yoursmahboob.wordpress.com sbi · yoursmahboob.wordpress.com iii p 101 speed tests for sbi bank...

382
SBI SBI BANK CLERK 101 SPEED TEST with 5 Practice Sets for Preliminary & Main Exams

Upload: others

Post on 16-Mar-2020

18 views

Category:

Documents


1 download

TRANSCRIPT

Page 1: yoursmahboob.wordpress.com SBI · yoursmahboob.wordpress.com iii P 101 Speed Tests for SBI Bank Clerk Exam 101 Speed Tests for SBI Bank Clerk Exam is revised and updated edition on

yoursmahboob.w

ordpress.com

SBISBIBANK CLERK

101 SPEED TEST

with 5 Practice Sets for Preliminary & Main Exams

Page 2: yoursmahboob.wordpress.com SBI · yoursmahboob.wordpress.com iii P 101 Speed Tests for SBI Bank Clerk Exam 101 Speed Tests for SBI Bank Clerk Exam is revised and updated edition on

yoursmahboob.w

ordpress.com

[ ii ]

• Head Office : B-32, Shivalik Main Road, Malviya Nagar, New Delhi-110017

• Sales Office : B-48, Shivalik Main Road, Malviya Nagar, New Delhi-110017

Tel. : 011-26691021 / 26691713

DISHA PUBLICATION

ALL RIGHTS RESERVED

© Copyright Publisher

No part of this publication may be reproduced in any form without prior permission of the publisher. The author and the

publisher do not take any legal responsibility for any errors or misrepresentations that might have crept in. We have tried

and made our best efforts to provide accurate up-to-date information in this book.

For further information about books from DISHA,

Log on to www.dishapublication.com or email to [email protected]

Typeset by Disha DTP Team

Page 3: yoursmahboob.wordpress.com SBI · yoursmahboob.wordpress.com iii P 101 Speed Tests for SBI Bank Clerk Exam 101 Speed Tests for SBI Bank Clerk Exam is revised and updated edition on

yoursmahboob.w

ordpress.com

[ iii ]

Preface101 Speed Tests for SBI Bank Clerk Exam

101 Speed Tests for SBI Bank Clerk Exam is revised and updated edition on the basis of Current Exam Patten. It contains all theIMPORTANT CONCEPTS which are required to crack this exam. The concepts are covered in the form of 101 SPEED TESTS.

No matter where you PREPARE from – a coaching or any textbook/ Guide — 101 SPEED TESTS provides you the rightASSESSMENT on each topic. Your performance provides you the right cues to IMPROVE your concepts so as to perform betterin the final examination.

It is to be noted here that these are not mere tests but act as a checklist of student’s learning and ability to apply concepts todifferent problems.

The book is based on the concept of TRP – Test, Revise and Practice. It aims at improving your SPEED followed by STRIKE RATEwhich will eventually lead to improving your SCORE.

How is this product different?

• 1st unique product with 101 speed tests.

• Each test is based on small topics which are most important for the SBI Bank Clerk exam. Each test contains around 25-30 MCQs on the latest pattern of the exam.

• The whole syllabus has been divided into 5 sections which are further distributed into 96 topics.

1. QUANTITATIVE APTITUDE is distributed into 28 topics.

2. REASONING is distributed into 24 topics.

3. ENGLISH LANGUAGE is distributed into 19 topics.

4. COMPUTER KNOWLEDGE/ MARKETING APTITUDE is distributed into 9 topics.

5. GENERAL AWARENESS is distributed into 16 topics.

• In the end of each section a Sectional Test is provided so as to sum up the whole section.

• Finally at the end 5 FULL TESTS-3 Tests for Prelim and 2 Tests for Mains are provided so as to give the candidates the realfeel of the final exam.

• In all, the book contains 3400+ Quality MCQ’s in the form of 101 tests.

• Solutions to each of the 101 tests are provided at the end of the book.

• The book provides Separate Tests. The book comes with perforation such that each test can be torn out of the book.

• Separate Time Limit, Maximum Marks, Cut-off, Qualifying Score is provided for each test.

• The book also provides a separate sheet, SCORE TRACKER where you can keep a record of your scores and performance.

• It is advised that the students should take each test very seriously and must attempt only after they have prepared thattopic.

• Once taken a test the candidates must spend time in analysing their performance which will provide you the right cuesto IMPROVE the concepts so as to perform better in the final examination.

It is our strong belief that if an aspirant works hard on the cues provided through each of the tests he/ she can improvehis/ her learning and finally the SCORE by at least 20%.

DISHA PUBLICATION

Page 4: yoursmahboob.wordpress.com SBI · yoursmahboob.wordpress.com iii P 101 Speed Tests for SBI Bank Clerk Exam 101 Speed Tests for SBI Bank Clerk Exam is revised and updated edition on

yoursmahboob.w

ordpress.com

[ iv ]

REASONING

1. ANALOGY 1-2

2. CLASSIFICATION 3-4

3. SERIES 5-6

4. ALPHABET TEST 7-8

5. CODING - DECODING 9-10

6. BLOOD RELATION 11-12

7. DIRECTION AND DISTANCE 13-14

8. TIME, SEQUENCE & RANKING TEST 15-16

9. SITTING ARRANGEMENT 17-18

10. ANALYTICAL PUZZLE - I 19-20

11. ANALYTICAL PUZZLE - II 21-22

12. VENN DIAGRAMS 23-24

13. SYLLOGISM - I 25-26

14. SYLLOGISM - II 27-28

15. SYMBOLS & CODES 29-30

16. ALPHA NUMERIC SEQUENCE PUZZLE 31-32

17. INPUT-OUTPUT 33-34

18. MATHEMATICAL OPERATIONS 35-36

19. CLOCK AND CALENDAR 37-38

20. DATA SUFFICIENCY 39-40

21. STATEMENT & CONCLUSION(MATHEMATICAL) 41-42

22. STATEMENT & CONCLUSION(LOGICAL) 43-44

23. PASSAGE BASED CONCLUSION-1 45-46

24. PASSAGE BASED CONCLUSION-2 47-48

25. CUBE & DICE 49-50

26. NON-VERBAL REASONING - I 51-52

27. NON-VERBAL REASONING - II 53-54

28. SECTION TEST : REASONING 55-58

QUANTITATIVE APTITUDE

29. NUMBER SYSTEM - I 59-60

30. NUMBER SYSTEM - II 61-62

31. LCM, HCF AND FRACTION 63-64

32. SQUARE, CUBE, INDICES & SURDS 65-66

33. SIMPLIFICATION 67-68

34. AVERAGE 69-70

35. PERCENTAGE 71-72

36. RATIO & PROPORTION 73-74

37. ALLIGATION AND MIXTURE 75-76

38. PROFIT, LOSS & DISCOUNT 77-78

39. SIMPLE INTEREST 79-80

40. COMPOUND INTEREST 81-82

41. DISTANCE, SPEED AND TIME 83-84

42. TIME & WORK / PIPE & CISTERN 85-86

43. PROBLEM BASED ON AGES 87-88

44. PERMUTATION & COMBINATION 89-90

45. PROBABILITY 91-92

46. AREA AND PERIMETER 93-94

47. VOLUME AND SURFACE AREA 95-96

48. GEOMETRY 97-98

49. NUMBER SERIES - I 99-100

50. NUMBER SERIES - II 101-102

51. DATA INTERPRETATION 103-104

52. SECTION TEST :QUANTITATIVE APTITUDE 105-106

ENGLISH LANGUAGE

53. READING COMPREHENSION - I 107-110

54. READING COMPREHENSION - II 111-114

55. READING COMPREHENSION - III 115-118

56. SYNONYMS 119-120

57. ANTONYMS 121-122

58. SENTENCE COMPLETION - I 123-124

Syllabus

Page 5: yoursmahboob.wordpress.com SBI · yoursmahboob.wordpress.com iii P 101 Speed Tests for SBI Bank Clerk Exam 101 Speed Tests for SBI Bank Clerk Exam is revised and updated edition on

yoursmahboob.w

ordpress.com

[ v ]

59. SENTENCE COMPLETION - II 125-126

60. SIMILAR SUBSTITUTION 127-128

61. CORRECT USAGE OFPREPOSITION 129-130

62. SENTENCE IMPROVEMENT 131-132

63. SPOTTING THE ERRORS - I 133-134

64. SPOTTING THE ERRORS - II 135-136

65. SPELLING TEST 137-138

66. ONE WORD SUBSTITUTION 139-140

67. PARA JUMBLES 141-142

68. IDIOMS AND PHRASES 143-144

69. CLOZE TEST - I 145-146

70. CLOZE TEST - II 147-148

71. SECTION TEST :ENGLISH LANGUAGE 149-152

COMPUTER KNOWLEDGE/ MARKETING APTITUDE

72. COMPUTER FUNDAMENTAL /BINARY SYSTEM/ OPERATING SYSTEM 153-154

73. MS OFFICE/ COMMANDS and 155-156SHORTCUT KEYS

74. SOFTWARES/ PROGRAMMING 157-158

75. INTERNET, NETWORKING AND 159-160COMPUTER ABBREVIATIONS

76. FUNDAMENTALS OF MARKETING, 161-162PRODUCT AND BRANDING

77. MARKET SITUATIONS BASED ON PRICE,DISTRIBUTION, PROMOTION ANDADVERTISING 163-164

78. MARKET SEGMENTATION,TARGETING AND POSITIONING 165-166

79. MODERN MARKETING /MARKETING IN BANKING INDUSTRY 167-168

80. SECTION TEST :COMPUTER KNOWLEDGE/MARKETING APTITUDE 169-170

GENERAL AWARENESS

81. HISTORY OF BANKING ANDITS DEVELOPMENT 171-172

82. RBI & ITS MONETARY POLICIES 173-174

83. BANKING PRODUCT & SERVICES 175-176

84. BANKING TERM/ TERMINOLOGY 177-178

85. MICRO FINANCE & ECONOMICS 179-180

86. FOREIGN TRADE 181-182

87. SOCIO-ECO-POLITICALENVIRONMENT OF INDIA 183-184

88. APPOINTMENT/ ELECTION/RESIGNATION 185-186

89. EVENTS/ ORGANISATION/SUMMITS 187-188

90. AWARDS AND HONOURS 189-190

91. BOOKS AND AUTHORS 191-192

92. SPORTS AND GAMES 193-194

93. SCIENCE AND TECHNOLOGY 195-196

94. CURRENT BANKING 197-198

95. CURRENT AFFAIRS 199-200

96. SECTION TEST :General & Financial Awareness 201-204

97. PRELIM FULL TEST - 1 205-212

98. PRELIM FULL TEST - 2 213-220

99. PRELIM FULL TEST - 3 221-228

100. Full MAIN TEST - 4 229-242

101. Full MAIN TEST - 5 243-256

101 SPEED TEST - SOLUTIONS 1-116

Page 6: yoursmahboob.wordpress.com SBI · yoursmahboob.wordpress.com iii P 101 Speed Tests for SBI Bank Clerk Exam 101 Speed Tests for SBI Bank Clerk Exam is revised and updated edition on

yoursmahboob.w

ordpress.com

[ vi ]

––

Page 7: yoursmahboob.wordpress.com SBI · yoursmahboob.wordpress.com iii P 101 Speed Tests for SBI Bank Clerk Exam 101 Speed Tests for SBI Bank Clerk Exam is revised and updated edition on

yoursmahboob.w

ordpress.com

[ vii ]

––

Page 8: yoursmahboob.wordpress.com SBI · yoursmahboob.wordpress.com iii P 101 Speed Tests for SBI Bank Clerk Exam 101 Speed Tests for SBI Bank Clerk Exam is revised and updated edition on

yoursmahboob.w

ordpress.com

[ viii ]

30

30

1515

15

15

15

15

15

15

25 30

15

151520

150

150

150150135

135

135

135135

20 12

150

––

25

25

2523

Page 9: yoursmahboob.wordpress.com SBI · yoursmahboob.wordpress.com iii P 101 Speed Tests for SBI Bank Clerk Exam 101 Speed Tests for SBI Bank Clerk Exam is revised and updated edition on

yoursmahboob.w

ordpress.com

1. A 'Square' is related to 'Cube' in the same way as a 'Circle' isrelated to

(a) Sphere (b) Circumference (c) Diameter(d) Area (e) None of these

2. In a certain code BRIGHT is written as JSCSGG. How isJOINED written in that code?

(a) HNIEFO (b) JPKEFO (c) JPKMDC(d) KPJCDM (e) None of these

3. '34' is related to '12' in the same way as '59' is related to

(a) 45 (b) 14 (c) 42(d) 38 (e) 47

4. 'Mustard' is related to 'Seed' in the same way as 'Carrot' isrelated to

(a) Fruit (b) Stem (c) Flower(d) Root (e) None of these

DIRECTIONS (Q. 5 - 24) : For each of the following questionsthere is some relationship between the two terms to the left of ::and the same relationship obtains between the two terms to itsright. In each of these questions the fourth term is missing. Thisterm is one of the alternatives among the five terms given below.Find out this term.

5. ADE : FGJ :: KNO : ?

(a) PQR (b) TPR (c) PQT(d) RQP (e) PRS

6. DGPGJ : MPQPS : : KNENQ : ....?

(a) RUFUX (b) RFUFX (c) RXUXF(d) RFUFX (e) None of these

7. UTS : EDC : : WVU : ?

(a) XWV (b) WXY (c) SJM(d) RPO (e) SRP

8. NUMBER : UNBMRE : : GHOST : ?

(a) HOGST (b) HOGTS (c) HGOST

(d) HGSOT (e) HGOTS

9. DRIVEN : EIDRVN : : BEGUM : ?

(a) EUBGM (b) MGBEU

(c) BGMEU (d) UEBGM

(e) BGMUE

10. PRLN : X Z T V : : J L F H : ?

(a) RTNP (b) NPRT (c) NRPT

(d) NTRP (e) RPNT

11. XWV : UTS : : LKJ : ... ?

(a) IHG (b) JKL (c) STU

(d) MNO (e) KIG

12. QYGO : SAIQ : : UCKS : ?

(a) WDMV (b) VFNU (c) WDLU

(d) VEMU (e) WEMU

13. YAWC : UESG : : QIOK : ?

(a) MINC (b) MIKE (c) KOME

(d) MMKO (e) LIME

14. CFIL : PSVY : : HKNQ : ?

(a) NPSV (b) LPSY (c) LORU

(d) MOQT (e) MPSU

15. 122 : 170 : : 290 : ?

(a) 299 (b) 315 (c) 332

(d) 344 (e) 362

RESPONSE

GRID

1. a b c d e 2. a b c d e 3. a b c d e 4. a b c d e 5. a b c d e

6. a b c d e 7. a b c d e 8. a b c d e 9. a b c d e 10. a b c d e

11. a b c d e 12. a b c d e 13. a b c d e 14. a b c d e 15. a b c d e

11Max. Marks : 30 No. of Qs. 30 Time : 20 min. Date : ........./......../................

Analogy

Page 10: yoursmahboob.wordpress.com SBI · yoursmahboob.wordpress.com iii P 101 Speed Tests for SBI Bank Clerk Exam 101 Speed Tests for SBI Bank Clerk Exam is revised and updated edition on

yoursmahboob.w

ordpress.com

2 SPEED TEST 116. 42 : 56 : : 110 : ?

(a) 132 (b) 136 (c) 144(d) 148 (e) 156

17. 12 : 20 : : 30 : ?(a) 15 (b) 32 (c) 35(d) 42 (e) 48

18. 3 : 10 : : 08 : ?(a) 10 (b) 13 (c) 17(d) 14 (e) 16

19. 08 : 28 : 15 : ?(a) 63 (b) 126 (c) 65(d) 124 (e) 26

20. 08 : 09 : : 64 : ?(a) 16 (b) 25 (c) 125(d) 32 (e) 20

21. 6 : 24 : : 5 : ?(a) 23 (b) 22 (c) 26(d) 20 (e) 19

22. 6 : 35 : : 11 : ?(a) 120 (b) 115 (c) 122(d) 121 (e) 124

23. 3 : 27 : : 4 : ?(a) 140 (b) 75 (c) 100(d) 80 (e) 64

24. 3 : 33

8 : : 5 : ?

(a)5

58

(b)3

58

(c)1

58

(d)5

28

(e)1

68

25. Which of the following pairs of words have the same

relationship as FAN : HEAT?

(a) Water : Drink (b) Light : Night

(c) Teach : Student (d) Air : Breathe

(e) Food : Hunger

26. A disease would always necessarily have

(a) Medicine (b) Bacteria (c) Cause

(d) Cure (e) Fever

27. 'Army' is related to 'Land' in the same way as 'Navy' is related

to _____.

(a) Ships (b) Battle (c) Water

(d) Admiral (e) Defence

28. A 'Tumbler' is related to 'Empty' in the same way as a 'Seat' is

related to

(a) Occupied (b) Person (c) Chair

(d) Sitting (e) Vacant

29. Secretly is to openly as silently is to__

(a) scarcely (b) impolitely (c) noisily

(d) quietly (e) None of these

30. Spring is to as coil as ring is to____

(a) rope (b) loop (c) cowl

(d) stretch (e) None of these

RESPONSE

GRID

16. a b c d e 17. a b c d e 18. a b c d e 19. a b c d e 20. a b c d e

21. a b c d e 22. a b c d e 23. a b c d e 24. a b c d e 25. a b c d e

26. a b c d e 27. a b c d e 28. a b c d e 29. a b c d e 30. a b c d e

Page 11: yoursmahboob.wordpress.com SBI · yoursmahboob.wordpress.com iii P 101 Speed Tests for SBI Bank Clerk Exam 101 Speed Tests for SBI Bank Clerk Exam is revised and updated edition on

yoursmahboob.w

ordpress.com

1. Four of the following five are alike in a certain way on thebasis of their positions in English alphabet and so form agroup. Which is the one that does not belong to that group.(a) HJG (b) PQN (c) DEB(d) TUR (e) KLI

2. Four of the following five are alike in a certain way and soform a group. Which is the one that does not belong to thatgroup ?(a) Rose (b) Jasmine (c) Hibiscus(d) Marigold (e) Lotus

3. Four of the following five pairs of alphas and numerals havesame relationship between their elements as in the case ofthe pair PROBLEM : 2948375 and hence form a group. Whichone does not belong to the group?(a) BORE : 8497 (b) MOEP : 5972(c) LBOR : 3849 (d) OMEP : 4572(e) EROL : 7943

4. Four of the following five pairs are alike in a certain wayand hence form a group. Which one does not belong tothat group?(a) DONE : NOED (b) WANT : NATW(c) WITH : TIHW (d) JUST : SUTJ(e) HAVE : AVEH

5. Four of the following five are alike in a certain way and soform a group. Which is the one that does not belong to thatgroup?(a) Hill (b) Valley (c) Dam(c) River (e) Mountain

6. Four of the following five are alike in a certain way and soform a group. Which is the one that does not belong to thatgroup?(a) 50 (b) 65 (c) 170(d) 255 (e) 290

7. Four of the following five are alike in a certain way and soform a group. Which is the one that does not belong to thatgroup?(a) 21 (b) 35 (c) 42(d) 56 (e) 49

8. Four of the following five are alike in a certain way and soform a group. Which is the one that does not belong to thatgroup?(a) Garlic (b) Ginger (c) Carrot(d) Radish (e) Brinjal

9. Four of the following five are alike in a certain way and soform a group. Which is the one that does not belong to thatgroup?(a) Clutch (b) Wheel (c) Break(d) Car (e) Gear

10. Four of the following five are alike in a certain way and soform a group. Which is the one that does not belong to thatgroup?(a) 196 (b) 256 (c) 529(d) 576 (e) 324

11. Four of the following five are alike in a certain way and soform a group. Which is the one that does not belong to thatgroup?(a) RPN (b) WSU (c) HDF(d) LHJ (e) QMO

12. Four of the following five are alike in a certain way and soform a group. Which is the one that does not belong to thatgroup?(a) 169 (b) 441 (c) 361(d) 529 (e) 289

13. Four of the following five are alike in a certain way and soform a group. Which is the one that does not belong to thatgroup?(a) PM (b) EB (c) TQ(d) IF (e) VY

14. Four of the following five are alike in a certain way and soform a group. Which is the one that does not belong to thatgroup?(a) 115 (b) 85 (c) 95(d) 75 (e) 155

15. Four of the following five are alike in a certain way and soform a group. Which is the one that does not belong to thatgroup?(a) 115 (b) 161 (c) 253(d) 391 (e) 345

RESPONSE

GRID

1. a b c d e 2. a b c d e 3. a b c d e 4. a b c d e 5. a b c d e

6. a b c d e 7. a b c d e 8. a b c d e 9. a b c d e 10. a b c d e

11. a b c d e 12. a b c d e 13. a b c d e 14. a b c d e 15. a b c d e

22Max. Marks : 30 No. of Qs. 30 Time : 20 min. Date : ........./......../................

Classification

Page 12: yoursmahboob.wordpress.com SBI · yoursmahboob.wordpress.com iii P 101 Speed Tests for SBI Bank Clerk Exam 101 Speed Tests for SBI Bank Clerk Exam is revised and updated edition on

yoursmahboob.w

ordpress.com

4 SPEED TEST 2

16. Four of the following five are alike in a certain way and soform a group. Which is the one that does not belong to thatgroup?

(a) OMQ (b) HFJ (c) TPR(d) TRV (e) VTX

17. Four of the following five are alike in a certain way based onthe English alphabetical series and so form a group. Whichis the one that does not belong to that group?

(a) MLJ (b) WVT (c) OMK(d) JIG (e) TSQ

18. Four of the following five are alike in a certain way and soform a group. Which is the one that does not belong to thatgroup?

(a) Diabetes (b) Smallpox

(c) Conjunctivitis (d) Chickenpox

(e) Plague

19. Four of the following five are alike in a certain way and soform a group. Which is the one that does not belong to thatgroup?

(a) Mustard (b) Jowar (c) Wheat(d) Paddy (e) Bajra

20. Four of the following five are alike in a certain way and soform a group. Which is the one that does not belong to thatgroup?

(a) 45 (b) 35 (c) 85(d) 25 (e) 75

21. Four of the following five are alike in a certain way andhence form a group. Which is the one that does not belongto that group?

(a) Succeed (b) Victory (c) Triumph(d) Compete (e) Win

22. Four of the following five are alike in a certain way andhence form a group. Which is the one that does not belongto that group?

(a) Fair (b) Impartial (c) Indifferent(d) Unbiased (e) Just

23. Four of the following five are alike in a certain way andhence form a group. Which is the one that does not belongto that group?(a) KP (b) BY (c) DW(d) HU (e) GT

24. Four of the following five are alike in a certain way andhence form a group. Which is the one that does not belongto that group?(a) JLNK (b) TVXU (c) ACEB(d) PRTQ (e) GJKH

25. Four of the following five are alike in a certain way andhence form a group. Which is the one that does not belongto that group?(a) GIJF (b) OQRN (c) KMNL(d) UWXT (e) CEFB

26. Four of the following five are alike in a certain way andhence form a group. Which is the one that does not belongto the group?(a) Break (b) Change (c) Split(d) Divide (e) Separate

27. Four of the following five are alike in a certain way andhence form a group. Which is the one that does not belongto the group?(a) Train (b) Instruct (c) Educate(d) Advocate (e) Coach

28. Four of the following five are alike in a certain way andhence form a group. Which is the one that does not belongto the group?(a) Extend (b) Higher (c) Upward(d) Rise (e) Ascend

29. Four of the following five are alike in a certain way and soform a group. Which is the one that does not belong to thatgroup?(a) Volume (b) Size (c) Large(d) Shape (e) Weight

30. Four of the following five are alike in a certain way and soform a group. Which is the one that does not belong to thatgroup?(a) Anxiety (b) Worry (c) Inhibition(d) Curiosity (e) Weariness

RESPONSE

GRID

16. a b c d e 17. a b c d e 18. a b c d e 19. a b c d e 20. a b c d e

21. a b c d e 22. a b c d e 23. a b c d e 24. a b c d e 25. a b c d e

26. a b c d e 27. a b c d e 28. a b c d e 29. a b c d e 30. a b c d e

Page 13: yoursmahboob.wordpress.com SBI · yoursmahboob.wordpress.com iii P 101 Speed Tests for SBI Bank Clerk Exam 101 Speed Tests for SBI Bank Clerk Exam is revised and updated edition on

yoursmahboob.w

ordpress.com

DIRECTIONS (Q. 1 - 12) : What should come in place of questionmark (?) in the following number series?

1. 1050 420 168 67.2 26.88 10.752 ?

(a) 4.3008 (b) 6.5038 (c) 4.4015(d) 5.6002 (e) None of these

2. 0 6 24 60 120 210 ?

(a) 343 (b) 280 (c) 335(d) 295 (e) None of these

3. 15 19 83 119 631 (?)

(a) 731 (b) 693 (c) 712(d) 683 (e) None of these

4. 19 26 40 68 124 (?)

(a) 246 (b) 238 (c) 236(d) 256 (e) None of these

5. 11 10 18 51 200 (?)

(a) 885 (b) 1025 (c) 865(d) 995 (e) None of these

6. 14 24 43 71 108 (?)

(a) 194 ( b) 154 (c) 145(d) 155 (e) None of these

7. 144 173 140 169 136 (?)

(a) 157 ( b) 148 (c) 164(d) 132 (e) None of these

8. 656 352 200 124 86 (?)

(a) 67 (b) 59 (c) 62(d) 57 (e) None of these

9. 12 18 36 102 360 (?)

(a) 1364 (b) 1386 (c) 1384(d) 1376 (e) None of these

10. 71 78 99 134 183 (?)

(a) 253 (b) 239 (c) 246(d) 253 (e) None of these

11. 342 337.5 328.5 315 297 (?)

(a) 265.5 (b) 274.5 (c) 270(d) 260 (e) None of these

12. 161 164 179 242 497 (?)

(a) 1540 (b) 1480 (c) 1520(d) 1440 (e) None of these

DIRECTIONS (Qs. 13 to 17): What will come in place of questionmark (?) in the following number series?

13. 12, 30, 56, 90, 132, ?

(a) 178 (b) 182 (c) 185(d) 189 (e) 196

14. 91, 381, 871, 1561, 2451, ?

(a) 3541 (b) 3621 (c) 3681(d) 3716 (e) 3772

15. 110, 440, 990, 1760, 2750, ?

(a) 3680 (b) 3610 (c) 37820(d) 3840 (e) 3960

RESPONSE

GRID

1. a b c d e 2. a b c d e 3. a b c d e 4. a b c d e 5. a b c d e

6. a b c d e 7. a b c d e 8. a b c d e 9. a b c d e 10. a b c d e

11. a b c d e 12. a b c d e 13. a b c d e 14. a b c d e 15. a b c d e

Max. Marks : 30 No. of Qs. 30 Time : 20 min. Date : ........./......../................

33 Series

Page 14: yoursmahboob.wordpress.com SBI · yoursmahboob.wordpress.com iii P 101 Speed Tests for SBI Bank Clerk Exam 101 Speed Tests for SBI Bank Clerk Exam is revised and updated edition on

yoursmahboob.w

ordpress.com

6 SPEED TEST 3

16. 5, 6, 11, 20, 33, 50, ?

(a) 64 (b) 71 (c) 78(d) 81 (e) 84

17. 2, 7, 24, 77, 238, 723, ?

(a) 1948 (b) 1984 (c) 2010(d) 2096 (e) 2180

DIRECTIONS (Q.18 - 22) :What should come in place of questionmark (?) in the following number series?

18. 980 484 260 112 50 ? 3.5

(a) 25 (b) 17 (c) 21(d) 29 (e) None of these

19. 1015 508 255 129 66.5 ? 20.875

(a) 34.50 (b) 35 (c) 35.30(d) 35.75 (e) None of these

20. 4.5 18 2.25 ? 1.6875 33.75

(a) 27 (b) 25.5 (c) 36(d) 40 (e) None of these

21. 59.76 58.66 56.46 52.06 ? 25.66

(a) 48.08 (b) 46.53 (c) 43.46(d) 43.26 (e) None of these

22. 36 157 301 470 ? 891

(a) 646 (b) 695 (c) 639(d) 669 (e) None of these

DIRECTIONS (Qs. 23 to 27) : Which one of the letters whensequentially placed at the gaps in the given letter series shallcomplete it?

23. a – c a – b c – b c c – b c a

(a) b b a b (b) b a b a (c) a a b b(d) b b a a (e) None of these

24. What will be the next term in ?

DCXW, FEVU, HGTS, ...........

(a) LKPO (b) ABYZ (c) JIRQ(d) LMRS (e) None of these

25. ZXVTR....

(a) O, K (b) N, M (c) K, S(d) M, N (e) P, N

26. C, e, G, i, K....

(a) O, K (b) m, O (c) k, M(d) M, k (e) O, p

27. m_ _ l m _ l _ m m _ l

(a) mllml (b) mlmll (c) llmlm

(d) mmlml (e) llmll

28. What should come next in the number series given below ?1 1 2 1 2 3 1 2 3 4 1 2 3 4 5 1 2 3 4 5 6 1 2 3 4 5 6(a) 5 (b) 2

(c) 8 (d) 1

(e) None of these

29. What should come next in the following letter series?A BC D P Q RS A BC D E P Q RS T A B C D E F P Q RS T

(a) A (b) V

(c) U (d) W

(e) None of these

30. What will come in place of question mark (?) in the followingseries ?

NDP, QWB, ZFR, ?

(a) SVJ (b) AFS

(c) IVS (d) SFA

(e) None of these

RESPONSE

GRID

16. a b c d e 17. a b c d e 18. a b c d e 19. a b c d e 20. a b c d e

21. a b c d e 22. a b c d e 23. a b c d e 24. a b c d e 25. a b c d e

26. a b c d e 27. a b c d e 28. a b c d e 29. a b c d e 30. a b c d e

Page 15: yoursmahboob.wordpress.com SBI · yoursmahboob.wordpress.com iii P 101 Speed Tests for SBI Bank Clerk Exam 101 Speed Tests for SBI Bank Clerk Exam is revised and updated edition on

yoursmahboob.w

ordpress.com

1. How many such pairs of letters are there in the wordEXCURSION, each of which has as many letters betweenthem in the word as they have in the English alphabet?(a) None (b) One (c) Two(d) Three (e) More than three

2. How many such pairs of letters are there in the word GOLDEN,each of which has as many letters between them in the wordas in the English alphabet?(a) None (b) One (c) Two(d) Three (e) More than three

3. How many such pairs of letters are there in the word STRIVEeach of which has as many letters between them in the wordas in the English alphabet ?(a) One (b) Two (c) Three(d) Four (e) None of these

4. How many meaningful English words can be formed with theletters 'ATN' using each letter only once in each word?(l) One (b) Two (c) Three(d) Four (e) More than four

5. The serial order of how many letters in the word CLIENT willnot differ than their serial order in the arrangement where theletters of the word are arranged alphabetically?(a) Four (b) One (c) Three(d) Two (e) None of these

6. How many meaningful English words can be made with theletters ENAL using each letter only once in each word?(a) One (b) Two (c) Three(d) Four (e) More than four

7. How many three - letter meaningful words can be formedfrom the word TEAR beginning with 'A' without repeatingany letter within that word?(a) One (b) Three (c) Five(d) Two (e) None of these

8. If all the letters in the word ARGUMENT are rearranged inalphabetical order and substituted by the letter immediatelyfollowing it in the English alphabet, what will be the newarrangement of letters?(a) BFHNOSUV (b) BFHONSWV(c) BFHNOUSV (d) BFHNOQUV(e) None of these

RESPONSE

GRID

1. a b c d e 2. a b c d e 3. a b c d e 4. a b c d e 5. a b c d e

6. a b c d e 7. a b c d e 8. a b c d e 9. a b c d e 10. a b c d e

11. a b c d e 12. a b c d e 13. a b c d e 14. a b c d e 15. a b c d e

Max. Marks : 30 No. of Qs. 30 Time : 20 min. Date : ........./......../................

9. If the letters of the word ARROGANCE are interchanged,first with fifth, second with sixth, third with seventh, fourthwith eighth and the position of the ninth remains unchanged,then what will the new arrangement of letters be?(a) GANACRROE (b) GANCRAROE"(c) GNACORRAE (d) GANCARROE(e) None of these

10. If all the letters in the word 'PRINCE' are rearranged inalphabetical order, then how many letter(s) will remainunchanged?(a) None (b) One (c) Two

(d) Three (e) More than three11. How many meaningful English words can be formed with

ESRO using each letter only once in each word?(a) None (b) One (c) Two(d) Three (e) More than three

12. How many such pairs of letters are there in the wordCONSTABLE, each of which has as many letters betweenthem in the word as in the English alphabet?(a) None (b) One (c) Two(d) Three (e) More than three

13. How many such pairs of letters are there in the wordPHYSICAL, each of which has as many letters between themin the word as they have in the English alphabet?(a) None (b) One (c) Two(d) Three (e) More than three

14. How many pairs of letters are there in the word 'SHIFTED'each of which has as many letters between its two letters asthere are between them in the English alphabet?(a) None (c) One (c) Two(d) Three (e) None of these

15. How many meaningful English words can be formed by usingany two letters of the word 'GOT'?(a) Three (b) Two (c) One

(d) More than three (e) None of these

44Alphabet Test

Page 16: yoursmahboob.wordpress.com SBI · yoursmahboob.wordpress.com iii P 101 Speed Tests for SBI Bank Clerk Exam 101 Speed Tests for SBI Bank Clerk Exam is revised and updated edition on

yoursmahboob.w

ordpress.com

8 SPEED TEST 4

RESPONSE

GRID

16. a b c d e 17. a b c d e 18. a b c d e 19. a b c d e 20. a b c d e

21. a b c d e 22. a b c d e 23. a b c d e 24. a b c d e 25. a b c d e

26. a b c d e 27. a b c d e 28. a b c d e 29. a b c d e 30. a b c d e

16. How many three-letter meaningful English words can beformed from the word NOTE beginning with T and withoutrepeating any letter within that word?(a) Three (b) One (c) Two(d) None (e) None of these

17. If all the letters in the word MERCIFUL are rearranged inalphabetical order and substituted by the alphabet precedingthem in the English alphabet, what will be the newarrangement of letters?(a) BDFIEKLQT (b) BDEHKLQT(c) BDEHLKQT (d) BDEJMLQT(e) None of these

18. If it is possible to make only one meaningful word, from thefirst, the third, the fifth and the eighth letters of wordENTERPRISE using each letter only once, first letter of theword is your answer. If more than one such word can bemade your answer is `X' and if no such word can be made,

your answer is Y'.

(a) P (b) S (c) T(d) X (e) Y

19. If in the word DISTANCE all the vowels are replaced by thenext letter and all the consonants are replaced by the previousletter and then all the letters are arranged alphabetically, whichletter will be third from the right?(a) M (b) F (c) R

(d) J (e) None of these20. After arranging the letters of the word FOLK in alphabetical

order, if each letter is substituted by the letter immediatelypreceding to it in the English alphabet, what will be theresultant form of the word?'(a) GLMP (b) EJKP (c) EKJN(d) EJKN (e) None of these

21. If A is denoted by 1, B by 2, C by 3, D by 1, E by 2, F by 3 andso on, what would be the sum of the digits for the word

MULE ?

(a) 7 (b) 8 (c) 9

(d) 10 (e) None of these22. How many meaningful English words can be made with the

letters ARTSE using each letter only once in each word?(a) None (b) One (c) Two(d) Three (e) More than three

23. If each vowel of the word WEBPAGE is substituted with thenext letter of the English alphabet, and each consonant issubstituted with the letter preceding it, which of the followingletters will appear thrice?(a) G (b) F (c) Q(d) V (e) None of these

24. How many meaningful four letter English words can beformed with the letters TPSI using each letter only once ineach word?(a) One (b) Two (c) Three(d) Four (e) More than four

25. How many meaningful five-letter words can be formed withthe letters SLIKL using each letter only once ?(a) One (b) Two (c) Three(d) More than three (e) None

26. If each vowel in the word HABITUAL is changed to the nextletter in the English alphabet and each consonant is changedto the previous letter in the English alphabet, which of thefollowing will be fourth from the left ?(a) A (b) S (c) J(d) H (e) None of these

27. The positions of how many alphabets will remain unchangedif each of the alphabets in the word WALKING is arranged inalphabetical order from left to right ?(a) None (b) One (c) Two(d) Three (e) More than three

28. If the first three letters of the word COMPREHENSION arereversed, then the last three letters are added and then theremaining letters are reversed and added, then which letterwill be exactly in the middle ?(a) H (b) N (c) R(d) S (e) None of these

29. If the first and second letters in the word DEPRESSION wereinterchanged, also the third and the fourth letters, the fifthand the sixth letters and so on, which of the following wouldbe the seventh letter from the right ?(a) R (b) O (c) S(d) P (e) None of these

30. Arrange the given words in alphabetical order. Which onecomes in the middle?(a) Restrict (b) Rocket (c) Robber

(d) Random (e) Restaurant

Page 17: yoursmahboob.wordpress.com SBI · yoursmahboob.wordpress.com iii P 101 Speed Tests for SBI Bank Clerk Exam 101 Speed Tests for SBI Bank Clerk Exam is revised and updated edition on

yoursmahboob.w

ordpress.com

1. In a certain code, PROSE is written as PPOQE. How wouldLIGHT be written in that code ?(a) LIGFT (b) LGGHT (c) LGGFT(d) JIEHR (e) None of these

2. If Z = 52 and ACT is equal to 48, then BAT will be equal to(a) 39 (b) 41 (c) 44(d) 46 (e) None of these

3. In a certain code, 15789 is written as EGKPT and 2346 iswritten as ALUR. How is 23549 written in that code ?(a) ALEUT (b) ALGTU (c) ALGUT(d) ALGRT (e) None of these

4. In a certain coding system, RBM STD BRO PUS means ‘thecat is beautiful’. TNH PUS DIM STD means ‘the dog isbrown’. PUS DIM BRO PUS CUS means ‘the dog has thecat’. What is the code for ‘has’ ?(a) CUS (b) BRO (c) DIM(d) STD (e) None of these

5. If NAXALITE is written in a certain code as LYVYJGRC’ howwill INTEGRATE be written in the same code ?(a) LGRECYPRC (b) GLRCEPYRC(c) PYWMNOPQR (d) BLACKHOLE

(e) None of these6. ENGLAND is written as 1234526 and FRANCE as 785291.

How will GREECE be written in this coding scheme ?(a) 381191 (b) 381911 (c) 394132(d) 562134 (e) None of these

7. In a certain code, CAT is written as SATC and DEAR is writtenas SEARD. How would SING be written in that code?(a) GNISS (b) SINGS (c) SGNIS(d) BGINS (e) None of these

8. PROMISE is coded as 1234567. What should be the code forMISER in that code language ?(a) 45672 (b) 43672 (c) 76543(d) 14572 (e) None of these

RESPONSE

GRID

1. a b c d e 2. a b c d e 3. a b c d e 4. a b c d e 5. a b c d e

6. a b c d e 7. a b c d e 8. a b c d e 9. a b c d e 10. a b c d e

11. a b c d e 12. a b c d e 13. a b c d e 14. a b c d e 15. a b c d e

Max. Marks : 30 No. of Qs. 30 Time : 20 min. Date : ........./......../................

9. If ‘light’ is called ‘morning’, ‘morning’ is called ‘dark’, ‘dark’is called ‘night’, ‘night’ is called ‘sunshine’ and ‘sunshine’ iscalled ‘dusk’, when do we sleep ?(a) Dusk (b) Dark (c) Night(d) Sunshine (e) None of these

10. If A + B = C, D – C = A and E – B = C, then what does D + Fstands for ? Provide your answer in letter terms as well as innumber terms.(a) J & 10 (b) A & 1 (c) C & 3(d) Q & 17 (e) None of these

11. In a certain code, ‘247’ means ‘spread red carpet’, ‘256’ means‘dust one carpet’ and ‘234’ means ‘one red carpet’. Whichdigit in that code means ‘dust’?(a) 2 (b) 3 (c) 5(d) 6 (e) None of these

12. If table is called chair, chair is called cot, cot is called pot andpot is called filter, where does a person sit?(a) pot (b) cot (c) chair(d) filter (e) None of these

13. In a certain code OVER is written as $#%*. and VIST is writtenas #+×–. How is SORE written in that code?(a) ×$*% (b) %×$*(c) ×*$% (d) None of these

(e) All of these14. In a certain code language PULSE' is written as `DRKTO'

and 'NEW is written as VDM'. How will 'PROBES' be writtenin that code language?(a) RDANQO (b) QSPCFT(c) TFCPSQ (d) OPNADR(e) None of these

15. If REMIT is written as *£3 7 and CONSUL is written as

= %8 b $5; then OCELOT will be written as(a) %=3587 (b) %=£5%7

(c) %=35% (d) %35%7

(e) None of these

55Coding - Decoding

Page 18: yoursmahboob.wordpress.com SBI · yoursmahboob.wordpress.com iii P 101 Speed Tests for SBI Bank Clerk Exam 101 Speed Tests for SBI Bank Clerk Exam is revised and updated edition on

yoursmahboob.w

ordpress.com

10 SPEED TEST 5

RESPONSE

GRID

16. a b c d e 17. a b c d e 18. a b c d e 19. a b c d e 20. a b c d e

21. a b c d e 22. a b c d e 23. a b c d e 24. a b c d e 25. a b c d e

26. a b c d e 27. a b c d e 28. a b c d e 29. a b c d e 30. a b c d e

16. If AMONG is written as NAOGM and SPINE is written asNSIEP, then LAMON will be written as(a) OALNM (b) MLONA (c) OLMNA(d) OLNMA (e) None of these

17. In a certain code SEAL is written as $75@ and DOSE iswritten as #8$7. How is SOLD written in that code?(a) $8@# (b) #87$ (c) #8$7(d) $5@# (e) None of these

18. If 'table' is called 'chair'; 'chair' is called cupboard', 'cupboard'is called 'chalk', 'chalk' is called 'book', 'book'is called 'duster' and 'duster' is called 'table', what does theteacher use to write on the black board?(a) book (b) cupboard (c) table(d) duster (e) None of these

19. in a certain code MOTHER' is written as OMHURF. Howwill ANSWER' be written in that code ?(a) NBWRRF (b) MAVSPE(c) NBWTRD (d) NBXSSE(e) None of these

20. In a certain code 'GROW' is written as '=@%#' and 'WITHIN'is written as '# ¸ + Ó ¸ D ' How is 'WING' written in thatcode?

(a) # ¸ D = (b) # % D = (c) % ¸ D =

(d) # ¸ Ó = (e) None of these21. If 'DO' is written as 'FQ' and 'IN' is written as 'KP' then how

would 'AT' be written?(a) CV (b) BS (c) CU(d) DV (e) None of these

22. lf 8 is written as B, 1 as R, 6 as K, 9 as O, 4 as M, 7 as W and3 as T, then how, would WROMBT be Written in the numericform?(a) 714983 (b) 719483 (c) 769483(d) 719486 (e) None of these

23. In a code language, PINK is written as QHOJ and BOLT iswritten as CNMS. How would MUST be written in that code?(a) NVTS (b) NTTS (c) NTRS(d) NITU (e) None of these

24. In a certain code,'LOCK' is written as MPBJ'' and BLOW' iswritten as CMNV'. How is 'WINE' written in that code?(a) VHOF (b) XJMD (c) XJOR(d) VHMD (e) None of these

25. In a certain code SOLDIER is written as JFSCRNK. How isGENIOUS written in that code?(a) PVTHHFO (b) PVTHFDM(c) PVTHMDF (d) TVPHFDM(e) None of these

26. If blue means green, green means black, black means white,white means pink, pink means red and red means orange,then what is the colour of blood?(a) Red (b) Black (c) White(d) Pink (e) None of these

DIRECTIONS (27 and 28) : Study the following information toanswer the given questions :

In a certain code ‘colours of the sky’ is written as ’ki la fa so’,‘rainbow colours’ is written as ‘ro ki’ and ‘sky high rocket’ iswritten as ‘la pe jo’ and ‘the rocket world’ is written as ‘pe so ne’.

27. Which of the following is the code for ’colours sky high’?

(a) ro jo la (b) fa la jo (c) la ki so(d) ki jo la (e) fa ki jo

28. Which of the following will/may represent ‘the’?

(a) Only ’fa’ (b) Either ‘fa’ or ’la’

(c) Only ’so’ (d) Only ’la’

(e) Either ‘so’ or ’fa’

29. In a certain code language DREA is written as BFSE, MINGis written as FMHL and TREA is written as BFSU How willTISE be written in that code ?

(a) DTHS (b) DSTV (c) DSHS

(d) FUGS (e) None of these

30. If table is called chair, chair is called cot, cot is called potand pot is called filter, where does a person sit?

(a) pot (b) cot (c) chair(d) filter (e) None of these

Page 19: yoursmahboob.wordpress.com SBI · yoursmahboob.wordpress.com iii P 101 Speed Tests for SBI Bank Clerk Exam 101 Speed Tests for SBI Bank Clerk Exam is revised and updated edition on

yoursmahboob.w

ordpress.com

1. Pointing to a photograph, Arun said, she is the mother of mybrother’s son’s wife’s daughter. How is Arun related to the lady?(a) Cousin (b) Daughter-in-law(c) Uncle (d) None of these(e) None of these

2. A’s mother is sister of B and has daughter C. How can A berelated to B from amongst the following ?(a) Niece (b) Uncle (c) Daughter(d) Father (e) None of these

3. Introducing Kamla, Mahesh said : His father is the only sonof my father. How was Mahesh related to Kamla ?(a) Brother (b) Father (c) Uncle(d) Son (e) None of these

4. Anil, introducing a girl in a party, said, she is the wife of thegrandson of my mother. How is Anil related to the girl?(a) Father (b) Grandfather(c) Husband (d) Father-in-law(e) None of these

5. A and B are married couple. X and Y are brother. X is thebrother of A. How Y is related to B ?(a) Brother (b) Brother-in-law(c) Son (d) Son-in-law(e) None of these

6. A man said to a woman, “Your mother’s husband’s sister ismy aunt.” How is the woman related to the man ?(a) Granddaughter (b) Daughter (c) Sister(d) Aunt (e) None of these

7. Showing a lady in the park, Vineet said, ‘She is the daughterof my grandfather’s only son’. How is Vineet related to thatlady ?(a) Father (b) Son (c) Brother(d) Mother (e) None of these

8. X told Y, “Though I am the son of your father, you are not mybrother”. How is X related to Y ?(a) Sister (b) Son (c) Daughter(d) None of these (e) None of these

9. Rahul’s mother is the only daughter of Monika’s father. Howis Monika’s husband related to Rahul?(a) Uncle (b) Father(c) Grandfather (d) Brother(e) None of these

RESPONSE

GRID

1. a b c d e 2. a b c d e 3. a b c d e 4. a b c d e 5. a b c d e

6. a b c d e 7. a b c d e 8. a b c d e 9. a b c d e 10. a b c d e

11. a b c d e 12. a b c d e 13. a b c d e 14. a b c d e 15. a b c d e

Max. Marks : 30 No. of Qs. 30 Time : 20 min. Date : ........./......../................

10. Ankit is related to Binny and Chinky, Daizy is Chinky’smother. Also Daizy is Binny’s sister and Aruna is Binny’ssister. How is Chinky related to Aruna?(a) Niece (b) Sister (c) Cousin(d) Aunt (e) None of these

11. Saroj is mother-in-law of Vani who is sister-in-law of Deepak.Rajesh is father of Ramesh, the only brother of Deepak. Howis Saroj related to Deepak?(a) Mother-in-law (b) Wife (c) Aunt(d) Mother (e) None of these

12. A man pointing to a photograph says, “The lady in thephotograph is my nephew’s maternal grandmother.” How isthe lady in the photograph related to the man’s sister whohas only brother and no other sister?(a) Sister-in-law (b) Cousin(c) Mother (d) Mother-in- law(e) None of these

13. A man pointing to a photograph says, “The lady in thephotograph is my nephew’s maternal grandmother and herson is my sister’s brother-in-law. How is the lady in thephotograph related to his sister who has no other sister?(a) Mother (b) Cousin(c) Mother-in-law (d) Sister-in-law(e) None of these

DIRECTIONS (Qs. 14 - 16) : Read the information given below toanswer the questions that follow.Rama and Mohan are a married couple having two daughtersnamed Smita and Devika. Devika is married to a man who is theson of Madhu and Jeewan. Romila is the daughter of Aman.Krishna who is Aman’s sister is married to Sunil and has two sonsAnuj and Ankur. Ankur is the grandson of Madhu and Jeewan.14. How is Krishna related to Devika ?

(a) Sister-in-law (b) Sister(c) Aunt (d) None of these(e) None of these

15. What is the relationship between Anuj and Romila?(a) Uncle – Niece (b) Father – Daughter(c) Husband – Wife (d) Cousins(e) None of these

66Blood Relation

Page 20: yoursmahboob.wordpress.com SBI · yoursmahboob.wordpress.com iii P 101 Speed Tests for SBI Bank Clerk Exam 101 Speed Tests for SBI Bank Clerk Exam is revised and updated edition on

yoursmahboob.w

ordpress.com

12 SPEED TEST 6

RESPONSE

GRID

16. a b c d e 17. a b c d e 18. a b c d e 19. a b c d e 20. a b c d e

21. a b c d e 22. a b c d e 23. a b c d e 24. a b c d e 25. a b c d e

26. a b c d e 27. a b c d e 28. a b c d e 29. a b c d e 30. a b c d e

16. Which of the following is true ?(a) Anuj is the son of Smita.(b) Romila is the cousin of Krishna.(c) Madhu is Sunil’s mother-in-law.(d) Jeewan is Devika’s maternal uncle.(e) None of these

DIRECTIONS (Qs. 17-20) : Read the information given below toanswer these questions:(i) In a family of six persons, A, B, C, D, E and F, there are two

married couples.(ii) D is the grandmother of A and the mother of B.(iii) C is the wife of B and the mother of F.(iv) F is the granddaughter of E.17. What is C to A?

(a) Daughter (b) Grandmother(c) Mother (d) Cannot be determined(e) None of these

18. How many male members are there in the Family?(a) Two (b) Three(c) Four (d) Cannot be determined(e) None of these

19. Which of the following is true?(a) A is the brother of F(b) A is the sister of F(c) D has two grandsons(d) None of these(e) None of these

20. Who among the following is one of the couples?(a) C D (b) D E(c) E B (d) Cannot be determined(e) None of these

DIRECTIONS (Q.21-25) : Read the following information carefullyand answer the questions which follow:

(i) 'P ¸ Q' means 'P is son of Q'.(ii) 'P × Q' means 'P is sister of Q'.(iii) 'P + Q' means 'P is brother of Q'.(iv) 'P – Q' means 'P is mother of Q'.

21. How is T related to S in the expression 'T × R + V S'?(a) Sister (b) Mother (c) Aunt(d) Uncle (e) None of these

22. How is T related to S in the expression 'T × R V – S'?(a) Father (b) Sister (c) Daughter(d) Aunt (e) None of these

23. How is S related to T in the expression 'T + R – V + S'?(a) Uncle (b) Nephew(c) Son (d) Cannot be determined(e) None of these

24. Which of the following means that 'S is the husband of T'?(a) T × R – V + S (b) T – R V × S(c) T – R + V S (d) T ¸ R × V + S(e) None of these

25. How is V related to T in the expression 'T ¸ R + V × S'?(a) Aunt (b) Nephew (c) Niece(d) Uncle (e) None of these

26. D is brother of B. M is brother of B. K is father of M. T is wifeof K. How is B related to T?(a) Son (b) Daughter(c) Son or Daughter (d) Data inadequate(e) None of these

DIRECTIONS (Qs. 27 - 30) : Study the following informationcarefully to answer the questions that follow.Adhir Mishra has three children : Urmila, Raghu and Sumit. Sumitmarried Roma, the eldest daughter of Mr. And Mrs. Mohan. TheMohans married their youngest daughter to the eldest son of Mr.and Mrs. Sharma and they had two children Sandeep and Shaifali.The Mohans have two more children, Roshan and Bimla, bothelder to Sheila. Sohan and Shivendar are sons of Sumit and Roma.Leela is the daughter of Sandeep.27. How is Mrs. Mohan related to Sumit ?

(a) Aunt (b) Mother-in-law(c) Mother (d) Sister-in-law(e) None of these

28. What is the surname of Sohan ?(a) Sharma (b) Mohan (c) Mishra(d) Raghu (e) None of these

29. What is the surname of Leela ?(a) Sharma (b) Mohan (c) Mishra(d) None of these (e) None of these

30. How is Shivendar related to Roma’s father ?(a) Son-in-law (b) Cousin (c) Son(d) Grandson (e) None of these

Page 21: yoursmahboob.wordpress.com SBI · yoursmahboob.wordpress.com iii P 101 Speed Tests for SBI Bank Clerk Exam 101 Speed Tests for SBI Bank Clerk Exam is revised and updated edition on

yoursmahboob.w

ordpress.com

1. At my house I am facing east, then I turn left and go 10 m,then turn right and go 5 m, and then I go 5 m towards thesouth and from there 5 m towards the west. In which directionam I from my house ?(a) East (b) West (c) North(d) South (e) None of these

2. My friend and I started walking simultaneously towards eachother from two places 100 m apart. After walking 30 m, myfriend turns left and goes 10 m, then he turns right and goes20 m and then turns right again and comes back to the roadon which he had started walking. If we walk with the samespeed, what is the distance between us at this point of time?(a) 50 m (b) 20 m (c) 30 m(d) 40 m (e) None of these

3. A watch reads 4:30 O’clock. If minute hand points towardsthe East, in which direction does the hour hand point ?(a) North-East (b) South-East(c) North-West (d) North(e) None of these

4. A man travels 3 km to the west, turns left and goes 3 km,turns right and goes 1 km, again turns right and goes 3 km.How far is he from the starting point ?(a) 7 km (b) 6 km (c) 5 km(d) 4 km (e) None of these

5. A and B start walking in opposite directions. A covers 3 km andB covers 4 km. Then A turns right and walks 4 km while B turnsleft and walks 3 km. How far is each from the starting point ?(a) 5 km (b) 4 km (c) 10 km(d) 8 km (e) None of these

6. Ram walks 10 m south from his house, turns left and walks25 m, again turns left and walks 40 m, then turns right andwalks 5 m to reach to the school. In which direction theschool is from his house ?(a) South-west (b) North-east (c) East(d) North (e) None of these

7. A river flows west to east and on the way turns left and goesin a semicircle round a hillock, and then turns left in a right-angle. In which direction is the river finally flowing ?(a) North (b) South (c) East(d) West (e) None of these

DIRECTIONS (Qs. 8 & 9) : Study the information given belowcarefully to answer these questions:On a playing ground, Dinesh, Kunal, Nitin, Atul and Prashant arestanding, as described below, facing the North.

RESPONSE

GRID

1. a b c d e 2. a b c d e 3. a b c d e 4. a b c d e 5. a b c d e

6. a b c d e 7. a b c d e 8. a b c d e 9. a b c d e 10. a b c d e

11. a b c d e 12. a b c d e 13. a b c d e

Max. Marks : 30 No. of Qs. 30 Time : 20 min. Date : ........./......../................

(I) Kunal is 40 metres to the right of Atul.(II) Dinesh is 60 metres to the south of Kunal(III) Nitin is 25 metres to the west of Atul.(IV) Prashant is 90 metres to the north of Dinesh.

8. Who is to the north-east of the person who is to the left ofKunal?(a) Dinesh (b) Nitin (c) Atul(d) None of these (e)

9. If a boy walks from Nitin, meets Atul followed by Kunal,Dinesh and Prashant, then how many metres has he walkedif he has travelled the straight distance all through?(a) 155 metres (b) 185 metres(c) 215 metres (d) 245 metres(e) None of these

10. A square field ABCD of side 90 m is so located that itsdiagonal AC is from north to south and the corner B is to thewest of D. Rohan and Rahul start walking along the sidesfrom B and C respectievely in the clockwise and anti-clockwise directions with speeds of 8 km/hr and 10 km/hr.Where shall they cross each other the second time ?(a) On AD at a distance of 30 m from A(b) On BC at a distance of 10 m from B(c) On AD at a distance of 30 m from D(d) On BC at a distance of 10 m from C(e) None of these

11. One morning after sunrise, Reeta and Kavita were talking toeach other face to face at Tilak square. If Kavita’s shadowwas exactly to the right of Reeta, then which direction wasKavita facing?(a) North (b) South (c) East(d) Data inadequate (e) None of these

12. I am facing west. I turn 45° in the clockwise direction andthen 180° in the same direction and then 270° anticlockwise.Which direction am I facing now?(a) South-West (b) South (c) West(d) North-west (e) None of these

13. Bhavika and Sunaina start simultaneously towards eachother from two places 100m apart. After walking 30 m Bhavikaturns left and goes 10 m, then she turns right and goes 20 mand then turns right again and comes back to the road onwhich she had started walking. If both Bhavika and Sunainawalk with the same speed, what is the distance betweenthem at this point of time?(a) 70 metres (b) 10 metres (c) 40 metres(d) 20 metres (e) None of these

77Direction andDistance

Page 22: yoursmahboob.wordpress.com SBI · yoursmahboob.wordpress.com iii P 101 Speed Tests for SBI Bank Clerk Exam 101 Speed Tests for SBI Bank Clerk Exam is revised and updated edition on

yoursmahboob.w

ordpress.com

14 SPEED TEST 7

RESPONSE

GRID

14. a b c d e 15. a b c d e 16. a b c d e 17. a b c d e 18. a b c d e

19. a b c d e 20. a b c d e 21. a b c d e 22. a b c d e 23. a b c d e

24. a b c d e 25. a b c d e 26. a b c d e 27. a b c d e 28. a b c d e

29. a b c d e 30. a b c d e

14. A man starts from a point and moves 3 km north, then turns towest and goes 2 km. He turns north and walks 1 km and thenmoves 5 km towards east. How far is he from the starting point?(a) 11 km (b) 10 km (c) 5 km(d) 8 km (e) None of these

15. Starting from Point X, Joy walked 15 metres towards West.He turned left and walked 20 metres. He again turned leftand walked 15 metres. After which he turned right andwalked for another 12 metres. How far is Joy from point X ifhe is facing North at present?(a) 27 m (b) 35 m (c) 32 m(d) 42 m (e) None of these

16. Town D is 12 km towards the North of town A. Town C is 15km towards the West of town D. Town B is 15 km towardsthe West of town A. How far and in which direction is townB from town C?(a) 15 km towards North (b) 12 km towards North(c) 3 km towards South (d) 12 km towards South(e) cannot be determined

17. Rahul started from point A and travelled 8 km towards theNorth to point B, he then turned right and travelled 7 km topoint C, from point C he took the first right and drove 5 kmto point D, he took another right and travelled 7 km to pointE and finally turned right and travelled for another 3 km topoint F. What is the distance between point F and B?(a) 1 km (b) 2 km (c) 3 km(d) 4 km (e) None of these

18. Meghna drives 10 km towards South, takes a right turn anddrives 6 km. She then takes another right turn, drives 10 kmand stops. How far is she from the starting point?(a) 16 km (b) 6 km (c) 4 km(d) 12 km (e) None of these

19. Vikas walked 10 metres towards North, took a left turn andwalked 15 metres, and again took a left turn and walked 10metres and stopped walking. Towards which direction washe facing when he stopped walking?(a) South (b) South-West(c) South-East (d) Cannot be determined(e) None of these

DIRECTIONS (Q. 20 & 21) : Study the following informationcarefully to answer these questions.A vehicle starts from point P and runs 10 km towards North. Ittakes a right turn and runs 15 km. It now runs 6 km after taking aleft turn. It finally takes a left turn, runs 15 km and stops at point Q.20. How far is point Q with respect to point P?

(a) 16 km (b) 25 km (c) 4 km(d) 0 km (e) None of these

21. Towards which direction was the vehicle moving before itstopped at point Q?(a) North (b) East (c) South(d) West (e) North-East

22. Raman starts from point P and walks towards South andstops at point Q. He now takes a right turn followed by a leftturn and stops at point R. He finally takes a left turn andstops at point S. If he walks 5 km before taking each turn,towards which direction will Raman have to walk from pointS to reach point Q?

(a) North (b) South (c) West(d) East (e) North-West

23. Town D is to the West of town M. Town R is to the South oftown D. Town K is to the East of town R. Town K is towardswhich direction of town D?(a) South (b) East(c) North-East (d) South-East(e) None of these

24. Mohan walked 30 metres towards South, took a left turnand walked 15 metres. He then took a right turn and walked20 metres. He again took a right turn and walked 15 metres.How far is he from the starting point?(a) 95 metres (b) 50 metres(c) 70 metres (d) Cannot be determined(e) None of these

25. W walked 40 metres towards West, took a left turn andwalked 30 metres. He then took a right turn and walked 20metres, He again took a right turn and walked 30 metres.How far was he from the starting point?(a) 70 metres (b) 60 metres(c) 90 metres (d) Cannot be determined(e) None of these

26. Town D is 13 km towards the East of town A. A bus startsfrom town A, travels 8 km towards West and takes a rightturn. After taking the right turn, it travels 5 km and reachestown B. From town B the bus takes a right turn again, travels21 km and stops. How far and towards which direction mustthe bus travel to reach town D?(a) 13 km towards South (b) 5 km towards West(c) 21 km towards South (d) 5 km towards South(e) None of these

27. A person travels 12 km due North, then 15 km due East, afterthat 15 km due West and then 18 km due South. How far ishe from the starting point?(a) 6 km (b) 12 km (c) 33 km(d) 60 km (e) 65 km

28. In a meeting, the map of a village was placed in such amanner that south-east becomes north, north-east becomeswest and so on. What will south become?(a) North (b) North-east (c) North-west(d) West (e) South

29. A school bus driver starts from the school, drives 2 kmtowards North, takes a left turn and drives for 5 km. He thentakes a left turn and drives for 8 km before taking a left turnagain and driving for further 5 km. The driver finally takes aleft turn and drives 1 km before stopping. How far andtowards which direction should the driver drive to reach theschool again?(a) 3 km towards North (b) 7 km towards East(c) 6 km towards South (d) 6 km towards West(e) 5 km towards North

30. Roma walked 25 metre towards south, took a right turn andwalked 15 metre. She then took a left turn and walked 25meter. Which direction is she now from her starting point?(a) South-east (b) South (c) South-west(d) North-west (e) None of these

Page 23: yoursmahboob.wordpress.com SBI · yoursmahboob.wordpress.com iii P 101 Speed Tests for SBI Bank Clerk Exam 101 Speed Tests for SBI Bank Clerk Exam is revised and updated edition on

yoursmahboob.w

ordpress.com

1. At an enquiry office at a railway station, a passenger wastold that a train for New Delhi has left 15 minutes ago, butafter every 45 minutes a train leaves for New Delhi. The nexttrain will leave at 8.30 p.m. At what time was this informationgiven to the passanger ?(a) 7.45 pm (b) 8.00 pm (c) 8.15 pm(d) 8.05 pm (e) None of these

2. A watch is a minute slow at 1 p.m. on Tuesday and 2 minutesfast at 1 p.m. on Thursday. When did it show the correct time?(a) 1:00 a.m. on Wednesday(b) 5:00 a.m. on Wednesday(c) 1:00 p.m. on Wednesday(d) 5:00 p.m. on Wednesday(e) None of these

3. Reaching the place of meeting on Tuesday 15 minutes before8.30 hours, Anuj found himself half an hour earlier than theman who was 40 minutes late. What was the scheduled timeof the meeting?(a) 8.00 hrs (b) 8.05 hrs (c) 8.15 hrs(d) 8.45 hrs (e) None of these

4. A clock gaining 2 min every hour was synchronised at midnightwith a clock losing 1 min every hour. How many minutes behindwill its minute hand be at eleven the following morning ?(a) 23 (b) 27 (c) 22(d) None of these (e)

5. Samant remembers that his brother’s birthday is after fifteenthbut before eighteenth of February, whereas his sisterremembers that her brother’s birthday is after sixteenth butbefore nineteenth of February. On which date of February isSamant’s brother’s birthday?(a) 18th (b) 16th (c) 19th(d) 17th (e) None of these

6. A is shorter than B but much taller than E. C is the tallest andD is little shorter than A. Who is the shortest ?(a) A (b) E (c) C(d) D (e) None of these

7. In a class of 35 students Kiran is placed 7th from the bottomwhereas Sohan is placed 9th from the top. Mohan is placed exactlyin between the two. What is Kiran’s position from Mohan?(a) 10 (b) 11 (c) 13(d) 12 (e) None of these

RESPONSE

GRID

1. a b c d e 2. a b c d e 3. a b c d e 4. a b c d e 5. a b c d e

6. a b c d e 7. a b c d e 8. a b c d e 9. a b c d e 10. a b c d e

11. a b c d e 12. a b c d e 13. a b c d e 14. a b c d e 15. a b c d e

Max. Marks : 32 No. of Qs. 32 Time : 20 min. Date : ........./......../................

DIRECTIONS (Qs. 8-11) : Read the following information carefullyand answer the question that follow:Six boys A, B, C, D, E and F are marching in a line. They arearranged according to their height, the tallest being at the backand the shortest in front,F is between B and A.E is shorter than D but taller than C who is taller than A.E and F have two boys between them.A is not the shortest among them.8. Where is E ?

(a) Between A and B (b) Between C and A(c) Between D and C (d) In front of C(e) None of these

9. Who is the tallest ?(a) B (b) D (c) F(d) A (e) None of these

10. If we start counting from the shortest, which boy is fourthone in the line ?(a) E (b) A (c) D(d) C (e) None of these

11. Who is the shortest ?(a) C (b) D (c) B(d) F (e) None of these

12. Five newly born babies were weighed by the doctor. In herreport, she stated that child A is lighter than child B, child Cis lighter than D and child B is lighter than child D, butheavier than child E. Which child is the heaviest?(a) E (b) D (c) C(d) A (e) None of these

13. Thirty-six vehicles are parked in a parking in a single row.After the first car, there is one scooter. After the second car,there are two scooters. After the third car, there are threescooters and so on. Work out the number of scooters in thesecond half of the row.(a) 10 (b) 12 (c) 15(d) 17 (e) None of these

14. In a row at a bus stop, A is 7th from the left and B is 9th fromthe right. Both of them interchange their positions and thusA becomes 11th from the left. How many people are there inthat row?(a) 18 (b) 19 (c) 20(d) 21 (e) None of these

15. A, B, C, D and E when arranged in descending order of theirweight from the top, A becomes third, E is between D and A whileC and D are not at the top. Who among them is the second?(a) C (b) B (c) E(d) Data inadequate (e) None of these

888Time, Sequence andRanking Test

Page 24: yoursmahboob.wordpress.com SBI · yoursmahboob.wordpress.com iii P 101 Speed Tests for SBI Bank Clerk Exam 101 Speed Tests for SBI Bank Clerk Exam is revised and updated edition on

yoursmahboob.w

ordpress.com

16 SPEED TEST 8

RESPONSE

GRID

16. a b c d e 17. a b c d e 18. a b c d e 19. a b c d e 20. a b c d e

21. a b c d e 22. a b c d e 23. a b c d e 24. a b c d e 25. a b c d e

26. a b c d e 27. a b c d e 28. a b c d e 29. a b c d e 30. a b c d e

31. a b c d e 32. a b c d e

DIRECTIONS (Qs. 16 & 17) : Read the relationships givenbelow to answer the questions that follow.Archana is elder than Suman. Arti is elder than Archana butyounger than Kusum. Kusum is elder than Suman. Suman isyounger than Arti and Gita is the eldest.

16. Who is the youngest ?(a) Archana (b) Suman (c) Arti(d) Kusum (e) None of these

17. Age wise, who is in the middle ?(a) Suman (b) Archna (c) Arti(d) Kusum (e) None of these

18. Five boys took part in a race. Raj finished before Mohit butbehind Gaurav. Ashish finished before Sanchit but behindMohit. Who won the race?(a) Raj (b) Gaurav (c) Mohit(d) Ashish (e) None of these

19. Five men A, B, C, D and E read newspaper. The one whoreads first gives it to C. The one who reads last had taken itfrom A. E was not the first or the last to read. There were tworeaders between B and A.B passed the newspaper to whom?(a) A (b) C (c) D(d) E (e) None of these

20. In a row of children, Bhushan is seventh from the left andMotilal is fourth from the right. When Bhushan and Motilalexchange positions, Bhushan will be fifteenth from the left.What will be Motilal’s position from the right ?(a) Eighth (b) Fourth (c) Eleventh(d) Twelfth (e) None of these

21. In a queue I am the last person while my friend is seventhfrom the front. If the person exactly between me and myfriend is on the 23rd position from the front, what is myposition in the queue ?(a) 37 (b) 36 (c) 38(d) 39 (e) None of these

22. Pratap correctly remembers that his mother's birthday isbefore twentythird April but after nineteenth April, whereashis sister correctly remembers that their mother's birthday isnot on or after twentysecond April. On which day in April isdefinitely their mother's birthday?(a) Twentieth(b) Twentyfirst(c) Twentieth or Twentyfirst(d) Cannot be determined(e) None of these

23. In a row of forty children, R is eleventh from the right endand there are fifteen children between R and M. What is M'sposition from the left end of the row?(a) Fourteenth (b) Fifteenth(c) Thirteenth (d) Cannot be determined(e) None of these

24. In a row of twenty-five children facing North, W is fifth tothe right of R, who is sixteenth from the right end of the row.What is W's position from the right end of the row?(a) Eleventh (b) Tenth (c) Twelfth(d) Data inadequate (e) None of these

25. Seema's watch is 6 minutes fast and the train, which shouldhave arrived at 7 p.m. was 14 minutes late. What time is it bySeema's watch when the train arrived?(a) 7 : 05 pm (b) 7 : 30 pm (c) 7 : 01 pm(d) 7 : 31 pm (e) None of these

26. In a row of children facing North, Ritesh is twelfth from theleft end. Sudhir who is twenty-second from the right end isfourth to the right of Ritesh. Total how many children arethere in the row?(a) 35 (b) 36 (c) 37(d) 34 (e) None of these

27. Among A, B, C, D and E each reaching school at a differenttime, C reaches before D and A and only after B. E is not thelast to reach school. Who among them reached school last?(a) D (b) A (c) C(d) Data inadequate (e) None of these

28. M is older than R. Q is younger than R and N. N is not as oldas M. Who among M, N, R and Q is the oldest?(a) M (b) R (c) M or R(d) Data inadequate (e) None of these

29. Akshay is 16th from the left end in the row of boys and Vijayis 18th from the right end. Avinash is 11th from Akshaytowards the right end and 3rd from Vijay towards the rightend. How many boys are there in the row?(a) Data inadequate (b) 42 (c) 40(d) 48 (e) None of these

30. Geeta is senior to Shilpa but not to Deepa. Gayatri is juniorto Reepa. No one is senior to Fatima. Who is most junior?(a) Shilpa (b) Geeta (c) Gayatri(d) Data inadequate (e) None of these

31. Among M, N, P, R and T each one has secured differentmarks in an examination. R secured more marks than M andT. N secured less marks than P. Who among them securedthird highest marks?(a) N (b) R (c) M(d) T (e) Data inadequate

32. In a column of girls Kamal is 11th from the front. Neela is3 places ahead of Sunita who is 22nd from the front.How many girls are there between Kamal and Neela in thecolumn?(a) Six (b) Eight(c) Seven (d) Cannot be determined(e) None of these

Page 25: yoursmahboob.wordpress.com SBI · yoursmahboob.wordpress.com iii P 101 Speed Tests for SBI Bank Clerk Exam 101 Speed Tests for SBI Bank Clerk Exam is revised and updated edition on

yoursmahboob.w

ordpress.com

1. A, B, C, D, E, F and G are sitting in a line facing east. C isimmediate right of D. B is at the extreme ends and has E as hisneighbour. G is between E and F. D is sitting third from thesouth end. Who are the persons sitting at the extreme ends?(a) A and E (b) A and B (c) F and B(d) C and D (e) None of these

DIRECTIONS (Qs. 2-4) : Read the following statements to answerthe questions that follow.Nine cricket fans are watching a match in a stadium. Seated in onerow, they are – J, K, L, M, N, O, P, Q and R. L is at the right of M andat third place to the right of N. K is at one end of the row. Q isimmediately next to O and P. O is at the third place to the left of K.J is right next to left of O.2. Who is sitting in the centre of the row ?

(a) L (b) O (c) J(d) Q (e) None of these

3. Who is at the other end of the row ?(a) R (b) J (c) P(d) N (e) None of these

4. Which of the following statements is true ?(a) R and P are neighbours.(b) There is one person between L and O.(c) M is at one extreme end.(d) N is two seats away from J.(e) None of these

5. A, B, C, D, E and F are sitting around a round table. A isbetween E and F, E is opposite D, and C is not in either of theneighbouring seats of E. Who is opposite to B ?(a) F (b) C (c) D(d) None of these (e)

6. Four girls (G1, G2, G3, G4) and three boys (B1, B2, B3) are tosit for a dinner such that no two boys should sit togethernor two girls. If they are successively sitting, what is theposition of B2 and G3?(a) 5th and 6th (b) 4th and 5th(c) 3rd and 4th (d) 2nd and 3rd(e) None of these

DIRECTIONS (Q. 7-10) : Study the following information carefullyand answer the questions given below:W, Y, T, M, R, H and D are seven persons, sitting around a circlefacing the centre. T is fourth to the right of M who is second to theright of R. W is third to the left of R. H is not an immediate neighbourof M. D is not an immediate neighbour of W.

RESPONSE

GRID

1. a b c d e 2. a b c d e 3. a b c d e 4. a b c d e 5. a b c d e6. a b c d e 7. a b c d e 8. a b c d e 9. a b c d e 10. a b c d e11. a b c d e 12. a b c d e 13. a b c d e 14. a b c d e 15. a b c d e

Max. Marks : 30 No. of Qs. 30 Time : 20 min. Date : ........./......../................

7. Who is to the immediate left of H?(a) W (b) T (c) R(d) Data inadequate (e) None of these(e) None of these

8. Who is third to the right of H ?(a) M (b) D (c) Y(d) R (e) None of these(e) None of these

9. Who is third to the right of D?(a) M (b) R (c) W(d) M (e) None of these(e) None of these

10. What is Y's position with respect toT?(a) Third to the right (b) Fourth to the left(c) Third to the left (d) Second of the left(e) None of these

DIRECTIONS (Q.11-15): Study the following information carefullyto answer these questions :Eight friends J, K, L, M, N, O, P and Q are sitting around a circlefacing the centre. J is not the neighour of N. L is third to the rightof K. Q is second to the left of N who is next to the right of L. O isnot the neighbour of N or K and is to the immediate left of P.11. Which of the following is the correct position of L?

(a) To the immediate right of N(b) To the immediate right of Q(c) To the immediate left of N(d) To the immediate left of Q(e) None of these

12. Which of the following pair of persons represent O'sneighbours?(a) L&N (b) P&K (c) M&P(d) N&P (e) None of these

13. Which of the following groups has the first person sittingbetween the other two persons?(a) PKJ (b) JQL (c) QNL(d) LMN (e) None of these

14. Who is to the immediate right of K?(a) J (b) P(c) Q (d) Cannot be determined(e) None of these

15. Who is to the immediate left of O?(a) P (b) L (c) Q(d) M (e) None of these

9899Sitting

Arrangement

Page 26: yoursmahboob.wordpress.com SBI · yoursmahboob.wordpress.com iii P 101 Speed Tests for SBI Bank Clerk Exam 101 Speed Tests for SBI Bank Clerk Exam is revised and updated edition on

yoursmahboob.w

ordpress.com

18 SPEED TEST 9

RESPONSE

GRID

16. a b c d e 17. a b c d e 18. a b c d e 19. a b c d e 20. a b c d e

21. a b c d e 22. a b c d e 23. a b c d e 24. a b c d e 25. a b c d e

26. a b c d e 27. a b c d e 28. a b c d e 29. a b c d e 30. a b c d e

DIRECTIONS (Q. 16 - 20): These questions are based on thebasis of following information. Study it carefully and answer thequestions.

Eight executives J, K, L, M, N, O, P and Q are sitting around acircular table for a meeting. J is second to the right of P who isthird to the right of K. M is second to the left of O who sitsbetween P and J. L is not a neighbour of K or N.

16. Who is to the immediate left of L?(a) Q (b) O (c) K(d) N (e) None of these

17. Who is to the immediate left of K?(a) N (b) J(c) Q (d) Cannot be determined(e) None of these

18. Which of the following is the correct position of N?(a) Second to the right of K(b) To the immediate left of K(c) To the immediate right of M(d) To the immediate right of K(e) None of these

19. Who is third to the right of P ?(a) L (b) J (c) Q(d) N (e) None of these

20. Which of the following groups of persons have the firstperson sitting between the other two?(a) PJO (b) OPJ (c) OPM(d) MPO (e) None of these

DIRECTIONS (21-25) : Study the following informationcarefully to answer these questions.

A, B, C, D, E, F, G and H are sitting around a circle facing thecentre. F is third to the right of C and second to the left of H. D isnot an immediate neighbour of C or H. E is to the immediate rightof A, who is second to the right of G.

21. Who is second to the left of C?(a) A (b) B (c) E(d) D (e) None of these

22. Who is to the immediate right of C?(a) E (b) B (c) D(d) B or D (e) None of these

23. Which of the following pairs of persons has first personsitting to the right of the second person?(a) CB (b) AE (c) FG(d) HA (e) DB

24. Who sits between G & D?(a) H (b) D (c) F(d) E (e) None of these

25. Which of the following is the correct position of B withrespect to H?I Second to the rightII Fourth to the rightIII Fourth to the leftIV Second to the left(a) Only I (b) Only II (c) Only III(d) Both II & III (e) None of these

DIRECTIONS (Q. 26 - 30) : Study the following informationcarefully and answer the given questions:

Eight friends P, Q, R, S, T, V, W and Y are sitting around a squaretable in such a way that four of them sit at four corners of thesquare while four sit in the middle of each of the four sides. Theones who sit at the four corners face the centre while those whosit in the middle of the sides face outside.P, who faces the centre, sits third to the right of V. T, who faces thecentre, is not an immediate neighbour of V. Only one person sitsbetween V and W. S sits second to right of Q. Q faces the centre.R is not an immediate neighbour of P.26. Who sits second to the left of Q?

(a) V (b) P (c) T(d) Y (e) Cannot be determined

27. What is the position of T with respect to V?(a) Fourth to the left (b) Second to the left(c) Third to the left (d) Third to the right(e) Second to the right

28. Four of the following five are alike in a certain way and soform a group. Which is the one that does not belong to thatgroup?(a) R (b) W (c) V(d) S (e) Y

29. Which of the following will come in place of the questionmark based upon the given seating arrangement?WP TR QW RS ?(a) Y T (b) V Y (c) V Q(d) P Y (e) Q V

30. Which of the following is true regarding R?(a) R is an immediate neighbour of V.(b) R faces the centre.(c) R sits exactly between T and S.(d) Q sits third to left of R(e) None of these

Page 27: yoursmahboob.wordpress.com SBI · yoursmahboob.wordpress.com iii P 101 Speed Tests for SBI Bank Clerk Exam 101 Speed Tests for SBI Bank Clerk Exam is revised and updated edition on

yoursmahboob.w

ordpress.com

DIRECTIONS (Qs. 1 - 5) : The questions below are based on thefollowing statements.Asha and Charu are good in Mathematics and Athletics. Deepaand Asha are good in Athletics and Studies, Charu and Beena aregood in General Knowledge and Mathematics. Deepa, Beena andEla are good in Studies and General Knowledge. Ela and Deepaare good in Studies and Arts.

1. Who is good in Studies, General Knowledge, Athletics & Arts?(a) Asha (b) Beena (c) Charu(d) Deepa (e) None of these

2. Who is good in Studies, General Knowledge and Mathematics?(a) Asha (b) Beena (c) Charu(d) Deepa (e) None of these

3. Who is good in Studies, Mathematics and Athletics ?(a) Asha (b) Beena (c) Charu(d) Deepa (e) None of these

4. Who is good in Athletics, General Knowledge and Mathematics?(a) Asha (b) Beena (c) Charu(d) Deepa (e) None of these

5. Who is good in Studies, General Knowledge and Arts butnot in Athletics ?(a) Asha (b) Beena (c) Charu(d) Ela (e) None of these

DIRECTIONS (Qs. 6 & 7) : Study the information given belowcarefully and then answer the questions that follow :Students joining certain university can choose from among sevencourses : Biology, Astronomy, English, Mathematics, History,Chemistry and Psychology.A student cannot take both English and Astronomy.Chemistry is a prerequisite for Biology and cannot be takenconcurrently with Biology. A student must take at least two of thethree courses, English, History and Psychology. No course maybe repeated.

6. What is the maximum number of courses that can a studentcan take without violating any of the conditions given above?(a) 3 (b) 4 (c) 5(d) 6 (e) None of these

7. If a student is taking Astronomy and Mathematics andwishes to take four courses, then in how many differentways can the other two courses be taken?(a) One (b) Two (c) Three(d) Four (e) None of these

RESPONSE

GRID

1. a b c d e 2. a b c d e 3. a b c d e 4. a b c d e 5. a b c d e

6. a b c d e 7. a b c d e 8. a b c d e 9. a b c d e 10. a b c d e

11. a b c d e 12. a b c d e 13. a b c d e 14. a b c d e 15. a b c d e

Max. Marks : 30 No. of Qs. 30 Time : 20 min. Date : ........./......../................

DIRECTIONS (Qs. 8 - 12) : Study the information given below toanswer the questions that follow :(i) There is a family of 5 persons A, B, C, D and E.(ii) They are working as a doctor, a teacher, a trader, a lawyer

and a farmer.(iii) B, an unmarried teacher, is the daughter of A.(iv) E, a lawyer, is the brother of C.(v) C is the husband of the only married couple in the family.(vi) Daughter-in-law of A is a doctor.

8. Which of the following is a group of female members in thefamily ?(a) A and D (b) D and E (c) A, C and E(d) B and D (e) None of these

9. Which of the following is the married couple ?(a) A and B (b) C and D (c) A and D(d) B and C (e) None of these

10. Which of the following is a group of male members in thefamily ?(a) A, B and C (b) B and D (c) A, C and E(d) A, C and D (e) None of these

11. Who is the doctor in the family ?(a) A (b) B (c) C(d) D (e) None of these

12. Who is the trader in the family ?(a) A (b) B (c) C(d) D (e) None of these

DIRECTIONS (Qs. 13 - 16) : Study the infomation given below toanswer these questions :There are four friends A, B, C and D. One of them is a Cricketer andstudies Chemistry and Biology. A and B play Football. BothFootball players study Maths. D is a Boxer. One football playeralso studies physics. The Boxer studies Maths and Accounts. Allthe friends study two subjects each and play one game each.

13. Who is the Cricketer ?(a) A (b) B (c) C(d) D (e) None of these

14. Who studies Accounts and plays Football ?(a) A alone (b) B alone (c) D(d) A or B (e) None of these

15. Who studies Physics?(a) A or B (b) A alone (c) B alone(d) D (e) None of these

1081010AnalyticalPuzzle - I

Page 28: yoursmahboob.wordpress.com SBI · yoursmahboob.wordpress.com iii P 101 Speed Tests for SBI Bank Clerk Exam 101 Speed Tests for SBI Bank Clerk Exam is revised and updated edition on

yoursmahboob.w

ordpress.com

20 SPEED TEST 10

RESPONSE

GRID

16. a b c d e 17. a b c d e 18. a b c d e 19. a b c d e 20. a b c d e

21. a b c d e 22. a b c d e 23. a b c d e 24. a b c d e 25. a b c d e

26. a b c d e 27. a b c d e 28. a b c d e 29. a b c d e 30. a b c d e

16. How many games are played and subjects studied by thefour friends ?(a) 1 game and 4 subjects(b) 2 games and 3 subjects(c) 3 games and 4 subjects(d) 3 games and 5 subjects(e) None of these

DIRECTIONS (Qs. 17 - 21) : Study the information given belowto answer these questions :(i) Six plays A, B, C, D, E and F are to be organised from

Monday to Saturday, i.e from 5th to 10th-one play each day.(ii) There are two plays between C and D and one play between

A and C.(iii) There is one play between F and E and E is to be organised

before F.(iv) B is to be organised before A, not necessarily immediately.(v) The organisation does not start with B

17. The organisation would start from which play ?(a) A (b) F(c) D (d) Cannot be determined(e) None of these

18. On which date the play E is to be organised ?(a) 5th (b) 7th(c) 6th (d) Cannot be determined(e) None of these

19. The organisation would end with which of the followingplays?(a) A (b) D(c) B (d) Cannot be determined(e) None of these

20. On which day the play B is organised ?(a) Tuesday (b) Friday(c) Thursday (d) None of these(e) None of these

21. Which of the following is the correct sequence of organisingplays?(a) AECFBD (b) DFECBA (c) BDEFCA(d) None of these (e)

DIRECTIONS (Qs. 22-25) : Read the following information toanswer these questions :In a school, there were five teachers. A and B were teaching Hindiand English. C and B were teaching English and Geography. Dand A were teaching Mathematics and Hindi. E and B were teachingHistory and French.

22. Who among the teachers was teaching maximum number ofsubjects?(a) A (b) C (c) B(d) D (e) None of these

23. Which of the following pairs was teaching both Geographyand Hindi?(a) A and B (b) C and A (c) B and C(d) None of these (e)

24. More than two teachers were teaching which subject?(a) History (b) French (c) Hindi(d) Geography (e) None of these

25. D, B and A were teaching which of the following subjects?(a) English only (b) Hindi only(c) Hindi and English (d) English and Geography(e) None of these

DIRECTIONS (26-30) : Study the following information carefullyand answer the given questions.Seven flights namely those of Jet Airways, British Airways, Delta,Quantas, Emirates, Lufthansa and Air India are scheduled to fly toLondon. There is only one flight to London on each of the sevendays of the week, starting from Monday and ending on Sunday.Delta flies on Wednesday. Air India flies the day next to BritishAirways. British Airways does not fly on Monday or Friday. Twoairlines fly between the days British Airways and Emirates fly.Emirates does not fly on Sunday. Quantas flies a day beforeLufthansa.

26. On which of the following days does Jet Airways fly ?(a) Friday (b) Sunday (c) Tuesday(d) Thursday (e) None of these

27. How many flights fly between Lufthansa and Delta ?(a) None (b) One (c) Two(d) Three (e) Five

28. Which of the following flights flies on Friday ?(a) Air India (b) Quantas (c) Emirates(d) Lufthansa (e) Jet Airways

29. If Delta postpones its flight to Sunday owing to some technicalreasons and all the flights scheduled for Thursday to Sundayare now made to take off a day ahead of the schedule, which ofthe following flights would now fly on Friday?(a) Lufthansa (b) Jet Airways(c) British Airways (d) Air India(e) Quantas

30. If Emirates is related to British Airways and Delta is relatedto Quantas in a certain way based upon the given flightschedule, then Jet Airways will be related to which of thefollowing based upon the same relationship ?(a) Lufthansa (b) Quantas (c) Delta(d) Air India (e) None of these

Page 29: yoursmahboob.wordpress.com SBI · yoursmahboob.wordpress.com iii P 101 Speed Tests for SBI Bank Clerk Exam 101 Speed Tests for SBI Bank Clerk Exam is revised and updated edition on

yoursmahboob.w

ordpress.com

DIRECTIONS (Q.1- 5) : Read the following passage carefullyand answer these questions given below it.

A group of seven friends A, B, C, D, E, F and G, work as Economist,Agriculture Officer, IT Officer, Terminal Operator, Clerk, ForexOfficer and Research Analyst, for Banks L, M, N, P, Q, R and S, butnot necessarily in the same order. C works for Bank N and isneither Research Analyst nor a Clerk. E is an IT Officer and worksfor Bank R.A works as Forex Officer and does not work for Bank Lor Q. The one who is an Agricultural Officer works for Bank M.The one who works for Bank L works as a Terminal Operator. Fworks for Bank Q. G works for Bank P as a Research Analyst. D isnot an Agricultural Officer.1. Who amongst the following works as an Agriculture Officer?

(a) C (b) B (c) F(d) D (e) None of these

2. What is the profession of C?(a) Terminal Operator (b) Agriculture Officer(c) Economist (d) Cannot be determined(e) None of these

3. For which bank does B work?(a) M (b) S (c) L(d) Either M or S (e) None of these

4. What is the profession of the person who works for Bank S?(a) Clerk (b) Agriculture Officer(c) Terminal Operator (d) Forex Officer(e) None of these

5. Which of the following combinations of person, professionand bank is correct?(a) A - Forex Officer - M(b) D - Clerk - L(c) F - Agriculture Officer - Q(d) B - Agriculture Officer - S(e) None of these

DIRECTIONS (Q.6-10) : Study the following information toanswer the given questions.

Each of seven plays viz. P, Q, R, S, T, V and W are scheduled to bestaged on a different day of a week starting from Monday andending on Sunday of the same week. Play V is scheduled onThursday. Two plays are scheduled to be held between Play Vand Play P. Only one play is scheduled between Play T and Play S.Play T is not scheduled on the day immediately before orimmediately after the day when Play V is scheduled. Play R isscheduled the day immediately before the day when Play W isscheduled. Play S is not scheduled after Play Q.6. How many plays are scheduled to be staged between Play R

and Play S?

(a) None (b) One (c) Two(d) Three (e) Four

7. Which of the following plays is scheduled on Saturday?(a) Q (b) W (c) R(d) S (e) T

8. R is related to S in a certain way. In the same way P isrelated to V based on the given schedule. Which of thefollowing is W related to the following the same pattern?(a) P (b) Q (c) R(d) T (e) Cannot be determined

9. On which of the following days is Play W scheduled?(a) Monday (b) Tuesday(c) Wednesday (d) Saturday(e) Cannot be determined

10. Which of the following plays is scheduled on Friday?(a) R (b) T (c) Q(d) W (e) S

DIRECTIONS (Q.11-Q.15) : Study the following informationcarefully and answer the questions given below:

P, Q, R, S, T, V and W are seven friends working in a call centre.Each of them has different day offs in a week from Monday toSunday not necessarily in the same order. They work in threedifferent shifts I, II and III with at least two of them in each shift.

R works in shift II and his day off is not Sunday. P's day offis Tuesday and he does not work in the same shift with either Q orW. None of those who work in shift I has day off either onWednesday or on Friday. V works with only T in shift III. S's dayoff is Sunday. V's day off is immediate next day of that of R's dayoff. T's day off is not on Wednesday. W's day off is not on theprevious day of P's day off. S works in shift I. Q does not work inthe same shift with R and his day off is not on Thursday.11. Which of the following is W's day off?

(a) Tuesday (b) Monday (c) Saturday(d) Data inadequate (e) None of these

12. Which of the following is R's day off?(a) Friday (b) Thursday (c) Tuesday(d) Wednesday (e) None of these

13. Which of the following groups of friends work in shift II?(a) RP (b) RV (c) QWS(d) Data inadequate (e) None of these

14. Which of the following is Q's day off?(a) Friday (b) Wednesday (c) Thursday(d) Monday (e) None of these

15. Which of the following groups of friends work in shift I?(a) RV (b) RP (c) QWS(d) Data inadequate (e) None of these

RESPONSE

GRID

1. a b c d e 2. a b c d e 3. a b c d e 4. a b c d e 5. a b c d e

6. a b c d e 7. a b c d e 8. a b c d e 9. a b c d e 10. a b c d e

11. a b c d e 12. a b c d e 13. a b c d e 14. a b c d e 15. a b c d e

Max. Marks : 30 No. of Qs. 30 Time : 20 min. Date : ........./......../................

1111AnalyticalPuzzle - II

Page 30: yoursmahboob.wordpress.com SBI · yoursmahboob.wordpress.com iii P 101 Speed Tests for SBI Bank Clerk Exam 101 Speed Tests for SBI Bank Clerk Exam is revised and updated edition on

yoursmahboob.w

ordpress.com

22 SPEED TEST 11

DIRECTIONS (Q.16-20) : Study the following informationcarefully and answer the questions given below:

(i) A, B, C, D, E, F, G and H are eight students, each having adifferent height

(ii) D is shorter than A but taller than G.(iii) E is taller than H but shorter than C.(iv) B is shorter than D but taller than F.(v) C is shorter than G.(vi) G is not as tall as F.16. Which of the following is definitely false?

(a) G is shorter than F (b) C is shorter than F(c) F is taller than C. (d) B is taller than E.(e) All are true

17. If another student J, who is taller than E but shorter than G,is added to the group, which of the following will be definitelytrue?(a) C and J are of the same height(b) J is shorter than D.(c) J is shorter than H.(d) J is taller than A.(e) None of these

18. Which of the following will definitely be the third from topwhen the eight students are arranged in descending orderof height?(a) B (b) F (c) G(d) B or G (e) Cannot be determined

19. How many of them are definitely shorter than F?(a) Three (b) Four (c) Five(d) Data inadequate (e) None of these

20. Which of the following is redundant to answer all the abovequestions?(a) (ii) only(b) (ii) and (iii) only(c) (iii) and (iv) only(d) (ii) and (v) only(e) All are necessary to answer the above questions.

DIRECTIONS (Q.21-Q.25) : Study the following informationcarefully and answer the questions given below:

A, B, C, D, E, F, G and H are eight employees of an organizationworking in three departments, viz Personnel, Administration andMarketing with not more than three of them in any department.Each of them has a different choice of sports from football, cricket,volleyball, badminton, lawn tennis, basketball, hockey and tabletennis, not necessarily in the same order.D works in Administration and does not like either football orcricket. F works in Personnel with only A, who likes table tennis.E and H do not work in the same department as D. C likes hockeyand does not work in Marketing. G does not work in Administrationand does not like either cricket or badminton. One of those whowork in Administration likes football. The one who likes volleyballworks in Personnel. None of those who work in Administrationlikes either badminton or lawn tennis. H does not like cricket.

21. Which of the following groups of employees work inAdministration department?(a) EGH (b) AF (c) BCD(d) BGD (e) Data inadequate

22. In which department does E work?(a) Personnel (b) Marketing(c) Administration (d) Data inadequate(e) None of these

23. Which of the following combinations of employeesdepartment-favourite sport is correct?(a) E-Administration-Cricket(b) F-Personnel-Lawn Tennis(c) H-Marketing-Lawn Tennis(d) B-Administration-Table Tennis(e) None of these

24. What is E's favourite sport?(a) Cricket (b) Badminton (c) Basketball(d) Lawn Tennis (e) None of these

25. What is G's favourite sport?(a) Cricket (b) Badminton (c) Basketball(d) Lawn Tennis (e) None of these

DIRECTIONS (Qs. 26-30): Study the following informationcarefully and answer the given questions.

P, Q, R, S, T, V and W are seven friends, all of a different heightand fatness.(i) Q is the thinnest and tallest among them.(ii) S is not as short as T, but shorter than R.(iii) W is not as fat as R, but fatter than V.(iv) R and T are taller than W but shorter than R(v) V is fatter than T and the shorter among them.(vi) R is third among them in fatness in descending order.

26. Who is the fattest among them?(a) P (b) S(c) Either P or S (d) Either T or S(e) None of these

27. Which of the given statements is not required to find outthe thickest among them?(a) (i) (b) (vi)(c) (iii) and (iv) (d) (v)(e) None of these

28. If they are made to stand in ascending order of their heights,who will come in the middle?(a) R (b) S (c) T(d) Data inadequate (e) None of these

29. If they are made to stand in ascending order of their fatness,who will be the second from the last?(a) S (b) V (c) W(d) Data inadequate (e) None of these

30. Who obtained the same place in height and fatness amongthem when arranged in ascending order of their height andfatness?(a) P (b) Q (c) R(d) S (e) None of these

RESPONSE

GRID

16. a b c d e 17. a b c d e 18. a b c d e 19. a b c d e 20. a b c d e

21. a b c d e 22. a b c d e 23. a b c d e 24. a b c d e 25. a b c d e

26. a b c d e 27. a b c d e 28. a b c d e 29. a b c d e 30. a b c d e

Page 31: yoursmahboob.wordpress.com SBI · yoursmahboob.wordpress.com iii P 101 Speed Tests for SBI Bank Clerk Exam 101 Speed Tests for SBI Bank Clerk Exam is revised and updated edition on

yoursmahboob.w

ordpress.com

1. The diagram represent the student who are singers, dancersand poets.

P Q

R

S

TU

Singers Dancers

Poets

V

Study the diagram and identify the region which representthe students who are both poets and singers but not dancer.(a) P + T + S (b) T(c) T + V + R + S (d) P + T + U + S(e) None of these

2.

ab

cd

e

g

f

School children

Artist

Singers

Above diagram represents school children, artist and singers.Study the diagram and identify the region. Which representsthose school children who are artist not singers.(a) a (b) b(c) f (d) e(e) None of these

DIRECTIONS (Qs. 3-9) : The following question are based onthe diagram given below.

6

7

8

9

10

11

12 13

144

5

3

RESPONSE

GRID

1. a b c d e 2. a b c d e 3. a b c d e 4. a b c d e 5. a b c d e

6. a b c d e 7. a b c d e 8. a b c d e 9. a b c d e 10. a b c d e

Max. Marks : 30 No. of Qs. 30 Time : 20 min. Date : ........./......../................Rectangle represents malesTriangle represents educatedCircle represents urbanSquare represents civil servants

3. Who among the following is an educated male who is not anurban resident?(a) 4 (b) 5 (c) 11(d) 9 (e) None of these

4. Who among the following is neither a civil servant noreducated but is urban and not a male?(a) 2 (b) 3 (c) 6(d) 10 (e) None of these

5. Who among the following is a female urban resident andalso a civil servant?(a) 6 (b) 7 (c) 10(d) 13 (e) None of these

6. Who among the following is an educated male who hailsfrom urban, a civil servants?(a) 6 (b) 7 (c) 10(d) 13 (e) None of these

7. Who among the following is uneducated and also an urbanmale?(a) 2 (b) 3 (c) 11(d) 12 (e) None of these

8. Who among the following is only a civil servant but neithera male or urban oriented and uneducated?(a) 7 (b) 8 (c) 9(d) 14 (e) None of these

9. Who among the following is a male urban oriented and alsoa civil servant but not educated?(a) 13 (b) 12 (c) 6(d) 10 (e) None of these

10. Which of the following diagram represents the relationshipamong sun, moon and stars?

(a) (b) (c) (d) (e)

12111081212Venn Diagrams

Page 32: yoursmahboob.wordpress.com SBI · yoursmahboob.wordpress.com iii P 101 Speed Tests for SBI Bank Clerk Exam 101 Speed Tests for SBI Bank Clerk Exam is revised and updated edition on

yoursmahboob.w

ordpress.com

24 SPEED TEST 12

RESPONSE

GRID

11. a b c d e 12. a b c d e 13. a b c d e 14. a b c d e 15. a b c d e

16. a b c d e 17. a b c d e 18. a b c d e 19. a b c d e 20. a b c d e

21. a b c d e 22. a b c d e 23. a b c d e 24. a b c d e 25. a b c d e

26. a b c d e 27. a b c d e 28. a b c d e 29. a b c d e 30. a b c d e

DIRECTIONS (Qs. 11-14) : Refer to the following Venndiagram :

16

12

920

1314

9

15

18

13 16

19

GEOGRAPHY

HISTORY

MATHEMATICS

SCIENCE

18

11. The number of students who took any three of the abovesubjects was

(a) 62 (b) 63

(c) 64 (d) 66(e) None of these

12. The number of students in total, who took History orMathematics or Science, was

(a) 183 (b) 190

(c) 424 (d) 430(e) None of these

13. The number of students who took both History andGeography among other subjects was

(a) 62 (b) 63

(c) 65 (d) 66(e) None of these

14. Which subject was taken by the largest number of students?

(a) Mathematics (b) Science

(c) Geography (d) History(e) None of these

DIRECTIONS (Qs. 15-30) : In each of these questions, threewords are related in some way. The relationship among the wordsin question can best represents by one of the five diagram.

(a) (b)

(c) (d)

(e)

15. Teachers, college, students.16. Mothers, Homo sapiens, Woman17. Cabinet, Home Minister, Minister.18. Parrots, Birds, Mice.19. Professors, Researchers, Scientists.20. Men, Rodents, Living beings.21. Parents, Mothers, Fathers.22. Nitrogen, Ice, Air.23. Musicians, Singers, Women24. Elephants, Carnivore, Tiger.25. Fish, Herring, Animal living in water26. Hospital, Nurse, Patient.27. Nose, Hand, Body.28. Rings, Ornaments, Diamond Rings.29. Furniture, Table, Books.30. Indoor games, Chess, Table tennis.

Page 33: yoursmahboob.wordpress.com SBI · yoursmahboob.wordpress.com iii P 101 Speed Tests for SBI Bank Clerk Exam 101 Speed Tests for SBI Bank Clerk Exam is revised and updated edition on

yoursmahboob.w

ordpress.com

DIRECTIONS (Q.1-5) : In each of the questions below are giventhree statements followed by two conclusions numbered I and II.You have to take the given statements to be true even if they seemto be at variance from commonly known facts. Read both of theconclusions and then decide which of the given conclusionslogically follows from the given statements disregarding commonlyknown facts.1. Statements: Some phones are computers.

All computers are radios.All radios are televisions.

Conclusions: I. All televisions are computers.II. Some radios are phones.

(a) None follows (b) Only I follows(c) Only II follows (d) Both I and II follow(e) None of these

2. Statements: All rings are fingers.Some ears are fingers.All ears are necklaces.

Conclusions: I. Some necklaces are fingers.II. Some necklaces are rings.

(a) None follows (b) Only I follows(c) Only II follows (d) Both I and II follow(e) None of these

3. Statements: Some bottles are cups.Some cups are plates.No spoon is a plate.

Conclusions: I. Some spoons are bottles.II. No bottle is a spoon.

(a) None follows (b) Only I follows(c) Only II follows (d) Either I or II follow(e) None of these

4. Statements: All pens are erasers.Some erasers are sharpeners.Some sharpeners are staples

Conclusions: I. Some sharpeners are pens.II. Some staples are erasers.

(a) None follows (b) Only I follows(c) Only II follows (d) Both I and II follow(e) None of these

5. Statements: All hills are trees.All trees are jungles.All jungles are bushes.

Conclusions: I. All trees are bushes.II. Some jungles are hills.

(a) None follows (b) Only I follows(c) Only II follows (d) Both I and II follow(e) None of these

RESPONSE

GRID

1. a b c d e 2. a b c d e 3. a b c d e 4. a b c d e 5. a b c d e

6. a b c d e 7. a b c d e 8. a b c d e 9. a b c d e 10. a b c d e

11. a b c d e

Max. Marks : 30 No. of Qs. 30 Time : 20 min. Date : ........./......../................

DIRECTIONS (6 - 10): In each question below arc two statementsfollowed by two conclusions numbered I and II. You have to takethe two given statements to be true even if they seem to be atvariance from commonly known facts and then decide which ofthe given conclusions logically follows from the given statementsdisregarding commonly known facts.Give answer (a) if only conclusion 1 follows.Give answer (b) if only conclusion II follows.Give answer (c) if either conclusion I or conclusion II follows.Give answer (d) if neither conclusion I nor conclusion II follows.Give answer (e) if both conclusions I and II follow.6. Statements : Some windows arc grills.

All glasses are grills.Conclusions : I. All grills are windows.

II. At least some grills are glasses.7. Statements : Some painters are artists. Some dancers are painters.

Conclusions : I. All artists are dancers.II. All painters are dancers.

8. Statements : All cabins are rooms.All rooms are buildings.

Conclusions: I. All buildings are rooms.II. All cabins are buildings.

9. Statements : All rings are necklaces.No necklace is a bracelet.

Conclusions: I. No ring is a bracelet.II. All necklaces are rings.

10. Statements : All hands are arms.Some hands are muscles.

Conclusions: I. Some muscles are arms.II. All muscles are arms.

DIRECTIONS (Qs. 11-15): In each question below are twostatements followed by two conclusions numbered I and II. Youhave to take the two given statements to be true even if they seemto be at variance from commonly known facts and then decidewhich of the given conclusions logically follows from the givenstatements disregarding commonly known facts. Give answer(a) if only conclusion I follows.(b) if only conclusion II follows.(c) if either conclusion I or II follows.(d) if neither conclusion I nor II follows.(e) if both conclusions I and II follow.11. Statements: No holiday is a vacation.

Some vacations are trips.Conclusions: I. No trip is a holiday.

II. Some holidays are definitely not trips.

1313Syllogism - I

Page 34: yoursmahboob.wordpress.com SBI · yoursmahboob.wordpress.com iii P 101 Speed Tests for SBI Bank Clerk Exam 101 Speed Tests for SBI Bank Clerk Exam is revised and updated edition on

yoursmahboob.w

ordpress.com

26 SPEED TEST 13

RESPONSE

GRID

12. a b c d e 13. a b c d e 14. a b c d e 15. a b c d e 16. a b c d e

17. a b c d e 18. a b c d e 19. a b c d e 20. a b c d e 21. a b c d e

22. a b c d e 23. a b c d e 24. a b c d e 25. a b c d e 26. a b c d e

27. a b c d e 28. a b c d e 29. a b c d e 30. a b c d e

12. Statements: Some kites are birds.No kite is an aeroplane.

Conclusions: I. All aeroplanes are birds.II. Some birds are definitely not kites

13. Statements: All metals are plastics.All plastics are fibres.

Conclusions: I. Atleast some fibres are metals.II. Some metals are not fibres.

14. Statements: All roads are streets.No street is a highway.

Conclusions: I. No highway is a road.II. All streets are roads.

15. Statements: Some animals are plants.All plants are rocks.

Conclusions: I. All plants are animals.II. Atleast some rocks are animals.

DIRECTIONS (Q.16-20) : In each questions below are two / threestatements followed by two conclusions numbered I and II. Youhave to take the two / three given statements to be true even if theyseem to be at variance from commonly known facts and then decidewhich of the given conclusions logically follows from the givenstatements disregarding commonly known facts. Give answer(a) if only conclusion I follows.(b) if only conclusion II follows.(c) if either conclusion I or II follows.(d) if neither conclusion I nor II follows.(e) if both conclusions I and II follows.16. Statements: No holiday is a vacation.

Some vacations are trips.Conclusions: I. No trip is a holiday.

II. Some holidays are definitely not trips.17. Statements: Some kites are birds.

No kite is an aeroplane.Conclusions: I. All aeroplanes are birds.

II. Some birds are definitely not kites.18. Statements: All metals are plastics.

All plastics are fibres.Conclusions: I. At least some fibres are metals

II. Some metals are not fibres.19. Statements: Some animals are plants.

All plants are rocks.Conclusions: I. All plants are animals.

II. Atleast some rocks are animals.20. Statements: Some institutes are banks.

All institutes are academies.All academies are schools.

Conclusions: I. All banks can never be schools.II. Any bank which is an institute in a school.

DIRECTIONS (Qs. 21-27) : In each group of questions below aretwo/three statements followed by two conclusions numbered I andII. You have to take the given statements to be true even if theyseem to be at variance from commonly known facts and then decidewhich of the given conclusions logically follows from the two/threestatements disregarding commonly known facts.Give answer (a) if only conclusion I follows;Give answer (b) if only conclusion II follows;

Give answer (c) if either conclusion I or conclusion II follows;Give answer (d) if neither conclusion I nor conclusion II follows;Give answer (e) if both conclusion I and conclusion II follow.21. Statements : Some exams are tests. No exam is a question.

Conclusions : I. No question is a test.II. Some tests are definitely not exams.

22. Statements : All forces are energies. All energies arepowers. No power is heat.

Conclusions : I. Some forces are definitely not powers.II. No heat is force.

23. Statements : All forces are energies. All energies arepowers. No power is heat.

Conclusions : I. No energy is heat.II. Some forces being heat is a possibility.

24. Statements : No note is a coin. Some coins are metals.All plastics are notes.

Conclusions : I. No coin is plastic.II. All plastics being metals is a possibility.

25. Statements : No note is a coin. Some coins are metals.All plastics are notes.

Conclusions : I. No metal is plastic.II. All notes are plastics.

26. Statements : Some symbols are figures. All symbols aregraphics.No graphic is a picture.

Conclusions : I. Some graphics are figures.II. No symbol is a picture.

27. Statements : All vacancies are jobs. Some jobs areoccupations.

Conclusions : I. All vacancies are occupations.II. All occupations being vacancies is apossibility.

DIRECTIONS (Qs. 28-30) : In each question below are two/threestatements followed by two conclusions numbered I and II. Youhave to take the two/three given statements to be true even ifthey seem to be at variance from commonly known facts and thendecide which of the given conclusions logically follows from thegiven statements disregarding commonly known facts.Give answer (a) if only conclusion I followsGive answer (b) if only conclusion II follows.Give answer (c) if either conclusion I or conclusion II follows.Give answer (d) if neither conclusion I nor conclusion II follows.Give answer (e) if both conclusion I and conclusion II follow.(Qs. 28-29) :Statements : All gliders are parachutes.

No parachute is an airplane.All airplanes are helicopters.

28. Conclusions : I. No glider is an airplane.II.All gliders being helicopters is a possibility.

29. Conclusions : I. No helicopter is a glider.II. All parachutes being helicopters is apossibility.

30. Statements : Some mails are chats.All updates are chats.

Conclusions : I. All mails being updates is a possibility.II. No update is a mail.

Page 35: yoursmahboob.wordpress.com SBI · yoursmahboob.wordpress.com iii P 101 Speed Tests for SBI Bank Clerk Exam 101 Speed Tests for SBI Bank Clerk Exam is revised and updated edition on

yoursmahboob.w

ordpress.com

DIRECTIONS (Q.1-5) : In each of the questions below are giventhree statements followed by three conclusions numbered I , IIand III. You have to take the given statements to be true even ifthey seem to be at variance from commonly known facts. Readall the conclusions and then decide which of the givenconclusions logically follows from the given statementsdisregarding commonly known facts.

1. Statements:Some flowers are bins.Some bins are handlesAll handles are sticks.Conclusions:I. Some sticks are bins.II. Some handles are flowers.III. Some sticks are flowers.(a) Only II follows (b) Only III follows(c) Only I and II follow (d) Only I and III follow(e) None of these

2. Statements:Some towers are windows.All windows are houses.Some houses are templesConclusions:I. Some towers are temples.II. Some houses are towers.III. Some temples are windows.(a) Only I follows (b) Only II follows(c) Only III follows (d) Only I and II follow(e) None of these

3. Statements:Some walls are doors.Some doors are cots.Some cots are chairs.Conclusions:I. Some chairs are doors.II. Some cots are walls.III. No chair is door.(a) Only II follows(b) Only III follows(c) Only either I or III follows(d) Only I follows(e) None of these

4. Statements:All trees are gardens.All gardens are stones.All stones are fences.Conclusions:I. Some fences are gardens.II. All gardens are stones.III. Some stones are trees.(a) Only I and II follows (b) Only I and III follows(c) Only II or III follow (d) All follow(e) None of these

5. Statements:All books are leaves.Some leaves are jungles.No jungle is box.Conclusions:I. Some jungles are books.II. No book is box.III. Some leaves are boxes.(a) None follows (b) Only I follows(c) Only II follows (d) Only III follows(e) Only I and II follow

DIRECTIONS (Q.6-25) : In each questions below are giventwo/three statements followed by two conclusions numbered Iand II. You have to take the given statements to be true even ifthey seem to be at variance with commonly known facts. Read allthe conclusions and then decide which of the given conclusionslogically follows from the given statements disregardingcommonly known facts. Give answer.

(a) if only conclusion I follows.(b) if only conclusion II follows.(c) if either conclusion I or II follows.(d) if neither conclusion I nor II follows.(e) if both conclusions I and II follows.6. Statements:

Some toys are desks.Some desks are pens.All pens are rods.Conclusions:I. Some rods are toys.II. Some pens are toys.

RESPONSE

GRID

1. a b c d e 2. a b c d e 3. a b c d e 4. a b c d e 5. a b c d e

6. a b c d e

Max. Marks : 25 No. of Qs. 25 Time : 18 min. Date : ........./......../................

1414Syllogism - II

Page 36: yoursmahboob.wordpress.com SBI · yoursmahboob.wordpress.com iii P 101 Speed Tests for SBI Bank Clerk Exam 101 Speed Tests for SBI Bank Clerk Exam is revised and updated edition on

yoursmahboob.w

ordpress.com

SPEED TEST 1428

7. Statements:Some table are huts.No hut is ring.All rings are bangles.Conclusions:I. Some bangles are tables.II. No bangles is table.

8. Statements:Some chairs are rooms.All rooms are trees.All trees are poles.Conclusions:I. Some poles are chairs.II. Some trees are chairs.

9-10. Statements:All buildings are houses.No house is an apartment.All apartments are flats.

9. Conclusions:I. No flat is a house.II. No building is an apartment.

10. Conclusions:I. All buildings being flats is a possibility.II. All apartments being building is a possibility.

11-12.Statements:Some oceans are seas.All oceans are rivers.No river is a canal.

11. Conclusions:I. All rivers can never be oceans.II. All canals being oceans is a possibility.

12. Conclusions:I. No ocean is a canal.II. At least some seas are rivers.

13-14. Statements:No day is night.All nights are noon.No noon is an evening.

13. Conclusions:I. No day is noon.II. No day is an evening.

14. Conclusions:I. No evening are nights.II. All days being noon is a possibility.

15-16. Statements:Some papers are boardsNo board is a card.

15. Conclusions:I. No card is a paper.II. Some papers are cards.

16. Conclusions:I. All cards being papers is a possibility.II. All boards being papers is a possibility.

17. Statements: All rings are circles.All squares are rings.No ellipse is a circle.

Conclusions: I. Some, rings being ellipses is a possibility.: II. At least some circles are squares.

18. Statements : No house is an apartment.Some bungalows are apartments.

Conclusions: I. No house is a bungalow.II. All bungalows are houses.

19. Statements: Some gases are liquids.All liquids are water.

Conclusions: I. All gases being water is a possibility.II. All such gases which are not water can

never be liquids.20. Statements: All minutes are seconds.

All seconds are hours.No second is a day.

Conclusions: I. No day is an hour.II. At least some hours are minutes.

(21-22): Statements: Some teachers are professors.Some lecturers are teachers.

21. Conclusions: I. All teachers as well as professors beinglecturers is a possibility.

II. All those teachers who are lecturers arealso professors.

22. Conclusions: I. No professor is a lecturer.II. All lecturers being professors is a

possibility.(23-24):

Statements: Some flowers are red.Some roses are flowers.

23. Conclusions:I. All those flowers which are roses are red.II. No rose is red.

24. Conclusions :I. All roses being red is a possibility.II. Some flowers can never be roses.

25. Statements: Some hills are mountains.All mountains are high.

Conclusions: I. All hills being high is a possibility.II. Some mountains can never be hills.

RESPONSE

GRID

7. a b c d e 8. a b c d e 9. a b c d e 10. a b c d e 11. a b c d e

12. a b c d e 13. a b c d e 14. a b c d e 15. a b c d e 16. a b c d e

17. a b c d e 18. a b c d e 19. a b c d e 20. a b c d e 21. a b c d e

22. a b c d e 23. a b c d e 24. a b c d e 25. a b c d e

Page 37: yoursmahboob.wordpress.com SBI · yoursmahboob.wordpress.com iii P 101 Speed Tests for SBI Bank Clerk Exam 101 Speed Tests for SBI Bank Clerk Exam is revised and updated edition on

yoursmahboob.w

ordpress.com

DIRECTIONS (Qs.1-5): In each question below is given a groupof letters followed by four combinations of digits/symbolsnumbered (a), (b), (c) and (d). You have to find out which of thecombinations correctly represents the group of letters based onthe coding system and the conditions given below and mark thenumber of that combination as your answer. If none of thecombinations correctly represents the group of letters, mark (e)i.e. ‘None of these’ as your answer.

Letters P M A E J K D R W H I U T F Digits/symbols

Conditions 4 $ 1 2 3 # 5 @ © 6 % 7 9d

(i) If the first letter is a consonant and the last letter is a vowel,the codes of both these are to be interchanged.

(ii) If both the first and the last letters are consonants both theseare to be coded as per the code of the last letter.

(iii) If the first letter is vowel and the last letter is a consonantboth these are to be coded as ‘ ’

Note: All the remaining letters are to be coded as per their originalcodes.1. ERWHKA

(a) 2@©6#1 (b) 1@©6#2 (c) 1@©6#I(d) 2@©6#2 (e) None of these

2. MPEKDU(a) $42#5 d (b) $42#5$ (c) d 42#5 d(d) d 425#$ (e) None of these

3. TMEIUF(a) 7$2%d 9 (b) 7$2%d 7 (c) 9$2%d 7(d) 9$2%d 9 (e) None of these

4. JTAERI(a) % 712@ 3 (b) 3712@3 (c) 712@(d) %712@% (e) None of these

5. UKTMIH(a) #7$%6 (b) 6#7$%d (c) #7$ %(d) 7#$%6 (e) None of these

DIRECTIONS (Qs.6-8): In each question below is given a groupof numbers/symbols followed by five combinations of letter codesnumbered (a), (b), (c), (d) and (e). You have to find out which ofthe combinations correctly represents the group of numbers/symbols based on the following coding system and the conditionsand mark the number of that combination as your answer.

RESPONSE

GRID

1. a b c d e 2. a b c d e 3. a b c d e 4. a b c d e 5. a b c d e

6. a b c d e 7. a b c d e 8. a b c d e 9. a b c d e 10. a b c d e

Max. Marks : 25 No. of Qs. 25 Time : 20 min. Date : ........./......../................

Number/9 4 & 5 % 3 # 7 6 @ 8 + 2 $

SymbolsLetter

X P J H B D K F S T N G R LCodes

Conditions:(i) If the first element is a symbol and the last element is a

number, then the codes for both are to be interchanged.(ii) If both the first and last elements are symbols, then the

last element is to be coded as the code for the firstelement.

(iii) If the group of elements contains only one symbol,then that symbols is to be coded as A.

6. 28%956(a) RNBXHS (b) RNAXSH(c) RNBXSH (d) RNAXHS(e) RNASHX

7. ©62+74(a) PSRGFT (b) TSRFGP(c) PSRFGT (d) PRSGFT(e) TSRGFP

8. +5963%(a) GHXSDG (b) GSHXDB(c) GHXDSG (d) GHSXDB(e) GXHSDG

9. In a certain code MODE is written as #8%6 and DEAF iswritten as %67$. How is FOAM written in that code?(a) $87# (b) $#7% (c) #87%(d) $87% (e) None of these

10. In a certain code WEAK is written as 5%9$ and WHEN iswritten as 5*%7. How HANK written in that code?(a) *9$7 (b) 9*$7 (c) $97*(d) 9*7$ (e) None of these

DIRECTIONS (Qs.11-15) : In each of these questions a group ofletters is given followed by four combinations of number/symbolnumbered (a), (b), (c) & (d). Letters are to be coded as per thescheme and conditions given below. You have to find out theserial number of the combination, which represents the letter group.Serial number of that combinations is your answer. If none of thecombinations is correct, your answer is (e) i.e. 'None of these'.

1515Symbols & Codes

Page 38: yoursmahboob.wordpress.com SBI · yoursmahboob.wordpress.com iii P 101 Speed Tests for SBI Bank Clerk Exam 101 Speed Tests for SBI Bank Clerk Exam is revised and updated edition on

yoursmahboob.w

ordpress.com

SPEED TEST 1530

RESPONSE

GRID

11. a b c d e 12. a b c d e 13. a b c d e 14. a b c d e 15. a b c d e

16. a b c d e 17. a b c d e 18. a b c d e 19. a b c d e 20. a b c d e

21. a b c d e 22. a b c d e 23. a b c d e 24. a b c d e 25. a b c d e

D K M B I N P R J A L S E Q G

% 3 7 H 4 @ $ 1 8 5 # 9 2 £ 6

Letters

Number /

Symbol Code

Conditions(i) If the first letter is a consonant and the last a vowel,

both are to be coded as the code of the vowel.(ii) If the first letter is a vowel and the last a consonant, the

codes for the first and the last are to be interchanged.(iii) If no vowel is present in the group of letters, the second

and the fifth letters are to be coded as ã .11. KQAPJE

(a) 3£5$82 (b) 3£58$2 (c) 2£5$82(d) 2£5$83 (e) None of these

12. EMANRB(a) *75@12 (b) 275@1* (c) ã 75@2ã(d) *75@1* (e) None of these

13. JAQDKP(a) 85£%38 (b) $5£%3$ (c) $5£%38(d) $5£3%8 (e) None of these

14. QDBGRM(a) £%*617 (b) $ã *6ã 7 (c) £%*167(d) % £*61ã (e) None of these

15. IKQLMS(a) 43£#74 (b) ã 3£#7ã (c) 4£3#74(d) 93£#74 (e) None of these

DIRECTIONS (Qs. 16-20): In each of these questions, a groupof digits is given followed by four combinations of letters andsymbols numbered (a), (b), (c) and (d). The group of digits is to becoded as per the scheme and conditions given below. The serialnumber of the combination which correctly represents the groupof digits is your answer. If none of the four combinations is correct,your answer is (e) i.e., 'None of these'.

Digits 5 8 4 3 6 2 9 0 7 1Code T J $ # H Q @ L % K

Conditions:(i) If the first as well as the last digit is odd, both are to be

coded as ©.(ii) If the first as well as the last digit is even, their codes are to

be swapped.(iii) If '0' is the last digit, it is to be coded as *.16. 270514

(a) ©%LTK© (b) $%LTKQ(c) Q%LTK$ (d) $%*TKQ(e) None of these

17. 364279(a) ©H$Q%© (b) #H$Q%@(c) ©H$Q%# (d) #H$Q%©(e) None of these

18. 875306(a) J%T#Ll1 (b) H%T#LH(c) H%T#LJ (d) J% oT#LJ(e) None of these

19. 592476(a) H COQ$%T (b) Q$% oT(c) H©Q$%OO (d) CCQ$%o©(e) None of these

20. 468910(l) $HJ©KL (b) LHJ@K$(c) *HJ@K$ (d) $HJ@K*(e) None of these

DIRECTIONS (Qs. 21-24) : In each of the questions given below,a group of digits is given followed by four combinations of letters/symbols numbered (a), (b),(c) and(d). You have to find out whichof the four combinations correctly represents the group of digitsbased on the letter/symbol codes and the conditions given below.If none of the four combinations represents the group of digitscorrectly, give (e) i.e. `None of these' as the answer.

Digit 2 8 3 9 4 7 6 5 1

Code B = T @ K $ © P C

Conditions:(i) If the first digit is odd and last digit is even, the codes

for the first and the last digits are to be interchanged.(ii) If the first as well as the last digit is even, both are to

be coded by the code for last digit.(iii) If the first as well as the last digit is odd, both are to

be coded as X.(iv) If the first digit is even and last digit is odd, both are

to be coded by the code for the first-digit.21. 2976581

(a) B@$©P = B (b) C@$©P = C(c) B@$©P = C (d) C@S©P = B(e) None of these

22. 7269534(a) $BC@PTK (b) KB©@PT$(c) $B©@PT$ (d) KB©@P=$(e) None of these

23. 8135246(a) = CTPBK = (b) ©CTP = K©(c) ©CTPBK© (d) CTPB$ =(e) None of these

24. 4352718(a) XTPB$CX (b) KTPB$C=(c) =TPB$CK (d) KTP$CK(e) None of these

25. if 1 is coded as $, 5 is coded as %, 9 is coded as , 3 is codedas +, 7 is coded as # and 4 is coded as?, what will be thecorrect code of the number 435971?(a) ? + % # $ (b) ? + % $ # (c) ? + % # $ (d) $ # % + ?(e) None of these

Page 39: yoursmahboob.wordpress.com SBI · yoursmahboob.wordpress.com iii P 101 Speed Tests for SBI Bank Clerk Exam 101 Speed Tests for SBI Bank Clerk Exam is revised and updated edition on

yoursmahboob.w

ordpress.com

DIRECTIONS (Q. 1-5) : Answers the questions given belowreferring to the following arrangement:

J * R 3 P L 2 # I N 7 O C @ K 5 D = M $ 6 B < A Q 41. Four of the following five are alike in a certain way as regards

their position in the above arrangement and so form a group.Which is the one that does not belong to that group?(a) 2 3 # (b) O I C (c) K O 5(d) # P I (e) B $ <

2. What will come in the place of the question mark (?) in thefollowing series based on the above arrangement?P R J # L 3 7 1 2 @ O N ?(a) D K C (b) 5 @ O (c) D K @(d) = 5 @ (e) None of these

3. It the above series is re-arranged in the reverse order,which will be the eleventh element to the left to thesixteenth element from the left end?(a) J (b) 6 (c) B(d) < (e) None of these

4. How many such numbers are there in the above arrangementeach of which is immediately preceded by a consonant andnot immediately followed by a symbol?(a) None (b) Two (c) Four(d) Three (e) None of these

5. How many such symbols are there in the above arrangementeach of which is immediately preceded by a number andimmediately followed a consonant?(a) One (b) Two (c) Three(d) More than three (e) None

DIRECTIONS (Qs. 6-10) : Study the following arrangement ofconsonants, vowels, numbers and symbols carefully and answerthe questions given below:

H @ F ! 3 U 6 % G I T * P L 8 $ Ù 9 S 2 7 & A M K + J ©D 4 # 5 & E

6. Which of the following is ninth to the right of the twentiethfrom the right end of the above arrangement ?(a) K (b) M (c) U(d) A (e) None of these

7. How many such consonants are there in the abovearrangement, each of which is immediately preceded by asymbol and also immediately followed by a symbol ?(a) None (b) One (c) Two(d) Three (e) More than three

RESPONSE

GRID

1. a b c d e 2. a b c d e 3. a b c d e 4. a b c d e 5. a b c d e

6. a b c d e 7. a b c d e 8. a b c d e 9. a b c d e 10. a b c d e

11. a b c d e 12. a b c d e 13. a b c d e 14. a b c d e 15. a b c d e

Max. Marks : 30 No. of Qs. 30 Time : 20 min. Date : ........./......../................8. If all the symbols are dropped from the arrangement, which

of the following will be the twelfth from the left end ?(a) 9 (b) 2 (c) S(d) 7 (e) None of these

9. Four of the following five are alike in a certain way based ontheir positions in the above arrangement and so form a group.Which is the one that does not belong to the group?(a) L$8 (b) AKM (c) @!F(d) 6%G (e) JD©

10. What should come in place of the question mark (?) in thefollowing series based on the above arrangement >F3U %IT L$ Ù ?(a) 927 (b) 7&A (c) 7AM(d) 2&A (e) 27&

DIRECTIONS (Qs. 11-18): Study the following arrangement ofletters/symbols and answer the questions given below:

D F J T $ # P R Z Q * C M A B @ H K L S + ?11. How many such symbols are there each of which is

immediately preceded by a symbol and immediately followedby a letter?(a) One (b) Two (c) Three(d) Four (e) None of these

12. If the order of the first half of the arrangement is reversedwhich of the following letters/symbols will be the fifth to theleft of the fifteenth letter/symbol from the left?(a) * (b) Q (c) T(d) J (e) None of these

13. If all the symbols of the above sequence are denoted by 7and each letter is denoted by 5, then what will be the sum ofall the elements of the sequence?(a) 142 (b) 138 (c) 132(d) 122 (e) None of these

14. If all the symbols from the above sequence are dropped,which letter will be seventh to the right of twelfth letter fromthe right?(a) H (b) B (c) K(d) A (e) None of these

15. Which of the following is related to ‘FT’ in the same way as‘DJ’ is related to‘? S’ ?(a) L+ (b) KS (c) HL(d) + L (e) None of these

161616Alpha NumericSequence Puzzle

Page 40: yoursmahboob.wordpress.com SBI · yoursmahboob.wordpress.com iii P 101 Speed Tests for SBI Bank Clerk Exam 101 Speed Tests for SBI Bank Clerk Exam is revised and updated edition on

yoursmahboob.w

ordpress.com

SPEED TEST 1632

RESPONSE

GRID

16. a b c d e 17. a b c d e 18. a b c d e 19. a b c d e 20. a b c d e

21. a b c d e 22. a b c d e 23. a b c d e 24. a b c d e 25. a b c d e

26. a b c d e 27. a b c d e 28. a b c d e 29. a b c d e 30. a b c d e

16. How many such letters are there in the above sequenceeach of which occupies the same position from the left inthe sequence as in the alphabet from left?(a) None (b) One (c) Two(d) Three (e) None of these

17. Four of the following five are alike on the basis of theirposition in the above sequence and hence form a group.Which of the following does not belong to that group?(a) DJ ? (b) T # L (c) FT +(d) PZ @ (e) J # S

18. Which of the following will be exactly midway between fifthelement from the left and eighth element from the right?(a) C (b) * (c) Q(d) M (e) None of these

19. Select the combination of numbers so that letters arrangedaccordingly will form a meaningful word.R A C E T1 2 3 4 5(a) 1, 2, 3, 4, 5 (b) 3, 2, 1, 4, 5(c) 5, 2, 3, 4, 1 (d) 5, 1, 2, 3, 4(e) None of these

20. Select the combination of numbers so that the lettersarranged accordingly will form a meaningful word.V A R S T E(a) 2, 3, 1, 6, 4, 5 (b) 4, 5, 2, 3, 1, 6(c) 6, 3, 4, 5, 2, 1 (d) 3, 2, 4, 5, 6, 1(e) None of these

DIRECTIONS (Qs. 21-25): Study the following arrangementcarefully and answer the questions given below:

J 1 # P 4 E K 3 A D $ R U M 9 N 5 1 % T V * H 2 ÷F 6 G 8 Q W

21. How many such numbers are there in the above arrangement,each of which is either immediately preceded by orimmediately followed by a vowel or both?(a) None (b) One (c) Two(d) Three (e) More than three

22. Which of the following is exactly in the middle between thetenth from the left and the eighth from the right end in theabove arrangement?(a) M (b) N (c) 1(d) 5 (e) None of these

23. If the order of the last fifteen elements in the abovearrangement is reversed, which of the following will be theninth to the right of the eleventh element from. the left end?(a) G (b) % (c) 8(d) 3 (e) None of these

24. How many such consonants are there in the abovearrangement, each of which is immediately preceded by asymbol but not immediately followed by either a number ora vowel?(a) None (b) One (c) Two(d) Three (e) More than three

25. Four of the following five are alike in a certain way based ontheir position in the above arrangement and so form a group.Which is the one that does not belong to that group?(a) A $ E (b) % V N (c) 2 F V(d) 4 K 1 (e) 6 Q ÷

DIRECTIONS (Qs. 26-30): Study the following arrangementcarefully and answer the questions given below:

M £ 5 T R E 3 $ P J 1 7 D 1 2 N A 4 F H 6 * U 9 # V B @ W26. If the positions of the first fourteen characters of the above

arrangement are reversed, which of the following will be thetwenty-second from the right end?(a) J (b) I (c) P(d) 3 (e) None of these

27. How many such numbers are there in the above arrangement,each of which is immediately preceded by a vowel andimmediately followed by a consonant?(a) None (b) One (c) Two(d) Three (e) More than three

28. What should come in place of the question mark (?) in theseries given below based on the above arrangement?R 3 £ P I E ? A F I(a) DNJ (b) D21 (c) IN1(d) N4D (e) None of these

29. How many such consonants are there in the abovearrangement each of which is immediately preceded by asymbol but not immediately followed by a number?(a) None (b) One (c) Two(d) Three (e) More than three

30. Which of the following is the fifth towards right of theseventeenth from the right end?(a) $ (b) 4 (c) 7(d) A (e) None of these

Page 41: yoursmahboob.wordpress.com SBI · yoursmahboob.wordpress.com iii P 101 Speed Tests for SBI Bank Clerk Exam 101 Speed Tests for SBI Bank Clerk Exam is revised and updated edition on

yoursmahboob.w

ordpress.com

DIRECTIONS (Qs. 1-5): Study the following information carefullyand answer the given questions:A word and number arrangement machine when given an inputline of words and numbers rearranges them following a particularrule in each step. The following is an illustration of input andrearrangement. (All numbers are two-digit numbers.)Input : good for everything 19 37 26 all 65Step I : all good for everything 19 37 26 65Step II : all 65 good for everything 19 37 26Step III : all 65 everything good for 19 37 26Step IV : all 65 everything 37 good for 19 26Step V : all 65 everything 37 for good 19 26Step VI : all 65 everything 37 for 26 good 19and Step VI is the last step of the rearrangement as the desiredarrangement is reached.As per the rules followed in the above steps, find out in each ofthe following questions the appropriate step for the given input.(All numbers are two-digit numbers.)1. Input: won 13 now 25 72 please go 47

How many steps will be required to complete the rearrangement?(a) Four (b) Five (c) Six(d) Three (e) None of these

2. Step III of an input is :car 81 desk 15 42 39 tall moreWhich of the following will be Step VI?(a) car 81 desk 42 39 15 tall more(b) car 81 desk 42 15 39 tall more(c) car 81 desk 42 more 39 15 tall(d) There will be no such step.(e) None of these

3. Step II of an input is:bell 53 town hall near 27 43 12How many more steps will be required to complete therearrangement?(a) Five (b) Four (c) Six(d) Three (e) None of these

4. Step II of an input isbox 93 25 year end 41 32 valueWhich of the following is definitely the input?(a) 25 year end box 93 41 32 value(b) 25 year end 93 41 32 value box(c) 9325 box year end 41 32 value(d) Cannot be determined(e) None of these

RESPONSE

GRID

1. a b c d e 2. a b c d e 3. a b c d e 4. a b c d e 5. a b c d e

6. a b c d e 7. a b c d e 8. a b c d e 9. a b c d e

Max. Marks : 23 No. of Qs. 23 Time : 18 min. Date : ........./......../................5. Input: paper dry 37 23 height call 62 51

Which of the following steps will be the last but one?(a) V (b) IV (c) VI(d) III (e) None of these

DIRECTIONS (Qs.6-10) : A word and number arrangementmachine when given an input line of words and numbers rearrangesthem following a particular rule in each step. The following is anillustration of an input and rearrangement.Input : 17 put show on 39 27 85 goldStep I : show 17 put on 39 27 85 goldStep II : show 85 17 put on 39 27 goldStep III : show 85 put 17 on 39 27 goldStep IV : show 85 put 39 17 on 27 goldStep V : show 85 put 39 on 17 27 goldStep VI : show 85 put 39 on 27 17 goldStep VII : show 85 put 39 on 27 gold 17and step VII is the last step of the rearrangement of the aboveinput.As per the rules followed in the above steps, find out in each ofthe following questions the appropriate step for the given input.6. Input : glass full 15 37 water now 85 67

Which of the following will be step VI of the above input?(a) 85 now 67 full glass 15 37(b) water 85 now 67 glass full 15 37(c) water 85 now 67 glass 37 full 15(d) There will be no such step.(e) None of these

7. Step II of an input is: ultra 73 12 16 mail sort 39 kite. Which ofthe following steps will be the last but one?(a) VIII (b) IX (c) VII(d) VI (e) None of these

8. Step III of an input is: win 75 voice 15 39 store gap 26. Whichof the following is definitely the input?(a) voice 15 win 75 39 store gap 26(b) voice win 75 15 39 store gap 26(c) 15 75 win voice store gap 26(d) Cannot be determined(e) None of these

9. Step II of an input is: tube 83 49 34 garden flower rat 56. Howmany steps will be required to complete the rearrangement?(a) Four (b) Five (c) Six(d) Three (e) None of these

171717Input-Output

Page 42: yoursmahboob.wordpress.com SBI · yoursmahboob.wordpress.com iii P 101 Speed Tests for SBI Bank Clerk Exam 101 Speed Tests for SBI Bank Clerk Exam is revised and updated edition on

yoursmahboob.w

ordpress.com

SPEED TEST 1734

RESPONSE

GRID

10. a b c d e 11. a b c d e 12. a b c d e 13. a b c d e 14. a b c d e

15. a b c d e 16. a b c d e 17. a b c d e 18. a b c d e 19. a b c d e

20. a b c d e 21. a b c d e 22. a b c d e 23. a b c d e

10. Input : hunt for 94 37 good 29 48 book.How many steps will be required to complete therearrangement?(a) Four (b) Five (c) Six(d) Seven (e) None of these

DIRECTIONS (Q.11 to 15) : Study the following information toanswer the given questions:A word and number arrangement machine when given an inputline of words and numbers rearranges them following a particularrule. The following is an illustration of input and rearrangement.(All the numbers are two-digit numbers.)Input : sine 88 71 cos theta 14 56 gamma delta 26Step I : cos sine 71 theta 14 56 gamma delta 26 88Step II : delta cos sine theta 14 56 gamma 26 88 71Step III : gamma delta cos sine theta 14 26 88 71 56Step IV : sine gamma delta cos theta 14 88 71 56 26Step V : theta sine gamma delta cos 88 71 56 26 14Step V is the last step of the rearrangement.As per the rules followed in the above steps, find out in each ofthe following questions the appropriate steps for the given input.Input for the questions:Input : for 52 all 96 25 jam road 15 hut 73 bus stop 38 46 (All thenumbers given in the arrangement are two digit numbers).11. Which word/ number would be at 8th position from the

right in step IV ?(a) 15 (b) road (c) hut(d) jam (e) stop

12. Which step number would be the following output? Bus allfor 52 25 jam road 15 hut stop 38 46 96 73.(a) There will be no such step.(b) III (c) II (d) V(e) VI

13. Which of the following would be step VII?(a) stop road jam hut for bus all 15 96 73 5246 38 25(b) road jam hut for bus all stop 15 25 38 46 52 73 96(c) stop road jam hut for bus all 96 73 52 46 38 25 15(d) jam hut for bus all 25 road stop 15 96 73 52 46 38(e) There will be no such step

14. Which word/number would be at 6th position from the leftin step V?(a) 25 (b) stop (c) jam(d) all (e) road

15. Which of the following would be step III?(a) hut for bus all 25 jam road 15 stop 38 96 73 52 46(b) for bus all 25 jam road 15 hut 38 stop 96 46 73 52(c) hut for bus all jam road 15 stop 38 96 73 52 46 25(d) for bus all 25 jam road 15 hut stop 38 46 96 73 52(e) None of these

DIRECTIONS (Q. 16 to 23) : Study the following informationcarefully and answer the given questions:A word and number arrangement machine when given an input lineof words and numbers rearranges them following a particular rule ineach step. The following is an illustration of input and rearrangement.

Input : 51 pour 32 start now 23 46 houseStep I : 23 51 pour 32 start now 46 houseStep II : 23 start 51 pour 32 now 46 houseStep III : 23 start 32 51 pour now 46 houseStep IV : 23 start 32 pour 51 now 46 houseStep V : 23 start 32 pour 46 51 now houseStep VI : 23 start 32 pour 46 now 51 houseand step VI is the last step of the rearrangementAs per the rules followed in the above steps, find out in each ofthe following questions the appropriate steps for the given input.16. Step II of an input is : 18 task bear cold dish 81 63 31

How many more steps will be required to complete therearrangement?(a) Three (b) Four (c) Five(d) Six (e) None of these

17. Input : 72 59 37 go for picnic 24 journeyHow many steps will it take to complete the rearrangement?(a) Three (b) Four (c) Five(d) Six (e) None of these

18. Input : nice flower 24 12 costly height 41 56Which of the following will be step III ?(a) 12 nice 34 height flower costly 41 56(b) 12 nice 34 height 41 flower costly 56(c) 12 nice 34 flower costly height 41 56(d) 12 nice flower 34 costly height 41 56(e) None of these

19. Step II of an input is : 16 victory 19 36 53 store lake town.Which of the following will be step V ?(a) 16 victory 19 town store 36 53 lake(b) 16 victory 19 town 36 store 53 lake(c) 16 victory 19 town 36 53 store lake(d) There will be no such step(e) None of these

20. Step III of an input is : 15 yes 29 ask for soap 42 37Which of the following is definitely the input?(a) ask yes 29 15 for soap 42 37(b) yes ask 15 29 for soap 42 37(c) 29 15 yes ask for soap 42 37(d) Cannot be determined(e) None of these

21. Input : milk pot 18 24 over goal 36 53Which of the following steps will be the last but one?(a) VI (b) V (c) VII(d) VIII (e) None of these

22. Step III of an input is : 36 win 44 95 86 ultra box queenHow many more steps will be required to complete therearrangement?(a) Three (b) Four (c) Five(d) Six (e) None of these

23. Input : new 22 model 27 pump 38 11 joinHow many steps will be required to complete therearrangement?(a) Four (b) Five (c) Six(d) Seven (e) None of these

Page 43: yoursmahboob.wordpress.com SBI · yoursmahboob.wordpress.com iii P 101 Speed Tests for SBI Bank Clerk Exam 101 Speed Tests for SBI Bank Clerk Exam is revised and updated edition on

yoursmahboob.w

ordpress.com

1. If ‘+’ means ‘minus’ ‘–’ means ‘multiplied by’ ‘¸’ means ‘plus’and ‘×’ means ‘divided by’, then10 × 5 3 – 2 + 3 = ?

(a) 5 (b) 21 (c)53

3(d) 18 (e) None of these

2. In the following question you have to identify the correctresponse from the given premises stated according to thefollowing symbols.If ‘+’ means ‘¸’ , ‘–’ means ‘×’, ‘¸’ means ‘+’ and ‘×’ means‘–’, then 63 × 24 + 8 4 + 2 – 3 = ?(a) 54 (b) 66 (c) 186(d) 48 (e) None of these

3. Which one of the following is correct?6 * 4 * 9 * 15(a) ×, = , – (b) ×, –, = (c) =, ×, –(d) –, ×, = (e) None of these

4. If > = , Ú = ×, < = +, Ù = –, + = <, × = =, – = >(a) 6 > 2 > 3 Ù 8 Ú 4 + 13(b) 6 Ù 2 < 3 > 8 < 4 – 13(c) 6 Ú 2 < 3 Ù 8 > 4 × 13 (d) 6 > 2 Ú 3 < 8 Ù 4 + 13(e) None of these

5. Find out the correct answer for the unsolved equation onthe basis of the given equations.If 6 * 5 = 91, 8 * 7 = 169, 10 * 7 = 211, then11 * 10 = ?(a) 331 (b) 993 (c) 678(d) 845 (e) None of these

6. If ‘–’ stands for division, ‘+’ for multiplication ‘¸’forsubtraction and ‘×’ for addition. Which one of the followingequation is correct?(a) 6 ¸ 20 × 12 + 7 – 1 = 70(b) 6 + 20 – 20 7 × 1 = 62(c) 6 – 20 12 × 7 + 1 = 57(d) 6 + 20 – 20 7 – 1 = 38(e) None of these

7. In an imaginary mathematical operation ‘+’ meansmultiplication, ‘×’ means subtraction, ‘¸’means addition and‘–’ means division. All other rules in mathematical operationare the same as in the existing system.Which one of the following gives the result of175 – 25 5 + 20 × 3 + 20 = ?(a) 160 (b) 2370 (c) 7 7(d) 240 (e) None of these

RESPONSE

GRID

1. a b c d e 2. a b c d e 3. a b c d e 4. a b c d e 5. a b c d e

6. a b c d e 7. a b c d e 8. a b c d e 9. a b c d e 10. a b c d e

11. a b c d e 12. a b c d e 13. a b c d e 14. a b c d e 15. a b c d e

Max. Marks : 30 No. of Qs. 30 Time : 20 min. Date : ........./......../................

8. If L stands for +, M stands for –, N stands for ×, P stands for¸, then 14 N 10 L 42 P 2 M 8 = ?(a) 153 (b) 216 (c) 248(d) 251 (e) None of these

9. It being given that: > denotes +, < denotes –, + denotes , –denotes =, = denotes ‘less than’ and × denotes ‘greater than’.Find which of the following is a correct statement.(a) 3 + 2 > 4 = 9 + 3 < 2(b) 3 > 2 > 4 = 18 + 3 < 1(c) 3 > 2 < 4 × 8 + 4 < 2(d) 3 + 2 < 4 × 9 + 3 < 3(e) None of these

10. If P denotes +, Q denotes –, R denotes × and S denotes ¸which of the following statements is correct?(a) 36 R 4 S 8 Q 7 P 4 = 10(b) 16 R 12 P 49 S 7 Q 9 = 200(c) 32 S 8 R 9 = 160 Q 12 R 12(d) 8 R 8 P 8 S 8 Q 8 = 57(e) None of these

11. If ‘×’ stands for ‘addition’, ‘<’ for substraction, + for division,> for multiplication, – for ‘equal to’, ¸ for ‘greater than’ and‘=’ for ‘less than’, then state which of the following is true?(a) 3 × 4 > 2 – 9 + 3 < 3(b) 5 × 3 < 7 8 + 4 × 1(c) 5 > 2 + 2 = 10 < 4 × 8(d) 3 × 2 < 4 16 > 2 + 4(e) None of these

DIRECTIONS (Qs. 12 - 16): In an imaginary language, the digits0, 1, 2, 3, 4, 5, 6, 7, 8 and 9 are substituted by a, b, c, d, e, f, g,h, i and j. And 10 is written as ba.

12. (cd + ef) × bc is equal to(a) 684 (b) 816 (c) 916(d) 1564 (e) None of these

13. dc × f – (bf – d) × d is equal to(a) abb (b) abe (c) bce(d) bcf (e) None of these

14. baf + fg – (ca × h/be) is equal to(a) 141 (b) 145 (c) 151(d) 161 (e) None of these

15. baf ¸ bf × d is equal to(a) df (b) cb (c) be(d) d (e) None of these

181818MathematicalOperations

Page 44: yoursmahboob.wordpress.com SBI · yoursmahboob.wordpress.com iii P 101 Speed Tests for SBI Bank Clerk Exam 101 Speed Tests for SBI Bank Clerk Exam is revised and updated edition on

yoursmahboob.w

ordpress.com

SPEED TEST 1836

RESPONSE

GRID

16. a b c d e 17. a b c d e 18. a b c d e 19. a b c d e 20. a b c d e

21. a b c d e 22. a b c d e 23. a b c d e 24. a b c d e 25. a b c d e

26. a b c d e 27. a b c d e 28. a b c d e 29. a b c d e 30. a b c d e

16. If ‘+’ means ‘×’, ‘×’ means ‘–’, ‘¸’ means ‘+’ and ‘–’ means‘¸’, then which of the following gives the result of175 – 25 5 + 20 × 3 ¸ 10 = ?(a) 77 (b) 160 (c) 240(d) 2370 (e) None of these

17. If + means ÷ ,– means ×, ¸ means + and × means –, then 36× 8 + 4 ÷ 6 + 2 –3 = ?(a) 2 (b) 18 (c) 43

(d)1

62

(e) None of these

18. If the given interchanges namely : signs + and ÷ andnumbers 2 and 4 are made in signs and numbers, which oneof the following four equations would be correct ?(a) 2 + 4 ÷ 3 = 3 (b) 4 + 2 ÷ 6 = 1.5(c) 4 ÷ 2 + 3 = 4 (d) 2 + 4 ÷ 6 = 8.(e) None of these

19. If L denotes x, M denotes ÷ , P denotes + and Q denotes–, than 8 P 36 M 6 Q 6 M 2 L 3 = ?

(a)136

(b) –1

6(c)

114

2(d) 5 (e) None of these

20. If X stands for’ addition’, < for ‘substraction’, + stands for‘division’, > for ‘multiplication’, –, stands for ‘equal to’,¸ for ‘greater than’ and = stands for ‘less than’, state whichof the following is true ?(a) 3 × 2 < 4 ¸ 16 > 2 + 4(b) 5 > 2 + 2 = 10 < 4 × 2(c) 3 × 4 > 2 – 9 + 3 < 3(d) 5 × 3 < 7 ¸ 8 + 4 × 1(e) None of these

21. If ‘20 – 10’ means 200, ‘8 ÷ 4’ means 12, ‘6 × 2’ means 4 and‘12 + 3’ means 4, then100 – 10 × 1000 ÷ 1000 + 100 × 10 = ?(a) 1090 (b) 0 (c) 1900(d) 20 (e) None of these

22. If × means +, ÷ means – , – means × and + means ÷, then 8 ×7 – 8 + 40 ÷ 2 = ?

(a) 1 (b)2

75

(c) 38

5(d) 44 (e) None of these

23. If ‘+’ means ‘×’ ; ‘–’ means ‘÷’ ; ‘ ×’ means ‘–’ and ‘÷’ means‘+’ then 9 + 8 ÷ 8 – 4 × 9 = ?(a) 26 (b) 17 (c) 65(d) 11 (e) None of these

24. If ‘÷’ means ‘+’ ; ‘–’ means ‘×’ ; ‘+’ means ‘÷’ and ‘×’ means‘–’ then 20 ÷ 12 × 4 + 8 – 6 = ?

(a)2

83

(b) 29 (c) 32

(d) 26 (e) None of these25. If ‘–’ means ‘×’ ; ‘×’ means ‘+’ ; ‘+’ means ‘÷’ and ‘÷’ means

‘–’ then 40 × 12 + 3 – 6 ÷ 60= ?(a) 44 (b) 7.95 (c) 16(d) 28 (e) None of these

26. If ‘ + ’ means ‘divided by’ ‘–’ means ‘added to ’ ‘x’ means‘subtracted from’ and ÷ means ‘multiplied by’ then what isthe value of 24 ¸ 12 – 18 + 9 ?(a) – 25 (b) 0.72 (c) 15.30(d) 290 (e) None of these

27. If ® stands for ‘addition’ ¬ stands for ‘subtraction’ ­stands for ‘division ;¯ stands for’ multiplication' Z stands

for equal to' then which of the following alternatives iscorrect?

(a) 7 43 6 1 Z 4

(b) 3 6 2 3 6 Z 5

(c) 5 7 3 2 Z 5

(d) 2 5 6 2 Z 6

(e) None of these28. Of ‘x’ Stands for ‘ addition’ ‘z’ for subtraction’ ‘+’ for division'

> for multiplication’ ‘–’ for equal to’ ‘+’ for ‘ greater than’and '=' for ' less than' state which of the following is true. ?(a) 3 x 4 > 2 – 9 + 3 < 3(b) 5 x 3 < 7 ¸ 8 + 4 x 1(c) 5 > 2 + 2 = 10 < 4 x 8(d) 3 x 2 < 4 ¸ 16 > 2 + 4(e) None of these

29. If ¸ means + – means ÷ x means – and + means x then

( )36 x 4 8 x 4

4 8 x 2 16 1

-+ + + = ?

(a) 0 (b) 8 (c) 12(d) 16 (e) None of these

30. If x means +, – means x , ÷ means + and + means – then(3 – 15 ÷ 19) x 8 + 6 = ?(a) – 1 (b) 2 (c) 4(d) 8 (e) None of these

Page 45: yoursmahboob.wordpress.com SBI · yoursmahboob.wordpress.com iii P 101 Speed Tests for SBI Bank Clerk Exam 101 Speed Tests for SBI Bank Clerk Exam is revised and updated edition on

yoursmahboob.w

ordpress.com

1. The number of times in a day the Hour-hand and the Minute-hand of a clock are at right angles, is(a) 44 (b) 48 (c) 24(d) 12 (e) None of these

2. An accurate clock shows the time as 3.00. After hour handhas moved 135°, the time would be(a) 7.30 (b) 6. 30 (c) 8.00(d) 9.30 (e) None of these

3. An accurate clock shows 8 O' clock in the morning.Throughout how many degrees will the hour hand rotate,when the clock shows 2 O' clock in the afternoon?(a) 150° (b) 144° (c) 168°(d) 180° (e) None of these

4. March 1, 2008 was Saturday. Which day was it on March 1,2002?(a) Thursday (b) Friday (c) Saturday(d) Sunday (e) None of these

5. How many times are an hour hand and a minute hand of aclock at right angles during their motion from 1.00 p.m. to10.00 p.m.?(a) 9 (b) 10 (c) 18(d) 20 (e) None of these

6. At what approximate time between 4 and 5 am will thehands of a clock be at right angle?(a) 4 : 40 am (b) 4 : 38 am (c) 4 : 35 am(d) 4 : 39 am (e) None of these

7. At what time between 3 and 4 o’clock, the hands of a clockcoincide?

(a)4

16 minutes past 311

(b)5

15 minutes past 361

(c)5

15 minutes to 260

(d)4

16 minutes to 411

(e) None of these

RESPONSE

GRID

1. a b c d e 2. a b c d e 3. a b c d e 4. a b c d e 5. a b c d e

6. a b c d e 7. a b c d e 8. a b c d e 9. a b c d e 10. a b c d e

11. a b c d e 12. a b c d e 13. a b c d e 14. a b c d e 15. a b c d e

Max. Marks : 30 No. of Qs. 30 Time : 20 min. Date : ........./......../................

8. At what time between 5.30 and 6 will the hands of a clock beat right angles?

(a)5

43 min. past 511

(b)7

43 min. past 511

(c) 40 min. past 5 (d) 45 min. past 5(e) None of these

9. At what time between 4 and 5 o’clock will the hands of awatch point in opposite directions?(a) 45 min. past 4 (b) 40 min. past 4

(c)4

50 min. past 411

(d)6

54 min. past 411

(e) None of these10. How much does a watch lose per day, if its hands coincide

every 64 minutes?

(a)8

32 min.11

(b)5

36 min.11

(c) 90 min.

(d) 96 min. (e) None of these11. The last day of a century cannot be

(a) Monday (b) Wednesday (c) Tuesday(d) Friday (e) None of these

12. Which of the following is not a leap year?(a) 700 (b) 800 (c) 1200(d) 2000 (e) None of these

13. How many days are there in x weeks x days?(a) 7 x2 (b) 8 x (c) 14 x(d) 7 (e) None of these

14. It was Sunday on Jan 1, 2006. What was the day of the weekon Jan 1, 2010?(a) Sunday (b) Saturday (c) Friday(d) Wednesday (e) None of these

15. On 8th Feb, 2005 it was Tuesday. What was the day of theweek on 8th Feb, 2004?(a) Tuesday (b) Monday (c) Sunday(d) Wednesday (e) None of these

191919Clock and Calendar

Page 46: yoursmahboob.wordpress.com SBI · yoursmahboob.wordpress.com iii P 101 Speed Tests for SBI Bank Clerk Exam 101 Speed Tests for SBI Bank Clerk Exam is revised and updated edition on

yoursmahboob.w

ordpress.com

SPEED TEST 1938

RESPONSE

GRID

16. a b c d e 17. a b c d e 18. a b c d e 19. a b c d e 20. a b c d e

21. a b c d e 22. a b c d e 23. a b c d e 24. a b c d e 25. a b c d e

26. a b c d e 27. a b c d e 28. a b c d e 29. a b c d e 30. a b c d e

16. The calendar for the year 2007 will be the same for the year.

(a) 2014 (b) 2016 (c) 2017(d) 2018 (e) None of these

17. Today is Monday. After 61 days, it will be

(a) Wednesday (b) Saturday (c) Tuesday(d) Thursday (e) None of these

18. What was the day of the week on 17th June, 1998?

(a) Monday (b) Tuesday

(c) Wednesday (d) Thursday

(e) None of these

19 If 21st July, 1999 is a wednesday, what would have beenthe day of the week on 21st July, 1947 ?

(a) Monday (b) Sunday (c) Thursday(d) Saturday (e) None of these

20. At an enquiry office at a railway station, a passenger wastold that a train for New Delhi has left 15 minutes ago, butafter every 45 minutes a train leaves for New Delhi. Thenext train will leave at 8.30 p.m. At what time was thisinformation given to the passanger ?

(a) 7.45 pm (b) 8.00 pm (c) 8.15 pm(d) 8.05 pm (e) None of these

21. A watch is a minute slow at 1 p.m. on Tuesday and 2 minutesfast at 1 p.m. on Thursday. When did it show the correcttime ?

(a) 1:00 a.m. on Wednesday

(b) 5:00 a.m. on Wednesday

(c) 1:00 p.m. on Wednesday

(d) 5:00 p.m. on Wednesday

(e) None of these

22. An application was received by inward clerk in the afternoonof a week day. Next day he forwarded it to the table of thesenior clerk, Who was on leave that day. The senior clerkput up the application to the desk officer next day in theevening. The desk officer studied the application anddisposed off the matter on the same day i.e., Friday. Whichday was the application received by the inward clerk ?

(a) Monday

(b) Wednesday

(c) Tuesday

(d) Previous week’s Saturday

(e) None of these

23. Ashish leaves his house at 20 minutes to seven in themorning reaches Kunal’s house in 25 minutes. They finishtheir breakfast in another 15 minutes and leave for theiroffice which takes another 35 minutes. At what time dothey leave Kunal’s house to reach their office?

(a) 7.40 a.m. (b) 7.20 a.m. (c) 7.45 a.m.(d) 8.15 a.m. (e) None of these

24. Reaching the place of meeting on Tuesday 15 minutesbefore 8.30 hours, Anuj found himself half an hour earlierthan the man who was 40 minutes late. What was thescheduled time of the meeting?

(a) 8.00 hrs (b) 8.05 hrs (c) 8.15 hrs(d) 8.45 hrs (e) None of these

25. A clock gaining 2 min every hour was synchronised atmidnight with a clock losing 1 min every hour. How manyminutes behind will its minute hand be at eleven thefollowing morning ?

(a) 23 (b) 27 (c) 22(d) None of these (e) None of these

26. Rama remembers that she met her brother on Saturday,which was after the 20th day of a particular month. If the1st day of that month was Tuesday, then on which date didRama meet her brother ?(a) 24th (b) 23rd (c) 25th(d) None of these

27. In 1

22

hours the hour hand of a clock rotates through an

angle of(a) 90° (b) 140° (c) 120°(d) 75° (e) None of these

28. On 27 March, 1995 was a Monday. Then what days of theweek was 1 November, 1994?(a) Monday (b) Sunday(c) Tuesday (d) Wednesday

29. 16 January 1997 was a Thursday. What day of the weekwas 4 January 2000?(a) Tuesday (b) Wednesday (c) Thursday(d) Friday (e) None of these

30. In a year 28th February is Tuesday; if the leap year isexcluded, then 28th March will be a(a) Sunday (b) Tuesday (c) Monday(d) Saturday (e) None of these

Page 47: yoursmahboob.wordpress.com SBI · yoursmahboob.wordpress.com iii P 101 Speed Tests for SBI Bank Clerk Exam 101 Speed Tests for SBI Bank Clerk Exam is revised and updated edition on

yoursmahboob.w

ordpress.com

DIRECTIONS (Qs. 1 - 20) : Each question below is followed bytwo statements I and II. You are to determine whether the datagiven in the statement is sufficient to answer the question. Youshould use the data and your knowledge of Mathematics to choosebetween the possible answers.Give answer (a) if the statement I alone is sufficient to answer the

question, but the statement II alone is notsufficient.

Give answer (b) if the statement II alone is sufficient to answerthe question, but the statement I alone is notsufficient.

Give answer (c) if both statements I and II together are needed toanswer the question.

Give answer (d) if either the statement I alone or the statement IIalone is sufficient to answer the question.

Give answer (e) if you cannot get the answer from the statementsI and II together, but need even more data.

1. What is the age of C, in a group of A, B, C, D and E, whoseaverage age is 45 years?I. Average of the ages of A and B is 53 years.II. Average of the ages of D and E is 47 years

2. Tower ‘P’ is in which direction with respect to tower ‘Q’?I. P is to the West of H, which is to the South of Q.II. F is to the West of Q and to the North of P.

3. How is K related to N?I. N is the brother of M, who is the daughter of K.II. F is the husband of K

4. What is Nidhi’s age?I. Nidhi is 3 times younger than Rani.II. Surekha is twice the age of Rani and the sum of theirages is 72.

5. What is Seema’s age?I. Seema’s age is half of Reema ageII. Reema is 5 years younger than her sister.

6. What is Deepali’s age?I. Deepali is two times younger than Nisha.II. Supriya is twice the age of Nisha.

RESPONSE

GRID

1. a b c d e 2. a b c d e 3. a b c d e 4. a b c d e 5. a b c d e

6. a b c d e 7. a b c d e 8. a b c d e 9. a b c d e 10. a b c d e

11. a b c d e 12. a b c d e 13. a b c d e 14. a b c d e 15. a b c d e

Max. Marks : 25 No. of Qs. 25 Time : 20 min. Date : ........./......../................7. In a row of girls facing North, what is D’s position from the

left end?I. D is twentieth from the right end.II. There are ten girls between B and D.

8. Town M is towards which direction of Town K?I. Town K is towards North-West of Town D.II. Town M is towards South - East of Town D.

9. How many daughters does P have?I. K and M are sisters of T.II. T’s father is husband of P’s mother.

10. Towards which direction is Village M from Village T?I. Village P is to the south of Village M and Village P is to

the west of Village T.II. Village K is to the east of Village M and Village K is to

the north of Village T.11. How is D related to M?

I. K and D are the only sisters of R.II. M is married to R’s father.

12. What is R’s position from the left end in a row of childrenfacing South?I. There are forty children in the row.II. D is tenth to the left of R and fifteenth from the rightend of the row.

13. Towards which direction was D facing when he started hisjourney?I. D walked 20 metres after he started, took a right turn

and walked 30 metres and again took a right turn andfaced West.

II. D walked 20 metres after he started, took a left turn andwalked 30 metres and again took a left turn and facedWest.

14. How many daughters does A have?I. A has four children.II. B and C are sisters of D who is son of A.

15. How far is A from the starting point?I. A moves 5 km. towards East, then 2 km. towards left,

10 km, towards right and finally. 2 km, towards rightand stops

II. A moves 2 km. towards East, then 2 km. towards right,13 km, towards left and finally, 2 km. towards left andstops.

202020Data Sufficiency

Page 48: yoursmahboob.wordpress.com SBI · yoursmahboob.wordpress.com iii P 101 Speed Tests for SBI Bank Clerk Exam 101 Speed Tests for SBI Bank Clerk Exam is revised and updated edition on

yoursmahboob.w

ordpress.com

SPEED TEST 2040

RESPONSE

GRID

16. a b c d e 17. a b c d e 18. a b c d e 19. a b c d e 20. a b c d e

21. a b c d e 22. a b c d e 23. a b c d e 24. a b c d e 25. a b c d e

16. In a row of 40 students facing North, how many students arethere between R and S?I. S’s position in the row is 15th from the right end.II. R’s position in the row is 4th from the left end.

17. How many children does Suneeta have?I. X is the only daughter of Suneeta.II. Y is brother of X.

18. Pole X is in which direction with respect to pole Y?I. Pole H is to the north-east of pole X and to the north of

pole Y.II. Pole R is to the east of pole X and to the north of pole

Y.19. How many children does Seema have?

I. Seema, the mother of Varsha’s sister has only one son.II. Varsha has only three siblings.

20. How is Anil related to Sanjay?I. Sanjay’s son is the brother of only sister of Anil.II. Radhika, the only daughter of Sanjay has only two

brothers.

DIRECTIONS (Qs. 21-25) : Each of the questions below consistsof a question and two statements numbered I and II given belowit. You have to decide whether the data provided in the statementsare sufficient to answer the question. Read both the statementsand Give answer

(a) if the data in statement I alone are sufficient to answer thequestion, while the data in statement II alone are not sufficientto answer the question.

(b) if the data in statement II alone are sufficient to answer thequestion, while the data in statement I alone are not sufficientto answer the question.

(c) If the data either in statement I alone or in statement II aloneare sufficient to answer the question.

(d) if the data given in both the statements I and II together arenot sufficient to answer the question.

(e) if the data given in both the statements I and II together arenecessary to answer the question.

21. In a row of girls facing North, what is D’s position from theleft end?I. D is twentieth from the right end.II. There are ten girls between Band D.

22. Town M is towards which direction of Town K?I. Town K is towards North-West of Town DII. Town M is towards South-East of Town D

23. How many daughters does P have?I. K and M are sisters of T.II. T’s father is husband of P’s mother.

24. On which day of the week from Monday to Sunday did Arunleave for London?I. Arun did not leave for London during the weekend.II. Arun’s brother left for London on Friday two days afterArun left for London.

25. How is ‘new’ written in a code language?I. ‘new good clothes’ is written as ‘5 3 9’ in that code

language.II. ‘good clothes are costly’ is written as ‘9673’ in that

code language.

Page 49: yoursmahboob.wordpress.com SBI · yoursmahboob.wordpress.com iii P 101 Speed Tests for SBI Bank Clerk Exam 101 Speed Tests for SBI Bank Clerk Exam is revised and updated edition on

yoursmahboob.w

ordpress.com

DIRECTIONS (Qs. 1-4): In these questions, relationships betweendifferent elements is shown in the statements. These statementsare followed by two conclusions.

Give answer (a) if only conclusion I follows.Give answer (b) if only conclusion II follows.Give answer (c) if either conclusion I or conclusion II follows.Give answer (d) if neither conclusion I nor conclusion IIfollows.Give answer (e) if both conclusions I and II follow.

1. Statement :

A < L < T < R £ H > KConclusions :I. H > LII. K > T

2. Statement :P = N > D ³ G < B = JConclusions :I. G < PII. G < J

3. Statement :F £ C ³ V = Z < X = UConclusions :I. V < UII. Z < F

4. Statement :Q £ E = I > N ³ R ³ SConclusions :I. E = SII. S £ N

DIRECTIONS (Qs. 5-9): In the following questions, the symbolsd , %, $, # and @ are used with the following meaning as illustratedbelow:

‘P $ Q’ means ‘P is not smaller than Q’.‘P @ Q’ means ‘P is not greater than Q’.‘P d Q’ means ‘P is neither smaller than nor equal to Q’.‘P # Q’ means ‘P is neither greater than nor equal to Q’.‘P % Q’ means ‘P is neither smaller than nor greater than Q’.Now in each of the following questions assuming the given

statements to be true, find which of the two conclusions I and IIgiven below them is/are definitely true?

RESPONSE

GRID

1. a b c d e 2. a b c d e 3. a b c d e 4. a b c d e 5. a b c d e

6. a b c d e 7. a b c d e 8. a b c d e 9. a b c d e 10. a b c d e

Max. Marks : 22 No. of Qs. 22 Time : 20 min. Date : ........./......../................

Give answer(a) if only Conclusion I is true.(b) if only Conclusion II is true.(c) if either Conclusion I or II is true.(d) if neither Conclusion I nor II is true.(e) if both Conclusions I and II are true.

5. Statements: F @ N, N d R, H @ RConclusions: I. H d N

II. F # R6. Statements: M # T, T@ K, K $ N

Conclusions: I. M # NII. K d M

7. Statements: T % H, H $ WConclusions: I. W # T

II. W % T8. Statements: N d K, K # D, D % M

Conclusions: I. M d KII. D d N

9. Statements: J $ B, B % R, R d FConclusions: I. F # B

II. R @ J

DIRECTIONS (Qs. 10-14): In these questions symbols ©, #, *, $and @ are used with different meanings as follows:

'A ©B' means ‘A is smaller than B’.A # B means ‘A is either smaller, than or equal to B’.`A * B' means ‘A is greater than B’.A $ B means ‘A is either greater than or equal to B’.A @ B means ‘A is neither smaller than nor greater than B’.In each of the following questions assuming the givenstatements to be true, find out which of the two conclusionsI and II given below them is/are definitely true.Give answer (a) if only conclusion I is true.Give answer (b) if onlyconclusion II is true.Give answer (c) if either conclusion I or conclusion II is true.Give answer (d) if neither conclusion I nor conclusion II istrue.Give answer (e) if both conclusions I and II are true.

10. Statements:V # S, S © L, L © JConclusions: I. V © L

II. S © J

2121Statement & Conclusion(Mathematical)

Page 50: yoursmahboob.wordpress.com SBI · yoursmahboob.wordpress.com iii P 101 Speed Tests for SBI Bank Clerk Exam 101 Speed Tests for SBI Bank Clerk Exam is revised and updated edition on

yoursmahboob.w

ordpress.com

SPEED TEST 2142

RESPONSE

GRID

11. a b c d e 12. a b c d e 13. a b c d e 14. a b c d e 15. a b c d e

16. a b c d e 17. a b c d e 18. a b c d e 19. a b c d e 20. a b c d e

21. a b c d e 22. a b c d e

11. Statements: M # R, R © J, J # HConclusions: I. M # H

II. R © H12. Statements:H $ F, F @ G, G « M

Conclusions: I. H « M II. H « G

13. Statements:R © J, J « T, T # LConclusions: I. R @ T

II. J @ L14. Statements: W @ T, T $ K, K « F

Conclusions: I. W $ K II. W @ K

DIRECTIONS (Qs.15-19) : In the following questions, the symbols@, #, %, $ and c are used with the following meaning as illustratedbelow:

'P # Q' means 'P is neither greater than nor equal to Q'.'P c Q' means 'P is neither equal to nor smaller than Q''P % Q' means 'P is neither smaller than nor greater than Q.'P $ Q' means 'P is not smaller than Q''P @ Q' means 'P is not greater than Q'.

Now in each of the following questions, assuming the givenstatements to be true, find which of the three conclusions I, II andIII given below them is/are definitely true and give your answeraccordingly.15. Statements: R @ D, D c W, B $ WW

Conclusions:I. W # R II. B c D III. W $ R(a) None is true (b) Only I is true(c) Only III is true (d) Only either I or III is true(e) All are true

16. Statements:H $ V, V % M, K c MConclusions: I. K c V

II. M @ HIII. H c K

(a) Only I and III are true (b) Only II and III are true(c) Only I and II are true (d) All are true(e) None of these

17. Statements:K # T, T $ B, B @ FConclusions: I. F $ T

II. K # BIII. T $ F

(a) None is true (b) Only I is true(c) Only I and II are true (d) Only II and III are true(e) All are true

18. Statements:Z # F, R @ F, D c RConclusions: I. Z # R

II. F # DIII. D @ Z

(a) None is true (b) Only I is true(c) Only III is true (d) Only either I or III is true(e) All are true

19. Statements:M c R, R % D, D @ NConclusions: I. M c N

II. N $ RIII. M c D

(a) Only I and II are true (b) Only II and III are true(c) Only I and III are true (d) All are true(e) None of these

DIRECTIONS (Qs.20-22) : In these questions, relationshipbetween different elements is shown in the statements. Thesestatements are followed by two conclusions.Mark answers if(a) Only conclusion I follows.(b) Only conclusion II follows.(c) Either conclusion I or II follows.(d) Neither conclusion I nor II follows.(e) Both conclusion I and II follows.20. Statement: P ³ Q = R > S > T

Conclusions: I. P ³ TII. T < Q

21. Statement: L £ M < N > O ³ PConclusions: I. O < M

II. P £ N22. Statement: A > B, B ³ C = D < E

Conclusions: I. C < AII. D £ B

Page 51: yoursmahboob.wordpress.com SBI · yoursmahboob.wordpress.com iii P 101 Speed Tests for SBI Bank Clerk Exam 101 Speed Tests for SBI Bank Clerk Exam is revised and updated edition on

yoursmahboob.w

ordpress.com

DIRECTIONS (Q. 1 to 18) : In each question below is given astatement followed by two conclusions numbered I and II. Youhave to assume everything in the statement to be true, thenconsider the two conclusions together and decide which of themlogically follows beyond a reasonable doubt from the informationgiven in the statement.

Give answer (a) if only conclusion I follows.Give answer (b) if only conclusion II follows.Give answer (c) if either I or II follows.Give answer (d) if neither I nor II follows.Give answer (e) if both I and II follows.1. Statement : Although we have rating agencies like Crisil.

ICRA, there is demand to have a separate rating agency forIT Companies to protect investors.Conclusions :I : Assessment of financial worth of IT Companies calls

for separate set of skills, insight and competencies.II : Now the investors investing in I. T. Companies will get

protection of their investment.2. Statement : Company "Y" will improve the manufacturing

facilities for the production of shaving kits as a result ofwhich capacity would increase and cost would be reduced –A spokesperson of the Company "Y".Conclusions :I : The products of Company "Y" will complete the market

norms in the quality and cost factor.II : There will be demand of shaving kits of Company "Y"?

3. Statement : During 1997-98 the total loss incurred by the111 Public Sector Units was to the tune of ` 6809 crore,which was converted into paid capitals by the Governmentof its total investment of 5129 crore.Conclusions :I : The Government is left with only one option that is to

privatise these units.II : The Government did not take care in the matter of

investments in these public sector units.

RESPONSE

GRID

1. a b c d e 2. a b c d e 3. a b c d e 4. a b c d e 5. a b c d e

6. a b c d e 7. a b c d e 8. a b c d e

Max. Marks : 20 No. of Qs. 20 Time : 15 min. Date : ........./......../................

4. Statement : Population increase coupled with depletingresources is going to be the scenario of many developingcountries in days to come.Conclusions :I : The population of developing countries will not

continue to increase in future.II : It will be very difficult for the governments of

developing countries to provide its people decentquality of life.

5. Statement : Mr. X is one of the probable candidatesshortlisted for the post of Director of K. L. M. Institute.Conclusions :I : Mr. X will be selected as Director of K. L. M. Institute.II : Mr. X will not be selected as Director of K. L. M.

Institute.6. Statement :

‘We follow some of the best and effective teaching learningpractices used by leading institutes all over the world’. — Astatement of professor of MN Institute.Conclusions :I. The MN Institute is one of the leading institutes of the

world.II. Whatever is being followed by world’s leading institutes

will definitely be good and useful.7. Statement :

In the absence of national health insurance or social securitycover, a person with limited resources has to depend ongovernment hospitals, which are crowded, overburdenedand understaffed.Conclusions :I. National health insurance is meant only for the affluent

sections of society.II. The government hospitals provide treatment on nominal

charges or free.8. Statement :

We do not need today in India extraordinary specialists butthose trained ordinary doctors who are dedicated to theirprofession.Conclusions :I. We should promote medical profession with dedicated

ordinary doctors rather than promoting high specialisedmedical education.

II. Extraordinary specialists are not dedicated to theirprofession.

222222Statement &Conclusion (Logical)

Page 52: yoursmahboob.wordpress.com SBI · yoursmahboob.wordpress.com iii P 101 Speed Tests for SBI Bank Clerk Exam 101 Speed Tests for SBI Bank Clerk Exam is revised and updated edition on

yoursmahboob.w

ordpress.com

SPEED TEST 2244

RESPONSE

GRID

9. a b c d e 10. a b c d e 11. a b c d e 12. a b c d e 13. a b c d e

14. a b c d e 15. a b c d e 16. a b c d e 17. a b c d e 18. a b c d e

19. a b c d e 20. a b c d e

9. Statement :“The Government will review the present policy of the dieselprice in view of further spurt in the international oil prices”— A spokesman of the Government.Conclusions :I. The Government will increase the price of the diesel after

the imminent spurt in the international oil prices.II. The Government will not increase the price of the diesel

even after the imminent spurt in the international oilprices.

10. Statement :Vegetable prices are soaring in the market.Conclusions :I. Vegetables are becoming a rare commodity.II. People cannot eat vegetables.

11. Statement :Being from a business family, Chandan was apparentlyconvinced by his parents and other family members to jointhe family trade.Conclusions :I. People should take up their family profession so that

family prospers.II. It is necessary to keep in family members happy by

choosing family’s business.12. Statement :

Global ecological issues have eclipsed local environmentalproblems which are being faced by the poor societies.Conclusions :I. Poor societies always have to suffer because of their

poverty.II. Global ecological issues are not so important. Rich

societies can bear with it.13. Statements :

Of the ten fishermen caught in a storm, nine managed toreturn to the shore.Praveen has not yet returned after four days.Conclusions :I. Praveen got killed in the storm.II. Praveen has survived the storm.

14. Statements :Now you don’t need an import licence to own a VCR.Conclusions :I. VCRs are now manufactured indigenously.II. VCRs are now freely permitted to be imported.

15. Statements :Just about everyone in Germany has been on a diet at onetime or the other and millions of them have learned that theweight they lose is all too easily regained.Still’ despite their frustration, few question the wisdom ofdieting.Conclusions :I. Germany should stop dieting.II. Germans do not learn from experience.

16. Statements :A study of planning commission reveals boom in revenues.However, this has been of little avail owing to soaringexpenditure. In the event, there has been a high dose ofdeficit financing, leading to marked rise in prices.Large financial outlays year after year had little impact onthe standard of living.Conclusions :I. A boom in revenues leads to soar in prices.II. Large financial outlays should be avoided.

17. Statements :The average number of students per teacher is 50 in the urbanarea whereas it is 60 in rural areas. The national average is 55.Conclusions :I. The student-teacher ratio in the rural areas is higher than

in the urban areas.II. More students study with the same teacher in the rural

areas as compared to those in the urban areas.18. Statement :

Morning walks are good for health.Conclusions :I. All healthy people go for morning walks.II. Evening walks are harmful.

19. Statement : The cost of manufacturing cars in state A is 30per cent less than the cost of manufacturing cars in state B.After transportation fee for the differential distances of statesA and B and the interstate taxes, it is cheaper to manufacturecars in state B than in state A for selling these cars in State C.Which of he following supports the conclusion draw in theabove statement?(a) The cost of transportation from state A to state C is

more than 30 per cent of the production cost.(b) The production cost of cars in state B is lower in

comparison to state A.(c) Only entry tax at state C is more for the production

originating in state A.(d) Entry tax at state C is more for the products originating

in state B.(e) The total of transportation cost of cars from state B to

state C and entry tax of cars at state C is less than 30per cent of the production cost of cars in state B.

20. Statement : There was a slow decline in the number ofpatients with flu-like symptoms visiting various healthfacilities in the city during the last fortnight.Which of the following substantiates the fact mentioned inthe above statement?(a) Majority of the people suffering from flu visit the health

facilities in the city.(b) There has been a continuous increase in the sale of

medicines for curing flu in the city limits.(c) People have started visiting the crowded places like

malls and cinema halls during the last fortnight after acautioned gap of one month.

(d) There is a sudden increase in the number of deathscaused by flu-like symptoms followed by respiratorycomplications.

(e) None of these

Page 53: yoursmahboob.wordpress.com SBI · yoursmahboob.wordpress.com iii P 101 Speed Tests for SBI Bank Clerk Exam 101 Speed Tests for SBI Bank Clerk Exam is revised and updated edition on

yoursmahboob.w

ordpress.com

DIRECTIONS (Qs. 1-3): Study the following information carefullyand answer the given questions.

The prospects for the Indian economy this year will be influenced by thebehaviour of the monsoon and expansion of commerce and trade. TheEleventh Plan has envisaged a growth target of 8%. If the agriculturesector does well and the world trade conditions improve then it is possibleto achieve a growth of 6-7%. We need to improve our economy and aimat a higher rate of growth in order to feed our population, maintain thestandard of living and improve the quality of life. It is now more than 10years since we have adopted reforms. We need to go forward inliberalisation but we cannot throw open the market for everything. Thereare sectors like village industries which need protection.1. Which of the following is an assumption which is implicit in the

facts stated in the above paragraph?(a) India should adopt economic policies of developed countries.(b) Free market strategy is beneficial for India, but not in all the

sectors.(c) Over the last few years, we have achieved sustained growth.(d) A very good monsoon is expected this year.(e) None of these

2. Which of the following is an inference which can be drawn from thefacts stated in the paragraph?(a) The world trade conditions don't affect Indian economy.(b) The world trade conditions have a major impact on Indian

economy.(c) Indian economy has been downgraded since last decade.(d) Govt should cut the subsidies in order to obtain sustained growth.(e) None of these

3. Which of the following is a conclusion which can be drawn fromthe facts stated in the above paragraph?(a) India may become a super economic power some day.(b) The standard of living of people has continuously degraded

in India.(c) Growth of Indian economy and a good monsoon are

complement of each other.(d) Indian economy is on the peak of growth.(e) None of these

DIRECTIONS (Qs. 4-6) : Study the following Information carefullyand answer the questions given below :Poverty measurement is an unsettled issue, both conceptually andmethodologically. Since poverty is a process as well as an outcome;many come out of it while others may be falling into it. The net effect ofthese two parallel processes Is a proportion commonly identified as the‘head count ratio’, but these ratios hide the fundamental dynamism thatcharacterises poverty in practice. The most recent poverty reestimatesby an expert group has also missed the crucial dynamism. In a studyconducted on 13.000 households which represented the entire country In1993-94 and again on 2004-05. it was found that in the ten-year period

RESPONSE

GRID

1. a b c d e 2. a b c d e 3. a b c d e 4. a b c d e 5. a b c d e

6. a b c d e

Max. Marks : 15 No. of Qs. 15 Time : 15 min. Date : ........./......../................

18.2% rural population moved out of poverty whereas another 22.1%fell into it over this period. This net increase of about four percentagepoints was seen to have a considerable variation across states and regions.4. Which of the following is a conclusion which can be drawn from

the facts slated in the above paragraph ?(a) Accurate estimates of number of people living below poverty

line in India is possible to be made.(b) Many expert groups in dia are not interested measure poverty

objectively.(c) Process of poverty measurement needs to take into account

various factors to tackle its dynamic nature.(d) People living below poverty line remain in that position for

a very long time.(e) None of these

5. Which of the following is an assumption which is implicit in thefacts stated In the above paragraph ?(a) It may not be possible to have an accurate poverty

measurement in India.(b) Level of poverty in India is static over the years.(c) Researchers avoid making conclusions on poverty

measurement data in India.(d) Government of India has a mechanism to measure level of

poverty effectively and accurately.(e) None of these

6. Which of the following is an inference which can be made fromthe facts stated in the above paragraph ?(a) Poverty measurement tools in India arc outdated.(b) Increase in number of persons falling into poverty varies

considerably across the country over a period of time.(c) Government of India has stopped measuring poverty related

studies.(d) People living in rural areas are more susceptible to fall into

poverty over the time(e) None of these

DIRECTIONS (Qs. 7 to 9) : Study the given information carefully andanswer the questions that follow:Indian Navy's now believes that its worst fear has come true, and believesit's unlikely to find any survivors onboard INS Sindhurakshak. The Navyalso came out with a statement, which said that 'the state of these twobodies and conditions within the submarine leads to firm conclusion thatfinding any surviving personnel within the submarine is unlikely'.This is for the first time that the Indian Navy has got first-handconfirmation of fatality since two massive explosions rocked INSSindhurakshak. A Navy release said that the bodies were shifted to INHSAsvini where a DNA identification test would be carried to identify thecrewmember.The damage and destruction within the submarine around the controlroom area indicates that the feasibility of locating bodies of personnel inthe forward part of the submarine is also very remote as the explosion

232323Passage BasedConclusion-1

Page 54: yoursmahboob.wordpress.com SBI · yoursmahboob.wordpress.com iii P 101 Speed Tests for SBI Bank Clerk Exam 101 Speed Tests for SBI Bank Clerk Exam is revised and updated edition on

yoursmahboob.w

ordpress.com

SPEED TEST 2346

RESPONSE

GRID

7. a b c d e 8. a b c d e 9. a b c d e 10. a b c d e 11. a b c d e

12. a b c d e 13. a b c d e 14. a b c d e 15. a b c d e

and very high temperatures, which melted steel within, would haveincinerated the bodies too, said Navy officials. But navy officials saidthat the divers would continue to search every inch of the submergedsubmarine till all bodies are either located or it can be stated with finalitythat no bodies remain to be found.7. Which of the following is the conclusion which can be drawn

from the facts stated in the above paragraph?(a) No personnel have survived in the INS Sindhurakshak tragedy.(b) It is impossible to trace the bodies of all the personnel died in

the incident.(c) The Navy is presently concentrating on reaching the interiors

of the submarine to locate and extricate any remaining bodiesthat may still be trapped within.

(d) Navy's rescue and salvage effort is proceeding slowly becauseit lacks expertise in conducting such an operation.

(e) It was a massive accident leading to the death of all the sailingcrew members.

8. Which of the following is an assumption which is implicit in thefacts stated in the above paragraph?(a) Some personnel could be found alive.(b) The submarine and its internal machinery could be reused

thus saving costs for the Navy.(c) The temperature inside the submerged submarine would be

very hot.(d) There is lack of divers who are equally skilled in using gas

cutters or other machinery required to break open jammedhatches.

(e) Looking at the level of destruction more dead bodies wouldbe found.

9. Which of the following is an inference which can be made fromthe facts stated in the above paragraph?(a) The explosion and very high temperatures, which melted

steel within, would have incinerated the bodies too.(b) Continuous efforts would lead to extrication of the bodies of

all the personnel.(c) The bodies are expected to be so badly charred that

identification will not be possible.(d) Submarines are not safe watercraft capable of carrying

independent operation underwater.(e) Old vessels and other crafts must be phased out so as to

avoid such accidents in future.

DIRECTIONS (Qs 10 to 12): Study the given information carefullyand answer the questions that follow:

Through advertisting, manufacturing exercises a high degree of controlover consumer's desires. However, the manufacturer assumes enormousrisks in attempting to predict what consumers will want and in producinggoods in quantity and distributing them in advance of final selection bythe consumers.10. Which of the following is the conclusion which can be drawn

from the facts stated in the above paragraph?(a) Distribute goods directly to the consumers.(b) Can eliminate the risk of overproduction by advertising.(c) Always take moderate and calculated risk.(d) Can predict with great accuracy the success of any product

they put on the market.(e) Must depend upon the final consumers for the success of

their undertakings.

11. Which of the following is an Assumption which is implicit in thefacts stated in the above paragraph?(a) Advertising guarantees the manufacturer in exhausting the

stocks he has produced.(b) Advertising stimulates consumers to buy more products.(c) Advertising leads to fierce competition.(d) A manufacturer manufactures a product because there is

demand in the market.(e) It is impossible to predict the demand of a product accurately.

12. Which of the following is an Inference which can be made fromthe facts stated in the above paragraph?(a) Manufacturers shall not try to predict the demand of a

product.(b) These days there are a lot of tools that helps in predicting the

demand of a product accurately.(c) Manufacturers must produce only what they can sell.(d) Consumer is the king.(e) Advertising can minimise the risk taken by manufacturers.

13. Our school district should not spend its money on the new VerbalAdvantage reading program. After all, our students get all the readingpractice they need by studying history and science.The argument above depends on which the following assumptions?(a) The Verbal Advantage program would not help the students

learn history and science.(b) Other reading programs are just as effective but less expensive

than the Verbal Advantage program.(c) The Verbal Advantage program involves only reading practice.

(d) Teaching students history and science is more importantthan teaching them reading skills.

(e) None of these14. Efficiency is all right in its place, in the shop, the factory, the store.

The trouble with efficiency is that it wants to rule our play as wellas our work; it won’t be content to reign in the shop, it follows ushome.It can be inferred from the above passage that(a) Efficiency can become all - pervading(b) Efficiency does not always pay(c) Efficiency can be more of a torture than a blessing(d) both (b) and (c) (e) None of these

15. The fare-paying capacity of people who travel on routes connectingto small towns is very low. Most successful airlines which operatein such regions have a large number of seats.Which of the following can be inferred from the above information?(a) Regional airlines are quite profitable.(b) People from cities are increasingly travelling to small towns.(c) Regional airlines have to charge low fares in order to be

profitable.(d) The number of people travelling from small towns to cities is

massive.(e) None of these

Page 55: yoursmahboob.wordpress.com SBI · yoursmahboob.wordpress.com iii P 101 Speed Tests for SBI Bank Clerk Exam 101 Speed Tests for SBI Bank Clerk Exam is revised and updated edition on

yoursmahboob.w

ordpress.com

DIRECTIONS (Qs. 1 to 3): Study the given information carefully andanswer the questions that follow:

In economics, a recession is a business cycle contraction, a generalslowdown in economic activity. Macroeconomic indicators such as GDP(Gross Domestic Product), employment, investment spending, capacityutilization, household income, business profits, and inflation fall, whilebankruptcies and the unemployment rate rise.Recessions generally occur when there is a widespread drop in spending(an adverse demand shock). This may be triggered by various events, suchas a financial crisis, an external trade shock, an adverse supply shock or thebursting of an economic bubble. Governments usually respond to recessionsby adopting expansionary macroeconomic policies, such as increasing moneysupply, increasing government spending and decreasing taxation.1. Which of the following is the conclusion which can be drawn from

the facts stated in the above paragraph?(a) Recession leads to financial crisis.(b) Demand and supply play a major role in determining the

fundamentals of an economy.(c) Cash is the king. So always keep your money safe.(d) In a recession, interest rates are reduced so as to enable people

to take loans which bring back the money in the economythus increasing spending and economic activity.

(e) Recession is a normal (albeit unpleasant) part of the businesscycle and brings in correction in the market.

2. Which of the following is an assumption which is implicit in thefacts stated in the above paragraph?(a) Recessions generally occur when there is a widespread drop

in spending.(b) When economic activity slows down, firms may decide to

reduce employment levels and save money rather than invest.(c) A recession generally lasts from six to 18 months.(d) Recession is the result of adverse supply shock or the bursting

of an economic bubble.(e) A slow growing nation can protect itself from recession.

3. Which of the following is an inference which can be made fromthe facts stated in the above paragraph?(a) Recession leads to decline in consumption, investment,

government spending, and net export activity.(b) In a recession people should generally try to watch their

spending and not take any undue risks that might put theirfuture financial goals in jeopardy.

(c) Recession affects both the developed and developing nationsof the world.

(d) Countries must reduce their imports drastically so as to avoidrecession.

(e) Recession is a business cycle contraction which brings ageneral slowdown in economic activity.

RESPONSE

GRID

1. a b c d e 2. a b c d e 3. a b c d e 4. a b c d e 5. a b c d e

6. a b c d e

Max. Marks : 15 No. of Qs. 15 Time : 15 min. Date : ........./......../................

DIRECTIONS (Q. 4 and 5) : In the following questions a paragraph isgiven. Read the paragraph carefully and answer the questions whichfollow each of these paragraph.

Fashion has become one of the largest fads among the youth. The amountof time wastage and expenditure on fashion is very large. What bothers,however, is the fact that fashion is here to stay despite countless argumentsagainst it. What is required, therefore, is that strong efforts should bemade in order to displace the excessive craze of fashion from the minds oftoday’s youth.4. Which of the following statements finds the least support by the

argument made by the author in the given paragraph?(a) Youngsters should be motivated to do constructive business

rather than wasting time on fashion.(b) The world of fashion being glamorous and glittery attracts

people towards itself.(c) Following the latest fashion increases the self-efficacy of

people, thus increasing their overall mental abilities.(d) Many universities have implemented a dress code to put a

check on the increasing fad amongst the youth which wasaffecting their grades.

(e) None of these5. Which of the following can be inferred from the given paragraph?

(a) The author has made strong efforts to wipe out fashion fromthe minds of youth.

(b) Steps need to be taken in order to control the growing fad offashion amongst the youth.

(c) The author is upset with the shift” of fashion from thetraditional ethnic wear to western outfits.

(d) Fashion world is responsible for lack of creativity among theyouth.

(e) None of these

DIRECTIONS (Qs. 6 to 8): Study the given information carefully andanswer the questions that follow:

Due to enormous profits involved in smuggling, hundreds of personshave been attracted towards this anti-national activity. Some of thembecame millionaires overnight. India has a vast coastline both on theEastern and Western Coast. It has been a heaven for smugglers who havebeen carrying on their activities with great impunity. There is no doubt,that from time to time certain seizures were made by the enforcementauthorities, during raids and ambush but even allowing these losses thesmugglers made huge profits.6. Which of the following is the conclusion which can be drawn from

the facts stated in the above paragraph?(a) smuggling hampers the economic development of a nation.(b) smuggling ought to the curbed.(c) authorities are taking strict measures to curb smuggling.(d) smuggling is fast increasing in our country owing to the quick

profit it entails.(e) None of these

24132424Passage BasedConclusion-2

Page 56: yoursmahboob.wordpress.com SBI · yoursmahboob.wordpress.com iii P 101 Speed Tests for SBI Bank Clerk Exam 101 Speed Tests for SBI Bank Clerk Exam is revised and updated edition on

yoursmahboob.w

ordpress.com

SPEED TEST 2448

RESPONSE

GRID

7. a b c d e 8. a b c d e 9. a b c d e 10. a b c d e 11. a b c d e

12. a b c d e 13. a b c d e 14. a b c d e 15. a b c d e

7. Which of the following is an assumption which is implicit in thefacts stated in the above paragraph?(a) Coastlines provide easy escape for the smugglers.(b) People are driven by money not by values.(c) Smugglers are smart enough to be caught.(d) Government can earn a lot of money from seizing smuggled

goods.(e) Smuggling cannot be stopped as smugglers will find new

ways of doing it.8. Which of the following is an inference which can be made from

the facts stated in the above paragraph?(a) The enforcement authorities lack in their efforts to curb

smuggling.(b) Authorities are taking bribe from smugglers there by

increasing their potential.(c) There is no measure taken by authorities.(d) Smuggling hampers the GOD of a nation.(e) None of these

DIRECTIONS (Qs. 9 to 11): Study the given information carefully andanswer the questions that follow:

Exports and imports, a swelling favourable balance of trade, investmentsand bank-balances, are not an index or a balance sheet of nationalprosperity. Till the beginning of the Second World War, English exportswere noticeably greater than what they are today. And yet England hasgreater national prosperity today than it ever had. Because the income ofaverage Englishmen, working as field and factory labourers, clerks,policemen, petty shopkeepers and shop assistants, domestic workersand other low-paid workers, has gone up.9. Which of the following is the conclusion which can be drawn

from the facts stated in the above paragraph?(a) a country’s economic standard can be best adjudged by per

capita income.(b) a country’s balance of trade is the main criteria of determining

its economic prosperity.(c) a nation’s economy strengthens with the increase in exports.(d) English trade has continually increased since the Second

World War.(e) None of these

10. Which of the following is an assumption which is implicit in thefacts stated in the above paragraph?(a) Increasing exports and decreasing imports helps a nation

prosper.(b) Prosperity of a nation is the outcome of prosperity of its

people.(c) Greater exports leads to higher GDP which leads to better

flow of money in the economy.(d) Second World War led to deterioration in the national

prosperity of England.(e) None of these

11. Which of the following is an inference which can be made fromthe facts stated in the above paragraph?(a) Countries must focus on increasing income of its people and

not an exports or imports.(b) Favourable balance of trade will automatically lead to higher

income of people.

(c) Increase in income of all leads to a better life style of people.(d) Increase in exports does not guarantee increase in income

levels of all class of people.(e) The gap between rich and poor determines national

prosperity.

DIRECTIONS (Qs. 12 to 14): Study the given information carefullyand answer the questions that follow:

The consumption of harmful drugs by the people can be prevented notonly by banning their sale in the market but also by instructing usersabout their dangerous effects which they must understand for their safety.Also the drug addicts may be provided with proper medical facilities fortheir rehabilitation. This will help in scaling down the use of drugs.12. Which of the following is the conclusion which can be drawn

from the facts stated in the above paragraph?(a) are on increase in the society(b) can always be reduced.(c) are due to lack of medical facilities.(d) can be eliminated with the help of banning their sale.(e) may be channelised through proper system.

13. Which of the following is an assumption which is implicit in thefacts stated in the above paragraph?(a) Such drugs are harmful and can lead to severe depression and

suicidal tendencies.(b) People using drugs harm the society.(c) Scaling down the use of such harmful drugs is beneficial for

the society.(d) Medical facilities needs to be improved in our country.(e) The government is not taking appropriate actions to curb the

use of harmful drugs.14. Which of the following is an inference which can be made from

the facts stated in the above paragraph?(a) Such harmful drugs should not be available at the first place.(b) Controlling drug addicts can scale down the use of drugs.(c) People using these drugs must be rehabilitated.(d) Awareness about the ill-effects of these drugs can scale down

their usage.(e) Bad company leads to spread of use of such drugs.

15. “If you want a hassle-free holiday package for city M, then joinonly our tour. Hurry up; only a few seats available” – Anadvertisement of XYZ Tourist Company.If the above statement is true then which of the following has beenassumed while making the statement?(a) No seats may be available with other tour operators for city

M .(b) Nowadays people have a lot of money to spend on their

comforts.(c) Travel packages offered by other tour operators are neither

cheap nor comfortable.(d) Many people desire convenience and comfort while going

for a holiday.(e) None of these

Page 57: yoursmahboob.wordpress.com SBI · yoursmahboob.wordpress.com iii P 101 Speed Tests for SBI Bank Clerk Exam 101 Speed Tests for SBI Bank Clerk Exam is revised and updated edition on

yoursmahboob.w

ordpress.com

1. Each of the six faces of a cube is numbered by one of thedigits from 1 to 6. This cube is shown in its four differentpositions in the figure I, II, III, and IV.

6

3 2

1

4 2

5

6 4

6

2 4

I II III IV

Consider the following statements.1. Figures II and III are sufficient to known as to which

face is opposite to the face numbered 6.2. figures II and III are sufficient to known as to which

face is opposite to the face numbered 4. .3. Figures I and IV are sufficient to known as to which

face is opposite to the face numbered 3.Which of the statements given above are correct?(a) 1 and 3 only (b) 1 and 2 only(c) 2 and 3 only (d) 1, 2 and 3(e) None of these

2. Six faces of a cube are numbered from 1 to 6, each facecarrying one different number. Further,1. The face 2 is opposite to the face 6.2. The face 1 is opposite to the face 5.3. The face 3 is between the face 1 and the face 54. The face 4 is adjacent to the face 2.Which one of the following is correct?(a) The face 2 is adjacent to the face 3(b) The face 6 is between the face 2 and the face 4(c) The face 1 is between the face 5 and the face 6(d) None of the above(e) None of these

3. Six squares are coloured, front and back, red (R), blue (B),yellow (Y), green (G), white (W) and orange (O) and arehinged together as shown in the figure given below. If theyare folded to form a cube, what would be the face oppositethe white face?

R BG Y O

W(a) R (b) G (c) B(d) O (e) None of these

4. Three views of a cube following a particular motion are givenbelow:

RESPONSE

GRID

1. a b c d e 2. a b c d e 3. a b c d e 4. a b c d e 5. a b c d e

6. a b c d e 7. a b c d e 8. a b c d e 9. a b c d e 10. a b c d e

Max. Marks : 20 No. of Qs. 20 Time : 15 min. Date : ........./......../................

AB

K H

MK

PH

B

What is the letter opposite to A?(a) H (b) P (c) B(d) M (e) None of these

DIRECTIONS (Qs. 5 -8) : A cube is coloured red on all faces. It iscut into 64 smaller cubes of equal size. Now, answer the followingquestions based on this statement :5. How many cubes have no face coloured?

(a) 24 (b) 16 (c) 8(d) 0 (e) None of these

6. How many cubes are there which have only one facecoloured?(a) 4 (b) 8 (c) 16(d) 24 (e) None of these

7. How many cubes have two red opposite faces?(a) 0 (b) 8 (c) 16(d) 24 (e) None of these

8. How many cubes have three faces coloured?(a) 24 (b) 16 (c) 8(d) 4 (e) None of these

9.

How many dots are there on the dice opposite to the onedot?(a) 2 (b) 4 (c) 5(d) 6 (e) None of these

10. Select from alternative, the box that can be formed by foldingthe sheet shown.

1

2

34

5

6

(a) 3 52

(b) 6 34

(c) 2 53

(d) 1 34

(e) None of these

132525Cube & Dice

Page 58: yoursmahboob.wordpress.com SBI · yoursmahboob.wordpress.com iii P 101 Speed Tests for SBI Bank Clerk Exam 101 Speed Tests for SBI Bank Clerk Exam is revised and updated edition on

yoursmahboob.w

ordpress.com

SPEED TEST 2550

RESPONSE

GRID

11. a b c d e 12. a b c d e 13. a b c d e 14. a b c d e 15. a b c d e

16. a b c d e 17. a b c d e 18. a b c d e 19. a b c d e 20. a b c d e

11. Select from alternative, the box that can be formed by foldingthe sheet shown.

×

= ¸

+

(a)

×

+ ¸ (b) =

¸ (c) ׸

(d) ×

¸

– (e) None of these

12. Select from the alternative, the box that can be formed byfolding the sheet shown in figure (X) :

(X)

(A) (B) (C) (D)(a) A only (b) A and C only(c) A , C and D only (d) A, B, C and D(e) None of these

13. Three positions of a cube are as shown below :

?

The figure on the face opposite the triangle is the :(a) pentagon (b) circle(c) question mark (d) rectangle(e) None of these

14. Which number is on the face opposite to 6 on the dicewhose four positions as shown below ?

6

32

6

2 4

5

6 4

1

4 2

(a) 1 (b) 2 (c) 3(d) 4 (e) None of these

15. A cube is painted white on all the sides. It is then cut into 64smaller cubes of equal sizes. How many of these smallercubes have no paint on any side?(a) 8 (b) 6 (c) 4(d) 1 (e) None of these

16. How many cubes are there in the following figure?

(a) 6 (b ) 10 (c) 12(d) 8 (e) None of these

DIRECTIONS (Qs. 17 - 19) : The questions below are to beanswered on the basis of the three views of a cube given as follows:

5 4 24 6 61 3 5

17. Which number is on the face opposite to 1 ?(a) 3 (b) 2 (c) 6(d) 4 (e) None of these

18. Which number is on the face opposite to 4 ?(a) 2 (b) 3 (c) 6(d) 1 (e) None of these

19. Which number is at the bottom face of figure 1?(a) 3 (b) 2 (c) 6(d) 1 (e) None of these

20. Four positions of a cube are shown below. Which symbolis opposite to the face having ‘ ’?

(a) (b) (c) (d)

÷ ÷ ÷

××

+ +

(a) D (b) – (c) ×(d) ÷ (e) None of these

Page 59: yoursmahboob.wordpress.com SBI · yoursmahboob.wordpress.com iii P 101 Speed Tests for SBI Bank Clerk Exam 101 Speed Tests for SBI Bank Clerk Exam is revised and updated edition on

yoursmahboob.w

ordpress.com

DIRECTIONS (Qs. 1 to 5): In each of the questions given below which one of the five Answer Figures on the right should come afterthe Problem Figures on the left, if the sequence were continued?

Problem Figures Answer Figures

1.

(a) (b) (c) (d) (e)

CO B K

SE

LSE

L O B

C

KKC

OB

LE S

CKB

OS

E

LL O

B

SE

K CCE

BK

SO

LES

C B K

O

C

LS

EL

B K

SO

CE

O

K

L

BL C

E

KB

S O

2.

(a) (b) (c) (d) (e)

3.

(a) (b) (c) (d) (e)

4.

(a) (b) (c) (d) (e)

o o

o oo

o o o o occ

c

c

c

c c c c c

5.

(a) (b) (c) (d) (e)

= H O L E

K

KO

H

HH

HHE

EE

C C

S

L LL

L LH

HHC

S

S S S

L

L LL

RESPONSE

GRID

1. a b c d e 2. a b c d e 3. a b c d e 4. a b c d e 5. a b c d e

Max. Marks : 15 No. of Qs. 15 Time : 15 min. Date : ........./......../................

26132626Non-verbalReasoning - I

Page 60: yoursmahboob.wordpress.com SBI · yoursmahboob.wordpress.com iii P 101 Speed Tests for SBI Bank Clerk Exam 101 Speed Tests for SBI Bank Clerk Exam is revised and updated edition on

yoursmahboob.w

ordpress.com

SPEED TEST 2652

RESPONSE

GRID

6. a b c d e 7. a b c d e 8. a b c d e 9. a b c d e 10. a b c d e

11. a b c d e 12. a b c d e 13. a b c d e 14. a b c d e 15. a b c d e

DIRECTIONS (Qs. 6 to 10) : In each of the following questions aseries begins with an unnumbered figure on the extreme left. Oneand only one of the five lettered figures in the series does not fitinto the series. The two unlabelled figures, one each on the extremeleft and the extreme right, fit into the series. You have to take asmany aspects into account as possible of the figures in the seriesand find out the one and only one of the five lettered figures whichdoes not fit into the series. The letter of that figure is the answer.

6.

(a) (b) (c) (d) (e)

s

­=

s s s s s s

­ = ­ = = ­ ­ = ­ = ­=

7.

(a) (b) (c) (d) (e)

N

N

× ×

8.

(a) (b) (c) (d) (e)

××

9.

(a) (b) (c) (d) (e)

c s

sc

c s

s c cs

c s c

s

10.

(a) (b) (c) (d) (e)

DIRECTIONS (Qs. 11-15) : In each of the questions given belowwhich one of the five answer figures on the bottom should comeafter the problem figures on the top if the sequence werecontinued?

11. Problem Figures

z s

oo s

z*

?o=

* *

=

Answer Figures

oz * o s z s * o s o =

**o*

(a) (b) (c) (d) (e)

12. Problem Figures

?Answer Figures

(a) (b) (c) (d) (e)

13. Problem Figures

?Answer Figures

(a) (b) (c) (d) (e)

14. Problem Figures

?Answer Figures

(a) (b) (c) (d) (e)

15. Problem Figures

?Answer Figures

(a) (b) (c) (d) (e)

Page 61: yoursmahboob.wordpress.com SBI · yoursmahboob.wordpress.com iii P 101 Speed Tests for SBI Bank Clerk Exam 101 Speed Tests for SBI Bank Clerk Exam is revised and updated edition on

yoursmahboob.w

ordpress.com

DIRECTIONS (Qs. 1 to 5): In each of the questions, there are twosets of figures. The figures on upper side are problem figuresmarked by letters (1), (2), (3), (4) and (5), and on the bottom sideare answer figures marked by numbers (a), (b), (c), (d) and (e). Aseries is established, if one of the five answer figures is placed inplace of the (?) sign in the problem figures. That figure is youranswer.

1. Problem Figures

(1) (2) (3) (4)

?(5)

Answer Figures

(a) (b) (c) (d) (e)

2. Problem Figures

(1) (2) (3) (4)

S·= 0

? C?

(5)

· = S

C

0 ?

S 0 C

· ?=

S C ?

= 0

·Answer Figures

(a) (b) (c) (d) (e)

S·=0

?

C ·=

SC

0

?

S0

C

·

?=

S C?

=

0

··

=S

C

0

?

3. Problem Figures

(1) (2) (3) (4)

?

(5)

× ×

×

Answer Figures

(a) (b) (c) (d) (e)4. Problem Figures

(1) (2) (3) (4)

?

(5)Answer Figures

(a) (b) (c) (d) (e)5. Problem Figures

(1) (2) (3) (4)

?

(5)Answer Figures

(a) (b) (c) (d) (e)

RESPONSE

GRID1. a b c d e 2. a b c d e 3. a b c d e 4. a b c d e 5. a b c d e

Max. Marks : 15 No. of Qs. 15 Time : 15 min. Date : ........./......../................

DIRECTIONS (Qs. 6 to 10): In each of the following questions, a related pair of figures is followed by five lettered pairs of figures. Selectthe pair that has relationship similar to that in the question figure. The best answer is to be selected from a group of fairly close choices.

6.

I II I II I II I II I II I II(a) (b) (c) (d) (e)

27132727Non-verbalReasoning - II

Page 62: yoursmahboob.wordpress.com SBI · yoursmahboob.wordpress.com iii P 101 Speed Tests for SBI Bank Clerk Exam 101 Speed Tests for SBI Bank Clerk Exam is revised and updated edition on

yoursmahboob.w

ordpress.com

SPEED TEST 2754

7.

I II I II I II I II I II I II(a) (b) (c) (d) (e)

8.

I II I II I II I II I II I II(a) (b) (c) (d) (e)

9.

I II I II I II I II I II I II(a) (b) (c) (d) (e)

10.

I II I II I II I II I II I II(a) (b) (c) (d) (e)

RESPONSE

GRID

6. a b c d e 7. a b c d e 8. a b c d e 9. a b c d e 10. a b c d e

11. a b c d e 12. a b c d e 13. a b c d e 14. a b c d e 15. a b c d e

DIRECTIONS (Qs. 11-15): In each of the questions given below which one of the following five answer figures should come after theproblem figures, if the sequence were continued?

Answer Figures

(a) (b) (c) (d) (e)

P

PPP

U U U P U

U

14. Problem Figures

s

z o

D zDC

o s s o

DCz

C C

D Ds s

o oz zC

Answer Figures

(a) (b) (c) (d) (e)o sz

D C D

C

C

D

s

s ssz

z C CDoo o

z o

z

D

15. Problem Figures

Answer Figures

(a) (b) (c) (d) (e)

11. Problem Figures

Z O

E V

E V

P T

P T

D S

D S

A Z

A Z

B

Answer Figures

B

=

B B = =

BA Z=A Z(a) (b) (c) (d) (e)

12. Problem Figures

DZ

T

T

DO

Z

T DZ

DO

TZ OT

DZ

Answer Figures

(a) (b) (c) (d) (e)

DZ

O

T

Z D O

OOT

DO T DZ T Z DZ T

13. Problem Figures

P U

U U

U

U

P P

P P

Page 63: yoursmahboob.wordpress.com SBI · yoursmahboob.wordpress.com iii P 101 Speed Tests for SBI Bank Clerk Exam 101 Speed Tests for SBI Bank Clerk Exam is revised and updated edition on

yoursmahboob.w

ordpress.com

1. How many meaningful three letter English words can beformed with the letters AER, using each letter only once ineach word ?(a) None (b) One(c) Two (d) Three(e) Four

2. If in a certain language LATE is coded as 8 & 4 $ and HIREis coded as 7*3$ then how will HAIL be coded in the samelanguage ?(a) 7 & 8* (b) &7*8(c) 7*& 8 (d) 7&*8(e) None of these

3. Four of the following five are alike in a certain way and soform a group. Which is the one that does not belong to thatgroup ?(a) Stem (b) Tree(c) Root (d) Branch(e) Leaf

4. Four of the following five are alike in a certain way and soform a group. Which is the one that does not belong to thatgroup ?(a) Clutch (b) Wheel(c) Break (d) Car(e) Gear

5. How many such pairs of letters are there in word SENDING,each of which has as many letters between its two letters asthere are between them in the English alphabets ?(a) None (b) One(c) Two (d) Three(e) More than three

6. Four of the following five are alike in a certain way and soform a group. Which is the one that does not belong to thatgroup?(a) 169 (b) 441(c) 361 (d) 529(e) 289

RESPONSE

GRID

1. a b c d e 2. a b c d e 3. a b c d e 4. a b c d e 5. a b c d e

6. a b c d e 7. a b c d e 8. a b c d e 9. a b c d e 10. a b c d e

11. a b c d e 12. a b c d e

Max. Marks : 40 No. of Qs. 40 Time : 25 min. Date : ........./......../................

7. In a certain code DISPLAY is written as RHCQZBM. How isGROUPED written in that code?(a) PSHTEFQ (b) NQFVCDO(c) NQFVEFQ (d) PSHTCDO(e) None of these

8. Among P, Q, R, T and W each having different weight, T isheavier than W and lighter than only P. Q is not the lightest.Who among them is definitely the lightest ?(a) R (b) W(c) R or W (d) Data inadequate(e) None of these

9. In a row of thirty children M is sixth to the right of R who istwelfth from the left end. What is M’s position from the rightend of the row ?(a) Twelfth (b) Thirteenth(c) Fourteenth (d) Data inadequate(e) None of these

10. What should come next in the following letter series?B D F H J L N A C E G I K M B D F H J L A C E G I K B D F H J(a) B (b) L(c) M (d) F(e) None of these

DIRECTIONS (Qs. 11-15) : Study the following arrangementcarefully and answer the questions given below

M, D, K, R, T, H, W and A are sitting around a circle facing at thecentre. D is second to the right of M who is fifth to the left of T. Kis third to the right of R who is second to the right of D. H issecond to the right of W.11. Who is second to the right of A ?

(a) M (b) D(c) K (d) Data inadequate(e) None of the above

12. Who is third to the left of M ?(a) A (b) T(c) H (d) D(e) Data inadequate

28132828Section Test :Reasoning

Page 64: yoursmahboob.wordpress.com SBI · yoursmahboob.wordpress.com iii P 101 Speed Tests for SBI Bank Clerk Exam 101 Speed Tests for SBI Bank Clerk Exam is revised and updated edition on

yoursmahboob.w

ordpress.com

SPEED TEST 2856

RESPONSE

GRID

13. a b c d e 14. a b c d e 15. a b c d e 16. a b c d e 17. a b c d e

18. a b c d e 19. a b c d e 20. a b c d e 21. a b c d e 22. a b c d e

23. a b c d e 24. a b c d e 25. a b c d e

13. Who is fourth to the right of H ?(a) A (b) T(c) R (d) K(e) None of these

14. In which of the following combinations is the first personsitting between the second and the third person ?(a) KMW (b) MWD(c) RHT (d) TAK(e) None of the above

15. In which of the following pairs is the second person sittingto the immediate left of the first person ?(a) MW (b) AK(c) TA (d) RH(e) WD

DIRECTIONS (Qs. 16 - 20): In each question below, there arethree of two statements followed by two conclusions numberedI and II. You have to take the three given statements to be trueeven if they seem to be at variance from commonly known factsand then decide which of the given conclusions logically followfrom the three statements.

Give answer (a) if only conclusion I follows.Give answer (b) if only conclusion II follows.Give answer (c) if either I or II follows.Give answer (d) if neither I nor II follows.Give answer (e) if both I and II follow.

16. Statements:All fish are birds.Some hens are fish.Conclusion:I. Some hens are birds.II. No birds are hens;

17. Statements:Some shoes are coats.Some coats are buttonsConclusions:I. No button is shoe.II. Some shoes are buttons.

18. Statements:All bats are boys.All boys are gloves.Conclusions:I. Some gloves are bats.II. All bats are gloves..

19. Statements:All puppies are tigers.All kittens are tigers.Conclusions:I. All puppies are kittens.II. All tigers are puppies.

20. Statements: Some doctors are nurses.All nurses are patients.Conclusions:I. All doctors are patients.II. Some patients are doctors.

DIRECTIONS (Qs.21-25) : Study the following information toanswer the given questions :

Six plays – A, B, C, D, E and F are to be staged on six days of theweek starting from Monday and ending on Saturday. Play C isstaged on Tuesday. Plays A, F and B are staged one after theother in the same order. Play D is not staged on Monday orWednesday.21. How many plays are staged after play A is staged ?

(a) One (b) Two(c) Three (d) Four(e) Cannot be determined

22. Four of the following five form a group based on the daysthat they are staged. Which one of them does not belong tothat group ?(a) EC (b) FD(c) CA (d) AF(e) BD

23. Which play is staged immediately before the day play E isstaged ?(a) B (b) A(c) F (d) D(e) There is no such play staged

24. If play D was staged on a Monday, which of the followingplays would definitely be staged on a Saturday (all the otherconditions given above remain the same) ?(a) B (b) A(c) E (d) E or B(e) Cannot be determined

25. Which play is staged on Thursday ?(a) B (b) E(c) D (d) F(e) Cannot be determined

Page 65: yoursmahboob.wordpress.com SBI · yoursmahboob.wordpress.com iii P 101 Speed Tests for SBI Bank Clerk Exam 101 Speed Tests for SBI Bank Clerk Exam is revised and updated edition on

yoursmahboob.w

ordpress.com

SPEED TEST 28 57

DIRECTIONS (Qs.26-28) : In each question a group of letters isgiven followed by four combinations of number/symbol numbered(a), (b), (c) and (d). Letters are to be coded as per the scheme andconditions given below. You have to find out the serial number ofthe combination, which represents the letter group. Serial numberof that combination is your answer. If none of the combinations iscorrect, your answer is (e) i.e. None of these.

Letters Q M S I N G D K A L P R B J ENumber/ 7 @ 4 # % $ 6 1 2 £ 5 * 9 8 3Symbol

Conditions :(i) If the first letter is a consonant and the last a vowel, both are

to be coded as the code of the vowel.(ii) If the first letter is vowel and the last a consonant, the codes

for the first and the last are to be interchanged.(iii) If no vowel is present in the group of letters, the second and

the fifth letters are to be coded as ©.26. BARNIS

(a) 9 2 * % # 4 (b) 9 2 4 # * %(c) 9 2 * # % 9 (d) 4 2 * # % 4(e) None of these

27. DMBNIA(a) 6 @ 9 % # 2 (b) 2 @ 9 % # 6(c) 2 @ 9 % # 6 (d) 2 © 9 % # 2(e) None of these

28. IJBRLG(a) # 8 9 * £ $ (b) # 8 9 * £ #(c) $ 8 9 * £ # (d) $ 8 9 * £ $(e) None of these

DIRECTIONS (Qs. 29-31) : Study the given information carefullyand answer the questions that follow:

Though the waste of time or the expenditure on fashions is verylarge, yet fashions have come to stay. They will not go, come whatmay. However, what is now required is that strong efforts shouldbe made to displace the excessive craze for fashion from the mindsof these youngsters.29. Which of the following is the conclusion which can be drawn

from the facts stated in the above paragraph?(a) Fashion is the need of the day.(b) The excessive craze for fashion is detrimental to one's

personality.(c) The hoard for fashion should be done away with so as

not to let down the constructive development.(d) Work and other activities should be valued more than

the outward appearance.(e) None of these

30. Which of the following is an Assumption which is implicit inthe facts stated in the above paragraph(a) Whatever you do you cannot protect yourself from

fashion.(b) Fashion changes the outward appearance and makes

one feel better.(c) Today's youth is highly influenced by fashion.(d) Normal clothing is good as it keeps a person grounded.(e) Fashion is spoiling the minds of the youngsters.

31. Which of the following is an Inference which can be madefrom the facts stated in the above paragraph?(a) Moderate investment of time and money by youth is

acceptable.(b) Fashion brings with itself feelings of materialism.(c) All of us have to adopt ourselves to latest fashion to

some bit.(d) Fashion helps in projecting a better image of one's

personality.(e) People who will avoid fashion completely will be

outdated.

DIRECTIONS (Qs. 32-35) % In the following questions, the symbols@, #, $, © and % are used with the following meaning as illustrated

below :

'P@ Q' means ‘P is not smaller than Q’,

'P# Q' means ‘P is not greater than Q’.

'P$ Q' means ‘P is neither greater than nor equal to Q’.

'P© Q' means ‘P is neither smaller than nor equal to Q’.

'P% Q' means ‘P is neither smaller than nor greater than Q’.

Now in each of the following questions, assuming the givenstatements to be true, find which of the two conclusions Iand II given below them is/are definitely true.Give answer (a) if only conclusion I is true.Give answer (b) if onlyconclusion II is true.Give answer (c) if either conclusion I or conclusion II is true.Give answer (d) if neither conclusion I nor conclusion II istrue.Give answer (e) if both conclusions I and II are true.

32. Statements: % V $ W, W@T, T#H

Conclusions: I. V©T II. H%W

33. Statements: H © M, M @ E, E $ C

Conclusions: I. C@M II. H©E

RESPONSE

GRID

26. a b c d e 27. a b c d e 28. a b c d e 29. a b c d e 30. a b c d e

31. a b c d e 32. a b c d e 33. a b c d e

Page 66: yoursmahboob.wordpress.com SBI · yoursmahboob.wordpress.com iii P 101 Speed Tests for SBI Bank Clerk Exam 101 Speed Tests for SBI Bank Clerk Exam is revised and updated edition on

yoursmahboob.w

ordpress.com

SPEED TEST 2858

34. Statements: N@J, J%R, R©H

Conclusions: I. R#N II. N©H

35. Statements: L@K, K©A, A$W

Conclusions: I . W$L II. L#W

DIRECTIONS (Qs. 36-40) : Study the following arrangementcarefully and answer the questions given below

D 5 d R @ A K © 3 9 B J E * F $ M P I 4 H 1 W 62 # U Q 8 T N

36. How many such numbers are there in the above arrangementeach of which is immediately preceded by a symbol andimmediately followed by a letter?(a) None (b) One(c) Two (d) Three(e) More than three

37. Which of the following is the ninth to the right of the twentysecond from the right end of the above arrangement ?(a) E (b) I(c) D (d) N(e) None of these

38. How many such symbols are there in the above arrangementeach of which is immediately preceded by a number andimmediately followed by a letter?(a) None (b) One(c) Two (d) Three(e) More than three

39. If all the numbers are dropped from the above arrangement,which of the following will be the eleventh from the left end?(a) B (b) H(c) $ (d) *(e) None of these

40. Four of the following five are alike in a certain way based ontheir positions in the above arrangement and so form a group.Which is the one that does not belong to that group?(a) 41I (b) 6#W(c) QTU (d) RAd(e) J9E

RESPONSE

GRID

34. a b c d e 35. a b c d e 36. a b c d e 37. a b c d e 38. a b c d e

39. a b c d e 40. a b c d e

Page 67: yoursmahboob.wordpress.com SBI · yoursmahboob.wordpress.com iii P 101 Speed Tests for SBI Bank Clerk Exam 101 Speed Tests for SBI Bank Clerk Exam is revised and updated edition on

yoursmahboob.w

ordpress.com

1. Which is the smallest of the following numbers ?

(a)3

2(b) – 1.5 (c)

3

5

(d) – 1.375 (e) None of these2. Three-fourth of 68 is less than two-thirds of 114 by

(a) 12 (b) 25 (c) 35(d) 48 (e) None of these

3. If one-third of a number is 3 more than one -fourth of thenumber, then the number is :(a) 18 (b) 24 (c) 30(d) 36 (e) None of these

4. Which is the smallest of the following numbers ?

(a) 7 (b)7

1(c)

7

7

(d)7

1(e) None of these

5. If one-eighth of a pencil is black, half of the remaining is

yellow and the remaining 2

13 cm is blue, then the length of

the pencil is :(a) 6 cm (b) 7 cm (c) 8 cm(d) 11 cm (e) None of these

6. A boy was asked to write 25 92 ´ but he wrote 2592. Thenumerical difference between the two is:(a) 0 (b) 3 (c) 2(d) 9 (e) None of these

7. In a group of buffaloes and ducks, the number of legs are24 more than twice the number of heads. What is the numberof buffaloes in the group ?(a) 6 (b) 12 (c) 8(d) 15 (e) None of these

RESPONSE

GRID

1. a b c d e 2. a b c d e 3. a b c d e 4. a b c d e 5. a b c d e

6. a b c d e 7. a b c d e 8. a b c d e 9. a b c d e 10. a b c d e

11. a b c d e 12. a b c d e 13. a b c d e 14. a b c d e 15. a b c d e

Max. Marks : 30 No. of Qs. 30 Time : 20 min. Date : ........./......../................

8. How many of the following numbers are divisible by 132 ?264, 396, 4, 762, 792, 968, 2178, 5184, 6336

(a) 4 (b) 5 (c) 6(d) 7 (e) None of these

9. Find the number which, when added to itself 13 times, gives112.(a) 9 (b) 7 (c) 8(d) 11 (e) None of these

10. The smaller number by which 3600 can be divided to make ita perfect cube is :(a) 9 (b) 50 (c) 300(d) 450 (e) None of these

11. The least number having four digits which is a perfect squareis :(a) 1004 (b) 1016 (c) 1036(d) 1226 (e) None of these

12. A positive integer, which added to 1000, gives a sum which isgreater than the product obtained when it is multiplied by1000. The positive integer is :(a) 1 (b) 5 (c) 2(d) 3 (e) None of these

13. Of the 120 people in the room, 5

3 are women. If

3

2 of the

people are married, then what is the maximum number ofwomen in the room who are unmarried?(a) 40 (b) 50 (c) 60(d) 80 (e) None of these

14. How many odd numbered pages are there in a book of 1089pages ?(a) 545 (b) 544 (c) 543(d) 547 (e) None of these

15. If p is a prime number greater than 3, then )1p( 2 - is always

divisible by :(a) 6 but not 12 (b) 24 (c) 12 but not 24(d) 16 (e) None of these

1329Number System - I 29

Page 68: yoursmahboob.wordpress.com SBI · yoursmahboob.wordpress.com iii P 101 Speed Tests for SBI Bank Clerk Exam 101 Speed Tests for SBI Bank Clerk Exam is revised and updated edition on

yoursmahboob.w

ordpress.com

60 SPEED TEST 29

RESPONSE

GRID

16. a b c d e 17. a b c d e 18. a b c d e 19. a b c d e 20. a b c d e

21. a b c d e 22. a b c d e 23. a b c d e 24. a b c d e 25. a b c d e

26. a b c d e 27. a b c d e 28. a b c d e 29. a b c d e 30. a b c d e

16. A man has 1044 candles. After burning all of them he canmake a new candle from 9 stubs left behind. Find the maximumnumber of candles that can be made :(a) 116 (b) 120 (c) 130(d) 136 (e) None of these

17. Find the whole number which when increased by 20 is equalto 69 times the reciprocal of the number:(a) 7 (b) 5 (c) 3(d) 2.5 (e) None of these

18. A clock strikes 4 taking 9 seconds. In order to strike 12 at thesame rate, the time taken is(a) 36 seconds (b) 27 seconds (c) 30 seconds(d) 33 seconds (e) None of these

19. The sum of the place values of 3 in the number 50, 35, 35 is(a) 3300 (b) 6 (c) 60(d) 3030 (e) None of these

20. How many numbers are there between 500 and 600 in which9 occurs only once?(a) 19 (b) 18 (c) 20(d) 21 (e) None of these

21. The product of two consecutive even numbers is 12768.

What is the greater number ?

(a) 110 (b) 108 (c) 114

(d) 112 (e) None of these

22. The product of two consecutive odd numbers is 19043.Which is the smaller number?(a) 137 (b) 131 (c) 133(d) 129 (e) None of these

23. The product of two successive numbers is 8556. What is thesmaller number?

(a) 89 (b) 94 (c) 90

(d) 92 (e) None of these

24. What is the least number to be added to 1500 to make it aperfect square?

(a) 20 (b) 21 (c) 22

(d) 23 (e) None of these25. How many pieces of 8.6 metres length cloth can be cut out

of a length of 455.8 metres cloth?

(a) 43 (b) 48 (c) 55(d) 53 (e) 62

26. If an amount of 15,487 is divided equally among 76 students,approximately how much amount will each student get?

(a) `206 (b) `210 (c) `204(d) `218 (e) `212

27. The difference between a number and one fifth of it is 84.What is the number?(a) 95 (b) 100 (c) 105(d) 108 (e) 112

28. A, B, C, D and E are five consecutive odd numbers The sumof A and C is 146. What is the value of E?(a) 75 (b) 81 (c) 71(d) 79 (e) None of these

29. The sum of five consecutive even numbers of set A is 280.What is the sum of a different set B of five consecutivenumbers whose lowest number is 71 less than double thelowest number of set A ?(a) 182 (b) 165 (c) 172(d) 175 (e) None of these

30. Deepak has some hens and some goats. If the total numberof animal heads is 90 and the total number of animal feet is248, what is the total number of goats Deepak has ?(a) 32 (b) 36 (c) 34(d) Cannot be determined(e) None of these

Page 69: yoursmahboob.wordpress.com SBI · yoursmahboob.wordpress.com iii P 101 Speed Tests for SBI Bank Clerk Exam 101 Speed Tests for SBI Bank Clerk Exam is revised and updated edition on

yoursmahboob.w

ordpress.com

1. Farah got married 8 years ago. Today her age is 127 times

her age at the time of her marriage. At present her daughter'sage is one-sixth of her age. What was her daughter's age 3years ago?(a) 6 years (b) 2 years(c) 3 years (d) Cannot be determined(e) None of these

2. If (12)3 is subtracted from the square of a number the answerso obtained is 976. What is the number?(a) 58 (b) 56 (c) 54(d) 52 (e) None of these

3. The cost of 5 chairs and 8 tables is ` 6,574. What is the costof 10 chairs and 16 tables?(a) ` 15674 (b) ` 16435 (c) ` 13148(d) Cannot be determined(e) None of these

4. If (56)2 is added to the square of a number, the answer soobtained is 4985. What is the number?(a) 52 (b) 43 (c) 65(d) 39 (e) None of these

5. The unit’s digit in the product 557135 1137 ´´ is :(a) 1 (b) 3 (c) 7(d) 9 (e) None of these

6. If p be a number between 0 and 1, which one of the followingwill be true ?

(a) p> p (b) pp

12>

(c)1

pp

< (d) 23 pp >

(e) None of these7. In a certain shop, 9 oranges cost as much as 5 apples, 5

apples cost as much as 3 mangoes, 4 mangoes cost asmuch as 9 lemons. If 3 lemons cost 48 paise, the price of anorange is :(a) 12 paise (b) 14 paise (c) 13 paise(d) 15 paise (e) None of these

RESPONSE

GRID

1. a b c d e 2. a b c d e 3. a b c d e 4. a b c d e 5. a b c d e

6. a b c d e 7. a b c d e 8. a b c d e 9. a b c d e 10. a b c d e

11. a b c d e 12. a b c d e 13. a b c d e 14. a b c d e

Max. Marks : 30 No. of Qs. 30 Time : 20 min. Date : ........./......../................

8. Given, n = 1 + x, where x is the product of four consecutiveintegers. Then which of the following is true ?A. n is an odd integerB. n is prime.C. n is a perfect square(a) Both A and C are correct.(b) Both A and B are correct.(c) Only A is correct(d) Only C is correct(e) None of these

9. 796 + , when divided by 8, would have a remainder :(a) 0 (b) 6 (c) 5(d) 4 (e) None of these

10. The remainder when 478 is divided by 342 is :(a) 0 (b) 1 (c) 21(d) 340 (e) None of these

11. Students of a class are preparing for a drill and are made tostands in a row. If 4 students are extra in a row, then therewould be 2 rows less. But there would be 4 more rows if 4students are less in a row. The number of students in theclass is :(a) 96 (b) 56 (c) 69(d) 65 (e) None of these

12. Three friends divided some bullets equally. After each of themshot 4 bullets, the total number of bullets remaining is equalto the bullets each had after division . Find the original numberof bullets:(a) 15 (b) 17 (c) 20(d) 18 (e) None of these

13. If x and y are negative, then which of the following statementsis/are always true ?I. x + y is positiveII. xy is positiveIII. x – y is positive(a) I only (b) II only (c) III only(d) I and II only (e) None of these

14. A student was asked to divide a number by 6 and add 12 tothe quotient. He, however, first added 12 to the number andthen divided it by 6, getting 112 as the answer. The correctanswer should have been :(a) 122 (b) 118 (c) 114(d) 124 (e) None of these

133030Number System - II

Page 70: yoursmahboob.wordpress.com SBI · yoursmahboob.wordpress.com iii P 101 Speed Tests for SBI Bank Clerk Exam 101 Speed Tests for SBI Bank Clerk Exam is revised and updated edition on

yoursmahboob.w

ordpress.com

62 SPEED TEST 30

RESPONSE

GRID

15. a b c d e 16. a b c d e 17. a b c d e 18. a b c d e 19. a b c d e

20. a b c d e 21. a b c d e 22. a b c d e 23. a b c d e 24. a b c d e

25. a b c d e 26. a b c d e 27. a b c d e 28. a b c d e 29. a b c d e

30. a b c d e

15. If Harry is 3

1rd the age of his father, George now, and was

4

1th the age of his father 5 years ago, then how old will

George be 5 years from now ?(a) 20 years (b) 45 years (c) 40 years(d) 50 years (e) None of these

16. On Monday, a certain animal shelter housed 55 cats anddogs. By Friday, exactly 1/5 of the cats and 1/4 of the dogshad been adopted; no new cats or dogs were brought to theshelter during this period. What is the greatest possiblenumber of pets that could have been adopted from the animalshelter between Monday and Friday ?(a) 11 (b) 12 (c) 13(d) 14 (e) None of these

17. The number 6n2 + 6n, for any natural number n, is alwaysdivisible by :(a) 6 only (b) 18 only (c) 12 only(d) 6 and 12 (e) None of these

18. A bus starts from city X. The number of women in the bus ishalf of the number of men. In city Y, 10 men leave the bus andfive women enter. Now, the number of men and women isequal. In the beginning, how many passengers entered thebus?(a) 15 (b) 30 (c) 36(d) 45 (e) None of these

19. In a class, there are 18 boys who are over 160 cm tall. If theseconstitute three-fourths of the boys and the total number ofboys is two-thirds of the total number of students in theclass, then what is the number of girls in the class?(a) 6 (b) 12 (c) 18(d) 24 (e) None of these

20. In a family, a couple has a son and a daughter. The age of thefather is three times of his daughter and the age of the son ishalf of his mother. The wife is nine years younger to herhusband and the brother is seven years older than his sister.What is the age of the mother?(a) 40 years (b) 50 years (c) 45 years(d) 60 years (e) None of these

21. Soma purchases National Savings Certificates every yearwhose value exceeds the previous year’s purchase by Rs.400. After 8 years, she finds that she has purchased certificateswhose total face value is Rs. 48,000. What is the face value ofthe certificates purchased by her in the first year ?(a) Rs. 4,300 (b) Rs. 4,500 (c) Rs. 4,400(d) Rs. 4,600 (e) None of these

22. The letters L, M, N, O, P, Q, R, S and T in their order aresubstituted by nine integers 1 to 9 but not in that order. 4 isassigned to P. The difference between P and T is 5. Thedifference between N and T is 3. What is the integerassigned to N?(a) 4 (b) 5 (c) 6(d) 7 (e) None of these

23. A number when divided by 765 leaves a remainder 42. Whatwill be the remainder if the number is divided by 17 ?(a) 8 (b) 7 (c) 6(d) 5 (e) None of these

24. If m and n are natural numbers such that 2m – 2n = 960, whatis the value of m ?(a) 10 (b) 12 (c) 16(d) Cannot be determined(e) None of these

25. At the first stop on his route, a driver uploaded 2/5 of thepackages in his van. After he uploaded another threepackages at his next stop, 1/2 of the original number ofpackages remained. How many packages were in the vanbefore the first delivery?(a) 25 (b) 10 (c) 30(d) 36 (e) None of these

26. A divisor is 2

3of the dividend and 2 times the remainder If

the remainder is 75, then find the dividend.(a) 85 (b) 145 (c) 225(d) 65 (e) None of these

27. The quotient arising from the division of 24162 by a certainnumber x is 89 and the remainder is 43. Find x.(a) 271 (b) 281 (c) 261(d) 251 (e) None of these

28. A boy multiplied a certain number x by 13. He found that theresulting product consisted of all nines entirely. Find thesmallest value of x.(a) 76913 (b) 76933 (c) 76923(d) 75933 (e) None of these

29. Find the unit’s digit in the product (2467)153 × (341)72.(a) 6 (b) 7 (c) 8(d) 9 (e) None of these

30. What is the digit in the unit place of 251 ?(a) 2 (b) 8 (c) 1(d) 4 (e) None of these

Page 71: yoursmahboob.wordpress.com SBI · yoursmahboob.wordpress.com iii P 101 Speed Tests for SBI Bank Clerk Exam 101 Speed Tests for SBI Bank Clerk Exam is revised and updated edition on

yoursmahboob.w

ordpress.com

1. If the L.C.M and H.C.F. of two numbers are 2400 and 16,one number is 480; find the second number.(a) 40 (b) 80 (c) 60(d) 50 (e) None of these

2. The L.C.M. of two number is 630 and their H.C.F. is 9. If thesum of numbers is 153, their difference is(a) 17 (b) 23 (c) 27(d) 33 (e) None of these

3. The H.C.F and L.C.M of two numbers are 21 and 4641respectively. If one of the numbers lies between 200 and300, then the two numbers are(a) 273, 357 (b) 273, 361 (c) 273, 359(d) 273, 363 (e) None of these

4. Suppose you have 108 green marbles and 144 red marbles.You decide to separate them into packages of equal numberof marbles. Find the maximum possible number of marblesin each package.(a) 4 (b) 36 (c) 9(d) 12 (e) None of these

5. One pendulum ticks 57 times in 58 seconds and another608 times in 609 seconds. If they started simultaneously,find the time after which they will tick together.

(a)211

19s (b)

1217

19s (c)

1218

19s

(d)1018

19s (e) None of these

6. From 3 drums of milk, 271, 331 and 451 respectively are tobe drawn out. To do it in a minimum time, the capacity ofthe measuring can be(a) 271 (b) 61 (c) 111(d) 31 (e) None of these

7. Product of two co-prime numbers is 117. Their L.C.M. shouldbe:(a) 1 (b) 117(c) equal to their H.C.F.(d) cannot be calculated(e) None of these

RESPONSE

GRID

1. a b c d e 2. a b c d e 3. a b c d e 4. a b c d e 5. a b c d e

6. a b c d e 7. a b c d e 8. a b c d e 9. a b c d e 10. a b c d e

11. a b c d e 12. a b c d e 13. a b c d e 14. a b c d e 15. a b c d e

Max. Marks : 30 No. of Qs. 30 Time : 20 min. Date : ........./......../................8. Four runners started running simultaneously from a point on

a circular track they took 200 sec, 300 sec, 360 sec and 450 secto complete one round, after how much time do they meet atthe starting point for the first time?(a) 1800 sec (b) 3600 sec (c) 2400 sec(d) 4800 sec (e) None of these

9. Philip, Tom and Brad start jogging around a circular field andcomplete a single round in 18, 22 and 30 seconds respectively,In how much time, will they meet again at the starting point ?(a) 3 min 15 sec (b) 21 min(c) 16 min 30 sec (d) 12 min(e) None of these

10. The numbers 11284 and 7655, when divided by a certainnumber of three digits, leave the same remainder. Find thatnumber of three digits.(a) 161 (b) 171 (c) 181(d) 191 (e) None of these

11. The LCM of two numbers is 2079 and their HCF is 27. if one ofthe numbers is189, find the other.(a) 273 (b) 279 (c) 297(d) 307 (e) None of these

12. Find the least number which, when divided by 18, 24, 30 and42, will leave in each case the same remainder 1.(a) 2521 (b) 2556 (c) 2456(d) 2473 (e) None of these

13. The HCF and LCM of two numbers are 44 and 264 respectively.If the first number is devided by 2, the quotient is 44. What isthe other number?(a) 108 (b) 44 (c) 124(d) 132 (e) None of these

14. The product of two number si 2160 and their HCF is 12. Findthe possible pairs of numbers.(a) 1 (b) 2 (c) 3(d) 4 (e) None of these

15. Three bells toll at intervals of 9, 12 and 15 minutes respectively.All the three begin to toll at 8 a.m. At what time will they tolltogether again?(a) 8.45 a.m. (b) 10. 30 a.m. (c) 11.00 a.m.(d) 1. 30 p.m. (e) None of these

31133131LCM, HCF andFraction

Page 72: yoursmahboob.wordpress.com SBI · yoursmahboob.wordpress.com iii P 101 Speed Tests for SBI Bank Clerk Exam 101 Speed Tests for SBI Bank Clerk Exam is revised and updated edition on

yoursmahboob.w

ordpress.com

64 SPEED TEST 31

RESPONSE

GRID

16. a b c d e 17. a b c d e 18. a b c d e 19. a b c d e 20. a b c d e

21. a b c d e 22. a b c d e 23. a b c d e 24. a b c d e 25. a b c d e

26. a b c d e 27. a b c d e 28. a b c d e 29. a b c d e 30. a b c d e

16. Four metal rods of lengths 78 cm, 104 cm, 117 cm and 169 cmare to be cut into parts of equal length. Each part must be aslong as possible. What is the maximum number of piecesthat can be cut?(a) 27 (b) 36 (c) 43(d) 400 (e) 402

17. The LCM and HCF of two numbers are 84 and 21, respectively.If the ratio of two numbers be 1 : 4, then the larger of the twonumbers is :(a) 21 (b) 48 (c) 84(d) 108 (e) None of these

18. Which of the following pair of fractions adds up to a numbergreater than 5 ?

(a)6

11,

5

13(b)

3

8,

4

11(c)

5

11,

3

7

(d)4

3,

3

5(e) None of these

19. Which of the following fractions is the least ?

(a)119

12(b)

10

1(c)

39

4

(d)69

7(e) None of these

20. Which of the following is not the reciprocal of 4

3

2÷øö

çèæ

?

(a)4

2

3÷øö

çèæ

(b)4

3

2-

÷øö

çèæ

(c)4

2

3 -

÷øö

çèæ

(d)4

4

2

3(e) None of these

21. A certain type of wooden board is sold only in lengths ofmultiples of 25 cm from 2 to 10 metres. A carpenter needs alarge quantity of this type of boards in 1.65 meter length. Forthe minimum waste, the lengths to be purchased should be:(a) 3.30 m (b) 6.60 m (c) 8.25 m(d) 9.95 m (e) None of these

22. Which of the following group of fractions is arranged inascending order ?

(a)17

6,

18

7,

16

5(b)

16

5,

17

6,

18

7

(c)18

7,

17

6,

16

5(d)

16

5,

18

7,

17

6

(e) None of these23. What is the least fraction which, when added to or subtracted

from 29 15

12 16+ , will make the result a whole number ?

(a)38

21 (b)38

31(c)

48

31

(d)48

17(e) None of these

24. If the fractions 7

5,

11

8,

3

2,

13

9are arranged in ascending

order, then the correct sequence is :

(a)7

5,

11

8,

3

2,

13

9(b)

11

8,

7

5,

13

9,

3

2

(c)13

9,

7

5,

11

8,

3

2(d)

13

9,

3

2,

11

8,

7

5

(e) None of these25. If the numerator and the denominator of a proper fraction are

increased by the same quantity, then the resulting fraction is:(a) always greater than the original fraction(b) always less than the original fraction(c) always equal to the original fraction(d) Can’t be determined(e) None of these

26. If x is a positive number, then which of the following fractionhas the greatest value ?

(a)x

x(b)

x

)1x( +(c)

)1x(

x

+

(d))3x(

)2x(

++

(e) None of these

27. Four bells begin to toll together and toll respectively at in-tervals of 6, 5, 7, 10 and 12 seconds. How many times theywill toll together in one hour excluding the one at the start ?(a) 7 times (b) 8 times (c) 9 times(d) 11 times (e) None of these

28. HCF of 3240, 3600 and a third number is 36 and their LCM is2254 7532 ´´´ . The third number is :

(a) 234 752 ´´ (b) 252 732 ´´

(c) 253 732 ´´ (d) 5 2 22 5 7´ ´(e) None of these

29. The LCM of two numbers is 4800 and their HCF is 160. If oneof the numbers is 480, then the other number is :(a) 16 (b) 16000 (c) 160(d) 1600 (e) None of these

30. The traffic lights at three different road crossings changeafter every 48 seconds, 72 seconds and 108 secondsrespectively. If they all change simultaneously at 8 : 20 hours,then at what time will they again change simultaneously ?(a) 8 : 20 : 08 hrs (b) 8 : 24 : 10 hrs(c) 8 : 27 : 12 hrs (d) 8 : 30 : 15 hrs(e) None of these

Page 73: yoursmahboob.wordpress.com SBI · yoursmahboob.wordpress.com iii P 101 Speed Tests for SBI Bank Clerk Exam 101 Speed Tests for SBI Bank Clerk Exam is revised and updated edition on

yoursmahboob.w

ordpress.com

DIRECTIONS (Qs. 1-25): What should come in place of thequestion mark (?) in the following questions?

1. 1190 7225 ? 3094¸ ´ =(a) 221 (b) 121 (c) 214(d) 241 (e) None of these

2. ( )25 1 ? 2 5- = -

(a) 6 (b) 6 2 5+ (c) 6 5

(d) 6 2 5- (e) None of these

3. ( )? 25

11 3 1836=

´ -

(a) 2 (b) 4 (c) 5(d) 6 (e) None of these

4. 12 145 6 34 ?´ ¸ + =(a) – 18 (b) (324)2 (c) 18

(d) 18 (e) None of these

5. ( ) ( ) ( )2 2 213 5 676 7 ?- - + =

(a) 10 (b) 20 (c) 5(d) (25)2 (e) 5

6. ( ) ( )2 2316 5 169 ?- + =

(a) (12)2 (b) 144 (c) 12(d) (144)2 (e) None of these

7. ( )2225 2304 ? 12+ = -

(a) 205 (b) 207 (c) 206(d) 208 (e) None of these

8. 450 890 685 ?+ + =(a) 43 (b) 45 (c) 55(d) 53 (e) None of these

9. 3 6859 ? 4= -(a) 26 (b) 25 (c) 23(d) 22 (e) None of these

RESPONSE

GRID

1. a b c d e 2. a b c d e 3. a b c d e 4. a b c d e 5. a b c d e

6. a b c d e 7. a b c d e 8. a b c d e 9. a b c d e 10. a b c d e

11. a b c d e 12. a b c d e 13. a b c d e 14. a b c d e 15. a b c d e

16. a b c d e 17. a b c d e 18. a b c d e

Max. Marks : 40 No. of Qs. 40 Time : 25 min. Date : ........./......../................

10. 7 447 21 73 26 ?´ ¸ + - =(a) 196 (b) 16 (c) 13(d) 169 (e) None of these

11.? 15 4 40

225

´ -=

(a) 20 (b) 45 (c) 25(d) 50 (e) None of these

12. ( )2656 164 ?¸ =

(a) 14 (b) 16 (c) 64(d) 256 (e) None of these

13. ( )2255 17 5 ?¸ ¸ =

(a) 9 (b) 3 (c) 3

(d) 27 (e) 8114. (32 × 42 × 5) 36 = (?)2 – 80

(a) (100)2 (b) 10 (c) 100

(d) 10 (e) None of these

15. ( )26 1 ? 2 6+ = +

(a) 7 (b) 6 (c) 4 6 7+

(d) 4 6 (e) None of these

16. 12 184 23 26 73 ?´ ¸ + - =

(a) 7 (b) ( )27 (c) 7

(d) (49)2 (e) None of these

17. ( ) ( ) ( )2 3 213 4 676 2 ?- - + =

(a) 3 (b) 9 (c) 81(d) 27 (e) 18

18. ( ) ( )74 676 42 ? 496´ - ´ =

(a) 1024 (b) 1296 (c) 1156(d) 1089 (e) None of these

3232Square, Cube,Indices & Surds

Page 74: yoursmahboob.wordpress.com SBI · yoursmahboob.wordpress.com iii P 101 Speed Tests for SBI Bank Clerk Exam 101 Speed Tests for SBI Bank Clerk Exam is revised and updated edition on

yoursmahboob.w

ordpress.com

66 SPEED TEST 32

RESPONSE

GRID

19. a b c d e 20. a b c d e 21. a b c d e 22. a b c d e 23. a b c d e

24. a b c d e 25. a b c d e 26. a b c d e 27. a b c d e 28. a b c d e

29. a b c d e 30. a b c d e 31. a b c d e 32. a b c d e 33. a b c d e

34. a b c d e 35. a b c d e 36. a b c d e 37. a b c d e 38. a b c d e

39. a b c d e 40. a b c d e

19. 97344 ?=(a) 302 (b) 322 (c) 292(d) 342 (e) None of these

20. ( ) ( ) ( )2 22 392 21 8 7 ?- + - =

(a) 4 (b) – 4 (c) 12(d) 2 (e) 6

21. 7365 + (5.4)2 + ? = 7437.16

(a) 1894 (b) 1681 (c) 1764(d) 2025 (e) None of these

22. ( )3 ? 756 67 804= ´ ¸

(a) 195112 (b) 250047 (c) 226981(d) 274625 (e) None of these

23. 17 51 152 289 ?+ + + =(a) 3 (b) 5 (c) 8(d) 11 (e) None of these

24. 217 52 12 ?+ + =(a) 18 (b) 16 (c) 12(d) 15 (e) 10

25. 10201 3136 ?- =(a) 45 (b) 42 (c) 46(d) 49 (e) None of these

DIRECTIONS (Qs. 26-30): What approximate value will come inplace of the question mark (?) in the following questions? (You arenot expected to calculate the exact value.)

26. If 21025 145= , then the value of 210.25 2.1025 ?+ =(a) 0.1595 (b) 1.595 (c) 159.5(d) 15.95 (e) None of these

27. What is the least number to be added to 2000 to make it aperfect square?(a) 25 (b) 64 (c) 36(d) 49 (e) None of these

28. If (22)3 is subtracted from the square of a number, the answerso obtained is 9516. What is the number?(a) 144 (b) 142 (c) 138(d) 136 (e) None of these

29. If the square of a number is subtracted from 4052 and thedifference is multiplied by 15, the answer so obtained is41340. What is the number?(a) 36 (b) 1024 (c) 32(d) 1296 (e) None of these

30. A gardener plants 34969 mango trees in his garden andarranges them so that there are so many rows as there aremango trees in each row. The number of rows is(a) 187 (b) 176 (c) 169(d) 158 (e) None of these

DIRECTIONS (Q. 31-40): What will come in place of questionmark (?) in the following questions?

31. 93 × 812 273 = (3)?

(a) 3 (b) 4 (c) 5(d) 6 (e) None of these

32. ( ) ( ) ( ) ( )( )3 5 2 ?9 81 27 3´ ¸ =

(a) 5 (b) 4 (c) 7(d) 6 (e) None of these

33. 81.1 × 42.7 × 23.3 = 2?

(a) 7.1 (b) 14 (c) 0.5(d) 9 (e) None of these

34. (49)3 ¸ (7)2 = ?(a) 2401 (b) 49 (c) 343(d) 7 (e) None of these

35. 643.1 × 84.3 = 8?

(a) 10.5 (b) 7.4 (c) 1.2(d) 13.3 (e) None of these

36. 87 × 26 ¸ 82.4 = 8?

(a) 10.6 (b) 9.6 (c) 8.6(d) 6.6 (e) None of these

37. (31)31 × (31)– 27 = ?(a) (961)2 (b) 4 (c) (31)2

(d) 29791 (e) None of these

38.( ){ }( ){ }

22

22

12?

12

-

- =

(a) 12 (b) 4.8 (c)12

144(d) 144 (e) None of these

39. 64 × 362 216 = 6(?)

(a) 3 (b) 4 (c) 5(d) 6 (e) None of these

40. ( ) ( ) ( )6 3 ?48 64 8 8´ ¸ =

(a) 3 (b) 4 (c) 5(d) 7 (e) None of these

Page 75: yoursmahboob.wordpress.com SBI · yoursmahboob.wordpress.com iii P 101 Speed Tests for SBI Bank Clerk Exam 101 Speed Tests for SBI Bank Clerk Exam is revised and updated edition on

yoursmahboob.w

ordpress.com

DIRECTIONS (Qs. 1 to 12) : What will come in place of qustionsmark (?) in the following questions?

1. 19.99 × 9.9 + 99.9 = ?(a) 129. 79 (b) 297. 801(c) 1009 (d) 296.91(e) None of these

2. 456.675 + 35.7683 × 67.909 – 58.876 = ?(a) 33382 (b) 3587(c) 1540 (d) 2756(e) 2830

3. {(52)2 + (45)2} ? = 8(a) 611.345 (b) 487.225(c) 591.125 (d) 372.425(e) None of these

4. (12.25)2 – 625 = ?

(a) 235.1625 (b) 125.0625

(c) 375.2625 (d) 465.3625

(e) None of these

5. 572 + 38 × 0.50 – 16 = ?

(a) 289 (b) 305

(c) 448 (d) 565

(e) None of these

6.3 2 11

? 10567 5 13´ ´ ´ =

(a) 7280 (b) 7296

(c) 7308 (d) 7316

(e) 7324

7. 6138 + 317576 = ? × 9

(a) 676 (b) 729

(c) 784 (d) 841

(e) 961

RESPONSE

GRID

1. a b c d e 2. a b c d e 3. a b c d e 4. a b c d e 5. a b c d e

6. a b c d e 7. a b c d e 8. a b c d e 9. a b c d e 10. a b c d e

11. a b c d e 12. a b c d e 13. a b c d e 14. a b c d e

Max. Marks : 30 No. of Qs. 30 Time : 20 min. Date : ........./......../................

8. (78700 ÷ 1748) + (3.79 × 121.24) = ?

(a) 305 (b) 415

(c) 525 (d) 635

(e) 745

9. 3 46080 74000 6560 ?¸ + =(a) 30 (b) 80

(c) 130 (d) 170

(e) 210

10.1 2 3

of of of 1715 ?8 3 5

=

(a) 80 (b) 85

(c) 90 (d) 95

(e) 7511. 561 ÷ 35.05 × 19.99 = ?

(a) 320 (b) 330(c) 315 (d) 325(e) 335

12. (15.01)2 × 730 ?=(a) 6125 (b) 6225(c) 6200 (d) 6075(e) 6250

DIRECTIONS (13-20): What will come in place of the questionmark (?) in the following questions?

13. (3325 25) × (152 16) = ?(a) 1269.4 (b) 1264.9(c) 1265.3 (d) 1263.5(e) None of these

14.1 2 2

5 2 3 ?5 15 3

+ + =

(a) 15 (b) 13

(c)11

15(d) 12

(e) None of these

3333Simplification

Page 76: yoursmahboob.wordpress.com SBI · yoursmahboob.wordpress.com iii P 101 Speed Tests for SBI Bank Clerk Exam 101 Speed Tests for SBI Bank Clerk Exam is revised and updated edition on

yoursmahboob.w

ordpress.com

68 SPEED TEST 33

RESPONSE

GRID

15. a b c d e 16. a b c d e 17. a b c d e 18. a b c d e 19. a b c d e

20. a b c d e 21. a b c d e 22. a b c d e 23. a b c d e 24. a b c d e

25. a b c d e 26. a b c d e 27. a b c d e 28. a b c d e 29. a b c d e

30. a b c d e

15. – 15 – 27– 88 – 63 + 255 = ?(a) 55 (b) 74(c) 62 (d) 59(e) None of these

16. (2525 × 0.25 ¸ 5) × 7 = ?(a) 889.43 (b) 883.75(c) 886.45 (d) 881.75(e) None of these

17.14 57 20

?19 70 21

´ ´ =

(a)2

7(b)

4

7

(c)2

9(d)

3

7(e) None of these

18. 32% of 500 + 162% of 50 = ?(a) 231 (b) 245(c) 237 (d) 247(e) None of these

19. 45316 + 52131 – 65229 = ? + 15151(a) 17063 (b) 17073(c) 17076 (d) 17067(e) None of these

20.184 4

?23 of 400

´=

(a) 7 (b) 9(c) 8 (d) 5(e) None of these

21. What will come in place of both the question marks (?) inthe following question ?

4 / 3

5 / 3

(?) 28

32 ?

1=

(a) 16 (b) 12(c) 18 (d) 14(e) None of these

22. 16% of 450 ÷ ? % of 250 = 4.8

(a) 12 (b) 6(c) 4 (d) 10(e) None of these

23. ? 11 1521- =

(a) 2500 (b) (28)2

(c) 28 (d) 50

(e) None of these

24. 8059 7263 ? 40- = ´

(a) 19.9 (b) 18.7(c) 15.9 (d) 17.7(e) None of these

25. 4 ? 4062 5´ = ¸(a) 203.1 (b) 213.1(c) 205.1 (d) 215.1(e) None of these

26. ( )3.5 80 2.5 ?´ ¸ =

(a) 122 (b) 111(c) 222 (d) 212(e) None of these

27. 13% of 258 – ? = 10(a) 23.45 (b) 24.53(c) 23.54 (d) 24.35(e) None of these

28.4 3 5

2 ?5 4 8´ ¸ =

(a)12

435

(b)12

135

(c)11

235

(d)13

325

(e) None of these29. 623.15 – 218.82 – 321.43 = ?

(a) 89.2 (b) 82.2(c) 89.9 (d) 79.2(e) None of these

30. 5437 – 3153 + 2284 = ? × 50(a) 96.66 (b) 91.36(c) 96.13 (d) 93. 16(e) None of these

Page 77: yoursmahboob.wordpress.com SBI · yoursmahboob.wordpress.com iii P 101 Speed Tests for SBI Bank Clerk Exam 101 Speed Tests for SBI Bank Clerk Exam is revised and updated edition on

yoursmahboob.w

ordpress.com

1. The average of five numbers is 281. The average of the firsttwo numbers is 280 and the average of the last two numbersis 178.5. What is the third number ?(a) 488 (b) 336(c) 228 (d) 464(e) None of these

2. The average age of 3 friends is 32 years. If the age of afourth friend is added, their average age comes to 31 years.What is the age of the fourth friend ?(a) 32 years (b) 28 years(c) 24 years (d) 26 years(e) None of these

3. Find the average of the following set of scores.965, 362, 189, 248, 461, 825, 524, 234(a) 476 (b) 504(c) 461 (d) 524(e) None of these

4. If the value of 21a + 21b=1134, what is the average of a+b?(a) 29 (b) 27(c) 58 (d) 54(e) None of these

5. Out of three given numbers, the first number is twice thesecond and thrice the third. If the average of the three num-bers is 154, what is the difference between the first and thethird number?(a) 126 (b) 42(c) 166 (d) 52(e) None of these

6. Average weight of three boys P, T and R is 1

543

kgs while

the average weight of three boys, T, F and G is 53 kgs. Whatis the average weight of P, T, R, F and G?(a) 53.8kgs (b) 52.4kgs(c) 53.2kgs (d) Cannot be determined(e) None of these

7. Find the average of the following set of scores221, 231, 441, 359, 665, 525(a) 399 (b) 428

(c) 407 (d) 415

(e) None of these

RESPONSE

GRID

1. a b c d e 2. a b c d e 3. a b c d e 4. a b c d e 5. a b c d e

6. a b c d e 7. a b c d e 8. a b c d e 9. a b c d e 10. a b c d e

11. a b c d e 12. a b c d e 13. a b c d e 14. a b c d e

Max. Marks : 30 No. of Qs. 30 Time : 20 min. Date : ........./......../................8. The average of 5 consecutive numbers A, B, C, D and E is 48.

What is the product of A and E?(a) 2162 (b) 2208(c) 2024 (d) 2800(e) None of these

9. If the value of 16a + 16b = 672, what is the average of a and b?(a) 44 (b) 21(c) 24 (d) 42(e) None of these

10. The sum of five numbers is 290. The average of the first twonumbers is 48.5 and the average of last two numbers is 53.5.What is the third number?(a) 72 (b) 84(c) 96 (d) 108(e) None of these

11. The average age of the family of five members is 24. If thepresent age of youngest member is 8 yr, then what was theaverage age of the family at the time of the birth of the youngestmember ?(a) 20 yr (b) 16 yr (c) 12 yr(d) 18 yr (e) 21 yr

12. The average of four consecutive odd numbers is 36. What isthe smallest of these numbers ?(a) 31 (b) 35 (c) 43(d) 47 (e) None of these

13. Average score of Rahul, Manish and Suresh is 63. Rahul’sscore is 15 less than Ajay and 10 more than Manish. If Ajayscored 30 marks more than the average scores of Rahul,Manish and Suresh, what is the sum of Manish’s and Suresh’sscores ?(a) 120 (b) 111(c) 117 (d) Cannot be determined(e) None of these

14. The total marks obtained by a student in physics, Chemistryand Mathematics together is 120 more than the marks obtainedby him in Chemistry. What are the average marks obtained byhim in Physics and Mathematics together ?(a) 60 (b) 120(c) 40 (d) Cannot be determined(e) None of these

3434Average

Page 78: yoursmahboob.wordpress.com SBI · yoursmahboob.wordpress.com iii P 101 Speed Tests for SBI Bank Clerk Exam 101 Speed Tests for SBI Bank Clerk Exam is revised and updated edition on

yoursmahboob.w

ordpress.com

70 SPEED TEST 34

RESPONSE

GRID

15. a b c d e 16. a b c d e 17. a b c d e 18. a b c d e 19. a b c d e

20. a b c d e 21. a b c d e 22. a b c d e 23. a b c d e 24. a b c d e

25. a b c d e 26. a b c d e 27. a b c d e 28. a b c d e 29. a b c d e

30. a b c d e

15. A man drives to his office at 60 km/hr and returns homealong the same route 30 km/hr. Find the average speed.(a) 50 km/hr (b) 45 km/hr (c) 40 km/hr(d) 55 km/hr (e) None of these

16. A cricketer has completed 10 innings and his average is 21.5runs. How many runs must he make in his next innings so asto raise his average to 24?(a) 69 (b) 59 (c) 49(d) 39 (e) None of these

17. One-third of a certain journey is covered at th rate of 25 km /hr, one-fourth at the rate of 30 km / hr and the rest at 50 km /hr. Find the average speed for the whole journey.

(a)1

33 km / hr3

(b)1

44 km / hr4

(c) 1

33 km/hr5

(d)1

44 km / hr5

(e) None of these

18. The average salary of all the workers in a workshop is` 8,000. The average salary of 7 technicians is `12,000 andthe average salary of the rest is Rs 6,000. The total numberof workers in the workshop is :(a) 21 (b) 20 (c) 23(d) 22 (e) None of these

19. A batsman makes a scores of 98 runs in his 19th inning andthus increases his average by 4. What is his average after19th inning ?(a) 22 (b) 24 (c) 28(d) 26 (e) None of these

20. The average weight of 45 students in a class is 52 kg. 5 ofthem whose average weight is 48 kg leave the class andother 5 students whose average weight is 54 kg join theclass. What is the new average weight (in kg) of the class ?

(a) 52.6 (b)3

252 (c)

3

152

(d) 62.5 (e) None of these21. The batting average of 40 innings of a cricket player is 50

runs. His highest score exceeds his lowest score by 172runs. If these two innings are excluded, the average of theremaining 38 innings is 48. His highest score was :(a) 172 (b) 173 (c) 174(d) 176 (e) None of these

22. A car owner buys petrol at Rs 7.50, Rs 8.00 and Rs 8.50 perlitre for three successive years. What approximately is hisaverage cost per litre of petrol if he spends Rs 4000 eachyear ?(a) Rs 8 (b) Rs 9 (c) Rs 7.98(d) Rs 8.50 (e) None of these

23. The average of three numbers is 135. The largest number is180 and the difference between the others is 25. The small-est number is :(a) 130 (b) 125 (c) 120(d) 100 (e) None of these

24. There are 30 student in a class. The average age of the first10 students is 12.5 years. The average age of the next 20students is 13.1 years. The average age of the whole classis:(a) 12.5 years (b) 12.7 years (c) 12.8 years(d) 12.9 years (e) None of these

25. The weight in kilograms of 10 students are 52, 45, 31, 35, 40,55, 60, 38, 44, 36. If 44 is replaced by 46 and 40 is replaced by35, then the new median will be :(a) 42 (b) 40.5 (c) 40(d) 41.5 (e) None of these

26. The average (arithmetic mean) of x and y is 40. If z = 10, thenwhat is the average of x, y and z ?

(a)3

216 (b) 30 (c) 25

(d)2

117 (e) None of these

27. The average of 11 numbers is 10.9. If the average of the firstsix numbers is 10.5 and that of the last six numbers is 11.4,then the middle number is :(a) 11.5 (b) 11.4 (c) 11.3(d) 11.0 (e) None of these

28. The average of 10 numbers is 40.2. Later it is found that twonumbers have been wrongly copied. The first is 18 greaterthan the actual number and the second number added is 13instead of 31. Find the correct average.(a) 40.2 (b) 40.4 (c) 40.6(d) 40.8 (e) None of these

29. Last year, a Home Appliance Store sold an average (arithmeticmean) of 42 microwave ovens per month. In the first 10months of this year, the store has sold an average (arithmeticmean) of only 20 microwave ovens per month. What was theaverage number of microwave ovens sold per month duringthe entire 22 months period ?(a) 21 (b) 30 (c) 31(d) 32 (e) None of these

30. The average temperature for the first four days of a week is40.2°C and that of the last four days is 41.3°C. If the averagetemperature for the whole week is 40.6°C, then the tempera-ture on the fourth day is :(a) 40.8°C (b) 38.5°C (c) 41.3°C(d) 41.8°C (e) None of these

Page 79: yoursmahboob.wordpress.com SBI · yoursmahboob.wordpress.com iii P 101 Speed Tests for SBI Bank Clerk Exam 101 Speed Tests for SBI Bank Clerk Exam is revised and updated edition on

yoursmahboob.w

ordpress.com

DIRECTIONS (Qs. 1-18): What should come in place of thequestion mark (?) in the following questions?

1. 76% of 1285 = 35% of 1256 + ?

(a) 543 (b) 537 (c) 547(d) 533 (e) None of these

2. (21.5% of 999)1/3 + (43% of 601)1/2 = ?

(a) 18 (b) 22 (c) 26(d) 30 (e) 33

3. 64.5% of 800 + 36.4% of 1500 = (?)2 + 38

(a) 32 (b) 38 (c) 42(d) 48 (e) 34

4. 41% of 601 – 250.17 = ? – 77% of 910

(a) 800 (b) 500 (c) 700(d) 650 (e) 550

5. 40.005% of 439.998 + ?% of 655.011 = 228.5

(a) 8 (b) 17 (c) 12(d) 20 (e) 5

6. 25% of 84 × 24% of 85 = ?

(a) 424.2 (b) 488.4 (c) 482.8(d) 428.4 (e) None of these

7. 20.06% of 599 + 10.01% of 901 = ?

(a) 150 (b) 210 (c) 250(d) 280 (e) 300

8. 14.2% of 5500 + 15.6% of ? = 1795

(a) 6500 (b) 6200 (c) 5600(d) 5800 (e) None of these

RESPONSE

GRID

1. a b c d e 2. a b c d e 3. a b c d e 4. a b c d e 5. a b c d e

6. a b c d e 7. a b c d e 8. a b c d e 9. a b c d e 10. a b c d e

11. a b c d e 12. a b c d e 13. a b c d e 14. a b c d e 15. a b c d e

Max. Marks : 30 No. of Qs. 30 Time : 20 min. Date : ........./......../................

9. 36% of 245 – 40% of 210 = 10 – ?

(a) 4.2 (b) 6.8 (c) 4.9(d) 5.6 (e) None of these

10.1

2 of 3842 + 15% of ? = 24499

(a) 3520 (b) 3250 (c) 3350(d) 3540 (e) None of these

11. 57% of 394 – 2.5% of 996 = ?

(a) 215 (b) 175 (c) 200(d) 180 (e) 205

12. 40% of 265 + 35% of 180 = 50% of ?

(a) 338 (b) 84.5 (c) 253.5(d) 169 (e) None of these

13.1 1

4 3 ?5 3´ + = 20% of 120

(a)1

1015

(b) 10 (c) 5

(d) 15 (e) None of these

14. 14% of 250 × ? % of 150 = 840

(a) 15 (b) 18 (c) 16(d) 12 (e) None of these

15. 18% of 609 + 27.5% of 450 = ?

(a) 220 (b) 233 (c) 267(d) 248 (e) 274

3535Percentage

Page 80: yoursmahboob.wordpress.com SBI · yoursmahboob.wordpress.com iii P 101 Speed Tests for SBI Bank Clerk Exam 101 Speed Tests for SBI Bank Clerk Exam is revised and updated edition on

yoursmahboob.w

ordpress.com

72 SPEED TEST 35

RESPONSE

GRID

16. a b c d e 17. a b c d e 18. a b c d e 19. a b c d e 20. a b c d e

21. a b c d e 22. a b c d e 23. a b c d e 24. a b c d e 25. a b c d e

26. a b c d e 27. a b c d e 28. a b c d e 29. a b c d e 30. a b c d e

16. 125% of 3060 – 85% of ? = 408(a) 3890 (b) 3940 (c) 4020(d) 4015 (e) None of these

17. If x% of 500 = y% of 300 and x% of y% of 200 = 60, then x = ?

(a) 10 2 (b) 20 2 (c) 15 2

(d) 30 2 (e) None of these

18. 185% of 400 + 35% of 240 = ? % of 1648(a) 85 (b) 75 (c) 125(d) 50 (e) None of these

19. What is 25% of 50% of 2

rd3

of 630 ?

(a) 36.5 (b) 52.5 (c) 45.5(d) 68.5 (e) None of these

20. There are 1225 employees in an organization, out of which40% got transferred to different places. How many suchemployees got transferred ?(a) 540 (b) 490(c) 630 (d) 710(e) None of these

21. If the numerator of a fraction is increased by 500% and thedenominator is increased by 300%, the resultant fraction

is4

27

. What was the original fraction?

(a)47

(b)127

(c)154

(d)65

(e) None of these

22. If the numerator of a fraction is increased by 250% and the

denominator is increased by 300%, the resultant fraction is7

9.

What is the original fraction ?

(a)8

11(b)

7

8

(c)8

9(d)

7

11

(e) None of these23. In an examination it is required to get 40% of the aggregate

marks to pass. A student gets 261 marks and is declared failedby 4% marks. What are the maximum aggregate marks astudent can get?

(a) 700 (b) 730(c) 745 (d) 765(e) None of these

24. The population of a town is 198000. It increases by 7% in the1st year and decreases by 5% in the 2nd year. What is thepopulation of the town at the end of 2 years?

(a) 211860 (b) 201267(c) 222453 (d) 198900

(e) None of these

25. Bovina spent ` 44,668 on her air tickets, 56,732 on buyinggifts for the family members and the remaining 22% of thetotal amount she had as cash with her. What was the totalamount?(a) ` 28]600 (b) ` 1]30]000(c) ` 1]01]400 (d) ` 33]800(e) None of these

26. Rubina decided to donate 16% of her monthly salary to anNGO. On the day of donation she changed her mind anddonated ` 6,567 which was 75% of what she had decidedearlier. How much is Rubina’s monthly salary?(a) ` 8]756 (b) ` 54]725(c) ` 6]56]700 (d) ` 45]696(e) None of these

27. In a college election between two candidates, one candidategot 55% of the total valid votes. 15% of the votes wereinvalid. If the total votes were 15,200, what is the number ofvalid votes the other candidate got ?(a) 7106 (b) 6840(c) 8360 (d) 5814(e) None of these

28. If the radius of a circle is diminished by 10%, the area isdiminished by(a) 36% (b) 20% (c) 19% (d) 10%(e) None of these

29. If A’s salary is 25% higher than B’s salary, then how muchper cent is B’s salary lower than A’s ?

(a) %3

116 (b) 20% (c) 25% (d)

3

133 %

(e) None of these30. A reduction of 20% in the price of an apple enable a man to

buy 10 apple more for ` 54. The reduced price of apple perdozen is(a) ` 4.32 (b) ` 12.96(c) ` 10.80 (d) ` 14.40(e) None of these

Page 81: yoursmahboob.wordpress.com SBI · yoursmahboob.wordpress.com iii P 101 Speed Tests for SBI Bank Clerk Exam 101 Speed Tests for SBI Bank Clerk Exam is revised and updated edition on

yoursmahboob.w

ordpress.com

1. The total number of students in a school is 819. If the numberof girls in the school is 364, then what is the respective ratioof the total number of boys to the total number of girls in theschool ?(a) 26 : 25 (b) 21 : 17(c) 18 : 13 (d) 5 : 4(e) None of these

2. If a dividend of ` 57,834 is to be divided among Meena,Urmila and Vaishali in the proportion of 3:2:1, find Urmila'sshare.(a) ` 19,281 (b) ` 17,350(c) ` 23,133 (d) ` 19,278(e) None of these

3. The ratio of the ages of Richa and Shelly is 5 : 8. The ratio oftheir ages 10 years hence would be 7:10. What is the presentage of Shelly?(a) 45years (b) 40years(c) 35years (d) 30years(e) 25years

4. A sum of money is to be divided equally amongst A, B, andC in the respective ratio of 3:4:5 and another sum of moneyis to be divided between E and F equally. If F got 1050 lessthan A, how much amount did B receive?(a) ` 750 (b) ` 2000(c) ` 1500 (d) Cannot be determined(e) None of these

5. The average age of a woman and her daughter is 42 years.The ratio of their ages is 2:1. What is the daughter's age?(a) 28 years (b) 48 years(c) 52 years (d) 32 years(e) None of these

6. The difference between the present ages of Arun and Deepakis 14 years. Seven years ago the ratio of their ages was 5 : 7respectively. What is Deepak’s present age?(a) 49 years (b) 42 years(c) 63 years (d) 35 years(e) None of these

7. Ratio of Rani’s and Komal’s age is 3 : 5. Ratio of Komal’s andPooja’s age is 2 : 3. If Rani is two-fifth Pooja’s age, what isRani’s age?

RESPONSE

GRID

1. a b c d e 2. a b c d e 3. a b c d e 4. a b c d e 5. a b c d e

6. a b c d e 7. a b c d e 8. a b c d e 9. a b c d e 10. a b c d e

11. a b c d e 12. a b c d e 13. a b c d e

Max. Marks : 30 No. of Qs. 30 Time : 20 min. Date : ........./......../................

(a) 10 years (b) 15 years(c) 24 years (d) Cannot be determined(e) None of these

8. In a college the students in Arts and Commerce faculties werein the ratio of 4 : 5 respectively. When 65 more students joinedCommerce faculty the ratio became 8 : 11. How many studentsare there in Arts faculty?(a) 520 (b) 650(c) 715 (d) Cannot be determined(e) None of these

9- The ratio between the boys and girls in a class is 6 : 5. If 8 moreboys join the class and two girls leave the class then the ratiobecomes 11 : 7. What is the number of boys in the class now?(a) 28 (b) 38(c) 44 (d) 36(e) None of these

10. At present Meena is eight times her daughter's age. Eightyears from now, the ratio of the ages of Meena and herdaughter will be 10 : 3. What is Meena's present age ?(a) 32 years (b) 40 years(c) 36 years (d) Cannot be determined(e) None of these

11. The ratio of the ages of a father and son is 17 : 7. 6 years agothe ratio of their ages was 3 : 1 . What is the father’s presentage?(a) 64 (b) 51(c) 48 (d) Cannot be determined(e) None of these

12. The ratio of the money with Rita an Sita is 7 : 15 and that withSita and Kavita is 7 : 16. If Rita has 490, how much moneydoes Kavita have?(a) 1050 (b) 2200 (c) 2400(d) 2800 (e) None of these

13. In two alloys, the ratio of iron and copper is 4 : 3 and 6 : 1,respectively. If 14 kg of the first alloy and 42 kg of the secondalloy is mixed together to form a new alloy, then what will bethe ratio of iron to copper in the new alloy ?(a) 11 : 3 (b) 11 : 8 (c) 8 : 1(d) 3 : 11 (e) None of these

3636Ratio & Proportion

Page 82: yoursmahboob.wordpress.com SBI · yoursmahboob.wordpress.com iii P 101 Speed Tests for SBI Bank Clerk Exam 101 Speed Tests for SBI Bank Clerk Exam is revised and updated edition on

yoursmahboob.w

ordpress.com

74 SPEED TEST 36

RESPONSE

GRID

14. a b c d e 15. a b c d e 16. a b c d e 17. a b c d e 18. a b c d e

19. a b c d e 20. a b c d e 21. a b c d e 22. a b c d e 23. a b c d e

24. a b c d e 25. a b c d e 26. a b c d e 27. a b c d e 28. a b c d e

29. a b c d e 30. a b c d e

14. The sides of a triangle are in the ratio 1 1 1

: :2 3 4

and its

perimeter is 104 cm. The length of the longest side is:(a) 52 cm (b) 48 cm (c) 32 cm(d) 26 cm (e) None of these

15. The ratio of two numbers is 4 : 7. If each of these numbersincreases by 30, their ratio will become 5 : 8 . What is theaverage of these two numbers?(a) 135 (b) 145 (c) 155(d) 165 (e) 175

16. If ` 782 be divided into three part, proportional to 1 2 3

: :2 3 4

,

then the first part is :(a) ` 182 (b) ` 190 (c) ` 196(d) ` 204 (e) None of these

17. The scale of map is 34

of cm = 1 km. If the distance on the

map between two towns is 60 cm, then the actual distance is(a) 60 km (b) 80 km (c) 75 km(d) 50 km (e) None of these

18. If A : B : C = 2 : 3 : 4, then A B C

: :B C A

is equal to :

(a) 4 : 9 : 16 (b) 8 : 9 : 12 (c) 8 : 9 : 16(d) 8 : 9 : 24 (e) None of these

19. The average age of three boys is 25 years and their ages arein the proportion 3: 5 : 7. The age of the youngest boy is:(a) 21 years (b) 18 years (c) 15 years(d) 9 years (e) None of these

20. The income of A and B are in the ratio 3 : 2 and expenses arein the ratio 5 : 3. If both save 200, what is the income of A?(a) ` 1000 (b) ` 1200 (c) ` 1500(d) ` 1800 (e) None of these

21. ` 750 is distributed among A, B and C such thatA’s share : B’s share = 2 : 3 and B’s share: C’s share = 6 : 5.The share of A is,(a) ` 150 (b) ` 175 (c) ` 200(d) ` 250 (e) None of these

22. Two number are in the ratio 5 : 4 and their difference is 10.What is the larger number ?(a) 30 (b) 40 (c) 50(d) 60 (e) None of these

23. An amount of money is to be distributed among P, Q and Rin the ratio 3 : 1 : 5. The difference between Q’s and R’s shareis Rs 3600. What is the total of P’s and Q’s share ?(a) Rs 5400 (b) Rs 3600 (c) Rs 2700(d) Rs 1800 (e) None of these

24. Mrs. X spends Rs 535 in purchasing some shirts and ties forher husband. If shirts cost Rs 43 each and the ties cost Rs 21each, then what is the ratio of the shirts to the ties, that arepurchased ?(a) 1 : 2 (b) 2 : 1 (c) 2 : 3(d) 3 : 4 (e) None of these

25. If a : b = 2 : 5, then the value of )b5a7(:)b3a2( ++ is :

(a)39

19 (b)13

99(c)

19

31

(d)31

19(e) None of these

26. If two numbers are in the ratio 6 : 13 and their least commonmultiple is 312, then the sum of the numbers is :(a) 75 (b) 57 (c) 76(d) 67 (e) None of these

27. If 5

23

b3a2

b3a5=

-+

, then the value of a : b is :

(a) 2 : 1 (b) 1 : 4 (c) 1 : 2(d) 4 : 1 (e) None of these

28. A bag contains Rs 216 in the form of one rupee, 50 paise and25 paise coins in the ratio of 2 : 3 : 4. The number of 50 paisecoins is :(a) 96 (b) 144 (c) 114(d) 141 (e) None of these

29. The ages of two persons are in the ratio of 5 : 7. Sixteen yearsago, their ages were in the ratio of 3: 5. Their present agesare:(a) 30 years and 44 years(b) 35 years and 52 years(c) 40 years and 56 years(d) 45 years and 60 years(e) None of these

30. In a mixture of 28 litres, the ratio of milk and water is 5 : 2.Another 2 litres of water is added to the mixture. The ratio ofmilk and water in the new mixture is:(a) 1: 1 (b) 2: 1 (c) 3: 1(d) 4: 1 (e) None of these

Page 83: yoursmahboob.wordpress.com SBI · yoursmahboob.wordpress.com iii P 101 Speed Tests for SBI Bank Clerk Exam 101 Speed Tests for SBI Bank Clerk Exam is revised and updated edition on

yoursmahboob.w

ordpress.com

1. One litre of water was mixed to 3 litres of sugar. Solutioncontaining 4% of sugar. What is the percentage of sugar inthe solution?(a) 3 (b) 4(c) 6 (d) Insuffficient data(e) None of these

2. A trader mixes 80 kg of tea at ` 15 per kg with 20 kg of tea atcost price of ` 20 per kg. In order to earn a profit of 25%,what should be the sale price of the mixed tea?(a) ` 23.75 (b) ` 22 (c) ` 20(d) ` 19.20 (e) None of these

3. Alcohol cost ` 3.50 per litre and kerosene oil cost 2.50 perlitre. In what proportion these should be mixed so that theresulting mixture may be 2.75 per litre?(a) 2 : 5 (b) 1 : 3 (c) 4 : 7(d) 2 : 3 (e) None of these

4. Pure milk costs 3.60 per litre. A milkman adds water to 25litres of pure milk and sells the mixture at 3 per litre. Howmany litres of water does he add?(a) 2 litres (b) 5 litres (c) 7 litres(d) 11 litres (e) None of these

5. In what ratio must water be mixed with milk to gain 20% byselling the mixture at cost price?(a) 1 : 3 (b) 1 : 5 (c) 1 : 7(d) 1 : 10 (e) None of these

6. A chemist has 10 litres of a solution that is 10 per cent nitricacid by volume. He wants to dilute the solution to 4 percent strength by adding water. How many litres of watermust he add ?(a) 15 (b) 20 (c) 18(d) 25 (e) None of these

7. In what ratio must water be mixed with milk to gain 2

163

%

on selling the mixture at cost price?(a) 1 : 6 (b) 6 : 1 (c) 2 : 3(d) 4 : 3 (e) None of these

RESPONSE

GRID

1. a b c d e 2. a b c d e 3. a b c d e 4. a b c d e 5. a b c d e

6. a b c d e 7. a b c d e 8. a b c d e 9. a b c d e 10. a b c d e

11. a b c d e 12. a b c d e

Max. Marks : 25 No. of Qs. 25 Time : 20 min. Date : ........./......../................

8. An alloy contains copper and zinc in the ratio 5 : 3 and anotheralloy contains copper and tin in the ratio 8 : 5. If equal weightsof both the alloys are melted together, then the weight of tinin the resulting alloy per kg will be:

(a)26

5(b)

26

5(c)

40

7

(d)7

40(e) None of these

9. The ratio of milk and water in 55 litres of adulterated milk is7 : 4. How much water must be added to make the mixture’sratio 7 : 6?(a) 5 lt (b) 10 lt (c) 15 lt(d) 25 lt (e) None of these

10. A and B are two alloys of gold and copper prepared by mixingmetals in the ratio 7 : 2 and 7 : 11 respectively. If equalquantities of the alloys are melted to form a third alloy C, thenthe ratio of gold and copper in alloy C will be(a) 5 : 7 (b) 5 : 9 (c) 7 : 5(d) 9 : 5 (e) None of these

11. Sameer bought 10 kg of tea at 45 per kg and 8 kg at 50 perkg. He mixed both the brands and sold it at a total profit of 32. What was the selling price per kg of the mixture?(a) ` 48 (b) ` 50 (c) ` 49(d) ` 47 (e) None of these

12. How many litres of pure alcohol must be added to 10 litres ofmixture which is 15% alcohol to make a mixture which will be25% alcohol?

(a)54

(b)52

(c)34

(d)4

3(e) None of these

3737Alligation andMixture

Page 84: yoursmahboob.wordpress.com SBI · yoursmahboob.wordpress.com iii P 101 Speed Tests for SBI Bank Clerk Exam 101 Speed Tests for SBI Bank Clerk Exam is revised and updated edition on

yoursmahboob.w

ordpress.com

76 SPEED TEST 37

RESPONSE

GRID

13. a b c d e 14. a b c d e 15. a b c d e 16. a b c d e 17. a b c d e

18. a b c d e 19. a b c d e 20. a b c d e 21. a b c d e 22. a b c d e

23. a b c d e 24. a b c d e 25. a b c d e

13. How many kg of custard powder costing ` 40 kg must bemixed with 16 kg of custard powder costing ` 55 kg so that25% may be gained by selling the mixture at 60 kg?(a) 11 kg (b) 14 kg (c) 12 kg(d) 20 kg (e) None of these

14. 300 gm of sugar solution has 40% sugar in it. How muchsugar should be added to make it 50% in the solution?(a) 40 gm (b) 50 gm (c) 60 gm(d) 70 gm (e) None of these

15. Gold is 19 times as heavy as water and copper 9 times. Inwhat ratio should these metals be mixed so that the mixturemay be 15 times as beavy as water?(a) 1 : 2 (b) 3 : 2 (c) 2 : 3(d) 4 : 5 (e) None of these

16. In a mixture of 60 litres, the ratio of milk to water is 2 : 1. If theratio of milk to water is to be 1 : 2, then amount of water to befurther added is ___________.(a) 20 (b) 40 (c) 60(d) 80 (e) None of these

17. In a mixture of milk and water the proportion of water byweight was 75%. If in 60 gm of mixture 15 gm water wasadded, what would be the percentage of water? (Weight ingm)(a) 75% (b) 88% (c) 90%(d) 100% (e) None of these

18. In what ratio must tea at Rs. 62 per kg be mixed with tea atRs. 72 per kg so that the mixture must be worth Rs. 64.50 perkg?(a) 3 : 1 (b) 3 : 2 (c) 4 : 3(d) 5 : 3 (e) None of these

19. Two vessels A and B contain spirit and water mixed in theratio 5 : 2 and 7 : 6 respectively. Find the ratio in which thesemixture be mixed to obtain a new mixture in vessel C contain-ing spirit and water in the ratio 8 : 5 ?(a) 4 : 3 (b) 3 : 4 (c) 5 : 6(d) 7 : 9 (e) None of these

20. If 50% of a 2 : 3 solution of milk and water is replaced withwater, then the concentration of the solution is reduced by:(a) 25% (b) 33.33% (c) 50%(d) 75% (e) None of these

21. In a mixture of 45 litres, the ratio of milk and water is 3 : 2.How much water must be added to make the ratio 9 : 11?(a) 10 litres (b) 15 litres (c) 17 litres(d) 20 litres (e) None of these

22. Three containers A, B and C are having mixtures of milk andwater in the ratio 1 : 5, 3 : 5 and 5 : 7, respectively. If thecapacities of the containers are in the ratio 5 : 4 : 5, then findthe ratio of the milk to the water if the mixtures of all the threecontainers are mixed together.(a) 51 : 115 (b) 52 : 115 (c) 53 : 115(d) 54 : 115 (e) None of these

23. Five litres of water is added to a certain quantity of pure milkcosting Rs. 3 per litre. If by selling the mixture at the sameprice as before, a profit of 20% is made, then what is theamount of pure milk in the mixture ?(a) 20 litres (b) 30 litres (c) 25 litres(d) 35 litres (e) None of these

24. How many kg of tea worth Rs 25 per kg must be blendedwith 30 kg of tea worth Rs 30 per kg so that by selling theblended variety at Rs 30 per kg there should be a gain of10%?(a) 32 kg (b) 40 kg (c) 36 kg(d) 42 kg (e) None of these

25. How many kg of sugar costing Rs 5.75 per kg should bemixed with 75 kg of cheaper sugar costing Rs 4.50 per kg sothat the mixture is worth Rs 5.50 per kg ?(a) 350 kg (b) 300 kg (c) 250 kg(d) 325 kg (e) None of these

Page 85: yoursmahboob.wordpress.com SBI · yoursmahboob.wordpress.com iii P 101 Speed Tests for SBI Bank Clerk Exam 101 Speed Tests for SBI Bank Clerk Exam is revised and updated edition on

yoursmahboob.w

ordpress.com

1. If the cost price is 96% of the selling price, then what is theprofit percent?(a) 4.5% (b) 4.2% (c) 4%(d) 3.8% (e) None of these

2. If the manufacturer gains 10%, the wholesale dealer 15%and the retailer 25%, then find the cost of production of atable, the retail price of which is 1265?(a) ` 800 (b) ` 1000 (c) ` 900(d) ` 600 (e) None of these

3. A man sold his book for Rs 891, thereby gaining 1

10of its

cost price. Find his cost price.(a) ` 850 (b) ` 810 (c) ` 851(d) ` 840 (e) None of these

4. A man buys 50 pencils for Rs 100 and sells 45 pencils for` 90. Find his gain or loss %.(a) 20% (b) 35% (c) 25%(d) No gain or loss (e) None of these

5. A dealer sold a mixer for 420 at a loss of 12.5%. At whatprice should he have sold it to gain 12.5%.(a) ` 620 (b) ` 540 (c) ` 650(d) ` 750 (e) None of these

6. A man sold 10 eggs for 5 rupees and gained 20%.Howmany eggs did he buy for 5 rupees?(a) 10 eggs (b) 12 eggs (c) 14 eggs(d) 16 eggs (e) None of these

7. Coconuts were purchased at ` 150 per hundred and sold at` 2 per coconut. If 2000 coconuts were sold, what was thetotal profit made?(a) ` 500 (b) ` 1000 (c) ` 1500(d) ` 2000 (e) None of these

8. A shopkeeper’s price is 50% above the cost price. If heallows his customer a discount of 30% what profit does hemake?(a) 5% (b) 10% (c) 15%(d) 20% (e) None of these

RESPONSE

GRID

1. a b c d e 2. a b c d e 3. a b c d e 4. a b c d e 5. a b c d e

6. a b c d e 7. a b c d e 8. a b c d e 9. a b c d e 10. a b c d e

11. a b c d e 12. a b c d e 13. a b c d e 14. a b c d e 15. a b c d e

Max. Marks : 30 No. of Qs. 30 Time : 20 min. Date : ........./......../................

9. A man buys milk at Rs 6 per litre and adds one third of waterto it and sells mixture at Rs 7.20 per litre. The gain is(a) 40% (b) 80% (c) 60%(d) 25% (e) None of these

10. A dishonest fruit seller professes to sell his goods at the costprice but weighs 800 grams for a kg weight. Find his gainpercent.(a) 100% (b) 150% (c) 50%(d) 200% (e) None of these

11. Two electronic musical instruments were purchased for` 8000. The first was sold at a profit of 40% and the second atloss of 40%. If the sale price was the same in both the cases,what was the cost price of two electronic musical instruments?(a) ` 2000, 5000 (b) ` 2200, 5500(c) ` 2400, 5000 (d) ` 2400, 5600(e) None of these

12. A tradesman is marketing his goods 20% above the cost priceof the goods. He gives 10% discount on cash payment, findhis gain percent.(a) 12% (b) 8% (c) 15%(d) 18% (e) None of these

13. A man sells his car for ` 5000 and loses something. Had hesold it for 5600, his gain would have been double the formerloss. Find the cost price.(a) ` 5500 (b) ` 5100 (c) ` 5400(d) ` 5200 (e) None of these

14. John sold a fan at a loss of 7%. If he had sold it for ` 48 more,he would have gained 5%. Find the cost price of the fan.(a) ` 350 (b) ` 480 (c) ` 240(d) ` 400 (e) None of these

15. The owner of an electronics shop charges his customer 22%more than the cost price. If a customer paid ` 10,980 for aDVD player, then what was the cost price of the DVD player?

(a) ` 8000 (b) ` 8800 (c) ` 9500

(d) ` 9200 (e) None of these

3838Profit, Loss &Discount

Page 86: yoursmahboob.wordpress.com SBI · yoursmahboob.wordpress.com iii P 101 Speed Tests for SBI Bank Clerk Exam 101 Speed Tests for SBI Bank Clerk Exam is revised and updated edition on

yoursmahboob.w

ordpress.com

78 SPEED TEST 38

RESPONSE

GRID

16. a b c d e 17. a b c d e 18. a b c d e 19. a b c d e 20. a b c d e

21. a b c d e 22. a b c d e 23. a b c d e 24. a b c d e 25. a b c d e

26. a b c d e 27. a b c d e 28. a b c d e 29. a b c d e 30. a b c d e

16- Cost of 24 bats and 32 sticks is `5,600. What is the price of3 bats and 4 sticks?(a) ` 1,400 (b) ` 2,800(c) ` 700 (d) Cannot be determined(e) None of these

17. The profit earned after selling an article for ` 1,754 is thesame as loss incurred after selling the article for` 1,492. What is the cost price of the article?(a) ` 1,623 (b) ` 1,523 (c) ` 1,689(d) ` 1,589 (e) None of these

18. Prathik sold a music system to Karthik at 20% gain and Karthiksold it to Swasthik at 40% gain. If Swasthik paid 10,500 forthe music system, what amount did Prathik pay for the same?(a) ` 8,240 (b) ` 7,500 (c) ` 6,250(d) Cannot be determined(e) None of these

19. The cost of 5 pendants and 8 chains is ` 1,45,785. Whatwould be the cost of 15 pendants and 24 chains?(a) ` 3,25,285 (b) ` 4,39,355(c) ` 5,50,000 (d) Cannot be determined(e) None of these

20. What price should a shopkeeper mark on an article costinghim Rs 153 to gain 20% after allowing a discount of 15% ?(a) Rs 162 (b) Rs 621 (c) Rs 216(d) Rs 226 (e) None of these

21. Three successive discounts of 10%, 12% and 15% amountto a single discount of :(a) 36.28 % (b) 34.68% (c) 37 %(d) 38% (e) None of these

22. A machine is sold at a profit of 10%. Had it been sold for Rs80 less, there would have been a loss of 10%. The cost priceof the machine is :(a) Rs 350 (b) Rs 400 (c) Rs 450(d) Rs 520 (e) None of these

23. If selling price is doubled then, the profit triples. What isprofit per cent :

(a)3

266 (b) 100 (c)

133

3(d) 125 (e) None of these

24. If the cost of 12 pencils is equal to the selling price of 10pencils, the profit percent in the transaction is :

(a)1

16 %3

(b) %2

122 (c) 20%

(d) 25% (e) None of these25. Two motor cars were sold for Rs 9,900 each, gaining 10% on

one and losing 10% on the other. The gain or loss percent inthe whole transaction is :(a) Neither loss no gain

(b)99

1% gain (c) %

99

100 profit

(d) 1% loss (e) None of these26. The retail price of a water geyser is Rs 1265. If the

manufacturer gains 10%, the wholesale dealer gains 15%and the retailer gains 25%, then the cost of the geyser is :(a) Rs 800 (b) Rs 900 (c) Rs 550(d) Rs 650 (e) None of these

27. A man buys a watch for Rs. 1950 in cash and sells it for Rs.2200 on credit of 1 year. If the rate of interest is 10% perannum, then the man(a) Gains Rs. 55 (b) Gains Rs. 50(c) Loses Rs. 30 (d) Gains Rs. 30(e) None of these

28. A shopkeeper marks up his goods to gain 35%. But he allows10% discount for cash payment. His profit on the cashtransaction in percentage, is

(a) 113

2(b) 25 (c)

121

2

(d)1

312

(e) None of these

29. A man sold two steel chairs for Rs. 500 each. On one hegains 20% and on other, he loses 12%. How much does hegain or lose in the whole transaction?(a) 1.5% gain (b) 2% gain (c) 1.5% loss(d) 2% loss (e) None of these

30. By selling 12 notebooks, the seller earns a profit equal to theselling price of two notebooks. What is his percentageprofit?

(a) 25% (b) 20% (c)2

163

%

(d) Data inadequate(e) None of these

Page 87: yoursmahboob.wordpress.com SBI · yoursmahboob.wordpress.com iii P 101 Speed Tests for SBI Bank Clerk Exam 101 Speed Tests for SBI Bank Clerk Exam is revised and updated edition on

yoursmahboob.w

ordpress.com

1. Ms. Sandhya deposits an amount of ` 31,400 to obtain asimple interest at the rate of 12 per cent per annum for 8years. What total amount will Ms. Sandhya get at the endof 8 years?(a) ` 31,444 (b) ` 61,544 (c) ` 41,544(d) ` 31,144 (e) None of these

2. Mr. Deepak invested an amount of 21,250 for 6 years. Atwhat rate of simple interest will he obtain the total amountof ` 26,350 at the end of 6 years?(a) 6 % p.a (b) 5 % p.a (c) 8 % p.a(d) 12 % p.a (e) None of these

3. Asmita invests an amount of 9535 at the rate of 4 per centper annum to obtain a total amount of 11442 on simpleinterest after a certain period. For how many year did sheinvest the amount to obtain the total sum?(a) 10 years (b) 2 years (c) 5 years(d) 4 years (e) None of these

4. Girish invested a certain amount at the rate of 8% p.a. for 6year to obtain an amount of ` 28,046. How much amountdid Girish obtain as simple interest?(a) `12,550 (b) `9,096 (c) `18,950(d) Cannot be determined(e) None of these

5. Mr. Anuraag Awasthi deposits an amount of ` 56500 toobtain a simple interest at the rate of 12% p.a. for 3 years.What total amount will Mr. Anuraag Awasthi get at the endof 3 year ?(a) `75680 (b) `77540 (c) `76840(d) `73420 (e) None of these

6. Veena obtained an amount of ` 8, 376/- as simple intereston a certain amount at 8% p.a. after 6 years. What is theamount invested by Veena?(a) 17,180 (b) 18,110 (c) 16,660(d) 17,450 (e) None of these

7. The simple interest accrued on a sum of certain principal is` 2000 in five years at the rate of 4% per annum. Whatwould be the compound interest accrued on same principalat same rate in two years ?(a) ` 716 (b) ` 724 (c) ` 824(d) ` 816 (e) None of these

RESPONSE

GRID

1. a b c d e 2. a b c d e 3. a b c d e 4. a b c d e 5. a b c d e

6. a b c d e 7. a b c d e 8. a b c d e 9. a b c d e 10. a b c d e

11. a b c d e 12. a b c d e 13. a b c d e 14. a b c d e 15. a b c d e

Max. Marks : 30 No. of Qs. 30 Time : 20 min. Date : ........./......../................

8. What total amount would Mithilesh get at the end of threeyears if he invests an amount of ` 11200 in a scheme, whichoffers simple interest 8.5% per annum for three years ?(a) ` 14056 (b) ` 14348 (c) ` 13852(d) ` 15064 (e) None of these

9. A sum of Rs. 2600 is lent out in two parts in such a way thatthe interest on one part at 10% for 5 years is equal to that onthe other part at 9% for 6 years. The sum lent out at 10% is________ .(a) 1250 (b) 1350 (c) 1450(d) 1550 (e) 1650

10. At a certain rate of simple interest, a certain sum doublesitself in 10 years. It will treble itself in years ________ .(a) 10 (b) 20 (c) 25(d) 30 (e) 45

11. A sum of money at simple interest amounts to 600 in 4 yearsand ` 650 in 6 years. Find the rate of interest per annum.(a) 3% (b) 5% (c) 9%(d) 10% (e) 15%

12. A person lent at certain sum of money at 4% simple interest;

and in 8 years the interest amounted to ` 340 less than thesum lent. Find the sum lent.(a) 500 (b) 600 (c) 1000(d) 1500 (e) 1700

13. In what time will 72 become 81 at 1

64

% p.a. simple interest?

(a) 1 year 6 months (b) 2 years (c) 1 years

(d)1

22

years (e) None of these

14. A sum of money lent out at simple interest amounts to` 1008 in 2 years and 1164 in 3½ years. Find the rate % p.a.

(a) 13% (b) 14% (c)1

12 %2

(d) 15% (e) None of these15. A sum of money lent out at simple interest amounts to 720

after 2 years and to 1,020 after a further period of 5 years.Find the sum and the rate %.(a) ` 500, 10% (b) ` 600, 10% (c) ` 500, 12%(d) ` 600, 12% (e) None of these

3939Simple Interest

Page 88: yoursmahboob.wordpress.com SBI · yoursmahboob.wordpress.com iii P 101 Speed Tests for SBI Bank Clerk Exam 101 Speed Tests for SBI Bank Clerk Exam is revised and updated edition on

yoursmahboob.w

ordpress.com

80 SPEED TEST 39

RESPONSE

GRID

16. a b c d e 17. a b c d e 18. a b c d e 19. a b c d e 20. a b c d e

21. a b c d e 22. a b c d e 23. a b c d e 24. a b c d e 25. a b c d e

26. a b c d e 27. a b c d e 28. a b c d e 29. a b c d e 30. a b c d e

16. On retirement, a person gets 1.53 lakhs of his provident fundwhich he invests in a scheme at 20% p.a. His monthly incomefrom this scheme will be(a) ` 2, 450 (b) ` 2,500 (c) ` 2, 550(d) ` 2, 600 (e) None of these

17. A sum was put at simple interest at a certain rate for 4 yearsHad it been put at 2% higher rate, it would have fetched ` 56more. Find the sum.(a) ` 500 (b) ` 600 (c) ` 700(d) ` 800 (e) None of these

18. The simple interest on ` 200 for 7 months at 5 paise perrupee per month is(a) ` 70 (b) ` 7 (c) ` 35(d) ` 30.50 (e) None of these

19. A father left a will of 68,000 to be divided between his twosons aged 10 years and 12 years such that they may getequal amount when each attains the age of 18 years If themoney is reckoned at 10% p.a., find how much each gets atthe time of the will.(a) ` 30,000, 38,000 (b) ` 28,000, 40,000(c) ` 32,000, 36,000 (d) cannot be determined.(e) None of these

20. In how many minimum number of complete years, the intereston ` 212.50 P at 3% per annum will be in exact number ofrupees?(a) 6 (b) 8 (c) 9(d) 7 (e) None of these

21. What annual instalment will discharge a debt of 4,200 duein 5 years at 10% simple interest?(a) ` 500 per year (b) ` 600 per year(c) ` 700 per year (d) ` 800 per year(e) None of these

22. A certain amount earns simple interest of 1750 after 7 yearsHad the interest been 2% more, how much more interestwould it have earned?(a) ` 35 (b) ` 245 (c) ` 350(d) Cannot be determined(e) None of these

23. What will be the ratio of simple interest earned by certainamount at the same rate of interest for 6 years and that for9 years?(a) 1 : 3 (b) 1 : 4 (c) 2 : 3(d) Data inadequate (e) None of these

24. Nitin borrowed some money at the rate of 6% p.a. for thefirst three years, 9% p.a. for the next five years and 13% p.a.for the period beyond eight years If the total interest paid byhim at the end of eleven years is 8160, how much moneydid he borrow?(a) ` 8000 (b) ` 10,000 (c) ` 12,000(d) Data inadequate (e) None of these

25. An automobile financier claims to be lending money at simpleinterest, but he includes the interest every six months forcalculating the principal. If he is charging an interest of 10%,the effective rate of interest becomes :(a) 10% (b) 10.25% (c) 10.5%(d) Data inadequate (e) None of these

26. A lent ` 5000 to B for 2 years and 3000 to C for 4 years onsimple interest at the same rate of interest and received` 2200 in all from both of them as interest. The rate of interestper annum is:

(a) 5% (b) 7% (c)1

78

%

(d) 10% (e) None of these27. The rates of simple interest in two banks A and B are in the

ratio 5 : 4. A person wants to deposit his total savings in twobanks in such a way that he received equal half-yearlyinterest from both. He should deposit the savings in banksA and B in the ratio.(a) 2 : 5 (b) 4 : 5 (c) 5 : 2(d) 5 : 4 (e) None of these

28. If a certain sum of money becomes double at simple interestin 12 years, what would be the rate of interest per annum ?

(a) 18

3(b) 10 (c) 12

(d) 14 (e) None of these29. Two equal sums were borrowed at 8% simple interest per

annum for 2 years and 3 years, respectively. The differencein the interests was Rs 56. The difference in the interestswas Rs 56. The sum borrowed were :(a) Rs 690 (b) Rs 700 (c) Rs 740(d) Rs 780 (e) None of these

30. How much interest will Rs 10,000 earn in 9 months at anannual rate of 6 percent ?(a) Rs 450 (b) Rs 460 (c) Rs 475(d) Rs 600 (e) None of these

Page 89: yoursmahboob.wordpress.com SBI · yoursmahboob.wordpress.com iii P 101 Speed Tests for SBI Bank Clerk Exam 101 Speed Tests for SBI Bank Clerk Exam is revised and updated edition on

yoursmahboob.w

ordpress.com

1. Sudhanshu invested 15,000 at interest @ 10% p.a for oneyear. If the interest is compounded every six months whatamount will Sudhanshu get at the end of the year?(a) ` 16,537.50 (b) ` 16,5000(c) ` 16,525.50 (d) ` 18,150(e) None of these

2. What would be the compound interest obtained on anamount of ` 1,250 at the rate of 8% p.a. after 2 year ?(a) ` 200 (b) ` 208 (c) `212

(d) `220 (e) None of these3. Shyam invests an amount of 5,690 at the rate of 5 per cent

per annum for 3 years. What approximate amount ofcompound interest will he obtain at the end of 3 years?

(a) ` 854 (b) ` 799 (c) ` 843

(d) 787 (e) 8974. The simple interest accrued on an amount of 84,000 at the

end of three year is ` 30,240. What would be the compoundinterest accrued on the same amount at the same rate in thesame period?(a) 30,013.95 (b) 31,013.95(c) 32,013.95 (d) 33,013.95(e) 34,013-95

5. What will be the difference between the compound interestand simple interest at the rate of 5% p.a. on an amount of` 4,000 at the end of two years?(a) ` 10 (b) ` 20(c) ` 30 (d) Data inadequate(e) None of these

6. Pamela invested an amount of 35,000 for two year at therate of 5% p.a. What amount of compound interest wouldshe receive at the end of two year ?(a) ` 3587.50 (b) ` 3500 (c) 3580.50(d) ` 3565.50 (e) None of these

RESPONSE

GRID

1. a b c d e 2. a b c d e 3. a b c d e 4. a b c d e 5. a b c d e

6. a b c d e 7. a b c d e 8. a b c d e 9. a b c d e 10. a b c d e

11. a b c d e 12. a b c d e 13. a b c d e

Max. Marks : 25 No. of Qs. 25 Time : 20 min. Date : ........./......../................

7. Rohit invested some amount at the rate of 6 pcpa and at theend of 2 yr he got 8730 simple interest. How much compoundinterest he will get on same amount and same rate of interestafter 2 yr.(a) ` 5820 (b) ` 5949.60 (c) ` 5900(d) ` 5994.60 (e) None of these

8. Sonika invested an amount of 5800 for 2 years. At what rateof compound interest will she get an amount of 594.5 at theend of two years ?(a) 5 p.c.p.a. (b) 4 p.c.p.a. (c) 6 p.c.p.a.(d) 8 p.c.p.a. (e) None of these

9. What would be the compound interest accrued on an amountof 7400 @ 13.5 p.c.p.a. at the end of two years ?(Rounded off to two digits after decimal)(a) ` 2136.87 (b) ` 2306.81 (c) ` 2032.18(d) ` 2132.87 (e) None of these

10. If the compound interest accrued on an amount of 14500 intwo years is 4676.25, what is the rate of interest p.c.p.a ?(a) 11 (b) 9 (c) 15(d) 18 (e) None of these

11. What will be the compound interest accrued on an amount of` 10000 @ per annum in two years if the interest iscompounded half-yearly ?(a) ` 4400 (b) ` 4600 (c) ` 4641(d) ` 4680 (e) None of these

12. What will be the difference between the simple interest andcompound interest earned on a sum of 985.00 at the rate of14% per annum at the end of two years ?(a) ` 16.408 (b) ` 14.214 (c) ` 19.218(d) ` 17.405 (e) None of these

13. A sum of money at compound interest amounts in two yearsto ` 2809, and in three years to ` 2977.54. Find the rate ofinterest and the original sum.(a) 2000 (b) 2100 (c) 2200(d) 2500 (e) 3000

4040Compound Interest

Page 90: yoursmahboob.wordpress.com SBI · yoursmahboob.wordpress.com iii P 101 Speed Tests for SBI Bank Clerk Exam 101 Speed Tests for SBI Bank Clerk Exam is revised and updated edition on

yoursmahboob.w

ordpress.com

82 SPEED TEST 40

RESPONSE

GRID

14. a b c d e 15. a b c d e 16. a b c d e 17. a b c d e 18. a b c d e

19. a b c d e 20. a b c d e 21. a b c d e 22. a b c d e 23. a b c d e

24. a b c d e 25. a b c d e

14. In what time will 6250 amount to 6632.55 at 4 p.c. compoundinterest payable half-yearly?

(a)3

5(b)

3

2(c)

3

4

(d)5

2(e)

9

1115. If the simple interest on a certain sum of money for 3 years at

5% is ` 150, find the corresponding CI.(a) 197 (b) 157.62 (c) 137.36(d) 117.17 (e) 127.34

16. A sum of money becomes eight times in 3 years if the rate iscompounded annually. In how much time, the same amountat the same compound interest rate will become sixteen times?(a) 6 years (b) 4 years (c) 8 years(d) 5 years (e) None of these

17. The difference between the simple interest and the compoundinterest compounded annually at the rate of 12% per annumon Rs 5000 for two years will be :(a) Rs 47.50 (b) Rs 63 (c) Rs 45(d) Rs 72 (e) None of these

18. Rahul borrowed a certain sum from Dhawan at a certain rateof simple interest for 2 years. He lent this sum to Ramesh atthe same rate of interest compounded annually for the sameperiod. At the end of two years, he received Rs 4200 ascompound interest but paid Rs 4000 only as simple interest.Find the rate of interest.(a) 12% (b) 25% (c) 35%(d) 10% (e) None of these

19. What will Rs 1000 be worth after three years if it earns interestat the rate of 5% compounded annually ?(a) Rs 1075 (b) Rs 1257 (c) Rs 1157(d) Rs 1300 (e) None of these

20. A sum of money invested at compound interest amounts in3 years to Rs 2,400 and in 4 years to Rs. 2,520. The interestrate per annum is :(a) 6% (b) 5% (c) 10%(d) 12% (e) None of these

21. A computer is available for Rs. 39,000 cash or Rs. 17,000 ascash down payment followed by five monthly installmentsof Rs. 4,800 each. What is the rate of interest under theinstalment plan ?(a) 35.71% p.a. (b) 37.71% p.a.(c) 36.71% p.a. (d) 38.71% p.a.(e) None of these

22. Under the Rural Housing Scheme, the Delhi DevelopmentAuthority (DDA) allotted a house to Kamal Raj for Rs.1,26,100. This payment is to be made in three equal annualinstalments. If the money is reckoned at 5% per annumcompound interest, then how much is to be paid by KamalRaj in each instalment ?(a) Rs. 45,205 (b) Rs. 47,405 (c) Rs. 46,305(d) Rs. 48,505 (e) None of these

23. A finance company declares that, at a certain compoundinterest rate, a sum of money deposited by anyone willbecome 8 times in three years. If the same amount is depositedat the same compound rate of interest, then in how manyyear will it become 16 times ?(a) 5 years (b) 4 years (c) 6 years(d) 7 years (e) None of these

24. Seema invested an amount of Rs. 16,000 for two years oncompound interest and received an amount of Rs. 17,640 onmaturity. What is the rate of interest ?(a) 5 % pa (b) 8 % pa(c) 4 % pa (d) Data inadequate(e) None of these

25. Two friends A and B jointly lent out Rs. 81,600 at 4% perannum compound interest. After 2 years A gets the sameamount as B gets after 3 years. The investment made by Bwas(a) Rs. 40,000 (b) Rs. 30,000 (c) Rs. 45,000(d) Rs. 38,000 (e) None of these

Page 91: yoursmahboob.wordpress.com SBI · yoursmahboob.wordpress.com iii P 101 Speed Tests for SBI Bank Clerk Exam 101 Speed Tests for SBI Bank Clerk Exam is revised and updated edition on

yoursmahboob.w

ordpress.com

1. How many seconds will a train 60 m in length, travelling atthe rate of 42 km an hour, rate to pass another train 84 mlong, proceeding in the same direction at the rate of 30 kman hour?(a) 42 (b) 43.2 (c) 45

(d) 50 (e) None of these2. A train takes 5 seconds to pass an electric pole. If the length

of the train is 120 metres, the time taken by it to cross arailway platform 180 metres long is _______ seconds.(a) 12.5 (b) 13.5 (c) 14.5(d) 15.5 (e) None of these

3. A train is running at the rate of 40 kmph. A man is alsogoing in the same direction parallel to the train at the speedof 25 kmph. If the train crosses the man in 48 seconds, thelength of the train is _______ metres.(a) 100 (b) 150 (c) 200(d) 250 (e) None of these

4. A train speeds past a pole in 15 seconds and speeds past aplatform 100 metres long in 25 seconds. Its length in metresis ________ .(a) 50 (b) 100 (c) 150(d) 200 (e) None of these

5. A bus covers a distance of 2,924 kms in 43 hours. What isthe speed of the bus?(a) 72 kmph (b) 60kmph(c) 68kmph (d) Cannot be determined(e) None of these

6. A bus travels at the speed of 49 kmph and reaches itsdestination in 7 hours. What is the distance covered by thebus?(a) 343 km (b) 283 km (c) 353 km(d) 245 km (e) 340 km

7. Nilesh goes to school from his village & returns at thespeed of 4 km/hr. If he takes 6 hours in all, then what is thedistance between the village and the school?(a) 6 km (b) 5 km(c) 4 km (d) Cannot be determined(e) None of these

RESPONSE

GRID

1. a b c d e 2. a b c d e 3. a b c d e 4. a b c d e 5. a b c d e

6. a b c d e 7. a b c d e 8. a b c d e 9. a b c d e 10. a b c d e

11. a b c d e 12. a b c d e 13. a b c d e

Max. Marks : 30 No. of Qs. 30 Time : 20 min. Date : ........./......../................

8. A 200 meter long train crosses a platform double its length in36 seconds. What is the speed of the train in km/hr ?(a) 60 (b) 48 (c) 64(d) 66 (e) None of these

9. A 160 meter long train running at a speed of 90 km/h crossesa platform in 18 seconds. What is the length of the platform inmeters?(a) 210 (b) 240 (c) 290(d) 310 (e) None of these

10. Excluding the stoppages, the speed of a bus is 64 km/hr andincluding the stoppages the speed of the bus is 48 km/hr. Forhow many minutes does the bus stop per hour?(a) 12.5 minutes (b) 15 minutes (c) 10 minutes(d) 18 minutes (e) None of these

11. The ratio between the speed of a train and a car is 18 : 13 .Also, a bus covered a distance of 480 kms. in 12 hours. Thespeed of the bus is five-ninth the speed of the train. Howmuch distance will the car cover in 5 hours ?(a) 250 km. (b) 280 km.(c) 260 km. (d) Cannot be determined(e) None of these

12. A 300 meter long train moving with an average speed of 126km/hr crosses a platform in 24 seconds. A man crosses thesame platform in 5 minutes. What is the speed of the man inmeters/second(a) 1.8 m/s (b) 1.2 m/s(c) 1.5 m/s (d) Cannot be determined(e) None of these

13. A bike covers a certain distance at the speed of 64 km/hr in 8hours. If a bike was to cover the same distance inapproximately 6 hours, at what approximate speed should thebike travel?(a) 80 km./hr. (b) 85 km/hr. (c) 90 km./hr.(d) 75 km/hr. (e) 70 km./hr

4141Distance, Speedand Time

Page 92: yoursmahboob.wordpress.com SBI · yoursmahboob.wordpress.com iii P 101 Speed Tests for SBI Bank Clerk Exam 101 Speed Tests for SBI Bank Clerk Exam is revised and updated edition on

yoursmahboob.w

ordpress.com

84 SPEED TEST 41

RESPONSE

GRID

14. a b c d e 15. a b c d e 16. a b c d e 17. a b c d e 18. a b c d e

19. a b c d e 20. a b c d e 21. a b c d e 22. a b c d e 23. a b c d e

24. a b c d e 25. a b c d e 26. a b c d e 27. a b c d e 28. a b c d e

29. a b c d e 30. a b c d e

14. The driver of a car driving @ 36 kmph locates a bus 40meters ahead of him. After 20 seconds the bus is 60 metersbehind. The speed of the bus is :(a) 36 kmph (b) 20 m/sec. (c) 72 m/sec.(d) 18 kmph (e) None of these

15. Two trains 100 meters and 120 meters long are running in thesame direction with speeds of 72 km/h and 54 km/h. In howmuch time will the first train cross the second?(a) 50 sec (b) 44 sec (c) 38 sec(d) 42 sec (e) None of these

16. A man covers a certain distance on a scooter. If the scootermoved 4 km/h faster, it would take 30 minutes less. If it moved2 km/h slower, it would have taken 20 minutes more. Find thedistance.

(a) 60 km (b) 58 km (c) 55 km(d) 50 km (e) None of these

17. A boat running downstream covers a distance of 16 km in 2hours while for covering the same distance upstream, it takes4 hours. What is the speed of the boat in still water?(a) 4 km/h (b) 6 km/h(c) 8 km/h (d) Data inadequate

(e) None of these18. In a 800 m race around a stadium having the circumference

of 200 m, the top runner meets the last runner on the 5thminute of the race. If the top runner runs at twice the speedof the last runner, what is the time taken by the top runner tofinish the race ?(a) 20 min (b) 15 min (c) 10 min(d) 5 min (e) None of these

19. A long distance runner runs 9 laps of a 400 meters trackeveryday. His timings (in minutes) for four consecutive daysare 88, 96, 89 and 87 resplectively. On an average, how manymeters/minute does the runner cover ?(a) 40 m/min (b) 45 m/min (c) 38 m/min(d) 49 m/min (e) None of these

20. A cyclist covers a distance of 750 m in 2 min 30 sec. What isthe speed in km/h of the cyclist ?(a) 18 km/h (b) 15 km/h (c) 20 km/h(d) 22 km/h (e) None of these

21. An aeroplane flies along the four sides of a square at thespeeds of 200, 400, 600 and 800 km/h. Find the average speedof the plane around the field.(a) 384 km/h (b) 370 km/h (c) 368 km/h(d) 378 km/h (e) None of these

22. R and S start walking each other at 10 AM at the speeds of3 km/h and 4 km/h respectively. They were initially 17.5 kmapart. At what time do they meet?(a) 2 : 30 PM (b) 11 : 30 AM (c) 1 : 30 PM(d) 12 : 30 PM (e) None of these

23. A train does a journey without stoppage in 8 hours, if it hadtravelled 5 km/h faster, it would have done the journey in 6hours 40 minutes. Find its original speed.(a) 25 km/h (b) 40 km/h (c) 45 km/h(d) 36.5 km/h (e) None of these

24. A car travels 25 km an hour faster than a bus for a journey of500 km. If the bus takes 10 hours more than the car, then thespeeds of the bus and the car are(a) 25 km/h and 40 km/h respectively(b) 25 km/h and 60 km/h respectively(c) 25 km/h and 50 km/h respectively(d) Cannot be determined(e) None of these

25. A thief goes away with a Maruti car at a speed of 40 km/h.The theft has been discovered after half an hour and theowner sets off in another car at 50 km/h. When will the ownerovertake the thief from the start.

(a)1

22

hours (b) 2 hr 20 min

(c) 1 hr 45 min (d) Cannot be determined(e) None of these

26. In a flight of 600 km, an aircraft was slowed down due to badweather. Its average speed for the trip was reduced by 200km/ hr and the time of flight increased by 30 minutes. Theduration of the flight is:(a) 1 hours (b) 2 hours (c) 3 hours(d) 4 hours (e) None of these

27. A train covers 180 km distance in 4 hours. Another traincovers the same distance in 1 hour less. What is the differencein the distances covered by these trains in one hour ?(a) 45 km (b) 9 km (c) 40 km(d) 25 km (e) None of these

28. The jogging track in a sports complex is 726 metres incircumference. Pradeep and his wife start from the same pointand walk in opposite directions at 4.5 km/h and 3.75 km/h,respectively. They will meet for the first time in :(a) 5.5 min (b) 6.0 min (c) 5.28 min(d) 4.9 min (e) None of these

29. The speed of a boat in still water is 15 km/h and the rate ofstream is 5 km/h. The distance travelled downstream in 24minutes is(a) 4 km (b) 8 km (c) 6 km(d) 16 km (e) None of these

30. A man makes his upward journey at 16 km/h and downwardjourney at 28 km/h. What is his average speed ?(a) 32 km/h (b) 56 km/h (c) 20.36 km/h(d) 22 km/h (e) None of these

Page 93: yoursmahboob.wordpress.com SBI · yoursmahboob.wordpress.com iii P 101 Speed Tests for SBI Bank Clerk Exam 101 Speed Tests for SBI Bank Clerk Exam is revised and updated edition on

yoursmahboob.w

ordpress.com

1. ‘A’ can complete a piece of work in 12 days. ‘A’ and ‘B’ togethercan complete the same piece of work in 8 days. In how manydays can ‘B’ alone complete the same piece of work?(a) 15 days (b) 18 days (c) 24 days(d) 28 days (e) None of these

2. George takes 8 hours to copy a 50 page manuscript whileSonia can copy the same manuscript in 6 hours. How manyhours would it take them to copy a 100 page manuscript, ifthey work together ?

(a)6

67

(b) 9 (c)5

97

(d) 14 (e) None of these3. A can finish a work in 18 days and B can do the same work

in half the time taken by A. Then, working together, whatpart of the same work can they finish in a day?

(a)1

6(b)

1

9(c)

2

5

(d)2

7(e) None of these

4. A and B can finish a work in 10 days while B and C can do itin 18 days. A started the work, worked for 5 days, then Bworked for 10 days and the remaining work was finished byC in 15 days. In how many days could C alone have finishedthe whole work ?(a) 30 days (b) 15 days (c) 45 days(d) 24 days (e) None of these

5. Two pipes A and B can fill a cistern in 10 and 15 minutesrespectively. Both fill pipes are opened together, but at theend of 3 minutes, ‘B’ is turned off. How much time will thecistern take to fill ?(a) 6 min (b) 8 min (c) 10 min(d) 12 min (e) None of these

6. A is 30% more efficient than B. How much time will they,working together, take to complete a job which A alone couldhave done in 23 days?(a) 11 days (b) 13 days

(c)3

2017

days (d) Cannot be determined

(e) None of these

RESPONSE

GRID

1. a b c d e 2. a b c d e 3. a b c d e 4. a b c d e 5. a b c d e

6. a b c d e 7. a b c d e 8. a b c d e 9. a b c d e 10. a b c d e

11. a b c d e 12. a b c d e 13. a b c d e

Max. Marks : 30 No. of Qs. 30 Time : 20 min. Date : ........./......../................

7. Sunil and Pradeep can complete a work in 5 days and 15 daysrespectively. They both work for one day and then Sunilleaves. In how many days will the remaining work be completedby Pradeep ?(a) 11 days (b) 12 days (c) 15 days(d) 8 days (e) None of these

8. If 6 men and 8 boys can do a piece of work in 10 days while 26men and 48 boys can do the same in 2 days, the time taken by15 men and 20 boys in doing the same work will be:(a) 4 days (b) 5 days (c) 6 days(d) 7 days (e) None of these

9. 12 men complete a work in 9 days. After they have worked for6 days, 6 more men join them. How many days will they take tocomplete the remaining work?(a) 2 days (b) 3 days (c) 4 days(d) 5 days (e) None of these

10. A and B can do a job in 16 days and 12 days respectively. Bhas started the work alone 4 days before finishing the job, Ajoins B. How many days has B worked alone?(a) 6 days (b) 4 days (c) 5 days(d) 7 days (e) None of these

11. Three taps A, B and C can fill a tank in 12, 15 and 20 hoursrespectively. If A is open all the time and B and C are open forone hour each alternately, then the tank will be full in :

(a) 6 hrs. (b)2

6 hrs.3

(c) 7 hrs.

(d)1

7 hrs.2

(e) None of these

12. Two pipes A and B when working alone can fill a tank in 36min. and 45 min. respectively. A waste pipe C can empty thetank in 30 min. First A and B are opened. After 7 min., C is alsoopened. In how much time will the tank be full ?(a) 1/60 (b) 1/30 (c) 7/20(d) 13/20 (e) None of these

13. A man can do a piece of work in 10 days but with the assistanceof his son, the work is done in 8 days. In how many days, hisson alone can do the same piece of work?(a) 15 days (b) 22 days (c) 30 days(d) 40 days (e) None of these

4242Time and Work /Pipe and Cistern

Page 94: yoursmahboob.wordpress.com SBI · yoursmahboob.wordpress.com iii P 101 Speed Tests for SBI Bank Clerk Exam 101 Speed Tests for SBI Bank Clerk Exam is revised and updated edition on

yoursmahboob.w

ordpress.com

86 SPEED TEST 42

RESPONSE

GRID

14. a b c d e 15. a b c d e 16. a b c d e 17. a b c d e 18. a b c d e

19. a b c d e 20. a b c d e 21. a b c d e 22. a b c d e 23. a b c d e

24. a b c d e 25. a b c d e 26. a b c d e 27. a b c d e 28. a b c d e

29. a b c d e 30. a b c d e

14. A can do 3

4of a work in 12 days. In how many days can he

finish 18

of the work?

(a) 6 days (b) 5 days (c) 3 days(d) 2 days (e) None of these

15. A can do a piece of work in 25 days and B in 20 days. Theywork together for 5 days and then A goes away. In howmany days will B finish the remaining work ?(a) 17 days (b) 11 days (c) 10 days(d) 15 days (e) None of these

16. A contractor undertakes to built a walls in 50 days. Heemploys 50 peoples for the same. However after 25 days hefinds that only 40% of the work is complete. How many moreman need to be employed to complete the work in time?(a) 25 (b) 30 (c) 35(d) 20 (e) None of these

17. A and B can finish a work in 10 days while B and C can do itin 18 days. A started the work, worked for 5 days, then Bworked for 10 days and the remaining work was finished by Cin 15 days. In how many days could C alone have finishedthe whole work ?(a) 30 days (b) 15 days (c) 45 days(d) 24 days (e) None of these

18. 12 men complete a work in 18 days. Six days after they hadstarted working, 4 men joined them. How many days will allof them take to complete the remaining work ?(a) 10 days (b) 12 days (c) 15 days(d) 9 days (e) None of these

19. 10 men can complete a piece of work in 15 days and 15 womencan complete the same work in 12 days. If all the 10 men and15 women work together, in how many days will the work getcompleted?

(a) 6 (b)1

63

(c)2

63

(d)2

73

(e) None of these

20. A can do a piece of work in 10 days, while B alone can do it in15 days. They work together for 5 days and the rest of thework is done by C in 2 days. If they get ` 450 for the wholework, how should they divide the money ?(a) ` 225, 150, 75 (b) ` 250, 100, 100(c) ` 200, 150, 100 (d) ` 175, 175, 100(e) None of these

21. 10 men and 15 women together can complete a work in 6days. It takes 100 days for one man alone to complete thesame work. How many days will be required for one womanalone to complete the same work?(a) 90 (b) 125 (c) 145(d) 105 (e) None of these

22. A contractor undertook to do a piece of work in 9 days. Heemployed certain number of laboures but 6 of them wereabsent from the very first day and the rest could finish thework in only 15 days. Find the number of men originallyemployed .(a) 15 (b) 6 (c) 13(d) 9 (e) None of these

23. After working for 8 days, Anil finds that only 1

3of the work

has been done. He employs Rakesh who is 60% efficient asAnil. How many more days will Anil take to complete the job?(a) 15 days (b) 12 days (c) 10 days(d) 8 days (e) None of these

24. 2 men and 3 boys can do a piece of work in 10 days while3 men and 2 boys can do the same work in 8 days. In howmany days can 2 men and 1 boy to the work ?

(a)1

122

days (b)1

112

days (c)1

152

days

(d)1

132

days (e) None of these

25. A can do a certain job in 12 days. B is 60% more efficient thanA. How many days B alone take to do the same job?

(a)1

72

(b) 11 (c)1

82

(d) 8 (e) None of these26. A man can do a piece of work in 5 days but with the help of

his son he can do it in 3 days. In what time can the son do italone?

(a)1

62

days (b) 7 days (c)1

72

(d) 8 days (e) None of these27. x is 3 times as faster as y and is able to complete the work in

40 days less than y. Then the time in which they can completethe work together?

(a) 15 days (b) 10 days (c)1

72

days

(d) 5 days (e) None of these28. Pipe A can fill a tank in 5 hours, pipe B in 10 hours and pipe C

in 30 hours. If all the pipes are open, in how many hours willthe tank be filled ?(a) 2 (b) 2.5 (c) 3(d) 3.5 (e) None of these

29. Two taps can fill a tank in 12 and 18 minutes respectively.Both are kept open for 2 minutes and the first is turned off. Inhow many minutes more will the tank be filled ?(a) 15 min. (b) 20 min. (c) 11 min.(d) 13 min. (e) None of these

30. A cistern has three pipes, A, B and C. The pipes A and B canfill it in 4 and 5 hours respectively and C can empty it in 2hours. If the pipes are opened in order at 1, 2 and 3 a.m.respectively, when will the cistern be empty ?(a) 3 p.m. (b) 4 p.m. (c) 5 p.m.(d) 6 p.m. (e) None of these

Page 95: yoursmahboob.wordpress.com SBI · yoursmahboob.wordpress.com iii P 101 Speed Tests for SBI Bank Clerk Exam 101 Speed Tests for SBI Bank Clerk Exam is revised and updated edition on

yoursmahboob.w

ordpress.com

1. The average age of a man and his son is 16 years. The ratioof their ages is 15 : 1 respectively. What is the son’s age?(a) 30 years (b) 32 years (c) 2 years(d) 4 years (e) None of these

2. The average age of a lady and her daughter is 28.5. Theratio of their ages is 14 : 5 respectively. What is the daugh-ters age?(a) 12 years (b) 15 years(c) 18 years (d) Cannot be determined(e) None of these

3. Present age of Sudha and Neeta are in the ratio of 6 : 7respectively. Five years ago their ages were in the ratio of5 : 6 respectively. What is Sudha’s present age?(a) 30 years (b) 35 years(c) 40 years (d) Cannot be determined(e) None of these

4. Average age of 36 children of the class is 15 years. 12 morechildren joined whose average age is 16 years. What is theaverage age of all the 48 children together?(a) 15.25 years (b) 15.5 years(c) 15.3 years (d) 15.4 years(e) None of these

5. Two years ago the ratio of the ages of Swati and Khyatiwas 5 : 7 respectively. Two years hence the ratio of theirages will be 7 : 9 respectively. What is the present age ofKhyati?(a) 16 years (b) 14.5 years(c) 12 years (d) Cannot be determined(e) None of these

6. The age of a man is 4 times that of his son. 5 yrs ago, theman was nine times as old as his son was at that time. Whatis the present age of the man?(a) 28 yrs (b) 32 yrs (c) 40 yrs(d) 42 yrs (e) None of these

7. After 5 yrs, the age of a father will be thrice the age of hisson, whereas five years ago, he was 7 times as old as hisson was. What are their present ages?(a) 30 yrs (b) 40 yrs (c) 50 yrs(d) 60 yrs (e) None of these

8. 10 Yrs ago, Sita’s mother was 4 times older than herdaughter. After 10 yrs, the mother will be two times olderthan the daughter. What is the present age of Sita?

RESPONSE

GRID

1. a b c d e 2. a b c d e 3. a b c d e 4. a b c d e 5. a b c d e

6. a b c d e 7. a b c d e 8. a b c d e 9. a b c d e 10. a b c d e

11. a b c d e 12. a b c d e 13. a b c d e 14. a b c d e 15. a b c d e

Max. Marks : 30 No. of Qs. 30 Time : 20 min. Date : ........./......../................

(a) 10 yrs (b) 30 yrs (c) 20 yrs(d) 40 yrs (e) None of these

9. Ten yrs ago, A was half of B in age. If the ratio of their presentages is 3 : 4, what will be the total of their present ages?(a) 25 (b) 35 (c) 45(d) 50 (e) None of these

10. The sum of the ages of a mother and her daughter is 50 yrs.Also 5 yrs ago, the mother’s age was 7 times the age of thedaughter. What are the present ages of the mother and thedaughter?(a) 35, 5 (b) 40, 10 (c) 30, 20(d) 25, 15 (e) None of these

11. The ratio of the father’s age to the son’s age is 4 : 1. Theproduct of their ages is 196. What will be the ratio of theirages after 5 years?(a) 7 : 3 (b) 14 : 9 (c) 11 : 4(d) 17 : 3 (e) None of these

12. The ratio of Rita’s age to the age of her mother is 3 : 11. Thedifference of their ages is 24 yrs. What will be the ratio of theirages after 3 yrs?(a) 1 : 2 (b) 1 : 3 (c) 3 : 7(d) 2 : 5 (e) None of these

13. A man’s age is 125% of what it was 10 years ago, but 1

83 %3

of what it will be after 10 years. What is his present age?(a) 30 yrs (b) 40 yrs (c) 50 yrs(d) 60 yrs (e) None of these

14. The age of a man is three times the sum of the ages of his twosons. Five years hence, his age will be double of the sum ofthe ages of his sons. The father's present age is(a) 40 years (b) 45 years (c) 50 years(d) 55 years (e) 65 years

15. The ratio between the present ages of P and Q is 3 : 4respectively. Four years hence Q will be 5 years older than P.What is P’s present age?(a) 15 years (b) 20 years(c) 25 years (d) Cannot be determined(e) None of these

4343Problem Basedon Ages

Page 96: yoursmahboob.wordpress.com SBI · yoursmahboob.wordpress.com iii P 101 Speed Tests for SBI Bank Clerk Exam 101 Speed Tests for SBI Bank Clerk Exam is revised and updated edition on

yoursmahboob.w

ordpress.com

88 SPEED TEST 43

RESPONSE

GRID

16. a b c d e 17. a b c d e 18. a b c d e 19. a b c d e 20. a b c d e

21. a b c d e 22. a b c d e 23. a b c d e 24. a b c d e 25. a b c d e

26. a b c d e 27. a b c d e 28. a b c d e 29. a b c d e 30. a b c d e

16. Present ages of Rama and Shyama are in the ratio of 4 : 5respectively. Five years hence the ratio of their ages be-comes 5 : 6 respectively. What is Rama’s present age?(a) 25 years (b) 22 years (c) 20 years(d) 30 years (e) None of these

17. In a family, a couple has a son and daughter. The age of thefather is three times that of his daughter and the age of theson is half of his mother. The wife is nine years younger toher husband and the brother is seven years older than hissister. What is the age of the mother?(a) 40 years (b) 45 years (c) 50 years(d) 60 years (e) 65 years

18. Ram’s present age is three times his son’s present age andtwo-fifth of his father’s present age. The average of thepresent ages of all of them is 46 years. What is the differencebetween the Ram’s son’s present age and Ram’s father’spresent age?(a) 68 years (b) 88 years(c) 58 years (d) Cannot be determined(e) None of these

19. Abhay’s age after six years will be three-seventh of hisfather’s age. Ten years ago, the ratio of their ages was 1 : 5.What is Abhay’s father’s age at present?(a) 30 yrs. (b) 40 yrs. (c) 50 yrs.(d) 60 yrs. (e) 70 years

20. The present ages of three persons are in proportions4 : 7 : 9. Eight years ago, the sum of their ages was 56. Findtheir present ages (in years).(a) 8, 20, 28 (b) 16, 28, 36 (c) 20, 35, 45(d) 25, 30, 40 (e) None of these

21. Tanya’s grandfather was 8 times older to her 16 years ago.He would be 3 times of her age 8 years from now. Eight yearsago, what was the ratio of Tanya’s age to that of her grand-father?(a) 1 : 2 (b) 1 : 5 (c) 3 : 8(d) 11 : 53 (e) None of these

22. Q is as much younger than R as he is older than T. If the sumof the ages of R and T is 50 years, what is definitely thedifference between R and Q’s age?(a) 1 year (b) 2 years (c) 25 years(d) Data inadequate (e) None of these

23. The sum of the ages of a father and his son is 45 years. Fiveyears ago, the product of their ages is 34. Find the presentage of father.(a) 32 years (b) 36 years (c) 38 years(d) 40 years (e) 39 years

24. The sum of the ages of 5 children born at the intervals of 3years each is 50 years. What is the age of the youngestchild?

(a) 4 years (b) 8 years (c) 10 years(d) 12 years (e) None of these

25. If 6 years are subtracted from the present age of Gagan andthe remainder is divided by 18, then the present age of hisgrandson Anup is obtained. If Anup is 2 years younger toMadan whose age is 5 years, then what is Gagan’s presentage?

(a) 48 years (b) 60 years (c) 84 years(d) 96 years (e) 100 years

26. The ratio between the school ages of Neelam and Shaan is5 : 6 respectively. If the ratio between the one-third age ofNeelam and half of Shaan’s age is 5 : 9, then what is theschool age of Shaan?

(a) 25 years

(b) 30 years

(c) Cannot be determined

(d) 35 years

(e) None of these

27. A is two years older than his son. In two years, his age willbe twice the age of his son. The present age of the son is:

(a) 7 (b) 8 (c) 9

(d) 10 (e) 11

28. Eighteen years ago, a father was three times as old as hisson. Now the father is only twice as old as his son. Then thesum of the present ages of the son and the father is:

(a) 54 (b) 72 (c) 105(d) 108 (e) 116

29. One year ago, Preeti was four times as old as her daughterSonal. Six years hence, Preeti’s age will exceed her daughter’sage by 9 years. The ratio of the present ages of Preeti andher daughter is :

(a) 9 : 2 (b) 11 : 3 (c) 12 : 5(d) 13 : 4 (e) 17 : 7

30. The present age of the father and the son are in the ratio of8:3. After 12 years the ratio of their ages will be 2:1. What isthe sum of the present age of the father and the son?

(a) 66yrs (b) 70yrs (c) 74yrs(d) 78yrs (e) 80yrs.

Page 97: yoursmahboob.wordpress.com SBI · yoursmahboob.wordpress.com iii P 101 Speed Tests for SBI Bank Clerk Exam 101 Speed Tests for SBI Bank Clerk Exam is revised and updated edition on

yoursmahboob.w

ordpress.com

1. In how many ways can six different rings be worn on fourfingers of one hand ?(a) 10 (b) 12 (c) 15(d) 16 (e) None of these

2. In how many ways can 7 persons be seated at a round tableif 2 particular persons must not sit next to each other ?(a) 5040 (b) 240 (c) 480(d) 720 (e) None of these

3. In how many different ways can the letters of the word‘MATHEMATICS’ be arranged so that the vowels alwayscome together ?(a) 10080 (b) 4989600 (c) 120960(d) 12960 (e) None of these

4. The number of ways in which four letters of the word‘MATHEMATICS’ can be arranged is(a) 136 (b) 2454 (c) 1680(d) 192 (e) None of these

5. In how many different ways can hte letters of the word‘PRETTY’ be arranged?(a) 120 (b) 36 (c) 360(d) 720 (e) None of these

6. In how many different ways can be letters of the word‘CYCLE’ be arranged?(a) 120 (b) 4240 (c) 30(d) 80 (e) None of these

7. In how many different ways can the letters of the wordTRUST be arranged?(a) 240 (b) 120 (c) 80(d) 25 (e) None of these

DIRECTIONS (Qs.8 & 9): Answer these questions on the basisof the information given below :From a group of 6 men and 4 women a Committee of 4 persons isto be formed.

RESPONSE

GRID

1. a b c d e 2. a b c d e 3. a b c d e 4. a b c d e 5. a b c d e

6. a b c d e 7. a b c d e 8. a b c d e 9. a b c d e 10. a b c d e

11. a b c d e 12. a b c d e 13. a b c d e 14. a b c d e 15. a b c d e

Max. Marks : 30 No. of Qs. 30 Time : 20 min. Date : ........./......../................

8. In how many different ways can it be done so that thecommittee has at least one woman?(a) 210 (b) 225 (c) 195(d) 185 (e) None of these

9. In how many different ways can it be done, so that thecommittee has at least 2 men?(a) 210 (b) 225 (c) 195(d) 185 (e) None of these

10. In how many ways can 5 boys be chosen from 6 boys and 4girls so as to include exactly one girl?(a) 252 (b) 210 (c) 126(d) 90 (e) 60

11. In how many different ways can the letters of the wordCORPORATION be arranged?(a) 3326400 (b) 1663200 (c) 831600(d) 415800 (e) 207900

12. In how many different ways can the letters of the word"COUNTRY" be arranged in such a way that the vowelsalways come together?(a) 720 (b) 1440 (c) 2880(d) 5040 (e) None of these

13. In how many different ways can the letters of the word‘PROBLEM’ be arranged ?(a) 5060 (b) 720 (c) 5040(d) 980 (e) None of these

14. How many different ways can the letters in the word ATTENDbe arranged?

(a) 60 (b) 120 (c) 240(d) 80 (e) None of these

15. In how many different ways can the letters of the word‘OFFICES’ be arranged?

(a) 2520 (b) 5040 (c) 1850(d) 1680 (e) None of these

4444Permutation andCombination

Page 98: yoursmahboob.wordpress.com SBI · yoursmahboob.wordpress.com iii P 101 Speed Tests for SBI Bank Clerk Exam 101 Speed Tests for SBI Bank Clerk Exam is revised and updated edition on

yoursmahboob.w

ordpress.com

90 SPEED TEST 44

RESPONSE

GRID

16. a b c d e 17. a b c d e 18. a b c d e 19. a b c d e 20. a b c d e

21. a b c d e 22. a b c d e 23. a b c d e 24. a b c d e 25. a b c d e

26. a b c d e 27. a b c d e 28. a b c d e 29. a b c d e 30. a b c d e

16. In how many different ways can the letters of the word‘ARMOUR’ be arranged?

(a) 720 (b) 300 (c) 640(d) 350 (e) None of these

17. In how many different ways can 4 boys and 3 girls bearranged in a row such that all boys stand together and allthe girls stand together?

(a) 75 (b) 576 (c) 288(d) 24 (e) None of these

18. In how many different ways can be letters of the wordSOFTWARE be arranged in such a way that the vowelsalways come together?(a) 13440 (b) 1440(c) 360 (d) 120(e) None of these

19. A bag contains 2 red, 3 green and 2 blue balls. 2 balls are tobe drawn randomly. What is the probability that the ballsdrawn contain no blue ball?

(a)5

7(b)

10

21

(c)2

7(d)

11

21

(e) None of these20. In how many different ways can the letters of the word

BOOKLET be arranged such that B and T always cometogether?(a) 360 (b) 720(c) 480 (d) 5040(e) None of these

21. In a box there are 8 red, 7 blue and 6 green balls. One ball ispicked up randomly. What is the probability that it is neitherred nor green?

(a)7

19(b)

2

3

(c)3

4(d)

9

21(e) None of these

22. Letters of the word DIRECTOR are arranged in such a waythat all the vowels come together. Find out the total numberof ways for making such arrangement.(a) 4320 (b) 2720(c) 2160 (d) 1120(e) None of these

23. How many three digit numbers can having only twoconsecutive digits identical is(a) 153 (b) 162(c) 168 (d) 163(e) None of these

24. In how many ways can the letters of the word 'PRAISE' bearranged. So that vowels do not come together?(a) 720 (b) 576(c) 440 (d) 144(e) None of these

25. The number of ways in which one or more balls can beselected out of 10 white, 9 green and 7 blue balls is(a) 892 (b) 881(c) 891 (d) 879(e) None of these

26. How many 3-digit numbers, each less than 600, can be formedfrom {1, 2, 3, 4, 7, 9} if repetition of digits is allowed?(a) 216 (b) 180(c) 144 (d) 120(e) None of these

27. If a secretary and a joint secretary are to be selected from acommittee of 11 members, then in how many ways can theybe selected ?(a) 110 (b) 55(c) 22 (d) 11(e) None of these

28. On a railway route there are 20 stations. What is the numberof different tickets required in order that it may be possibleto travel from every station to every other station?(a) 40 (b) 380(c) 400 (d) 420(e) None of these

29. What is the number of five-digit numbers formed with 0, 1, 2,3, 4 without any repetition of digits?(a) 24 (b) 48(c) 96 (d) 120(e) None of these

30. What is the number of three-digit odd numbers formed byusing the digits 1, 2, 3, 4, 5, 6 if repetition of digits is allowed?(a) 60 (b) 108(c) 120 (d) 216(e) None of these

Page 99: yoursmahboob.wordpress.com SBI · yoursmahboob.wordpress.com iii P 101 Speed Tests for SBI Bank Clerk Exam 101 Speed Tests for SBI Bank Clerk Exam is revised and updated edition on

yoursmahboob.w

ordpress.com

1. Tickets numbered 1 to 20 are mixed up and then a ticket isdrawn at random. What is the probability that the tickerdrawn bears a number which is a multiple of 3?

(a)3

10(b)

3

20(c)

2

5

(d)12

(e) None of these

2. In a lottery, there are 10 prizes and 25 blanks. A lottery isdrawn at random. What is the probability of getting a prize?

(a)1

10(b)

2

5(c)

2

7

(d)57

(e) None of these

3. One card is drawn at random from a pack of 52 cards. Whatis the probability that the card drawn is a face card?

(a)1

13(b)

4

13(c)

1

4

(d)952

(e) None of these

4. One card is drawn from a pack of 52 cards. What is theprobability that the card drawn is either a red card or aking?

(a)12

(b)6

13(c)

713

(c)2752

(e) None of these

5. A bag contains 6 black and 8 white balls. One ball is drawnat random. What is the probability that the ball drawn iswhite?

(a)3

4(b)

4

7(c)

1

8

(d)3

7(e) None of these

RESPONSE

GRID

1. a b c d e 2. a b c d e 3. a b c d e 4. a b c d e 5. a b c d e

6. a b c d e 7. a b c d e 8. a b c d e 9. a b c d e 10. a b c d e

11. a b c d e

Max. Marks : 25 No. of Qs. 25 Time : 20 min. Date : ........./......../................6. In a box, there are 8 red, 7 blue and 6 green balls. One ball is

picked up randomly. What is the probability that it is neitherred nor green?

(a)23

(b)34

(c)7

19

(d)8

21(e)

9

217. Two dice are tossed. The probability that the total score is a

prime number is :

(a)16

(b)5

12(c)

12

(d)79

(e) None of these

8. In a simultaneous throw of two coins, the probability of gettingat least one head is

(a)1

2(b)

1

3(c)

2

3

(d)34

(e) None of these

9. Three unbiased coins are tossed. What is the probability ofgetting at least 2 heads?

(a)1

4(b)

1

2(c)

1

3

(d)1

8(e) None of these

10. In a single throw of a die, what is the probability of getting anumber greater than 4?

(a)12

(b)13

(c)23

(d)1

4(e) None of these

11. In a simultaneous throw of two dice, what is the probabilityof getting a total of 7?

(a)1

6(b)

1

4(c)

2

3

(d)34

(e) None of these

4545Probability

Page 100: yoursmahboob.wordpress.com SBI · yoursmahboob.wordpress.com iii P 101 Speed Tests for SBI Bank Clerk Exam 101 Speed Tests for SBI Bank Clerk Exam is revised and updated edition on

yoursmahboob.w

ordpress.com

92 SPEED TEST 45

RESPONSE

GRID

12. a b c d e 13. a b c d e 14. a b c d e 15. a b c d e 16. a b c d e

17. a b c d e 18. a b c d e 19. a b c d e 20. a b c d e 21. a b c d e

22. a b c d e 23. a b c d e 24. a b c d e 25. a b c d e

12. What is the probability of getting a sum 9 from two throwsof a date?

(a)1

6(b)

1

8(c)

1

9

(d)1

12(e) None of these

13. In a simultaneous throw of two dice, what is the probabilityof getting a doublet?

(a)1

6(b)

1

4(c)

2

3

(d)3

7(e) None of these

14. In a simultaneous throw of two dice, what is the probabilityof getting a total of 10 or 11?

(a)1

4(b)

1

6(c)

7

12

(d)5

36(e) None of these

15. A coin is tossed three times. What is the probability ofgetting head and tail (HTH) or tail and head (THT)alternatively ?(a) 1/4 (b) 1/5 (c) 1/6(d) 1/8 (e) None of these

16. In a lottery, 16 tickets are sold and 4 prizes are awarded. If aperson buys 4 tickets,what is the probability of his winninga prize?

(a) 4

4

16(b)

175256

(c)14

(d)81

256(e) None of these

17. Two letters are drawn at random from the word ‘HOME’.What is the probability that both the letters are vowels?(a) 1/ 6 (b) 5/ 6 (c) 1/ 2(d) 1/ 3 (e) None of these

18. Three dice are thrown. What is the probability that the samenumber will appear on each of them?

(a)1

6(b)

1

18(c)

1

24

(d)1

36(e) None of these

19. What is the probability that a leap year selected at randomcontains 53 Mondays?

(a)1

7(b)

2

7(c)

7

366

(d)26

183(e) None of these

20. What is the probability of getting a sum of 7 with two dice?

(a)1

6(b)

1

3(c)

1

12

(d)5

36(e) None of these

21. A bag contains 5 white, 7 red and 8 black balls. If 4 balls aredrawn one by one with replacement, what is the probabilitythat all are white ?

(a)256

1(b)

16

1(c)

20

4

(d)8

4(e) None of these

22. Two dice are tossed. The probability that the total score is aprime number is :

(a)6

1(b)

12

5(c)

2

1

(d)9

7(e) None of these

23. A bag contains 3 white balls and 2 black balls. Another bagcontains 2 white balls and 4 black balls. A bag is taken and aball is picked at random from it. The probability that the ballwill be white is:

(a)11

7 (b)30

7(c)

11

5

(d)15

7(e) None of these

24. Out of 20 consecutive positive integers, two are chosen atrandom. The probability that their sum is odd is(a) 19/20 (b) 10/19 (c) 1/20(d) 9/19 (e) None of these

25. An unbiased die is tossed. What is the probability of gettinga multiple of 3.

(a)1

2(b)

1

3(c)

2

5

(d)16

(e) None of these

Page 101: yoursmahboob.wordpress.com SBI · yoursmahboob.wordpress.com iii P 101 Speed Tests for SBI Bank Clerk Exam 101 Speed Tests for SBI Bank Clerk Exam is revised and updated edition on

yoursmahboob.w

ordpress.com

1. The circumference of a circle is 44 metres. Find the area ofthe circle.(a) 154 m2 (b) 160 m2 (c) 175 m2

(d) 168 m2 (e) None of these2. The length and breadth of a rectangle are in the ratio 9 : 5.

If its area is 720 m2, find its perimeter.(a) 112 metre (b) 115 metre(c) 110 metre (d) 118 metre(e) None of these

3. A circle and a rectangle have the same perimeter. The sidesof the rectangle are 18 cm and 26 cm. What is the area of thecircle ?(a) 88 cm2 (b) 154 cm2 (c) 1250 cm2

(d) 616 cm2 (e) None of these4. The cost of carpeting a room 18m long with a carpet 75 cm

wide at 4.50 per metre is 810. The breadth of the room is:(a) 7 m (b) 7.5 m (c) 8 m(d) 8.5 m (e) None of these

5. If the perimeter and diagonal of a rectangle are 14 and 5 cmsrespectively, find its area.(a) 12 cm2 (b) 16 cm2 (c) 20 cm2

(d) 24 cm2 (e) None of these6. In an isoscele right angled triangle, the perimeter is 20 metre.

Find its area.(a) 9,320 m2 (b) 8,750 m2 (c) 7,980 m2

(d) 8,150 m2 (e) None of these7. The diameter of a garden roller is 1.4 m and it is 2 m long.

How much area will it cover in 5 revolutions ? 22

use7

æ öp =ç ÷è ø

(a) 40 m2 (b) 44 m2 (c) 48 m2

(d) 36 m2 (e) None of these8. The area of a triangle is 615 m2. If one of its sides is 123

metre, find the length of the perpendicular dropped on thatside from opposite vertex.(a) 15 metres (b) 12 metres(c) 10 metres (d) 9 metres(e) None of these

9. A horse is tethered to one corner of a rectangular grassyfield 40 m by 24 m with a rope 14 m long. Over how mucharea of the field can it graze?(a) 154 cm2 (b) 308 m2 (c) 150 m2

(d) 145 cm2 (e) None of these

RESPONSE

GRID

1. a b c d e 2. a b c d e 3. a b c d e 4. a b c d e 5. a b c d e

6. a b c d e 7. a b c d e 8. a b c d e 9. a b c d e 10. a b c d e

11. a b c d e 12. a b c d e 13. a b c d e 14. a b c d e 15. a b c d e

16. a b c d e 17. a b c d e 18. a b c d e

Max. Marks : 35 No. of Qs. 35 Time : 25 min. Date : ........./......../................

10. How many plants will be there in a circular bed whose outeredge measure 30 cms, allowing 4 cm2 for each plant ?(a) 18 (b) 750 (c) 24(d) 120 (e) None of these

11. A rectangular plot 15 m ×10 m, has a path of grass outside it.If the area of grassy pathway is 54 m2, find the width of thepath.(a) 4 m (b) 3 m (c) 2 m(d) 1 m (e) None of these

12. If the area of a circle decreases by 36%, then the radius of acircle decreases by(a) 20% (b) 18% (c) 36%(d) 64% (e) None of these

13. The floor of a rectangular room is 15 m long and 12 m wide.The room is surrounded by a verandah of width 2 m on allits sides. The area of the verandah is :(a) 124 m2 (b) 120 m2 (c) 108 m2

(d) 58 m2 (e) None of these14. A circular grass lawn of 35 metres in radius has a path 7

metres wide running around it on the outside. Find the area ofpath.(a) 1694 m2 (b) 1700 m2 (c) 1598 m2

(d) 1900 m2 (e) None of these15. The radius of the wheel of a bus is 70 cms and the speed of

the bus is 66 km/h, then the r.p.m. (revolutions per minutes) ofthe wheel is(a) 200 (b) 250 (c) 300(d) 330 (e) None of these

16. The altitude drawn to the base of an isosceles triangle is 8 cmand the perimeter is 32 cm. The area of the triangle is(a) 72 cm2 (b) 60 cm2 (c) 66 cm2

(d) 67 cm2 (e) None of these17. The area of a square field is 576 km2. How long will it take for

a horse to run around at the speed of 12 km/h ?(a) 12 h (b) 10 h (c) 8 h(d) 6 h (e) None of these

18. The area of a rectangular field is 144 m2. If the length hadbeen 6 metres more, the area would have been 54 m2 more.The original length of the field is(a) 22 metres (b) 18 metres (c) 16 metres(d) 24 metres (e) None of these

4646Area and Perimeter

Page 102: yoursmahboob.wordpress.com SBI · yoursmahboob.wordpress.com iii P 101 Speed Tests for SBI Bank Clerk Exam 101 Speed Tests for SBI Bank Clerk Exam is revised and updated edition on

yoursmahboob.w

ordpress.com

94 SPEED TEST 46

RESPONSE

GRID

19. a b c d e 20. a b c d e 21. a b c d e 22. a b c d e 23. a b c d e

24. a b c d e 25. a b c d e 26. a b c d e 27. a b c d e 28. a b c d e

29. a b c d e 30. a b c d e 31. a b c d e 32. a b c d e 33. a b c d e

34. a b c d e 35. a b c d e

19. The ratio between the length and the breadth of a rectangularpark is 3 : 2. If a man cycling along the boundary of the parkat the speed of 12km / hr completes one round in 8 minutes,then the area of the park (in sq. m) is:(a) 15360 (b) 153600 (c) 30720(d) 307200 (e) None of these

20. A wire can be bent in the form of a circle of radius 56 cm. If itis bent in the form of a square, then its area will be:(a) 3520 cm2 (b) 6400 cm2 (c) 7744 cm2

(d) 8800 cm2 (e) None of these21. The length of a room is double its breadth. The cost of

colouring the ceiling at 25 per sq. m is ` 5,000 and the costof painting the four walls at ` 240 per sq. m is ` 64,800. Findthe height of the room.(a) 4.5 m (b) 4 m (c) 3.5 m(d) 5 m (e) None of these

22. The surface area of a cube is 150 m2. The length of itsdiagonal is

(a) 5 3 m (b) 5 m (c)10

3 m

(d) 15 m (e) None of these23. The length and breadth of a playground are 36m and 21 m

respectively. Poles are required to be fixed all along theboundary at a distance 3m apart. The number of polesrequired will be(a) 39 (b) 38 (c) 37(d) 40 (e) None of these

24. What would be the area of a square whose diagonal measures28 cm?(a) 288 sq cm (b) 514 sq cm (c) 428 sq cm(d) 392 sq cm (e) None of these

25. The perimeter of a square is one-fourth the perimeter of arectangle. If the perimeter of the square is 44 cm and thelength of the rectangle is 51 cm, what is the differencebetween the breadth of the rectangle and the side of thesquare?(a) 30 cm (b) 18 cm (c) 26 cm(d) 32 cm (e) None of these

26. The area of a rectangle is equal to the area of a circle withcircumference equal to 220 metres. What is the length of therectangle if its breadth is 50 metres?(a) 56 metres (b) 83 metres (c) 77 metres(d) 69 metres (e) None of these

27. A man riding a bicycle, completes one lap of a circular fieldalong its circumference at the speed of 79.2 km/hr in 2 minutes40 seconds. What is the area of the field?(a) 985600 sq metre (b) 848500 sq metre(c) 795600 sq metre (d) Cannot be determined(e) None of these

28. The perimeter of a square is double the perimeter of arectangle. The area of the rectangle is 240 sq cm. What is thearea of the square ?(a) 100 sq cm (b) 36 cq cm(c) 81 sq cm (d) Cannot be determined(e) None of these

29. If the perimeter of a square is equal to the radius of a circlewhose area is 39424 sq. cm, what is the area of the square ?(a) 1225 sq. cm (b) 441 sq. cm(c) 784 sq. cm (d) Cannot be determined(e) None of these

30. Two poles, 15 m and 30 m high, stand upright in a playground.If their feet be 36 m apart, find the distance between theirtops.(a) 35 cm (b) 39 cm (c) 45 cm(d) 50 cm (e) None of these

31. A semi-circle is constructed on each side of a square oflength 2m. Find the area of the whole figure.(a) (5 + 3p) m2 (b) (4 + 3p) m2 (c) (4 + p) m2

(d) (4 + 2p) m2 (e) None of these32. In a quadrilateral, the length of one of its diagonal is 23 cm

and the perpendiculars drawn on this diagonal from othertwo vertices measure 17 cm and 7 cm respectively. Find thearea of the quadrilateral.(a) 250 cm2 (b) 276 cm2 (c) 300 cm2

(d) 325 cm2 (e) None of these33. A circular wire of radius 42 cm is cut and bent in the form of

a rectangle whose sides are in the ratio of 6 : 5. Find thesmaller side of the rectangle.(a) 50 cm (b) 60 cm (c) 70 cm(d) 80 cm (e) None of these

34. The following figure contains three squares with areas of100, 16 and 49 sq. units respectively laying side by side asshown. By how much should the area of the middle squarebe reduced in order that the total length PQ of the resultingthree squares is 19?

10049

16

QP

(a) 12 (b) 4 (c) 3(d) 2 (e) 6

35. A rectangle has perimeter of 50 metres. If its length is 13metres more than its breadth, then its area is:(a) 124 m2 (b) 144 m2 (c) 114 m2

(d) 104 m2 (e) 117m2

Page 103: yoursmahboob.wordpress.com SBI · yoursmahboob.wordpress.com iii P 101 Speed Tests for SBI Bank Clerk Exam 101 Speed Tests for SBI Bank Clerk Exam is revised and updated edition on

yoursmahboob.w

ordpress.com

1. A cylindrical bucket of height 36 cm and radius 21 cm isfilled with sand. The bucket is emptied on the ground anda conical heap of sand is formed, the height of the heapbeing 12 cm. The radius of the heap at the base is :(a) 63 cm (b) 53 cm(c) 56 cm (d) 66 cm(e) None of these

2. A metal cube of edge 12 cm is melted and formed into threesmaller cubes. If the edges of two smaller cubes are 6 cmand 8 cm, then find the edge of the third smaller cube.(a) 10 cm (b) 14 cm(c) 12 cm (d) 16 cm(e) None of these

3. A well 22.5 deep and of diameter 7 m has to be dug out. Findthe cost of plastering its inner curved surface at 3 per sq.metre.(a) ` 1465 (b) ` 1485(c) ` 1475 (d) ` 1495(e) None of these

4. A copper sphere of radius 3 cm is beaten and drawn into awire of diametre 0.2 cm. The length of the wire is(a) 9 m (b) 12 m(c) 18 m (d) 36 m(e) None of these

5. If the volume of a sphere is divided by its surface area, theresult is 27 cms. The radius of the sphere is(a) 9 cms (b) 27 cms(c) 81 cms (d) 243 cms(e) None of these

6. A cistern 6 m long and 4 m wide contains water up to adepth of 1 m 25 cm. The total area of the wet surface is:(a) 49 m2 (b) 50 m2

(c) 53.5 m2 (d) 55 m2

(e) None of these

RESPONSE

GRID

1. a b c d e 2. a b c d e 3. a b c d e 4. a b c d e 5. a b c d e

6. a b c d e 7. a b c d e 8. a b c d e 9. a b c d e 10. a b c d e

11. a b c d e 12. a b c d e

Max. Marks : 25 No. of Qs. 25 Time : 20 min. Date : ........./......../................

7. The length of a cold storage is double its breadth. Its heightis 3 metres. The area of its four walls (including the doors) is108 m2. Find its volume.(a) 215 m3 (b) 216 m3

(c) 217 m3 (d) 218 m3

(e) None of these8. A cube of 384 cm2 surface area is melt to make x number of

small cubes each of 96 mm2 surface area. The value of x is(a) 80,000 (b) 8(c) 8,000 (d) 800(e) None of these

9. A conical vessel, whose internal radius is 12 cm and height 50cm, is full of liquid. The contents are emptied into a cylindricalvessel with internal radius 10 cm. Find the height to which theliquid rises in the cylindrical vessel.(a) 18 cm (b) 22 cm(c) 24 cm (d) 20 cm(e) None of these

10. A hollow sphere of internal and external diameters 4 cm and 8cm respectively is melted into a cone of base diamater 8 cm.The height of the cone is:(a) 12 cm (b) 14 cm(c) 15 cm (d) 18 cm(e) None of these

11. A cone of height 9 cm with diameter of its base 18 cm iscarved out from a wooden solid sphere of radius 9 cm. Thepercentage of the wood wasted is:(a) 25% (b) 30%(c) 50% (d) 75%(e) None of these

12. A monument has 50 cylindrical pillars each of diameter 50 cmand height 4 m. What will be the labour charges for gettingthese pillars cleaned at the rate of 50 paise per sq. m?

(use 3.14)p =(a) ` 237 (b) ` 157(c) ` 257 (d) ` 353(e) None of these

4747Volume andSurface Area

Page 104: yoursmahboob.wordpress.com SBI · yoursmahboob.wordpress.com iii P 101 Speed Tests for SBI Bank Clerk Exam 101 Speed Tests for SBI Bank Clerk Exam is revised and updated edition on

yoursmahboob.w

ordpress.com

96 SPEED TEST 47

RESPONSE

GRID

13. a b c d e 14. a b c d e 15. a b c d e 16. a b c d e 17. a b c d e

18. a b c d e 19. a b c d e 20. a b c d e 21. a b c d e 22. a b c d e

23. a b c d e 24. a b c d e 25. a b c d e

13. A conical vessel of base radius 2 cm and height 3 cm is filledwith kerosene. This liquid leaks through a hole in the bottomand collects in a cylindrical jar of radius 2 cm. The kerosenelevel in the jar is(a) p cm (b) 1.5 cm(c) 1 cm (d) 3 cm(e) None of these

14. In a swimming pool measuring 90 m by 40 m, 150 men take adip. If the average displacement of water by a man is 8 cubicmetres, what will be the rise in water level?(a) 33.33 cm (b) 30 cm (c) 20 cm (d) 25 cm(e) None of these

15. Three cubes of a metal are of edges 3 cm, 4 cm and 5 cm.These are melted together and from the melted material,another cube is formed. The edge of this cube is :(a) 8 cm (b) 10 cm(c) 9 cm (d) 6 cm(e) None of these

16. A cylindrical bath tub of radius 12 cm contains water to adepth of 20 cm. A spherical iron ball is dropped into the tuband thus the level of water is raised by 6.75 cm. What is theradius of the ball?(a) 8 cm (b) 9 cm(c) 12 cm (d) 7 cm(e) None of these

17. The volume of a cube is numerically equal to the sum of thelengths of its edges. What is its total surface area in squareunits ?(a) 64 (b) 144(c) 36 (d) 72(e) None of these

18. The volume of a cube whose surface area is 726m2, is :(a) 1300 m3 (b) 1331 m3

(c) 1452 m3 (d) 1542 m3

(e) None of these19. If a cistern is 3 m long, 2 m wide and 1 m deep, its capacity in

litres is :(a) 6 (b) 600(c) 6,000 (d) 60,000(e) None of these

20. The curved surface of a right circular cone of height 15 cmand base diameter 16 cm is :(a) 120p cm2 (b) 60 p cm2

(c) 136 p cm2 (d) 68p cm2

(e) None of these21. The weight of a solid cone having diameter 14 cm and vertical

height 51 cm is ....., if the material of solid cone weighs 10grams per cubic cm :(a) 16.18 kg (b) 17.25 kg(c) 26.18 kg (d) 71.40 kg(e) None of these

22. A hemispherical bowl is made of steel 0.5 cm thick. The insideradius of bowl being 4 cm. The volume of the steel used inmaking the bowl is :(a) 55.83 cm2 (b) 56.83 cm2

(c) 57.83 cm2 (d) 58.83 cm2

(e) None of these23. Consider the volumes of the following :

A. A parallelopiped of length 5 cm, breadth 3 cm and height4 cm.

B. A cube having each side 4 cm.C. A cylinder of radius 3 cm and length 3 cm.D. A sphere of radius 3 cm.The volumes of these in the decreasing order is :(a) A, B, C and D (b) A, C, B and D(c) D, B, C and A (d) D, C, B and A(e) None of these

24. Two solid spheres of radii 1 cm and 2 cm were melted andcombined to form a bigger sphere. The radius of the biggersphere is:

(a) 1

32(b)

133

(c) 138 (d)

139

(e) None of these25. The inner diameter of a circular building is 54 cm and the

base of the wall occupies a space of 352 cm2. The thicknessof the wall is :(a) 29 cm (b) 2 cm(c) 4 cm (d) 58 cm(e) None of these

Page 105: yoursmahboob.wordpress.com SBI · yoursmahboob.wordpress.com iii P 101 Speed Tests for SBI Bank Clerk Exam 101 Speed Tests for SBI Bank Clerk Exam is revised and updated edition on

yoursmahboob.w

ordpress.com

1. In triangle ABC, angle B is a right angle. If (AC) is 6 cm, andD is the mid-point of side AC. The length of BD is

C

D

A

B

(a) 4 cm (b) cm6(c) 3 cm (d) 3.5 cm(e) None of these

2. AB is diameter of the circle and the points C and D are onthe circumference such that .30CAD °=Ð What is the

measure of ?ACDÐ

A B

CD

70°

(a) 40° (b) 50°(c) 30° (d) 90°(e) None of these

3. The sum of the interior angles of a polygon is 1620°. Thenumber of sides of the polygon are :(a) 9 (b) 11(c) 15 (d) 12(e) None of these

4. A point P is 13 cm. from the centre of a circle. The length ofthe tangent drawn from P to the circle is 12cm. Find theradius of the circle.

O 13cm.

12cm.

P

T

r

RESPONSE

GRID

1. a b c d e 2. a b c d e 3. a b c d e 4. a b c d e 5. a b c d e

6. a b c d e 7. a b c d e 8. a b c d e 9. a b c d e

Max. Marks : 25 No. of Qs. 25 Time : 20 min. Date : ........./......../................

(a) 8 cm (b) 4 cm(c) 5 cm (d) 7 cm(e) None of these

5. If the angles of a triangle are in the ratio 5 : 3 : 2, then thetriangle could be :(a) obtuse (b) acute(c) right (d) isosceles(e) None of these

6. In the figure, AB = 8, BC = 7 m, ABCÐ = 1200. Find AC.

B CM

A

80120

(a) 11 (b) 12(c) 13 (d) 14(e) None of these

7. The perimeters of two similar triangles ABC and PQR are36 cm, and 24 cm, respectively. If PQ = 10 cm, then thelength of AB is :(a) 16 cm (b) 12 cm(c) 14 cm (d) 15 cm(e) None of these

8. Two isosceles triangles have equal vertical angles and theirareas are in the ratio 9 : 16. The ratio of their correspondingheights is :(a) 3 : 4 (b) 4 : 3(c) 2 : 1 (d) 1 : 2(e) None of these

9. The circumcentre of a triangle is always the point ofintersection of the :(a) medians(b) angle bisectors(c) perpendicular bisectors of sides(d) perpendiculars dropped from the vertices on the

opposite sides of the triangle.(e) None of these

4848Geometry

Page 106: yoursmahboob.wordpress.com SBI · yoursmahboob.wordpress.com iii P 101 Speed Tests for SBI Bank Clerk Exam 101 Speed Tests for SBI Bank Clerk Exam is revised and updated edition on

yoursmahboob.w

ordpress.com

98 SPEED TEST 48

RESPONSE

GRID

10. a b c d e 11. a b c d e 12. a b c d e 13. a b c d e 14. a b c d e

15. a b c d e 16. a b c d e 17. a b c d e 18. a b c d e 19. a b c d e

20. a b c d e 21. a b c d e 22. a b c d e 23. a b c d e 24. a b c d e

25. a b c d e

10. In a triangle ABC, Ð =A x , Ð =B y and 20Ð = +C y .If 4x – y = 10, then the triangle is :(a) Right-angled (b) Obtuse-angled(c) Equilateral (d) Cannot be determined(e) None of these

11. If the sides of a right triangle are x, x + 1 and x – 1, then itshypotenuse is :(a) 5 (b) 4(c) 1 (d) 0(e) None of these

12. Two circles touch each other internally. Their radii are 2 cmand 3 cm. The biggest chord of the outer circle which isoutside the inner circle is of length

(a) 2 2 cm (b) 3 2 cm

(c) 2 3 cm (d) 4 2 cm

(e) None of these

13. An angle is equal to 1

3rd of its supplement. Find its measure.

(a) 60° (b) 80°(c) 90° (d) 45°(e) None of these

14. An angle measuring 270° is an example of(a) acute angle (d) obtuse angle(c) right angle (d) reflex angle(e) None of these

15. Find the angle which is equal to its supplementary angle(a) 45° (b) 180°(c) 90° (d) 360°(e) None of these

16. Three angles of a quadrilaterals are in the ratio1 : 2 : 3. The sum of the least and the greatest of these anglesis equal to 180°. All the angles of the quadrilateral will be :(a) 30°, 90°, 45°, 60°(b) 45°, 90°, 135°, 90°(c) 120°, 150°, 210°, 360°(d) Cannot be determined(e) None of these

17. The number of degrees in four and one-third right angles is(a) 405 (b) 390(c) 395 (d) 400(e) None of these

18. Number of degrees in two and half right angles is__________ .(a) 245 (b) 225(c) 200 (d) 180(e) None of these

19. Number of degrees in five and two-third of a right angle is(a) 510 (b) 490(c) 486 (d) 480(e) None of these

20. An angle is two-third of its complement and one-fourth ofits supplement, then the angle is(a) 46º (b) 56º(c) 36º (d) 40º(e) None of these

21. The measures of the four angles of a quadrilateral are in theratio of 1 : 2 : 3 : 4.What is the measure of fourth angle?(a) 144º (b) 135º(c) 125º (d) 150º(e) None of these

22. Find the length of a chord which is at a distance of 3 cm fromthe centre of a circle of radius 5 cm(a) 8 cm (b) 10 cm(c) 12 cm (d) 6 cm(e) None of these

23. Two adjacent angles of a parallelogram are in the ratio 2 : 3.Find the measures of all angles(a) 72o, 108o, 72o, 108o

(b) 72o, 108o, 70o, 110o

(c) 80o, 100o, 80o, 100o

(d) Cannot be determined(e) None of these

24. If the angle of triangle are in the ratio of 4 : 3 : 2, then thetriangle(a) is obtuse angled triangle(b) has one angle greater than 80°(c) is a right triangle(d) is acute angled triangle(e) None of these

25. One of the angles of a parallelogram is 45°. What will be thesum of the larger angle and twice the smaller angle of theparallelogram ?(a) 228° (b) 224°(c) 225° (d) 222°(e) None of these

Page 107: yoursmahboob.wordpress.com SBI · yoursmahboob.wordpress.com iii P 101 Speed Tests for SBI Bank Clerk Exam 101 Speed Tests for SBI Bank Clerk Exam is revised and updated edition on

yoursmahboob.w

ordpress.com

DIRECTIONS (Qs. 1-5): What should come in place of questionmark (? ) in the following number series ?1. 121 117 108 92 67 ?

(a) 31 (b) 29(c) 41 (d) 37(e) None of these

2. 50 26 14 ? 5 3.5(a) 6 (b) 8(c) 10 (d) 12(e) None of these

3. 3 23 43 ? 83 103(a) 33 (b) 53(c) 63 (d) 73(e) None of these

4. 748 737 715 682 638 ?(a) 594 (b) 572(c) 581 (d) 563(e) None of these

5. 1 9 25 49 81 ? 169(a) 100 (b) 64(c) 81 (d) 121(e) None of these

DIRECTIONS (Qs. 6-10) : What should come in place of questionmark (?) in the following number series?

6. 36 20 ? 8 6 5(a) 10 (b) 12(c) 14 (d) 16(e) None of these

7. 668 656 632 584 ? 296(a) 392 (b) 438(c) 488 (d) 536(e) None of these

8. 1 121 441 961 1681 ?(a) 2701 (b) 2511(c) 2611 (d) 2801(e) None of these

RESPONSE

GRID

1. a b c d e 2. a b c d e 3. a b c d e 4. a b c d e 5. a b c d e

6. a b c d e 7. a b c d e 8. a b c d e 9. a b c d e 10. a b c d e

11. a b c d e 12. a b c d e 13. a b c d e 14. a b c d e 15. a b c d e

Max. Marks : 30 No. of Qs. 30 Time : 20 min. Date : ........./......../................

9. 9 49 201 1009 ? 20209 80841

(a) 4054 (b) 4049

(c) 4050 (d) 4041

(e) None of these

10. 31 35 44 60 85 ?

(a) 121 (b) 111

(c) 109 (d) 97

(e) None of these

DIRECTIONS (Q.11-15) : What will come in place of the questionmark (?) in the following number series?11. 7 9 12 16 ?

(a) 2 2 (b) 19(c) 20 (d) 21(e) None of these

12. 384 192 96 48 ?(a) 36 (b) 28(c) 24 (d) 32(e) None of these

13. 5 6 14 45 ?(a) 183 (b) 185(c) 138 (d) 139(e) None of these

14. 8 9 13 22 ?(a) 30 (b) 31(c) 34 (d) 36(e) None of these

15. 6 11 21 41 ?(a) 81 (b) 61(c) 71 (d) 91(e) None of these

4949Number Series - I

Page 108: yoursmahboob.wordpress.com SBI · yoursmahboob.wordpress.com iii P 101 Speed Tests for SBI Bank Clerk Exam 101 Speed Tests for SBI Bank Clerk Exam is revised and updated edition on

yoursmahboob.w

ordpress.com

100 SPEED TEST 49

RESPONSE

GRID

16. a b c d e 17. a b c d e 18. a b c d e 19. a b c d e 20. a b c d e

21. a b c d e 22. a b c d e 23. a b c d e 24. a b c d e 25. a b c d e

26. a b c d e 27. a b c d e 28. a b c d e 29. a b c d e 30. a b c d e

DIRECTIONS (Qs. 16-20): What will come in place of questionmark (?) in the following number series?16. 7 13 25 49 ?

(a) 99 (b) 97(c) 89 (d) 87(e) None of these

17. 5 6 10 19 ?(a) 28 (b) 37(c) 36 (d) 35(e) None of these

18. 8 9 20 63 ?(a) 256 (b) 252(c) 246 (d) 242(e) None of these

19. 11 13 16 20 ?(a) 24 (b) 26(c) 28 (d) 27(e) None of these

20. 608 304 152 76 ?(a) 39 (b) 36(c) 38 (d) 37(e) None of these

DIRECTIONS (Qs. 21-25) % In the following number series, a wrong

number is given. Find out that wrong number.

21. 2 11 38 197 1172 8227 65806

(a) 11 (b) 38(c) 197 (d) 1172

(e) 8227

22. 16 19 21 30 46 71 107

(a) 19 (b) 21

(c) 30 (d) 46(e) 71

23. 7 9 16 25 41 68 107 173

(a) 107 (b) 16(c) 41 (d) 68

(e) 25

24. 4 2 3.5 7.5 26.25 118.125(a) 118.125 (b) 26.25(c) 3.5 (d) 2(e) 7.5

25. 16 4 2 1.5 1.75 1.875(a) 1.875 (b) 1.75(c) 1.5 (d) 2(e) 4

DIRECTIONS (Q. 26-30) : In the following number series onlyone number is wrong. Find out the wrong number.26. 4 6 18 49 201 1011

(a) 1011 (b) 201

(c) 18 (d) 49

(e) None of these

27. 48 72 108 162 243 366

(a) 72 (b) 108

(c) 162 (d) 243

(e) None of these

28. 2 54 300 1220 3674 7350

(a) 3674 (b) 1220

(c) 300 (d) 54

(e) None of these

29. 8 27 64 125 218 343

(a) 27 (b) 218

(c) 125 (d) 343

(e) None of these

30. 19 68 102 129 145 154

(a) 154 (b) 129

(c) 145 (d) 102

(e) None of these

Page 109: yoursmahboob.wordpress.com SBI · yoursmahboob.wordpress.com iii P 101 Speed Tests for SBI Bank Clerk Exam 101 Speed Tests for SBI Bank Clerk Exam is revised and updated edition on

yoursmahboob.w

ordpress.com

DIRECTIONS (Qs.1-5) : What should come in place of thequestion mark (?) in the following number series ?1. 353 354 351 356 349 ?

(a) 348 (b) 358(c) 338 (d) 385(e) 340

2. 1 5 13 29 ? 125 253(a) 83 (b) 69(c) 61 (d) 65(e) 81

3. 45 57 81 117 165 ?(a) 235 (b) 215(c) 205 (d) 245(e) 225

4. 17 18 26 53 117 ? 458(a) 342 (b) 142(c) 257 (d) 262(e) 242

5.1 1 3 1 1 3

11 1 1 ?4 2 4 4 2 4

(a) 2 (b) 4

(c) 6 (d)1

15

(e)2

13

DIRECTIONS (Qs.6-10) : What should come in place of thequestion mark (?) in the following number series?6. 4 19 49 94 154 ?

(a) 223 (b) 225(c) 229 (d) 239(e) None of these

7.1 1 1

1 1 2 2 3 ?2 2 2

(a)1

32

(b)1

23

(c) 4 (d)1

34

(e) None of these

RESPONSE

GRID

1. a b c d e 2. a b c d e 3. a b c d e 4. a b c d e 5. a b c d e

6. a b c d e 7. a b c d e 8. a b c d e 9. a b c d e 10. a b c d e

11. a b c d e 12. a b c d e 13. a b c d e 14. a b c d e 15. a b c d e

Max. Marks : 30 No. of Qs. 30 Time : 20 min. Date : ........./......../................

8. 101 103 99 105 97 ? 95(a) 93 (b) 104(c) 108 (d) 107

(e) None of these

9. 3 219 344 408 ? 443 444

(a) 416 (b) 435(c) 423 (d) 428

(e) None of these

10. 7 10 16 28 52 ? 196(a) 100 (b) 90(c) 160 (d) 150(e) None of these

DIRECTIONS (Qs. 11-15) : What should come in place of thequestion mark (?) in the following number series ?11. 1 1 2 6 ? 120

(a) 24 (b) 60(c) 100 (d) 30(e) None of these

12. 7 8 16 43 ? 232(a) 204 (b) 107(c) 119 (d) 89(e) None of these

13. 4 13 17 ? 30 39(a) 29 (b) 21(c) 26 (d) 19(e) None of these

14. 982 977 952 827 822 ?(a) 779 (b) 817(c) 789 (d) 697(e) None of these

15. 41472 5184 576 72 8 ?(a) 0 (b) 9(c) 1 (d) 8(e) None of these

Number Series - II 505050

Page 110: yoursmahboob.wordpress.com SBI · yoursmahboob.wordpress.com iii P 101 Speed Tests for SBI Bank Clerk Exam 101 Speed Tests for SBI Bank Clerk Exam is revised and updated edition on

yoursmahboob.w

ordpress.com

102 SPEED TEST 50

RESPONSE

GRID

16. a b c d e 17. a b c d e 18. a b c d e 19. a b c d e 20. a b c d e

21. a b c d e 22. a b c d e 23. a b c d e 24. a b c d e 25. a b c d e

26. a b c d e 27. a b c d e 28. a b c d e 29. a b c d e 30. a b c d e

DIRECTIONS (Qs. 16-20) : What should come in place of thequestion mark (?) in the following number series?

16. 64 54 69 49 74 44 ?

(a) 89 (b) 69

(c) 59 (d) 99

(e) None of these

17. 4000 2008 1012 ? 265 140.5 78.25

(a) 506 (b) 514

(c) 520 (d) 512

(e) None of these

18. 5 5 15 75 ? 4725 51975

(a) 520 (b) 450

(c) 525 (d) 300

(e) None of these

19. 52 26 26 39 78 ? 585

(a) 195 (b) 156

(c) 234 (d) 117

(e) None of these

20. 29, 23, ?, 17, 13, 11, 7

(a) 19 (b) 21

(c) 23 (d) 27

(e) None of these

DIRECTIONS (Qs. 21 to 25): In each of these questions, a numberseries is given. In each series, only one number is wrong. Find outthe wrong number.

21. 3601 3602 1803 604 154 36 12

(a) 3602 (b) 1803 (c) 604(d) 154 (e) 36

22. 4 12 42 196 1005 6066 42511

(a) 12 (b) 42 (c) 1005(d) 196 (e) 6066

23. 2 8 12 20 30 42 56

(a) 8 (b) 42 (c) 30(d) 20 (e) 12

24. 32 16 24 65 210 945 5197.5

(a) 945 (b) 16 (c) 24(d) 210 (e) 65

25. 7 13 25 49 97 194 385

(a) 13 (b) 49 (c) 97(d) 194 (e) 25

DIRECTIONS (Qs. 26 - 30) :In each of these questions, a numberseries is given. In each series, only one number is wrong number.Find out the wrong number.

26. 16 19 21 30 46 71 107

(a) 19 (b) 21 (c) 30(d) 46 (e) 71

27. 7 9 16 25 41 68 107 173

(a) 107 (b) 16 (c) 41(d) 68 (e) 25

28. 32 16 24 65 210 945 5197.5

(a) 945 (b) 16 (c) 24(d) 210 (e) 65

29. 850 600 550 500 475 462.5 456.25

(a) 600 (b) 550 (c) 500(d) 4625 (e) None of these

30. 8 12 24 46 72 108 216

(a) 12 (b) 24 (c) 46(d) 72 (e) None of these

Page 111: yoursmahboob.wordpress.com SBI · yoursmahboob.wordpress.com iii P 101 Speed Tests for SBI Bank Clerk Exam 101 Speed Tests for SBI Bank Clerk Exam is revised and updated edition on

yoursmahboob.w

ordpress.com

DIRECTIONS (Q. 1-5): Study the following graph carefully to answer these questions.Quantity of Various Items Sold and Price per kg

0

5

10

15

20

25

30

A B C D E FItems

0

10

20

30

40

50

60

Price in per kg`Quantity sold in quintals

Quantity

Pri

ce

RESPONSE

GRID

1. a b c d e 2. a b c d e 3. a b c d e 4. a b c d e 5. a b c d e

6. a b c d e 7. a b c d e

Max. Marks : 20 No. of Qs. 20 Time : 20 min. Date : ........./......../................

DIRECTIONS (Qs. 6 - 10): Study the following graph carefullyand answer the questions given below it.

Number of Students Studying in Various Colleges fromVarious Faculties (Number in Thousands)

51.2

65

44

30

40

50

30

56

36.5 33

60

25

0

10

20

30

40

50

60

70

80

H I J KColleges

Arts

Science

Commerce

Nu

mbe

r of

Stu

den

ts

6. What is the difference between the total number of studentsstudying in college H and those studying in college K ?(a) 16100 (b) 15800 (c) 16300(d) 16700 (e) None of these

7. What is the total number of students studying in all thecolleges together ?(a) 520900 (b) 520700 (c) 610200(d) 510800 (e) None of these

1. If the quantity sold of item D increased by 50% and theprice reduced by 10%. What was the total value of thequantity sold for item D ?(a) ` 675 (b) ` 6750 (c) `67550(d) ` 67500 (e) None of these

2. Approximately, what is the average price per kg of items A,B & C ?(a) ` 9.50 (b) ` 8 (c) ` 7.50(d) ` 9 (e) ` 10.50

3. What is the ratio between the total values of quantity soldfor items E & F respectively ?(a) 15 : 14 (b) 3 : 2 (c) 5 : 7(d) 7 : 5 (e) None of these

4. Total value of the quantity sold for item C is what per centof the total value of the quantity sold for item E ?(a) 111 (b) 85 (c) 90(d) 87.5 (e) None of these

5. If the price as well as the quantity sold is increased by 20%for item A, what is the total value of quantity sold for itemA?(a) ` 48500 (b) ` 49000 (c) ` 42000(d) ` 50400 (e) None of these

5151Data Interpretation

Page 112: yoursmahboob.wordpress.com SBI · yoursmahboob.wordpress.com iii P 101 Speed Tests for SBI Bank Clerk Exam 101 Speed Tests for SBI Bank Clerk Exam is revised and updated edition on

yoursmahboob.w

ordpress.com

104 SPEED TEST 51

RESPONSE

GRID

8. a b c d e 9. a b c d e 10. a b c d e 11. a b c d e 12. a b c d e

13. a b c d e 14. a b c d e 15. a b c d e 16. a b c d e 17. a b c d e

18. a b c d e 19. a b c d e 20. a b c d e

8. What is the respective ratio of the students from the facultyof Science from colleges H and I together to the studentsfrom the same faculty from colleges J and K together ?(a) 43 : 45 (b) 41 : 43 (c) 45 : 43(d) 43 : 41 (e) None of these

9. The number of students from the faculty of Science fromcollege I are approximately what per cent of the total numberof students studying in that college ?(a) 34 (b) 37 (c) 29(d) 31 (e) 39

10. What is the average number of students from the faculty ofCommerce from all the colleges together ?(a) 36825 (b) 38655 (c) 35625(d) 36585 (e) None of these

DIRECTIONS (Qs. 11-15) : Study the following Pie-chart carefullyto answer these questions.Percentagewise Distribution of Teachers who Teach six DifferentSubjects

Total Number of Teachers = 2000Percentage of Teachers

English7%

History27%

Mathematic14%

sPhysics15%

Economics25%

Biology12%

11. If five-seventh of the teachers who teach Mathematics arefemale, then number of male Mathematics teachers isapproximately what percentage of the total number ofteachers who teach English ?(a) 57 (b) 42 (c) 63(d) 69 (e) 51

12. What is the difference between the total number of teacherswho teach English and History together and the total numberof teachers who teach Mathematics and Biology together ?(a) 146 (b) 156 (c) 180(d) 160 (e) None of these

13. If the percentage of Biology teachers is increased by 40 percent and percentage of History teachers decreased by 20per cent then what will be the total number of Biology andHistory teachers together ?

(a) 634 (b) 654 (c) 658(d) 778 (e) None of these

14. What is the approximate average number of teachers teachingEconomics, History and Biology together ?(a) 400 (b) 420 (c) 450(d) 480 (e) 470

15. What is the respective ratio of the number of teachers whoteach Biology and the number of teachers who teach Physics?(a) 6 : 7 (b) 4 : 7 (c) 3 : 5(d) 4 : 5 (e) None of these

DIRECTIONS (Q. 16-20) : Study the following table carefully toanswer the questions that follow.

Number of flights cancelled by five differentairlines in six different years

Airline Year

P Q R S T

2005 240 450 305 365 640 2006 420 600 470 446 258 2007 600 680 546 430 610 2008 160 208 708 550 586 2009 140 640 656 250 654 2010 290 363 880 195 483

16. What was the difference between the highest number offlights cancelled by airline - Q and the lowest number offlights cancelled by airline-T out of all the six years ?(a) 446 (b) 456 (c) 432(d) 442 (e) None of these

17. What was the approximate percentage increase in numberof flights cancelled by airline-S in the year 2008 as comparedto previous year ?(a) 127 (b) 27 (c) 150(d) 45 (e) 117

18. What was the average number of flights cancelled by theairlines P, R, S and T in the year 2008 ?(a) 551.5 (b) 501 (c) 405(d) 442.4 (e) None of these

19. In 2010, 40% flights are cancelled by airline-R due to badweather and technical fault. How many flights are cancelledby airline-R due to technical fault ?(a) 528 (b) 568 (c) 468(d) 548 (e) None of these

20. What is the approximate percentage of cancelled flights byairline’s-P and R in 2007 compared to cancelled flights byairline-S in 2005 ?(a) 356 (b) 280 (c) 265(d) 340 (e) 314

Page 113: yoursmahboob.wordpress.com SBI · yoursmahboob.wordpress.com iii P 101 Speed Tests for SBI Bank Clerk Exam 101 Speed Tests for SBI Bank Clerk Exam is revised and updated edition on

yoursmahboob.w

ordpress.com

RESPONSE

GRID

1. a b c d e 2. a b c d e 3. a b c d e 4. a b c d e 5. a b c d e

6. a b c d e 7. a b c d e 8. a b c d e 9. a b c d e 10. a b c d e

11. a b c d e 12. a b c d e 13. a b c d e 14. a b c d e 15. a b c d e

16. a b c d e 17. a b c d e 18. a b c d e 19. a b c d e

Max. Marks : 40 No. of Qs. 40 Time : 25 min. Date : ........./......../................

11. Cost of 12 belts and 30 wallets is Rs 8940. What is the cost of4 belts and 10 wallets?(a) Rs 2890 (b) Rs 2980 (c) Rs 2780(d) Rs 2870 (e) None of these

12. Ghanshyam purchased an article for Rs 1850. At what priceshould he sell it so that 30% profit is earned?(a) Rs 2450 (b) Rs 2245 (c) Rs 2405(d) Rs 2425 (e) None of the above

13. What is the compound interest accrued on an amount of Rs8500 in two years @ interest 10% per annum?(a) Rs 1875 (b) Rs 1885 (c) Rs 1775(d) Rs 1765 (e) None of these

14. A train running at the speed of 60 kmph crosses a 200 m longplatform in 27 s. What is the length of the train ?(a) 250 m (b) 200 m (c) 240 m(d) 450 m (e) None of these

15. 10 men can complete a piece of work in 8 days. In how manydays can 16 men complete that work?(a) 4 days (b) 5 days (c) 6 days(d) 3 days (e) None of these

16. Find the average of following set of numbers.76, 48, 84, 66, 70, 64(a) 72 (b) 66 (c) 68(d) 64 (e) None of these

17. If the numerator of a certain fractions increased by 100% andthe denominator is increased by 200%; the new fraction thus

formed is 4

21. What is the original fraction?

(a)2

7(b)

3

7(c)

2

5

(d)47

(e) None of these

18. In how many different ways can the letters of the word'SIMPLE' be arranged?(a) 520 (b) 120 (c) 5040(d) 270 (e) None of these

19. The ratio of the ages of A and B seven years ago was 3 : 4respectively. The ratio of their ages nine years from nowwill be 7 : 8 respectively. What is B’s age at present ?(a) 16 years (b) 19 years (c) 28 years(d) 23 years (e) None of these

DIRECTIONS (Q.1-10) : What will come in place of questionmark (?) in the following questions ?1. 48% of 525 + ? % of 350 = 399

(a) 42 (b) 46 (c) 28(d) 26 (e) None of these

2.37

of 45

of 58

of 490 = ?

(a) 115 (b) 105 (c) 108(d) 116 (e) None of these

3. ? + 172 = 335(a) 46 (b) 42 (c) 1764(d) 2116 (e) None of these

4. 125% of 560 + 22% of 450 = ?(a) 799 (b) 700 (c) 782(d) 749 (e) None of these

5. 2 228 5 15 6 ?

7 256 (13)´ - ´ =

+ +

(a)27

115(b)

22117

(c)25

117

(d)22

115(e) None of these

6. 18.76 + 222.24 + 3242.15 = ?(a) 3384.15 (b) 3483.15 (c) 3283.25(d) 3383.25 (e) None of these

7. 784 ÷ 16 ÷ 7 = ?(a) 49 (b) 14 (c) 21(d) 7 (e) None of these

8.32

of 455 + 58

of 456 = ?

(a) 448 (b) 476 (c) 480(d) 464 (e) None of these

9. 1.05% of 2500 + 2.5% of 440 = ?(a) 37.50 (b) 37.25 (c) 370.25(d) 372.50 (e) None of these

10. 4900 ÷ 28 × 444 ÷ 12 = ?(a) 6575 (b) 6475 (c) 6455(d) 6745 (e) None of these

5252Section Test :Quantitative Aptitude

Page 114: yoursmahboob.wordpress.com SBI · yoursmahboob.wordpress.com iii P 101 Speed Tests for SBI Bank Clerk Exam 101 Speed Tests for SBI Bank Clerk Exam is revised and updated edition on

yoursmahboob.w

ordpress.com

106 SPEED TEST 52

RESPONSE

GRID

20. a b c d e 21. a b c d e 22. a b c d e 23. a b c d e 24. a b c d e

25. a b c d e 26. a b c d e 27. a b c d e 28. a b c d e 29. a b c d e

30. a b c d e 31. a b c d e 32. a b c d e 33. a b c d e 34. a b c d e

35. a b c d e 36. a b c d e 37. a b c d e 38. a b c d e 39. a b c d e

40. a b c d e

20. The sum of three consecutive odd numbers is 1383. Whatis the largest number ?(a) 463 (b) 49 (c) 457(d) 461 (e) None of these

DIRECTIONS (Q. 21-25) : What should come in place of ques-tion mark (?) in the following number series?21. 8 52 ? 1287 4504.5 11261.25 16891.875

(a) 462 (b) 286 (c) 194(d) 328 (e) None of these

22. 3 42 504 ? 40320 241920 967680(a) 6048 (b) 5544 (c) 4536(d) 5040 (e) None of these

23 403 400 394 382 358 310 ?(a) 244 (b) 210 (c) 214(d) 256 (e) None of these

24. 7 8 4 13 –3 22 ?(a) –7 (b) –10 (c) –12(d) –14 (e) None of these

25. 250000 62500 12500 3125 625 ? 31.25(a) 156.25 (b) 172.25 (c) 125(d) 150 (e) None of these

26. Out of the fractions 1 7 3 5

, , ,2 8 4 6

and 67

, what is the differ-

ence between the largest and the smallest fraction?

(a)7

13(b)

3

8(c)

4

7

(d)1

6(e) None of these

27. What will come in place of both questions marks(?) in thefollowing equation?

0.6

1.4

(?) 26

104 (?)=

(a) 58 (b) –48 (c) –56(d) 42 (e) –52

28. The perimeter of a square is thrice the perimeter of a rectange.If the perimeter of the square is 84 cm and the length of therectangel is 8 cm, what is the difference between the breadthof the rectangle and the sidce of the square?(a) 15 cm (b) 19 cm (c) 10 cm(d) 8 cm (e) None of these

29. The area of a circle is equal to the area of a rectangel withperimeter equal to 42 m and breadth equal to 8.5 m. What isthe area of the circle?(a) 116.25 sq m (b) 104.25 sq m (c) 146.25 sq m(d) 128.25 sq m (e) None of these

30. The product of 5% of a positive number and 3% of the samenumber is 504.6 What is half of that number?(a) 290 (b) 340 (c) 680(d) 580 (e) None of these

31. 4 women and 12 children together take four days to completea piece of work. How many days will four children alone taketo complete the piece of work if two women alone cancomplete the piece of work in 16 days?(a) 32 (b) 24 (c) 16(d) 12 (e) None of these

32. The average of four consecutive odd number A, B, C and Drespectively is 40. What is the product of B and D?(a) 1599 (b) 1591 (c) 1763(d) 1677 (e) None of these

33. Anu walks 2.31 km in three weeks by walking an equaldistance each day. How many metres does she walk eachday?(a) 110 m (b) 90 m (c) 140 m(d) 120 m (e) None of these

34. A man riding a bicycle completes one lap of a square fieldalong its perimeter at the speed of 43.2 km/hr in 1 minute 20seconds. What is the area of the field?(a) 52900 sq m (b) 57600 sq m (c) 48400 sq m(d) Can’t be determined(e) None of these

35. On Teacher’s Day, 4800 sweets were to be equally distributedamong a certain number of children. But on that particularday 100 children were absent. Hence, each child got foursweets extra. How many children were originally supposedto be there?(a) 300 (b) 400 (c) 540(d) 500 (e) Can’t be determined.

36. The ratio of the monthly oncomes of Sneha, Tina and Akrutiis 95:110:116. If Sneha’s annual income is 3,42,000, what isAkruit’s annual income?(a) `3,96,900 (b) `5,63,500 (c) `4,17,600(d) `3,88,000 (e) None of these

37. A truck covers a distance of 256 km at the speed of 32 km/hr.What is the average speed of a car which travels a distanceof 160 km more than the truck in the same time?(a) 46 kmh–1 (b) 52 kmh–1 (c) 49 kmh–1

(d) 64 kmh–1 (e) None of these38. In an examination, the maximum aggregate marks is 1020. In

order to pass the exam a student is required to obtain 663marks out of the aggregate marks. Shreya obtained 612marks. By what per cent did Shreya fail the exam?(a) 5% (b) 8% (c) 7%(d) Can’t be determined(e) None of these

39. The average height of 21 girls was recorded as 148 cm. Whenthe teacher’s height was included, the average of theirheights increased by 1 cm. What was the height of theteacher?(a) 156 cm (b) 168 cm (c) 170 cm(d) 162 cm (e) None of these

40. What would be the area of a circle whose diameter is 35 cm?(a) 962.5 sq cm (b) 875.5 sq cm (c) 981.5 sq cm(d) 886.5 sq cm (e) None of these

Page 115: yoursmahboob.wordpress.com SBI · yoursmahboob.wordpress.com iii P 101 Speed Tests for SBI Bank Clerk Exam 101 Speed Tests for SBI Bank Clerk Exam is revised and updated edition on

yoursmahboob.w

ordpress.com

DIRECTIONS : Read the following passages carefully and answerthe questions given below it. Certain words are printed in bold tohelp you to locate them while answering some of the questions.

Passage 1We find that today the unity and integrity of the nation isthreatened by the divisive forces of regionalism, linguism andcommunal loyalties which are gaining ascendancy in nationallife and seeking to tear apart and destroy national integrity. Wetend to forget that India is one nation and we are all Indians firstand Indians last. It is time we remind ourselves what the greatvisionary and builder of modern India Jawaharlal Nehru said,“Who dies if India lives, who lives if India dies?” We must realise,and this is unfortunately what many in public life tend to overlook,sometimes out of ignorance of the forces of history andsometimes deliberately with a view to promoting their self-interest, that national interest must inevitably and forever prevailover any other considerations proceeding from regional, linguisticor communal attachments. The history of India over the pastcenturies bears witness to the fact that India was at no time asingle political unit. Even during the reign of the Maurya dynasty,though a large part of the country was under the sovereignty ofthe Mauryan kings, there were considerable portions of theterritory which were under the rule of independent kingdoms. Soalso during the Mughal rule which extended over large parts ofthe territory of India, there were independent rulers who enjoyedpolitical sovereignty over the territories of their respectivekingdoms. It is an interesting fact of history that India was forgedinto a nation, neither on account of a common language nor onaccount of the continued existence of a single political regimeover its territories but on account of a common culture evolvedover the centuries. It is cultural unity—something morefundamental and enduring than any other bond which may unitethe people of a country together which has welded this countryinto a nation. But until the advent of the British rule, it was notconstituted into a single political unit. There were, throughoutthe period of history for which we have fairly authenticatedaccounts, various kingdoms and principalities which wereoccasionally engaged in conflict with one another. During theBritish rule, India became a compact political unit having onesingle political regime over its entire territories and this led to theevolution of the concept of a nation. This concept of one nationtook firm roots in the minds and hearts of the people during thestruggle for independence under the leadership of Mahatma

RESPONSE

GRID

1. a b c d e 2. a b c d e 3. a b c d e 4. a b c d e

Max. Marks : 30 No. of Qs. 30 Time : 20 min. Date : ........./......../................

Gandhi. He has rightly been called the Father of the Nation becauseit was he who awakened in the people of this country a sense ofnational consciousness and instilled in them a high sense ofpatriotism without which it is not possible to build a country intonationhood. By the time the Constitution of India came to be enacted,insurgent India, breaking a new path of non-violent revolution andfighting to free itself from the shackles of foreign domination, hademerged into nationhood and “the people of India” were inspired bya new enthusiasm, a high and noble spirit of sacrifice and above all,a strong sense of nationalism and in the Constitution which theyframed. They set about the task of a strong nation based on certaincherished values for which they had fought.1. The author has quoted Jawaharlal Nehru to emphasise the

point that(a) national interest must enjoy supreme importance(b) India is going to survive even if the world is under the

spell of destruction(c) the world will be destroyed if India is on the threshold of

destruction(d) the survival of the world depends only upon the well

being of India(e) None of these

2. What, according to the author, is the impact of the divisiveforces on our nation?(a) They promote a sense of regional pride.(b) They help people to form linguistic groups.(c) They separate groups of people and create enmity among

them.(d) They encourage among people the sense of loyalty to

their community.(e) They remind us of our national pride.

3. “Communal loyalties” have been considered by the author as(a) a good quality to be cherished(b) of no consequence to the nation(c) a very important aspect for nation-building(d) a threat to the solidarity of the nation(e) None of these

4. Which of the following was instrumental in holding thedifferent people of India together?(a) A common national language(b) A common cultural heritage(c) The endurance level of the people(d) Fundamentalist bent of mind of the people(e) None of these

5353ReadingComprehension - I

Page 116: yoursmahboob.wordpress.com SBI · yoursmahboob.wordpress.com iii P 101 Speed Tests for SBI Bank Clerk Exam 101 Speed Tests for SBI Bank Clerk Exam is revised and updated edition on

yoursmahboob.w

ordpress.com

108 SPEED TEST 53

RESPONSE

GRID

5. a b c d e 6. a b c d e 7. a b c d e 8. a b c d e 9. a b c d e

10. a b c d e 11. a b c d e 12. a b c d e 13. a b c d e 14. a b c d e

15. a b c d e

5. The passage appears to have been written with the purpose of(a) giving a piece of advice to politicians of free India(b) assessing the patriotic values and sacrifices made by

people for India’s freedom(c) justifying the teaching of Mahatma Gandhi and its

impact on the people(d) giving a historical account of how India evolved as a

nation(e) None of these

6. History shows that India, which was not a political unitearlier, became so(a) during the reign of Maurya dynasty(b) during the Mughal rule(c) after one-national-language policy was adopted(d) during the regime of independent rulers(e) during the British rule

7. Why do people tend to overlook the paramount importanceof national interest?(A) Because they are unaware of the imperative need of

the day(B) Because they give undue importance to their selfish

motives(C) Because historical events force them to do so(a) Only A (b) Only B(c) Only C (d) A and B only(e) B and C only

8. The “people of India”, as highlighted by the author in the lastsentence of the passage, refer to(a) the people of one unified nation(b) the subjects of several independent rulers(c) the patriots who sacrificed themselves in the freedom

struggle(d) the people who were instrumental in writing the

Constitution(e) None of these

9. India’s insurgence was for(a) breaking the path of non-violence(b) having one common national language(c) insisting on a unique cultural identity(d) several independent sovereign rulers(e) None of these

10. Transformation of our country into nationhood was possiblebecause of(A) People’s spontaneously referring to Mahatma Gandhi

as the Father of the Nation(B) People’s sense of national consciousness(C) Generation of a high sense of dedication to the nation

among the people(a) A and B only (b) A and C only(c) B and C only (d) All the three(e) None of these

DIRECTIONS (Qs. 11-13): Choose the word/group of words whichis most nearly the SAME in meaning as the word given in bold asused in the passage.11. awakened

(a) moved (b) segregated(c) extracted (d) kindled(e) supported

12. cherished(a) maintained carefully(b) available abundantly(c) managed tactfully(d) accepted happily(e) protected lovingly

13. authenticated(a) established (b) documented(c) hearsay (d) audited(e) maintained

DIRECTIONS (Qs. 14 & 15) : Choose the word which is mostOPPOSITE in meaning of the word given in bold as used in thepassage.14. considerable

(a) inconsiderate (b) uncountable(c) unfathomable (d) irresolute(e) negligible

15. deliberately(a) reluctantly (b) unintentionally(c) unauthorisedly (d) wrongly(d) notoriously

Passage - 2In a reversal of the norm elsewhere, in India policymakers andeconomists have become optimists while bosses do the worrying.The country’s Central Bank has predicted that the country’seconomy is likely to grow at a double digit rate during the next 20-30 years. India has the capability with its vast labour and laudedentrepreneurial spirit. But the private sector which is supposed todo the heavy lifting that turns India from the world’s tenth largesteconomy to its third largest by 2030 has become fed up. Businesspeople often carp about India’s problems but their irritation thistime has a nervous edge. In the first quarter of 2011, GDP grew atan annual rate of 7.8 percent; in 2005-07 it managed 9-10 percent.The economy may be slowing naturally as the low interest ratesand public spending that got India through the global crisis arebelatedly withdrawn. At the same time the surge in inflation causedby exorbitant food prices has spread more widely, casting doubtover whether India can grow at 8-10 percent in the medium termwithout overheating.In India, as in many fast growing nations, the confidence to investdepends on the conviction that the long term trajectory is intactand it is that which is in doubt. Big Indian firms too sometimesseem happier to invest abroad than at home, in deals that areoften hailed as symbols of the country’s growing clout butsometimes speak to its weaknesses – purchases of naturalresources that India has in abundance but struggles to get out ofthe ground. In fact a further dip in investment could be self

Page 117: yoursmahboob.wordpress.com SBI · yoursmahboob.wordpress.com iii P 101 Speed Tests for SBI Bank Clerk Exam 101 Speed Tests for SBI Bank Clerk Exam is revised and updated edition on

yoursmahboob.w

ordpress.com

109SPEED TEST 53fulfilling: if fewer roads, ports and factories are built, this will hurtboth short term growth figures and reduce the economy’s longterm capacity.There is a view that because a fair amount of growth is assuredthe government need not try very hard. The liberalization reformsthat began in 1991 freed markets for products and gave rise tovibrant competition, at the same time what economists call factormarkets, those for basic inputs like land, power, labour etc remainunreformed and largely under state control, which createsdifficulties. Clearances today can take three to four years andmany employers are keen to replace workers with machinesdespite an abundance of labor force. This can be attributed tolabor laws which are inimical to employee creation and aneducation system that means finding quality manpower a majorproblem. In fact the Planning Commission concluded that evenachieving 9 percent growth will need marked policy action inunreformed sectors. Twenty years age it was said that yardstickagainst which India should be measured was its potential and itis clear that there remains much to do.16. Which of the following can be said about the Indian

economy at present?(a) It can comfortably achieve double digit growth rare at

present.(b) High food prices have led to overheating of the

economy.(c) Citizens are affluent owing to laxity in regulation.(d) Private sector confidence in India’s growth potential

is high.(e) Unreformed sectors are a drag on economic growth.

17. Why are employers reluctant to hire Indian labour force?(1) India’s labour force is overqualified for the

employment opportunities available.(2) High attrition rate among employees stemming from

their entrepreneurial spirit.(3) Labour laws are not conducive to generating

emploment.(a) Only (3)(b) All (1), (2) and (3)(c) Only (1) and (3)(d) Only (1) and (2)(e) None of these

18. What is the state of India’s basic input sectors at present?(a) These sectors attract Foreign Direct Investment

because of their vast potential.(b) These sectors are lagging as projects are usually

awarded to foreign companies.(c) These sectors are stagnating and badly in need of

reforms.(d) These sectors are well regulated as these are governed

by the State.(e) None of these

19. What is the author’s main objective in writing the passage?(a) Showcasing the potential of India’s growth potential to

entice foreign investors .(b) Exhorting India to implement measures to live up to its

potential.(c) Recommending India’s model of development to other

developing countries(d) Berating the private sector for not bidding for

infrastructure development projects.(e) Criticising the measures taken by India during the global

economic crisis.20. What impact has the GDP growth of 7.8 percent had?

(1) Indian Industry is anxious about India’s economicgrowth.

(2) India has achieved status as the world’s third largesteconomy at present.

(3) Foreign investment in India has drastically increased.(a) Only (1)(b) All (1), (2) and (3)(c) Only (1) and (3)(d) Only (1) and (2)(e) None of these

21. Which of the following is most similar in meaning to theword CLOUT given in bold as used in the passage?(a) Strike(b) Standing(c) Force(d) Launch(e) Achieve

22. Which of the following is most opposite in meaning to theword MARKED given in bold as used in the passage?(a) Decreased(b) Ignored(c) Clear(d) Assessed(e) Imperceptible

23. What measures do experts suggest be taken to ensure targetedeconomic growth?(a) Loweing of interest rates to help industries hit by

recession.(b) Prolonged financial support for basic input industries.(c) Incentives to Indian companies to invest in infrastucture.(d) Formulation of policies and their implementation in factor

markets(e) Stringent implementation of licensing system.

Passage - 3

In many countries, a combustible mixture of authoritarianism,unemployment and youth has given rise to disaffection withstrongmen rulers which has in turn spill over into uprising. Youngpeople in these countries are far better educated than their parentswere. In 1990 the average Egyptian had 4.4 years of schooling; by2010 the figure had risen to 7.1 years. Could it be that education, bymaking people less willing to put up with restrictions on freedom

RESPONSE

GRID

16. a b c d e 17. a b c d e 18. a b c d e 19. a b c d e 20. a b c d e

21. a b c d e 22. a b c d e 23. a b c d e

Page 118: yoursmahboob.wordpress.com SBI · yoursmahboob.wordpress.com iii P 101 Speed Tests for SBI Bank Clerk Exam 101 Speed Tests for SBI Bank Clerk Exam is revised and updated edition on

yoursmahboob.w

ordpress.com

110 SPEED TEST 53and more willing to question authority, promotes democratization.Ideas about the links between education, Income and democracyare at the heart of what social scientists have long studied. Sincethen plenty of economists and political scientists have looked forstatistical evidence of a causal link between education anddemocratization. Many have pointed to the strong correlation thatexists between levels of education and measures like the pluralismof party politics and the existence of civil liberties. The patternsare similar when income and democracy are considered. There areoutliers, of course – until recently, many Arab countries managed tocombine energy-based wealth and decent education withundemocratic political systems. But some deduce from the overallpicture that as China and other authoritarian states get more educatedand richer, their people will agitate for greater political freedom,culminating in a shift to a more democratic form of government.This apparently reasonable intuition is shakier than it seems. Criticsof the hypothesis point out that correlation is hardly causation.The general trend over the past half century may have been towardsrising living standards, a wider spread of basic education and moredemocracy, but it is entirely possible that this is being by anothervariable. Even if the correlation were not spurious, it would be difficultto know which way causation ran. Does more education lead togreater democracy? Or are more democratic countries better ateducating their citizens? A recent NBER paper compared a group ofKenyan girls in 69 primary school whose students were randomlyselected to receive a scholarship with similar students in schoolswhich received no such financial aid. Previous studies has shownthat the scholarship programme led to higher test scores andincreased the likelihood that girls enrolled in secondary school.Overall, it significantly increased the amount of education obtained.For the new study the authors tried to see how the extra schoolinghad affected the political and social attitudes of the women inquestion. Findings suggested that education may make people moreinterested in improving their own lives but they may not necessarilysee democracy as the way to do it. Even in established democracies,more education does not always mean either more active politicalparticipation or greater faith in democracy. Poorer and less educatedpeople often vote in larger numbers than their more educatedcompatriots, who often express disdain for the messiness ofdemocracy, yearning for the kind of government that would dealstrongly with the corrupt and build highways, railway lines andbridges at a dizzying pace of authoritarian China.24. Which of the following most aptly describes the central

theme of the passage?(a) Democratic nations are richer and have a better track

record of educating their citizens.(b) Education does not necessarily lead to greater

enthusiasm for a democratic form of government(c) Educated societies with autocratic form of government

enjoy a better quality of life than democracies.(d) Citizens can fulfill their personal aspirations only under

a democratic form of government.(e) Democracy makes citizens more intolerant as it does

not restrict personal freedoms

25. Which of the following is most similar in meaning to theword PROMOTES given in bold as used in the passage?(a) Up grades (b) Prefers(c) Recommends (d) Advocates(e) Publicizes

26. What conclusion can be drawn from the statistics cited aboutEgypt’s education system?(a) Job prospects have been on the rise in Egypt in recent

times.(b) Authoritarian leaders have played a vital role in

reforming Egypt’s education system.(c) Egypt has one of the youngest and best educated

demographies in the world.(d) Egypt is likely to be successful vibrant democracy.(e) There has been a rise in education levels in Egypt in

recent times.27. In the context of the passage which of the following

characterize (s) democracies?(1) Active participation of majority of educated citizens in

electoral process.(2) Fast paced economic growth and accountability of those

in power.(3) Better standards of living and access to higher education.(a) All (1), (2) and (3) (b) Only (2) and (3)(c) Only (3) (d) Only (1) and (2)(e) None of these

28. What according to the author has led to uprisings inauthoritarian countries?(a) Lack of access to education.(b) Vast numbers of uneducated and unemployable youth.(c) Frustration with the existing system of governance.(d) Unavailability of natural energy resources like coal and

oil.(e) Government’s overambitious plans for development.

29. Which of the following is/are true about China in the contextof the passage?(1) China’s citizens are in favor of a more representative

form of government.(2) China has made huge strides in infrastructure

developments.(3) China is in the midst of a political revolution.(a) None (b) Only (1)(c) Only (1) and (3) (d) Only (2)(e) All (1), (2) and (3)

30. What does the phrase “messiness of democracy” conveyin the context of the passage?(a) Democratic nations are chaotic on account of individual

freedoms.(b) Most democratic countries frequently have violent

revolts among their citizens.(c) The divide between the poor and educated is growing

wider in democracies.(d) High levels of pollution on account of frantic pace of

infrastructure development.(e) Resigned acceptance of intrinsic corruption in the

education system.

RESPONSE

GRID

24. a b c d e 25. a b c d e 26. a b c d e 27. a b c d e 28. a b c d e

29. a b c d e 30. a b c d e

Page 119: yoursmahboob.wordpress.com SBI · yoursmahboob.wordpress.com iii P 101 Speed Tests for SBI Bank Clerk Exam 101 Speed Tests for SBI Bank Clerk Exam is revised and updated edition on

yoursmahboob.w

ordpress.com

DIRECTIONS (Qs. 1-9): Read the following passage carefullyand answer the questions given below it. Certain words/phrasesin the passage are printed in bold to help you locate them whileanswering some of the questions.

In a country where consumers have traditionally had a rawdeal, the Consumer Protection Act was one of the mostprogressive acts of legislation introduced in 1986. Before this, ashop could get away easily with the line “goods once sold willnot be taken back or exchanged” or a car parking contractor with“park at your own risk”. It is not that things have changed nowbut at least a legislation is in place and a forum is available toseek redressal . One of the basic limitations of this act is itsmystification and general ignorance. No consumer agency orgroup has made its provisions general, nor has any redressalcommission or forum. Restricted as it is by a lack of in frastructureand personnel and great verdicts to encourage consumers. Thelegislation is comprehensive. It gives consumers the right toredress against defective goods, deficient services and unfairtrade practices. Consumer courts must deliver their judgementswithin 40 days, but rarely is this deadline adhered to. Thisreviewer had a first-hand experience of the chairman of a consumercourt in Delhi who adjourned a case against a foreign airline fortwo years on the grounds that he did not have staff to type theorders. His replacement found the backlog so shocking that hedismissed several cases without applying his mind, in the processworking against the interests of consumers. But what is moreimportant is that the law has it that a consumer can approachcourt on his own without having to pay legal fees. In practice,this does not happen. The chairperson of the NationalCommission, who is a sitting judge, is so attuned to deliveringjudgments which can stand scrutiny in a civil court of law that itis insisted upon that a consumer must be represented by a lawyer.If not, cases are adjourned with impunity and set for anotherday. Girimaji’s attempt is creditable in that it is the first of its kindand has addressed almost all possible angles. She has discussedredressals in complaints about housing, basic telephony, railtransportation, power supply, life insurance and medicalnegligence. There are even tips on how to file a complaint. But itis mired in the case files of the National/ State Commissions ofthe Consumer Forum. A useful dimension would have been acomparison with the Law of Torts practised abroad. It is necessaryhere also, especially in an era of economic liberalisation, when theconsumer is like ly to be swept off his feet by free-market forces.

RESPONSE

GRID1. a b c d e 2. a b c d e 3. a b c d e 4. a b c d e

Max. Marks : 30 No. of Qs. 30 Time : 20 min. Date : ........./......../................

1. Why is the consumer likely to be swept off his feet?(a) He is easily taken in by the deceptive publicity.(b) He is wooed by the charm of foreign brands readily

available in the market.(c) He is not aware of the Law of Torts as practised abroad.(d) He is not aware of the benefits of the consumer rights.(e) The Consumer Protection Act has been implemented andhe can seek redressal.

2. What does ‘lack of... verdicts’ imply?(a) A lack of the basis of the system, trained staff and

decisions based on fact(b) A paucity of funds, jury and judgement(c) A lack of resources, employees and final decision based

on facts(d) Not having the required manpower, economy and

decisive ruling(e) None of these

3. Which of the following statements is/are true?A. Girimaji’s attempt is comprehensive but could have done

with an angle or two more.B. Though the Act allows the consumer to approach the

court on his own, yet a lawyer to represent him is insistedupon.

C. Despite the Act, much remains the same.(a) Only A and C (b) Only A and B(c) Only B and C (d) Only B and C(e) None of these

4. What does the author mean by ‘mystification of the Act’?(a) The mysterious Act is yet to be resolved.(b) The consumer is wary of the Act.(c) The Act is not easily accessible.(d) The consumer remains unaware of his rights and

privileges.(e) The plight of the consumer is yet to end.

5454ReadingComprehension - II

Page 120: yoursmahboob.wordpress.com SBI · yoursmahboob.wordpress.com iii P 101 Speed Tests for SBI Bank Clerk Exam 101 Speed Tests for SBI Bank Clerk Exam is revised and updated edition on

yoursmahboob.w

ordpress.com

112 SPEED TEST 54

RESPONSE

GRID

5. a b c d e 6. a b c d e 7. a b c d e 8. a b c d e 9. a b c d e

10. a b c d e 11. a b c d e 12. a b c d e 13. a b c d e 14. a b c d e

15. a b c d e

5. Which of the following best describes the judge’sreplacement?(a) He was partial towards the airline as it was a foreign

one.(b) He never bothered to safeguard the interests of the

reviewer.(c) He dismissed cases without even giving a second

thought to what cases came to him.(d) He was apathetic and uninterested about the direction

the case might head in.(e) He passed irrelevant verdicts indifferently.

6. What does the Act broadly cover?(a) It protects the right to redress.(b) It is a forum that protects the redresser.(c) It shields the consumer from deceptive and unfair trade

practices.(d) It enables the plaintiff to fight his case free of cost.(e) None of these

7. Which of the following is a limitation of the Act?(a) It does not cover the international law of torts.(b) It is not comprehensive with regard to liberal economy.(c) No forum or commission has come forward to bring it

to light.(d) Its red-tapism(e) None of these

8. How has Girimaji’s attempt been creditable?(a) It has given the Act a new dimension.(b) She has brought all the loopholes in the Act to the

consumer’s notice.(c) She has looked at the Act in a very disinterested and

impersonal manner.(d) She has discussed the law in the most explicit manner.(e) Her implicit dialogue with the consumer has made him

aware of his rights.9. What is the functionary role of the chairman of the National

Commission?(a) To be the titular head of the commission(b) To be accountable to the public(c) To prevent any dissent arising out of his verdicts and

Acts(d) To adjourn the cases with impunity(e) None of these

DIRECTIONS (Qs. 10-12): Choose the word which is most SIMILARin meaning to the word printed in bold as used in the passage.

10. Forum(a) Dias (b) Podium (c) Platform(d) Stage (e) None of these

11. Attuned(a) Brought into harmony(b) Adjusted(c) Hazardous(d) Out of tune(e) Malpractice

12. Adjourned(a) Stopped (b) Postponed (c) Decided(d) Cleared (e) Pended

DIRECTIONS (Qs. 13-15): Select the word which is most OPPOSITEin meaning of the word printed in bold as used in the passage.

13. Impunity(a) Penalised(b) Fine(c) Sentence(d) Freedom from punishment(e) None of these

14. Mired(a) Buried (b) Muddy (c) Steeped(d) Free (e) None of these

15. Redressal(l) Plea(b) Justice(c) Sue for compensation(d) Not to compensate(e) Put right

DIRECTIONS (Qs. 16-23): Read the following passage carefullyand answer the questions given below it. Certain words / phraseshave been printed in bold to help you locate them while answeringsome of the questions.

Amartya Sen wrote about the Indian tradition of skepticismand heterodoxy of opinion that led to high levels of intellectualargument. The power sector in India is a victim of this tradition atits worst. Instead of forcefully communicating, supporting andhonestly and firmly implementing policies, people just debatethem. It is argued that central undertakings produce power atlower tariffs and must therefore build most of the required extracapacities. This is a delusion. They no longer have access tolow-cost government funds.

Uncertainty about payment remains a reason for thehesitation of private investment. They had to sell only to SEBs(State Electricity Boards). SEB balance sheets are cleaner after the“securitisation” of the Rs 40,000 crore or so owed by SEBs tocentral government undertakings, now shown as debt instruments.But state governments have not implemented agreed plans toensure repayment when due. The current annual losses of aroundRs 28,000 crore make repayment highly uncertain. The centralundertakings that are their main suppliers have payment securitybecause the government will come to their help. Private enterprisesdo not have such assurance and are concerned about paymentsecurity, that must be resolved.

Page 121: yoursmahboob.wordpress.com SBI · yoursmahboob.wordpress.com iii P 101 Speed Tests for SBI Bank Clerk Exam 101 Speed Tests for SBI Bank Clerk Exam is revised and updated edition on

yoursmahboob.w

ordpress.com

113SPEED TEST 54

By the late 1990s, improving the SEB finances wasrecognised as fundamental to power reform. Unbundling SEBs,working under corporate discipline and even privatisation andnot vertically integrated state enterprises, are necessary forefficient and financially viable electricity enterprises. Sincegovernment will not distance itself from managing them,privatising is an option. The Delhi model has worked. But itreceives no public support.

The Electricity Act 2003, the APRDP (Accelerated PowerReform and Development Programme) with its incentives andpenalties, and the creation of independent regulatorycommissions, were the means to bring about reforms to improvefinancial viability of power sector. Implementation has been half-hearted and results disappointing. The concurrent nature ofelectricity in the Constitution impedes power sector improvement.States are more responsive to populist pressures than the centralgovernment, and less inclined to take drastic action againstelectricity thieves.

Captive power would add significantly to capacity. However,captive generation, three years after the Act enabled it, has addedlittle to capacity because rules for open access were delayed.Redefined captive generation avoids state vetoes on purchaseor sale of electricity except to state electricity enterprises.Mandating open access on state-owned wires to powerregardless of ownership and customer would encourageelectricity trading. The Act recognised electricity trading as aseparate activity. A surcharge on transmission charges will payfor cross-subsidies. These were to be eliminated in time. Rulesfor open access and the quantum of surcharge by each statecommission (under broad principles defined by the centralcommission) have yet to be announced by some. The few whohave announced the surcharge have kept it so high that no tradingcan take place.

16. The author thinks it appropriate to(a) discuss any policy in details and make it fool proof

instead of implementing it hastily.(b) follow Indian tradition meticulously as skepticism is

essential for major decisions.(c) divert our energies from fruitlessly contracting policies

to supporting its implementation whole-heartedly.(d) intellectual arguments and conceptualisation of every

policy is definitely better than its enforcement.(e) none of these

17. Why are the central undertakings not capable of generatingpower at low cost?(a) Due to paucity of low-cost funds(b) Due to their access to Government funds(c) Due to their delusion about government funds(d) Because of their extra capacities(e) None of these

18. Which of the following is the reason for apathy of privateinvestors in power sector?(a) Their hesitation(b) Uncertainly of their survival(c) Cut-throat competition(d) Lack of guarantee of timely returns(e) None of these

19. What was the serious omission on the part of the StateGovernment?(a) Agreement for late recovery of dues(b) Reluctance to repay to private investors as per agreed

plan(c) Non-implementation of recovery due to unplanned and

haphazard polices(d) Lack of assurance from private enterprises(e) None of these

20. Which of the following is/are considered necessary forimproving performance of electricity enterprises?(A) Corporate work culture(B) Privatisation(C) Properly integrated state enterprises(a) All the three (b) (a) and (b) only(c) (a) and (c) only (d) (b) and (c) only(e) None of these

21. The example of “Delhi Model” quoted by the authorunderlines his feelings ofA. happiness about its success.B. unhappiness for lack of public supportC. disgust towards privatisation.(a) (a) and (b) only (b) (b) and (c) only(c) (a) and (c) only (d) All the three(e) None of these

22. Which of the following was/were not considered as theinstrument(s) to accomplish financial well-being of powersector?(a) The Electricity Act 2003(b) The APRDP with its incentives and penalties(c) Setting up of independent regulatory commissions(d) States vulnerability to populist pressures(e) Taking drastic action against electricity thieves.

23. Why were the results of the power sector reforms NOT ashad been anticipated?(a) The means to bring about reforms were illconceived.(b) The enforcement of the reform means was inadequate

and apathetic.(c) The Act and the reform measures were contradicting

with each other.(d) The incentives on the one hand and penalties on the

other created dissatisfaction.(e) None of these

RESPONSE

GRID

16. a b c d e 17. a b c d e 18. a b c d e 19. a b c d e 20. a b c d e

21. a b c d e 22. a b c d e 23. a b c d e

Page 122: yoursmahboob.wordpress.com SBI · yoursmahboob.wordpress.com iii P 101 Speed Tests for SBI Bank Clerk Exam 101 Speed Tests for SBI Bank Clerk Exam is revised and updated edition on

yoursmahboob.w

ordpress.com

114 SPEED TEST 54

DIRECTIONS (Qs. 24-30) : Read the following passage and answerthe questions given below it. Certain words/phrases are given inhold to bold you to locate them while answering some of thequestions.

We have inherited the tradition of secrecy about the budgetfrom Britain where also the system has been strongly attacked byeminent economists and political scientists including Peter Jay.Sir Richard Clarke, who was the originating genius of nearly everyimportant development in the British budgeting techniques duringthe last two decades, has spoken out about the abuse of budgetsecrecy: “The problems of long-term tax policy should surely bedebated openly with the facts on the table. In my opinion, allgovernments should have just the same duty to publish theirexpenditure policy. Indeed, this obligation to publish taxationpolicy is really essential for the control of public expenditure inorder to get realistic taxation implications.” Realising thatdemocracy flourishes best on the principles of open government,more and more democracies are having an open public debate onbudget proposals before introducing the appropriate Bill in thelegislature. In the United States the budget is conveyed in amessage by the President to the Congress, which comes well inadvance of the date when the Bill is introduced in the Congress.In Finland the Parliament and the people are already discussing inJune the tentative budget proposals which are to be introduced inthe Finnish Parliament in September. Every budget contains acartload of figures in black and white - but the dark figures representthe myriad lights and shades of India’s life, the contrasting tonesof poverty and wealth, and of bread so dear and flesh and bloodso cheap, the deep tints of adventure and enterprise and man’sageless struggle for a brighter morning. The Union budget shouldnot be an annual scourge but a part of presentation of annualaccounts of a partnership between the Government and the people.That partnership would work much better when the nonsensicalsecrecy is replaced by openness and public consultations, resultingin fair laws and the people’s acceptance of their moral duty to pay.24. How do the British economists and political scientists react

to budget secrecy? They are(a) in favour of having a mix of secrecy and openness.(b) indifferent to the budgeting techniques and taxation

policies.(c) very critical about maintenance of budget secrecy.(d) advocates of not disclosing in advance the budget

contents.(e) None of these

25. The author thinks that openness in budget is essential as itleads to(a) prevention of tax implications(b) people’s reluctance to accept their moral duties(c) exaggerated revelation of the strengths and

weaknesses of economy(d) making our country on par with Finland(e) None of these

26. The author seems to be in favour of(a) maintaining secrecy of budget(b) judicious blend of secrecy and openness(c) transparency in budget proposals(d) replacement of public constitution by secrecy(e) None of these

27. The secrecy of the budget is maintained by all of thefollowing countries exceptA. FinlandB. IndiaC. United States(a) Only A (b) Only B (c) Only C(d) A and C (e) B and C

28. Which of the following statements is definitely TRUE in thecontext of the passage?(a) The British Government has been religiously

maintaining budget secrecy.(b) Budget secrecy is likely to lead to corrupt practices.(c) Consulting unjustifiable taxes with public helps make

them accept those taxes.(d) There should be no control on public expenditure in

democratic condition.(e) None of these

29. Sir Richard Clarke seems to deserve the credit for(a) transformation in the British budgetary techniques.(b) maintenance of secrecy of the British budget.(c) detection of abuse of transparency in budget.(d) bringing down the tax load on British people.(e) None of these

30. From the contents of the passage, it can be inferred that theauthor is(a) authoritarian in his approach.(b) a democratic person.(c) unaware of India’s recent economic developments.(d) a conservative person.(e) None of these

RESPONSE

GRID

24. a b c d e 25. a b c d e 26. a b c d e 27. a b c d e 28. a b c d e

29. a b c d e 30. a b c d e

Page 123: yoursmahboob.wordpress.com SBI · yoursmahboob.wordpress.com iii P 101 Speed Tests for SBI Bank Clerk Exam 101 Speed Tests for SBI Bank Clerk Exam is revised and updated edition on

yoursmahboob.w

ordpress.com

DIRECTION : Read the following passages to answer the givenquestion bused on it. Some words/phrases are printed in bold tohelp you locate them while answering some of the questions.

Political ploys initially hailed as master-strokes often endup as flops. The ` 60,000 crore farm loan waiver announced inthe budget writes off 100% of overdues of small and marginalfarmers holding upto two hectares, and 25% of overdues of largerfarmers. While India has enjoyed 8%-9% GDP growth for thepast few years, the boom has bypassed many rural areas andfarmer distress and suicides have made newspaper headlines.Various attempts to provide relief (employment guarantee scheme,public distribution system) have made little impact, thanks tohuge leakages from the government’s lousy delivery systems.So, many economists think the loan waiver is a worthwhilealternative to provide relief.

However the poorest rural folk are landless labourers whoget neither farm loans nor waivers. Half of the small and marginalfarmers get no loans from banks, and depend entirely onmoneylenders, and will not benefit. Besides, rural India is full ofthe family holdings rather than individual holdings and familyholdings will typically be much larger than two hectares even fordirt-poor farmers, who will, therefore, be denied the 100% waiver. Itwill thus fail in both economic and political objectives. IRDP loansto the rural poor in the 1980s demonstrated that crooked bankofficials demand bribes amounting to one third the intendedbenefits. Very few of the intended beneficiaries who merited reliefreceived it. After the last farm loan waiver will similarly slow downfresh loans to deserving farmers. While overdues to co-operativesmay be higher, economist Snrjit Bhalla says less then 5% of farmerloans to banks are overdue ie overdues exist for only 2.25 millionout of 90 million farmers. If so, then the 95% who have repaid loanswill not benefit. They will be angry at being penalised for honesty.

The budget thus grossly overestimates the number ofbeneficiaries. It also underestimates the negative effects of thewaiver encouraging wilful default in the future and discouragingfresh bank lending for some years. Instead of trying to reach theneedy, through a plethora of leaky schemes we should transfercash directly to the needy using new technology like biometricsmart cards, which are now being used in many countries, andmobile phones bank accounts. Then benefits can go directly tophone accounts operable only by those with biometric cards,ending the massive leakages of current schemes.

RESPONSE

GRID

1. a b c d e 2. a b c d e 3. a b c d e

Max. Marks : 30 No. of Qs. 30 Time : 20 min. Date : ........./......../................

The political benefits of the loan waiver have also beenexaggerated since if only a small fraction of farm families benefit,and many of these have to pay bribes to get the actual benefit, willthe waiver really be a massive vote-winner? Members of joint familieswill feel aggrieved that, despite having less than one hectare perhead, their family holding is too large to qualify for the 100% waiver.All finance ministers, of central or state governments, give awayfreebies in their last budgets, hoping to win electoral regards. yet,four-fifth of all incumbent government are voted out. This showsthat beneficiaries of favours are not notably grateful, while thosenot so favoured may feel aggrieved, and vote for the opposition.That seems to be why election budgets constantly fail to winelections in India and the loan waiver will not change that pattern.1. Why do economists feel that loan waivers will benefit farmers

in distress?(a) It will improve the standard of living of those farmers

who can afford to repay their loans but are exempted.(b) Other government relief measures have proved

ineffective,(c) Suicide rates of farmers have declined after the

announcement of the waiver.(d) Farmers will be motivated to increase the size of their

family holdings not individual holdings.(e) The government will be forced to re-examine and improve

the public distribution system.2. What message will the loan waiver send to farmers who have

repaid loans?(a) The Government will readily provide them with loans in

the future.(b) As opposed to money lenders banks are a safer and

more reliable source of credit.(c) Honesty is the best policy.(d) It is beneficial to take loans from co-operatives since

their rates of interest are lower.(e) They will be angry at being penalised for honesty.

3. What is the author's suggestion to provide aid to farmers?(a) Families should split their joint holding to take advantage

of the loan waiver.(b) The government should increase the reach of the

employment guarantee scheme.(c) Loans should be disbursed directly into bank accounts

of the farmers using the latest technology.(d) Government should ensure that loans waivers can be

implemented over the number of years.(e) Rural infrastructure can be improved using schemes

which were successful abroad.

5555ReadingComprehension - III

Page 124: yoursmahboob.wordpress.com SBI · yoursmahboob.wordpress.com iii P 101 Speed Tests for SBI Bank Clerk Exam 101 Speed Tests for SBI Bank Clerk Exam is revised and updated edition on

yoursmahboob.w

ordpress.com

116 SPEED TEST 55

RESPONSE

GRID

4. a b c d e 5. a b c d e 6. a b c d e 7. a b c d e 8. a b c d e

9. a b c d e 10. a b c d e 11. a b c d e 12. a b c d e 13. a b c d e

14. a b c d e 15. a b c d e

4. What was the outcome of IRDP loans to the rural poor?(a) The percentage of bank loan sanctioned to family

owned farms increased.(b) The loans· benefited dishonest moneylenders not

landless labourers.(c) Corrupt bank officials were the unintended beneficiaries

of the loans.(d) It resulted in tne Government sanctioning thrice the

amount for the current loan waiver.(e) None of these.

5. What are the terms of the loan waiver?(1) One-fourth of the overdue loans of landless labourers

will be written off.(2) The ` 60,000 crore loan waiver has been sanctioned for

2.25 million marginal farmers.(3) Any farmer with between 26 per cent to 100 per cent of

their loan repayments overdue will be penalised.(a) Only (1) (b) On1y (2)(c) Both (2) and (3) (d) All (1), (2) and (3)(e) None of these

6. What is the author's view of the loan waiver?(a) It will have an adverse psychological impact on those

who cannot avail of the waiver.(b) It is a justified measure in view of the high suicide rate

among landless labourers.(c) It makes sound economic and political sense in the

existing scenario.(d) It will ensure that the benefits of India's high GDP are

felt by the rural poor.(e) None of these

7. Which of the following cannot be said about loan waiver?(1) Small and marginal farmers will benefit the most.(2) The loan waiver penalises deserving farmers.(3) A large percentage ie ninety five per cent of distressed

farmers will benefit.(a) Only (3) (b) Both (1) and (3)(c) Only (1) (d) Both (2) and (3)(e) None of these

8. Which of the following will definitely be an impact of loanwaivers?(1) Family holdings will be split into individual holdings

not exceeding one hectare.(2) The public distributipn system will be revamped.(3) Opposition will definitely win the election.(a) None (b) Only (1)(c) Both (1) and (2) (d) Only (3)(e) All (1), (2) and (3)

9. What impact will the loan waiver have on banks?(a) Banks have to bear the entire brunt of the write off.(b) Loss of trust in banks by big farmers.(c) Corruption among bank staff will increase.(d) Farmers will make it a habit to default on loans(e) None of these

10. According to the author what is the government's motive insanctioning the loan waiver?(a) To encourage farmers to opt for bank loans from money

lenders.(b) To raise 90 million farmers out of indebtedness.(c) To provide relief to those marginal farmers who have

the means to but have not repaid their loans(d) To ensure they will be re-elected(e) None of these

DIRECTION (Q. 11-13) : Choose the word which is most nearlythe SAME in meaning to the word printed in bold as used in thepassage.11. incumbent

(a) mandatory (b) present(c) incapable (d) tazy(e) officious

12. ploys(a) surveys (b) entreaties(c) ruses (d) sliders(d) assurances

13. aggrieved(a) vindicated (b) intimidated(c) offensive (d) wronged(d) disputed

DIRECTIONS (Q. 14 & 15) : Choose the word which is most OPPOSITEin meaning to the word printed in bold as used in the passage.14. plethora

(a) dearth (b) missing(c) superfluous (d) sufficient(e) least

15. merited(a) ranked· (b) unqualified for(c) lacked (d) inept at(e) unworthy of

DIRECTIONS (Q. 16-25): Read the following passage to answerthe given questions based on it, Some words/phrases are printed inbold to help you locate them while answermg some of the questions.

The e-waste (Management and Handling) Rules, 2011,notified by the Ministry of Environmynt and Forests, have thepotential to turn a growing problem into a developmentalopportunity. With almost half-a-year to go before the rules take

Page 125: yoursmahboob.wordpress.com SBI · yoursmahboob.wordpress.com iii P 101 Speed Tests for SBI Bank Clerk Exam 101 Speed Tests for SBI Bank Clerk Exam is revised and updated edition on

yoursmahboob.w

ordpress.com

117SPEED TEST 55

effect, there is enough time to create the necessary infrastructurefor collection, dismantling and recycling of electronic waste. Thefocus must be on sincere and efficient implementation. Onlydecisive action can reduce the pollution and health costsassociated with India's hazardous waste recycling industry. IfIndia can achieve a transformation, it will be creating a wholenew-employment sector that provides good wages and workingconditions for tens of thousands. The legacy response of theStates to even the basic law on urban waste, the Municipal SolidWastes (Management and Handling) Rules has been one ofindifference; many cities continue to simply bum the garbage ordump it in lakes. With the emphasis now on segregation of wasteat source and recovery of materials. it should be feasible toimplement both sets of rules efficiently. A welcome feature of thenew e-waste rules is the emphasis on extended producerresponsibility. In other words, producers must take responsibilityfor the disposal of end-of-life products. For this provision towork, they must ensure that consumers who sell scrap get someform of financial incentive.

The e-waste rules, which derive from those pertaining tohazardous waste, are scheduled to come into force on May 1,2012. Sound as they are, the task of scientifically disposing a fewhundred thousand tonnes of trash electronics annually dependsheavily on a system of oversight by State Pollutions ControlBoards (PCBs). Unfortunately, most PCBs remain unaccountableand often lack the resources for active enforcement. It must bepointed out that, although agencies handling e-waste must obtainenvironmental clearances and be authorised and registered bythe PCBs even under the Hazardous Wastes (Management,Handling and Transboundary Movement) Rules 2008, there hasbeen little practical impact. Over 95 per cent of electronic waste iscollected and recycled by the informal sector. The way forward isfor the PCBs to be made accountable for enforcement of the e-waste rules and the levy of penalties under environmental laws.Clearly, the first order priority is to create a system that will absorbthe 80,000-strong workforce in the informal sector into theproposed scheme for scientific recycling. Facilities must be createdto upgrade the skills of these workers through training and theiroccupational health must be ensured.Recycling of e-waste is one of the biggest challenges today. Insuch a time, when globalisation and information technology aregrowing at a pace which could only be imagined few years back,e-waste and its hazards have become more prominent over aperiod of time and should be given immediate attention.16. What, according to the passage, is important now for

e-waste management?(a) Making rules (b) Reviewing rules(c) Implementing rules (d) Notifying rules(e) Amending rules

17. Which of the following can be one ofthe by-products ofeffective e-waste management? .(a) India can guide other countries in doing so.(b) It will promote international understanding.(c) It will promote national integration.(d) It will create a new employment sector.(e) It will further empowet judiciary.

18. Which of the following rules has not been indicated in thepassage?(a) e-waste Rules 20 11(b) Pollution Check Rules(c) Hazardous Wastes Rules, 2008(d) Municipal Solid Wastes Rules(e) All these have been indicated

19. “Both sets of rules” is being referred to which of the following?(a) Solid wastes and hazardous wastes(b) e-waste and hazardous waste(c) Solid waste and e-waste(d) e-waste and e-production(e) Solid waste and recycling waste

20. e-waste rules have been derived from those pertaining to(a) Hazardous waste (b) PC waste(c) Computer waste (d) Municipal solid waste(d) National waste

21. Which of the following will help implement “both sets ofrules”?(a) Employment opportunities(b) International collaboration(c) Financial incentive(d) Segregation of waste at source(e) Health costs

22. e-waste Rules came/come into force from(a) 2008 (b) 2009(c) 2010 (d) 2011(e) 2012

23. Which of the following best explains the meaning of thephrase “which could only be imagined few years back” asused in the passage?(a) It was doomed.(b) It took us few years.(c) It took us back by few years(d) Imagination is better than IT.(e) None of these

24. Which of the following is true in the context of the passage?(a) No city dumps its waste in lakes.(b) Some cities burn garbage.(c) PCBs have adequate resources for'active enforcement.(d) e-waste was a much bigger chajlenge in the past.(e) None of these

RESPONSE

GRID

16. a b c d e 17. a b c d e 18. a b c d e 19. a b c d e 20. a b c d e

21. a b c d e 22. a b c d e 23. a b c d e 24. a b c d e

Page 126: yoursmahboob.wordpress.com SBI · yoursmahboob.wordpress.com iii P 101 Speed Tests for SBI Bank Clerk Exam 101 Speed Tests for SBI Bank Clerk Exam is revised and updated edition on

yoursmahboob.w

ordpress.com

118 SPEED TEST 5525. Which of the following is not true 10 the context of the

passage?(a) Some fonn offmancial incentive is recommended for

the producers.(b) Some financial incentive is recommended for the

consumers.(c) e-waste will be a few hundred thousand tonnes.(d) The agencies handling e-waste have to obtain

environmental clearances.(e) Those involved in e-waste management would need to

upgrade their skills.

DIRECTIONS (Q. 26-28): Choose the word which is most nearlythe same in meaning of the word printed in bold, as used in thepassage.26. Clearance

(a) Cleaning (b) Permisssion(c) Sale (d) Remedy(e) Clarity

RESPONSE

GRID

25. a b c d e 26. a b c d e 27. a b c d e 28. a b c d e 29. a b c d e

30. a b c d e

27. Turn(a) Throw (b) Chance(c) Send (d) Transform(e) Rotate

28. Potential(a) Intelligence (b) Aptitude(c) Possibility (d) Portion(e) Will

DIRECTIONS (Q. 29 & 30): Choose the word which is mostopposite in meaning of the word printed in bold as used in thepassage.29. Feasible

(a) Unattended (b) Physical(c) Practical (d) Unviabie(e) Wasteful

30. Indifference(a) Interest (b) Difference(c) Ignorance (d) Rule-bound(e) Insignificance

Page 127: yoursmahboob.wordpress.com SBI · yoursmahboob.wordpress.com iii P 101 Speed Tests for SBI Bank Clerk Exam 101 Speed Tests for SBI Bank Clerk Exam is revised and updated edition on

yoursmahboob.w

ordpress.com

DIRECTIONS (Qs. 1-25): Select the synonyms of the word givenin CAPITAL letters.

1. VICARIOUS(a) Ambitious(b) Not experienced personally(c) Nostalgic(d) Vindictive(e) None of these

2. CRAVEN(a) Greedy (b) Cowardly(c) Flattering (d) Restless(e) None of these

3. TEPID(a) Irreversible (b) Causing fatigue(c) Fast moving (d) Lukewarm(e) None of these

4. TENUOUS(a) Contentious (b) Dark(c) Slender (d) Malfunctioning(e) None of these

5. PROBITY(a) Integrity (b) Impudence(c) Profane (d) Preface(e) None of these

6. MUSTY(a) Certainty (b) Stale(c) Modern (d) Mysterious(e) None of these

7. ALLEVIATE(a) To release (b) To lessen(c) To deprive (d) To deceive(e) None of these

8. MOROSE(a) Humble (b) Morsel(c) Sullen (d) Repugnant(e) None of these

RESPONSE

GRID

1. a b c d e 2. a b c d e 3. a b c d e 4. a b c d e 5. a b c d e

6. a b c d e 7. a b c d e 8. a b c d e 9. a b c d e 10. a b c d e

11. a b c d e 12. a b c d e 13. a b c d e 14. a b c d e 15. a b c d e

16. a b c d e 17. a b c d e

9. PROTAGONIST(a) Talented child (b) Reserved person(c) Leading character (d) Fearless(e) None of these

10. FACTITIOUS(a) Humorous (b) Truthful(c) Artificial (d) Causing fatigue(e) None of these

11. HOSPITABLE(a) Convivial (b) Liberal(c) Congenital (d) Welcoming(e) None of these

12. SCARCELY(a) Hardly (b) Always(c) Sometimes (d) Frequently(e) None of these

13. DISDAIN(a) Disown (b) Condemn(c) Hate (d) Criticise(e) None of these

14. ABSURD(a) Senseless (b) Clean(c) Abrupt (d) Candid(e) None of these

15. PHILANTHROPY(a) Generosity (b) Perversity(c) Perjury (d) Flaunting(e) None of these

16. MUTUAL(a) Reciprocal (b) Agreed(c) Common (d) Conjugal(e) None of these

17. WEIRD(a) Beastly (b) Unpleasant(c) Frightening (d) Unnatural(e) None of these

Max. Marks : 40 No. of Qs. 40 Time : 25 min. Date : ........./......../................

5656Synonyms

Page 128: yoursmahboob.wordpress.com SBI · yoursmahboob.wordpress.com iii P 101 Speed Tests for SBI Bank Clerk Exam 101 Speed Tests for SBI Bank Clerk Exam is revised and updated edition on

yoursmahboob.w

ordpress.com

120 SPEED TEST 5618. PESSIMISTIC

(a) Indifferent (b) Ascetic(c) Unsettle (d) Not hopeful(e) None of these

19. ANALOGOUS(a) Unsuitable (b) Uncritical(c) Similar (d) Disproportionate(e) None of these

20. EXAGGERATE(a) Bluff (b) Overstate(c) Explain (d) Underestimate(e) None of these

21. EVIDENT(a) Prominent (b) Seen(c) Observed (d) Quite clear(e) None of these

22. PENALIZE(a) Persecute (b) Punish(c) Torture (d) Ruin(e) None of these

23. REMEDIAL(a) Punitive (b) Stringent(c) Corrective (d) Strict(e) None of these

24. TRIVIAL(a) Unimportant (b) Transparent(c) Important (d) Unexpected(e) None of these

25. INCREDIBLE(a) Hard to believe (b) Considerable(c) Inconsistent (d) Unsatisfactory(e) None of these

DIRECTIONS (Qs. 26-40): Select the synonym of the wordoccuring in the sentence in CAPITAL letters as per the context.26. After weeks of FRENETIC activity, the ground was ready

for the big match.(a) Strenuous (b) Hurried(c) Excited (d) Hectic(e) None of these

27. The policemen kept on questioning the criminal but failed toELICIT the truth.(a) Evoke (b) Wrest(c) Obtain (d) Extort(e) None of these

28. Teaching in universities is now considered a LUCRATIVEjob.(a) Risky (b) Profitable(c) Honourable (d) Undignified(e) None of these

29. His descriptions are VIVID.(a) Detailed (b) Categorical

(c) Clear (d) Ambiguous(e) None of these

30. Friends have always DEPLORED my unsociable nature.(a) Deprived (b) Implored(c) Denied (d) Regretted(e) None of these

31. People fear him because of his VINDICTIVE nature.(a) Violent (b) Cruel(c) Revengeful (d) Irritable(e) None of these

32. He always has a very PRAGMATIC approach to life.(a) Practical (b) Proficient(c) Potent (d) Patronizing(e) None of these

33. The song had a SOPORIFIC effect on the child.(a) Soothing (b) Terrific(c) Supreme (d) Sleep-inducing(e) None of these

34. His bad behaviour EVOKED punishment.(a) Escaped (b) Called for(c) Produced (d) Summoned(e) None of these

35. The boy said that pain had ABATED.(a) Reduced (b) Vanished(c) Increased (d) Stabilised(e) None of these

36. The queen was aware of the INSOLENT behaviour of thelords.(a) Violent (b) Polite(e) None of these(c) Insulting (d) Frivolous

37. The newspaper reports were MENDACIOUS(a) mischievous (b) truthful(c) provocative (d) false(e) None of these

38. He INDUCES human beings to want things they don’t want.(a) Influences (b) Dictates(c) Persuades (d) Appreciates(e) None of these

39. His information is not AUTHENTIC.(a) Real (b) Reliable(c) Believable (d) Genuine(e) None of these

40. It is the difficulty or SCARCITY of a thing that it makes itprecious.(a) poverty (b) absence(c) insufficiency (d) disappearance(e) None of these

RESPONSE

GRID

18. a b c d e 19. a b c d e 20. a b c d e 21. a b c d e 22. a b c d e

23. a b c d e 24. a b c d e 25. a b c d e 26. a b c d e 27. a b c d e

28. a b c d e 29. a b c d e 30. a b c d e 31. a b c d e 32. a b c d e

33. a b c d e 34. a b c d e 35. a b c d e 36. a b c d e 37. a b c d e

38. a b c d e 39. a b c d e 40. a b c d e

Page 129: yoursmahboob.wordpress.com SBI · yoursmahboob.wordpress.com iii P 101 Speed Tests for SBI Bank Clerk Exam 101 Speed Tests for SBI Bank Clerk Exam is revised and updated edition on

yoursmahboob.w

ordpress.com

DIRECTIONS (Qs. 1-20): Select the antonyms of the word givenin CAPITAL letters.1. REINFORCING

(a) contradicting (b) wishing(c) jolting (d) forcing(e) re-inventing

2. BEFRIENDED(a) recoiled (b) killed(c) accepted (d) mistrusted(e) ignored

3. FRAIL(a) Unhealthy (b) Massive(c) Rich (d) Robust(e) Civilised

4. SPLENDIDLY(a) Wisely (b) Unfairly(c) Rudely (d) Reluctantly(e) Unimpressively

5. CHARMED(a) Offended (b) Stunned(c) Repulsed (d) Jealous(e) Outraged

6. OBLIGATORY(a) Doubtful (b) Voluntary(c) Sincerely (d) Faithfully(e) None of these

7. OBSCURE(a) Suitable (b) Apt(c) Thalamus (d) Clear(e) None of these

8. REPULSIVE(a) Attractive (b) Colourful(c) Unattractive (d) Striking(e) None of these

9. VITAL(a) Trivial (b) Peripheral(c) Optional (d) Superficial(e) None of these

10. INHIBIT(a) Pamper (b) Breed(c) Accept (d) Promote(e) None of these

RESPONSE

GRID

1. a b c d e 2. a b c d e 3. a b c d e 4. a b c d e 5. a b c d e

6. a b c d e 7. a b c d e 8. a b c d e 9. a b c d e 10. a b c d e

11. a b c d e 12. a b c d e 13. a b c d e 14. a b c d e 15. a b c d e

16. a b c d e 17. a b c d e 18. a b c d e 19. a b c d e 20. a b c d e

11. EXPLOIT(a) Utilize (b) Alert(c) Support (d) Neglect(e) None of these

12. SHARP(a) Bleak (b) Blunt(c) Bright (d) Blond(e) None of these

13. CONDEMN(a) Censure (b) Approve(c) Recommend (d) Praise(e) None of these

14. RELUCTANT(a) Avoiding (b) Anxious(c) Refuse (d) Eager(e) None of these

15. SCARCITY(a) Plenty (b) Prosperity(c) Facility (d) Simplicity(e) None of these

16. BLEAK(a) Bright (b) Confusing(c) Uncertain (d) Great(e) None of these

17. STERN(a) Violent (b) Generous(c) Mild (d) Forgiving(e) None of these

18. SUPERFICIAL(a) Profound (b) Difficult(c) Secretive (d) Mystical(e) None of these

19. ELEGANCE(a) Balance (b) Indelicacy(c) Clumsiness (d) Savagery(e) None of these

20. COARSE(a) Pleasing (b) Rude(c) Polished (d) Soft(e) None of these

Max. Marks : 40 No. of Qs. 40 Time : 25 min. Date : ........./......../................

5757Antonyms

Page 130: yoursmahboob.wordpress.com SBI · yoursmahboob.wordpress.com iii P 101 Speed Tests for SBI Bank Clerk Exam 101 Speed Tests for SBI Bank Clerk Exam is revised and updated edition on

yoursmahboob.w

ordpress.com

122 SPEED TEST 57

RESPONSE

GRID

21. a b c d e 22. a b c d e 23. a b c d e 24. a b c d e 25. a b c d e

26. a b c d e 27. a b c d e 28. a b c d e 29. a b c d e 30. a b c d e

31. a b c d e 32. a b c d e 33. a b c d e 34. a b c d e 35. a b c d e

36. a b c d e 37. a b c d e 38. a b c d e 39. a b c d e 40. a b c d e

DIRECTIONS (Qs. 21-40): Select the antonym of the wordoccuring in the sentence in CAPITAL letters as per the context.21. He is a man of EXTRAVAGANT habits.

(a) Sensible (b) Careful(c) Economical (d) Balanced(e) None of these

22. They employ only DILIGENT workers.(a) Unskilled (b) Lazy(c) Careless (d) Idle(e) None of these

23. The characters in this story are not all FICTITIOUS.(a) Common (b) Factual(c) Real (d) Genuine(e) None of these

24. He is a GENEROUS man.(a) Stingy (b) Uncharitable(c) Selfish (d) Ignoble(e) None of these

25. The issue raised in the form can be IGNORED.(a) Removed (b) Considered(c) Set aside (d) Debated(e) None of these

26. After swallowing the frog had become LETHARGIC.(a) Aggressive (b) Dull(c) Active (d) Hungry(e) None of these

27. Dust storms and polluted rivers have made it HAZARDOUSto breathe the air and drink the water.(a) Convenient (b) Risky(c) Wrong (d) Safe(e) None of these

28. Only hard work can ENRICH our country.(a) Impoverish (b) Improve(c) Increase (d) Involve(e) None of these

29. The story you have just told is INCREDIBLE.(a) Credible (b) Fantastic(c) Probable (d) Believable(e) None of these

30. The doctor found the patient INERT.

(a) Active (b) Lazy(c) Strong (d) Resolute(e) None of these

31. The boy comes of an AFFLUENT family.(a) Poor (b) Ordinary(c) Infamous (d) Backward(e) None of these

32. The young athlete is ENERGETIC enough to run tenthousand meters at a stretch.(a) inactive (b) dull(c) gloomy (d) lethargic(e) None of these

33. He has a SECURE position in the entrance examination.(a) rigid (b) precarious(c) static (d) secondary(e) None of these

34. The criminal was known to the police by VARIOUS names.(a) separate (b) distinct(c) identical (d) similar(e) None of these

35. RECESSION is a major cause of unemployment.(a) Education (b) Inflation(c) Poverty (d) Computerization(e) None of these

36. The accused emphatically DENIED the charge in the court.(a) Accepted (b) Agreed(c) Asserted (d) Affirmed(e) None of these

37. The three states signed a pact to have COLLECTIVEeconomy.(a) distributive (b) disintegrative(c) individual (d) divided(e) None of these

38. The child was ABDUCTED when he was going to school.(a) seized (b) set free(c) kidnapped (d) ransomed(e) None of these

39. The government has ENHANCED the tuition fees in schoolsand colleges.(a) magnified (b) aggravated(c) decreased (d) augmented(e) None of these

40. Children ADORE the cricket players.(a) Discuss (b) Condemn(c) Benefit (d) Check(e) None of these

Page 131: yoursmahboob.wordpress.com SBI · yoursmahboob.wordpress.com iii P 101 Speed Tests for SBI Bank Clerk Exam 101 Speed Tests for SBI Bank Clerk Exam is revised and updated edition on

yoursmahboob.w

ordpress.com

DIRECTIONS: Select the correct word or phrase to complete agrammatical sentence. In case of more than one blank, the differentwords given in the options shall fill in the corrosponding order.

1. As soon as the visitor’s dishonest purpose was discoveredhe was ........ the door.(a) show with (b) shown to(c) shown (d) shown out of(e) None of these

2. As soon as my attention was ........ the dangerous state ofthe staircase, I got it repaired.(a) drawn for (b) drawn upon(c) drawn near (d) drawn to(e) None of these

3. I wish my brother -........ here to listen to this entertaininglecture.(a) would be (b) has been(c) is (d) were(e) None of these

4. Did you think you ........ somewhere before?(a) have seen me (b) saw me(c) had seen me (d) would see me(e) None of these

5. Do not force me to ........ you on this issue; I am not at allconvinced.(a) agree upon (b) concur with(c) join over (d) equate with(e) None of these

6. Having ........ only in salt water before, I found it a littledifficult to swim in fresh water.(a) swam (b) swum(c) had swam (d) swimming(e) None of these

7. If I were you, I ............... be careful with my words.(a) will (b) would(c) shall (d) should(e) None of these

RESPONSE

GRID

1. a b c d e 2. a b c d e 3. a b c d e 4. a b c d e 5. a b c d e

6. a b c d e 7. a b c d e 8. a b c d e 9. a b c d e 10. a b c d e

11. a b c d e 12. a b c d e 13. a b c d e 14. a b c d e 15. a b c d e

Max. Marks : 30 No. of Qs. 30 Time : 20 min. Date : ........./......../................

8. Of the two assistants we employed last month, I findRaman .............. hard working.(a) most (b) more(c) least (d) only(e) None of these

9. It is earth’s gravity which .............. people their weight.(a) gives (b) give(c) giving (d) given(e) None of these

10. Total weight of all the ants in the world is much greaterthan ............... .(a) to all human beings(b) that of all human beings(c) is of all human beings(d) that of the all human beings(e) None of these

11. It is good form to use the name of the person ............... .(a) who are greeting(b) you are greeting(c) which you are greeting(d) greeting for you(e) None of these

12. .............. that increasing numbers of compact disc players willbe bought by consumers in the years to come.(a) They are anticipated(b) In anticipation(c) Anticipating(d) It is anticipated(e) None of these

13. He was frightened ...............(a) to be killed (b) to being killed(c) for being killed (d) of being killed(e) None of these

14. Capitalist society .............. profit as a valued goal.(a) which regards (b) regarding(c) regards (d) was regarded(e) None of these

15. The impact of two vehicles can cause a lot of .............. to both.(a) damage (b) damages(c) damaging (d) damagings(e) None of these

5858SentenceCompletion - I

Page 132: yoursmahboob.wordpress.com SBI · yoursmahboob.wordpress.com iii P 101 Speed Tests for SBI Bank Clerk Exam 101 Speed Tests for SBI Bank Clerk Exam is revised and updated edition on

yoursmahboob.w

ordpress.com

124 SPEED TEST 58

RESPONSE

GRID

16. a b c d e 17. a b c d e 18. a b c d e 19. a b c d e 20. a b c d e

21. a b c d e 22. a b c d e 23. a b c d e 24. a b c d e 25. a b c d e

26. a b c d e 27. a b c d e 28. a b c d e 29. a b c d e 30. a b c d e

16. The conditions necessary .............. this project have not beenmet.(a) of completion (b) for the complete of(c) of complete (d) for the completion of(e) None of these

17. The weather in the far north is not .............. it is down south.(a) like humid as (b) as humid as(c) humid as (d) so humid that(e) None of these

18. .............. the reactions of people with amnesia, scientists arelearning about the process of memory of the brain.(a) By studying (b) To study(c) They study (d) They are studying(e) None of these

19. After the election .............. a new stage.(a) the entering nation(b) the nation will enter(c) to enter the nation(d) will the nation enter(e) None of these

20. I hope she ............... .(a) must come (b) should come(c) will come (d) must be coming(e) None of these

21. An increase in population, without an increase ineconomic level, .............. result in a lower standard ofliving.(a) tends to (b) tending to(c) will tend (d) tends(e) None of these

22. .............. as President, a candidate must win a majority ofvotes.(a) Elected (b) To be elected(c) Having elected (d) Electing(e) None of these

23. Encounters between people from different countries canresult in misunderstandings .............. different conceptionsabout space.(a) because they (b) is because they(c) is because their (d) of their(e) None of these

DIRECTIONS : Select the correct word or phrase to complete agrammatical and idiomatic sentence.

24. If you are really not feeling well. you .......... a doctor.(a) should better see (b) may Sec(c) had better see (d) would rather see(e) None of these

25. If only I.......... his address, I would most certainly have toldyou.(a) know (b) knew(c) had known (d) off(e) None of these

26. The marathon race is intended to test one’s endurancemore...........(a) than his speed (b) than how fast one runs(c) than one’s speed (d) lain off(e) None of these

27. She expects me to type the letter in five minutes .......... isimpossible.(a) that (b) which(c) what (d) but(e) None of these

28. If only you had spoken clearly, you(a) would not be misunderstood(b) would not have been misunderstanding.(c) would not have been misunderstood.(d) would not have misunderstood.(e) None of these

29. It is high time that he .......... himself.(a) had reformed (b) will reform(c) has to reform (d) reformed(e) None of these

30. .........., a bus almost ran over him.(a) Running across the road(b) Running on the road(c) When he ran across the road(d) When he was running through the road.(e) None of these

Page 133: yoursmahboob.wordpress.com SBI · yoursmahboob.wordpress.com iii P 101 Speed Tests for SBI Bank Clerk Exam 101 Speed Tests for SBI Bank Clerk Exam is revised and updated edition on

yoursmahboob.w

ordpress.com

DIRECTIONS: Select the correct word or phrase to complete agrammatical and idiomatic sentence.

1. These essays are intellectually .............. and representvarious levels of complexity.(a) revealing (b) modern(c) superior (d) demanding(e) persistent

2. The soldiers are instructed to .......... restraint and handlethe situation peacefully.(a) exercise (b) control(c) enforce (d) remain(e) None of these

3. Since one connot read every book, one should be contentwith making a ........ selection.(a) normal (b) standard(c) sample (d) moderate(e) judicious

4. Some people ........ themselves into believing that they areindispensable to the organisation they work for.(a) keep (b) fool(c) force (d) denigrate(e) delude

5. How do you expect that country to progress when hergovernment is corrupt, ..... and still lergely feudal?(a) devalued (b) dwinding(c) demobilised (d) demeaning(e) None of these

6. The truck was ...... the trafic and the policeman asked thedriver to move off.(a) failing (b) obstructing(c) obviating (d) hiding(e) disturbing

7. The paternalistic attitude is so ingrained to themanagements that they will have to ........ try to change it.(a) casually (b) slowly(c) subtly (d) inadvertently(e) None of these

RESPONSE

GRID

1. a b c d e 2. a b c d e 3. a b c d e 4. a b c d e 5. a b c d e

6. a b c d e 7. a b c d e 8. a b c d e 9. a b c d e 10. a b c d e

11. a b c d e 12. a b c d e 13. a b c d e 14. a b c d e 15. a b c d e

16. a b c d e

Max. Marks : 35 No. of Qs. 35 Time : 20 min. Date : ........./......../................

8. His interest in the study of human behaviour is indeed very.........(a) strong (b) large(c) broad (d) vast(e) deep

9. The improvement made by changes in the system was .... andid not warrant the large expenses.(a) large (b) small(c) minute (d) marginal(e) uncertain

10. He is too ..... to be deceived easily.(a) strong (b) modern(c) kind (d) honest(e) intelligent

11. There has been a ...... lack of efficiency in all the crucial areasof the working of Public Sector Undertakings.(a) positive (b) surprising(c) conspicuous (d) stimulative(e) insignificant

12. I will be leaving for Delhi tonight and ........... to return by thisweek end.(a) waiting (b) plan(c) going (d) likely(e) making

13. Ravi’s behaviour is worthy of ....... by all the youngsters.(a) trial (b) emulation(c) following (d) exploration(e) experiment

14. The only way to ...... the country from the evils ofcommunalism is to enforce the rule of law.(a) eradicate (b) mobilise(c) extricate (d) purge(e) strengthen

15. Even at the risk of economic loss, he ...... refused to take thebeaten track.(a) repeatedly (b) stead fastly(c) regularly (d) continuously(e) None of these

16. On his sudden demise, may emotions were so complicatedthat it was ......... how I felt.(a) unreasonable (b) impossible(c) inexplicable (d) unimaginable(e) None of these

5959SentenceCompletion - II

Page 134: yoursmahboob.wordpress.com SBI · yoursmahboob.wordpress.com iii P 101 Speed Tests for SBI Bank Clerk Exam 101 Speed Tests for SBI Bank Clerk Exam is revised and updated edition on

yoursmahboob.w

ordpress.com

126 SPEED TEST 59

RESPONSE

GRID

17. a b c d e 18. a b c d e 19. a b c d e 20. a b c d e 21. a b c d e

22. a b c d e 23. a b c d e 24. a b c d e 25. a b c d e 26. a b c d e

27. a b c d e 28. a b c d e 29. a b c d e 30. a b c d e 31. a b c d e

32. a b c d e 33. a b c d e 34. a b c d e 35. a b c d e

17. Two of the fugitives managed to remain free by adeptlyavoiding the ....... of the police.(a) torture (b) pursuit(c) discovery (d) following(e) None of these

18. Experts fail to understand the ..... behind the decision tomove coal by road when there is enough rail capacity in thissector(a) ideology (b) judgement(c) rationale (d) politics(e) logistics

19. Automobile manufacturers are reviving up to launch acompaign designed to increase consumer ........ about thenew emmission control.(a) production (b) education(c) capacity (d) knowledge(e) awareness

20. His logic ......... everyone, including the expects.(a) teased (b) defied(c) surprised (d) confounded(e) overwhelmed

21. The factory went into a state of suspended ....... today withall its workers on strike.(a) symbiosis (b) animation(c) ways (d) condition(e) mortification

22. It is not fair to cast ....... on honest and innocent persons.(a) aspiration (b) aspersions(c) inspiration (d) adulation(e) None of these

23. You must ....... your career with all seriousness.(a) direct (b) complete(c) follow (d) manage(e) pursue

24. The villagers .......... the death of their leader by keeping allthe shops closed.(a) announced (b) protested(c) mourned (d) consoled(e) None of these

25. These medicines are ........ for curing cold.(a) proper (b) real(c) effective (d) capable(e) powerful

26. The poor ones continue to ........... out a living inspite ofeconomic liberalisation in that country.(a) find (b) go(c) bring (d) manage(e) None of these

27. I will write a letter to you tentatively ....... the dates of theprogramme.(a) involving (b) urging(c) guiding (d) indicating(e) propagating

28. Contemporary economic development differs ....... form theIndustrial Revolution of the 19th century.(a) naturally (b) usually(c) literally (d) specially(e) markedly

29. The word gharana points to the ...... concepts of stylisticindividuality and handing down of tradition within familyconfines.(a) joint (b) conflicting(c) dual (d) contradictory(e) extraordinary

30. It ws the help he got from his friends which ....... him throughthe tragedy.(a) helped (b) boosted(c) perked (d) supported(e) sustained

31. The criminals managed to escape from the prison eventhrough two armed policemen were ....... vigil over them.(a) taking (b) putting(c) guarding (d) keeping(e) looking

32. The speaker did not properly use the time as he went on .....on one point alone.(a) dilating (b) devoting(c) deliberating (d) diluting(e) distributing

33. Ravi had to drop his plan of going to picnic as he had certain..... to meet during that period.(a) preparations (b) observations(c) urgencies (d) transactions(e) commitments

34. Even in today’s modern society, people ....... god to bringrains.(a) provoke (b) evoke(c) propitiate (d) superimpose(e) None of these

35. The good is often ....... with their bones.(a) buried (b) covered(c) exhumed (d) interred(e) fleshed

Page 135: yoursmahboob.wordpress.com SBI · yoursmahboob.wordpress.com iii P 101 Speed Tests for SBI Bank Clerk Exam 101 Speed Tests for SBI Bank Clerk Exam is revised and updated edition on

yoursmahboob.w

ordpress.com

DIRECTIONS : In each of these questions, two sentences (I)and (II) are given. Each sentence has a blank in it. Five words (a),(b), (c), (d) and (e) are suggested. Out of these, only one fits atboth the places in the context of each sentence. Option of thatword is the answer.

1. I. He is__________ with whatever little he has.II. They kept the ____________ of the communication

a secret.(a) happy (b) matter(c) gist (d) content(e) sense

2. I. It is hard to believe the ___________ of operationsinvolved in this activity.

II. The map is drawn to a ___________ of 1 inch to 50km.

(a) magnitude (b) size(c) scale (d) proportion(e) significance

3. I. Heavy snow did ___________ the rescue efforts.II. The food was kept in a ___________.(a) delay (b) bundle(c) basket (d) hamper(e) holder

4. I. They left ___________ after breakfast.II. It is difficult to find a ___________ person for this job.(a) right (b) immediately(c) suitable (d) best(e) soon

5. I. He would always do ___________ was told by hissuperiors.

II. He appeared on stage ___________ a narrator of thedrama.

(a) as (b) what(c) about (d) whatever(e) always

RESPONSE

GRID

1. a b c d e 2. a b c d e 3. a b c d e 4. a b c d e 5. a b c d e

6. a b c d e 7. a b c d e 8. a b c d e 9. a b c d e 10. a b c d e

11. a b c d e 12. a b c d e

6. I. The truck stopped ___________.II. We take a ___________ walk every day.(a) suddenly (b) long(c) short (d) distant(e) near

7. I. I got the grains ___________ in the machine.II. I do not have any ___________ for doubting him.(a) done (b) basis(c) ground (d) crushed(e) tune

8. I. We were asked to design a ___________ of the dam.II. This Institute is a ___________ of modern thinking.(a) picture (b) type(c) function (d) fabric(e) model

9. I. Keep a ___________ grip on the railing.II. He was ___________ asleep.(a) fast (b) firm(c) deep (d) strong(e) sure

10. I. He asked me to ___________ over the fence.II. We should keep the valuables in the ___________.(a) vault (b) cross(c) safe (d) tie(e) locker

11. I. He has now become a ___________ to reckon with.II. It is better not to use ___________ to prove one’s point.(a) model (b) force(c) coercion (d) name(e) influence

12. I. He tried his ___________ best to score distinction inthis exam.

II. It pays to keep ___________ head in an emergency.(a) cool (b) utmost(c) very (d) possible(e) level

Max. Marks : 25 No. of Qs. 25 Time : 20 min. Date : ........./......../................

6060SimilarSubstitution

Page 136: yoursmahboob.wordpress.com SBI · yoursmahboob.wordpress.com iii P 101 Speed Tests for SBI Bank Clerk Exam 101 Speed Tests for SBI Bank Clerk Exam is revised and updated edition on

yoursmahboob.w

ordpress.com

128 SPEED TEST 60

RESPONSE

GRID

13. a b c d e 14. a b c d e 15. a b c d e 16. a b c d e 17. a b c d e

18. a b c d e 19. a b c d e 20. a b c d e 21. a b c d e 22. a b c d e

23. a b c d e 24. a b c d e 25. a b c d e

13. I. The system is working with ___________ to gettingthings done.

II. ___________ must be commanded and notdemanded.

(a) status (b) relation(c) attitude (d) respect(e) honour

14. I. There is a complaint against him that he ___________the mistakes of his juniors.

II. A good thing about this house is that it ___________the sea.

(a) ignores (b) promotes(c) examines (d) overlooks(e) faces

15. I. Out of the total loans ___________ by the bank, thelargest share was for infrastructure.

II. The trees are ___________ throughout the area.(a) disbursed (b) covered(c) distributed (d) spanned(e) extended

16. I. Boats take more time going against the ___________of the river.

II. She keeps herself abreast of ___________ events.(a) low (b) latest(c) water (d) all(e) current

17. I. While trying to open the door, the ___________ broke.II. It is not difficult to ___________ tricky situations.(a) handle (b) knob(c) bracket (d) overcome(e) win

18. I. This course teaches you not to ___________ totemptations.

II. We hope to increase our ___________ of rice thisyear.

(a) succumb (b) produce(c) yield (d) share(e) submit

19. I. When you play your radio at high ___________ itdisturbs others.

II. We have just received a latest ___________ of thisencyclopedia.

(a) edition (b) volume

(c) channel (d) frequency(e) pitch

20. I. It helps to rinse one’s mouth early morning with a___________ of salt and water.

II. You can always refer to this reference material to findthe ___________ to these problems.

(a) mixture (b) answers(c) liquid (d) fix(e) solution

21. I. Our office decided to organize a party for the……………….. couple.

II. She …………… him in conversation to while away

some time.(a) new (b) engaged(c) pledged (d) held(e) encountered

22. I. It is required that you fill out these two ………………..to register for the job.

II. This subject has many practical ……………......... in dayto day life.

(a) forms (b) applications(c) relevance (d) statements (e) views

23. I. The next ……………….. of this case is after two months.II. After the accident her ……………….. has been affected.

(a) dates (b) balance(c) evidence (d) health(e) hearing

24. I. As a last ………………. we had to accept these termsand conditions.

II. This place has become a good tourist ………… now.

(a) resort (b) step(c) attraction (d) spot (e) means

25. I. The organization decided on an hourly minimum…………………….. of 35.

II. Will he ……………a war on these fronts to improvethings?

(a) pay (b) declare (c) pose(d) wage (e) campaign

Page 137: yoursmahboob.wordpress.com SBI · yoursmahboob.wordpress.com iii P 101 Speed Tests for SBI Bank Clerk Exam 101 Speed Tests for SBI Bank Clerk Exam is revised and updated edition on

yoursmahboob.w

ordpress.com

DIRECTIONS : Select the correct word or phrase to complete agrammatical sentence. In case of more than one blank, the differentwords given in the options shall fill in the corresponding order.

1. The patient was cheered ........ by the news that she waslikely to be discharged in a day or two.(a) on (b) up(c) out (d) down(e) None of these

2. The thieves had driven ten miles before the police caught........them.(a) on with (b) upon(c) up to (d) up with(e) None of these

3. The way he is currying favour ........ his rich neighbour issickening.(a) of (b) with(c) for (d) to(e) None of these

4. Because of his distrust........ every one, he could never gainany good friend.(a) by (b) of(c) with (d) in(e) None of these

5. The new Twenty Point Programme is designed to be arenewed assault ........ poverty.(a) for (b) into(c) on (d) upon(e) None of these

6. If you have a good project but are lacking ........finance, youshould enter into partnership ........an affluent person.(a) of, with (b) for, with(c) in, with (d) in, of(e) None of these

7. He was operated ........ an abscess in his leg.(a) for (b) against(c) on for (d) on against(e) None of these

8. Living a simple life ........ one’s won resources is preferable........ leading a luxurious life on borrowed funds.

Max. Marks : 35 No. of Qs. 35 Time : 20 min. Date : ........./......../................

(a) with, than (b) within, to(c) within, against (d) on, from(e) None of these

9. I doubt whether the accommodation in that place isadequate........our needs.(a) according to (b) with(c) for (d) against(e) None of these

10. The teacher warned the students to desist ........making noise.(a) from (b) about(c) with (d) by(e) None of these

11. It was customary ........ devotees going to that temple to takea bath in the nearby tank first.(a) with (b) on(c) for (d) about(e) None of these

12. Absorbed ........ his own thought, he paid scant attention towhat was happening ........ him.(a) with, about (b) in, around(c) by, to (d) of, besides(e) None of these

13. Though young, he proved a worthy rival ........ his opponentwho had much more experience.(a) to (b) against(c) for (d) with(e) None of these

14. He was of a charitable disposition, but did not like a numberof his relatives trying to live ........ him without trying to earntheir living.(a) with (b) near(c) off (d) through(e) None of these

15. The by-election, ........ closely fought, resulted ........ a surpriselandslide for the ruling party.(a) if, at (b) since, with(c) whether, in (d) though, in(e) None of these

RESPONSE

GRID

1. a b c d e 2. a b c d e 3. a b c d e 4. a b c d e 5. a b c d e

6. a b c d e 7. a b c d e 8. a b c d e 9. a b c d e 10. a b c d e

11. a b c d e 12. a b c d e 13. a b c d e 14. a b c d e 15. a b c d e

6161Correct Usage ofPreposition

Page 138: yoursmahboob.wordpress.com SBI · yoursmahboob.wordpress.com iii P 101 Speed Tests for SBI Bank Clerk Exam 101 Speed Tests for SBI Bank Clerk Exam is revised and updated edition on

yoursmahboob.w

ordpress.com

130 SPEED TEST 61

RESPONSE

GRID

16. a b c d e 17. a b c d e 18. a b c d e 19. a b c d e 20. a b c d e

21. a b c d e 22. a b c d e 23. a b c d e 24. a b c d e 25. a b c d e

26. a b c d e 27. a b c d e 28. a b c d e 29. a b c d e 30. a b c d e

31. a b c d e 32. a b c d e 33. a b c d e 34. a b c d e 35. a b c d e

16. You cannot be too sensitive........ criticism, if you have chosena political career.(a) about (b) with(c) for (d) to(e) None of these

17. ........ we are good friends his views differ ........mine on manyissues.(a) Despite, with (b) Because, off(c) Although, from (d) Nevertheless, form(e) None of these

18. A peculiar custom prevailing........ Toads is the sacrifice ofbuffaloes on ceremonial occasions.(a) over (b) with(c) by (d) among(e) None of these

19. However poor one may be, one can be happy only if one hasthe right attitude to life; happiness lies........ contentment.(a) through (b) in(c) with (d) over(e) None of these

20. Being himself very quick ........ arithmetical calculations, hedid not need a calculator.(a) ahout (b) for(c) over (d) in(e) None of these

21. Though accused of partiality........ his home team, the umpirehad a clear conscience and believed in the correctness ofhis decisions.(a) with (b) towards(c) for (d) against(e) None of these

22. Though I would not recommend it, I have no objection........your going to that movie.(a) for (b) against(c) to (d) upon(e) None of these

23. Do not force me to ........ you on this issue; I am not at allconvinced.(a) agree upon (b) concur with(c) join over (d) equate with(e) None of these

24. The court acquited him ........ all the charges.(a) from (b) against(c) for (d) of(e) None of these

25. There is no point in counting........ his support he isnotoriously undependable.(a) for (b) on(c) with (d) through(e) None of these

26. His claims of close acquaintance........ the high and the mightyare unbelievable.(a) about (b) towards(c) with (d) from(e) None of these

27. Compared........China, India’s progress in the agricultural fieldhas been commendable.(a) against (b) over(c) towards (d) to(e) None of these

28. The court found him guilty and imposed ........ him a fine ofRs.500.(a) to (b) for(c) upon (d) against(e) None of these

29. It is believed that hypertension is most often the cause ........heart attack.(a) towards (b) with(c) in (d) of(e) None of these

30. Ravi was ........ Nagpur ........ 2nd Jan, 67 ........4 in the morning.(a) at, on, at (b) in, on, at(c) in, in, about (d) at, at, at(e) None of these

31. .............. a very long time this city has been prosperous.(a) Since (b) For(c) From (d) Till(e) None of these

32. The mother of the dead child was overwhelmed .............grief.(a) by (b) with(c) from (d) for(e) None of these

33. There is no use discussing .............. prohibition.(a) on (b) about(c) of (d) for(e) None of these

34. .............. business, a merger is a combination of two or morecorporations under one management.(a) At (b) In(c) The (d) On(e) None of these

35. How is life enhanced? A beautiful passage from Tagorecomes .............. mind.(a) in (b) to(c) from (d) out(e) None of these

Page 139: yoursmahboob.wordpress.com SBI · yoursmahboob.wordpress.com iii P 101 Speed Tests for SBI Bank Clerk Exam 101 Speed Tests for SBI Bank Clerk Exam is revised and updated edition on

yoursmahboob.w

ordpress.com

DIRECTIONS (Qs.1-20) : Which of the following phrases (a),(b), (c) and (d) given below each sentences should replace thephrase printed in bold in the sentence to make it grammaticallycorrect? If the sentence is correct as it is given and ‘No correctionis required’, mark (e) as the answer.1. Starting out my own business at this time would affect the

financial stability of my family.(a) Starting up my (b) For starting with(c) To start out mine (d) By starting my(e) No correction required

2. Use a tactic for mixing the inferior with good quality rice isdishonest and you will lose your license.(a) Using tacti as (b) Using a tactic like(c) To use tactics (d) Used to tactics like(e) No correction required

3. The company will invest more six hundred crores in thenext five years to expand its operations in Britain.(a) will future invest (b) has invested more than(c) have invested over (d) will be invested above(e) No correction required

4. Several of our projects have delayed because the equipmentwe ordered was delivered late.(a) have been delayed when(b) delayed because of(c) are delayed since (d) were delayed with(e) No correction required

5. The committee has ruled out the possible raising taxes forthis financial year.(a) possibly raised (b) possible rise of(c) possibility to raise (d) possibility of raising(e) No correction required

6. The company has set up a foundation which helps studentswho do not have the necessary funds to study ahead.(a) further to study (b) of studying more(c) to study onward (d) for higher studies(e) No correction required

7. If this land is used to cultivate crops it will be additionallysource of income for the villagers.(a) a source of additional (b) an additionally source(c) an additional source (d) additionally the source(e) No correction required

RESPONSE

GRID

1. a b c d e 2. a b c d e 3. a b c d e 4. a b c d e 5. a b c d e

6. a b c d e 7. a b c d e 8. a b c d e 9. a b c d e 10. a b c d e

11. a b c d e 12. a b c d e 13. a b c d e 14. a b c d e 15. a b c d e

Max. Marks : 30 No. of Qs. 30 Time : 20 min. Date : ........./......../................

8. Belonged to this cadre, you are eligible for facilities such asfree air travel and accommodation.(a) Since you belong to (b) Whoever belongs(c) For belonging to (d) To belong in(e) No correction required

9. The bank has hired a consultant who will look into any issueswhich arise during the merger.(a) is looking over (b) will be looked after(c) will look out (d) looks down on(e) No correction required

10. I had severe doubts about if I successfully run a company,but my father encouraged me.(a) if I am successful in (b) how should I successfully(c) whether I would successfully(d) that I would succeed to(e) No correction required

11. As it was a dark and stormy night, Lata was too scared to gohome alone.(a) very scary to (b) much scared to(c) as scared to (d) to scared too(e) No correction required

12. Since it was her engagement party, Riya was dress to kill.(a) dresses to kill (b) dressed to kill(c) dressed to killings (d) dressing to killed(e) No correction required

13. Ramesh worries endlessly about his son’s future as he wasso poor in studies.(a) worry endless (b) worried endless(c) worried endlessly (d) worries endless(e) No correction required

14. Now that the actual criminal had been caught, Kunal washappy that he was finally let of the hook.(a) off the hook (b) of the hookings(c) off the hooks (d) of the hooks(e) No correction required

15. The little boy appeared all of a sudden out of nowhere andtake everyone by surprise.(a) took everyone as surprised(b) take everyone with surprised(c) took everyone by surprises(d) took everyone by surprise(e) No correction required

6262SentenceImprovement

Page 140: yoursmahboob.wordpress.com SBI · yoursmahboob.wordpress.com iii P 101 Speed Tests for SBI Bank Clerk Exam 101 Speed Tests for SBI Bank Clerk Exam is revised and updated edition on

yoursmahboob.w

ordpress.com

132 SPEED TEST 6216. A young and successful executive was travelling down a

neighbourhood street, going a bit to fast in his new car.(a) a bit too fastly (b) a bit as fast(c) a bit to fastly (d) a bit too fast(e) No correction required

17. All she could think about was the beautiful dress and howshe could earn enough money to buy it.(a) All she can think (b) All she could thought(c) All she can thought (d) All she can thinking(e) No correction required

18. He told his employer of his plans to leave the business tolead a more leisure life.(a) more leisurely life (b) many leisurely life(c) many leisured life (d) more leisurely live(e) No correction required

19. Padma could convince anyone with her talks as she had thegift of the gabbing.(a) gifting of the gabbing (b) gift of the gab(c) gifting of the gab (d) gift of the gab(e) No correction required

20. For countries undergoing a recession, large cuts in publicspending seem to be the ordering of the day.(a) be the ordering of days(b) being the order of the day(c) be the order of the day(d) being the ordering of days(e) No correction required

DIRECTIONS (Qs.21-30) : In each of the following sentences,an idiomatic expression or a proverb is highlighted. Select thealternative which best describes its use in the sentence.21. The team put their plan into execution the very next day.

(a) proposed a plan.(b) discussed their plan.(c) started thinking about a plan(d) started carrying out their plan.(e) None of these

22. Mrs. Nayak opened the discussion on the “alarming rate ofpoverty in India”.(a) started the discussion.(b) gave her opinion in the discussion.(c) did not agree on the discussion.(d) welcomed the people to the discussion.(e) None of these

23. The course of events made it necessary for Joseph to start working.(a) events that were planned.(b) long list of future events.(c) A succession of unexpected events.(d) nature of events that followed after Joseph joined work.(e) None of these

24. The new law on “Right to Food Safety” will come into forcenext month.(a) be forced upon the people.(b) be associated from next month onwards.(c) be implemented next month.(d) be withdrawn next month.(e) be widely rejected next month.

25. When the girl wanted to stay out past midnight, her fatherput his foot down.(a) gave in to her request(b) walked away disapprovingly.(c) obstructed her from leaving the house.(d) requested her to be home on time.(e) None of these

26. In all likelihood the missing boy has run away to the forest(a) with good intentions (b) there's no chance(c) without doubt (d) in most probability(e) None of these

27. The parents were completely in the dark concerning theirdaughter's plans.(a) ignorant about (b) ashamed of(c) pretending to be unaware(d) unhappy about (e) None of these

28. I am in touch with the police, and they will be here in ten minutes.(a) in communication with(b) in close proximity with(c) in good terms with (d) familiar with(e) None of these

29. I stumbled upon some interesting old letters in myGrandfather's desk.(a) deliberately went through(b) surveyed (c) tripped over(d) discovered by chance (e) None of these

30. The secretary made an entry of the arrangement.(a) initiated discussion (b) made a record(c) brought notice (d) showed approval(e) None of these

RESPONSE

GRID

16. a b c d e 17. a b c d e 18. a b c d e 19. a b c d e 20. a b c d e

21. a b c d e 22. a b c d e 23. a b c d e 24. a b c d e 25. a b c d e

26. a b c d e 27. a b c d e 28. a b c d e 29. a b c d e 30. a b c d e

Page 141: yoursmahboob.wordpress.com SBI · yoursmahboob.wordpress.com iii P 101 Speed Tests for SBI Bank Clerk Exam 101 Speed Tests for SBI Bank Clerk Exam is revised and updated edition on

yoursmahboob.w

ordpress.com

DIRECTIONS (Qs.1-41) : Read each sentence to find out whetherthere is any grammatical error or idiomatic error in it. The error, ifany, will be in one part of the sentence. The number of that partis the answer. If there is no error, the answer is (e). (Ignore errorsof punctuation, if any.)1. His proposal had (a) / to be send to (b) / the President of

the company (c) / for her approval (d). No error (e).2. Each tuesday evening we visited (a) / the farmers in the

area (b) / and held a meeting (c) / to discuss the problemsthey faced (c). No error (e).

3. Though our training facilities (a) / are limited only a (b) /few employees have been (c) / selected for training (d). Noerror (e).

4. During the interview (a) / the panel asked me (b) / severaltechnical questions (c) / and I answered all of it (d). Noerror (e).

5. He decided to work for (a) / an NGO, but most of his (b) /classmates opted for high paid (c) / jobs in multinationalcompanies (d). No error (e).

6. It is necessarily to maintain (a)/ a record of all transactions(b) / in case the auditors (c)/ want to see it. (d)/ No error (e).

7. Very few young trainees (a)/ willingly undertake (b)/ aposting to a branch (c)/ located in a rural area (d)/ No error(e).

8. He has travelled (a)/ all over the world (b)/ yet he speaks(c)/ several languages fluently (d)/. No error (e).

9. A successful company is (a)/ any that makes a good (b)/profit and provides (c)/ high returns to its shareholders (d).No error (e).

RESPONSE

GRID

1. a b c d e 2. a b c d e 3. a b c d e 4. a b c d e 5. a b c d e

6. a b c d e 7. a b c d e 8. a b c d e 9. a b c d e 10. a b c d e

11. a b c d e 12. a b c d e 13. a b c d e 14. a b c d e 15. a b c d e

16. a b c d e 17. a b c d e 18. a b c d e 19. a b c d e 20. a b c d e

Max. Marks : 41 No. of Qs. 41 Time : 25 min. Date : ........./......../................

10. The agreement on (a)/ which all of us have (b)/ worked sohard will (c)/ be sign tomorrow (d). No error (e).

11. Nuclear waste will still being (a)/ radioactive even after twentythousand years, (b)/ so it must be disposed (c)/ of verycarefully. (d)/ No error (e)

12. My friend lived at the top (a)/ of an old house (b)/ which attichad been (c)/ converted into a flat. (d)/ No error (e)

13. A public safety advertising (a)/ campaign in Russia (b)/ hopeto draw attention (c)/ of pedestrians crossing the road.(d)/ No error (e)

14. A cash prize was (a)/ award to the most (b)/ successful salesmanof the year (c)/ by the President of the company. (d)/ No error(e)

15. The Renaissance was (a)/ a time to ‘re-awakening’ (b)/ in boththe arts (c)/ and the sciences. (4)/ No error (e)

16. In times of crisis, (a) / the Bhagavad Gita gives light (b)/ andguide to the mind tortured by doubt (c)/ and torn by conflictof duties. (d)/ No Error (e)

17. It was not easy for late Raja Ram Mohan Roy (a)/ to root outthe custom of sati (b) / because a majority of (c)/ the educatedclass does not support him. (d) / No Error (e)

18. Deplete of the Ozone layer (a) / and the greenhouse effect (b)/ are two long-term effects (c)/of air pollution. (d)/ No Error (e)

19. Most of the people which (a)/ have been victims (b) / of extremeviolence (c)/ are too frightened to report it to the police. (d)/No Error (e)

20. The doctor helps (a)/ to reducing human suffering (b)/bycuring diseases (c)/ and improving health. (d)/ No Error (e)

6363Spotting theErrors - I

Page 142: yoursmahboob.wordpress.com SBI · yoursmahboob.wordpress.com iii P 101 Speed Tests for SBI Bank Clerk Exam 101 Speed Tests for SBI Bank Clerk Exam is revised and updated edition on

yoursmahboob.w

ordpress.com

134 SPEED TEST 63

21. The shepherd counted (a) / his sheep and found (b) / thatone of (c) / them is missing. (d) / No Error (e)

22. The teacher were (a) / impressed by her performance (b) /and asked her to (c) / participate in the competition. (d) NoError (e)

23. She asked her (a) / son for help her (b) / find a place to bury(c) / the gold ornaments (d) No Error (e)

24. The painter was (a) / ask to paint a (b) / picture of the king,(c) / sitting on his throne (d) No Error (e)

25. The story was (a) / about how an (b) / intelligent man hadsaving (c) / himself from being robbed (d) No Error (e)

26. The decline of her moral (a) / was caused by a lot of (b) /factors that were once (c) / fascinating to her. (d) / No Error(e)

27. He took me to a restaurant (a) / and ordered for two cups(b) / of cold coffee (c) / which the waiter brought in an hour.(d) / No Error (e)

28. There are some animals (a) / that can live (b) / both in wateran land (c) / without any difficulty. (d) / No Error (e)

29. During his tour (a) / to the south (b) he visited not only toChennai (c) / but also Karnataka. (d) / No Error (e)

30. The President Mr. Kalam (a) / is much sought after (b) / byschool students and (c) / is invited for many functions.(d) / No Error (e)

31. His mother is not well (a) / but he (b) / does not look for her(c) / properly. (d) / No Error (e)

32. We may have to await for (a) / a new political revival (b) / toeradicate the (c) / corruption from our economy. (d) / NoError (e)

33. When she was (a) / in jail (b) / she was debarred to send(c) / a letter even to her son. (d) / No Error (e)

34. Despite of the best efforts (a) / put by the doctors (b) / thecondition of the patient (c) / is deteriorating from bad toworse. (d) / No Error (e)

35. The militant yielded for (a) / the temptation and fell (b) / intothe trap (c) / of police. (d) / No Error (e)

36. Many people in India (a) / are dying from hunger (b) / butgovernment seems (c) / to be ignorant of such crude fact. (d)/ No Error (e)

37. In difficult time (a) she prefers keeping her counsel (b) ratherthan wandering (c) / here and there for relief. (d) / No Error(e)

38. The persons who are (a) / suffering from diabetes are (b) /advised to substitute (c) / saccharine by sugar. (d) / No Error(e)

39. He always says (a) / that he prefers to go (b) / home to stayin (c) / a hotel at night. (d) / No Error (e)

40. Hardly had we settled down (a) / for the rest (b) / when wewere startled by the (c) / strange sound of trumpets. (d) / NoError (e)

41. He was able to (a) / free himself with (b) / the debts by (c) /working day and night. (d) / No Error (e)

RESPONSE

GRID

21. a b c d e 22. a b c d e 23. a b c d e 24. a b c d e 25. a b c d e

26. a b c d e 27. a b c d e 28. a b c d e 29. a b c d e 30. a b c d e

31. a b c d e 32. a b c d e 33. a b c d e 34. a b c d e 35. a b c d e

36. a b c d e 37. a b c d e 38. a b c d e 39. a b c d e 40. a b c d e

41. a b c d e

Page 143: yoursmahboob.wordpress.com SBI · yoursmahboob.wordpress.com iii P 101 Speed Tests for SBI Bank Clerk Exam 101 Speed Tests for SBI Bank Clerk Exam is revised and updated edition on

yoursmahboob.w

ordpress.com

DIRECTIONS : Read each sentence to find out whether there isany error in it. The error, if any, will be in one part of the sentence.The number of this part is the answer. If there is no error, theanswer is (e).

1. The banker’s association (a) / has submitted a memorandum(b) / for the fulfilment of (c) / their demands. (d) / No Error(e)

2. Five quintals of wooden coal (a) / are (b) / his annualrequirement (c) / for the unit. (d) / No Error (e)

3. Dickens have (a) / vehemently criticised (b) / the philosophy(c) in ‘Hard Times’. (d) / No Error (e)

4. All his money (a) / is spent (b) / and all his (c) / hopesruined. (d) / No Error (e)

5. This rule may (a) / and ought to be (b) / disregarded for (c)/ the time being. (d) / No Error (e)

6. Why come people don't get (a) / what they deserve (b) /and why others get what they don't deserve (c) / is a matterdecided by luck. (d)/No error(e)

7. The committee is thankful to Mr. Roy (a) / for preparing notonly the main report (b) / but also for preparing (c) theagenda notes and minutes. (d) / No error (e)

8. In order to save petrol (a ) / motorists must have to (b) / bevery cautious (c) / while driving along the highways. (d) /No error (e)

9. No country can long endure (a) / if its foundations (b) /were not laid deep (c) / in the material prosperity. (d) / Noerror (e)

10. Due to certain inevitable circumstances (a) / the scheduledprogramme had to be (b) / post poned indefinite (c) / butthe members could not be informed. (d) / No error (e)

RESPONSE

GRID

1. a b c d e 2. a b c d e 3. a b c d e 4. a b c d e 5. a b c d e

6. a b c d e 7. a b c d e 8. a b c d e 9. a b c d e 10. a b c d e

11. a b c d e 12. a b c d e 13. a b c d e 14. a b c d e 15. a b c d e

16. a b c d e 17. a b c d e 18. a b c d e 19. a b c d e 20. a b c d e

Max. Marks : 40 No. of Qs. 40 Time : 25 min. Date : ........./......../................

11. All three products help wean smokers (a) / from cigarettes byproviding small doses (b) / of nicotine designed to replacethe basic level (c) / of average smoker's day's nicotineconsumption. (d) No error (e)

12. The main lesson is that (a) / the mosquito eradicationcampaigns (b) / such as the one the French conducted (c) /needs to be followed up. ( d) / No error (e)

13. By arresting the local criminals (a) / and encouraging goodpeople, (b) / we can end (c) / hostilities of that area (d) / Noerror (e)

14. We admired thre way(a) / he had completed all his work (b) /and appreciation the method (c) / adopted by him. (d) / Noerror

15. I was being astonished (a) / when I heard that (b) / he had leftthe country (c) / without informing anyone of us . (d) / No.error (e)

16. The Head of the Department, along with his colleagues (a) /are coming to attend (b) / the conference which is (c) /scheduled this afternoon. (d) / No error (e)

17. Govind loved his Guru immensely (a) / and gave him fullestloyalty, (b) / yet he had his own (c) / independent way ofthinking (d) / No error (e)

18. In a very harsh tone, (a) / he shouted at his servants (b) / andtold them that (c) / he does not neEid their services (d) ./ Noerror (e)

19. This is an important difference in (a) / that it marks the firstmove towards (b) / an institution for money - earningproposition. (d) / No error (e)

20. The ultimate problem of physics (a)/ is to reduce matter byanalysis (b) / to its lowest condition of divisibility (c) / No.error(d)

6464Spotting theErrors - II

Page 144: yoursmahboob.wordpress.com SBI · yoursmahboob.wordpress.com iii P 101 Speed Tests for SBI Bank Clerk Exam 101 Speed Tests for SBI Bank Clerk Exam is revised and updated edition on

yoursmahboob.w

ordpress.com

136 SPEED TEST 6421. It was astonished (a) / to find that I scarcely had (b) / enough

money (c) / to pay the bills. (d) / No error (e)

22. We can not handle (a)./ this complicated case to day (b) /unless full details are not given (c) / to us by now. (d) / Noerror (e)

23. Even after worked in the office (a) / for as many as fifteenyears, (b) / he still does not understand (c) / the basicobjectives of the work. (d) / No error (e)

24. Neither the earthquake (a) / nor the subsequent fire ( b) /was able to dampen (c) / the spirit of the residents, (d) / Noerror (e)

25. Our school is making (a) / every possible effort (b) / to providebest facilities (c) / land person attention for each child (d) /No error (e)

26. Our neighbours had repeated (a) / the same illogical sequenceof activites (b) / if we had not brought the (c) /facts to theirnotice. ( d) / No error (e)

27. This has forced them to focus at (a) I how to reach and servetheir customers, (b) / rather than, say, pumping money (c) /into fancy graphics that look good in management meetings.(d) / No error (e)

28. The loss of forests thus entail (a) / large social and economiccosts; (b) the lives of more than one billion people (c) / arealready affected by loss of this forest cover (d) / No error (e)

29. Not one of the children (a) / has ever sang (b) / on anyoccasion (c) / in public before. (d) / No error (e)

30. Reasonable ambition, if supported (a) / at persistent efforts,(b) / is likely to yield (c) / the desied results. (d) / No error (e)

31. I would have lost (a) / my luggage and other belonging (b) /if I would have left the compartment (c) / and gone out tofetch drinking water. (d) / No errro (e)

32. Whether this happens, and whether the BIFR will (a) / onceagain reconsiger the Sirmour package (b) / are questionsthose will be (c) / answered in the comming months. (d) / Noerror (e)

33. We now look forward for (a) / some great achievements (b) /which to some extent (c) / can restore the country's prestigeonce again (d) / No error (e)

34. Honesty and integrity are (a) / the qualityes which cannotbe (b) / done away with (c) / and hence assume a lot ofimportance (d) / No error (e)

35. The foreign funds are cheaper than those (a) / available thedomestic market and (b) / the company is competent that (c)/ it will soon touch its earlier annual turnover, (d) / No error(e)

36. Honesty, integrity and being intelligent (a) / are the qualitieswhich (b) / we look for when (c) / we interview applicants (d)/ No error (e)

37. Not only the judges acquitted (a) / him of all the charges (b)/ levelled against him, but (c) / also commended all his actions.(d) /No error (e)

38. One of the most effective (a) / solutions is that (b) / sheshould work on Sunday (c) / and complete the assignment.(d) / No error (e)

39. Our system of assigning (a) I different jobs to different people(b) / should be based on (c) / their strengths and weaknesses(d) / No error (e)

40. There he stood on the dais, (a) / debunked the manner inwhich the company was run ( b) / by the former chairmanand managing director, (c) / and promising higher growthand industry. (d) No error (e)

RESPONSE

GRID

21. a b c d e 22. a b c d e 23. a b c d e 24. a b c d e 25. a b c d e

26. a b c d e 27. a b c d e 28. a b c d e 29. a b c d e 30. a b c d e

31. a b c d e 32. a b c d e 33. a b c d e 34. a b c d e 35. a b c d e

36. a b c d e 37. a b c d e 38. a b c d e 39. a b c d e 40. a b c d e

Page 145: yoursmahboob.wordpress.com SBI · yoursmahboob.wordpress.com iii P 101 Speed Tests for SBI Bank Clerk Exam 101 Speed Tests for SBI Bank Clerk Exam is revised and updated edition on

yoursmahboob.w

ordpress.com

DIRECTIONS: Choose the correct spelling of the given word.

1. (a) Coimmission (b) Comision

(c) Comission (d) Commision

(e) None of these

2. (a) Jewelery (b) Jewellry

(c) Jwellry (d) Jewellery

(e) None of these

3. (a) Sattellite (b) Satellite

(c) Sattelite (d) Satelite

(e) None of these

4. (a) Ocasion (b) Ocassion

(c) Occasion (d) Occassion

(e) None of these

5. (a) Comettee (b) Committe

(c) Comittee (d) Committee

(e) None of these

6. (a) Achievment (b) Acheivment

(c) Achievement (d) Achevement

(e) None of these

7. (a) Hetrogenous (b) Hetrogeneous

(c) Heterogenous (d) Heterogeneous

(e) None of these

8. (a) Foreigner (b) Forienor

(c) Foriegnor (d) Foreiner

(e) None of these

9. (a) Colaboration (b) Collaberation

(c) Colaberation (d) Collaboration

(e) None of these

10. (a) Acurrate (b) Accurate

(c) Acurate (d) Accuratte

(e) None of these

RESPONSE

GRID

1. a b c d e 2. a b c d e 3. a b c d e 4. a b c d e 5. a b c d e

6. a b c d e 7. a b c d e 8. a b c d e 9. a b c d e 10. a b c d e

11. a b c d e 12. a b c d e 13. a b c d e 14. a b c d e 15. a b c d e

16. a b c d e 17. a b c d e 18. a b c d e 19. a b c d e 20. a b c d e

Max. Marks : 40 No. of Qs. 40 Time : 25 min. Date : ........./......../................

11. (a) Entreprenuer (b) Entrepraneur

(c) Entrapreneur (d) Entrepreneur

(e) None of these

12. (a) Skillful (b) Skillfull

(c) Skilful (d) Skilfull

(e) None of these

13. (a) Varstile (b) Verstile

(c) Versatile (d) Vorstyle

(e) None of these

14. (a) Correspondant (b) Corraspondent

(c) Corraspondant (d) Correspondent

(e) None of these

15. (a) Etiquete (b) Etiquette

(c) Ettiquete (d) Ettiquette

(e) None of these

16. (a) Necessary (b) Necesarry

(c) Necesary (d) Neccessary

(e) None of these

17. (a) Homeopathy (b) Homoepathy

(c) Homiopathy (d) Homoeopathy

(e) None of these

18. (a) Lieutinant (b) Lieutenant

(c) Leutenant (d) Liutenant

(e) None of these

19. (a) Paralelogram (b) Paralellogram

(c) Parallelogram (d) Parallellogram

(e) None of these

20. (a) Milennium (b) Millenium

(c) Millennium (d) Milenium

(e) None of these

6565Spelling Test

Page 146: yoursmahboob.wordpress.com SBI · yoursmahboob.wordpress.com iii P 101 Speed Tests for SBI Bank Clerk Exam 101 Speed Tests for SBI Bank Clerk Exam is revised and updated edition on

yoursmahboob.w

ordpress.com

138 SPEED TEST 65

RESPONSE

GRID

21. a b c d e 22. a b c d e 23. a b c d e 24. a b c d e 25. a b c d e

26. a b c d e 27. a b c d e 28. a b c d e 29. a b c d e 30. a b c d e

31. a b c d e 32. a b c d e 33. a b c d e 34. a b c d e 35. a b c d e

36. a b c d e 37. a b c d e 38. a b c d e 39. a b c d e 40. a b c d e

DIRECTIONS (Qs. 21-40) : In each question below a sentencewith four words printed in bold type is given. These are numberedas (a), (b), (c) and (d). One of these four words printed in bold maybe either wrongly spelt or inappropriate in the context of thesentence. Find out the word which is wrongly spelt or inappropriateif any. The number of that word is your answer. If all the wordsprinted in bold are correctly spelt and also appropriate in thecontext of the sentence, mark (e) i.e. ‘All correct’ as your answer.

21. Under existing (a) / regulations we are not permitted (b) / toowe (c) / more than a forty percent share (d) / of the familybusiness. All correct (e).

22. In case of any land dispute (a) / panchayat officials (b) / willdetermine (c) / how the property is to be dividend (d) / Allcorrect (e).

23. The World Bank has consented (a) / to sanction (b) / thenecessary (c) / finance (d) / for the project. All correct (e).

24. To obtain (a) / a refund you will have to fill (b) / a claim (c)/ with the appropriate (d) / authority. All correct (e).

25. Experts predict (a) / there will be shortage (b) / of investment(c) / in the infrastructure (d) / sector. All correct (c).

26. In order to succeed (a)/ it is crucial (b)/ for an organisationto constantly (c)/ improve (d)/. All correct (e).

27. With some assistance (a)/ from her son she was enable (b)/to settle (c)/ her debts (d)/ on time. All correct (e).

28. We have prepared a detailed (a)/ report giving various (b)/solutions (c)/ to resort (d)/ the problem. All correct (e).

29. RBI has attempted (a)/ to spend (b)/ financial (c)/ awareness(d) / through this programme. All correct (e).

30. Though the government initiated (a)/ a large sum (b)/ ofmoney in the scheme (c)/ it was a failure (d)/. All correct (e).

31. It is not unusual (a)/for guests of the hotel to carry(b)/souveniers(c)/ back with them when they return (d)/to theirhomes. All correct (e)

32. She vested(a)/ her time in chatting(b)/ over the phone andultimately(c)/ ended up not finishing (d)/ her work. Allcorrect (e)

33. She had not eaten(a)/ anything(b)/ for a very long time nowand her stomach(c)/ was groling(d)/ All correct(e)

34. Half of the harm(a)/ that is done in this world(b)/ is due topeople(c)/ who want to feel important(d)/ All correct(e)

35. Life is like a mirror(a)/ smile at it and its charmeng(b)/frown(c)/ at it and it becomes sinister(d)/ All correct(e)

36. The Whole (a)/ time she walked with her child in her armsthe only thing (b)/ that worried (c)/ her was her son’s feature.(d)/ All correct (e)

37. When the young artist returned (a)/ to his village, his familyheld a festive (b)/ dinner on it’s lawn to celebrate his triumpant(c)/ homecoming. (d)/ All correct (e)

38. Had she not suppressed (a)/ all the details of her Company’sproject (b)/ her Company would have bagged (c)/ thecontract. (d)/ All correct (e)

39. She trusted Mira with all her heart (a)/ and thus handled (b)/over her life’s (c)/ savings to her instantly. (d)/ All correct (e).

40. It is difficullt (a)/ to see the picture (b)/ when you are inside(c)/ the frame. (d)/ All correct (e)

Page 147: yoursmahboob.wordpress.com SBI · yoursmahboob.wordpress.com iii P 101 Speed Tests for SBI Bank Clerk Exam 101 Speed Tests for SBI Bank Clerk Exam is revised and updated edition on

yoursmahboob.w

ordpress.com

DIRECTIONS (Qs.1-25) : In each of the following questions,out of the four alternatives, choose the one which can besubstituted for the given words/sentence.

1. List of headings of the business to be transacted at ameeting(a) Schedule (b) Agenda(c) Proceedings (d) Excerpts(e) None of these

2. Through which light cannot pass(a) Dull (b) Dark(c) Obscure (d) Opaque(e) None of these

3. Stealing from the writings of others(a) Copying (b) Reframing(c) Reproducing (d) Plagiarism(e) None of these

4. Constant effort to achieve something(a) Perseverance (b) Attempt(c) Enthusiasm (d) Vigour(e) None of these

5. A person not sure of the existence of God(a) Theist (b) Atheist(c) Agnostic (d) Cynic(e) None of these

6. One who deserts his religion(a) Deserter (b) Turn-coat(c) Fanatic (d) Apostate(e) None of these

7. One who uses fear as a weapon of power(a) Terrorist (b) Militant(c) Extremist (d) Anarchist(e) None of these

RESPONSE

GRID

1. a b c d e 2. a b c d e 3. a b c d e 4. a b c d e 5. a b c d e

6. a b c d e 7. a b c d e 8. a b c d e 9. a b c d e 10. a b c d e

11. a b c d e 12. a b c d e 13. a b c d e 14. a b c d e 15. a b c d e

Max. Marks : 30 No. of Qs. 30 Time : 20 min. Date : ........./......../................

8. The original inhabitants of a country(a) Aborigines (b) Citizens(c) Natives (d) Primitive(e) None of these

9. One desirous of getting money(a) Avaracious (b) Voracious(c) Garrulous (d) Greedy(e) None of these

10. Be the embodiment or perfect example of(a) Signify (b) Characterise(c) Personify (d) Masquerade(e) None of these

11. Cutting for stone in the bladder(a) Dichotomy (b) Tubectomy(c) Vasectomy (d) Lithotomy(e) None of these

12. That which makes it difficult to recognise the presence of realnature of somebody or something(a) Cover (b) Mask(c) Pretence (d) Camouflage(e) None of these

13. Yearly celebration of a date or an event(a) Centenary (b) Jubilee(c) Anniversary (d) Birthday(e) None of these

14. One who has suddenly gained new wealth, Power or prestige(a) Aristocrat (b) Affluent(c) Maverick (d) Parvenu(e) None of these

15. Code of diplomatic etiquette and precedence(a) Formalism (b) Statesmanship(c) Protocol (d) Hierarchy(e) None of these

6666One WordSubstitution

Page 148: yoursmahboob.wordpress.com SBI · yoursmahboob.wordpress.com iii P 101 Speed Tests for SBI Bank Clerk Exam 101 Speed Tests for SBI Bank Clerk Exam is revised and updated edition on

yoursmahboob.w

ordpress.com

140 SPEED TEST 66

16. Of outstanding significance(a) Meaningful (b) Ominous(c) Evident (d) Monumental(e) Rational

17. One who promotes the idea of absence of government ofany kind, when every man should be a law into himself(a) Anarchist (b) Belligerent(c) Iconoclast (d) Agnostic(e) None of these

18. Land so surrounded by water as to be almost an island(a) Archipelago (b) Isthmus(c) Peninsula (d) Lagoon(e) None of these

19. That which cannot be done without(a) Irrevocable (b) Impracticable(c) Indispensable (d) Impossible(e) None of these

20. One who travels from place to place(a) Itinerant (b) Mendicant(c) Journeyman (d) Tramp(e) None of these

21. An act or notion to look back in the past(a) Retrospective (b) Postnatal(c) Retrogressive (d) Primitive(e) None of these

22. Medicine to counteract the effect of a poison(a) Emetic (b) Antidote(c) Anti-venom (d) Antiseptic(e) None of these

23. A collection of poems(a) Pathology (b) Anthology(c) Oncology (d) Pedology(e) None of these

24. One who studies mankind(a) Anthropologist (b) Physicist(c) Pathologist (d) Philanthropist(e) None of these

DIRECTIONS (Qs.26-30) : Choose the one word for the italicsentences.

25. An opinion contrary to popular belief(a) Paradox (b) Orthodoxy(c) Hearsay (d) Heresy(e) None of these

26. Rajesh was a hater of learning and knowledge.(a) misogynist (b) misologist(c) misanthropist (d) bibliophile(e) None of these

27. The bus has to go back and forth every six hours.(a) travel (b) run(c) cross (d) shuttle(e) commute

28. A man can be sentenced to death for killing another humanbeing.(a) fratricide (b) regicide(c) homicide (d) genocide(e) None of these

29. A careful preservation and protection of wildlife is the needof the hour.(a) Embarkment (b) Promotion(c) Conservation (d) Management(e) Enhancement

30. The officer was not willing to take a definite stand on thatpoint.(a) vague (b) evasive(c) ambiguous (d) complex(e) None of these

RESPONSE

GRID

16. a b c d e 17. a b c d e 18. a b c d e 19. a b c d e 20. a b c d e

21. a b c d e 22. a b c d e 23. a b c d e 24. a b c d e 25. a b c d e

26. a b c d e 27. a b c d e 28. a b c d e 29. a b c d e 30. a b c d e

Page 149: yoursmahboob.wordpress.com SBI · yoursmahboob.wordpress.com iii P 101 Speed Tests for SBI Bank Clerk Exam 101 Speed Tests for SBI Bank Clerk Exam is revised and updated edition on

yoursmahboob.w

ordpress.com

DIRECTIONS (Qs. 1 to 5): Rearrange the following five sentences(A), (B), (C), (D) and (E) in the proper sequence to form a meaningfulparagraph, then answer the questions given below them.

A. It will take extraordinary political commitment and liberalpublic funding during the 11th Plan for affordable housingto become a credible goal.

B. The National Urban Housing and Habitat Policy of theUnited Progressive Alliance Government seeks to makeaccess to housing, long acknowledged as a fundamentalright, a reality for all.

C. The task is staggering even if we go by conservative estimates.D. The housing shortage to be met during the Plan is 26.53

million units, which include the backlog from the 10th Plan.E. If the existing stock of poor quality dwellings and the

growing urbanization–driven demand are taken intoaccount, the real deficit will be even higher.

1. Which of the following should be the FIRST sentence?(a) A (b) B (c) C(d) D (e) E

2. Which of the following should be the SECOND sentence?(a) A (b) B (c) C(d) D (e) E

3. Which of the following should be the THIRD sentence?(a) A (b) B (c) C(d) D (e) E

4. Which of the following should be the FOURTH sentence?(a) A (b) B (c) C(d) D (e) E

5. Which of the following should be the FIFTH (LAST)sentence?(a) A (b) B (c) C(d) D (e) E

DIRECTIONS (Qs. 6 to10): Rearrange the following five sentences(A), (B), (C), (D) and (E) and in the proper sequence to form ameaningful paragraph, then answer the questions given below them.A. The upsurge of public activism against the setting up of

Special Economic Zones, which eventually forced the StateGovernment to announce the scrapping of all 15 suchprojects, is an impressive case in point.

B. Early last year, a similar agitation coerced the government intocalling for a revision of the Goa Regional Plan 2011, acontroversial document that opened up large swathes of land,including green belts and coastal stretches, for construction.

Max. Marks : 30 No. of Qs. 30 Time : 20 min. Date : ........./......../................

C. The broad–based agitation against SEZs has demonstratedthe power of popular protest in the State.

D. Those opposed to the projects had questioned the proprietyof the government acquiring large tracts of land and thenselling them to promoters at low prices.

E. A coastal State with an area of 3,700 square kilometers and apopulation of about 1.4 million, Goa has always been extremelysensitive to the impact of unrestrained economic development.

6. Which of the following should be the FIRST sentence?(a) A (b) B (c) C(d) D (e) E

7. Which of the following should be the SECOND sentence?(a) A (b) B (c) C(d) D (e) E

8. Which of the following should be the THIRD sentence?(a) A (b) B (c) C(d) D (e) E

9. Which of the following should be the FOURTH sentence?(a) A (b) B (c) C(d) D (e) E

10. Which of the following should be the FIFTH (LAST) sentence?(a) A (b) B (c) C(d) D (e) E

DIRECTIONS (Qs. 11 to 15): Rearrange the following five sentences(A), (B), (C), (D) and (E) in the proper sequence to form a meaningfulparagraph, then answer the questions given below them.

A. The British government plans to insist that spouses shouldhave to learn English before they are allowed into Britain tojoin their husbands or wives have run into a barrage ofopposition and warnings that the idea could breach humanrights laws.

B. The responses to an official consultation on the proposalpublished on Thursday was more than two to one against theproposal, with many warning it could break up marriagesbecause many cannot afford or access English lessons.

C. Immigration lawyers have told ministers that spouses andfiances should not be barred from joining a partner in theU.K. for language reasons and that the plan could breach thehuman rights convention's guarantees to the right to marryand have a family life.

D. The anonymised responses were 68 to 31 against the pre-entry english test for spouses.

E. Other immigration organizations said the measure woulddiscriminate against those from rural areas in South Asia,where the opportunities to learn English are limited.

RESPONSE

GRID

1. a b c d e 2. a b c d e 3. a b c d e 4. a b c d e 5. a b c d e

6. a b c d e 7. a b c d e 8. a b c d e 9. a b c d e 10. a b c d e

6767Para Jumbles

Page 150: yoursmahboob.wordpress.com SBI · yoursmahboob.wordpress.com iii P 101 Speed Tests for SBI Bank Clerk Exam 101 Speed Tests for SBI Bank Clerk Exam is revised and updated edition on

yoursmahboob.w

ordpress.com

142 SPEED TEST 67

RESPONSE

GRID

11. a b c d e 12. a b c d e 13. a b c d e 14. a b c d e 15. a b c d e

16. a b c d e 17. a b c d e 18. a b c d e 19. a b c d e 20. a b c d e

21. a b c d e 22. a b c d e 23. a b c d e 24. a b c d e 25. a b c d e

26. a b c d e 27. a b c d e 28. a b c d e 29. a b c d e 30. a b c d e

11. Which of the following should be the FIRST sentence?(a) A (b) B (c) C(d) D (e) E

12. Which of the following should be the SECOND sentence?(a) A (b) B (c) C(d) D (e) E

13. Which of the following should be the THIRD sentence?(a) A (b) B (c) C(d) D (e) E

14. Which of the following should be the FOURTH sentence?(a) A (b) B (c) C(d) D (e) E

15. Which of the following should be the FIFTH (LAST)sentence?(a) A (b) B (c) C(d) D (e) E

DIRECTIONS (Qs. 16 to 20) : Rearrange the following six sentences(A), (B), (C), (D), (E) and (F) in the proper sequence to from ameaningful paragraph; then answer the questions given below them.(A) He immediately acknowledged Mohan’s good work and

invited him to his home for dinner.(B) One day a wealthy merchant sent his son’s bicycle to the

shop for repair.(C) The next day the merchant came to claim the bicycle and

noticed that it was shiny.(D) After repairing the bicycle, Mohan cleaned it up and made it

look new.(E) Once upon a time, there was a boy named Mohan who

worked as an apprentice in a bicycle shop.(F) Other apprentices in the shop laughed at Mohan for doing

unnecessary work.16. Which of the following should be the SECOND sentence

after rearrangement?(a) A (b) B (c) C(d) D (e) F

17. Which of the following should be the THIRD sentence afterrearrangement?(a) A (b) B (c) C(d) D (e) E

18. Which of the following should be the FIRST sentence afterrearrangement?(a) A (b) B (c) C(d) D (e) E

19. Which of the following should be the LAST (SIXTH)sentence after rearrangement?(a) A (b) B (c) D(d) E (e) F

20. Which of the following should be the FOURTH sentenceafter rearrangement?(a) B (b) C (c) D(d) E (e) F

DIRECTIONS (Qs. 21 to 25) : Rearrange the following sixsentences (A), (B), (C), (D), (E) and (F) in the proper sequence tofrom a meaningful paragraph; then answer the questions givenbelow them.(A) At first he got scared, but then he thought, “I have never

worshipped her; that is why I am not able to get anythingfrom my land.”

(B) One day unable to tolerate the summer heat, he went to restunder a big banyan tree.

(C) He rushed to his village and placed his humble offering ofmilk in a bowl before the snake.

(D) Vishnu Raman was a poor Brahmin and a farmer by profession.(E) The next day when he returned, he was rewarded with a gold

coin in the bowl he left behind.(F) Just as he was preparing to lie down he saw a huge cobra

swaying with his hood open.21. Which of the following should be the SECOND sentence

after rearrangement?(a) B (b) C (c) E(d) D (e) F

22. Which of the following should be the FIRST sentence afterrearrangement?(a) A (b) D (c) F(d) C (e) E

23. Which of the following should be the FIFTH sentence afterrearrangement?(a) F (b) D (c) C(d) B (e) E

24. Which of the following should be the SIXTH (LAST)sentence after rearrangement?(a) D (b) B (c) C(d) E (e) F

25. Which of the following should be the FOURTH sentenceafter rearrangement?(a) E (b) F (c) B(d) A (e) D

DIRECTIONS (Qs. 26 to 30) : In each of the following items someparts have been jumbled up. You are required to rearrange these partswhich are labelled P, Q, R, S to produce the correct sentence. Choosethe proper sequence and mark in your Answer Sheet accordingly.26. Feeling flattered by praise of the fox to the piece of cheese

(P) / the crow began to crow (Q) / it held on its beak (R) /unmindful of what will / happen (S).The proper sequence should be(a) SPRQ (b) QSRP (c) RSPQ(d) QSPR (e) PQSR

27. There is that the woman is a kitchen-maid and (P) / thetraditional belief (Q) / an instrument of man’s pleasure (R) /and a child-bearing machine (S).The proper sequence should be(a) QPSR (b) RQPS (c) QSPR(d) RSPQ (e) PSRQ

28. The rapid endangerment and death of many minoritylanguages not only among logistics and anthropologists(P) / with issues of cultural identity (Q) / is a matter ofwidespread concern (R) / but among all concerned (S).The proper sequence should be(a) PSRQ (b) RQPS (c) RPSQ(d) QRPS (e) SPQR

29. Violence even before she is born (P) / and can happenthroughout a woman’s life (Q) / against women (R) / takesmany forms (S).The proper sequence should be(a) RPSQ (b) PQSR (c) RSQP(d) SQRP (e) QPRS

30. I saw two roads covered with the yellow (P) / directions in aforest (Q) / branching in two different (R) leaves of autumn (S).The proper sequence should be(a) PQRS (b) PQSR (c) PRSQ(d) PSRQ (e) QSRP

Page 151: yoursmahboob.wordpress.com SBI · yoursmahboob.wordpress.com iii P 101 Speed Tests for SBI Bank Clerk Exam 101 Speed Tests for SBI Bank Clerk Exam is revised and updated edition on

yoursmahboob.w

ordpress.com

DIRECTIONS (Qs. 1 to 36): In each of the following sentences,an idiomatic expression or a proverb is highlighted Select thealternative which best describes its use in the sentence.

1. I have a bone to pick with you in this matter.(a) Am in agreement (b) Am angry(c) Am indebted (d) Will join hands(e) None of these

2. The new CM stuck his neck out today and promised 10kgs.free wheat a month for all rural families.(a) took an oath (b) took a risk(c) extended help (d) caused embarrassment(e) None of these

3. Harassed by repeated acts of injustice he decided to puthis foot down.(a) not to yield (b) resign(c) to accept the proposal unconditionally(d) withdraw (e) none o fthese

4. The class could not keep a straight face on hearing thestrange pronunciation of the new teacher.(a) remain silent (b) remain serious(c) remain mute (d) remain disturbed(e) none of these

5. His speech went down well with the majority of the audience.(a) found acceptance with(b) was attentively listened to by(c) was appreciated by (d) was applauded by(e) none of these

6. Rohit has bitten off more than he chew.(a) Is trying to do much (b) Is very greedy(c) Is always hungry (d) Has little regard for others(e) none of these

7. The detective left no stone unturned to trace the culprit.(a) took no pains (b) did very irrelevant things(c) resorted to illegitimate practices(d) used all available means(e) none of these

8. He believes in the policy of making hay while the sun shines.(a) giving bribes to get his work done(b) seeking advice from one and all(c) helping those who help him(d) making the best use of a favourable situation(e) none of these

9. His friends advised him to be fair and square in his dealings.(a) Careful (b) Considerate(c) Polite (d) Upright(e) none of these

Max. Marks : 36 No. of Qs. 36 Time : 25 min. Date : ........./......../................

10. We should give a wide berth to bad characters.(a) give publicity to (b) publicly condemn(c) keep away from (d) not sympathise with(e) none of these

11. The authorities took him to task for his negligence.(a) gave him additional work (b) suspended his assignment(c) reprimanded him (d) forced him to resign(e) none of these

12. In spite of the immense pressure exerted by the militants, theGovernment has decided not to give in.(a) accede (b) yield(c) oblige (d) confirm(e) none of these

13. Their business is now on its last legs.(a) About to fructify (b) About to perish(c) About to produce results(d) About to take off (e) none of these

14. He went back on his promise to vote for me.(a) withdrew (b) forgot(c) reinforced (d) supported(e) none of these

15. The old beggar ran amuck & began to throw stones at the passerby.(a) became desperate (b) ran about wildly(c) become annoyed (d) felt disgusted(e) none of these

16. Turban is in vogue in some communities.(a) in fashion (b) out of use(c) vaguely used (d) never used(e) none of these

17. The old man was cut to the quick when his rich son refusedto recognise him.(a) surprised (b) hurt intensely(c) annoyed (d) irritated(e) none of these

18. I requested him to put in a word for me.(a) introduce (b) assist(c) support (d) recommend(e) none of these

19. The dacoit murdered the man in cold blood.(a) coldly (b) boldly(c) ruthlessly (d) deliberately(e) none of these

20. He is always picking holes in every project.(a) creating problems in (b) finding fault with(c) suggesting improvement in(d) asking irrelevant questions on(e) None of these

RESPONSE

GRID

1. a b c d e 2. a b c d e 3. a b c d e 4. a b c d e 5. a b c d e

6. a b c d e 7. a b c d e 8. a b c d e 9. a b c d e 10. a b c d e

11. a b c d e 12. a b c d e 13. a b c d e 14. a b c d e 15. a b c d e

16. a b c d e 17. a b c d e 18. a b c d e 19. a b c d e 20. a b c d e

6868Idioms andPhrases

Page 152: yoursmahboob.wordpress.com SBI · yoursmahboob.wordpress.com iii P 101 Speed Tests for SBI Bank Clerk Exam 101 Speed Tests for SBI Bank Clerk Exam is revised and updated edition on

yoursmahboob.w

ordpress.com

144 SPEED TEST 68

RESPONSE

GRID

21. a b c d e 22. a b c d e 23. a b c d e 24. a b c d e 25. a b c d e

26. a b c d e 27. a b c d e 28. a b c d e 29. a b c d e 30. a b c d e

31. a b c d e 32. a b c d e 33. a b c d e 34. a b c d e 35. a b c d e

36. a b c d e

21. Pt. Nehru was born with a silver spoon in his mouth.(a) born in a middle class family(b) born in a wealthy family(c) born in a royal family(d) born in a family of nationalists(e) none of these

22. The arrival of the mother-in-law in the family proved a rift inthe lute.(a) caused unnecessary worries(b) brought about disharmony(c) caused a pleasant atmosphere(d) brought about a disciplined atmosphere(e) none of these

23. Having sold off his factory, he is now a gentleman at large.(a) Has no serious occupation(b) Is living comfortably(c) Is respected by everybody(d) Is held in high esteem (e) none of these

24. Though he has lot of money, yet all his plans are built upon sand.(a) established on insecure foundations(b) based on inexperience (c) resting on cheap material(d) resting on immature ideas(e) none of these

25. There has been bad blood between the two communities evenbefore shouting.(a) Impure blood (b) Ill feeling(c) Bloody fights (d) Quarrels(e) none of these

26. The curious neighbors were disappointed as the youngcouple's quarrel was just a storm in a tea cup.(a) violent quarrel (b) fuss about a trifle(c) brittle situation (d) quarrel about tea cups(e) none of these

27. My father strained every nerve to enable me to get settled in life.(a) worked very hard (b) spent a huge amount(c) tried all tricks (d) bribed several persons(e) none of these

28. Madhuri might scream blue murder, but I feel Deepali shouldget the promotion since she is better qualified for the job.(a) Someone has been murdered with some blue liquid(b) Someone is being murdered and has become blue(c) Suffer from persecution complex(d) Make a great deal of noise and object vehemently(e) none of these

29. Why do you wish to tread on the toes?(a) To give offence to them(b) To follow them grudgingly(c) To treat them indifferently(d) To be kicked by them (e) None of these

30. The autographed bat from the famous cricketer SunilGavaskar is worth a jew's eye.(a) Not a worthy possession(b) unnecessary (c) A costly items(d) A possession of high value(e) None of these

31. The speaker gave a bird's eye view of the politicalconditions in the country.(a) a personal view (b) a general view(c) a biased view (d) a detailed presentation(e) None of these

32. The stunt that I recently attempted was a piece of cake(a) The stunt that I recently attempted was enjoyable to watch(b) The stunt that I recently attempted was very challenging(c) The stunt that I recently attempted was celebrated by all(d) The stunt that I recently attempted turned out to be a failure(e) The stunt that I recently attempted was a simple task

33. The boy broke the window and took to his heels.(a) The boy broke the window and fell on his heels(b) The boy broke the window and ran away(c) The boy broke the window with his heels(d) The boy ran into the window(e) The boy broke the window and robbed a pair of heels

34. I pledged myself to serve the king faithfully.(a) I made a mistake by promising to serve the king

faithfully(b) I made a fool of myself in order to serve the king(c) I boasted about serving the king faithfully(d) I was forcibly made to serve the king(e) I made a solemn and formal promise to serve the king

faithfully35. There is a crying need for improvements to our public

transport system.(a) There is an obvious need for improvements to our public

transport system(b) There is a well documented need for improvements to

our public transport system.(c) There is a minor need for improvements to our public

transport system.(d) There is a serious need for improvements to our public

transport system.(e) There is no urgency for improvements to our public

transport system.36. In an old bookshop I happened to light upon a volume that

belonged to my grandfather.(a) In an old bookshop I happened to discard a volume

that belonged to my grandfather.(b) In an old bookshop I happened to purchase a volume

that belonged to my grandfather.(c) In an old bookshop I happened to look for a volume

that belonged to my grandfather.(d) In an old bookshop I happened to discover by chance

a volume that belonged to my grandfather(e) In an old bookshop I happened to reveal a volume that

belonged to my grandfather.

Page 153: yoursmahboob.wordpress.com SBI · yoursmahboob.wordpress.com iii P 101 Speed Tests for SBI Bank Clerk Exam 101 Speed Tests for SBI Bank Clerk Exam is revised and updated edition on

yoursmahboob.w

ordpress.com

DIRECTIONS (Qs.1-34) : In the following passage there areblanks each of which has been numbered. These numbers areprinted below the passage and against each, five words aresuggested, one of which fits the blank appropriately. Find outthe approptiate words in each case.

Passage - 1When we (1) started thirty years ago in 1977, we did not

know anything about how to run a bank for the poor. We thereforelooked at how others ran their operations and (2) from theirmistakes. In Bangladesh, conventional banks and credit co-operatives always (3) lump sum repayments. This created (4)problems because repaying in a lump sum was a mental hurdlefor borrowers. They tended to delay repayment and get furtherinto debt in the (5). In the end they usually (6) totally on the loan,which was a loss to the bank. In structuring our own loans, Idecided to ask for a daily payment, Monitoring repayment was(7) and it filled people with (8) that they could repay their loans.1. (a) firstly (b) freshly

(c) foremost (d) initially(e) recently

2. (a) copied (b) observed(c) learned (d) understood(e) improving

3. (a) asked (b) insisted(c) demanded (d) settled(e) lend

4. (a) severe (b) no(c) additionally (d) variety(e) plenty

5. (a) time (b) process(c) return (d) event(e) action

6. (a) neglected (b) abandoned(c) defaulted (d) depended(e) disappointed

7. (a) benefit (b) easier(c) reckless (d) disorganised(e) secure

8. (a) sense (b) confidence(c) challenge (d) doubt(e) believe

Max. Marks : 34 No. of Qs. 34 Time : 20 min. Date : ........./......../................

Passage - 2Today, twenty-two years after the bank (9), it has over a

thousand branches all over the country and the staff (10) abouttwenty-three lakh borrowers. We decided to operate (11) fromconventional banks who would ask their clients to come to theiroffice. Many people in rural areas found this (12). Our bank istherefore based on the (13) that people should not come to thebank but that the bank should go to the people. Our loans are also(14)- we give them for activities for candlemaking to tyre repair. Wealso keep (15) checks on the borrower through weekly visits. Wedo this to make certain that the family of the borrower is (16) fromthe loan.9. (a) inaugurated (b) origin

(c) commence (d) existed(e) began

10. (a) handle (b) assemble(c) cope (d) interact(e) deal

11. (a) identically (b) differently(c) similar (d) reverse(e) opposite

12. (a) threatening (b) worried(c) upset (d) panicking(e) anxious

13. (a) advantage (b) principle(c) discipline (d) opportunity(e) chance

14. (a) diverse (b) worth(c) vary (d) disburse(e) contrast

15. (a) daily (b) consistently(c) regular (d) often(e) frequently

16. (a) progress (b) benefiting(c) serving (d) welfare(e) obliged

Passage - 3The Government seems to be in right earnest to ensure more

(17) in governance. The Prime Minister’s announcement that hisGovernment is (18) drafting legislation to establish the citizen’sright to information is indeed welcome. Though the talk on theright to information is not new, we may (19) the bill to be brought

RESPONSE

GRID

1. a b c d e 2. a b c d e 3. a b c d e 4. a b c d e 5. a b c d e

6. a b c d e 7. a b c d e 8. a b c d e 9. a b c d e 10. a b c d e

11. a b c d e 12. a b c d e 13. a b c d e 14. a b c d e 15. a b c d e

16. a b c d e

Cloze Test - I 6969

Page 154: yoursmahboob.wordpress.com SBI · yoursmahboob.wordpress.com iii P 101 Speed Tests for SBI Bank Clerk Exam 101 Speed Tests for SBI Bank Clerk Exam is revised and updated edition on

yoursmahboob.w

ordpress.com

146 SPEED TEST 69

RESPONSE

GRID

17. a b c d e 18. a b c d e 19. a b c d e 20. a b c d e 21. a b c d e

22. a b c d e 23. a b c d e 24. a b c d e 25. a b c d e 26. a b c d e

27. a b c d e 28. a b c d e 29. a b c d e 30. a b c d e 31. a b c d e

32. a b c d e 33. a b c d e 34. a b c d e

early this time. The previous Government had set up a high-levelcommittee to prepare a draft bill. But nothing has been heard aboutthe matter since, (20) the committee did quite some work. Theissue, however, has come to such a pass that a solution cannot be(21) further. Sunlight is the best disinfectant, a foreign judge oncesaid, while (22) the unwarranted secrecy in an administrativesystem. When those in authority know that people have the rightto ask questions and the government is under the (23) to providethem with answers, (24) of authority, or of public finances, forpersonal or party ends is less likely to happen.17. (a) strictness (b) rudeness (c) leniency

(d) economy (e) transparency18. (a) personally (b) busy (c) not

(d) reluctantly (e) absolutely19. (a) expect (b) wait (c) try

(d) frustrate (e) appeal20. (a) even (b) as (c) because

(d) until (e) though21. (a) found (b) expected (c) delayed

(d) looked (e) longed22. (a) nurturing (b) criticising (c) demanding

(d) appreciating (e) upholding23. (a) pretention (b) affect (c) substance

(d) obligation (e) property24. (a) misuse (b) governance (c) dishonour

(d) curbing (e) breachPassage - 4

Today the economies of African countries are growing at5 per cent every year.

Rich countries usually help poorer ones (25) Africancountries through donations and aid. Their (26) are not alwayssuccessful as loans are (27) not used for the projects for whichthey are (28). China, however, has found a different (29) to helpAfrica - by trading more with the (30). In 2009 China’s trade withAfrican countries was $ 90 billion - (31) than the U.S., which was

$ 86 billion. (32) countries have now begun to notice the (33)available in Africa. China’s attitude has (34) the way the worlddeals with poor countries. “Trade not aid” is the new mantra ofAfrican nations.25. (a) belonging (b) similarly

(c) compared (d) with(e) like

26. (a) efforts (b) practices(c) challenges (d) achievements(e) attempt

27. (a) given (b) approved(c) regular (d) often(e) being

28. (a) grant (b) sanctioned(c) took (d) hired(e) apply

29. (a) goal (b) fund(c) way (d) skill(e) dream

30. (a) countries (b) others(c) abroad (d) neighbours(e) poor

31. (a) further (b) extra(c) more (d) less(e) high

32. (a) Recently (b) Any(c) Friendly (d) Many(e) While

33. (a) differences (b) supply(c) quantity (d) people(e) opportunities

34. (a) substitute (b) changed(c) exchanged (d) transform(e) convert

Page 155: yoursmahboob.wordpress.com SBI · yoursmahboob.wordpress.com iii P 101 Speed Tests for SBI Bank Clerk Exam 101 Speed Tests for SBI Bank Clerk Exam is revised and updated edition on

yoursmahboob.w

ordpress.com

DIRECTIONS : In the following passage there are blanks, eachof which has been numbered. These numbers are printed belowthe passage, against each, five words are suggested, one of whichfits the blank appropriately. Find out the appropriate word ineach case.

Passage - 1

Emperor Akbar was fond of (1) tricky questions to Birbal. Oneday he asked Birbal what he would (2) if he were given a choicebetween justice and a gold coin. “The gold coin,” said Birbal.Akbar was (3) aback. He had known Birbal for many years andhe knew that Birbal was a just person. Then how could he choosethe gold coin. “You would prefer a gold coin to justice ?” Heasked, incredulously. “Yes,” said Birbal. The other courtiers wereamazed by Birbal’s (4) of idiocy. For years they had been tryingto discredit Birbal in the emperor’s eyes but without success andnow the man had gone and (5) it himself ! They could notbelieve their good fortune. “I would have been dismayed if eventhe lowliest of my servants had said this,” continued the em-peror. “But coming from you it’s shocking - and sad. I did not (6)you were so debased ! I never expected this from you. Howcould you be so shallow ?”One (7) for what one does not have, Your Majesty !” said Birbal,quietly. “You have (8) to it that in our country justice is availableto everybody. So as justice is already available to me and as I’malways (9) of money I said I would choose the gold coin.” Theemperor laughed. He thought to himself, ‘I should have knownthat Birbal would come up with a witty reply as always.’ He wasso pleased with Birbal’s reply that he gave him (10) one but athousand gold coins.1. (a) showing (b) asking

(c) naming (d) finding(e) telling

2. (a) look (b) said(c) think (d) choose(e) find

3. (a) pushed (b) fallen(c) pulled (d) sent(e) taken

4. (a) idea (b) display(c) reply (d) place(e) showing

5. (a) speak (b) thought(c) done (d) create(e) told

Max. Marks : 35 No. of Qs. 35 Time : 20 min. Date : ........./......../................

6. (a) felt (b) said(c) know (d) accept(e) saw

7. (a) asks (b) chooses(c) look (d) find(e) wish

8. (a) sure (b) put(c) shown (d) seen(e) made

9. (a) no (b) rich(c) short (d) poor(e) plenty

10. (a) but (b) not(c) and (d) so(e) only

Passage - 2Once upon a time, two friends were (11) through the desert. Duringsome point of the (12) they had an argument, and one friend slappedthe other one in the face. The one who got slapped was (13), butwithout saying anything, he wrote in the sand, “Today my bestfriend slapped me in the face.” They kept on walking (14) theyfound an oasis, where they (15) to take a bath. The one, who hadbeen slapped, got (16) in the quicksand and started drowning, butthe friend saved him. After the friend (17) from the near drowninghe wrote on a stone, “Today my best friend saved my life.” Thefriend who had slapped and saved his best friend asked him, “AfterI hurt you, you wrote in the sand and (18) you write on a stone,why?” The other friend (19), “When someone hurt us, we shouldwrite it down in sand where wind of forgiveness can erase it away.But, when someone does something good for us, we must (20) it instone where no wind can ever erase it.”11. (a) crawling (b) speaking

(c) swimming (d) walking(e) dancing

12. (a) journey (b) sand(c) running (d) border(e) hunt

13. (a) dead (b) captured(c) presentable (d) missing(e) hurt

14. (a) as (b) until(c) from (d) with(e) through

15. (a) decided (b) fell(c) made (d) want(e) left

RESPONSE

GRID

1. a b c d e 2. a b c d e 3. a b c d e 4. a b c d e 5. a b c d e

6. a b c d e 7. a b c d e 8. a b c d e 9. a b c d e 10. a b c d e

11. a b c d e 12. a b c d e 13. a b c d e 14. a b c d e 15. a b c d e

70Cloze Test - II 70

Page 156: yoursmahboob.wordpress.com SBI · yoursmahboob.wordpress.com iii P 101 Speed Tests for SBI Bank Clerk Exam 101 Speed Tests for SBI Bank Clerk Exam is revised and updated edition on

yoursmahboob.w

ordpress.com

148 SPEED TEST 70

RESPONSE

GRID

16. a b c d e 17. a b c d e 18. a b c d e 19. a b c d e 20. a b c d e

21. a b c d e 22. a b c d e 23. a b c d e 24. a b c d e 25. a b c d e

26. a b c d e 27. a b c d e 28. a b c d e 29. a b c d e 30. a b c d e

31. a b c d e 32. a b c d e 33. a b c d e 34. a b c d e 35. a b c d e

16. (a) home (b) stuck(c) blended (d) mixed(e) sitting

17. (a) separated (b) leaked(c) died (d) recovered(e) saved

18. (a) so (b) how(c) when (d) tomorrow(e) now

19. (a) called (b) tell(c) replied (d) questioned(e) asked

20. (a) talk (b) push(c) engrave (d) add(e) bury

Passage - 3The Right of Children to Free and Compulsory Education (RTE)Act, 2009, which came (21) effect in April this year, is meant totransform the education sector and take India closer to the goal ofuniversal schooling. But with admissions to the new academicsession just (22) the corner, it is fast becoming clear that (23) wellintentioned ideas into (24) will take some doing. For a start, theguidelines for admissions under the RTE prohibit schools fromconducting any sort of student profiling. The stress on a randomyet justifiable admission process means that schools will have toresort to something as quirky as a lottery system. However, leavingadmission to a good school to pure (25) will only incentivisemanipulations, defeating the very essence of RTE.

The main problem facing the education sector is that of aresource crunch. The provisions for ensuring universal access toeducation are all very well, (26) we have the infrastructure in placefirst. Brick and mortar schools need to precede open admissionand not the (27) way around. In that sense, legislators’ assessmentof ground realities is (28) target when they endorse the closure oftens of thousands of low-cost private schools for not meeting theminimum standards of land plot, building specifications andplayground area as laid out in the RTE Act. Instead of bearingdown (29) on private schools for failing to conform to abstractbureaucratic criteria, efforts to bring about universal educationshould focus on upgrading and expanding the existinggovernment school infrastructure to accommodate all. Only thencan we ensure the much needed supply-demand (30) in theeducation sector.21. (a) with (b) for (c) on

(d) into (e) in22. (a) around (b) near (c) into

(d) about (e) reaching23. (a) forming (b) translating (c) having

(d) taking (e) framing

24. (a) affect (b) ideas (c) practice(d) concept (e) procedure

25. (a) benefit (b) merit (c) chance(d) basis (e) method

26. (a) unless (b) until (c) executed(d) provided (e) exercised

27. (a) other (b) any (c) two(d) differ (e) after

28. (a) on (b) of (c) often(d) taken (e) off

29. (a) soft (b) more (c) less(d) only (e) hard

30. (a) need (b) equilibrium(c) expectation (d) attempt(e) aspects

Passage - 4The Bhagavad Gita is a poem of 700 verses which is a part of theMahabharata. It is the only philosophical song existing in alllanguages. Its popularity and influence have never waned. It (31)light and guidance to the troubled mind in times of crisis. It is inthe (32) of a dialogue between Arjuna and Krishna on thebattlefield. Arjuna’s mind is troubled at the thought of the killingsof his friends and relatives. He cannot conceive of any gain. Arjunais the (33) of the tortured spirit of man tom by conflictingobligations and molalities.

The dialogue proceeds and takes upto the higher level ofindividual duty and social behaviour, application of ethics topractical life and social outlook that should govern all. An attemptis (34) to reconcile the three paths of human advancement - thepath of knowledge, the path of action and the path of faith. Butmore (35) is laid on faith. There is a call of action to meet theobligations of life, keeping in view the spiritual background andthe large purpose of the universe.31. (a) provides (b) shines

(c) enforces (d) secures(e) seeks

32. (a) programme (b) constitution(c) part (d) formation(e) form

33. (a) conceived (b) dream(c) source (d) figures(e) symbol

34. (a) generated (b) made(c) established (d) coined(e) given

35. (a) important (b) Significant(c) declaration (d) emphasis(e) blessings

Page 157: yoursmahboob.wordpress.com SBI · yoursmahboob.wordpress.com iii P 101 Speed Tests for SBI Bank Clerk Exam 101 Speed Tests for SBI Bank Clerk Exam is revised and updated edition on

yoursmahboob.w

ordpress.com

DIRECTIONS : Read the following passage carefully and answerthe questions given below it. Certain words/group of words havebeen printed in bold to help you locate them while answeringsome of the questions.

Unemployment is the problem of every modern nation. Evenindustrialised nations are not able to ensure a job for everyone.Following the conventional strategy of creating employment,governments of many developing countries try to attractemployers (business houses/industrialists) by offering taxrebates and many other facilities so that they locate theirupcoming plants on their soil, and thereby create industrialemployment. But there is a limit to what industry can bring.Also, industrial plants often create toxic waste which results inair and water pollution and environmental problems which canoutweigh whatever employment benefit industrial employmentbrings. In addition, they don't bring as substantial relief to thedwindling economy of the host country as they seem to promise,as the profits of such foreign investments are carried back to theparent company and foreign shareholders abroad.

Self-employment has none of these drawbacks. The problemis that self-employment is not as obviously glamorous as a shinynew factory. But profits from self-employment remain in thecountry where they are produced. It is too small to createenvironmental hazards. It also puts the poor person in charge ofhis or her own working hours and conditions. The hours areflexible and can adapt to fit any family situation. It allows peopleto choose between running a business full-time, or part-time whenthey face a crisis, or to put their business on hold and work full-time for a salary. Self-employment is tailor-made for anyone whois street-smart and has many acquired and inherited traditionalskills, rather than learning acquired from books and technicalschools. This means the illiterate and the poor can exploit theirstrengths, rather than be held back by their weaknesses. It allowsa person to turn their hobbies into gainful employment. It allowsindividuals who cannot work well in a rigid hierarchy to run theirown show.

Financing the poor to start their own little ventures elevatestheir sense of pride and self-respect. It offers a way out of welfaredependency, not just to become wage slaves, but to open a store

Max. Marks : 40 No. of Qs. 40 Time : 20 min. Date : ........./......../................

or start a manufacturing business. It can help those who havefound a job and are still nonetheless poor. It gives the victims ofprejudice who would not be hired because of their colour or nationalorigin a chance to earn a living. The average cost of creating self-employment is ten, twenty or hundred times lesser than creatingindustry-based employment. It helps isolated poor people gain self-confidence step by step.

Obviously self-employment has limits, but in many cases it isthe only solution to help those whom economies refuse to hire andtaxpayers do not want to carry on their shoulders. The policy neededfor the eradication of poverty must be much wider and deeper thanthe policy for the provision of mere employment. The real eradicationof poverty begins when people are able to control their own fate.Poor people are like bonsai trees. When you plant the best seed ofthe tallest tree in a flower-pot, you get a replica of the tallest tree,only inches tall. There is nothing wrong with the seed you planted;only the soil-base that is too inadequate. Poor people are bonsaipeople. There is nothing wrong in their seeds. Simply, society nevergave them the base to grow on. All it takes to get the poor peopleout of poverty is for us to create an enabling environment for them.Once the poor can unleash their energy and creativity, poverty willdisappear very quickly.

DIRECTIONS (Q. 1-3) : Choose the word/group of words which isMOST OPPOSITE in MEANING to the word/group of words printedin bold as used in the passage.

1. RIGID(a) Unstructured (b) Flexible(c) Soft (d) Gentle(e) Calm

2. STEP BY STEP(a) All at once (b) In quick succession(c) In slow motion (d) In a nutshell(e) Once and for all

3. OFFERING(a) Stealing (b) Permitting(c) Refusing (d) Protesting(e) Questioning

RESPONSE

GRID1. a b c d e 2. a b c d e 3. a b c d e

7171Section Test :English Language

Page 158: yoursmahboob.wordpress.com SBI · yoursmahboob.wordpress.com iii P 101 Speed Tests for SBI Bank Clerk Exam 101 Speed Tests for SBI Bank Clerk Exam is revised and updated edition on

yoursmahboob.w

ordpress.com

150 SPEED TEST 71

DIRECTIONS (Q. 4-15): Choose the word/group of words whichis MOST SIMILAR in MEANING to the word printed in bold asused in the passage.

4. REMAIN(a) Left-over (b) Stay(c) Stagnate (d) Continue(e) Linger

5. HIRED(a) Allowed (b) Rented(c) Authorised (d) Employed(e) Delegated

6. LIMIT(a) Maximum (b) Finish(c) Cap (d) Decrease(e) Barrier

7. Which of the following is a reason foreign investments donot strengthen the economies of host nations?(a) The parent company pays all the profit as tax to its

nation.(b) The profit of such enterprises does not remain in the

host nation; rather it goes back to the share holdersand owners of the parent company.

(c) The employees of the parent company demand extrapay from profits that the companies earn from factoriesin another nation.

(d) The profit earned by such enterprises is too less toprovide for anything beyond the salaries of employees.

(e) None of these8. What is the tone of the passage?

(a) Offensive (b) Satirical(c) Analytical (d) Humorous(e) Speculative

9. Which of the following is an advantage(s) that self-employment has over industry-based employment?(A) The work timings are highly flexible.(B) Starting one's own venture is an easy task and needs

no investment as financiers are readily available.(C) Self-employment makes one a master of other people

and thus satisfies their need to control others.(a) Only(C) (b) Only (A)(c) Only (B) (d) Only (A) and (B)(e) All (A), (B) and (C)

10. Which of the following may be inferred about self-employment?(A) Self-employment slowly but steadily strengthens the

economy of the country.(B) Self-employment checks unemployment.(C) As a strategy of providing employment, self-

employment is still unexplored.(a) Only (B) (b) Only (B) and (C)(c) Only (A) (d) Only (A) and (B)(e) Only (A) and (C)

11. What does the author indicate by the example of a bonsaitree?(A) When provided the right kind of financial help, poor

people in can flourish.(B) The poor people are as capable as the well-to-do class.(C) Conventional (industrial) employment can help the

poor people create their own base.(a) Only (A) (b) Only (B)(c) Only (A) and (B) (d) Only (A) and (C)(e) Only (B) and (C)

12. The author claims that self-employment is "tailor-made" forpeople with certain qualities. Which of the following are thequalities of such people?(A) They have an unconventional approach to all things.(B) They are street-smart.(C) They possess many acquired and traditional skills.(a) Only (A) (b) Only (C)(c) Only (B) and (C) (d) Only (A) and (C)(e) Only (B)

13. Which of the following outweighs the employment benefitsthat foreign industrialists bring?(a) Huge industries set up by them cause environmental

pollution.(b) They employ more number of people belonging to their

native nations than the host nations' unemployed.(c) They evade many taxes that could be a source of revenue

for the host nation.(d) They manufacture products that have no market in the

host nation.(e) They practise discrimination on grounds of gender

when providing employment to host nations' residents.

RESPONSE

GRID

4. a b c d e 5. a b c d e 6. a b c d e 7. a b c d e 8. a b c d e

9. a b c d e 10. a b c d e 11. a b c d e 12. a b c d e 13. a b c d e

Page 159: yoursmahboob.wordpress.com SBI · yoursmahboob.wordpress.com iii P 101 Speed Tests for SBI Bank Clerk Exam 101 Speed Tests for SBI Bank Clerk Exam is revised and updated edition on

yoursmahboob.w

ordpress.com

151SPEED TEST 71

14. Which of the following may be an appropriate title for thepassage?(a) Addressing conventional employment in developed

nations(b) Varied strategies and approaches to eradicating

poverty(c) Limitations of industrial employment(d) How is poverty linked to conventional (industrial)

employment?(e) Role of self-employment in battling unemployment and

eradication of poverty15. Which of the following is TRUE as per the passage?

(a) Self-employment is beneficial only for developingeconomies.

(b) Self-employment is not as glamorous as conventional(industrial) employment.

(c) Finance for poor is readily available in the developednations of the world.

(d) Small-scale industries produce as much toxic wasteas big industries.

(e) None is true

DIRECTIONS (Q. 16-20): Pick out the most effective word fromthe given words to fill in the blank to make the sentencemeaningfully complete.16. The government is planning to set ___________ family

welfare centres for slums in cities.(a) another (b) with(c) for (d) in(e) up

17. Economic independence and education have ___________women more assertive.(a) prepared (b) made(c) marked (d) resulted(e) adjusted

18. In the modern world, the ___________ of change andscientific innovation is unusually rapid.(a) supplies (b) context(c) pace (d) fantasy(e) requirement

19. The unprecedented economic growth of China has___________ worldwide attention.(a) perceived (b) proposed(c) neither (d) astonished(e) attracted

20. Each business activity ___________ employment to peoplewho would otherwise be unemployed.(a) taking (b) finds(c) creates (d) provides(e) given

DIRECTIONS (Q. 21-30) : In the following passage, there areblanks, each of which has been numbered. These numbers areprinted below the passage and against each five words aresuggested, one of which fits the blank appropriately. Find out theappropriate word in each case.

The latest technology (21) put to use or about to arrive inmarket must be (22) to all entrepreneurs. The reason is that it mayhave an (23) effect on business. Valve radios gave way to transistorradios and with micro chips, technology is giving way to digitalequipment. Business has (24) the same but the technology haskept changing. A notable feature is that the size of the receiversdecreased (25) so did the use of its material and consequently itsprice. The traditional flour mills are losing business (26) customersnow buy flour (27) from the market. As a result of this, the businessis (28). Following the same lines as technology, the social trendsalso go on changing and influence the market. The Indian sareesare being taken (29) by readymade stitched clothes. Everyentrepreneur must note such changes in the environment and alsothe technology and plan in (30)with these to ensure the success ofhis endeavour.21. (a) to (b) needed

(c) decided (d) besides(e) being

22. (a) hoped (b) welcome(c) released (d) known(e) aware

23. (a) approximate (b) huge(c) uniform (d) excellence(e) enormous

24. (a) maintained (b) remained(c) often (d) mentioned(e) become

25. (a) mainly (b) and(c) how (d) also(e) some

26. (a) reason (b) due(c) young (d) as(e) old

RESPONSE

GRID

14. a b c d e 15. a b c d e 16. a b c d e 17. a b c d e 18. a b c d e

19. a b c d e 20. a b c d e 21. a b c d e 22. a b c d e 23. a b c d e

24. a b c d e 25. a b c d e 26. a b c d e

Page 160: yoursmahboob.wordpress.com SBI · yoursmahboob.wordpress.com iii P 101 Speed Tests for SBI Bank Clerk Exam 101 Speed Tests for SBI Bank Clerk Exam is revised and updated edition on

yoursmahboob.w

ordpress.com

152 SPEED TEST 7127. (a) knowingly (b) ease

(c) cheap (d) directly(e) forcefully

28. (a) shrinking (b) blooming(c) returned (d) same(e) small

29. (a) against (b) to(c) over (d) up(e) for

30. (a) lines (b) relativity(c) accordance (d) proper(e) toning

DIRECTIONS (Q. 31-35): Which of the phrases (a), (b), (c) and(d) given below should replace the phrase given in bold in thefollowing sentences to make the sentence grammatically correct?If the sentence is correct as it is and there is no correction requiredmark (e) ie ‘No correction required’ as the answer.31. During the recession many companies will be forced to

lay off workers.(a) have the force to (b) be forced into(c) forcibly have (d) forcefully(e) No correction required

32. He wanted nothing else expecting to sleep after a Stressfulday at work.(a) nothing better than (b) anything else unless(c) nothing but having (d) nothing else than(e) No correction required

33. Ramesh took charge of the project, within a few days ofhaving appointed ?(a) having an appointment(b) being appointed(c) after being appointed (d) appointing(e) No correction required

34. It is difficult to work with him because he is one of thosepersons who think he is always right.

(a) think they are always

(b) always thinks he is (c) is always thinking they are(d) always think his (e) No correction required

35. Foreign businesses in developing countries have usuallyproblems with lack of infrastructure and rigid laws.

(a) usual problems as (b) usually problems on

(c) as usual problems like (d) the usual problems of

(e) No correction required

DIRECTIONS (Qs. 36-40) : Read each sentence to find out whetherthere is any grammatical mistake/error in it. The error if any, will bein one part of the sentence. Mark the number of the part with erroras your answer. If there is no error, mark (e).

36. The cost of constructing (a) / houses are increased (b) /because of the high (c) / price of cement. (d) / No error (e).

37. According to the Twelfth (a) / Five Year Plan, India should(b) / invest one trillion dollars (c) / in infrastructure projects.(d) / No error (e)

38. To increase the selling (a) / of products in rural areas (b) / thecompany will hire (c) / over five hundred trainees. (d) / Noerror (e)

39. We have spent (a) / most of the profits (b) / that we earn (c)/ last year on purchasing new computers, (d) / No error (e).

40. The Government has (a) / promised to revise (b) / thepension scheme for bank (c) / staff since next year. (d) / Noerror (e)

RESPONSE

GRID

27. a b c d e 28. a b c d e 29. a b c d e 30. a b c d e 31. a b c d e

32. a b c d e 33. a b c d e 34. a b c d e 35. a b c d e 36. a b c d e

37. a b c d e 38. a b c d e 39. a b c d e 40. a b c d e

Page 161: yoursmahboob.wordpress.com SBI · yoursmahboob.wordpress.com iii P 101 Speed Tests for SBI Bank Clerk Exam 101 Speed Tests for SBI Bank Clerk Exam is revised and updated edition on

yoursmahboob.w

ordpress.com

1. Most of the commonly used personal computers/laptopsdo not have a command key known as ________.

(a) Turnover (b) Shift (c) Alter(d) Delete (e) Insert

2. Which of the following is NOT a hardware of a computer?

(a) Monitor (b) Key Board

(c) Windows (d) Central Processing Unit

(e) Mouse

3. Most of the commonly available personal computers/laptopshave a keyboard popularly known as ________.

(a) QWERTY (b) QOLTY (c) ALTER(d) UCLIF (e) None of these

4. Computers send and receive data in the form of ________signals.

(a) Analog (b) Digital

(c) Modulated (d) Demodulated

(e) All of these

5. The smallest unit of information a computer can understandand process is known as a_______.

(a) digit (b) byte (c) megabyte(d) kilobyte (e) bit

6. A byte can represent any number between 0 and_____

(a) 2 (b) 255 (c) 256(d) 1024 (e) 1025

7. External devices such as printers, keyboards and modemsare known as

(a) add-on devices.

(b) peripherals.

(c) extra hardware devices.

(d) PC expansion slot add-ons.

(e) special-buys

8. The most common pointing input device is the

(a) trackball (b) touchpad

(c) touchscreen (d) mouse

(e) scanner

9. How many megabytes make a gigabyte?

(a) 1024 (b) 128

(c) 256 (d) 512

(e) 64

10. Which of the following categories would include a keyboard?

(a) Printing Device (b) Output Device

(c) Pointing Device (d) Storage Device

(e) Input Device

11. Which of the following is hardware and not software ?

(a) Excel (b) Printer driver

(c) Operating System (d) Power Point

(e) CPU

12. A _________ is an electronic device that process data,converting it into information.

(a) computer (b) processor

(c) case (d) stylus

(e) None of these

13. A computer works on a _________ number system.

(a) binary (b) octal

(c) decimal (d) hexadecimal

(e) None of these

14. The physical components of a computer system is _______(a) Software (b) Hardware

(c) ALU (d) Control Unit

(e) None of these

15. Files deleted from the hard disk are sent to the ______.

(a) recycle bin (b) floppy disk

(c) clipboard (d) motherboard

(e) None of these

16. Which of the following is NOT a famous operating system?

(a) Windows Vista (b) Mac OS X (c) Linux(d) Sun OS (e) Virtual Box

RESPONSE

GRID

1. a b c d e 2. a b c d e 3. a b c d e 4. a b c d e 5. a b c d e

6. a b c d e 7. a b c d e 8. a b c d e 9. a b c d e 10. a b c d e

11. a b c d e 12. a b c d e 13. a b c d e 14. a b c d e 15. a b c d e

16. a b c d e

Max. Marks : 30 No. of Qs. 30 Time : 20 min. Date : ........./......../................

7272Computer Fundamentals/Binary System/

Operating System

Page 162: yoursmahboob.wordpress.com SBI · yoursmahboob.wordpress.com iii P 101 Speed Tests for SBI Bank Clerk Exam 101 Speed Tests for SBI Bank Clerk Exam is revised and updated edition on

yoursmahboob.w

ordpress.com

154 SPEED TEST 72

RESPONSE

GRID

17. a b c d e 18. a b c d e 19. a b c d e 20. a b c d e 21. a b c d e

22. a b c d e 23. a b c d e 24. a b c d e 25. a b c d e 26. a b c d e

27. a b c d e 28. a b c d e 29. a b c d e 30. a b c d e

17. Window 95, Windows 98 and Windows NT are known as

(a) processors (b) domain names (c) modems

(d) operating systems (e) None of these

18. Two different files can have the same name if

(a) they are in different folders

(b) they are on different drives

(c) Never

(d) the names are capitalised differently

(e) None of these

19. Every computer has a(n) ____________; many also have__________.

(a) operating system; a client system

(b) operating system; instruction sets

(c) application programs; an operating system

(d) application programs; a client system

(e) operating system; application programs

20. Applications are often referred to as

(a) data file (b) executable files

(c) system software (d) the operating system

(e) None of these

21. The process of transferring files from a computer on theInternet to your computer is called __________.

(a) downloading (b) uploading (c) FTP

(d) JPEG (e) downsizing

22. The operating system called UNIX is typically used for

(a) Desktop computers

(b) Laptop computers

(c) Super computers

(d) Web servers

(e) All of these

23. When data changes in multiple lists and all lists are notupdated, this causes

(a) data redundancy (b) information overload

(c) duplicate data (d) data inconsistency

(e) data repetition

24. Which process checks to ensure the components of thecomputer are operating and connected properly?

(a) Booting (b) Processing (c) Saving

(d) Editing (e) Starting

25. Executing more than one program concurrently by one useron one computer is known as

(a) multi-programming (b) multi-processing

(c) time sharing (d) multi-tasking

(e) multi-action

26. All computers must have

(a) a word processing software

(b) an operating system

(c) an attached printer

(d) a virus checking program

(e) None of these

27. When you instal a new program on your computer,it is typi-cally added to the _________ menu.

(a) All Programs (b) Select Programs

(c) Start Programs (d) Desktop Programs

(e) None of these

28. The operating system, that is self-contained in a device andresident in the ROM .is

(a) Batch Operating System

(b) Real-time Operating System

(c) Embedded Operating System

(d) Mutli-Processor Operating System

(e) None of these

29. If you change Windows 98 to Windows XP, you are actuallyperforming

(a) upstart (b) upgrade

(c) update (d) patch

(e) None of these

30. What happens when we try to delete the files on the floppy?

(a) The files get moved to the Recycle Bin

(b) Files on a floppy cannot be deleted

(c) The files get deleted and can be restored again fromRecycle Bin .

(d) The files get deleted and cannot be restored again

(e) The file gets copied on the Hard disk

Page 163: yoursmahboob.wordpress.com SBI · yoursmahboob.wordpress.com iii P 101 Speed Tests for SBI Bank Clerk Exam 101 Speed Tests for SBI Bank Clerk Exam is revised and updated edition on

yoursmahboob.w

ordpress.com

1. The name of a Microsoft Office Word document is displayedin both the ___________ and the taskbar.(a) menu bar (b) taskbar(c) Formatting toolbar (d) Standard toolbar(e) title bar

2. Microsoft Office is an example of a _________.(a) closed-source software(b) open-source software(c) horizontal-market software(d) vertical-market software(e) compiler

3. You cannot link Excel worksheet data to a Word document___________.(a) with the right drag method(b) with the hyperlink(c) with the copy and paste special commands(d) with the copy and paste buttons on the standard

commands(e) All of these

4. In Excel, Charts are created using which option?(a) Chart Wizard (b) Pivot Table(c) Pie Chart (d) Bar Chart(e) None of these

5. Each cell in a Microsoft Office Excel document is referred toby its cell address, which is the(a) cell’s column label(b) cell’s column label and worksheet tab name(c) cell’s row label(d) cell’s row and column labels(e) cell’s contents

6. This Excel feature includes functions to calculate an Average,Minimum, Maximum and Count.(a) Format (b) Number (c) AutoSum(d) Calculate (e) MIN

7. This is a set of values that you want to chart in Excel.(a) Object (b) Numbers (c) Data Mart(d) Formulas (e) Data series

8. The .xls extension is used for ________ files.(a) Windows (b) Access (c) PowerPoint(d) Word (e) Excel

9. In word, when you indent a paragraph, you(a) push the text in with respect to the margin(b) change the margins on the page(c) move the text up by one line(d) move the text down by one line(e) None of these

10. Using Print Preview is useful when you want to-(a) Colour the document(b) Save the document(c) Delete the document(d) Copy the document(e) View how trip document will appear when printed

11. In word, you can change Page Margins by(a) Dragging the scroll box on the scroll bars(b) Deleting the margin boundaries on the Ruler(c) Dragging the margin boundaries on the Ruler(d) Clicking the right mouse button on the Ruler(e) None of these

12. In Excel, this is a prerecorded formula that provides a shortcutfor complex calculations(a) Value (b) Data Series(c) Function (d) Field(e) None of these

13. This is not a function category in Excel(a) Logical (b) Data Series(c) Financial (d) Text(e) None of these

14. Text in a column is generally aligned _________(a) justified (b) right(c) center (d) left(e) None of the above

15. In Excel _________ contains one or more worksheets.(a) Template (b) Workbook(c) Active cell (d) Label(e) None of these

RESPONSE

GRID

1. a b c d e 2. a b c d e 3. a b c d e 4. a b c d e 5. a b c d e6. a b c d e 7. a b c d e 8. a b c d e 9. a b c d e 10. a b c d e11. a b c d e 12. a b c d e 13. a b c d e 14. a b c d e 15. a b c d e

Max. Marks : 30 No. of Qs. 30 Time : 20 min. Date : ........./......../................

7373MS Office/ Commandsand Shortcut Keys

Page 164: yoursmahboob.wordpress.com SBI · yoursmahboob.wordpress.com iii P 101 Speed Tests for SBI Bank Clerk Exam 101 Speed Tests for SBI Bank Clerk Exam is revised and updated edition on

yoursmahboob.w

ordpress.com

156 SPEED TEST 73

RESPONSE

GRID

16. a b c d e 17. a b c d e 18. a b c d e 19. a b c d e 20. a b c d e

21. a b c d e 22. a b c d e 23. a b c d e 24. a b c d e 25. a b c d e

26. a b c d e 27. a b c d e 28. a b c d e 29. a b c d e 30. a b c d e

16. You click at B to make the text _________.(a) Italics(b) Underlined(c) Italics and Underlined(d) Bold(e) None of these

17. The shortcut key Ctrl + F in Word is used for(a) To view document in full view(b) To open the Formula dialog box(c) To save the file(d) To open the Find and Replace dialog box(e) None of these

18. To restart the computer ________ key is used.(a) Del + Ctrl (b) Backspace + Ctrl(c) Ctrl + Alt + Del (d) Reset(e) None of these

19. ctrl, shift and alt are called ________ keys.(a) adjustment (b) function(c) modifier (d) alphanumeric(e) None of these

20. To select or unselect one word to the right(a) CTRL + SHIFT (b) CTRL + SHIFT + UP Arrow(c) CTRL + SHIFT + Down Arrow(d) CTRL + SHIFT + Right Arrow(e) None of these

21. To maximize or restore a selected window(a) CTRL + F7 (b) CTRL + F10(c) CTRL + F8 (d) CTRL + F9(e) None of these

22. To Copy a picture of the selected window to the clipboard(a) ALT + TAB (b) Ctrl + TAB(c) ALT + Print Screen(d) Both (a) and (b)(e) None of these

23. ________ is the key to close a selected drop - down list;cancel a command and close a dialog box.(a) TAB (b) SHIFT(c) ESC (d) F10(e) None of these

24. ________ is the function key to display save-as box.(a) F5 (b) F6(c) F9 (d) F12(e) None of these

25. ________ is the Keyboard shortcut key to insert auto sum(a) ALT (b) ALT=(c) ALT+ (d) ALT–(e) ALT+CTRL

26. To restart the computer the following combination of keysis used(a) Del + Ctrl (b) Backspace + Ctrl(c) Esc + Ctrl (d) Insert + Esc(e) Ctrl + Alt + Del

27. To make a copy of the current document to disk(a) Use the “save” command(b) This cannot be done(c) Use the “duplicate” command(d) Copy the document(e) Use the “save as” command

28. What type of keys are ‘ctrl’ and ‘shift’?(a) adjustment (b) function(c) modifier (d) alphanumeric(e) None of these

29. Which among the following key is used for checkinggrammar and spelling?(a) F3 (b) F5(c) F7 (d) F2(e) None of these

30. While browsing the internet , which of the following key isused for full screen view?(a) F3 (b) F5(c) F11 (d) F9(e) None of these

Page 165: yoursmahboob.wordpress.com SBI · yoursmahboob.wordpress.com iii P 101 Speed Tests for SBI Bank Clerk Exam 101 Speed Tests for SBI Bank Clerk Exam is revised and updated edition on

yoursmahboob.w

ordpress.com

1. What kind of software would you most likely use to keeptrack of a billing account?(a) Word processing (b) Electronic publishing(c) Spreadsheet (d) Web authoring(e) None of these

2. The two major categories of software include(a) operating system and utility(b) Personal productivity and system(c) system and application(d) system and utility(e) None of these

3. A directory within a directory is called.(a) Mini Directory (b) Junior Directory(c) Part Directory (d) Sub Directory(e) None of these

4. Which of the following is not a common feature of softwareapplications?(a) Menus (b) Windows(c) Help (d) Search(e) None of these

5. Software applies __________, also called algorithms, toprocess data.(a) arithmetic (b) procedures(c) objects (d) rules(e) None of these

6. Application software is designed to accomplish _________.(a) real-world tasks (b) computer-centric tasks(c) gaming tasks (d) operating-system tasks(e) None of these

7. Which is the best definition of a software package?(a) An add-on for your computer such as additional

memory(b) A set of computer programs used for a certain function

such as word processing(c) A protection you can buy for a computer(d) The box, manual and license agreement that accom-

pany commercial software.(e) None of these

RESPONSE

GRID

1. a b c d e 2. a b c d e 3. a b c d e 4. a b c d e 5. a b c d e

6. a b c d e 7. a b c d e 8. a b c d e 9. a b c d e 10. a b c d e

11. a b c d e 12. a b c d e 13. a b c d e 14. a b c d e 15. a b c d e

Max. Marks : 30 No. of Qs. 30 Time : 20 min. Date : ........./......../................

8. A sales clerk at a checkout counter scanning a tag on anitem rather than keying it into the system, is using _______.(a) input automation (b) item data automation(c) scanning automation(d) source data automation(e) None of these

9. A document that explains how to use a software program iscalled ______ manual(a) User (b) System(c) Software (d) Program(e) Technical

10. The process that deals with the technical and managementissues of software development is ______(a) Delivery process (b) Control process(c) Software process (d) Testing process(e) Monitoring process

11. Linux is a type of ________ software.(a) Shareware (b) Commercial(c) Proprietary (d) Open Source(e) Hidden type

12. The _________ of a system includes the programs orinstructions.(a) hardware (b) icon(c) information (d) software(e) None of these

13. The software that allows users to surf the Internet is calleda/ an _________(a) Search engine (b) Internet Service Provider (ISP)(c) Multimedia application(d) Browser (e) None of these

14. The ______ of software contains lists of commands andoptions.(a) menu bar (b) tool bar(c) title bar (d) formula bar(e) None of these

15. ____ is a procedure that requires users to enter anidentification code and a matching password.(a) Paging (b) Logging on(c) Time-sharing (d) Multitasking(e) None of these

7474Softwares/Programming

Page 166: yoursmahboob.wordpress.com SBI · yoursmahboob.wordpress.com iii P 101 Speed Tests for SBI Bank Clerk Exam 101 Speed Tests for SBI Bank Clerk Exam is revised and updated edition on

yoursmahboob.w

ordpress.com

158 SPEED TEST 74

RESPONSE

GRID

16. a b c d e 17. a b c d e 18. a b c d e 19. a b c d e 20. a b c d e

21. a b c d e 22. a b c d e 23. a b c d e 24. a b c d e 25. a b c d e

26. a b c d e 27. a b c d e 28. a b c d e 29. a b c d e 30. a b c d e

16. A program that works like a calculator for keeping track ofmoney and making budgets __________.(a) calculator (b) scholastic (c) keyboard(d) spreadsheet (e) None of these

17. Compiling creates a(n) ___________.(a) program specification(b) algorithm (c) executable program(d) subroutine (e) None of these

18. Multiprogramming systems:(a) are easier to develop than single programming systems.(b) execute each job faster.(c) execute more jobs in the same time period.(d) use only one large mainframe computer.(e) None of these

19. A ________ contains specific rules and words that expressthe logical steps of an algorithm.(a) programming language(b) programming structure(c) syntax(d) logic chart(e) None of these

20. _________ is a set of keywords, symbols, and a system ofrules for constructing statements by which humans cancommunicate the instructions to be executed by a computer.(a) A computer program(b) A programming language(c) An assembler(d) Syntax(e) None of these

21. ________ is the process of finding errors in software code.(a) Compiling(b) Assembling(c) Interpreting(d) Debugging(e) None of these

22. Documentation of computer programs is important so that(a) users can learn how to use the program(b) other programmers can know how to maintain the

program(c) the programmer can see why the code is written that

way while hunting for sources of error(d) All of the above(e) None of the above

23. A program that enables you to perform calculationsinvolving rows and columns of numbers is called a_________.(a) spreadsheet program(b) word processor(c) graphics package(d) window(e) None of the above

24. ______ is a feature for scheduling and multiprogrammingto provide an economical interactive system of two or moreusers(a) Time sharing (b) Multitasking(c) Time tracing (d) Multiprocessing(e) None of these

25. A programming language having a ______ is slow inexecution(a) Interpreter (b) Compiler(c) Assembler (d) Linker(e) none of these

26. Assembly language is(a) Machine Language(b) High-level programming language(c) A low-level programming language(d) Language for assembling computers(e) None of these

27. What is correcting erros in a program called?(a) Compiling (b) Debugging(c) Grinding (d) Interpreting(e) None of these

28. A (n)___is a program that makes the computer easier to use.(a) utility (b) application(c) operating system (d) network(e) None of these

29. What is used in most programs that is a part of a programand guides the user through certain steps?(a) Software (b) Wizard(c) Wiki (d) Language(e) None of these

30. Which of the following was used in programming the firstcomputers?(a) Object code (b) Source Code(c) Machine Language (d) Assembly Language(e) None of these

Page 167: yoursmahboob.wordpress.com SBI · yoursmahboob.wordpress.com iii P 101 Speed Tests for SBI Bank Clerk Exam 101 Speed Tests for SBI Bank Clerk Exam is revised and updated edition on

yoursmahboob.w

ordpress.com

1. Which of the following is a Web browser?(a) Paint (b) Power Point(c) Fire fox (d) Word(e) All are Web browsers

2. Junk e-mail is also called __________.(a) spam (b) spoof(c) sniffer script (d) spool(e) None of these

3. Most World Wide Web pages contain commands in thelanguage _________.(a) NIH (b) URL(c) HTML (d) IRC(e) FTP

4. Computers connected to a LAN (local Area Network)can _________.(a) run faster(b) go on line(c) share information and/or share peripheral equipment(d) E-mail(e) None of these

5. What does a Web site address uniquely specify?(a) Web browser (b) Web site(c) PDA (d) Storage(e) Hard-disk

6. Web pages are saved in __________ format.(a) http:// (b) HTML(c) DOC (d) URL(e) None of these

7. What are the two parts of an E-mail address?(a) User name and street address(b) Legal name and phone number(c) User name and domain name(d) Initials and password(e) login name and password

8. An educational institution would generally have thefollowing in its domain name –(a) .org (b) .edu(c) .inst (d) .com(e) .sch

RESPONSE

GRID

1. a b c d e 2. a b c d e 3. a b c d e 4. a b c d e 5. a b c d e

6. a b c d e 7. a b c d e 8. a b c d e 9. a b c d e 10. a b c d e

11. a b c d e 12. a b c d e 13. a b c d e 14. a b c d e 15. a b c d e

Max. Marks : 30 No. of Qs. 30 Time : 20 min. Date : ........./......../................

9. Which of the following is not a term pertaining to theInternet?

(a) Keyboard (b) Link

(c) Browser (d) Search Engine

(e) Hyperlink

10. In a web site, the ‘home’ page refers to –

(a) the best page (b) the last page

(c) the first page (d) the most recent page

(e) the oldest page

11. An e-mail address typically consists of a user ID followedby the _________ sign and the name of the e-mail serverthat manages the user’s electronic post office box.

(a) @ (b) #

(c) & (d) «

(e) None of these

12. A Web __________ consists of one or more Web pageslocated on a Web server.

(a) hub (b) site

(c) story (d) template

(e) None of these

13. Programs such as Internet Explorer that serve as navigablewindows into the Web are called

(a) Hypertext (b) Networks

(c) Internet (d) Web browsers

(e) None of these

14. A word in a web page that, when clicked, opens anotherdocument.

(a) anchor (b) URL

(c) hyperlink (d) reference

(e) None of these

15. WWW stands for ________.

(a) World Work Web (b) Wide Work Web

(c) Wide World Web (d) World Wide Web

(e) None of the above.

7575Internet, Networking andComputer Abbreviations

Page 168: yoursmahboob.wordpress.com SBI · yoursmahboob.wordpress.com iii P 101 Speed Tests for SBI Bank Clerk Exam 101 Speed Tests for SBI Bank Clerk Exam is revised and updated edition on

yoursmahboob.w

ordpress.com

SPEED TEST 75160

16. The collection of links throughout the Internet creates aninterconnected network called the _________.(a) WWW (b) Web(c) World Wide Web (d) All of the above(e) Wide Area Web

17. ASCII stands for(a) American Special Computer for Information Interaction(b) American Standard Computer for Information

Interchange(c) American Special Code for Information Interchange(d) American Special Computer for Information

Interchange(e) American Standard Code for Information Interchange

18. POST stands for(a) Power on Self Test(b) Program on Self Test(c) Power on System Test(d) Program on System Test(e) Power Off System Test

19. CPU stands for _________(a) Computer Processing Unit(b) Central Processing Unit(c) Computer Protection Unit(d) Central Processing Upload(e) None of the above

20. OCR stands for _________(a) Optical Character Recognition(b) Optical CPU Recognition(c) Optimal Character Rendering(d) Other Character Restoration(e) None of these

21. Line printer speed is specified in terms of :(a) LPM (Line per minute)(b) CPM (Character per minute)(c) DPM(d) Any of the above(e) None of these

22. What does ‘DOS’ stand for ?(a) Disk Originating System(b) Dynamic Operating System(c) Disk Operating System(d) Default Operating System(e) None of these

23. FAT stands for(a) File Activity Table(b) File Allocation Table(c) File Access Tape(d) File Accommodation Table(e) None of these

24. ‘IMAP’ in computer language stands for(a) Internet Message Access Protocol(b) Internet Money Access Protocol(c) Intrusion Message Access Protocol(d) Internal Message Access Protocol(e) International Marketing Authority Protocol

25. The BIOS is the abbreviation of(a) Basic Input Output System(b) Best Input Output System(c) Basic Input Output Symbol(d) Base Input Output System(e) None of these

26. The computer abbreviation KB usually means(a) Key Block (b) Kernal Boot(c) Key Byte (d) Kit Bit(e) Kilo Byte

27. URL stands for(a) Universal Resource List(b) Universal Research List(c) Uniform Resource List(d) Uniform Research Locater(e) Uniform Resource Locater

28. The meaning of double-click is(a) pushing and releasing the main mouse button twice in

rapid succession when the on-tree mouse pointer ispositioned over the desired item

(b) appearance of an icon(c) to take a selection from the document and move it to

the clipboard(d) All of the above(e) None of these

29. BPS stands for(a) Bits Per Second (b) Bits Per Season(c) Bytes Per Second (d) Bits Per System(e) None of these

30. Reusable optical storage will typically have theacronym _________(a) CD (b) DVD(c) ROM (d) RW(e) None of these

RESPONSE

GRID

16. a b c d e 17. a b c d e 18. a b c d e 19. a b c d e 20. a b c d e

21. a b c d e 22. a b c d e 23. a b c d e 24. a b c d e 25. a b c d e

26. a b c d e 27. a b c d e 28. a b c d e 29. a b c d e 30. a b c d e

Page 169: yoursmahboob.wordpress.com SBI · yoursmahboob.wordpress.com iii P 101 Speed Tests for SBI Bank Clerk Exam 101 Speed Tests for SBI Bank Clerk Exam is revised and updated edition on

yoursmahboob.w

ordpress.com

1. Marketing is:(a) Only selling(b) meeting human & social needs while earning profits(c) focus on customer(d) focus on producing goods/ service(e) Both (b) and (c)

2. Long term objective of marketing is(a) customer satisfaction(b) profit maximisation(c) cost cutting(d) profit maximisation with customer(e) None of these

3. Market information means(a) knowledge of companies(b) cross-country information(c) knowledge of related markets(d) knowledge of current customers(e) None of these

4. Marketing and Selling are(a) not required if profit is high(b) not required if sales are high(c) not required in monopolistic conditions(d) All of the above(e) None of these

5. Direct Marketing is neccessary for(a) having a focussed approach(b) boosting sales(c) better customer contacts(d) All of the above(e) None of these

6. Marketing is required for(a) boosting production(b) reducing costs(c) boosting profits(d) improving customer service(e) All of the above

7. Marketing is successful when(a) demand exceeds supply(b) supply exceeds demand(c) exports are heavy and costly(d) salesmen are effective(e) All the above

8. In marketing the benefits of selling extend to(a) Only products and services(b) Only after sales services

RESPONSE

GRID

1. a b c d e 2. a b c d e 3. a b c d e 4. a b c d e 5. a b c d e

6. a b c d e 7. a b c d e 8. a b c d e 9. a b c d e 10. a b c d e

11. a b c d e 12. a b c d e 13. a b c d e 14. a b c d e 15. a b c d e

Max. Marks : 30 No. of Qs. 30 Time : 20 min. Date : ........./......../................

(c) Lifelong relationship with the buyer(d) All of these(e) None of these

9. A Market Plan is ________.(a) company's prospectus(b) Memorandum of Association(c) document for marketing strategies(d) business goals(e) action plan for better production

10. Marketing helps in _______.(a) boosting production(b) getting new clients(c) interacting with strangers(d) All of these(e) None of these

11. Marketing is the art of ________.(a) buying more (b) paying more(c) selling more (d) talking more(e) only (a) & (b)

12. Selling is _______.(a) different from Marketing(b) a sub-function of marketing(c) same as Marketing(d) more than Marketing(e) All of these

13. Social Marketing is _______.(a) Share market prices(b) Marketing by the entire society(c) Internet Marketing(d) Marketing for a social cause(e) Society bye-laws

14. Marketing is the combined study of(a) Buyer’s behaviour and consumer tasks(b) Product demand and Product supply(c) Brand building and Publicity(d) Sales force abilities and customer response(e) All of the above

15. The sales process begins with(a) customer identification(b) lead generation(c) sales presentation(d) sales closure(e) sales meet

7676Fundamentals ofMarketing,

Product and Branding

Page 170: yoursmahboob.wordpress.com SBI · yoursmahboob.wordpress.com iii P 101 Speed Tests for SBI Bank Clerk Exam 101 Speed Tests for SBI Bank Clerk Exam is revised and updated edition on

yoursmahboob.w

ordpress.com

162 SPEED TEST 76

RESPONSE

GRID

16. a b c d e 17. a b c d e 18. a b c d e 19. a b c d e 20. a b c d e

21. a b c d e 22. a b c d e 23. a b c d e 24. a b c d e 25. a b c d e

26. a b c d e 27. a b c d e 28. a b c d e 29. a b c d e 30. a b c d e

16. XXX is selecting and analyzing a target market anddeveloping a marketing mix to gain long-run competitiveadvantages. XXX is creating a.(a) Corporate strategy (b) Target design(c) Mix strategy (d) Marketing strategy(e) None of these

17. Critical success factors for a firm includes ________.(a) Changing lifestyles and attitudes(b) Low-cost production efficiency(c) Both (a) and (b)(d) Marketing strategy(e) None of the above

18. A differentiated product may be unique by itself but it willonly be successful only ________ .(a) if it satisfies customers' needs(b) if price differential is minimal(c) if brand can be classed as aspirational(d) differentiated products will always be successful(e) None of these

19. Establishing and maintaining a distinctive place in the marketfor an organization/product is ________.(a) Profiling (b) Profiling segmentation(c) Segmentation (d) Positioning(e) None of these

20. _____ of ad means how frequently you should expose yourtarget group to your message.(a) Frequency (b) Copy(c) Copy strategy (d) Media(e) None of these

21. Market is divided into groups on the basis of age, familysize, gender, income, occupation, education, religion, race,generation, nationality, or social class is the best descriptionof _____ .(a) Demographics (b) Psychographics(c) Behavioral (d) Geographic(e) None of these

22. What approach should a brand manager adopt to know thestatus of a brand in terms of consumer perceptions?(a) Compare two or three brands(b) Analyze market segmentations(b) Select potential target markets(c) Understand customer's needs(e) None of these

23. Introducing additional items in the same product categoryby adding new flavors, forms, colors, ingredients or packagesizes, under the same brand name, is _____.(a) Line extensions (b) Product mix(c) Interactive marketing (d) Service intangibility(e) None of these

24. To investigate new markets _____ management function isimportant.(a) Finance functions. (b) Marketing.(c) Production. (d) HRM.(e) None of these

25. _____ is a “category killer”.(a) Products which stock must have products that need to

be stocked by retailers due to consumer demand likeCoca Cola & Kellogg's.

(b) These are speciality stores with a deep product line tobe sold in restricted shop space

(c) These are retail outlets with a narrow product focusbut sell products at low prices by bulk buying, lowmargins and selling high volumes

(d) These are retail outlets with a wide product focus butwith a wide width and depth to products

(e) These are retail outlets with a narrow product focusbut with wide width and depth

26. Product life cycle theory maximizes profit at.(a) Developed Stage (b) Early Stage(c) Matured Stage (d) Declined Stage(e) Cannot be predicted

27. Marketing Plans are used for(a) doing Research by Marketing students(b) planning Departments(c) purchase of consumable items from retail outlets(d) All of the above(e) None of these

28. Product mix means(a) distributing mix products(b) collecting ideas to sell better(c) satisfying the customer(d) bundle of products required by the customer(e) products designed by the Company

29. Buyer Resistance' means _____.(a) Buyer's interest in the product(b) Buyer being aggressive with the seller(c) Buyer's hesitation in buying the product(d) Buyer becoming a seller(e) Buyer purchasing the product

30. Direct Marketing is useful for _____ .(a) Designing Products(b) Sending e-mails(c) Increased production(d) Bigger job opportunities(e) None of these

Page 171: yoursmahboob.wordpress.com SBI · yoursmahboob.wordpress.com iii P 101 Speed Tests for SBI Bank Clerk Exam 101 Speed Tests for SBI Bank Clerk Exam is revised and updated edition on

yoursmahboob.w

ordpress.com

1. A company's own retail outlets are meant _____.(a) To avoid the threat of distributors' power(b) To own and control the distribution channel(c) Distribution is profitable(d) All of the above(e) None of the above

2. Advertising _____ can attract consumers only if it is basedon their needs.(a) Reach (b) Copy(c) Frequency (d) Media(e) None of these

3. Major source of power in a distribution channel is the _____.(a) Company (b) Brand(c) Distributor (d) Customer(e) None of these

4. Factors affecting choice of distribution channel include_____.(a) Customer value (b) Sales revenues(c) Both [a] and [b] (d) Customer services(e) None of the above

5. Air India runs TV commercials that show its staff going outof their way to help customers. An important secondaryaudience for these ads is:(a) Civil aviation authority(b) Competitors(c) Air India employees(d) Air travelers(e) None of these

6. Pricing models offers opportunity to set different prices fordifferent needs is _____.(a) Segment pricing(b) Skim pricing(c) Value-in-use pricing(d) Strategic account pricing(e) None of these

7. Advertising helps sales promotion by creating awarenessand comprehension that creates _____.(a) Customer pull (b) Customer push(c) Customer loyal (d) Customer image(e) None of these

RESPONSE

GRID

1. a b c d e 2. a b c d e 3. a b c d e 4. a b c d e 5. a b c d e

6. a b c d e 7. a b c d e 8. a b c d e 9. a b c d e 10. a b c d e

11. a b c d e 12. a b c d e 13. a b c d e 14. a b c d e

Max. Marks : 30 No. of Qs. 30 Time : 20 min. Date : ........./......../................

8. Advertising copy _____.(a) Provides continuity in a brand's advertising(b) Help a brand achieve distinctiveness(c) Provides a common benchmark on which all concerned

in the company and the agency can evaluate merits ofadvertising campaign

(d) All of the above(e) None of these

9. Duration of _____ should be short and should not berepeated.(a) Sales promos (b) Market promos(c) Brand promos (d) Product promos(e) None of these

10. An effective advertising campaign _____(a) Revolves around a strong central idea(b) Should appeal to consumers self interest(c) Must not be generalised(d) All of the above(e) None of these

11. Prices of luxury product are explained by _____ .(a) Plus-one pricing(b) Skim pricing(c) Strategic account pricing(d) Segment pricing(e) None of these

12. _____ about Place/Distribution decisions is CORRECT?(a) Product classes are not related to Place objectives(b) The product life cycle is not related to Place objectives(c) Place decisions are short-term decisions that are easy

to change(d) Different market segments may require separate Place

arrangements.(e) None of these

13. Addition of 2.25 liter bottle by Coca Cola will ____ .(a) Increase customer base and usage(b) Enhance customer loyalty(c) Generate more profit(d) Develop brand image(e) All of the above

14. Communication through a news story about an organizationand its products that is transmitted through a mass mediumat no charge is ____.(a) Advertising (b) Sales promotion(c) Personal selling (d) Publicity(e) None of these

7777Market Situations Based onPrice, Distribution,

Promotion and Advertising

Page 172: yoursmahboob.wordpress.com SBI · yoursmahboob.wordpress.com iii P 101 Speed Tests for SBI Bank Clerk Exam 101 Speed Tests for SBI Bank Clerk Exam is revised and updated edition on

yoursmahboob.w

ordpress.com

164 SPEED TEST 77

RESPONSE

GRID

15. a b c d e 16. a b c d e 17. a b c d e 18. a b c d e 19. a b c d e

20. a b c d e 21. a b c d e 22. a b c d e 23. a b c d e 24. a b c d e

25. a b c d e 26. a b c d e 27. a b c d e 28. a b c d e 29. a b c d e

30. a b c d e

15. _____ is a disadvantage of using an agent in a channel ofdistribution?(a) Length of channel (b) Cost factor(c) Lack of control (d) Speed of distribution (e) Absence of market knowledge

16. _____ enables a company to control channels ofdistribution?(a) Vertical marketing system(b) Franchising(c) Exclusive dealing arrangements(d) Vertical integration(e) All of the above

17. Distribution where a limited number of outlets in ageographical area to sell its products are used is called_____?(a) Exhaustive distribution(b) Exclusive distribution(c) Intensive distribution(d) Selective distribution(e) Segmented distribution

18. Power in marketing channels is now with _____ ?(a) Distributor (b) Manufacturer(c) Retailer (d) Wholesaler(e) Consumer

19. ______ is a strength associated with franchising?(a) Reduces marketing promotional and administration

costs(b) Goal conflict does not arise(c) Reduces levels of channel conflict within the channel(d) Combine the strengths of a large sophisticated

marketing-oriented organization with the energy andmotivation of a locally owned outlet

(e) All of the above20. Best method of advertisement is

(a) glow sign boards (b) internet(c) Television (d) Consumer awareness(e) customer satisfaction

21. A form of distribution in which manufacturer makes an agreementwith a middleman in each market stipulating that the distributionof the product will be confined in that area is:(a) mass distribution(b) exclusive agency distribution(c) selective distribution(d) price based distribution(e) None of these

22. If Marketing is done effectively, _____ is not required(a) Advertisment (b) Pubilicity(c) Market Research (d) Market Segmentation(e) None of these

23. Sales forecasting involves(a) sales planning (b) sales pricing(c) distribution channels (d) consumer tastes(e) All of these

24. Proper pricing is essential for ____.(a) extra charges for extra services(b) levy of VAT(c) good customer service(d) putting burden on the customer(e) service with extra facilities

25. Selling skills are measured by ____.(a) number of goods sold(b) amount of profit earned(c) number of customers converted(d) All of the above(e) None of these

26. Promotion means(a) additional responsibility(b) undertaking research in marketing(c) advertisement and publicity for marketing(d) going up the promotional ladder(e) All of the above

27. Negotiation skills help in ____.(a) evolving a consensus(b) breaking the ice(c) carry marketing further(d) Mutual win-win situation(e) All of these

28. Relationship Marketing is useful for(a) trade between relatives(b) trade between sister concerns(c) cross-selling of products(d) preparing a list of relatives(e) There is no such term as Relationship Marketing

29. Good Public Relations means improved _____.(a) Marketing skills (b) Brand Image(c) Customer Service (d) All of these(e) None of these

30. One method of Market Monitoring is ____.(a) Monitor Performance of sales staff.(b) Monitor the SENSEX(c) Monitor Media Outlets(d) Monitor profits(e) None of these

Page 173: yoursmahboob.wordpress.com SBI · yoursmahboob.wordpress.com iii P 101 Speed Tests for SBI Bank Clerk Exam 101 Speed Tests for SBI Bank Clerk Exam is revised and updated edition on

yoursmahboob.w

ordpress.com

1. 4 P's of Marketing are ____.(a) Primary Marketing Techniques(b) Person, Place, Product and Promotion(c) Promoting Authority(d) Purpose, Place, Passion, and Product(e) None of these

2. Qualities essential in good marketing are ____.(a) aggressiveness (b) pushy(c) perseverance (d) politeness(e) Only (c) and (d)

3. ______ is not a part of 7 P's of Marketing.(a) Product (b) Price(c) Production (d) Promotion(e) People

4. Market Share means _____.(a) share market (b) share prices(c) IPOs (d) Scope for marketing(e) Share of business among pproducers

5. If lots of customers like the brand and are inclined to bebound into a contract, they would be known as _____.(a) Loyal customer (b) Difficult customer(c) Potential customer (d) Finicky customer(e) None of these

6. _____ evokes a hierarchical set of customer response effectsi.e. building awareness, comprehension, intentions, andactions.(a) Distribution (b) Communication(c) Merchandizing (d) Branding(e) None of these

7. Apart from the 'four Ps' of marketing mix, the three additionalelements of service brands are people, process, and ____ .(a) Physical evidence(b) Physiological evidence(c) Psychological evidence(d) Packaging(e) None of these

8. Unique Selling Proposition (USP) started in _____.(a) Advertising era (b) Image era(c) Product era (d) Positioning era(e) None of these

RESPONSE

GRID

1. a b c d e 2. a b c d e 3. a b c d e 4. a b c d e 5. a b c d e

6. a b c d e 7. a b c d e 8. a b c d e 9. a b c d e 10. a b c d e

11. a b c d e 12. a b c d e 13. a b c d e 14. a b c d e 15. a b c d e

Max. Marks : 30 No. of Qs. 30 Time : 20 min. Date : ........./......../................

9. Positioning has to stem from the point of view of _____.(a) Customers' (b) Competitors'(c) General Managers' (d) Brand Owners'(e) None of these

10. _____ is a marketer's major positioning tool that has a directimpact on product performance and is linked to customervalue.(a) Product quality (b) Social marketing(c) Specialty marketing (d) Production quality(e) None of these

11. ______ determines why customers buy?(a) Customer needs analysis(b) Brand-based customer model(c) Good brand promise(d) Brand management process(e) None of these

12. Marketers need to position their brands clearly in targetcustomers' minds. The strongest brands go beyond attributesor benefit positioning. They are positioned on the basis of_____.(a) Desirable benefit(b) Good packaging(c) Strong beliefs and values(d) Service inseparability(e) None of these

13. Target marketing featuring customized marketingprogrammes is _______.(a) Individual marketing (b) Segment marketing(c) Family marketing (d) Local marketing(e) Niche marketing

14. _____ segmentation divides the market into different unitslike nations, states, regions, cities or neighbourhoods.(a) Geographic (b) Demographic(c) Psychographic (d) Behavioral(e) Socio-economic

15. _____ segmentation is where the market is divided on thebasis of age, family size, life cycle, gender, income,occupation, education and religion .(a) Geographic (b) Demographic(c) Psychographic (d) Behavioral(e) Socio-economic

7878Market Segmentation,Targeting and Positioning

Page 174: yoursmahboob.wordpress.com SBI · yoursmahboob.wordpress.com iii P 101 Speed Tests for SBI Bank Clerk Exam 101 Speed Tests for SBI Bank Clerk Exam is revised and updated edition on

yoursmahboob.w

ordpress.com

166 SPEED TEST 7816. To be useful market segments must assess on _____ criteria.

(a) five (b) two(c) three (d) four(e) Six

17. Market Segmentation means(a) dividing the market into groups(b) segmenting by age(c) segmenting by tastes(d) geographic segmenting(e) None of the above

18. In Marketing, Market penetration means(a) entering customers houses(b) covering stores and shops(c) covering a wide market(d) All of the above(e) None of these

19. Rural Marketing is more effective if arranged through.(a) melas(b) village fairs(c) door to door campaign(d) All of these(e) None of these

20. ‘Buyer Resistance’ means _____.(a) Buyer’s interest in the product(b) Buyer fighting with the seller(c) Buyer’s hesitation in buying the product(d) Buyer becoming a seller(e) Buyer buying the product

21. Opportunities for growth and expansion are identified byfinding _____(a) Customers' beliefs about the segment(b) Customers believe about competitors(c) Customer's perceptions about the brand(d) Customer's response about the products(e) None of these

22. What explains Maslow's Motivation Theory best?(a) Importance of motivation of customer development(b) When people are driven by particular need at particular

times(c) When human needs are arranged in a hierarchy(d) All of these(e) Only (b) and (c)

23. Customer Database is used by(a) individuals (b) institutions(c) builders (d) marketing experts(e) None of these

24. Motivation is essential to effective marketing. What otherqualities are required for marketing?(a) Confidence(b) Effective Communication skills(c) Team work(d) Perseverance(e) All of the above

25. Market Penetration is possible through(a) more calls to the same buyers(b) more calls to potential buyers(c) surrogate marketing(d) alternate marketing(e) All of these

26. Market penetration connotes(a) covering a wider area(b) entering sellers' houses(c) covering all shops and business houses(d) All of these(e) None of these

27. Lead generation means ____.(a) Tips for selling tactics(b) Tips for more efficient production(c) Develop leaders(d) sources for prospective clients(e) None of these

28. Marketing orientation focuses on _____?(a) Customers (b) Suppliers(c) Competitors (d) Employees(e) All of the above

29. ‘Push’ marketing style requires _____.(a) Proper planning(b) Good pushing strength(c) Teamwork(d) Ability to identify products(e) Aggressive marketing

30. CRM (Customer Relationship Management) is _____.(a) A pre-sales activity(b) A tool for lead generation(c) An on going daily activity(d) Task of a DSA(e) Customer complaint cell

RESPONSE

GRID

16. a b c d e 17. a b c d e 18. a b c d e 19. a b c d e 20. a b c d e

21. a b c d e 22. a b c d e 23. a b c d e 24. a b c d e 25. a b c d e

26. a b c d e 27. a b c d e 28. a b c d e 29. a b c d e 30. a b c d e

Page 175: yoursmahboob.wordpress.com SBI · yoursmahboob.wordpress.com iii P 101 Speed Tests for SBI Bank Clerk Exam 101 Speed Tests for SBI Bank Clerk Exam is revised and updated edition on

yoursmahboob.w

ordpress.com

1. Modern marketing includes(a) publicity on internet(b) advertisement on internet(c) Bulk emails(d) telemarketing(e) All of these

2. e-marketing is same as _______.(a) virtual marketing (b) digital marketing(c) real time marketing (d) All of these(e) None of these

3. Digital marketing is selling ______.(a) digital goods (b) calculators(c) through internet (d) All of these(e) None of these

4. Online Marketing is useful for ______.(a) Selling Old Products(b) Sending e-mails(c) Increasing production(d) Additional job opportunities(e) Higher expenses

5. Online value proposition should _______.(a) Be communicated to site visitors and in all marketing

communications(b) Be a clear differentiator from online competitors(c) Target market segment(s) that the proposition will

appeal to(d) Given financial back up(e) All of the above

6. Achieving marketing objectives through use of electroniccommunications technology is _______.(a) E-marketing (b) E-business(c) Internet marketing (d) E-commerce(e) None of the above

7. Direct online contribution effectiveness is the _______.(a) Reach of audience volume of a site(b) Proportion of sales influenced by the web site(c) Proportion of business turnover achieved by

e-commerce transactions(d) First and third option above(e) None of the above

8. Modern marketing EXCLUDES _______.(a) Digital marketing

RESPONSE

GRID

1. a b c d e 2. a b c d e 3. a b c d e 4. a b c d e 5. a b c d e

6. a b c d e 7. a b c d e 8. a b c d e 9. a b c d e 10. a b c d e

11. a b c d e 12. a b c d e 13. a b c d e 14. a b c d e 15. a b c d e

Max. Marks : 30 No. of Qs. 30 Time : 20 min. Date : ........./......../................

(b) Tele-marketing(c) Door-to-door marketing(d) E-mail solicitation(e) None of these

9. Web marketing involves(a) Selling web cameras (b) Web advertisements(c) e-mail chatting (d) Browsing the web(e) Door-to-door canvassing

10. Online Marketing is mostly useful for marketing of(a) saving accounts (b) credit cards(c) home loans (d) NRI deposits(e) business accounts

11. The best promotional tool in any marketing is _______(a) Pamphlets(b) Newsletters(c) Word of mouth publicity(d) Regional Advertisement(e) Viral marketing

12. Internet marketers are using ________ as a form of word ofmouth, or word of mouse, to draw attention to their sites.(a) event marketing (b) subliminal marketing(c) viral marketing (d) place marketing(e) public relations

13. The major marketing developments as we enter the newmillennium can be summed up in a single theme:(a) innovation (b) the Internet(c) virtuality (d) connectedness(e) None of these

14. Today marketing is:(a) Product driven(b) Services driven(c) Improvement of bottom line (Profitability)(d) Cost conscious(e) Customer driven market

15. Modern marketing calls for more than developing a goodproduct, pricing it attractively, and making it accessible.Companies must also ________ with present and potentialstakeholders, and the general public.(a) attract (b) reach(c) relate to (d) advertise to(e) communicate

7979Modern Marketing /Marketing in

Banking Industry

Page 176: yoursmahboob.wordpress.com SBI · yoursmahboob.wordpress.com iii P 101 Speed Tests for SBI Bank Clerk Exam 101 Speed Tests for SBI Bank Clerk Exam is revised and updated edition on

yoursmahboob.w

ordpress.com

168 SPEED TEST 7916. In today's changing banking scenario, aggressive promotion

of business is necessary where the competition exists on(a) branch up-keep(b) expeditious service(c) use of advanced digital technology(d) good customer service(e) All of these

17. Bank marketing means(a) selling by banks(b) buying by banks(c) merger of banks(d) selling bank's products and services(e) selling products in banks

18. Marketing is not required in ________.(a) Selling Credit/ Debit Cards(b) Net Banking (c) Corporate Loans(d) Retail Loans (e) All of these

19. Marketing of Internet Banking means(a) meeting of Banks on the net(b) net practice(c) marketing usage of Banking transactions through

internet(d) transactions with foreign banks(e) All of the above

20. Marketing in banks is(a) a one-day function (b) a one-man function(c) a one-off affair (d) All of these(e) None of these

21. Target group for marketing Internet Banking is _______.(a) all customers(b) all literate customers(c) all computer literate customers(d) only borrowers(e) All of these

22. Digital Banking is available through _______.(a) Mobile phones (b) Internet(c) Telephones (d) All of these(e) None of these

23. Bancassurance can be sold to _____.(a) Banks (b) Insurance companies(c) Insurance agents (d) Bank customers(e) All of the above

24. Difference between Direct and indirect Bank Marketing is(a) Direct Marketing is to Bank's employees. Indirect is to

outsiders(b) Direct Marketing is to outsiders. Indirect is to

employees(c) Direct Marketing is to Bank's owners. Indirect is to

outsiders(d) Direct Marketing is to other Bank's employees. Indirect

is to outsiders(e) None of these

25. Target market for Home Loans is(a) builders (b) housing societies(c) agriculturists (d) All of these(e) None of these

26. The USP of a Credit Card is(a) cashless operations(b) only for HNIs(c) only for men(d) only for employed persons(e) transactions through cheque book

27. EMI can be a marketing tool if(a) EMI is increasing(b) It is very high(c) It is very low(d) EMI has no impact on marketing(e) EMI is a flat rate

28. Banks sell insurance for(a) increasing deposits(b) increasing loans(c) increasing clients(d) earning more profits(e) taking over insurance companies

29. Savings Accounts can be opened by _______(a) All individuals fulfilling KYC norms(b) All tax payers only(c) All individuals above the age of 18(d) All businessmen only(e) All students below the age of 18

30. A short term loan is repayable within _______(a) 20 years(b) 3 years(c) As per the borrowers’ wish(d) As per the guarantor’s wish(e) There is no need to repay short term loans

RESPONSE

GRID

16. a b c d e 17. a b c d e 18. a b c d e 19. a b c d e 20. a b c d e

21. a b c d e 22. a b c d e 23. a b c d e 24. a b c d e 25. a b c d e

26. a b c d e 27. a b c d e 28. a b c d e 29. a b c d e 30. a b c d e

Page 177: yoursmahboob.wordpress.com SBI · yoursmahboob.wordpress.com iii P 101 Speed Tests for SBI Bank Clerk Exam 101 Speed Tests for SBI Bank Clerk Exam is revised and updated edition on

yoursmahboob.w

ordpress.com

1. Devices that enter information and let you communicate with thecomputer are called _______.(a) Software (b) Output devices(c) Hardware (d) Input devices(e) Input/Output devices

2. What is the function of the Central Processing Unit of a Computer?(a) Creates invoices(b) Performs calculations and processing(c) Deletes Data(d) Corrupts the data(e) None of these

3. All the characters that a device can use is called its ?(a) Skill Set (b) Character Alphabet(c) Character Codes (d) Keyboard Characters(e) Character Set

4. If your computer keeps rebooting itself, then it is likely that_______(a) It has a virus(b) It does not have enough memory(c) There is no printer(d) There has been a power surge(e) It needs a CD-ROM

5. What utility to you use to transfer files and exchange messages ?(a) Web browsers (b) WWW(c) Email (d) Hypertext(e) search engines

6. Which unit controls the movement of signals between CPU and I/O ?(a) ALU (b) Control Unit(c) Memory Unit (d) Secondary Storage(e) None of these

7. The three main parts of the processor are _______(a) ALU, Control Unit and Registers(b) ALU, Control Unit and RAM(c) Cache, Control Unit and Registers(d) Control Unit, Registers and RAM(e) RAM, ROM and CD-ROM

8. Which of the following does not relate to Input Unit ?(a) If accepts data from the outside world.(b) It converts data into binary code that is understandable by

the computer(c) It converts binary data into the human readable form that is

understandable to the users.(d) It sends data in binary form to the computer for further

processing(e) None of these

9. Which of the following terms is just the collection of networks thatcan be joined together ?

Max. Marks : 40 No. of Qs. 40 Time : 25 min. Date : ........./......../................

(a) virtual private network (b) LAN(c) intranet (d) extranet(e) internet

10. Tangible, physical computer equipment that can be seen andtouched is called _______(a) hardware (b) software(c) storage (d) input/output(e) None of these

11. Which of the following is the second largest measurement of RAM?(a) Terabyte (b) Megabyte(c) Byte (d) Gigabyte(e) Mega Hertz

12. What resides on the motherboard and connects the CPU to othercomponents on the motherboard ?(a) Input Unit (b) System Bus(c) ALU (d) Primary Memory(e) None of these

13. Supercomputers _______(a) are smaller in size and processing capability than mainframe

computers(b) are common in majority of households(c) contain thousands of microprocessors(d) are rarely used by researchers due to their lack of computing

capacity(e) are of the same size as laptops

14. The basic computer processing cycle consists of _______(a) input, processing and output(b) systems and application(c) data, information and applications(d) hardware, software and storage(e) None of these

15. The system unit(a) coordinates input and output devices(b) is the container that houses electronic components(c) is a combination of hardware and software(d) controls and manipulates data(e) does the arithmetic operations

16. System software(a) allows the user to diagnose and troubleshoot the device(b) is a programming language(c) is part of a productivity suite(d) is an optional form of software(e) helps the computer manage internal resources

17. Microsoft's Messenger allows users to(a) bypass a browser to surf the Web(b) create a blog(c) communicate via direct live communication(d) identify and eliminate spam(e) make graphic presentations

RESPONSE

GRID

1. a b c d e 2. a b c d e 3. a b c d e 4. a b c d e 5. a b c d e

6. a b c d e 7. a b c d e 8. a b c d e 9. a b c d e 10. a b c d e

11. a b c d e 12. a b c d e 13. a b c d e 14. a b c d e 15. a b c d e

16. a b c d e 17. a b c d e

8080Section Test :Computer Knowledge/

Marketing Aptitude

Page 178: yoursmahboob.wordpress.com SBI · yoursmahboob.wordpress.com iii P 101 Speed Tests for SBI Bank Clerk Exam 101 Speed Tests for SBI Bank Clerk Exam is revised and updated edition on

yoursmahboob.w

ordpress.com

170 SPEED TEST 8018. What is the short-cut key to highlight the entire column?

(a) Ctrl + Page up (b) Ctrl + Page down(c) Ctrl + Enter (d) Ctrl + Space bar(e) Ctrl + C

19. To save the current document or to open a previously saveddocument, _________ is used.(a) file menu (b) tools menu(c) view menu (d) edit(e) review menu

20. In a computer system, _________ device is functionally oppositeof a keyboard.(a) joystick (b) mouse(c) trackball (d) printer(e) scanner

21. Marketing in banks is defined as(a) Negotiable Instruments Act(b) Banking Regulation Act(c) Reserve Bank of India Act(d) Companies Act(e) None of these

22. In marketing terms, attitude can best be defined as a(a) rude behaviour of salesperson(b) rude behaviour of consumer(c) mental state of consumer(d) ego of the marketing executive(e) None of these

23. According to product life cycle theory, the profit is maximum in(a) developed Stage (b) early stage(c) matured Stage (d) declined Stage(e) None of these

24. In banks, loans & advances are considered as(a) assets (b) liabilities(c) resources (d) cause of expenditure(e) None of these

25. A firm is productively efficient when(a) it is producing its product or service at the lowest unit cost

that it can.(b) it is selling at the lowest price possible.(c) it has the highest labour productivity that it can.(d) it is making what its customers want.(e) None of these

26. Vegetable market in India is nearly an example of(a) perfect Competition (b) monopoly(c) oligopoly (d) imperfect competition(e) high monopsony

27. Marketing in banks is(a) a one-day function (b) a one-man function(c) a one-off affair (d) All of these(e) None of these

28. Modern methods of marketing include(a) publicity on the net (b) advertisement on the net(c) soliciting business through e-mails(d) telemarketing (e) All of these

29. In a selling process in today’s world(a) only standard products are sold(b) no customization required(c) the seller need not have product knowledge(d) the seller should aim at customer satisfaction(e) only quantum of sales matters

30. In Marketing Mix 4P’s imply(a) Product, Price, Place, Promotion(b) Product, Price, Policy, Place(c) Product, Place, Promotion, Policy

(d) Place, People, Product, Policy(e) Promotion, Product, Price, People

31. For an economic organization like bank, MIS means(a) Middle Income Scheme(b) Management Information System(c) Management of Information & Science(d) Marketing Information System(e) Only (b) and (c)

32. Market segmentation is done by firms for all of the followingreasons, except(a) to increase market share.(b) to assist new product development.(c) so that they can develop multi-purpose advertising

campaigns.(d) to extend products into new markets.(e) None of these

33. Which among the following is not a tool for marketing planningappraisal?(a) External appraisal (b) Internal appraisal(c) Gap analysis (d) SWOT analysis(e) PCOT analysis

34. Cross-selling covers(a) identifying customer needs(b) matching the products to customer needs(c) convincing the customers of product benefits(d) responding to questions and objections of customers(e) All of these

35. Market information means(a) knowledge of shops and bazaars(b) knowledge of shopping malls(c) knowledge of customer profile and product mix(d) knowledge of various languages(e) None of these

36. Market research is needed for(a) deciding the market area(b) deciding the right product to be sold(c) making proper marketing decisions(d) deciding right time to sell(e) All of these

37. A product life cycle(a) shows how a products sales or profits, depending on the

units used, may rise and fall over its life.(b) tells you how long a product will sell for and make a profit.(c) is divided into three stages.(d) shows how profitable a product will be.(e) None of these

38. The long term objective of marketing is(a) customer satisfaction (b) profit maximisation(c) cost cutting(d) profit maximisation with customer(e) None of these

39. Proper pricing is needed for(a) extra charges for extra services(b) levy of VAT(c) good customer service(d) putting burden on the customer(e) service with extra facilities

40. All of the following are examples of unfair competition, except(a) two firms agreeing to fix their prices.(b) three companies agreeing to share a market between them.(c) a monopolist charging excess prices for its product.(d) exploiting a patent that the firm has on a product it has

developed itself.(e) None of these

RESPONSE

GRID

18. a b c d e 19. a b c d e 20. a b c d e 21. a b c d e 22. a b c d e

23. a b c d e 24. a b c d e 25. a b c d e 26. a b c d e 27. a b c d e

28. a b c d e 29. a b c d e 30. a b c d e 31. a b c d e 32. a b c d e

33. a b c d e 34. a b c d e 35. a b c d e 36. a b c d e 37. a b c d e

38. a b c d e 39. a b c d e 40. a b c d e

Page 179: yoursmahboob.wordpress.com SBI · yoursmahboob.wordpress.com iii P 101 Speed Tests for SBI Bank Clerk Exam 101 Speed Tests for SBI Bank Clerk Exam is revised and updated edition on

yoursmahboob.w

ordpress.com

1. Reserve Bank of India was established on(a) April 12, 1932 (b) April 1, 1935(c) May 2, 1943 (d) November 13, 1941(e) None of these

2. Reserve Bank of India was nationalised on(a) 21 May, 1948 (b) 13 July, 1951(c) 1 January, 1949 (d) 12 October, 1951(e) None of these

3. Six Commercial banks were nationalised by the Governmenton(a) April 15, 1980 (b) May 10, 1981(c) July 21, 1982 (d) May 13, 1984(e) None of these

4. The 14 major banks were nationalised by the Governmenton(a) June 19, 1967 (b) July 19, 1969(c) May 21, 1962 (d) May 12, 1963(e) None of these

5. Which of the following is the first commercial bank?(a) State Bank of India(b) Oudh Commercial Bank a(c) Union Bank of India(d) Indian Bank(e) None of these

6. State Bank of India was established on(a) May 2, 1951 (b) June 21, 1952(c) July 1, 1955 (d) August 12, 1956(e) None of these

7. NABARD (National Bank for Agriculture and RuralDevelopment) was established in(a) 1979 (b) 1980(c) 1981 (d) 1982(e) 1985

8. Regional Rural Banks were established on(a) July 3, 1970 (b) April 14, 1971(c) October 2, 1975 (d) November 23, 1978(e) None of these

9. Lead Bank system was started on the recommendations of(a) Raja Challaia Committee(b) Kelkar Committee(c) Nariman Committee(d) Malhotra Committee(e) None of these

10. Lead Bank system was started in(a) 1967 (b) 1968(c) 1969 (d) 1970(e) 1972

11. Reserve Bank of India (RBI) follows the Minimum ReservesSystem in issuing currency since(a) 1950 (b) 1952(c) 1954 (d) 1956(e) 1951

12. Reserve Bank of India started the system of ombudsman toresolve grievances of customers in(a) 1994 (b) 1995(c) 1996 (d) 1997(e) 1998

13. The credit control methods adopted by the Reserve Bank are(a) Quantitative controls(b) Qualitative controls(c) Fixed controls(d) Both (a) and (b)(e) None of these

14. Which of the following Committees were appointed by theGovernment for restructur ing the Regional RuralDevelopment Banks?(a) Bhandar Committee (b) K.Basu Committee(c) Raj Committee (d) Both (a) and ((b)(e) None of these

15. The concept of 'Universal Banking' was implemented in Indiaon the recommendations of:(a) Abid Hussain Committee(b) R H Khan Committee(c) S Padmanabhan Committee(d) Y H Malegam Committee(e) None of these

RESPONSE

GRID

1. a b c d e 2. a b c d e 3. a b c d e 4. a b c d e 5. a b c d e

6. a b c d e 7. a b c d e 8. a b c d e 9. a b c d e 10. a b c d e

11. a b c d e 12. a b c d e 13. a b c d e 14. a b c d e 15. a b c d e

Max. Marks : 30 No. of Qs. 30 Time : 20 min. Date : ........./......../................

8181History of Bankingand its Development

Page 180: yoursmahboob.wordpress.com SBI · yoursmahboob.wordpress.com iii P 101 Speed Tests for SBI Bank Clerk Exam 101 Speed Tests for SBI Bank Clerk Exam is revised and updated edition on

yoursmahboob.w

ordpress.com

172 SPEED TEST 81

16. Oudh Commercial Bank was established in(a) 1878 (b) 1879(c) 1880 (d) 1881(e) None of these

17. Which of the following is the first Private bank establishedbased on the recommendations of the NarasimhanCommittee?(a) UTI Bank Ltd. (b) Union Bank(c) Bank of Baroda (d) Dena Bank(e) None of these

18. In 1993, a Nationalized bank was merged with Punjab NationalBank (PNB), what is the name of that Bank?(a) Bank of Baroda (b) Global Trust Bank(c) New Bank of India (d) Bank of India(e) None of these

19. The biggest commercial bank in India is(a) RBI (b) SBI(c) IDBI (d) Exim Bank of India(e) None of these

20. ICICI is a(a) Financial Institution(b) Rural Development Bank(c) Cooperative Bank(d) Space Research Institute(e) None of these

21. Banking amendment bill reduced SBI holding in its sevensubsidiary banks from(a) 90% to 75%(b) 85% to 50%(c) 75% or more to 51%(d) 65% or more to 49%(e) None of these

22. When was paper currency first started in India?(a) 1810 (b) 1715(c) 1635 (d) 1542(e) 1902

23. Consider the following statements.(1) Scheduled Banks are those Banks which are included

in the Second Scheduled of the Reserve Bank Act, 1934.(2) There are 10 Non-Scheduled commercial Banks

operating in the country.(3) Co-operative banks are organized &managed on the

principle of co-operation, self-help and mutual help.

RESPONSE

GRID

16. a b c d e 17. a b c d e 18. a b c d e 19. a b c d e 20. a b c d e

21. a b c d e 22. a b c d e 23. a b c d e 24. a b c d e 25. a b c d e

26. a b c d e 27. a b c d e 28. a b c d e 29. a b c d e 30. a b c d e

Which of the following are correct?(a) 1 and 3 (b) only 2(c) 2 and 3 (d) only 1(e) All the above are correct.

24. Security printing press was established in 1982 at?(a) Kolkata (b) New Delhi(c) Bombay (d) Hyderabad(e) Nasik

25. Six private sector banks were nationalised on April 15, 1980,whose reserves were more than?(a) 100 Crores (b) 200 crores(c) 300 crores (d) 400 crores(e) 500 crores

26. Scheduled banks are those?(a) Includued in the 2nd schedule of the Banking

Regulation Act-1949(b) Includued in the 2nd schedule of the Companies Act-

1956(c) Includued in the 2nd schedule of the Reserve Bank of

India Act -1934(d) Bank Nationalization Act -1969(e) None of these

27. When RBI has decided to circulate 'Plastic Currency Notes'in the market ?(a) July 1, 1999 (b) July 1, 2010(c) July 1, 2011 (d) Sept. 1, 2011(e) None of these

28. When was addopted, New strategy for Rural Lending :Service Area Approach ?(a) April 1, 1989 (b) March 1, 2007(c) April 1, 2010 (d) April 1, 2011(e) None of these

29. India Brand Equity Fund' was established in the year:(a) 1992 (b) 1998 (c) 1995(d) 1996 (e) 1997

30. Which was the first Indian Bank to introduce credit card?(a) State Bank of India(b) Central Bank of India(c) Union Bank of India(d) ICICI(e) None of these

Page 181: yoursmahboob.wordpress.com SBI · yoursmahboob.wordpress.com iii P 101 Speed Tests for SBI Bank Clerk Exam 101 Speed Tests for SBI Bank Clerk Exam is revised and updated edition on

yoursmahboob.w

ordpress.com

1. The accounting year of RBI occurs between the months of ?(a) April – March (b) March – February(c) July – June (d) August – July(e) None of these

2. In which article RBI permitted to the co-operative Banks forspecial Account Supervision?(a) Art – 30 (b) Art – 31 (c) Art – 33(d) Art – 32 (e) None of these

3. Open market operations of RBI refers to?(a) buying and selling of shares(b) auctioning of foreign exchange(c) trading in securities(d) transactions in gold.(e) None of these

4. Monetary policy in India is formulated and implemented by?(a) Government of India(b) Reserve Bank of India(c) Indian Banks Association(d) FICCI(e) None of these

5. Reserve bank of India follows which system for the issue ofcurrency?(a) Minimum Reserve System(b) Proportionate Reserve System(c) Both of the above(d) None of the above(e) None of these

6. Which of the following controls credit creation by the com-mercial Banks in India?(a) Ministry of Finance (b) Reserve Bank of India(c) Government of India (d) State Bank of India(e) None of these

7. Note issuing department of Reserve Bank of India shouldalways passes the minimum gold stock worth ?

(a) ` 85 crore (b) ` 115 crore(c) ` 200 crore (d) None of above(e) None of these

8. In India which agency is entrusted with the collection ofdata of capital formation ?(a) RBI and CSO (b) RBI and SBI(c) RBI and Other Bank (d) CSO and NSSO(e) None of these

9. The Bank rate is the rate at which ?(a) a bank lends to the public(b) the RBI lends to the public(c) RBI gives credit to the Commercial Banks(d) the Government of India lends to other countries.(e) None of these

10. An increase in CRR by the Reserve Bank of India result in ?(a) decrease in debt of the government(b) reduction in liquidity in the economy(c) attracting more FDI in the country(d) more flow of credit to desired sectors(e) None of these

11. Which of the following provides the largest credit to agri-culture and allied sectors ?(a) Co-operative Banks with RBI(b) Regional Rural Banks(c) Commercial Banks(d) Co-operative and Regional Rural Banks(e) None of these

12. Who published the financial report on currency and finance ?(a) RBI (b) CSO (c) WTO(d) NSSO (e) None of these

13. Who is the custodian of India's foreign exchange Funds?(a) RBI (b) SBI(c) ICICI (d) Central Bank(e) None of these

RESPONSE

GRID

1. a b c d e 2. a b c d e 3. a b c d e 4. a b c d e 5. a b c d e6. a b c d e 7. a b c d e 8. a b c d e 9. a b c d e 10. a b c d e11. a b c d e 12. a b c d e 13. a b c d e

Max. Marks : 25 No. of Qs. 25 Time : 20 min. Date : ........./......../................

8282RBI & itsMonetary Policies

Page 182: yoursmahboob.wordpress.com SBI · yoursmahboob.wordpress.com iii P 101 Speed Tests for SBI Bank Clerk Exam 101 Speed Tests for SBI Bank Clerk Exam is revised and updated edition on

yoursmahboob.w

ordpress.com

174 SPEED TEST 82

14. Which of the following authority sanctions foreign exchangefor the import of goods ?(a) Any Nationalised Bank(b) Exchange Bank

(c) Reserve Bank of India(d) Ministry of Finance

(e) None of these15. How many posts of Deputy Governor in Reserve Bank of

India ?(a) 1 (b) 2 (c) 3(d) 4 (e) None of these

16. Which among the following institutions regulates the exter-nal commercial borrowings ?

(a) SEBI(b) Ministry of Finance

(c) Ministry of Commerce(d) Reserve Bank of India

(e) None of these17. Which of the following statements is not correct ?

(a) RBI is the Central Bank of the country(b) RBI is the Banker of the Central and the state Govern-

ments

(c) RBI is the custodian of the country’s Foreign ExchangeReserve

(d) RBI was established in 1949.(e) None of these

18. RBI has tweaked the rules for lending rates and stipulatedthat fixed rate loans of up to how many years will be offeredby lenders based on marginal cost of funding?(a) 2 (b) 3(c) 4 (d) 5(e) None of these

19. RBI has relaxed norms for which category of banks?(a) Banks part of the cleaning up exercises of power utilities(b) Banks holding debt ridden State Electricity Boards

bonds

RESPONSE

GRID

14. a b c d e 15. a b c d e 16. a b c d e 17. a b c d e 18. a b c d e19. a b c d e 20. a b c d e 21. a b c d e 22. a b c d e 23. a b c d e24. a b c d e 25. a b c d e

(c) Banks holding debt ridden Country Electricity Boardsbonds

(d) Both a and b(e) Neither (a) and (b)

20. Which of the following are reasons for adverse impact oncard use in India?(1) Less ATMs(2) Less number of cards(3) Less Point of Sales terminals(a) 1, 2 (b) 1, 3(c) 2 (d) All of the above(e) None of these

21. RBI will purchase back indexed bonds maturing in whichyear?(a) 2022 (b) 2023(c) 2024 (d) 2025(e) None of these

22. Which of the following statements is/are correct aboutDeputy Governor of RBI?1. The tenure of Deputy Governor is five years or till the

age of 62, whichever is earlier.2. RBI has provision for three deputy governors.3. RBI has provision for four deputy governors.(a) Only 1 (b) Only 2(c) Only 3 (d) Both 1 and 2(e) None of these

23. Urijit Patel, Deputy Governor of RBI has been reappointedfor 3 years. How many deputy governors of RBI are there?(a) 3 (b) 4(c) 5 (d) 6(e) None of these

24. Since when has the Reserve Bank of India been successfullyoperating the instrument of selective credit control in thiscountry?(a) 1939 (b) 1951(c) 1956 (d) 1961(e) None of these

25. India is a member of the International Monetary Fund since_____.(a) 1934 (b) 1935(c) 1947 (d) 1949(e) None of these

Page 183: yoursmahboob.wordpress.com SBI · yoursmahboob.wordpress.com iii P 101 Speed Tests for SBI Bank Clerk Exam 101 Speed Tests for SBI Bank Clerk Exam is revised and updated edition on

yoursmahboob.w

ordpress.com

1. Which of the following is not the part of the ScheduledBanking structure in India?(a) Money Lenders(b) Public Sector Banks(c) Private Sector Banks(d) Regional Rural Banks(e) State Cooperative Banks

2. Small Savings Scheme like national savings certificates,Public Provident Fund, Monthly Income Schemes are popularamong the salaried people. Which financial institutionsmanage these schemes?(a) Public Sector Banks (b) Commercial Banks(c) Post Offices (d) Co-operative Banks(e) None of these

3. Which of the following is/are the right(s) of customertowards his banker?(a) To receive a statement of his account from a banker(b) To sue the bank for any loss and damages(c) To sue the banker for not maintaining the secrecy of

his account(d) All of the above(e) None of these

4. Which of the following factors is not required to beconsidered to analyze the repayment capacity of a borrower?(a) Working capital management(b) Personal educational qualifications(c) Financial leverage(d) Interest rate risk management(e) None of these

5. Which of the following is a facilitating service of core loanproducts of retail banking services?(a) Current or savings accounts(b) Legal services for documentation(c) Delivery of loan at promised time period(d) Flexibility in prepayment of loan(e) None of these

6. Funded Services under corporate banking does not include?(a) Working Capital Finance(b) Bill Discounting(c) Export Credit(d) Letters of Credit(e) None of these

7. ''Swabhiman'' Scheme is related-(a) Rich Customers of the Bank(b) RRBs(c) To provide basic banking services to bankless villages(d) Both (b) and (c)(e) None of these

8. A centralized database with online connectivity to branches,internet as well as ATM-network which has been adoptedby almost all major banks of our country is known as?(a) Investment Banking(b) Core Banking(c) Mobile Banking(d) National Banking(e) Specialized Banking

9. The Reverse Mortgage scheme is launched to give benefitto which of the following groups of society?(a) Persons below 60 yrs(b) Senior Citizens(c) Unemployed youth(d) Orphans(e) All of the above

10. Which of the following scheme is not meant for investmentpurposes?(a) National saving certificate(b) Infrastructure bonds(c) Mutual funds(d) Letter of credit(e) None of these

RESPONSE

GRID

1. a b c d e 2. a b c d e 3. a b c d e 4. a b c d e 5. a b c d e6. a b c d e 7. a b c d e 8. a b c d e 9. a b c d e 10. a b c d e

Max. Marks : 21 No. of Qs. 21 Time : 15 min. Date : ........./......../................

8383Banking Productand Services

Page 184: yoursmahboob.wordpress.com SBI · yoursmahboob.wordpress.com iii P 101 Speed Tests for SBI Bank Clerk Exam 101 Speed Tests for SBI Bank Clerk Exam is revised and updated edition on

yoursmahboob.w

ordpress.com

176 SPEED TEST 83

11. Systematic investment Plans relates to:(a) Mutual Funds(b) Life Insurance Companies(c) Commercial Banks(d) Post office savings schemes(e) None of these

12. Which of the following is an example of cash less purchase?(a) Debit card (b) Credit card(c) ATM withdrawal (d) All of the above(e) None of these

13. Which one of the following is not an electronic bankingdelivery channel?(a) Mobile Vans(d) Mobile Phone Banking(c) Internet Banking(d) Tele Banking(e) ATM

14. Now-a-days Banks are selling third party products. Exampleof third party product is:(a) Mutual funds(b) Term deposits(c) Credit cards(d) All of these(e) None of these

15. What are White Label ATMs(a) ATMs set up and run by non-banking entities(b) ATMs set up and run by banking entities(c) ATMs in rural areas(d) ATMs in Defence areas(e) ATM set up in Uttaranchal

RESPONSE

GRID

11. a b c d e 12. a b c d e 13. a b c d e 14. a b c d e 15. a b c d e16. a b c d e 17. a b c d e 18. a b c d e 19. a b c d e 20. a b c d e21. a b c d e

16. If a bank ties up with a retail vendor and then both of themsponsor a credit card, then such card would be known as __:(a) Retail Credit Card (b) Vendor Card(c) Co-branded Card (d) Cash back Card(e) None of these

17. In terms of Banking Terminology, Affinity Card refers to __:(a) The Credit Cards that are linked to special organizations(b) The Credit Cards with zero interest rate on repayments(c) The Credit Cards exclusively for the Bank's employees(d) All of above(e) None of these

18. From which country, the concept of Credit Card originated?(a) United Kingdom (b) United States(c) France (d) Australia(e) None of these

19. Which is the first credit card facility to be recognisedworldwide?(a) Visa card (b) Maestro card(c) MasterCard (d) Diner Card(e) None of these

20. In terms of Credit Cards, what is a Hot Card?(a) Newly issued Card (b) Invalid Card(c) Stolen Card (d) Unpaid Card Hide(e) None of these

21. What does the cirrus logo on ATM / debit cards signifies__?(a) have cash access facility anywhere in India only(b) have cash access facility outside the India only(c) have cash access facility in or outside the India(d) have cash access facility in and outside the India(e) None of these

Page 185: yoursmahboob.wordpress.com SBI · yoursmahboob.wordpress.com iii P 101 Speed Tests for SBI Bank Clerk Exam 101 Speed Tests for SBI Bank Clerk Exam is revised and updated edition on

yoursmahboob.w

ordpress.com

1. What is the full form of ‘FINO’, a term we see frequently infinancial newspapers?(a) Financial Investment Network and Operations(b) Farmers’ Investment in National Organisation(c) Farmers Inclusion News and Operations(d) Financial Inclusion Network and Operations(e) None of these

2. What does the letter 'L' denote in the term ‘LAF’ as referredevery now and then in relation to monetary policy of theRBI?(a) Liquidity (b) Liability(c) Leveraged (d) Longitudinal(e) Linear

3. Which of the following terms is NOT a financial term?(a) Investment(b) El Nino effect(c) Core banking Solution(d) RTGS(e) All are financial terms

4. The term 'Smart Money" refers to __________ .(a) Foreign Currency (b) Internet Banking(c) US Dollars (d) Travelers' cheques(e) Credit Cards

5. We often come across the term SWIFT in financialnewspapers. What is the expanded form of this term?(a) Society for Worldwide Interbank Financial

Telecommunication(b) Secure Worldwide Interbank Financial Telecommunica-

tion.(c) Society for Worldwide Intra-bank Financial

Transaction.(d) Security for Worldwide Interbank Financial

Transaction(e) None of these

6. Many a time we read in financial newspapers about theperformance of the “core sectors” in the economy. Whichof the following is NOT included in the same?(a) Coal (b) Automobiles(c) Steel (d) Cement(e) Oil & Petroleum

7. The Standing Committee on Finance headed by YashwantSinha has recommended that CSR should be mandatory forall the companies. CSR stands for

(a) Company Social Representation(b) Company Service Responsibility(c) Corporate Social Responsibility(d) Corporate Security Responsibility(e) None of these

8. In a bid to standardise and enhance the security features incheque forms, it has now been made mandatory for banksto issue new format of cheques called(a) CTS-2010 (b) CTS-2011(c) CTS-2012 (d) CTS-2013(e) None of these

9. The “Four Eyes” principle (mentioned by the Reserve Bankof India) refers to:(a) Lenders (b) Borrowers(c) Wealth Managers (d) Micro-Fananciers(e) None of these

10. NRE deposit is(a) Non Resident External deposit(b) Non Resident Extra deposit(c) Non Resident Exchange deposit(d) Non Refundable External deposit(e) Non Resident Extended deposit

11. Which of the following is NOT a banking-related term?(a) SME Finance (b) Overdraft(c) Drawing power (d) Sanctioning Authority(e) Equinox

12. What does the acronym LAF stand for?(a) Liquidity Adjustment Fund(b) Liquidity Adjustment Facility(c) Liquidity Adjustment Finance(d) Liquidity Adjustment Factor(e) None of these

13. Which of the following terms is used in Banking Field?(a) Interest rate swap (b) Input devices(c) Sedimentary (d) Zero hour(e) Privilege motion

14. What is “wholesale banking”?(a) It is a bank-to-bank or B2B dealing.(b) It is a bank-to-customer dealing.(c) It is a bank-to-trustworthy customer dealing.(d) It is a bank-to-government dealing(e) None of these

RESPONSE

GRID

1. a b c d e 2. a b c d e 3. a b c d e 4. a b c d e 5. a b c d e6. a b c d e 7. a b c d e 8. a b c d e 9. a b c d e 10. a b c d e11. a b c d e 12. a b c d e 13. a b c d e 14. a b c d e

Max. Marks : 30 No. of Qs. 30 Time : 20 min. Date : ........./......../................

8484Banking Term/Terminology

Page 186: yoursmahboob.wordpress.com SBI · yoursmahboob.wordpress.com iii P 101 Speed Tests for SBI Bank Clerk Exam 101 Speed Tests for SBI Bank Clerk Exam is revised and updated edition on

yoursmahboob.w

ordpress.com

178 SPEED TEST 8415. Trade between India and China is in a state of “Payment

imbalance”. What does this mean in real terms?(1) China imports less from India but India imports more

from China.(2) China delays payments to exporters.(3) India wants payments in US Dollars but wants to pay

in Yuan.(a) Only 1 (b) Only 2(c) Only 3 (d) All 1, 2 and 3(e) None of these

16. Banks are promoting “Branch less Banking” which means?(1) Banks will not reduce number of branches. Number of

branches will be restricted and will concentrate onspecified core business.

(2) Banks will launch/operate multiple delivery channelslike ATMs, Mobile Banking/Internet Banking etc makingvisit to a branch unnecessary.

(3) Banks will issue only debit or credit cards for dailyfinancial transactions. Cheques/Cash payment will notbe allowed.

(a) Only 1 (b) Only 2(c) Only 1 and 2 (d) Only 2 and 3(e) All 1, 2 and 3

17. NBFCs are an important part of the Indian financial system.What is the full form of this term?(a) New Banking Financial Companies(b) Non-Banking Financial Companies(c) Neo Banking Financial Confederation(d) Non-Banking Fiscal Companies(e) All of these

18. BCSBI stands for(a) Banking Codes and Standards Board of India(b) Banking Credit and Standards Board of India(c) Banking Codes and Service Board of Inida.(d) Banking Credit and Service Board of India.(e) None of these

19. What does the term ‘bancassurance’ mean ?(a) Assurance from the bank to its account holder

regarding safety of his money(b) A special product designed by the bank(c) Selling of insurance policies by banks(d) Understanding between banks and insurance

companies(e) None of these

20. SEZ stands for:(a) Southern Economic Zone(b) South European zone(c) Special Economic Zone(d) Special Eastern Zone(e) None of these

21. The abbreviation IRDA stands for:(a) Industrial Research and Development Authority of

India(b) Insurance Research and Development Authority of

India(c) Insurance Regulation Development Authority of India

(d) Industrial Research and Demands Agency(e) None of these

22. Hard Currency is defined as currency :(a) which can hardly be used for international transactions(b) which is used in times of war(c) which loses its value very fast(d) traded in foreign exchange market for which demand

is persistently relative to the supply(e) None of these

23. The terms “bull” and “bear” are used in the :(a) Bihar Government’s Animal Husbandry Department(b) Income Tax Department(c) CBI(d) Stock Exchange(e) None of these

24. We read a term 'ECB' in the financial newspapers. What isthe full form of ECB?(a) Essential Credit and Borrowing(b) Essential Commercial Borrowing(c) External Credit and Business(d) External Commercial Borrowing(e) None of these

25. Green Banking means(a) financing of irrigation projects by banks(b) development of forestry by banks(c) financing of environment friendly projects by banks(d) development of Railway by banks.(e) None of these

26. DTAA stands for(a) Direct Tariff Avoidance Agreement(b) Double Taxation Avoidance Agreement(c) Direct Taxation Avoidance Agreement(d) Double Tariff Avoidance Agreement(e) None of these

27. The rate at which the Reserve Bank of India lends to thecommercial banks in very short term against the backing ofthe Government securities is known as?(a) Bank rate (b) Repo rate(c) Reverse Repo (d) Discount rate(e) None of these

28. A letter of credit (L(C) wherein the credit available to thecustomer gets reinstated after the bill is paid is known as?(a) Back to back LC (b) Red clause LC(c) Back to front LC (d) Revolving LC(e) None of these

29. Which of the following is not an imperfect note?(a) Washed note (b) Bleached note(c) Mutilated note (d) Oiled note(e) None of these

30. Which of the following is not shown as an asset in thebalance sheet of a BanK?(a) Investment (b) Advances(c) Cash Balances with other banks(d) Borrowings(e) None of these

RESPONSE

GRID

15. a b c d e 16. a b c d e 17. a b c d e 18. a b c d e 19. a b c d e20. a b c d e 21. a b c d e 22. a b c d e 23. a b c d e 24. a b c d e25. a b c d e 26. a b c d e 27. a b c d e 28. a b c d e 29. a b c d e30. a b c d e

Page 187: yoursmahboob.wordpress.com SBI · yoursmahboob.wordpress.com iii P 101 Speed Tests for SBI Bank Clerk Exam 101 Speed Tests for SBI Bank Clerk Exam is revised and updated edition on

yoursmahboob.w

ordpress.com

1. Which sector of Indian economy contributes longest to GrossNational Product ?(a) Primary sector (b) Secondary sector(c) Tertiary sector (d) Public sector(e) None of these

2. The main source of National Income in India is-(a) Service sector (b) Agriculture(c) Industrial sector (d) Trade sector(e) None of these

3. Which one of the following is not a tax levied by thegovernment of India ?(a) Service tax (b) Education(c) Custom duty (d) Toll tax(e) None of these

4. The most appropriate measure of a country’s economicgrowth is its-(a) Gross Domestic Product (GDP)(b) Net Domestic Product (NDP)(c) Net National Product (NNP)(d) Per Capita Product (PCP)(e) None of these

5. FEMA (Foreign Exchange Management (Act) was finallyimplemented in the year(a) 1991 (b) 1997(c) 2002 (d) 2007(e) None of these

6. The most common measure of estimating inflation in India is-(a) Price Index(b) Wholesale Price Index(c) Consumer Price Index(d) Price Index of Industrial Goods(e) None of these

7. The National Income of India is estimated by-(a) National Sample Survey Organization(b) Ministry of Finance(c) Reserve Bank of India(d) Central Statistical Organization(e) None of these

8. In India, which one among the following formulates the fiscalpolicy?(a) Planning Commission(b) Finance Commission

(c) Finance Ministry(d) Reserve Bank of India(e) None of these

9. In which of the following financial years the devaluation ofrupee in India took place twice ?(a) 1966-67 (b) 1991-92(c) 1990-91 (d) 1989-90(e) None of these

10. VAT is imposed-(a) On first stage of production(b) Directly on consumer(c) On all stages between production and final sale(d) On final stage of production(e) None of these

11. Term Balance of payment is used in relation to which of thefollowing ?(a) Annual sale of a factory(b) Tax collection(c) Exports and imports(d) None of the above(e) None of these

12. The Indian economy can be described as(a) a backward and stagnant economy(b) a developing economy(c) an underdeveloped economy(d) a developed economy(e) None of these

13. Loans to poor people by banks have many limitationsincluding lack of security and high operating cost. So tohelp them which type of finance system developed ?(a) Ponzi schemes(b) Micro Finance System(c) Money Laundering Schemes(d) Money tampering finance(e) None of these

14. The beneficiaries of Micro finance business are _______.(a) Land Less labour(b) Marginal farmers(c) Vendors in the small markets(d) All the above(e) None of these

RESPONSE

GRID

1. a b c d e 2. a b c d e 3. a b c d e 4. a b c d e 5. a b c d e6. a b c d e 7. a b c d e 8. a b c d e 9. a b c d e 10. a b c d e11. a b c d e 12. a b c d e 13. a b c d e 14. a b c d e

Max. Marks : 30 No. of Qs. 30 Time : 20 min. Date : ........./......../................

8585Micro Finance andEconomics

Page 188: yoursmahboob.wordpress.com SBI · yoursmahboob.wordpress.com iii P 101 Speed Tests for SBI Bank Clerk Exam 101 Speed Tests for SBI Bank Clerk Exam is revised and updated edition on

yoursmahboob.w

ordpress.com

180 SPEED TEST 8515. One of the delivery channel for Micro Finance is SHG model.

SHG means ?(a) Soar Help Group (b) Sake Help Group(c) Self Hope Group (d) Self Help Group(e) None of these

16. In the Not-For-Profit Micro Finance Institutes, which amongthe following are included ?(a) Societies(b) Public Trusts(c) Non-Profit Companies(d) All of these(e) None of these

17. Non-banking financial companies, producer companies andLAB come under the category of For-Profit-MFIs. LABmeans ?(a) Loan Area Banks (b) Legal Area Banks(c) Local Axis Banks (d) Local Area Banks(e) None of these

18. On which among the following dates FERA was replaced byFEMA?(a) June 1, 2000 (b) June 11, 2000(c) April 1, 2001 (d) April 1, 2002(e) None of these

19. What is the minimum net owned funds (NOF) mandatoryfor a Infrastructure Finance Company (IFC) in India?(a) ` 200 Crore (b) ` 300 Crore(c) ` 500 Crore (d) ` 100 Crore(e) None of these

20. In India, Infrastructure Debt Fund can be established as aTrust or a company. Which of the following regulated theInfrastructure Debt Fund set up as a trust?(a) SEBI (b) RBI(c) IrDA (d) Ministry of Corporate Affairs(e) None of these

21. What is the cap on loan amount given out by a Non-BankingFinancial Company - Micro Finance Institution (NBFC-MFI)?(a) ` 50000 (b) ` 60000(c) ` 70000 (d) ` 75000(e) None of these

22. Which of the following is are the parties in FactoringBusiness?(a) buyer(b) seller(c) buyer and seller(d) buyer, seller and financial institution(e) None of these

23. What is the minimum tenure of deposits to be taken byNBFCs?(A) 6 months (b) 12 months(c) 2 years (d) 3 years(e) None of these

24. Which among the following is considered to be the mostliquid asset?(a) Gold (b) Money(c) Land (d) Treasury bonds(e) None of these

25. Who is author of the ancient book on economics,Arthashastra?(a) Kautilya (b) Chanakya(c) Sushrut (d) Bhattacharya(e) None of these

26. Currency notes and coins are called as:(a) Flat money (b) Legal tenders(c) Fiat money (d) Both b and c(e) None of these

27. What is the currency deposit ratio (cdr)?(a) ratio of money held by the public in currency to that of

money held in bank deposits(b) ratio of money held by public in bank deposits to that

of money held by public in currency(c) ratio of money held in demand drafts to that of money

held in treasury bonds(d) ratio of money held in demand drafts to that of money

held in mutual bonds(e) None of these

28. What is the reserve deposit ratio (rdr)?(a) the proportion of money RBI lends to commercial banks(b) the proportion of total deposits commercial banks keep

as reserves(c) the total proportion of money that commercial banks

lend to the customers(d) the total proportion of money that commercial banks

lend to the money RBI tends(e) None of these

29. What is the Cash Reserve Ratio (CRR)?(a) the fraction of the deposits that commercial banks lendto the customers(b) the fraction of the deposits that RBI must keep withcommercial banks(c) the fraction of the deposits that commercial banks mustkeep with RBI(d) the fraction of the deposits that private banks mustkeep with RBI(e) None of these

30. In monetary terminology, what is called the 'monetary base'or 'high powered money'?(a) the total assets of RBI(b) the total liability of RBI(c) the total debt of the government(d) the total foreign exchange of RBI(e) None of these

RESPONSE

GRID

15. a b c d e 16. a b c d e 17. a b c d e 18. a b c d e 19. a b c d e20. a b c d e 21. a b c d e 22. a b c d e 23. a b c d e 24. a b c d e25. a b c d e 26. a b c d e 27. a b c d e28. a b c d e 29. a b c d e 30. a b c d e

Page 189: yoursmahboob.wordpress.com SBI · yoursmahboob.wordpress.com iii P 101 Speed Tests for SBI Bank Clerk Exam 101 Speed Tests for SBI Bank Clerk Exam is revised and updated edition on

yoursmahboob.w

ordpress.com

1. ‘Eco mark’ is given to the Indian products that are ?(a) pure and unadulterated(b) rich in proteins(c) Environment Friendly(d) Economically viable(e) None of these

2. The earlier name of WTO was ?(a) UNCTAD (b) GATT(c) UNIDO (d) OECD(e) None of these

3. ‘World Development Report’ is an annual publication of ?(a) UNDP (b) IBRD(c) WTO (d) IMF(e) None of these

4. India has the maximum volume of foreign trade with ?(a) USA (b) Japan(c) Germany (d) UAE(e) None of these

5. Participatory notes (PNs) are associated with which one ofthe followings ?(a) Consolidated food(b) Foreign Institutional Investors(c) UNDP(d) Kyoto protocal(e) None of these

6. What is the purpose of India Brand-Equity Fund ?(a) To promote in bound tourism(b) To make ‘Made in India’ a label of quality(c) To organise trade fairs(d) To provide venture capitals to IT sector(e) None of these

7. A trade policy consists of :(a) Export-Import policy(b) Licencing policy(c) Dumping(d) Double pricing(e) None of these

8. FERA in India has been replaced by ?(a) FEPA (b) FEMA(c) FENA (d) FETA(e) None of these

9. TRIPS and TRIMS are the term associated with ?(a) IMF (b) WTO(c) IBRD (d) IDA(e) None of these

10. SEZ act was passed by the parliament in the year ?(a) 2004 (b) 2005(c) 2006 (d) 2007(e) None of these

11. How many members recently in WTO ?(a) 158 (b) 159(c) 160 (d) 162(e) None of these

12. Which of the following is a part of capital account?(a) Private capital (b) Banking capital(c) Official capital (d) All the above(e) None of these

13. The investment in productive assets and participation inmanagement as stake holders in business enterprises is(a) FDI(b) FII(c) Balance of payment(d) SDR(e) None of these

14. The portfolio investment by foreign institutional investorsis called(a) FDI(b) FII(c) Balance of payment(d) SDR(e) None of these

15. Which of the following is international trade:(a) Trade between provinces(b) Trade between regions(c) Trade between countries(d) (b) and (c) of above(e) None of these

RESPONSE

GRID

1. a b c d e 2. a b c d e 3. a b c d e 4. a b c d e 5. a b c d e6. a b c d e 7. a b c d e 8. a b c d e 9. a b c d e 10. a b c d e11. a b c d e 12. a b c d e 13. a b c d e 14. a b c d e 15. a b c d e

Max. Marks : 30 No. of Qs. 30 Time : 20 min. Date : ........./......../................

8686Foreign Trade

Page 190: yoursmahboob.wordpress.com SBI · yoursmahboob.wordpress.com iii P 101 Speed Tests for SBI Bank Clerk Exam 101 Speed Tests for SBI Bank Clerk Exam is revised and updated edition on

yoursmahboob.w

ordpress.com

182 SPEED TEST 86

16. Which is NOT an advantage of international trade:(a) Export of surplus production(b) Import of defence material(c) Dependence on foreign countries(d) Availability of cheap raw materials(e) None of these

17. Trade between two countries can be useful if cost ratios ofgoods are:(a) Equal (b) Different(c) Undetermined (d) Decreasing(e) None of these

18. Foreign trade creates among countries:(a) Conflicts (b) Cooperation(c) Hatred (d) Both (a) & (b)(e) None of these

19. All are advantages of foreign trade EXCEPT:(a) People get foreign exchange(b) Nations compete(c) Cheaper goods(d) Optimum utilisation of country's resources(e) None of these

20. Govt. policy about exports and imports is called:(a) Monetary policy(b) Fiscal policy(c) Commercial policy(d) Finance policy(e) None of these

21. What would encourage trade between two countries:(a) Different tax system(b) Frontier checks(c) National currencies(d) Reduced tariffs(e) None of these

22. Foreign trade:(a) Increases employment opportunities(b) Increases international mobility of labour(c) Increases competition(d) All of the above(e) None of these

23. Special Economic Zones are :(a) situated outside India, but subject to RBI control(b) treated as foreign territory and not Indian laws(c) governed by international and not Indian laws(d) prohibited from buying from Domestic Tariff Area(e) None of these

24. The Imports and Exports (Control) Act came into enforcementfrom(a) 1947 (b) 1950.(c) 1951 (d) 1955(e) None of these

25. The IEC number is issued by the(a) Central Government(b) State Government(c) Director General of Foreign trade(d) Ministry of commerce(e) None of these

26. The IEC number is a number with(a) 7 digits (b) 8 digits(c) 9 digits (d) 10 digits(e) None of these

27. The Director General of Foreign trade is appointed by(a) Central Government(b) State Government(c) Ministry of commerce(d) Chief justice of the Supreme Court(e) None of these

28. The foreign Trade (Regulation) Rules was passed in theyear(a) 1991 (b) 1992(c) 1993 (d) 1994(e) None of these

29. The apex body of the Foreign Trade is(a) The Central Government(b) The State Government(c) The Ministry of Commerce(d) All the above(e) None of these

30. The tenure of the Foreign Trade policy is(a) 3 years (b) 5 years(c) 1 year (d) 7 years(e) None of these

RESPONSE

GRID

16. a b c d e 17. a b c d e 18. a b c d e 19. a b c d e 20. a b c d e21. a b c d e 22. a b c d e 23. a b c d e 24. a b c d e 25. a b c d e26. a b c d e 27. a b c d e 28. a b c d e 29. a b c d e 30. a b c d e

Page 191: yoursmahboob.wordpress.com SBI · yoursmahboob.wordpress.com iii P 101 Speed Tests for SBI Bank Clerk Exam 101 Speed Tests for SBI Bank Clerk Exam is revised and updated edition on

yoursmahboob.w

ordpress.com

1. Swabhiman scheme launched in India is associated with(a) Rural women rights(b) Rural old people care(c) Rural banking(d) Rural food security(e) None of these

2. In which one of the following years the unorganised workerssocial security Act was passed ?(a) 2004 (b) 2006(c) 2008 (d) 2010(e) None of these

3. Swadhar is the scheme for(a) unique identification(b) self employment for males(c) women in difficult circumstances(d) common home of senior citizens.(e) None of these

4. The aim of Pradhan Mantri Gramodaya Yojana is(a) Meeting rural needs like primary education, health care,

drinking water, housing, rural roads.(b) Alleviating poverty through microenterprises.(c) Generating employment in rural areas.(d) Strengthening Panchayati Raj system in rural areas.(e) None of these

5. Twenty Point Economic Programme was first launched inthe year(a) 1969 (b) 1975(c) 1977 (d) 1980(e) None of these

6. In India disguised unemployment is a prominent featuremainly of(a) Primary sector (b) Secondary sector(c) Tertiary sector (d) Social sector(e) None of these

7. Golden Quadrangle project is associated with thedevelopment of(a) Highways (b) Ports(c) Power Grids (d) Tourism Network(e) None of these

8. Which one of the following is the objective of National FoodSecurity Mission ?(a) To increase production of rice(b) To increase production of wheat(c) To increase production of pulses(d) All the above(e) None of these

9. Which one of the following durations is related to XII FiveYear Plan in India ?(a) 2007-12 (b) 2005-10(c) 2012-17 (d) 2010-15(e) None of these

10. Which of the following is not a measure of reducinginequalities ?(a) Minimum needs programme(b) Liberalization of economy(c) Taxation(d) Land reforms(e) None of these

11. Which one of the following scheme subsumed the ValmikiAwas Yojana ?(a) Integrated Housing and slum development Programme.(b) Sampoorna Grameen Swarozgar Yojana(c) Rajiv Awas Yojana(d) Integrated Rural development Programme(e) None of these

12. Poverty level in India is established on the basis of-(a) Per capita income in different states(b) House hold average income(c) House hold consumer expenditure(d) Slum population in the country.(e) None of these

13. Nirmal Bharat Abhiyan Yojana is associated with-(a) Development of villages(b) Community toilets in slum areas(c) Construction of house far low income groups(d) Development of roads(e) None of these

14. Crop Insurance Scheme in India was started in(a) 1945 (b) 1980(c) 1985 (d) 1988(e) None of these

RESPONSE

GRID

1. a b c d e 2. a b c d e 3. a b c d e 4. a b c d e 5. a b c d e6. a b c d e 7. a b c d e 8. a b c d e 9. a b c d e 10. a b c d e11. a b c d e 12. a b c d e 13. a b c d e 14. a b c d e

Max. Marks : 25 No. of Qs. 25 Time : 20 min. Date : ........./......../................

8787Socio-Eco-PoliticalEnvironment of India

Page 192: yoursmahboob.wordpress.com SBI · yoursmahboob.wordpress.com iii P 101 Speed Tests for SBI Bank Clerk Exam 101 Speed Tests for SBI Bank Clerk Exam is revised and updated edition on

yoursmahboob.w

ordpress.com

184 SPEED TEST 8715. Which of the following scheme is not for rural development ?

(a) PMGSY (b) SGSY(c) RGGVY (d) SJSRY(e) None of these

16. Who is the chairman of 21st law commission ?(a) Justice A.R. Lakshmanan(b) Justice P.V. Reddy(c) Justice D.K. Jain(d) Justice Balbir Singh Chauhan(e) None of these

17. According to the Integrated child development serviceswhich age group of children are focused ?(a) upto 4 years (b) upto 5 years(c) upto 6 years (d) upto 8 years(e) None of these

18. Which age group of women are eligible for Indira Gandhiwidow Pension Scheme ?(a) 40-50 (b) 50-60(c) 40-59 (d) 40-79(e) None of these

19. Under which project India is developing guided missiledestroyers?(a) Project 20B (b) Project 15B(c) Project 75B (d) Project 40A(e) None of these

20. E-Samiksha is an online project monitoring system launchedby ____?(a) Indian Railways (b) NHAI(c) RBI (d) FIPB(e) None of these

RESPONSE

GRID

15. a b c d e 16. a b c d e 17. a b c d e 18. a b c d e 19. a b c d e

20. a b c d e 21. a b c d e 22. a b c d e 23. a b c d e 24. a b c d e

25. a b c d e

21. India is implementing "Capacity Building for IndustrialPollution Management (CBIPM)" project with the supportof __?(a) Asian Development Bank(b) Japan Central Bank(c) World Bank(d) Reserve Bank of India(e) None of these

22. Who is the chairman of the Panel set up to look into variouscontentious issues relating to inter-linking of rivers?(a) B N Navalawala (b) Hardip Singh Puri(c) Prodipto Ghosh (d) Gopalakrishnan(e) None of these

23. Who has been appointed as new Chief ElectionCommissioner of India?(a) Nasim Zaidi (b) Sayed Nazim(c) H Hari Shankar (d) V S Chikkamata(e) None of these

24. What is the name of the new scheme to empower and enableminority youths, announcement for which was made in thebudget?(a) New Udaan (b) Nayi Soch(c) Nayi Umang (d) Nayi Manzil(e) None of these

25. Union Human Resource and Development (HRD) MinisterSmriti Irani introduced the logo winner of for the NewEducation Policy (NEP) who emerged as the winner in thecompetition was held through MyGov Platform. What is hisname?(a) Nawab Shaikh (b) Ali Ahmed(c) Nawaj Shaikh (d) Mansood Pataudi(e) None of these

Page 193: yoursmahboob.wordpress.com SBI · yoursmahboob.wordpress.com iii P 101 Speed Tests for SBI Bank Clerk Exam 101 Speed Tests for SBI Bank Clerk Exam is revised and updated edition on

yoursmahboob.w

ordpress.com

1. Who among the following has been appointed as permanentrepresentative of India to UNESCO?(a) Ruchir Kamboj(b) V S Oberoi(c) Vijay Keshav Ghokhale(d) Sudeer Kulkarni(e) None of these

2. Who among the followings is the chairman of “NationalCommission for Backward Classes”?(a) Justice V Eswaraih(b) Justice M N Rao(c) Justice Narendra Babu(d) Justice Gopala Gowda(e) None of these

3. Who among the followings is the Chairperson of the NationalCommission for Minorities?(a) K N Daruwall(b) Tsering Namgyal Shanoo(c) Wajahat Habibullah(d) Ajaib Singh(e) Shri Naseem Ahmed

4. Who among the following is the current Chairperson of“National Green Tribunal” of India?(a) Justice Jyothimani(b) Justice Kingaonkar(c) Justice Swatanter Kumar(d) Justice U D Salvi(e) None of these

5. Who among the following is the Chairman of “NationalGanga River Basin Authority”?(a) President (b) Prime Minister(c) Vice-president (d) Home Minister(e) None of these

6. Who among the followings has been appointed as ChiefInformation Commissioner?(a) Mrs Deepak Sandhu(b) Mrs Sushma Singh(c) Rajiv Mathur(d) Vijay Sharma(e) None of these

7. Who among the following has been appointed asChairperson of National School of Drama?(a) Girish Carnad (b) Ratan Thiyam(c) U R Anantha Murthy(d) Anupam Khair(e) None of these

8. Who is appointed as the chairman of The Association ofMutual Funds in India (AMFI).(a) Sundeep Sikka (b) Sandesh Kirkire(c) Pawan Kumar (d) S. M. Patel(e) Leo Puri

9. Who took charge as the new commissioner of Delhi Policeon February 2016?(a) BK Gupta (b) Neeraj Kumar(c) Bhim Sain Bassi (d) G.V. Prakash Kumar(e) Alok Kumar Verma

10. Who is recently appointed as the Chief Justice of MeghalayaHigh Court?(a) M. Subhashini(b) Kamal Ganzouri(c) Prafulla Chandra Pant(d) P. Sathasivam(e) T.Nand kumar Singh

11. Who among the following is the CEO of PepsiCo Asia,Middle East and Africa region?(a) Abhinav Gupta (b) Sanjay Singh(c) Sanjeev Chadha (d) Nanda Kishore(e) Indira Nooyi

12. Who is appointed as the Vice Chairman of NITI Aayog?(a) Jagdish Bhagwati (b) T. N. Srinivasan(c) Arvind Panagariya (d) Montek Singh Ahluwalia(e) None of these

13. Who replaced Sujatha Singh as India's Foreign Secretary(a) Nirupama Rao (b) Ranjan Mathai(c) Ajit Doval (d) S Jaishankar(e) None of these

14. _____ has been appointed as an Election Commissioner ofIndia ?(a) Anil Swarup (b) C.R. Viswanath(c) Achal Kumar Jyoti (d) Ashok Kumar Gupta(e) None of These

RESPONSE

GRID

1. a b c d e 2. a b c d e 3. a b c d e 4. a b c d e 5. a b c d e

6. a b c d e 7. a b c d e 8. a b c d e 9. a b c d e 10. a b c d e

11. a b c d e 12. a b c d e 13. a b c d e 14. a b c d e

Max. Marks : 30 No. of Qs. 30 Time : 20 min. Date : ........./......../................

8888Appointment/Election/Resignation

Page 194: yoursmahboob.wordpress.com SBI · yoursmahboob.wordpress.com iii P 101 Speed Tests for SBI Bank Clerk Exam 101 Speed Tests for SBI Bank Clerk Exam is revised and updated edition on

yoursmahboob.w

ordpress.com

186 SPEED TEST 8815. Who is the incumbent Managing Director of the

International Monetary Fund (IMF)?(a) Christine Lagarde (b) Dominique Strauss-Kahn(c) Ban Ki-moon (d) Jim Yong Kim(e) None of these

16. Who is the incumbent chairman of the National Bank forAgriculture and Rural Development (NABARD)?(a) Atulesh Jindal(b) Satynarayan Mohanty(c) Harsh Kumar Bhanwala(d) Upendra Kumar Sinha(e) None of these

17. Who has been appointed as an Asia-Pacific member of theInternational Coordinating Committee Bureau of NHRI-ICC?(a) Arun Jaitley (b) HL Dattu(c) R M Lodha (d) H L Narayanaswamy(e) None of these

18. Who has been appointed as the advisor to the Khadi andVillage Industries Commission (KVIC) for promotion of khadiwithin India and globally?(a) Payal Singhal (b) Ritu Beri(c) Tarun Tahiliani (d) Manish Arora(e) None of these

19. Who is the newly appointed Governor of Central Bank ofBangladesh?(a) Fazle Kabir (b) Abdul Hamid(c) AMA Muhith (d) Shahana Rahman(e) None of these

20. Which Indian scholar has been appointed as the ViceChancellor of the University of Canberra?(a) H Deep Saini (b) Sagar Mal(c) Mandeep Singh (d) Stephen Parker(e) None of these

21. Who has been appointed as the new chairman of BombayStock exchange (BSE)?(a) Sudhakar Rao (b) Sethurathnam Ravi(c) Prem Kumar (d) S Ramadorai(e) none of these

22. Who of the following Indian-American investor took chargeas Alternate Executive Director at International MonetaryFund (IMF)?(a) Nikhil Chopra (b) Sunil Sabharwal(c) Vikrant Singh (d) Dheerendra Payasi(e) None of these

23. Which Indian umpire has been promoted as the InternationalOutdoor Umpire by the International Hockey Federation(FIH)?(a) Kumar Dixit (b) Jitendra Rai(c) Nepoleon Singh (d) Mahendra Singh(e) None of these

24. Who among the following has taken over charge as DeputyChairman of Kolkata Dock System?(a) Sudheer K Juneja(b) Amit Kundan(c) S Balaji Arunkumar(d) K P Oli(e) None of these

25. Who among the following has been appointed as ChiefExecutive Officer of its agri-machinery business by EscortLimited?(a) Ravi A Menon(b) Ajit Kumar Sinha(c) O P Juneja(d) Ram Kumar Adhikari(e) None of these

26. Consider the following statements:1. Nita Ambani has been named the most powerful

businesswoman in Asia.2. Arundhati Bhattacharya has been ranked third on the

2016 'Asia's 50 Power Businesswomen' list3. The list has been released by Forbes.Which of the above statements are correct?(a) Only 1 & 2 (b) Only 1 & 3(c) Only 2 & 3 (d) All are correct(e) None of these

27. Who among the following persons has appointed as theCEO of Suzlon Group?(a) Anita Ghosh(b) J P Chalasani(c) Ajit Chaudhary(d) K S Sharma(e) None of these

28. Who among the following has taken charge as the GeneralManager (GM) of Pune unit of Bharat Electronics Ltd ?(a) M D Yadav(b) M K Sharma(c) Dinesh Kumar Batra(d) Advesh Singh Sharma(e) None of these

29. Who among the following has been appointed as non-official Director of CONCOR?(a) R K Mishra (b) Sanjeev Shah(c) M K Chaubey (d) Ajit Shah(e) None of these

30. Who assumed charge as the new secretary of RailwayBoard?(a) Sanjoy Mookerjee (b) Gangaram Agarwal(c) R. K. Verma (d) P. C. Gajbhiye'(e) None of these

RESPONSE

GRID

15. a b c d e 16. a b c d e 17. a b c d e 18. a b c d e 19. a b c d e

20. a b c d e 21. a b c d e 22. a b c d e 23. a b c d e 24. a b c d e

25. a b c d e 26. a b c d e 27. a b c d e 28. a b c d e 29. a b c d e

30. a b c d e

Page 195: yoursmahboob.wordpress.com SBI · yoursmahboob.wordpress.com iii P 101 Speed Tests for SBI Bank Clerk Exam 101 Speed Tests for SBI Bank Clerk Exam is revised and updated edition on

yoursmahboob.w

ordpress.com

1. Which of the following countries is not a member of ASEAN ?(a) Thailand (b) Vietnam(c) Myanmar (d) Brazil(e) None of these

2. Project ‘Sankalp’ is associated with the eradication of -(a) Polio (b) HIV/AIDS(c) Illiteracy (d) Tuberculosis(e) None of these

3. The 19th SAARC summit will be which held in country-(a) India (b) Nepal(c) Dhaka (d) Pakistan(e) None of these

4. The 21th summit of ASEAN will be organised in-(a) UAE (b) Brunei(c) Mascow (d) Laos(e) None of these

5. 8th BRICS summit held in which country-(a) Brazil (b) Russia(c) S. Africa (d) India(e) None of these

6. In which country the 2016 Annual Meeting of WEF organised ?(a) Switzerland (b) France(c) Germany (d) USA(e) None of these

7. How many members in the APEC nations organisation ?(a) 20 (b) 21(c) 22 (d) 23(e) None of these

8. The 17th NAM summit will be held in the country-(a) Iran (b) Kahira(c) India (d) Venezuela(e) None of these

9. In which country the G-20 2016 summit will be organised ?(a) Mexico (b) South Korea(c) China (d) USA(e) None of these

10. The 42nd summit of G-7 will be held in ________ .(a) USA (b) UK(c) Canada (d) Japan(e) None of these

11. Which of the following countries became the newmembers of NAM ?(a) Azerbaijan and Fiji(b) Syria and Comoras(c) Surinam and Guyana(d) Columbia and Cyprus(e) None of these

12. The 2016 NATO summit will be organised in which country ?(a) Russia (b) Poland(c) Canada (d) France(e) None of these

13. In which country first woman elected as a president ?(a) North Korea (b) South Korea(c) Japan (d) China(e) None of these

14. Who is elected as a new pope of Roman Catholic Churchrecently ?(a) Mario Bergoylio (b) George Allensary(c) Telesfor Toppo (d) Ivan Dias(e) None of these

15. The new organisation named “UN-women” created byunited nations came into existence on ?(a) 1 July 2010 (b) 1 July 2011(c) 1 July 2012 (d) 1 July 2013(e) None of these

RESPONSE

GRID

1. a b c d e 2. a b c d e 3. a b c d e 4. a b c d e 5. a b c d e

6. a b c d e 7. a b c d e 8. a b c d e 9. a b c d e 10. a b c d e

11. a b c d e 12. a b c d e 13. a b c d e 14. a b c d e 15. a b c d e

Max. Marks : 30 No. of Qs. 30 Time : 20 min. Date : ........./......../................

8989Events/Organisation/Summits

Page 196: yoursmahboob.wordpress.com SBI · yoursmahboob.wordpress.com iii P 101 Speed Tests for SBI Bank Clerk Exam 101 Speed Tests for SBI Bank Clerk Exam is revised and updated edition on

yoursmahboob.w

ordpress.com

188 SPEED TEST 89

RESPONSE

GRID

16. a b c d e 17. a b c d e 18. a b c d e 19. a b c d e 20. a b c d e

21. a b c d e 22. a b c d e 23. a b c d e 24. a b c d e 25. a b c d e

26. a b c d e 27. a b c d e 28. a b c d e 29. a b c d e 30. a b c d e

16. How many members are associated with BIMSTEC ?(a) 7 (b) 8(c) 9 (d) 10(e) None of these

17. What is the Ranking of India in Global Hunger Index list ?(a) 63rd (b) 66th(c) 67th (d) 68th(e) None of these

18. How many members are associated with the organisation‘MERCOSUR’ ?(a) 5 (b) 6(c) 7 (d) 8(e) None of these

19. The 9th world Hindi Conference held in which country ?(a) S. Africa (b) Sri Lanka(c) Russia (d) USA(e) None of these

20. How many points are mentioned to the achievement ofMillionium Development Goal LMDG-2015 of UN?(a) 6 (b) 7(c) 8 (d) 9(e) None of these

21. Where is the headquarter of F.A.O. ?(a) Italy (b) Paris(c) Jeneva (d) Moscow(e) None of these

22. (SAMPRITI-III), a special security forces exercise organisedbetween the countries of -(a) India-Sri Lanka (b) India-Bangladesh(c) India-Russia (d) India-USA(e) None of these

23. What does EAEU stand for?(a) Eurasian Economic Union(b) European Economic Union(c) European Energy Union(d) Eurasian Energy Union(e) None of these

24. Nuclear Security Summit 2016 took place on which day?(a) April 1, 2016 (b) April 2, 2016(c) April 3, 2016 (d) April 4, 2016(e) None of these

25. 13th Edition of World Spice Congress was held in whichstate?(a) Gujarat (b) Rajasthan(c) Kerala (d) Karnataka(e) None of these

26. The 12th Conference of Central Council of Health and FamilyWelfare was held to enforce which policy?(a) Draft National Health Policy(b) Draft National Family Welfare Policy(c) Draft National Welfare Policy(d) Draft National Health Welfare Policy(e) None of these

27. Annual Conference of State Minorities Commission whichcommenced on 24th February 2016 has which theme(s)?(a) "Sabka Saath,Sabka Vikas"(b) "Minority Welfare Schemes of Government of India-

An Overview"(c) "Functioning of State Minorities Commissions-

Problems and Challenges(d) Both (b) and (c)(e) None of these

28. Which maritime summit is being held for the first time inIndia in April?(a) Maritime India Summit(b) Maritime Cluster Development Summit(c) Maritime Smart Cities Summit(d) Maritime Inland Waterways Summit(e) None of these

29. Which website was launched for the Maritime Indian Summit2016?(a) www.maritimeinvestment.com(b) www.indianmaritime.in(c) www.indiamaritimeinvestment.com(d) www.maritimeinvest.in(e) None of these

30. 10th WTO Ministerial Conference was held in which city?(a) Nairobi(b) Durban(c) London(d) New York(e) None of these

Page 197: yoursmahboob.wordpress.com SBI · yoursmahboob.wordpress.com iii P 101 Speed Tests for SBI Bank Clerk Exam 101 Speed Tests for SBI Bank Clerk Exam is revised and updated edition on

yoursmahboob.w

ordpress.com

1. Jnanpith Award is given for which field?(a) Journalism (b) Music(c) Science (d) Literature(e) None of these

2. Highest award given to civilian in India is(a) Bharat Ratna (b) Padma Vibhushan(c) Sharam Award (d) Padma Bhushan(e) None of these

3. In which year National Film Awards were initiated?(a) 1952 (b) 1953(c) 1954 (d) 1955(e) None of these

4. Vyas Samman is awarded annually by(a) Azim Premji Foundation(b) Times Group(c) KK Birla Foundation(d) Ministry of Culture(e) None of these

5. Saraswati Samman is given to which field?(a) Sanskrit Literature(b) Science(c) Literature(d) Social Harmony(e) None of these

6. The second highest Gallantry award is(a) Mahavir Chakra(b) Vir Chakra(c) Arjuna Award(d) Ashok Chakra(e) None of these

7. Which of the following states conferred the Bihari Puraskar?(a) Uttar Pradesh(b) Bihar(c) Madhya Pradesh(d) Rajasthan(e) None of these

8. Tansen Samman is conferred in the field of :(a) Music (b) Literature(c) Science (d) Journalism(e) None of these

9. ‘Ashoka Chakra’ is awarded for(a) the most conspicuous bravery or self sacrifice on land,

air or sea but not in the presence of the enemy(b) acts of gallantry in the presence of enemy(c) gallantry by children(d) outstanding contribution to literature(e) None of these

10. Shanthi Swaroop Bhatnagar awards are given for(a) exploring new dimensions in creative writing in Indian

languages(b) outstanding contribution to science(c) creating mass awareness on environmental issues(d) excellence in film direction(e) None of these

11. The prestigious Ramon Magsaysay Award was conferredupon Mr.Arvind Kejriwal in which of the following category?(a) Emergent Leadership(b) Literature(c) Community Welfare(d) Government Service(e) None of these

12. B. C. Roy Award is given in the field of(a) Medicine (b) Music(c) Journalism (d) Environment(e) None of these

13. The Pampa Prashasti is the highest literacy award given bywhich of the following states?(a) Karnataka(b) Kerala(c) Andhra Pradesh(d) Maharashtra(e) None of these

14. Which among the following states has won the 10th Na-tional Award for Excellence work in Mahatma Gandhi Na-tional Rural Employment Guarantee Act (MGNREGA)?(a) Madhya Pradesh(b) Karnataka(c) West Bengal(d) Haryana(e) None of these

RESPONSE

GRID

1. a b c d e 2. a b c d e 3. a b c d e 4. a b c d e 5. a b c d e6. a b c d e 7. a b c d e 8. a b c d e 9. a b c d e 10. a b c d e11. a b c d e 12. a b c d e 13. a b c d e 14. a b c d e

Max. Marks : 30 No. of Qs. 30 Time : 20 min. Date : ........./......../................

9090Awards andHonours

Page 198: yoursmahboob.wordpress.com SBI · yoursmahboob.wordpress.com iii P 101 Speed Tests for SBI Bank Clerk Exam 101 Speed Tests for SBI Bank Clerk Exam is revised and updated edition on

yoursmahboob.w

ordpress.com

190 SPEED TEST 90

RESPONSE

GRID

15. a b c d e 16. a b c d e 17. a b c d e 18. a b c d e 19. a b c d e20. a b c d e 21. a b c d e 22. a b c d e 23. a b c d e 24. a b c d e25. a b c d e 26. a b c d e 27. a b c d e 28. a b c d e 29. a b c d e30. a b c d e

15. Which of the following famous financial journals of interna-tional repute confers ‘Finance minister of the year’ Award?(a) Dalal Street(b) Euromoney(c) Business Standard(d) Money Matters(e) None of these

16. The Nobel prize was instituted by which country?(a) USA (b) UK(c) Russia (d) Sweden(e) None of these

17. When did the Nobel prize in the Economics Scienceslaunched?(a) 1901 (b) 1942(c) 1967 (d) 1975(e) None of these

18. The Academy award is also known as(a) Oscar Award (b) BAFTA Award(c) Matthews Award (d) Palm d’ore(e) None of these

19. Confucius peace prize is given by(a) Sri Lanka (b) India(c) China (d) S. Korea(e) None of these

20. Pulitzer prize was established in(a) 1917 (b) 1918(c) 1922 (d) 1928(e) None of these

21. Nobel prizes are distributed annually at(a) Manila (b) New York(c) Stockholm (d) Geneva(e) None of these

22. BAFTA prize is distributed by(a) UK (b) Russia(c) India (d) USA(e) None of these

23. Golden Globe award is given by(a) UK (b) France(c) USA (d) China(e) None of these

24. Palme d’or prize is given by(a) France (b) USA(c) UK (d) Indonesia(e) None of these

25. Which of the following is an award instituted by UNESCO?(a) Kalinga Award(b) Pulitzer prize(c) Stirling prize(d) Pritzker prize(e) None of these

26. International Gandhi Peace prize is instituted in(a) 1995 (b) 1996(c) 1997 (d) 1998(e) None of these

27. The Stirling prize is a British prize for excellence in(a) Medicine (b) Science(c) Architecture (d) Literature(e) None of these

28. Which of the following award is given by World EconomicForum?(a) Crystal Award(b) Kalinga prize(c) Pulitzer Award(d) Abel prize(e) None of these

29. Magsaysay award is given by(a) USA (b) UK(c) Malaysia (d) Philippines(e) None of these

30. Booker prize is given to the field of :(a) Fiction (b) Poetry(c) Drama (d) Essay(e) None of these

Page 199: yoursmahboob.wordpress.com SBI · yoursmahboob.wordpress.com iii P 101 Speed Tests for SBI Bank Clerk Exam 101 Speed Tests for SBI Bank Clerk Exam is revised and updated edition on

yoursmahboob.w

ordpress.com

1. Who is the author of “An Uncertain Glory: India and itsContradictions”?(a) Amatya Sen & Michael Bush(b) Amartya Sen & Satya Paul(c) Amartya Sen & Jean Dreze(d) Amartya Sen & Zeenat Shaukat(e) Amartya Sen & Salman Rushdie

2. Who is the author of “Ambedkar Speaks (Triology)”?(a) Dr Dheerendra (b) Dr Satyendra Singh(c) Dr Kiran Yadav (d) Dr Narendra Jadhav(e) Akhilesh Yadav

3. Pax Indica is recently authored book by?(a) Narendra Modi (b) Atal Bihari Vajpayee(c) Shashi Tharoor (d) A P J Abdul Kalam(e) None of these

4. Who is the author of “The Outsider”?(a) Jimmy Connors (b) Amartya Sen(c) Jean Dreze (d) Zeenat Shaukat(e) Salman Rushdie

5. Name the book authored by Garima Sanjay which wasreleased by Hamid Ansari. The book highlights how a manblames the luck or someone for his failures?(a) Yaadein (b) Smritiyan(c) Divine Journey (d) Lucky Me(e) Man by Nature

6. Who is the author of “Women of Vision”?(a) Amish Tripathi (b) Preeti Shenoy(c) Durjoy Dutta (d) Alam Srinivas(e) Ravinder Singh

7. Who among the following is the author of “Religion, Law &Society - Across the Globe”?(a) Salman Rushdie (b) Tahir Mahmood(c) Jim Herley (d) Mohd.Raza(e) None of these

RESPONSE

GRID

1. a b c d e 2. a b c d e 3. a b c d e 4. a b c d e 5. a b c d e

6. a b c d e 7. a b c d e 8. a b c d e 9. a b c d e 10. a b c d e

11. a b c d e 12. a b c d e 13. a b c d e 14. a b c d e

Max. Marks : 30 No. of Qs. 30 Time : 20 min. Date : ........./......../................

9191Books and Authors

8. Who among the following is the author of “How to GetFilthy Rich in Rising Asia”?(a) Mohsin Hamid (b) Tahir Mahmood(c) Mohd Hamid (d) Mohd.Qazi(e) Mohamed Yusuf

9. Who has written the book “Walking with Lions: Tales froma Diplomatic Past”?(a) P.Chidambaram (b) K.Natwar Singh(c) Sashwant Sinha (d) Jaswant Singh(e) Manmohan Singh

10. A book ‘Fault Lines’ is written by -(a) Raghuram Rajan (b) Subba Rao(c) Vimal Jalan (d) Montek Singh Ahluwalia(e) None of these

11. Which of the following books has been written by VikramSeth?(a) My God Died Young(b) Islamic Bomb (c) Look Back in Anger(d) A Suitable Boy (e) None of these

12. Who is the author of the book India 2020?(a) Nibal Singh (b) R.K.Narayan(c) Sidney Shelton (d) Dr.A.P.J.Abdul Kalam(e) None of these

13. The book ‘My Nation My Life’ was written by(a) L.K.Advani (b) T.N.Seshan(c) Fervez Musharaf (d) Manmohan Singh(e) None of these

14. The famous book ‘Anandmath’ was authored by(a) Sarojini Naidu(b) Bankim Chandra Chottapadhya(c) Sri Aurobindo(d) Rabindrnath Tagore(e) None of these

Page 200: yoursmahboob.wordpress.com SBI · yoursmahboob.wordpress.com iii P 101 Speed Tests for SBI Bank Clerk Exam 101 Speed Tests for SBI Bank Clerk Exam is revised and updated edition on

yoursmahboob.w

ordpress.com

192 SPEED TEST 91

RESPONSE

GRID

15. a b c d e 16. a b c d e 17. a b c d e 18. a b c d e 19. a b c d e20. a b c d e 21. a b c d e 22. a b c d e 23. a b c d e 24. a b c d e25. a b c d e 26. a b c d e 27. a b c d e 28. a b c d e 29. a b c d e30. a b c d e

15. Who is the author of “INDIA : The Future is Now”.(a) Robin Sharma (b) Shashi Tharoor(c) Amartya Sen (d) Arundhati Roy(e) None of these

16. ‘Freedom from Fear’ is a book written by ?(a) Benzir Bhutto (b) Corazon Aquino(c) Aung san suu Kyi (d) Nayantara Seghal(e) None of these

17. ‘India of our Dreams’ is a book written by(a) Dr. S. Radhakrishnan(b) Dr. C. Subramanian(c) M.V. Kamath(d) Dr. Rajendra Prasad(e) None of these

18. Which of the following books is written by Sunil Gavaskar?(a) A Brief History of Time(b) A Sense of Time (c) Sunny Days(d) Great Expectations (e) None of these

19. Which of the following is NOT written by MunshiPremchand?(a) Gaban (b) Guide(c) Godan (d) Manasorovar(e) None of these

20. Who is the author of the book ‘Beyond the Lines : AnAutobiography’ ?(a) General J. J. Singh (b) Kuldip Nayar(c) Ray Bradbury (d) Khushwant Singh(e) None of these

21. Who among the following is the author of the book “Indiaand Malaysia: Intertwined Strands”?(a) Veena Sikri (b) Naveen Bandopadhya(c) Sushma Singh (d) Ravichandran Nayak(e) None of these

22. The Global Competitiveness Report is published from timeto time by ___?(a) World Bank(b) International Monetary Fund(c) Yale University(d) World Economic Forum(e) None of these

23. Which of the following books is written by Panini?(a) Mudrarakshas (b) Ashtadhyayi(c) Prem Vatika (d) Bijak(e) None of these

24. The book ‘Mitakshara’ is written by(a) Vatsyayana (b) Jeemaatwahan(c) Vigyaneshwar (d) Shudrak(e) None of these

25. Which of the following books is written by Kautilya?(a) Daybhag (b) Rajtarangini(c) Arthashastra (d) Mitakshara(e) None of these

26. Post Office’ is written by?(a) R.K. Narayan(b) Mulk Raj Anand(c) R.K Laxman(d) Rabindra Nath Tagore(e) None of these

27. Who is called the Father of English Poetry?(a) Charles Dickens (b) Milton(c) Chaucer (d) Wordsworth(e) None of these

28. Who wrote Jungle Book?(a) Mohd. Salim(b) Rudyard Kipling(c) Sibhu(d) R.K. Narayan(e) None of these

29. Who wrote ‘War and Peace’?(a) Leo Tolstoy (b) Mahatma Gandhi(c) Charles Dickens (d) Kipling(e) None of these

30. Who wrote the book ‘The Prince’?(a) Bernard Shaw (b) Niccolo Machiaveli(c) V.S. Sharma (d) Emile Zola(e) None of these

Page 201: yoursmahboob.wordpress.com SBI · yoursmahboob.wordpress.com iii P 101 Speed Tests for SBI Bank Clerk Exam 101 Speed Tests for SBI Bank Clerk Exam is revised and updated edition on

yoursmahboob.w

ordpress.com

1. The Olympic Museum was opened at which of the followingplaces?(a) Rome (b) Berlin(c) Lausanne (d) Athens(e) None of these

2. With which sport the term’ Caddie’ is associated?(a) Polo (b) Golf(c) Bridge (d) Billiards(e) None of these

3. Champion Trophy is associated with(a) Football (b) Hockey(c) Cricket (d) Chess(e) None of these

4. Rangaswami Cup is associated with(a) Wrestling (b) Football(c) Hockey (d) Golf(e) None of these

5. ‘Grand Slam’ is associated with the game of(a) Lawn Tennis (b) Hockey(c) Football (d) Swimming(e) None of these

6. First youth Olympic games was held in(a) Japan (b) China(c) North Korea (d) Singapore(e) None of these

7. ‘Subroto Cup’ is associated with(a) Badminton (b) Cricket(c) Chess (d) Football(e) None of these

8. Wankhede Stadium is situated in(a) Mumbai (b) Delhi(c) Lucknow (d) Bangalore(e) None of these

9. The term ‘Gambit’ is associated with(a) Chess (b) Tennis(c) Basketball (d) Baseball(e) None of these

RESPONSE

GRID

1. a b c d e 2. a b c d e 3. a b c d e 4. a b c d e 5. a b c d e6. a b c d e 7. a b c d e 8. a b c d e 9. a b c d e 10. a b c d e11. a b c d e 12. a b c d e 13. a b c d e 14. a b c d e 15. a b c d e16. a b c d e 17. a b c d e

Max. Marks : 30 No. of Qs. 30 Time : 20 min. Date : ........./......../................

9292Sports and Games

10. ‘Ashes’ is the term associated with which of the followingsports?(a) Cricket (b) Badminton(c) Basketball (d) Football(e) None of these

11. National Sports Day is celebrated on(a) 29th Aug. (b) 4th Dec.(c) 14th Nov. (d) 28th Oct.(e) None of these

12. The term ‘bogey’ is associated with(a) Cricket (b) Chess(c) Golf (d) Baseball(e) None of these

13. FINA is governing body of which sports?(a) Cricket (b) Archery(c) Water polo (d) Polo(e) None of these

14. Which of the following trophies/cups associated with thegame of Hockey?(a) Derby (b) Aga Khan Cup(c) Merdeka (d) Vizzy Trophy(e) None of these

15. The ‘Dronacharya Award’ is given to(a) Coaches (b) Sportspersons(c) Umpires (d) Sports Editors(e) None of these

16. When did the Wimbledon Grand Slam Tennis tournamentstart?(a) 1857 (b) 1877(c) 1897 (d) 1898(e) None of these

17. In which year, the Grand Master title of Chess started?(a) 1971 (b) 1972(c) 1973 (d) 1974(e) None of these

Page 202: yoursmahboob.wordpress.com SBI · yoursmahboob.wordpress.com iii P 101 Speed Tests for SBI Bank Clerk Exam 101 Speed Tests for SBI Bank Clerk Exam is revised and updated edition on

yoursmahboob.w

ordpress.com

194 SPEED TEST 9218. Duleep Trophy is associated with the game of

(a) Hockey (b) Badminton(c) Football (d) Cricket(e) None of these

19. Which game is associated with Queensberry rules?(a) Weight lifting (b) Boxing(c) Golf (d) Polo(e) None of these

20. How many players are there in Kho-Kho?(a) 9 (b) 10(c) 8 (d) 7(e) None of these

21. What is the National Game of Russia?(a) Chess (b) Hockey(c) Table Tennis (d) Baseball(e) None of these

22. Hurlington stadium is associated with(a) Polo (b) Cricket(c) Boxing (d) Golf(e) None of these

23. First Olympic Games were held in-(a) 776 BC. (b) 798 BC.(c) 876 BC. (d) 898 BC.(e) None of these

24. With which game is Bully associated?(a) Cricket (b) Football(c) Golf (d) Hockey(e) None of these

25. Indian Sports Research Institute is located at(a) Patiala (b) Delhi(c) Cochin (d) Poona(e) None of these

26. Who was the first Indian to win an individual medal in Olym-pics?(a) PT Usha(b) Karnam Malleshwari(c) Deepika Kumari(d) Sania Nehwal

27. Which county did Ravi Shastri play for?(a) Glamorgan (b) Leicestershire(c) Gloucestershire (d) Lancashire(e) None of these

28. The first World Cup Hockey was played in(a) Amsterdom, 1972(b) Barcelona, 1971(c) Kualalumpur, 1975(d) Mumbai, 1976(e) None of these

29. The normal length of a football ground must be(a) 110 – 120 m (b) 100 – 110 m(c) 90 – 100 m (d) 120 – 130 m(e) None of these

30. India first won the Olympic Hockey gold at(a) Amsterdam (b) Los Angeles(c) Mumbai (d) Tokyo(e) None of these

RESPONSE

GRID

18. a b c d e 19. a b c d e 20. a b c d e 21. a b c d e 22. a b c d e23. a b c d e 24. a b c d e 25. a b c d e 26. a b c d e 27. a b c d e28. a b c d e 29. a b c d e 30. a b c d e

Page 203: yoursmahboob.wordpress.com SBI · yoursmahboob.wordpress.com iii P 101 Speed Tests for SBI Bank Clerk Exam 101 Speed Tests for SBI Bank Clerk Exam is revised and updated edition on

yoursmahboob.w

ordpress.com

1. Line connecting the points on a map that have the sametemperature is called?(a) Isobar (b) Isotherm(c) Isohyet (d) Isohels(e) None of these

2. Rearing of silkworms for the production of raw silk is called?(a) Horticulture (b) Sericulture(c) Viticulture (d) Apiculture(e) None of these

3. In which of the following seas India has building Tsunamiwarning device?(a) Arabian Sea (b) South China Sea(c) Bay of Bengal (d) Indian Ocean(e) None of these

4. GlaxoSmithKline developed world’s first malaria vaccinecalled ___?(a) RTS, S (b) MTS, S(c) RMS, S (d) MRS, S(e) None of these

5. With which of the following Kepler’s Laws are related to_______?(a) Motion of Milkyway(b) Motion of planets around sun(c) Rotation of Earth on Its own axis(d) Motion of Moon around earth(e) None of these

6. Where is located the Centre for Wind Energy Technology(C-WET)?(a) Hyderabad (b) Chennai(c) Kochi (d) Kolkata(e) None of these

7. Which of the following statements is true about “TomTato”?(a) A new mammal species discovered in UK(b) A plant produces both tomatoes and potatoes(c) A man eating fish species(d) New element discovered under deep sea(e) None of these

8. What is the name of Australia’s most powerful supercomputer ?(a) Spirit (b) Raijin(c) Aizen (d) Shinto(e) None of these

9. Which was the first atomic submarine of India?(a) INS Chakra (b) INS Vikrant(c) INS Dhanush (d) INS Viraat(e) None of these

10. Vikram Sarabhai Space Centre (VSSC) is at(a) Thiruvananthapuram(b) Mumbai(c) Hyderabad(d) Bengaluru(e) None of these

11. The first fertilizer plant in India was established in(a) Trombay (b) Nangal(c) Alwaye (d) Sindri(e) None of these

12. Name of first indigenously developed Super Computer ofIndia is?(a) Param (b) Aryabhatt(c) Apsara (d) Tejas(e) None of these

13. Which of the following is the principal ore of Mercury?(a) Bauxite (b) Cinnabar(c) Hematite (d) Galena(e) None of these

14. Name the Chinese supercomputer which is declared thefastest computer of the world-(a) Tianhe-2 (b) Chinhane 1(c) Kisova (d) Techo-1(e) None of these

RESPONSE

GRID

1. a b c d e 2. a b c d e 3. a b c d e 4. a b c d e 5. a b c d e6. a b c d e 7. a b c d e 8. a b c d e 9. a b c d e 10. a b c d e11. a b c d e 12. a b c d e 13. a b c d e 14. a b c d e

Max. Marks : 25 No. of Qs. 25 Time : 20 min. Date : ........./......../................

9393Science andTechnology

Page 204: yoursmahboob.wordpress.com SBI · yoursmahboob.wordpress.com iii P 101 Speed Tests for SBI Bank Clerk Exam 101 Speed Tests for SBI Bank Clerk Exam is revised and updated edition on

yoursmahboob.w

ordpress.com

196 SPEED TEST 93

RESPONSE

GRID

15. a b c d e 16. a b c d e 17. a b c d e 18. a b c d e 19. a b c d e

20. a b c d e 21. a b c d e 22. a b c d e 23. a b c d e 24. a b c d e

25. a b c d e

15. Name the organization that launched a series of satelliteswhich is supposed to provide fast, cheap Internet and phoneservice to remote rural areas in 180 countries.(a) A2B Networks (b) R2R Networks(c) O3B Networks (d) M2M Networks(e) B2B Networks

16. Name the place in India where Early Tsunami WarningSystem have been installed(a) Rangachang (b) Kanyakumari(c) Chilka (d) Mysore(e) Ootkamandalam

17. Researchers have developed Eco-friendly batteries whichcan be used in power plants or to store solar energy.Thesebatteries are made up using which material(a) Carbon, Tin, Sodium(b) Wood, Tin, Carbon(c) Wood, Tin and Sodium(d) Sodium, carbon, Tin(e) Nickel, Carbon, Zinc

18. Which of the following is an indigenously built light combataircraft of India?(a) Akash (b) Vikrant(c) Tejas (d) Arjun(e) None of these

19. Name the broadband and telecommunications providerwhich launched world's first Firefox OS smartphone?(a) Telefonica (b) Apple(c) Samsung (d) Telenor(e) None of these

20. The First Navigation Satellite launched by ISRO(a) PSLV C 2 (b) IRNSS-1A(c) ISS-1A (d) INSAT(e) Edusat

21. The Rotavirus vaccine ROTOVAC developed by Indianscientists cures which disease?(a) Diarrohea (b) Cancer(c) Diabetes (d) Arthritis(e) Pneumonia

22. India's first DNA Forensic Laboratory is established in whichcity(a) Gurgaon (b) Mumbai(c) Delhi (d) Kanpur(e) Banglore

23. "A Boy and His Atom" - World's smallest movie made withone of the smallest particles of any element in the universe:atoms is developed by which organization(a) Microsoft(b) Infosys(c) IBM(d) Toshiba(e) Sony

24. Which among the following is the First cruise missile testfired by India?(a) Aakash(b) Nirbhay(c) Agni-3(d) Aakash-2(e) Prithvi-3

25. First Indian to go into space(a) Mohan Sharma(b) Rakesh Chaudhary(c) Rakesh Sharma(d) Suresh Sharma(e) None of these

Page 205: yoursmahboob.wordpress.com SBI · yoursmahboob.wordpress.com iii P 101 Speed Tests for SBI Bank Clerk Exam 101 Speed Tests for SBI Bank Clerk Exam is revised and updated edition on

yoursmahboob.w

ordpress.com

1. India's first Aadhaar enabled ATM has been launched bywhich bank? (a) State Bank of India (b) ICICI Bank (c) DCB Bank (d) Axis Bank(e) Indian Bank

2. Which is the first bank to commence the sale of Indian GoldCoin (IGC) in the domestic market?(a) State Bank of India (b) Indian Overseas Bank(c) Punjab National Bank (d) Bank of Baroda(e) Indian Bank

3. Who is the newly appointed Governor of Central Bank ofBangladesh?(a) Fazle Kabir (b) Abdul Hamid(c) AMA Muhith (d) Shahana Rahman(e) None of these

4. India's first contact-less mobile payment solution "iTap" hasbeen launched by which bank?(a) State Bank of India (b) HDFC Bank(c) Axis Bank Ltd (d) ICICI Bank(e) None of these

5. Which bank has launched iWork@home programme for itswomen employees?(a) Yes bank (b) Exim Bank(c) ICICI Bank (d) Axis Bank Ltd.(e) HDFC Bank

6. Which of the following banks will become the India's firstSmall Finance Bank (SFB)?(a) Ujjivan Financial Services Pvt. Ltd(b) Capital Local Area Bank Ltd(c) Au Financiers Ltd(d) Janalakshmi Financial Services Pvt. Ltd(e) None of these

7. Which bank has won the first ever Green Bond Pioneer Award2016?(a) State Bank of India (b) Bank of Baroda(c) Punjab National Bank (d) Yes Bank(e) ICICI Bank

RESPONSE

GRID

1. a b c d e 2. a b c d e 3. a b c d e 4. a b c d e 5. a b c d e6. a b c d e 7. a b c d e 8. a b c d e 9. a b c d e 10. a b c d e11. a b c d e 12. a b c d e 13. a b c d e 14. a b c d e

Max. Marks : 23 No. of Qs. 23 Time : 15 min. Date : ........./......../................

8. Which bank has launched 'Japan Desk' to facilitate Japanesecorporates investing in India?(a) State Bank of India (b) Punjab National Bank(c) Bank of Baroda (d) Dena Bank(e) ICICI Bank

9. Which of the following agreement is signed between ReserveBank of India (RBI) and UAE Central Bank?(a) Monetary Exchange Agreement(b) Currency Swap Agreement(c) Anti-Money Laundering Agreement(d) Rupee Exchange Agreement(e) Money Exchange Agreement

10. Who has been appointed as Vice-President of AsianInfrastructure Investment Bank (AIIB)?(a) Kamal Vatta (b) D J Pandian(c) Mitali Saran (d) Sreenivasan Kumar(e) None of these

11. Who has been appointed from India as Senior Director inthe World Bank?(a) Amitabh Singh (b) Saroj Kumar Jha(c) Suvarna Kumar (d) Nikhil Srivastav(e) None of these

12. Which bank has recently raised 900 crore by way of issuingBasel-III compliant tier-II bonds?(a) IDBI Bank (b) Central Bank of India(c) Syndicate Bank (d) Canara Bank(e) State Bank Of India

13. Which public sector bank has bagged three awards at the'National Payments Excellence Awards 2015, organized byNational Payments Corporation of India (NPCI)?(a) Bank of Baroda (b) State Bank of India(c) Allahabad Bank (d) Corporation Bank(e) None of these

14. Which bank has recently launched Green PIN facility, underwhich a customer can obtain duplicate PIN for debit cardinstantly through SMS request?(a) Punjab National Bank (b) ICICI Bank(c) Dena Bank (d) HDFC Bank(e) None of these

9494Current Banking

Page 206: yoursmahboob.wordpress.com SBI · yoursmahboob.wordpress.com iii P 101 Speed Tests for SBI Bank Clerk Exam 101 Speed Tests for SBI Bank Clerk Exam is revised and updated edition on

yoursmahboob.w

ordpress.com

198 SPEED TEST 94

RESPONSE

GRID

15. a b c d e 16. a b c d e 17. a b c d e 18. a b c d e 19. a b c d e

20. a b c d e 21. a b c d e 22. a b c d e 23. a b c d e

15. Reserve Bank has imposed a penalty of Rs. 1 crore on whichassociate bank of SBI for violation of its instructionsincluding reporting of data to Central Repository ofInformation on Large Credits (CRILC)?(a) State Bank of Patiala(b) State Bank of Bikaner and Jaipur(c) State Bank of Hyderabad(d) State Bank of Travancore(e) None of these

16. Who among the following has been reappointed as DeputyGovernor of RBI for three year term?(a) Rakesh Mohan (b) Urjit Patel(c) H R Khan (d) S S Mundra(e) None of these

17. In January 2016, which bank had launched ImmediatePayment Service (IMPS) across branches to provide inter-bank electronic transfer service, capable of processingperson to account remittances?(a) Central Bank of India (b) Canara Bank(c) Andhra Bank (d) Corporation Bank(e) None of these

18. Which private sector bank will partner with FINO PayTechto foray into the payments bank space?(a) Axis Bank (b) HDFC Bank(c) Yes Bank (d) ICICI Bank(e) None of these

19. Which country has recently allowed Six central banks andinternational financial institutions, including Reserve Bankof India (RBI), to enter China's interbank foreign exchangemarket from January 2016?

(a) South Korea (b) China(c) Brazil (d) South Africa(e) None of these

20. Under what name, State Bank of India has launched aspecialised branch in Bengaluru that caters to startups?(a) InCube (b) Foster(c) KickStart (d) SBIHatch(e) None of these

21. Which public sector bank has tied up with e-commerce majorSnapdeal to offer instant working capital loans to its onlinee-commerce sellers?(a) Bank of Baroda (b) State Bank of India(c) Corporation Bank (d) Syndicate Bank(e) None of these

22. The 2016 World Development Report by World Bank notedthat almost 1.063 billion Indians were offline even thoughIndia ranked among the top five nations in terms of the totalnumber of Internet users. What is the title of 2016 WorldDevelopment Report?(a) 'Technology Inclusion'(b) 'Digital Dividends'(c) 'Sustainable Technology'(d) 'Bridging Technology Divide'(e) None of these

23. Which Central Bank has imposed negative interest rate inits monetary policy statement of January 2016?(a) Bank of England (b) People's Bank of China(c) Bank of Japan (d) European Central bank(e) None of these

Page 207: yoursmahboob.wordpress.com SBI · yoursmahboob.wordpress.com iii P 101 Speed Tests for SBI Bank Clerk Exam 101 Speed Tests for SBI Bank Clerk Exam is revised and updated edition on

yoursmahboob.w

ordpress.com

1. The Pradhan Mantri Ujjwal Yojana will provide -(a) free electricity to the rural households.(b) free higher education to the girl child who belongs to a

poor family.(c) LPG to women of all households below poverty line.(d) scholarships to the children of the people below

poverty line.(e) None of these

2. Who is the Vice Chairman of NITI AAYOG in India?(a) A P Shan(b) Arvind Pangariya(c) Rakesh Tripath(d) B.P. Jeevan Reddy(e) None of these

3. Who is the winner of the prestigious 2016 Abel Prize?(a) Endre Szemeredi (b) Andrew Wiles(c) John Rognes (d) Michael Atiyah(e) None of these

4. What is the rank of India in the United Nations, WorldHappiness Index 2016?(a) 118 (b) 105(c) 98 (d) 125(e) None of these

5. Where was 13th meeting of Broadband Commission forSustainable Development held?(a) New York (b) Dubai(c) Switzerland (d) Mexico(e) None of these

6. March 15 is celebrated across the world as?(a) World Human Rights Day(b) World Health Day(c) World Consumer Rights Day(d) World Sleep Day(e) None of these

7. The fifth edition of Indian Aviation show was conducted inwhich city?(a) Odisha (b) Mumbai(c) Bangalore (d) Hyderabad(e) None of these

RESPONSE

GRID

1. a b c d e 2. a b c d e 3. a b c d e 4. a b c d e 5. a b c d e6. a b c d e 7. a b c d e 8. a b c d e 9. a b c d e 10. a b c d e11. a b c d e 12. a b c d e 13. a b c d e 14. a b c d e 15. a b c d e

Max. Marks : 30 No. of Qs. 30 Time : 15 min. Date : ........./......../................

8. Where was the fourth Nuclear Security Summit (NSS) held?(a) London (b) Hong Kong(c) Washington (d) New Delhi(e) None of these

9. Who was appointed as the advisor of the Andhra PradeshGovernment by Chief Minister Chandrababu Naidu?(a) Baba Ram dev(b) Kodali Venkateswara Rao(c) Hari Krishna(d) Chaganti Koteswara Rao(e) None of these

10. Who was sworn-in as the President of Kosovo in April 2016?(a) Jakup Krasniqi (b) Hashim Thaci(c) Atifete Jahjaga (d) Fadil Hoxha(e) None of these

11. Which team topped the April 2016 edition of FIFA or Coca-Cola World Rankings of Soccer?(a) Argentina (b) Brazil(c) France (d) Italy(e) None of these

12. Which online furniture store was acquired by the FutureGroup in the first week of April 2016?(a) FabFurnish.com (b) UrbanLadder.com(c) Pepperfry.com (d) Ezaara.com(e) None of these

13. The book 'Endurance: My Year in Space and Our Journey toMars' as announced by the publisher Alfred A. Knopf, is astory of which famous personality?(a) Scott Kelly (b) Mark Kelly(c) Mikhail Korniyenko (d) Timothy Kopra(e) None of these

14. Who is going to succeed Vijay Mallya as the chairman ofUnited Spirits Limited?(a) Anand Kripalu (b) M.K. Sharma(c) Nicholas Blazquez (d) Angus Mc Dowell(e) None of these

15. What is the theme of 2016 World Health Day (WHD)?(a) Healthy heart beat, Healthy blood pressure(b) Beat Diabetes(c) Healthy blood pressure(d) Save lives: Make hospitals safe in emergencies(e) None of these

9595Current Affairs

Page 208: yoursmahboob.wordpress.com SBI · yoursmahboob.wordpress.com iii P 101 Speed Tests for SBI Bank Clerk Exam 101 Speed Tests for SBI Bank Clerk Exam is revised and updated edition on

yoursmahboob.w

ordpress.com

200 SPEED TEST 95

RESPONSE

GRID

16. a b c d e 17. a b c d e 18. a b c d e 19. a b c d e 20. a b c d e

21. a b c d e 22. a b c d e 23. a b c d e 24. a b c d e 25. a b c d e

26. a b c d e 27. a b c d e 28. a b c d e 29. a b c d e 30. a b c d e

16. Who won the 2016 Formula 1 Rolex Australian Grand Prix atMelbourne Grand Prix?(a) Sebastian Vettel (b) Nico Rosberg(c) Daniel Ricciardo (d) Lewis Hamilton(e) None of these

17. Which unique Biosphere Reserve of India in the WesternGhats is among 20 new sites added by the UNESCO to itsWorld Network of Biosphere Reserves?(a) Great Nicobar(b) Simlipal Biosphere Reserve(c) Agasthyamala Biosphere(d) Nanda Devi Biosphere Reserve(e) None of these

18. Who was conferred with skoch Lifetime Achievement Awardin 2016?(a) Venkaiah Naidu (b) Arun Jaitley(c) Rajnath Singh (d) L K Advani(e) None of these

19. Bedaquiline drug launched by the Health Ministry will beused to treat which disease?(a) Dengue (b) Tuberculosis(c) Malaria (d) Zika Virus(e) None of these

20. Which veteran singer was awarded the Global Green HeroAward 2016?(a) Stevie Wonder (b) Lionel Richie(c) Whitney Houston (d) None of the above(e) None of these

21. What was the theme of International women's Day in 2016?(a) Equality for women is progress for all(b) Planet 50-50 by 2030: Step It up for Gender Equality(c) Empowering women, Empowering Humanity: Picture it!(d) Empowerment of women for sustainable Development(e) None of these

22. The Indian women's team on 6 March 2016 defeated whichcountry to bag gold medal at the Second Division of theWorld Team Table Tennis Championship?(a) China (b) Brazil(c) Luxembourg (d) Japan(e) None of these

23. India extended 2 billion US dollar line of Credit to whichcountry for implementing Socio-economic developmentprojects?(a) Bangladesh (b) Nepal(c) Sri Lanka (d) Myanmar(e) None of these

24. Who was appointed as the Chairman of CentralAdministrative Tribunal in April 2016?(a) Justice (retd) Permod Kohli(b) Justice Syed Rafat Alam(c) Justice A P Shah(d) None of them(e) None of these

25. Which has become the first state to pass land title bill?(a) Rajasthan (b) Bihar(c) Punjab (d) Karnataka(e) None of these

26. Who on 6 April 2016 was appointed the Head of LamborghiniIndia Operations?(a) Vincet Parker (b) Sharad Agarwal(c) Himanshu Sharma (d) Vivek Agnihotri(e) None of these

27. Who was awarded the UNESCO or Guillermo Cano WorldPress Freedom Prize 2016?(a) Mazen Darwish (b) Khadija Ismayilova(c) Ahmet ??k (d) None of the above(e) None of these

28. Who wrote the book titled "India's War: The Making ofModern South Asia 1939-1945"?(a) Srinath Raghavan (b) Mini Kapoor(c) Sreedhar Chandan (d) Suhasini Haidar(e) None of these

29. Who has won the 2016 US Masters golf tournament?(a) Jordan Spieth (b) Lee Westwood(c) Danny Willett (d) J Donaldson(e) None of these

30. Whose birth anniversary was celebrated on April 13, 2016at the UN headquarters?(a) B R Ambedkar (b) Annie Besant(c) Florence Nightingale (d) George Washington(e) None of these

Page 209: yoursmahboob.wordpress.com SBI · yoursmahboob.wordpress.com iii P 101 Speed Tests for SBI Bank Clerk Exam 101 Speed Tests for SBI Bank Clerk Exam is revised and updated edition on

yoursmahboob.w

ordpress.com

1. Which of the following games is not included in Olympicgames?(a) Football (b) Golf(c) Badminton (d) Hockey(e) Table Tennis

2. Which of the following awards is given only to individualsand organisations of Asian countries for excellence in theirrespective fields?(a) Booker Prize (b) Nobel Prize(c) Templeton Prize (d) Ramon Magsaysay Award(e) Oscar awards

3. Which Indian city is known as the ‘Garden City’?(a) Udaipur (b) Chandigarh(c) Srinagar (d) Bangaluru(e) Mysore

4. Who amongst the following is the author of the English novel‘The White Tiger’?(a) Kiran Desai (b) V. S. Naipaul(c) Shashi Tharoor (d) Shobha De(e) Aravind Adiga

5. Under the RTI Act the time for disposal of request forinformation in cases concerning life and liberty is _________.(a) 30 days (b) 15 days(c) 10 days (d) 7 days(e) 48 hours

6. World ‘No Tobacco Day’ is observed every year on__________.(a) 31st January (b) 31st May(c) 31st July (d) 31st October(e) 31st December

7. Which of the following is not a Consumer Right as perConsumer Protection Act 1986?(a) Right to Consumer Education(b) Right to Seek Redressal(c) Right to Safety(d) Right to be Informed(e) Right to Negotiate

Max. Marks : 50 No. of Qs. 50 Time : 30 min. Date : ........./......../................

8. Which of the following is not a Public Sector Bank?(a) Corporation Bank (b) United Bank of India(c) Vijaya Bank (d) Bank of Maharashtra(e) Federal Bank

9. A form of intentional weather modification by changing totrying the amount or type of precipitation that falls from cloudsby dispersing substances into air that serve as cloudcondensation is called ___________.(a) Cloud Computing (b) Cloud Technology(c) Cloud Seeding (d) Cloud Control(e) Cloud Engineering

10. The North Atlantic Treaty Organisation (NATO) isheadquartered at _________.(a) Camp David, Maryland USA(b) Geneva(c) New York(d) Sweden(e) Brussels

11. Which of the following is the abbreviated name of a ruralsports initiative introduced by the Government of India topromote youth and social development through sports?(a) NLMP (b) PYKKA(c) CGHS (d) AABY(e) SGSRY

12. National Sample Survey Organisation is an organisation of(a) Ministry of statistics and programme implementation(b) Planning Commission(c) Home Affairs(d) RBI(e) None of these

13. License to establish a commercial bank in India is issuedby—(a) Ministry of Finance (b) Registrar of companies(c) Reserve Bank of India (d) Planning Commission(e) None of these

RESPONSE

GRID

1. a b c d e 2. a b c d e 3. a b c d e 4. a b c d e 5. a b c d e6. a b c d e 7. a b c d e 8. a b c d e 9. a b c d e 10. a b c d e11. a b c d e 12. a b c d e 13. a b c d e

9696Section Test :General & Financial

Awareness

Page 210: yoursmahboob.wordpress.com SBI · yoursmahboob.wordpress.com iii P 101 Speed Tests for SBI Bank Clerk Exam 101 Speed Tests for SBI Bank Clerk Exam is revised and updated edition on

yoursmahboob.w

ordpress.com

202 SPEED TEST 96

14. On which one of the following issues can SEBI penalize anycompany in India?(A) Violation of Banking Regulation Act.(B) Violation of foreign portfolio investment guidelines.(C) For violation of Negotiable Instrument Act.(a) Only (A) (b) All (A), (B) & (C)(c) Only (A) & (B) (d) Only (B) & (C)(e) Only (B)

15. Which one the following is the west flowing river?(a) Narmada (b) Krishna(c) Godavari (d) Cauvery(e) Ganga

16. Which of the following best defines a floating-rate bond?(a) A bond with a fixed interest rate and has better yield

than varying interest rate bond(b) A bond with a fixed interest rate and has lower yield

than varying interest rate bond(c) A bond with a varying interest rate and has better yield

than fixed interest rate bond(d) A bond with a varying interest rate and has lower yield

than fixed interest rate bond(e) None of these

17. Once a Budget has been presented in the Parliament, thegovernment has to get all money bills related to the unionbudget passed within __?(a) 30 Days (b) 60 Days (c) 75 Days(d) 90 Days (e) None of these

18. Which among the following regulate the commodity marketsin India?1. RBI2. SEBI3. Forward Market CommissionChoose the correct option from the codes given below:(a) Only 1 & 2 (b) Only 2 & 3 (c) Only 1 & 3(d) Only 3 (e) None of these

19. The credit policy of a bank does not consists of?(a) Lending policies(b) Quality control(c) Loan product mix(d) Advertising of loan products(e) None of these

RESPONSE

GRID

14. a b c d e 15. a b c d e 16. a b c d e 17. a b c d e 18. a b c d e19. a b c d e 20. a b c d e 21. a b c d e 22. a b c d e 23. a b c d e24. a b c d e 25. a b c d e 26. a b c d e

20. India vision-2020 is prepared by(a) S.C. Gupta Committee(b) Panth Committee(c) Malhotra Commitee

(d) Narasimha Committee

(e) Kelkar Committee

21. The National Maritime Day of India is celebrated on whichday?

(a) April 3 (b) April 4

(c) April 5 (d) April 6

(e) None of these22. The "Mullaperiyar Dam" has been a subject of controversy

between which of the following two states?

(a) U.P and Uttarakhand

(b) Kerala & Tamil Nadu(c) Telangana and Andhra Pradesh

(d) Kerala & Karnataka

(e) None of these

23. The headquarters of the Confederation of Indian Industry(CII) is located at_____.

(a) Mumbai (b) Chandigarh

(c) New Delhi (d) Chennai

(e) None of these24. Which of the following is India's first state to link government

colleges to national cancer grid?

(a) Rajasthan (b) Maharashtra

(c) Madhya Pradesh (d) Odisha Hide(e) None of these

25. Which of the following villages has become the first liquor-

free village in Rajasthan?

(a) Dausa (b) Kachhabali(c) Churu (d) Achalpura

(e) None of these

26. Kakrapara Atomic Power Station' is located in which state

of India?(a) Gujarat (b) Karnataka

(c) Maharashtra (d) Rajasthan

(e) None of these

Page 211: yoursmahboob.wordpress.com SBI · yoursmahboob.wordpress.com iii P 101 Speed Tests for SBI Bank Clerk Exam 101 Speed Tests for SBI Bank Clerk Exam is revised and updated edition on

yoursmahboob.w

ordpress.com

203SPEED TEST 96

RESPONSE

GRID

27. a b c d e 28. a b c d e 29. a b c d e 30. a b c d e 31. a b c d e32. a b c d e 33. a b c d e 34. a b c d e 35. a b c d e 36. a b c d e

37. a b c d e 38. a b c d e 39. a b c d e 40. a b c d e 41. a b c d e

27. Which committee has recommended change in the structureand ecosystem in the Indian cricket board?(a) Kirit Parikh committee(b) RM Lodha committee(c) Bibek Debroy committee(d) Naresh Chandra committee(e) None of these

28. Shuklaphanta Wildlife Reserve' is located in which country?(a) Nepal (b) Bhutan(c) Myanmar (d) Sri Lanka(e) None of these

29. Which of the following virus is responsible for diarrhoeaamong infants and young children?(a) Parvovirus (b) Norwalk virus(c) Megavirus (d) Rotavirus(e) None of these

30. The International Day of Remembrance of the Victims ofSlavery and the Transatlantic Slave Trade", is observed onwhich date?(a) March 23 (b) March 24(c) March 25 (d) March 26(e) None of these

31. Swayam Shikshan Prayog (SSP) is an organisation whichaims to:(a) promote empowerment of women(b) protect the rights of children(c) protect the human rights of people(d) take care of Mentally or Physically Challenged Persons(e) None of these

32. What does BEPZA stands for?(a) Bangladesh Export Processing Zones Authority(b) Brazil Export Processing Zones Authority(c) Belgium Export Processing Zones Authority(d) Bahamas Export Processing Zones Authority(e) None of these

33. Which is the India's oldest paramilitary force?(a) Indian Coast Guard(b) Special Frontier Force(c) Assam Rifles(d) None of the above(e) None of these

34. Gurmeet Singh is associated with which sports?(a) Race walk (b) Swimming(c) Hockey (d) Kabaddi(e) None of these

35. Which is the first bank to commence the sale of Indian GoldCoin (IGC) in the domestic market?(a) State Bank of India(b) Indian Overseas Bank(c) Punjab National Bank(d) Bank of Baroda(e) None of these

36. Which organisation is providing loan support for "MadhyaPradesh Citizen Access to Responsive Services Project"?(a) World Bank (WB)(b) International Monetary Fund (IMF)(c) Asian Development Bank (ADB)(d) Asian Infrastructure Investment Bank (AIIB)(e) None of these

37. Which is the National Intelligence Agency of Israel?(a) GRU (b) NSIS(c) CISEN (d) Mossad(e) None of these

38. Which organisation has recently released the e-Atlas ofGender Inequality in Education?(a) UNESCO (b) UNICEF(c) UNFPA (d) UNSDN(e) None of these

39. Songkran festival is celebrated in which country?(a) India (b) Thailand(c) Indonesia (d) China(e) None of these

40. India's first contact-less mobile payment solution "iTap" hasbeen launched by which bank?(a) State Bank of India(b) HDFC Bank(c) Axis Bank Ltd(d) ICICI Bank(e) None of these

41. Which state government has launched awarenessprogramme to protect sparrows?(a) Madhya Pradesh (b) Uttar Pradesh(c) Odisha (d) Punjab(e) None of these

Page 212: yoursmahboob.wordpress.com SBI · yoursmahboob.wordpress.com iii P 101 Speed Tests for SBI Bank Clerk Exam 101 Speed Tests for SBI Bank Clerk Exam is revised and updated edition on

yoursmahboob.w

ordpress.com

204 SPEED TEST 96

RESPONSE

GRID

42. a b c d e 43. a b c d e 44. a b c d e 45. a b c d e 46. a b c d e

47. a b c d e 48. a b c d e 49. a b c d e 50. a b c d e

42. What is the new name of the LED based Domestic EfficientLighting Programme (DELP)?(a) Jyoti (b) Ujala(c) Disha (d) Prakash

43. Who is the first Indian classical musician to perform atUnited Nations?(a) Pt. Bhimsen Joshi (b) Pt. Jasraj(c) Ustad Jakir Hussain (d) MS Subbulakshmi(e) None of these

44. Which of the following two seas surround the CrimeanPeninsula?(a) Black Sea and Sea of Azov(b) Wadden Sea and Black Sea(c) Caspian Sea and Sea of Azov(d) Black Sea and Mediterranean Sea(e) None of these

45. Which of the following is not a correct statement aboutInfrasound?(a) Infrasound has a frequency lower than 50 hertz(b) Infrasound is produced / detected by elephants and

whales(c) Infrasound can travel longer distances than high

frequency sound waves(d) All are correct(e) None of these

46. Which bank has launched iWork@home programme for itswomen employees?(a) Yes bank (b) Exim Bank(c) ICICI Bank (d) Axis Bank Ltd.(e) None of these

47. Which state has launched Mukhyamantri Santwana HarishYojana?(a) Himachal Pradesh(b) Tamil Nadu(c) Karnataka(d) Odisha(e) None of these

48. Which of the following banks will become the India's firstSmall Finance Bank (SFB)?(a) Ujjivan Financial Services Pvt. Ltd(b) Capital Local Area Bank Ltd(c) Au Financiers Ltd(d) Janalakshmi Financial Services Pvt. Ltd(e) None of these

49. India's first solar powered ferry will come up in which state?(a) Gujarat(b) Kerala(c) Goa(d) Andhra Pradesh(e) None of these

50. India's first self cleaning smart toilets have been installed inwhich city?(a) Chennai(b) Bengaluru(c) Kolkata(d) Mumbai a(e) None of these

Page 213: yoursmahboob.wordpress.com SBI · yoursmahboob.wordpress.com iii P 101 Speed Tests for SBI Bank Clerk Exam 101 Speed Tests for SBI Bank Clerk Exam is revised and updated edition on

yoursmahboob.w

ordpress.com

Part-I : Numerical Ability

DIRECTIONS (Qs. 1-15): What will come in place of the questionmark (?) in the following questions.

1.5 4 11of of of 848=?

11 5 6(a) 216 (b) 222(c) 208 (d) 212(e) None of these

2. 1.4% of 750 + 2.2% of 480 = ?(a) 21.06 (b) 21.16(c) 20.88 (d) 21.18(e) None of these

3.3 2

of 116 of 87=?4 3

(a) 31 (b) 27(c) 29 (d) 26(e) None of these

4. 6.96 ÷ 1.2 – 18.24 ÷ 7.6 = ?(a) 3.4 (b) 3.14(c) 3.04 (d) 3.24(e) None of these

5. 32.25 × 2.4 × 1.6 = ?(a) 128.84 (b) 123.84(c) 112.88 (d) 112.84(e) None of these

6. 136% of 250 + ? % of 550 = 670(a) 64 (b) 55(c) 56 (d) 65(e) None of these

7. 448 ÷ 16 × 35 = ?(a) 850 (b) 890(c) 950 (d) 980(e) None of these

8.314 × 25 – 5 ?

24 5 8 9

(a)9

164

(b)6475

(c)11

164

(d)11

175

(e) None of these

9. 78.45 + 128.85 + 1122.25 = ?(a) 1329.55 (b) 1239.55(c) 1329.45 (d) 1239.45(e) None of these

10. 8729 – 4376 + 1245 = ? + 2785(a) 2713 (b) 2823(c) 2833 (d) 2733(e) None of these

11.2 5 717 4 ? 465 8 8

(a)3

325

(b)3

335

(c)2335

(d)2325

(e) None of these12. 5616 ÷ 18 ÷ 8 = ?

(a) 36 (b) 76(c) 49 (d) 39(e) None of these

13. 420 ÷ 28 × 288 ÷ 32 = ?(a) 235 (b) 236(c) 138 (d) 132(e) None of these

14. 222 ? 516(a) 1029 (b) 1024(c) 1124 (d) 1128(e) None of these

15. 45% of 660 + 28% of 450 = ?(a) 413 (b) 428(c) 423 (d) 418(e) None of these

DIRECTIONS (Qs. 16–20) : What will come in place of thequestion mark (?) in the following number series.16. 12 16 24 40 ?

(a) 76 (b) 72(c) 84 (d) 88(e) None of these

17. 9 19 39 79 ?(a) 139 (b) 129(c) 159 (d) 149(e) None of these

Max. Marks : 100 No. of Qs. 100 Time : 1 hr. Date : ........./......../................

9797Prelim Test - 1

Page 214: yoursmahboob.wordpress.com SBI · yoursmahboob.wordpress.com iii P 101 Speed Tests for SBI Bank Clerk Exam 101 Speed Tests for SBI Bank Clerk Exam is revised and updated edition on

yoursmahboob.w

ordpress.com

SPEED TEST 97206

18. 8 17 42 91 ?(a) 170 (b) 142(c) 140 (d) 172(e) None of these

19. 7 8 18 57 ?(a) 244 (b) 174(c) 186 (d) 226(e) None of these

20. 3840 960 240 60 ?(a) 20 (b) 18(c) 12 (d) 22(e) None of these

21. 75% of a number is equal to three seventh of another number.What is the ratio between the first number and the secondnumber respectively?(a) 4 : 7 (b) 7 : 4(c) 12 : 7 (d) 7 : 12(e) None of these

22. A 275 metre long train crosses a platform of equal length in33 seconds. What is the speed of the train in kmph ?(a) 66 (b) 60(c) 64 (d) 72(e) None of these

23. What is compound interest accrued on an amount of 45,000in two years at the rate of 9 p.c.p.a?(a) ` 8,600 (b) ` 8,565.40(c) ` 8,464.50 (d) ` 8,540(e) None of these

24. If the fractions 9 7 5 4, , ,

11 9 6 5and

1113

are arranged in

ascending order, which one will be the fourth?

(a)9

11(b)

79

(c)56

(d)45

(e)1113

25. Srikant and Vividh started a business investing amounts of` 1, 85,000 and 2, 25,000 respectively, If Vividh’s share inthe profit earned by them is 9,000, what is the total profitearned by them together?(a) ` 17,400 (b) ` 16,400(c) ` 16,800 (d) ` 17,800(e) None of these

26. Present ages of father and son are in the ratio of 6 : 1respectively. Four years after the ratio of their ages willbecome 4 : 1 respectively. What is the son’s present age?(a) 10 years (b) 6 years(c) 4 years (d) 8 years(e) None of these

27. 65% of a number is more than its 25

th by 140. What is 30%

of that number?(a) 186 (b) 168(c) 164 (d) 182(e) None of these

28. Number obtained by interchanging the digit of a two digitnumber is more than the original number by 27 and the sumof the digits is 13. What is the original number?(a) 58 (b) 67(c) 76 (d) 85(e) None of these

29. 22 men can complete a job in 16 days. In how many days will32 men complete that job?(a) 14 (b) 12(c) 16 (d) 9(e) None of these

30. A, B, C, D and E are five consecutive odd numbers. Averageof A and C is 59. What is the smallest number?(a) 65 (b) 63(c) 61 (d) 57(e) None of these

Direction (Qs. 31-35) : Study the following table carefully andanswer accordingly :

The distribution of marks (out of 150) obtained by 180 students ineach of the five subjects.

Marks Sub 0-29 30-59 60-89 90-119 120-150

Maths 22 47 74 25 12

Science 39 38 67 22 14

Hindi 19 59 47 36 19

English 24 41 58 34 23

Geography 42 32 52 41 13Average of five subjects

27 45 60 31 17

31. If for passing, the student has to obtain minimum 60% marksin the average of five subjects, how many students willpass?(a) 108 (b) 58(c) 48 (d) 72(e) None of these

32. How many students will pass in Geography if minimumpassing marks is 40%?(a) 74 (b) 106(c) 96 (d) Can’t say(e) None of these

33. How many students have obtained 60 or more marks in atleast one of the five subjects?(a) 111 (b) 103(c) 108 (d) 106(e) Data inadequate

34. If the criteria for distinction is minimum 75% marks in Maths,how many students will get distinction?(a) 37 (b) 27(c) 12 (d) Can’t say(e) None of these

Page 215: yoursmahboob.wordpress.com SBI · yoursmahboob.wordpress.com iii P 101 Speed Tests for SBI Bank Clerk Exam 101 Speed Tests for SBI Bank Clerk Exam is revised and updated edition on

yoursmahboob.w

ordpress.com

SPEED TEST 97 20735. The no. of students who obtained more than or equal to

40% marks in Science is what per cent less than that ofthose who scored less than 60% in Hindi?(a) 17.60% (b) 15.40%(c) 19.80% (d) 24.30%(e) None of these

Part-II : Reasoning Ability36. How many meaningful three letter English words can be

formed with the letters AER, using each letter only once ineach word ?(a) None (b) One(c) three (d) two(e) Four

37. Each vowel of the word ADJECTIVE is substituted with thenext letter of the English alphabetical series, and eachconsonant is substituted with the letter preceding it. Howmany vowels are present in the new arrangement ?(a) Four (b) One(c) Two (d) Three(e) None of these

38. In a certain code ‘na pa ka so’ means ‘birds fly very high’,‘ri so la pa’ means ‘birds are very beautiful’ and ‘ti me kabo’ means ‘the parrots could fly’. Which of the following isthe code for ‘high’in that language ?(a) na (b) ka(c) bo (d) so(e) None of these

39. If the digits in the number 86435192 are arranged inascending order, what will be the difference between thedigits which are second from the right and fourth from theleft in the new arrangement ?(a) One (b) Two(c) Three (d) Four(e) None

40. If it is possible to make only one meaningful word with theThird, Seventh, Eighth and Tenth letters of the wordCOMPATIBILITY, which of the following would be the lastletter of that word ? If no such word can be made, give ‘X’as your answer and if more than one such word can beformed, give your answer as ‘Y’.(a) I (b) B(c) L (d) X(e) Y

41. In a certain code FINE is written HGPC. How is SLIT writtenin that code ?(a) UTGR (b) UTKR(c) TUGR (d) RUGT(e) None of these

42. If in a certain language LATE is coded as 8 & 4 $ and HIREis coded as 7*3$ then how will HAIL be coded in the samelanguage ?(a) 7 & 8* (b) &7*8(c) 7*& 8 (d) 7&*8(e) None of these

43. Four of the following five are alike in a certain way and soform a group. Which is the one that does not belong to thatgroup ?(a) Stem (b) Tree(c) Root (d) Branch(e) Leaf

44. If ‘Apple’ is called ‘Orange’, ‘Orange’ is called ‘Peach’,‘Peach’ is called ‘Patato’, ‘Potato’ is called ‘Banana’,‘Banana’ is called ‘Papaya’ and ‘Papaya’ is called ‘Guava’,which of the following grows underground ?(a) Potato (b) Guava(c) Apple (d) Banana(e) None of these

45. How many such pairs of letters are there in word ENGLISH,each of which has as many letters between its two letters asthere are between them in the English alphabets ?(a) None (b) One(c) Two (d) Three(e) More than three

DIRECTIONS (Qs. 46-50) : In each of the questions below aregiven three statements followed by two conclusions numbered Iand II. You have to take the given statements to be true even ifthey seem to be at variance from commonly known facts. Readboth of the conclusions and then decide which of the givenconclusions logically follows from the given statementsdisregarding commonly known facts.

Read the statements and the conclusions which follow it and giveanswer(a) If only conclusion I is true.(b) If only conclusion II is true.(c) If either conclusion I or conclusion II is true.(d) If neither conclusion I nor conclusion II is true.(e) If both conclusions I and II are true.46. Statements :

All stars are suns.Some suns are planets.All planets are satellites.Conclusions :I. Some satellites are stars.II. No star is a satellite.

47. Statements :All curtains are rods.Some rods are sheets.Some sheets are pillows.Conclusions:I. Some pillows are rods.II. Some rods are curtains.

48. Statements :All switches are plugs.Some plugs are bulbs.All bulbs are sockets.Conclusions:I. Some sockets are plugs.II. Some plugs are switches.

Page 216: yoursmahboob.wordpress.com SBI · yoursmahboob.wordpress.com iii P 101 Speed Tests for SBI Bank Clerk Exam 101 Speed Tests for SBI Bank Clerk Exam is revised and updated edition on

yoursmahboob.w

ordpress.com

SPEED TEST 97208

49. Statements :All fishes are birds.All birds are rats.All rats are cows.Conclusions :I. All birds are cows.II. All rats are fishes.

50. Statements :Some walls are windows.Some windows are doors.All doors are roofs.Conclusions :I. Some doors are walls.II. No roof is a window.

DIRECTIONS (Qs. 51-55) : Read the following informationcarefully and answer the questions, which follow :

‘A - B’ means ‘A is father of B’.‘A + B’ means ‘A is daughter of B’.‘A ÷ B’ means ‘A is son of B’.‘A × B’ means ‘A is wife of B’.51. How is P related to T in the expression ‘P + S – T’ ?

(a) Sister (b) Wife(c) Son (d) Daughter(e) None of these

52. In the expression ‘P × Q – T’ how is T related to P ?(a) Daughter (b) Sister(c) Mother (d) Can’t be determined(e) None of these

53. Which of the following means T is wife of P ?(a) P × S ÷ T (b) P ÷ S × T(c) P – S ÷ T (d) P + T ÷ S(e) None of these

54. Which of the following means P is grandson of S ?(a) P + Q – S (b) P ÷ Q × S(c) P ÷ Q + S (d) P × Q ÷ S(e) None of these

55. In the expression ‘P + Q × T’ how is T related to P ?(a) Mother (b) Father(c) Son (d) Brother(e) None of these

DIRECTIONS (Qs. 56-60) : In each question a group of letters isgiven followed by four combinations of number/symbol numbered(a), (b), (c) and (d). Letters are to be coded as per the scheme andconditions given below. You have to find out the serial number ofthe combination, which represents the letter group. Serialnumber of that combination is your answer. If none of thecombinations is correct, your answer is (e) i.e. None of these.

Letters Q M S I N G D K A L P R B J ENumber/ 7 @ 4 # % $ 6 1 2 £ 5 * 9 8 3Symbol

Conditions :(i) If the first letter is a consonant and the last a vowel, both are

to be coded as the code of the vowel.(ii) If the first letter is vowel and the last a consonant, the codes

for the first and the last are to be interchanged.

(iii) If no vowel is present in the group of letters, the second andthe fifth letters are to be coded as ©.

56. BARNIS(a) 9 2 * % # 4 (b) 9 2 4 # * %(c) 9 2 * # % 9 (d) 4 2 * # % 4(e) None of these

57. DMBNIA(a) 6 @ 9 % # 2 (b) 2 @ 9 % # 6(c) 2 @ 9 % # 6 (d) 2 @ 9 % # 2(e) None of these

58. IJBRLG(a) # 8 9 * £ $ (b) # 8 9 * £ #(c) $ 8 9 * £ # (d) $ 8 9 * £ $(e) None of these

59. BKGQJN(a) 9 © $ 7© % (b) © 9 $ 7 % ©(c) 9 1 $ 7 8 % (d) % 1 $ 7 8 9(e) None of these

60. EGAKRL(a) # £ $ 2 1 * (b) £ $ 2 1 * 3(c) £ $ 2 1 * # (d) # £ $ 2 1 #(e) None of these

DIRECTIONS (Qs. 61-66) : Study the following informationcarefully to answer these questions.

Eight persons A, B, C, D, E, F, G and H work for three differentcompanies namely X, Y and Z. Not more than three persons workfor a company. There are only two ladies in the group who havedifferent specialisations and work for different companies. Of thegroup of friends, two have specialisation in each HR, Finance andMarketing. One member is an engineer and one is a doctor. H isan HR specialist and works with a Marketing specialist B whodoes not work for company Y. C is an engineer and his sisterworks in company Z. D is a specialist in HR working in companyX while her friend G is a finance specialist and works for companyZ. No two persons having the same specialisation work together.Marketing specialist F work for company Y and his friend A whois a Finance expert works for company X in which only twospecialists work. No lady is a marketing specialist or a doctor.61. Which of the following combinations is correct ?

(a) C - Z - Engineer (b) E - X - Doctor(c) H – X – HR (d) C – Y – Engineer(e) None of these

62. For which of the following companies does C work?(a) Y (b) X(c) Z (d) Data inadequate(e) None of these

63. Which of the following pairs represents the two ladies inthe group ?(a) A and D (b) B and D(c) D and G (d) Data inadequate(e) None of these

64. Which of the following represents the pair working in thesame company ?(a) D and C (b) A and B(c) A and E (d) H and F(e) None of these

Page 217: yoursmahboob.wordpress.com SBI · yoursmahboob.wordpress.com iii P 101 Speed Tests for SBI Bank Clerk Exam 101 Speed Tests for SBI Bank Clerk Exam is revised and updated edition on

yoursmahboob.w

ordpress.com

SPEED TEST 97 20965. Who amongst the friends is a doctor ?

(a) H (b) E(c) C (d) Either E or C(e) None of these

66. Four of the following five are alike in a certain way and soform a group. Which is the one that does not belong to thegroup?(a) 56 (b) 35(c) 49 (d) 42(e) 51

67. If it is possible to make only one meaningful English wordfrom the second, the fifth, the seventh and the eighth lettersof the word PHYSICAL, using each letter only once, secondletter of that word is your answer. If more than one suchword can be formed your answer is M. If no such word canbe formed your answer is N.(a) I (b) A(c) L (d) M(e) N

DIRECTIONS (Qs. 68-70) : Study the following paragraph andanswer the questions that follow :

Culture in the literal sense of the term is the art andmanifestation of human intellectual achievement regardedcollectively. It refers to the cumulative deposit of knowledge,experience, beliefs, values, attitudes, meanings and materialobjects and possessions acquired by a group of people in thecourse of generations through individual and group striving. Theculture of India refers to the religions, beliefs, customs, traditions,languages, ceremonies, arts, values and the way of life of thepeople of India. Heritage on the other hand denotes or relates tothings of special architectural, historical or natural value that arepreserved and sustained as valued objects such as historicbuildings and monuments as valued objects such as historicbuildings and monuments68. Which of the following conclusion which can be drawn

from the facts stated in the above passage?(a) Culture is an imaginary term by the people of that

country.(b) Culture is related with our political activities.(c) Culture is an integrated sense of art, knowledge,

customs, beliefs, languages, values life-styles etc.(d) Culture is mostly affected with economic activities and

thoughts.(e) None of these

69. Which statement strengthens the conclusion of theparagraph?(a) Life-style, clothing and fooding is also a part of culture.(b) Geographical situation does not affect the culture.(c) Scientific knowledge is also included in culture.(d) Culture does not affect one’s life.(e) None of these

70. Which statement weakens the conclusion of the paragraph?(a) Our heritage is also a part of our culture.(b) Methods of worship is an important part of culture.(c) Culture is a hypothetical and imaginary term. Which

can not be determined by some points.(d) Culture has an extensive sense which is determined by

life-style, custums, beliefs, languages, values etc.(e) None of these

Part-III : English Language

DIRECTIONS (Qs. 71-80) : Read the following passage carefullyand answer the questions given below it. Certain words/phraseshave been printed in bold to help you locate them while answeringsome of the questions.

Once upon a time a dishonest king had a man called theValuer in his court. The Valuer set the price which ought to bepaid for horses and elephants and the other animals. He also setthe price on jewellery and gold, and things of that kind. This manwas honest and just, and set the proper price to be paid to theowners of the goods. The king, however, was not pleased withthis Valuer, because he was honest. "If I had another sort of aman as Valuer, I might gain more riches," he thought.

One day the king saw a stupid, miserly peasant come intothe palace yard. The king sent for the fellow and asked him if hewould like to be the Valuer. The peasant said he would like theposition. So the king had him made Valuer. He sent the honestValuer away from the palace.

Then the peasant began to set the prices on horses andelephants, upon gold and jewels. He did not know their value, sohe would say anything he chose. As the king had made him Valuer,the people had to sell their goods for the price he set. By and bya horse-dealer brought five hundred horses to the court of thisking. The Valuer came and said they were worth a mere measureof rice. So the king ordered the horse-dealer to be given the measureof rice, and the horses to be put in the palace stables.

The horse-dealer then went to see the honest man who hadbeen the Valuer, and told him what had happened. "What shall Ido?" asked the horse-dealer. "I think you can give a present tothe Valuer which will make him do and say what you want him todo and say," said the man. "Go to him and give him a fine present,then say to him: "You said the horses are worth a measure of ricebut now tell what a measure of rice is worth! Can you value thatstanding in your place by the king?" If the says he can, go wthhim to the king, and I will be there, too."

The horse-dealer thought this was a good idea. So he gavea fine present to the Valuer, and said what the other man had toldhim to say., The stupid Valuer took the present, and said: "Yes, Ican go before the king with you and tell what a measure of rice isworth. I can value that now." Well, let us go at once," said thehorse-dealer. So they went before the king and his ministers inthe palace.

The horse-dealer bowed down before the king, and said: "OKing, I have learned that a measure of rice is the value of my fivehundred horses. But will the king be pleased to ask the Valuerwhat is the valuer of the measure of rice". The king, not knowingwhat had happened, asked, "How now, Valuer, what are fivehundred horses worth?" "A measure of rice, O King!" said he."Very good, then! If five hundred horses are worth a measure ofrice, what is the measure of rice worth?" "The measure of rice isworth your whole city," replied the foolish fellow.

The ministers clapped their hands, laughing, and saying,"What a foolish Valuer! How can such a man hold that office? Weused to think this great city was beyond price, but this man saysit is worth only a measure of rice, "Then the king was ashamed,and drove out the foolish fellow. "I tried to please the king bysetting a low price on the horses, and now see what has happenedto me!" said the Valuer, as he ran away from the laughing crowd.

Page 218: yoursmahboob.wordpress.com SBI · yoursmahboob.wordpress.com iii P 101 Speed Tests for SBI Bank Clerk Exam 101 Speed Tests for SBI Bank Clerk Exam is revised and updated edition on

yoursmahboob.w

ordpress.com

SPEED TEST 97210

71. Who did the king appoint as the new Valuer?(a) A minsister (b) A horse merchant(c) Himself (d) A stingy peasant(e) None of these

72. Why was the king not happy with the old Valuer?(a) As the Valuer was not good at his work(b) As he had dishonoured the king(c) As the Valuer had been dishonest with the king about

the prices that he set for goods(d) As the king believed that he was not earning much

because of the Valuer's honesty(e) None of these

73. Which of the following words can be used to describe theking?(1) Smart (2) Dishonest(3) Cheat(a) Only (1) (b) Only (2)(c) Only (2) and (3) (d) Only (1) and (3)(e) All the three (1), (2) and (3)

74. What can possibly be the moral of the story?(a) Slow and steady wins the race(b) Change is the only permanent thing in life(c) An honest answer is the sign of true friendship(d) Haste makes waste(e) No legacy is so rich as honesty

75. Why did the Ministers laugh at the new Valuer?(a) As he had sold the king's city at a very low price(b) As he had displayed his stupidity by quoting an

abysmally low price on the king's city(c) As he had cheated the horse dealer(d) As he had not claculated the price of the five hundred

horses correctly(e) None of these

76. What did the new Valuer do when he got the present fromthe horse dealer?(a) He accepted the present and resigned from his post as

was requested by the horse-dealer(b) He accepted the present and agreed to state the worth

of a measure of rice in the presence of the King(c) He accepted the present and immediately returned the

horse-dealer's horses(d) He refused to accept the present from the horse-dealer

and asked him to leave the premises(e) None of these

77. Which of the following can be said about the old Valuer?(1) He was honest. (2) He was intelligent.(3) He was revengeful.(a) Only (1) (b) Only (3)(c) Only (1) and (2) (d) Only (1) and (3)(e) All the three (1), (2) and (3)

78. What was the worth of a measure of rice according to thenew Valuer?(a) The king's entire city(b) The king's life(c) Two horses(d) Not mentioned in the passage(e) None of these

79. Why did the horse-dealer go to meet the old Valuer?(a) As the new Valuer had set a very inappropriate price

for his five hundred horses(b) As his five hundred horses were stolen from him by

the king(c) As he was a very good friend of the old Valuer(d) As the king head requested him to do so(e) None of these

80. What advice did the old Valuer give to the horse-dealer?(a) He asked the horse-dealer to inquire with the king about

the worth of a measure of rice(b) He asked the horse-dealer to bribe the new Valuer and

get his horses back(c) He asked the horse-dealer to forget about his horses

and go on with his life(d) He asked the horse-dealer to publicize his plight and

thus get his horses back(e) None of these

DIRECTIONS (Qs. 81-85) : Rearrange the following sixsentences (A), (B), (C), (D), (E) and (F) in the proper sequence toform a meaningful paragraph; then answer the questions givenbelow them.

(A) The woodcutter thankfully broke off from work and sat downto eat the delicious meal that his wife had sent for him.

(B) He was in a good mood that particular morning and soonstarted singing as he swung his axe at the log of wood infront of him.

(C) After he had eaten his meal and taken rest for a while thewoodcutter got back to work.

(D) The hours passed and the sun became hotter than ever andvery soon perspiraton started breaking out on thewoodcuter's hands and face.

(E) One hot summer's morning a woodcutter was hard at work,chopping wood into small pieces, so that he could sell themin the market.

(F) As it neared afternoon, his wife sent their little son to himwith food for the afternoon.

81. Which of the following should be the FOURTH sentenceafter rearrangement?(a) F (b) E(c) D (d) C(e) B

82. Which of the following should be the FIRSTsentence afterrearrangement?(a) A (b) B(c) C (d) D(e) E

83. Which of the following should be the SECOND sentenceafter rearrangement?(a) A (b) B(c) C (d) D(e) F

84. Which of the following should be the LAST (SIXTH)sentence after rearrangement?(a) A (b) B(c) C (d) D(e) E

Page 219: yoursmahboob.wordpress.com SBI · yoursmahboob.wordpress.com iii P 101 Speed Tests for SBI Bank Clerk Exam 101 Speed Tests for SBI Bank Clerk Exam is revised and updated edition on

yoursmahboob.w

ordpress.com

SPEED TEST 97 21185. Which of the following should be the THIRD sentence after

rearrangement?(a) A (b) B(c) C (d) D(e) E

DIRECTIONS (Qs. 86-88) : Which of the phrases (a), (b), (c) and(d) given below each sentence should replace the phrase printedin bold in the sentence to make it grammatically correct? If thesentence is correct as it is given and no correction is required,mark (e) as the answer.86. As it was already afternoon, Rohan decided to check out of

the hotel and go home.(a) for checking out (b) to checking out(c) to check outing (d) to checked out(e) No correction required

87. Five people which ignored an evacuation order were trappedin a mountain region encircled by a wildfire.(a) who ignored an (b) those ignoring an(c) who ignores a (d) that ignored a(e) No correction required

88. Since she was the most popular model on the ramp, shethought no end to herself.(a) no ending to herself (b) no ends of herself(c) no end of herself (d) no end with herself(e) No correction required

DIRECTIONS (Qs. 89-90) : In each question below, a sentencewith four words printed in bold type is given. These are numberedas (a), (b), (c) and (d). One of these four words printed in bold maybe either wrongly spelt or inappropriate in the context of thesentence. Find out the word which is wrongly spelt orinappropriate, if any. The number of that word is your answer. Ifall the words printed in bold are correctly spelt and alsoappropriate in the context of the sentence, mark (e) i.e., Allcorrect' as your answer.89. The city's fashion-conscious ladies (a)/ came together at a

city hotel to check out an exibition (b)/ by various (c)/designers (d)/ and labels. All correct (e)

90. The ministry's proposal (a)/ for an autonomous (b)/overarching authority (c)/ for higher education and researchwas finally approval. (d)/ All correct (e)

DIRECTIONS (Qs. 91-100) : In the following passage there areblanks, each of which has been numbered. These numbers areprinted below the passage and against each, five words aresuggested, one of which fits the blank appropriately. Find out theappropriate word in each case.

There was once a gardener who (91) care of the king'sgarden. In the garden, lived a family of monkeys. Seeing thegardener at work, the monkeys soon (92) to imitate him. As thegardener tended the plants and weeded and watered the garden,

he was amused to see that the monkeys also did (93) what he did.The gardener was happy that he had so much unpaid help.

One day the gardener wanted to (94) a fair in the city. Hehad an idea. He called the chief of the monkeys and said to him, "Ihave to go out for the day. Can you and your family water myplants like you (95) do? I promise you that if you help me I will(96) sweets for you from the fair.: The monkeys agreed. But afterthe gardener had left, they had a (97). How much water were theyto pour for each plant? then one of them said. "For plants with bigroots, we must pour (98) of water and for the ones with smallroots, we pour only a litle water." So, the monkeys (99) out eachplant and then pushed it back again after looking at the root. Asa result, many plants (100) and died. On his return, the gardenerrealized that he had been very foolish to trust a bunch of meremonkeys to do his job.91. (a) took (b) was

(c) great (d) handle(e) mended

92. (a) try (b) told(c) were (d) bent(e) learnt

93. (a) main (b) exactly(c) many (d) because(e) too

94. (a) call (b) make(c) stall (d) go(e) visit

95. (a) forcefully (b) hardly(c) usually (d) costly(e) truly

96. (a) ask (b) bring(c) got (d) throw(e) create

97. (a) party (b) time(c) answer (d) doubt(e) water

98. (a) body (b) many(c) lots (d) weight(e) quantity

99. (a) thrashed (b) saw(c) stick (d) pulled(e) splashed

100. (a) withered (b) crushed(c) killed (d) grew(e) smiled

Page 220: yoursmahboob.wordpress.com SBI · yoursmahboob.wordpress.com iii P 101 Speed Tests for SBI Bank Clerk Exam 101 Speed Tests for SBI Bank Clerk Exam is revised and updated edition on

yoursmahboob.w

ordpress.com

SPEED TEST 97212

Test Code : ............................... Time taken : ......................... Date : ..........................

RESPONSE SHEET

(a) (b) (c) (d) (e)

1.

2.

3.

4.

5.

6.

7.

8.

9.

10.

11.

12.

13.

14.

15.

16.

17.

18.

19.

20.

21.

22.

23.

24.

25.

(a) (b) (c) (d) (e)

51.

52.

53.

54.

55.

56.

57.

58.

59.

60.

61.

62.

63.

64.

65.

66.

67.

68.

69.

70.

71.

72.

73.

74.

75.

(a) (b) (c) (d) (e)

26.

27.

28.

29.

30.

31.

32.

33.

34.

35.

36.

37.

38.

39.

40.

41.

42.

43.

44.

45.

46.

47.

48.

49.

50.

(a) (b) (c) (d) (e)

76.

77.

78.

79.

80.

81.

82.

83.

84.

85.

86.

87.

88.

89.

90.

91.

92.

93.

94.

95.

96.

97.

98.

99.

100.

Page 221: yoursmahboob.wordpress.com SBI · yoursmahboob.wordpress.com iii P 101 Speed Tests for SBI Bank Clerk Exam 101 Speed Tests for SBI Bank Clerk Exam is revised and updated edition on

yoursmahboob.w

ordpress.com

Part-I : Numerical Ability

DIRECTIONS (Qs. 1-10): What will come in place of questionmark (?) in the following questions?

1.35 of

47 of

512

of 1015 = ?

(a) 220 (b) 340(c) 240 (d) 145(e) None of these

2. 1.5 × 0.025 + (?)2 = 0.1(a) 0.28 (b) 0.27(c) 0.25 (d) 0.235(e) None of these

3. 21.5 0.0225 ?(a) 0.3375 (b) 3.275(c) 32.75 (d) 0.0375(e) None of these

4. 0.0289 12 1.5 ?(a) 1.36 (b) 2.06(c) 13.90 (d) 14.80(e) None of these

5. 125% of 260 + ?% of 700 = 500(a) 32 (b) 56(c) 23 (d) 46(e) None of these

6. 45% of 750 – 25% of 480 = ?(a) 216 (b) 217.50(c) 245 (d) 236.50(e) None of these

7. 8.5 3.875 75 75 ?

(a) 4.9 (b) 3.6(c) 3.3 (d) 4.7(e) None of these

8. 39798 + 3798 + 378 = ?(a) 49532 (b) 43984(c) 43974 (d) 43576(e) None of these

9.7 3 13 7 1 ?11 11 2

(a)101311

(b)614

11

(c)91411

(d)171022

(e) None of these10. 1080 12 10 = ?

(a) 900 (b) 90(c) 120 (d) 12(e) None of these

11. The number zero (0) is surrounded by the same 2-digitnumber on both (left and right) the sides; for example, 25025,67067, etc. The largest number that always divides such anumber is(a) 7 (b) 11(c) 13 (d) 1001(e) None of these

12. The number of 3-digit number exactly divisible by 5 is(a) 181 (b) 180(c) 179 (d) 199(e) None of these

13. If a certain sum of money becomes double at simple interestin 12 years, what would be the rate of interest per annum ?

(a) 183

(b) 10

(c) 12 (d) 14(e) None of these

14. Three successive discounts of 10%, 12% and 15% amountto a single discount of(a) 36.28 % (b) 34.68%(c) 37 % (d) 32.68%(e) None of these

15. The ratio of the prices of two houses A and B was 4 : 5 lastyear. This year, the price of A is increased by 25% and thatof B by 50000. If their prices are now in the ratio 9 : 10, theprice of A last year was(a) ` 3,60,000 (b) ` 4,50,000(c) ` 4,80,000 (d) ` 5,00,000(e) None of these

Max. Marks : 100 No. of Qs. 100 Time : 1 hr. Date : ........./......../................

98

9898Prelim Test - 2

Page 222: yoursmahboob.wordpress.com SBI · yoursmahboob.wordpress.com iii P 101 Speed Tests for SBI Bank Clerk Exam 101 Speed Tests for SBI Bank Clerk Exam is revised and updated edition on

yoursmahboob.w

ordpress.com

SPEED TEST 98214

16. During a journey of 80 km a train covers first 60km with aspeed of 40 km/h and completes the remaining distance witha speed of 20 km/h. What is the average speed of the trainduring the whole journey?

(a) 30 km/h (b) 32 km/h

(c) 36 km/h (d) 40 km/h

(e) None of these

17. An aeroplane takes off 30 minutes later than the scheduledtime and in order to reach its destination 1500 km away intime, it has to increase its speed by 250 km/h from its usualspeed. Find its usual speed.(a) 1000 km/h (b) 750 km/h(c) 850 km/h (d) 650 km/h(e) None of these

18. A man arranges to pay off a debt of Rs 3,600 in 40 annualinstalments which form an AP. When 30 of the instalmentsare paid, he dies leaving one-third of the debt unpaid. Findthe value of the first instalment.(a) 55 (b) 53(c) 51 (d) 49(e) None of these

19. A tank 30 m long, 20 m wide and 12 m deep is dug in a field500 m long and 30 m wide. By how much will the level of thefield rise, if the earth dug out of the tank is evenly spreadover the field?(a) 0.33 m (b) 0.5 m(c) 0.25 m (d) 0.4 m(e) None of these

DIRECTIONS (Qs. 20-21) : Find the next term in the given seriesin each of the questions below.

20. 198, 194, 185, 169, ....(a) 136 (b) 144(c) 9 (d) 92(e) None of these

21. 6, 9, 7, 10, 8, 11, ....(a) 12 (b) 13(c) 9 (d) 14(e) None of these

DIRECTIONS (Qs. 22-28): Find out the approximate value whichshould replace the question mark (?) in the following questions.(You are not expected to find out the exact value).

22. 10609 7938.81 ?(a) 9200 (b) 81973.(c) 8553.3 (d) 8682.7(e) None of these

23. 18.4% of 656 + 12.7% of 864 = ?(a) 253 (b) 231 (c) 211(d) 241 (e) None of these

24. (98.4)2 + (33.6)2 = ?(a) 10812 (b) 18012 (c) 10910(d) 18102 (e) None of these

25. 8959 ? 4 5 = 26.35(a) 15 (b) 25 (c) 30(d) 17 (e) None of these

26. 3739 + 164 × 27 = ?(a) 10540 (b) 4000 (c) 8400(d) 8200 (e) None of these

27. Rajeev consistently runs 415 meters every day except onSunday when he runs 500 meters. How many kilometers willhe run in two weeks-? (in this question week starts fromMonday)(a) 5.98 kms. (b) 5.86 kms. (c) 5.96 kms.(d) 5.88 knis. (e) None of these

28. Amit got 44 marks in Hindi. 55 marks in Science. 77 marks inMaths. 79 marks in Social Science and 76 marks in English.The maximum marks of each subject are 100. How muchoverall percentage of marks did he get?(a) 66.2 (b) 64.2 (c) 72.2(d) 74.2 (e) None of these

DIRECTIONS (Qs. 29 - 30) : What will come in place of questionmark (?) in the following number series?

29. 7 11 19 35 67 (?)(a) 121 (b) 153 (c) 141(d) 133 (e) None of these

30. 5 6 10 19 35 (?)(a) 55 (b) 65 (c) 60(d) 70 (e) None of these

Directions (Qs. 31-35) : Study the following data carefully andanswer accordingly.

Following chart shows the number of students indifferent universities

IndraprasthaUni 13%

Delhi Uni35%

Hamdard14%

J.N.U20%

Jamia18%

Total no. of students = 120,000

Page 223: yoursmahboob.wordpress.com SBI · yoursmahboob.wordpress.com iii P 101 Speed Tests for SBI Bank Clerk Exam 101 Speed Tests for SBI Bank Clerk Exam is revised and updated edition on

yoursmahboob.w

ordpress.com

SPEED TEST 98 215Percentage of listeners of different FM channels in

National Capital Region

Radio M irchi

Radio City

Red FM

FM Gold

Rainbow

Indrapras tha 76% 72% 46% 54% 48%Hamdard 63% 64% 59% 47% 53%

JNU 52% 65% 64% 51% 54%DU 82% 44% 32% 35% 45%

Jamia 75% 32% 36% 52% 64%

FM ChannelsUnivers ities

31. How many students of JNU listen to Radio city?(a) 15200 (b) 15600(c) 14400 (d) 14600(e) None of these

32. The no. of Indraprastha students listening to Rainbow iswhat per cent of the no. of Jamia students listening FMGold?(a) 65 (b) 56(c) 68 (d) 58(e) None of these

33. From which of the following universities, the no. of studentsliking Red FM is minimum?(a) Indraprastha (b) Jamia(c) JNU (d) DU(e) Hamdard

34. How many students of Indraprastha and Jamia togetherlisten to Red FM?(a) 12562 (b) 12872(c) 14952 (d) 14272(e) None of these

35. Which of the following channels is the most popular amongthe students of Hamdard and JNU?(a) Radio Mirchi (b) Radio city(c) Red FM (d) FM Gold(e) Rainbow

Part-II : Reasoning Ability36. If ‘football’ is called cricket, ‘cricket’ is called ‘basketball’

‘basketball’ is called ‘badminton’, ‘badminton’ is called‘volleyball’, ‘volleyball’ is called ‘hockey’ and ‘hockey’ iscalled ‘golf’, which of the following games is not playedusing a ball ?(a) Volleyball (b) Basketball(c) Hockey (d) Cricket(e) None of these

37. If it is possible to make only one meaningful word with theFirst, Second, Third and Fifth letters of the wordTECHNOLOGY, which of the following would be the thirdletter of that word ? If no such word can be made, give ‘X’ asyour answer and if more than one such word can be formed,give your answer as ‘Y’.(a) C (b) T(c) N (d) X(e) Y

38. If each of the digits in the number 92581473 are arranged inascending order, what will be the difference between thedigits which are fourth from the right and third from the leftin the new arrangement ?(a) One (b) Two(c) Three (d) Four(e) None

39. In a certain code ‘ja ki mo pe’ means ‘at a frog’s leap’, ‘mo laki so’ means ‘take a leap ahead’ and ‘re bo ja na’ means‘insects are frog’s diet’. Which of the following is the codefor ‘at’ in that language ?(a) ja (b) pe(c) bo (d) re(e) None of these

40. If in a certain language WEAK is coded as 9%2$ and SKITis coded as #$7@, then how will WAIT be coded in thesame language ?(a) 9267 (b) 9276(c) 92 @ 6 (d) 9 @ 67(e) None of these

41. How many such pairs of letters are there in word SENDING,each of which has as many letters between its two letters asthere are between them in the English alphabets ?(a) None (b) One(c) Two (d) Three(e) More than three

42. Each vowel of the word GLADIOLUS is substituted withthe next letter of the English alphabetical series, and eachconsonant is substituted with the letter preceding it. Howmany vowels are present in the new arrangement ?(a) Four (b) One(c) Two (d) Three(e) None of these

DIRECTIONS (Qs. 43-47) : In each of these questions a group ofletters is given followed by four combinations of number/symbolnumbered (a), (b), (c) and (d). Letters are to be coded as per thescheme and conditions given below. You have to find out the serialnumber of the combination, which represents the letter group.Serial number of the combination is your answer. If none of thecombinations is correct, your answer is (e) i.e. ‘None of these’.

Letters M B D K Q L I R J S N P A E GNumber/ Symbol @ 3 7 % * 4 # 1 2 £ 8 5 9 $ 6

Conditions :(i) If the first letter is a vowel and the last a consonant, both are

to be coded as the code for the consonant.(ii) If the first letter is a consonant and the last a vowel, the

codes for the first and the last are to be interchanged.(iii) If no vowel is present in the group of letters, the first and

the last letters are to be coded as ©.43. GQRDBN

(a) ©* 173© (b) 6*1738(c) 6*1736 (d) 8*1738(e) None of these

44. IPEBQS(a) #5$3*£ (b) #53$*#(c) £5$3*£ (d) £5$3*#(e) None of these

Page 224: yoursmahboob.wordpress.com SBI · yoursmahboob.wordpress.com iii P 101 Speed Tests for SBI Bank Clerk Exam 101 Speed Tests for SBI Bank Clerk Exam is revised and updated edition on

yoursmahboob.w

ordpress.com

SPEED TEST 98216

45. RMAPSI(a) 1@95£# (b) 1@95£1(c) #@95£# (d) #@95£1(e) None of these

46. AREMQN(a) 91$@*8 (b) 81$@*8(c) 81$@*9 (d) 91$@*9(e) None of these

47. KJBPRD(a) 923517 (b) 723517(c) %23519 (d) ©2915©(e) None of these

DIRECTIONS (Qs. 48-52) : Read the following informationcarefully and answer the questions, which follow :

‘A – B’ means ‘A is daughter of B,.‘A + B’ means ‘A is wife of B’‘A ÷ B’ means ‘A is father of B’‘A × B’ means ‘A is son of B’.

48. In the expression ‘P × R – S’ how is P related to S ?(a) Father (b) Grandfather(c) Grandson (d) Sister(e) None of these

49. Which of the following means S is son-in-law of P?(a) P + R × S (b) P ÷ R × S(c) P + R ÷ S (d) P ÷ R + S(e) None of these

50. In the expression ‘P – Q + S’ how is S related to P?(a) Mother(b) Father(c) Brother(d) Cannot be determined(e) None of these

51. How is P related to S in the expression ‘P × R ÷ S’?(a) Brother (b) Wife(c) Son (d) Sister(e) None of these

52. How is S related to P in the expression ‘P + R ÷ S’?(a) Son (b) Daughter(c) Daughter- in - law (d) Sister(e) None of these

DIRECTIONS (Qs. 53-55) : Study the sets of numbers givenbelow and answer the questions, which follow :

972 682 189 298 75153. If one is added to the lowest number and two is added to

the highest number, what will be the difference between thesecond digit of the smallest number and third digit of thehighest number ?(a) 5 (b) 7(c) 9 (d) 8(e) None of these

54. If in each number, first and the last digits are interchanged,which of the following will be the third highest number ?(a) 972 (b) 682(c) 189 (d) 298(e) 751

55. If in each number, all the three digits are arranged indescending order, which of the following will be the thirdhighest number?(a) 972 (b) 682(c) 189 (d) 298(e) 751

DIRECTIONS (Qs. 56-60) : In each of the questions below aregiven three statements followed by two conclusions numbered Iand II. You have to take the given statements to be true even ifthey seem to be at variance from commonly known facts. Readboth of the conclusions and then decide which of the givenconclusions logically follows from the given statementsdisregarding commonly known facts.Read the statements and conclusions which follow it andGive answer (a) if only conclusion I is true.Give answer (b) if only conclusion II is true.Give answer (c) if either conclusion I or conclusion II is true.Give answer (d) if neither conclusion I nor conclusion II is true.Give answer (e) if both conclusions I and II are true.56. Statements :

No pen is a mobile.Some mobiles are bottles.All bottles are papers.Conclusions :I. Some papers are pens.II. All bottles are pens.

57. Statements :All computers are radios.All radios are televisions.Some televisions are watches.Conclusions :I. Some watches are computers.II Some televisions are computers.

58. Statements :Some desks are chairs.Some chairs are doors.Some doors are walls.Conclusions :I. Some walls are chairs.II. No chair is a wall.

59. Statements :All stars are fishes.Some fishes are moons.All moons are birds.Conclusions :I. Some birds are fishes.II. Some stars are moons.

60. Statements :All leaves are roots.All stems are roots.All roots are flowers.Conclusions :1. Some flowers are stems.II. Some flowers are leaves.

Page 225: yoursmahboob.wordpress.com SBI · yoursmahboob.wordpress.com iii P 101 Speed Tests for SBI Bank Clerk Exam 101 Speed Tests for SBI Bank Clerk Exam is revised and updated edition on

yoursmahboob.w

ordpress.com

SPEED TEST 98 217

DIRECTIONS (Qs. 61-67) : Study the following informationcarefully to answer these questions.

A group of people has six family members and an advocate.These are L, M, N, O, P, Q and R and having different professions.Each one of them is a journalist, businessman, architect, doctorand pilot but not necessarily in this order. There are three malesand three females in the family out of which there are two marriedcouples. M is a businessman and is the father of P. N is a housewifeand is daughter-in-law of O. L is neither a pilot nor a journalist. Ris an advocate. N is not the mother of P and O is not married to M.No lady is a journalist.61. Which of the following groups represents the three ladies

in the group ?(a) N, P, L (b) P, L, N(c) L, N, O (d) O, P, L(e) None of these

62. Who is married to Q ?(a) N (b) O(c) L (d) Can’t be determined(e) None of these

63. Who among the following family members is an architect ?(a) L (b) O(c) P (d) Can’t be determined(e) None of these

64. Which of the following is the profession of P ?(a) Architect (b) Pilot(c) Architect or pilot (d) Journalist(e) None of these

65. How is Q related to O ?(a) Father (b) Mother(c) Mother-in- law (d) Son - in - law(e) None of these

66. If in the number 5608391467, the position of the first and thesixth digits are interchanged, the second and seventh digitsare interchanged and so on upto the fifth and the tenthdigits. Then which will be the fourth digit from the left endafter interchange?(a) 1 (b) 6(c) 8 (d) 3(e) None of these

67. What will come next in the following series?ababcabcdabcdeabcdefabcd(a) g (b) f(c) e (d) a(e) None of these

DIRECTIONS (Qs. 68-70) : Study the following paragraph andanswer the questions that follow :

History as a discipline has always attracted the imaginationof the students because of its simple, lucid and easy tocomprehend nature. At the same time it is vast and time consumingtherefore it becomes imperative to plan a comprehensive strategyto ensure a sound preparation. The nature of the subject is itselfstatic however the questions asked over the last few years compelsus to understand the dynamics of the discipline as well as theforces, trends and patterns including institutions that have shapedhistory at any given point of time.

68. The passage best support the statement that–(a) History has a short and lucid curriculum to study so it

attracts the students.(b) History has an extensive curriculum and its questions

are time taking.(c) History is lucid and easy discipline. So, it does not

need a comprehensive strategic to ensure a soundpreparation.

(d) History is an important subject in the examination pointof view.

(e) None of these69. Which of the following assumption which can be implicit in

the facts stated in the above passage?(a) History is an important subject for the competition

points of view.(b) Nature of this subject is static.(c) A comprehensive strategy is needed to ensure a sound

preparation of history.(d) History attracts the student with its easy nature.(e) None of these

70. Which of the following is a conclusion which can be drawnfrom the facts stated in the above passage.(a) History is an easy subject nevertheless students do

not like to opt it as an optional paper.(b) History is a boring and wearisome subject.(c) There is not enough study material for the sound

preparation of history.(d) Questions are not being asked from history in the

competitive exams.(e) None of these

Part-III : English LanguageDIRECTIONS (Qs. 71-80) : Read the following passage carefullyand answer the questions given below it. Certain words havebeen printed in bold to help you locate them while answeringsome of the questions.

Govind’s father was a rich landlord, who was loved andrespected by all his tenants. When he died, he left large tracts ofland to Govind. But Govind did not spend a single day lookingafter his land. He had a funny idea, that there existed a magicpotion which, if it was poured on any object, would turn it intogold. He spent all his time trying to learn more about this potion.People took advantage of him and cheated him. His wife grewanxious. Given the amount of money Govind was spending, shewas sure that they would soon be paupers.

One day, a widely respected sage who had been to theHimalayas came to their town. Govind asked him about the potion.To his surprise the sage answered, “I have learnt how to brewsuch a potion. But it is a difficult process.” “Tell me!” insistedGovind, hardly able to believe his luck. “You have to collect thedew which settles on the leaves of a banana tree every morningduring winter. There is a condition, though. The tree should beplanted and watered regularly with your own hands. Store thecollected dew in an earthen vessel and when you have five litres,bring it to me. I will recite a sacred mantra to transform the dewinto the potion. A drop of the potion will be sufficient to changeany object into gold.”

Page 226: yoursmahboob.wordpress.com SBI · yoursmahboob.wordpress.com iii P 101 Speed Tests for SBI Bank Clerk Exam 101 Speed Tests for SBI Bank Clerk Exam is revised and updated edition on

yoursmahboob.w

ordpress.com

SPEED TEST 98218

Govind was worried “Winter is only for a few months in theyear. It will take me years to collect the dew.” “You can plant asmany trees as you want,” replied the sage. Govind went homeand after talking to his wife, began clearing the large fields whichhas been lying vacant for years. He planted rows of bananasaplings. He tended them with great care. His wife helped him too.She would take the banana crop to market and get a good price.Over the years the plantation grew and finally after six yearsGovind had five litres of dew. He went to the sage who smiled,uttered a mantra and sprinkled a few drops of dew on a coppervessel. To Govind’s dismay, nothing happened. “you have cheatedme!” he shouted at the sage.

The sage however smiled. Govind’s wife then came forwardwith a box. The sage opened it and revealed stacks of gold coinsinside. Turning to Govind he said, “you worked hard on yourland and created a plantation. Your wife sold the produce in themarket. It was your hard work which created this wealth, not magic.If I had told you this earlier, you would not have listened.” Govindunderstood the wisdom behind the sage’s words and workedeven harder from that day on.71. Why did Govind’s father give him large tracts of land?

(a) It was his way of instilling a sense of responsibility inhis son

(b) Govind was his only son and sole heir(c) To provide Govind with sufficient funds to pursue his

interest of discovering a magic potion(d) He wanted Govind to continue to look after the tenants(e) None of these

72. Which of the following can be said about the sage?(a) He was cunning and plotted with Govind’s wife to cheat

him.(b) He had no magical powers as such and used to swindle

people(c) He was a good judge of people(d) He did not deserve his good reputation(e) He was dishonest because he had cheated Govind out

of his gold73. Why was Govind’s wife worried ?

(a) Govind had no knowledge of farming and could notcultivate the land he had inherited from his father

(b) Govind had not friends because he was obsessed withfinding a potion which would turn any thing into gold

(c) Govind was only interested in studying under differentsages and neglected his family duties

(d) Since Govind had devoted all his time and wealth tofinding a magic potion, they would soon be poor

(e) Govind’s experiments to find a magic potion weredangerous

74. Why did Govind’s wife help him in the fields?A. To support her husband in his endeavour to find a

magic potion.B. The sage had advised her to help her husband succeed.C. He needed someone to help him collect the dew.(a) Only (B) (b) Only (A)(c) Both (A) and (B) (d) All (A), (B) and (C)(e) None of these

75. Why did Govind decide to cultivate a banana crop?(a) The soil of his land was suitable only for cultivating bananas(b) It was the most highly priced commodity in the region(c) It could be grown at any time of the year including winter(d) His wife pressurised him to do so(e) The ingredient for the magic potion could only be

obtained from a banana tree76. What made Govind angry with the sage?

(a) The sage had conspired with Govind’s wife againsthim

(b) He had forgotten the magic spell and all Govind’s hardwork was in vain

(c) He had lost a good deal of money in cultivatingbananas

(d) The sage had made a fool of him in front of othervillagers

(e) None of theseDIRECTIONS (Qs. 77-78) : Choose the word which is most similarin meaning to the word printed in bold as used in the passage.77. SPEND

(a) pay (b) bought(c) devote (d) settle(e) empty

78. LYING(a) sleeping (b) dishonest(c) relaxing (d) remaining(e) untruthful

DIRECTIONS (Qs. 79-80) : Choose the word which is mostopposite in meaning to the word printed in bold as used in thepassage.79. DISMAY

(a) joy (b) interest(c) desire (d) humour(e) luck

80. TENDED(a) negligible (b) watched(c) inclined (d) ignored(e) spoil

DIRECTIONS (Qs. 81-85) : Rearrange the following sixsentences (A), (B), (C), (D), (E) and (F) in the proper sequenceto form a meaningful paragraph; then answer the questions givenbelow them.

A. The hall was filled with children, teachers, students, familymembers and those who were close to him.

B. Normally such ceremonies are attended by important peoplelike industrialists, politicians and VIP’s.

C. What I saw when I stepped into the hall amazed me.D. I went home with the feeling that it was a most unusual oath

taking ceremony with only those who were ‘important’ tohim present.

E. When he was elected President, he invited me to the swearingin ceremony in the Central Hall of Parliament.

F. However in this case everyone who attended the ceremonyseemed to know him personally.

Page 227: yoursmahboob.wordpress.com SBI · yoursmahboob.wordpress.com iii P 101 Speed Tests for SBI Bank Clerk Exam 101 Speed Tests for SBI Bank Clerk Exam is revised and updated edition on

yoursmahboob.w

ordpress.com

SPEED TEST 98 21981. Which of the following should be the FIRST sentence after

rearrangement ?(a) A (b) B(c) C (d) D(e) E

82. Which of the following should be the SECOND sentenceafter rearrangement ?(a) B (b) C(c) D (d) E(e) F

83. Which of the following should be the THIRD sentence afterrearrangement ?(a) A (b) B(c) C (d) D(e) E

84. Which of the following should be the FIFTH sentence afterrearrangement ?(a) B (b) C(c) D (d) E(e) F

85. Which of the following should be the LAST (SIXTH)sentence after rearrangement ?(a) A (b) B(c) C (d) D(e) E

DIRECTIONS (Qs. 86-90) : Read each sentence to find outwhether there is any grammatical error or idiomatic error in it.The error, if any, will be in one part of the sentence. The numberof that part is the answer. If there is no error, the answer is (e).(Ignore errors of punctuation, if any.)86. His proposal had (a) / to be send to (b) / the President of the

company (c) / for her approval (d). No error (e).87. Each tuesday evening we visited (a) / the farmers in the

area (b) / and held a meeting (c) / to discuss the problemsthey faced (c). No error (e).

88. Though our training facilities (a) / are limited only a (b) / fewemployees have been (c) / selected for training (d). No error(e).

89. During the interview (a) / the panel asked me (b) / severaltechnical questions (c) / and I answered all of it (d). No error(e).

90. He decided to work for (a) / an NGO, but most of his (b) /classmates opted for high paid (c) / jobs in multinationalcompanies (d). No error (e).

DIRECTIONS (Qs. 91-98) : In the following passage there areblanks each of which has been numbered. These numbers areprinted below the passage and against each, five words aresuggested, one of which fits the blank appropriately. Find out theapproptiate words in each case.

When we 91 started thirty years ago in 1977, we did notknow anything about how to run a bank for the poor. We thereforelooked at how others ran their operations and 92 from their

mistakes. In Bangladesh, conventional banks and credit co-operatives always 93 lump sum repayments. This created 94problems because repaying in a lump sum was a mental hurdle forborrowers. They tended to delay repayment and get further intodebt in the 95. In the end they usually 96 totally on the loan,which was a loss to the bank. In structuring our own loans, Idecided to ask for a daily payment, Monitoring repayment was 97and it filled people with 98 that they could repay their loans.91. (a) firstly (b) freshly

(c) foremost (d) initially(e) recently

92. (a) copied (b) observed(c) learned (d) understood(e) improving

93. (a) asked (b) insisted(c) demanded (d) settled(e) lend

94. (a) severe (b) no(c) additionally (d) variety(e) plenty

95. (a) time (b) process(c) return (d) event(e) action

96. (a) neglected (b) abandoned(c) defaulted (d) depended(e) disappointed

97. (a) benefit (b) easier(c) reckless (d) disorganised(e) secure

98. (a) sense (b) confidence(c) challenge (d) doubt(e) believe

DIRECTIONS (Qs. 99 to 100): In each of the following sentences,an idiomatic expression or a proverb is highlighted. Select thealternative which best describes its use in the sentence.99. Having sold off his factory, he is now a gentleman at large.

(a) Has no serious occupation(b) Is living comfortably(c) Is respected by everybody(d) Is held in high esteem(e) None of these

100. Though he has lot of money, yet all his plans are built uponsand.(a) established on insecure foundations(b) based on inexperience(c) resting on cheap material(d) resting on immature ideas(e) None of these

Page 228: yoursmahboob.wordpress.com SBI · yoursmahboob.wordpress.com iii P 101 Speed Tests for SBI Bank Clerk Exam 101 Speed Tests for SBI Bank Clerk Exam is revised and updated edition on

yoursmahboob.w

ordpress.com

SPEED TEST 98220

Test Code : ............................... Time taken : ......................... Date : ..........................

RESPONSE SHEET

(a) (b) (c) (d) (e)

1.

2.

3.

4.

5.

6.

7.

8.

9.

10.

11.

12.

13.

14.

15.

16.

17.

18.

19.

20.

21.

22.

23.

24.

25.

(a) (b) (c) (d) (e)

51.

52.

53.

54.

55.

56.

57.

58.

59.

60.

61.

62.

63.

64.

65.

66.

67.

68.

69.

70.

71.

72.

73.

74.

75.

(a) (b) (c) (d) (e)

26.

27.

28.

29.

30.

31.

32.

33.

34.

35.

36.

37.

38.

39.

40.

41.

42.

43.

44.

45.

46.

47.

48.

49.

50.

(a) (b) (c) (d) (e)

76.

77.

78.

79.

80.

81.

82.

83.

84.

85.

86.

87.

88.

89.

90.

91.

92.

93.

94.

95.

96.

97.

98.

99.

100.

Page 229: yoursmahboob.wordpress.com SBI · yoursmahboob.wordpress.com iii P 101 Speed Tests for SBI Bank Clerk Exam 101 Speed Tests for SBI Bank Clerk Exam is revised and updated edition on

yoursmahboob.w

ordpress.com

Part-I : Numerical Ability

DIRECTIONS (Q. 1-10) : What should come in place of thequestion mark (?) in the following questions?

1. ? 75(a) –5625 (b) 75(c) 1500 (d) Cannot be determined(e) None of these

2.21 7 1 ?8 72 171

(a)9

19 (b)13

(c)5

19 (d)3

19(e) None of these

3.1 2 14 6 5 ?2 3 3

(a)1152

(b)2163

(c)1162

(d) 17

(e) None of these4. 792.02 + 101.32 - 306.76 = ?

(a) 893.34 (b) 1200.10(c) 997.11 (d) 586.58(e) None of these

5. 300% of 150 = ?% of 600(a) 75 (b) 45

(c) 450 (d) 13312

(e) None of these6. 34.95 + 240.016 + 23.9800 = ?

(a) 299.09 (b) 298.0946(c) 298.111 (d) 298.946(e) None of these

7. 48.95 – 32.006 = ?(a) 16.089 (b) 16.944(c) 16.35 (d) 16.89(e) None of these

8. 3889 + 12.952 – ? = 3854.002(a) 47.95 (b) 47.752(c) 47.095 (d) 47.932(e) None of these

9. ? + 72.64 = 74.64(a) 145.28 (b) –2.00(c) –145.28 (d) 147.28(e) None of these

10. 6.25 0.0025 = ?(a) 1800 (b) 2300(c) 1700 (d) 2500(e) None of these

11. Which is the smallest of the following numbers ?

(a) 7 (b)7

1

(c)77 (d)

71

(e) None of these12. Two equal sums were borrowed at 8% simple interest per

annum for 2 years and 3 years, respectively. The differencein the interests was 56. The difference in the interests was`56. The sum borrowed were(a) `690 (b) `700(c) `740 (d) `780(e) None of these

13. A machine is sold at a profit of 10%. Had it been sold for ` 80less, there would have been a loss of 10%. The cost price ofthe machine is(a) `350 (b) `400(c) `450 (d) `520(e) None of these

14. A jar of oil was four fifths full. When six bottles of oil weretaken out and four bottles of oil were poured into, it wasthree fourths full. How many bottles of oil were containedby the jar ?(a) 10 (b) 20(c) 30 (d) 40(e) None of these

15. During a journey of 80 km a train covers first 60km with aspeed of 40 km/h and completes the remaining distance witha speed of 20 km/h. What is the average speed of the trainduring the whole journey?(a) 30 km/h (b) 32 km/h(c) 36 km/h (d) 40 km/h(e) None of these

Max. Marks : 100 No. of Qs. 100 Time : 1 hr. Date : ........./......../................

9999Prelim Test - 3

Page 230: yoursmahboob.wordpress.com SBI · yoursmahboob.wordpress.com iii P 101 Speed Tests for SBI Bank Clerk Exam 101 Speed Tests for SBI Bank Clerk Exam is revised and updated edition on

yoursmahboob.w

ordpress.com

SPEED TEST 99222

16. An aeroplane takes off 30 minutes later than the scheduledtime and in order to reach its destination 1500 km away intime, it has to increase its speed by 250 km/h from its usualspeed. Find its usual speed.(a) 1000 km/h (b) 750 km/h(c) 850 km/h (d) 650 km/h(e) None of these

17. In an examination 35% of the candidates failed in one subjectand 42% failed in another subject. While 15% failed in boththe subjects. If 2500 candidates appeared at the examination,how many students passed in either subject but not in both?(a) 325 (b) 1175(c) 2125 (d) 1230(e) None of these

18. If the length of a certain rectangle is decreased by 4 cm andthe width is increased by 3 cm, a square with the same areaas the original rectangle would result. The perimeter of theoriginal rectangle (in centimetres) is :(a) 44 (b) 46(c) 48 (d) 50(e) None of these

DIRECTION (Qs. 19-21): What will come in place of the questionmark (?) in the following number series?

19. 2 9 30 105 ? 2195(a) 432 (b) 426(c) 440 (d) 436(e) None of these

20. 3 4 12 45 ? 1005(a) 152 (b) 198(c) 144 (d) 192(e) None of these

21. 1 3 9 31 ? 651(a) 97 (b) 127(c) 129 (d) 109(e) None of these

DIRECTIONS (Qs. 22-23): Find out the approximate value whichshould replace the question mark (?) in the following questions.(You are not expected to find out the exact value.)

22. 3.7 0.9989 ?95 95 95(a) 1.9 (b) 3(c) 2.99 (d) 3.6(e) 2.7

233.00110004.987 of 1891.992 = ?

(a) 2500 (b) 1230(c) 1640 (d) 1525(e) 2130

24. 12.25 × ? × 21.6 = 3545 .64(a) 20 (b) 12(c) 15 (d) 13(e) None of these

25. 3 4096 ?(a) 16 (b) 26(c) 18 (d) 24(e) None of these

26. 6 men can complete a piece of work in 20 days. In how manydays will 8 men complete the same piece of work?(a) 12 days (b) 14 days(c) 15 days (d) 16 days(e) None of these

27. What will come in place of both the question marks (?) inthe following equation?

363 (?)(?) 3

(a) 43 (b) 33(c) 37 (d) 47(e) None of these

28. Raju decided to marry 3 years after he gets a job. He was 17years old when he passed class 12th. After passing class12th', he had completed his graduation course in 3 yearsand PG Course in 2 years. He got the job exactly 1 year aftercompleting his PG Course. At what age will he get married?(a) 27 years (b) 26 years(c) 28 years (d) 23 years(e) None of these

DIRECTIONS (Qs. 29-33): Study the following table carefully toanswer the questions that follow:

Number of Students studying in Six Different Colleges over theYears

CollegeP Q R S T U

Year2004 2500 2250 2450 2150 2020 23002005 2040 2300 2400 2200 2090 21202006 2100 2150 2330 2250 2180 22602007 2280 2600 2260 2340 2250 24902008 2540 2240 2120 2380 2310 25202009 2320 2440 2500 2480 2400 2440

29. What is the total number of students from all the collegestogether in the year 2005?(a) 10350 (b) 13150(c) 15310 (d) 11350(e) None of these

30. What is the per cent increase in the number of students inCollege T in the year 2007 from the previous year? (roundedoff to two digits after decimal)(a) 8.33 (b) 5.18(c) 6.63 (d) 3.21(e) None of these

31. The number of students in college P in the year 2008 formsapproximately what per cent of the total number of studentsin the college in all the year together.

Page 231: yoursmahboob.wordpress.com SBI · yoursmahboob.wordpress.com iii P 101 Speed Tests for SBI Bank Clerk Exam 101 Speed Tests for SBI Bank Clerk Exam is revised and updated edition on

yoursmahboob.w

ordpress.com

SPEED TEST 99 223(a) 11 (b) 31(c) 18 (d) 26(e) 23

32. What is the ratio of the total number of students in CollegeS in the years 2006 and 2009 together to the total number ofstudents in College U in the same years?(a) 473 : 470 (b) 470 : 473(c) 371 : 390 (d) 390 : 371(e) None of these

33. What is the average number of students in all the collegestogether in the year 2004? (rounded off to the nearest integer)(a) 2208 (b) 2196(c) 2144 (d) 2324(e) 2278

34. The angles of a triangle are in the ratio of 5 : 6 : 7. respectively.What is the sum of the smallest angle and the largest angletogether?(a) 130° (b) 100°(c) 110° (d) 140°(e) None of these

35. What least number should be subtracted from 536 to make ita perfect square?(a) 7 (b) 5(c) 23 (d) 18

(e) None of these

Part-II : Reasoning Ability36. How many such pairs of letters are there in the word

CHANNEL each of which has as many letters between themin the word as in the English alphabet?(a) None(b) One(c) Two(d) Three(e) More than three

37. How many meaningful English words can be made with theletters ATLE using each letter only once in each word ?(a) None (b) One(c) Two (d) Three(e) More than three

38. In a certain code GROWN is written as 7 @ % 36 and NAMEis written as 64 $. How is GEAR· written in that code?(a) 74$@ (b) 7$4@(c) 7%4@ (d) 7@$4(e) None of these

39. How many pairs of digits are there in the number 6315784each of which has as many digits between them in the numberas when the digits are rearranged in descending order?(a) None (b) One(c) Two (d) Three(e) More than three

40. What should come next in the following letter series?B D F H J L N A C E G I K M B D F H J L A C E G I K B D F H J(a) B (b) L(c) M (d) F(e) None of these

41. Four of the following five are alike in a certain way and soform a group. Which is the one that does not belong to thatgroup?(a) 169 (b) 441(c) 361 (d) 529(e) 289

42. Among P, Q, R, T and W each having different weight, T isheavier than W and lighter than only P. Q is not the lightest.Who among them is definitely the lightest ?(a) R (b) W(c) R or W (d) Data inadequate(e) None of these

DIRECTIONS (Q. 43-47) : In each question below are threestatements followed by three conclusions numbered I, II and III.You have to take the three given statements to be true even if theyseem to be at variance from commonly known facts and thendecide which of the given conclusions logically follows from thethree given statements disregarding commonly known facts. Thendecide which of the answers (a), (b), (c), (d) and (e) is the correctanswer and indicate it on the answer sheet.43. Statements : Some desks are chairs. All chairs are tables.

Some tables are mats.Conclusions : I. Some mats are desks.

II. Some tables are desks.III.Some mats are chairs.

(a) Only I follows (b) Only II follows(c) Only III follows (d) II and III follow(e) None of the above

44. Statements : All sweets are fruits. No fruit is pencil. Somepencils are glasses.Conclusions : I. Some glasses are sweets.

II. Some pencils are sweets.III.No glass is sweet.

(a) Only I follows (b) Only II follows(c) Only III follows (d) either I or III follows(e) None of the above

45. Statements : Some books are flowers. Some flowers arechains. Some chains are hammers.Conclusions : I. Some hammers are flowers.

II. Some chairs are books.III.Some hammers are books.

(a) None follows (b) Only I follows(c) Only II follows (d) Only III follows(e) II and III follow

46. Statements : All roofs are cameras. Some cameras arephotographs.Some photographs are stores.Conclusions : I. Some stores are cameras.

II. Some stores are roofs.III.Some cameras are roofs.

(a) Only I follows (b) Only II follows(c) Only III follows (d) II and III follow(e) None of the above

Page 232: yoursmahboob.wordpress.com SBI · yoursmahboob.wordpress.com iii P 101 Speed Tests for SBI Bank Clerk Exam 101 Speed Tests for SBI Bank Clerk Exam is revised and updated edition on

yoursmahboob.w

ordpress.com

SPEED TEST 99224

47. Statements : Some nails are horses. All horses are tablets.All tablets are crows.Conclusions : I. Some crows are nails.

II. Some tablets are nails.III.Some crows are horses.

(a) Only I follows (b) I and II follows(c) I and III follow (d) II and III follow(e) All I, II and III follow

DIRECTIONS (Q. 48-52) : Study the following arrangementcarefully and answer the questions given below

D 5 R @ A K © 3 9 B J E F $ M P I 4 H 1 W6 2 # U Q 8 T N

48. How many such numbers are there in the above arrangementeach of which is immediately preceded by a symbol andimmediately followed by a letter?(a) None (b) One(c) Two (d) Three(e) More than three

49. Which of the following is the ninth to the right of the twentysecond from the right end of the above arrangement ?(a) E (b) I(c) D (d) N(e) None of these

50. How many such symbols are there in the above arrangementeach of which is immediately preceded by a number andimmediately followed by a letter?(a) None (b) One(c) Two (d) Three(e) More than three

51. If all the numbers are dropped from the above arrangement,which of the following will be the eleventh from the leftend?(a) B (b) H(c) $ (d) (e) None of these

52. How many such consonants are there in the abovearrangement each of which is immediately preceded by anumber and immediately followed by another consonant ?(a) None (b) One(c) Two (d) Three(e) More than three

DIRECTIONS (Q. 53-57) : In the following questions, the symbols#, %, @, © and are used with the following meanings illustrated.

‘P % Q’ means ‘P is not greater than Q’.‘P Q’ means ‘P is not smaller than Q’.‘P # Q’ means ‘P is neither equal to nor smaller than Q’.‘P © Q’ means ‘P is neither equal to nor greater than Q’.‘P @ Q’ means ‘P is neither smaller than nor greater than Q’.In each question, three statements showing relationships havebeen given, which are followed by three conclusions I, II and III.Assuming that the given statements are true, find out whichconclusion(s) is/are definitely true.53. Statements : M © K, K T, T © J

Conclusions :I. J # K II. T # MIII. M # J

(a) None is true (b) Only I is true(c) Only II is true (d) Only III is true(e) II and III are true

54. Statements : F @ T, T % M, M # RConclusions :I. R © T II. F @MIII. F©M(a) Only I is true (b) Only II is true(c) Only III is true (d) either II or III is true(e) II and III are true

55. Statements : J H, H @ B, B % NConclusions :I. N H II. N @ JIII. J B(a) I and II are true (b) II and III are true(c) I and III are true (d) All I, II and III are true(e) None of the above

56. Statements : B # T, T © K, K % MConclusions :I. K # BII. M # TIII. B # M(a) Only I is true(b) Only II is true(c) Only III is true(d) II and III are true(e) None of the above

57. Statements : D % F, F K, K @ RConclusions :I. R % FII. R % DIII. R@ D(a) Only I is true (b) Only II is true(c) Only III is true (d) I and II are true(e) None of the above

DIRECTIONS (Q. 58-62) : Study the following arrangementcarefully and answer the questions given below

M, D, K, R, T, H, W and A are sitting around a circle facingat the centre. D is second to the right of M who is fifth to the leftof T. K is third to the right of R who is second to the right of D. His second to the right of W.58. Who is second to the right of A ?

(a) M (b) D(c) K (d) Data inadequate(e) None of the above

59. Who is third to the left of M ?(a) A (b) T(c) H (d) D(e) Data inadequate

60. Who is fourth to the right of H ?(a) A (b) T(c) R (d) K(e) None of these

Page 233: yoursmahboob.wordpress.com SBI · yoursmahboob.wordpress.com iii P 101 Speed Tests for SBI Bank Clerk Exam 101 Speed Tests for SBI Bank Clerk Exam is revised and updated edition on

yoursmahboob.w

ordpress.com

SPEED TEST 99 22561. In which of the following combinations is the first person

sitting between the second and the third person ?(a) KMW(b) MWD(c) RHT(d) TAK(e) None of the above

62. If A and W interchange their positions who will be third tothe left of R ?(a) M (b) D(c) A (d) K(e) None of these

DIRECTIONS (Qs. 63-65) : Study the following paragraph andanswer the questions that follow :

The Government of India is responsible for ensuring thedefense of India and every part thereof. The Supreme Command ofthe Armed Forces vests in the President. The responsibility fornational defense rests with the Cabinet. This is discharged throughthe Ministry of Defense, which provides the policy framework andwherewithal to the Armed Forces to discharge their responsibilitiesin the context of the defense of the country. Ministry of Defensecomprises of four Departments viz. Department of Defense (DOD),Department of Defense Production (DDP), Department of DefenseResearch & Development (DDR&D) and Department of Ex-Servicemen Welfare and also Finance Division63. Which of the following is a conclusion which can be drawn

from the facts stated in the given passage?(a) President of India is responsible for the defense of the

country.(b) The government of India is responsible for defense of

the country and ministry of defense performs the dutywith help of its various departments.

(c) People of India are themselves responsible four theirsecurity.

(d) Indians are so bold that there is no need of security.(e) None of these

64. Which statement strengthens the conclusion of thepassages?(a) All ministries of the government co-operate to each

other and prime minister co-ordinates them.(b) There are many falts in our defense system which

come-out time to time.(c) Now-a-days the ministry of defense is not working

properly consequently internal and external securityis in danger.

(d) People of the country is themselves aware for theirsecurity.

(e) None of these65. Which statement weakens the conclusion of the passage.

(a) President of India is the supreme commander of theArmed Forces.

(b) Due to lack of co-ordination ministry of defense isunable to determine the security of people.

(c) Ministry of defense is divided in four variousdepartments.

(d) Department of defense production provides the socialsecurity to its ex-servicemen.

(e) None of these66. Four of the following five are alike in a certain way and so

form a group. Which is the one that does not belong to thatgroup?(a) Water (b) Juice(c) Petrol (d) Sugar(e) None of these

67. How many meaningful English words can be made with theletters EIND using each letter only once in each word ?(a) None (b) One(c) Two (d) Three(e) None of these

68. Four of the following five are alike in a certain way and soform a group. Which of the following does not belong tothat group?(a) 343 (b) 64(c) 75 (d) 27(e) 216

69. How many such digits are there in the number 586972 eachof which is as far away from the beginning of the number aswhen the digits are arranged in discending order.(a) None (b) One(c) Two (d) Three(e) More than Three

70. What should come next in the following number series ?9 8 7 6 5 4 3 2 1 8 7 6 5 4 3 2 1 7 6 5 4 3 2 1(1) 9 (2) 8(c) 7 (d) 6(e) None of these

Part-III : English LanguageDIRECTIONS (71-80) : Read the following passage carefullyand answer the questions given below it. Certain words/phraseshave been printed in bold to help you locate them while answeringsome of the questions.

Once upon a time there lived a vicious king, Raja Shankara–short-tempered and temperamental. "God I am" he said to hisimage as he stared into the mirror everyday, many times a day, Hewas obsessed with himself. He loved no one but himself. He wasblinded towards the injustice in his kingdom because he had littletime for his subjects. He wasted most of his time in pouring milkand honey over himself.

Interruption in his possessed life was dealt with sternreprimanding and sometimes on petty issues he would behead hisservants. Provoked by his evil advisor Twishar, he went on withhis self indulged life, unaware of the plot his very devoted advisorwas planning. A plot to dethrone the king, rule the kingdom withhis wicked ways only to harness wealth and the reputation of aking.

One morning the king went on his usual moring horsebackrounds but returned with a very sad look on his face. He lockedhimself inside his platial room only to unlock it at sundown. Just

Page 234: yoursmahboob.wordpress.com SBI · yoursmahboob.wordpress.com iii P 101 Speed Tests for SBI Bank Clerk Exam 101 Speed Tests for SBI Bank Clerk Exam is revised and updated edition on

yoursmahboob.w

ordpress.com

SPEED TEST 99226

as the doors creaked open and Raja Shankara emerged from it, hiswife rushed to embrace him. She feared a damaging incident hadoccurred.

The king spoke seldom that day and awoke the next day tomake a proclamation to his servants and subjects. The wholekingdom feared what was in store for them from their angry king.But to their surprise he said to all gathered, "From now on I will bea different king. A softer and a patient king."

True to his words from that day on, the king had truly turnedon a new leaf; he cleaned out the corruption and injustice in atender manner with punishments aimed to renew the person fromwithin.

One fine day his evil advisor gathered courage to ask thereason for his paradigm shift. And the king answered. When Iwent on horseback that morning a month ago, I noticed a dogbrutally chasing a cat. The cat managed to sneak into a hole onlyafter the dog bit her leg, maiming her for life. soon afterwards, thedog barked at a farmer who picked up a sharp stone and hit itstraight in the dog's eye. Bleeding profusely, the dog yelped inpain. As the farmer walked on, he slipped on the edge of the roadand broke his head.

All this happened in a matter of minutes before me and thenI realized that evil begets evil. I thought about it deeply and wasready to give up my worldly life for the betterment of my subjects.I wanted to give up the evil in me as I did not want evil to encounterme.

Sniggering away the immoral advisor thought what a perfecttime it was to dethrone the king, because the Raja had grown kindhearted and patient and would not endeavour a combat. Thinkinghow he would plan his attack, he stumbled over a step that tookhim hurling down the remaining steps, bringing him to a stop witha crash. He howled in pain only to discover he had broken thebones in both his legs.71. How can Raja Shankara be described before his

transformation?(1) He was unjust(2) He was preoccupied with himself(3) He was cruel(a) Only (1) (b) Only (2)(c) Only (3) (d) Only (1) adn (2)(e) All the three (1), (2) and (3)

72. Why was the king not happy with the old Valuer?(a) As the Valuer was not good at his work(b) As he had dishonoured the king(c) As the Valuer had been dishonest with the king about

the prices that he set for goods(d) As the king beloeved that he was not earning much

because of the Valuer's honesty(e) None of these

73. What proclamation did the Raja make to his subjects?(a) That he was giving up his throne for the betterment of

the kingdom(b) That his advisor would be the king from then on(c) That he would be a better king to them than he had

been all this while

(d) That his subjects were not good enough to deservebetter treatment than what was already being metedout to them

(e) None of these74. What did Raja Shankara's wife think about the Raja's peculiar

behaviour that particular day?(a) She was afraid that something really bad had happened(b) She was afraid that the Raja would beat her up because

of his unusual mood(c) She thought that he was in his usual sour mood(d) She thought that the Raja had received threats to his

life from his servants(e) None of these

75. What was the reason for Raja Shankara's change inbehaviour?(a) His advisor's words had made him realize his mistake(b) He had felt bad for a poor family on his tour around his

kingdom(c) His wife had betrayed him and hence he was upset(d) He had realized that doing good to people would bring

good to him(e) He had realized that evil begets evil.

DIRECTIONS (76-78) : Choose the word/group of words whichis most similar in meaning to the word/group of words printed inbold as used in the passage.76. BEGETS

(a) produces (b) loses(c) expects (d) avoids(e) calls

77. STERN(a) hard (b) tall(c) easy (d) tight(e) severe

78. GATHERED(a) partied (b) assembled(c) dispersed (d) pooled(e) collated

DIRECTIONS (79-80) : Choose the word/group of words whichis most opposite in meaning to the word/group of words printedin bold as used in the passage,79. PETTY

(a) tremendous (b) huge(c) vast (d) important(e) frugal

80. BRUTALLY(a) cruelly (b) partly(c) gently (d) rarely(e) harmfully

DIRECTIONS (81-83) : Which of the phrases (a), (b), (c) and (d)given below each sentence should replace the phrase printed inbold in the sentence to make it grammatically correct? If thesentece is correct as it is given and no correction is required,mark (e) as the answer.

Page 235: yoursmahboob.wordpress.com SBI · yoursmahboob.wordpress.com iii P 101 Speed Tests for SBI Bank Clerk Exam 101 Speed Tests for SBI Bank Clerk Exam is revised and updated edition on

yoursmahboob.w

ordpress.com

SPEED TEST 99 22781. Her entry to the office party was restrict as an official

enquiry had ben constituted against her.(a) was restricting(b) is restricted(c) was restricted(d) is restricting(e) No correction required

82. Rima was at her wit's end trying to figure out what to buyfor her frind's birthday.(a) at her witting end(b) at her wit ends(c) to her wit's end(d) so wit's end(e) No correction required

83. Pritesh while away his time in playing games on the computerinstead of studying.(a) whiled away his time(b) whiled against his time(c) whiling away his time(d) while awayed his time(e) No correction required

DIRECTIONS (84-88) : In each question below, a sentence withfour words printed in bold type is given. These are numbered as(a), (b), (c) and (d). One of these four words printed in bold may beeither wrongly spelt or inappropriate in the context of thesentence. Find out the word which is wrongly spelt orinappropriate, if any. The number of that word is your answer. Ifall the words printed in bold are correctly spelt and alsoappropriate in the context of the sentence, mark (e) i.e., 'Allcorrect' as your answer.

84. Discussion (a)/ is an exchange of knowledge (b)/ whereasarguement (c)/ is a depiction (d)/ of ignorance. All correct(e).

85. He was arrested (a)/ for the crime (b)/ and was charged (c)/with attempt (d)/ to murder. All correct (e)

86. commit (a)/ yourself to lifelong learning (b)/ as the mostvaluable (c)/ aset (d)/ you will have is your mind.All correct (e)

87. Belive (a)/ that life is worth (b)/ living and your belief willcreate (c)/ the fact. (d)/ All correct (e)

88. The best educated (1)/ human bing (2)/ is the one whounderstands (3)/ most about the life in which (4)/ he isplaced. All correct (e).

DIRECTIONS (89-98) : In the following passage there are blanks,each of which has been numbered. These numbers are printedbelow the passage and against each, five words are suggested,one of of which fits the blank appropriately. Find out theappropriate word in each case.

Once upon a time, two friends were (89) through the desert.During some point of the (90) they had an argument, and onefriend slapped the other one in the face. The one who got slappedwas (91), but without saying anything, he wrote in the sand,"Today my best friend slapped me in the face." They kept on

walking (92) they found an oasis, where they (93) to take a both.The one, who had been slapped, got (94) in the quicksand andstarted drowing, but the friend saved him. After the friend (95)from the near drowning, he wrote on a stone, "The friend who hadslapped and saved his best friend asked him, "After I hurt you,you wrote in the sand and (96) you write on a stone, why?" Theother friend (97), "When someone hurts us, we should write itdown in sand where winds of forgiveness can erase it away. But,when someone does something good for us, we must (98) it instone where no wind can ever erase it."89. (a) crawling (b) speaking

(c) swimming (d) walking(e) dancing

90. (a) journey (b) sand(c) running (d) border(e) hunt

91. (a) dead (b) captured(c) presentable (d) missing(e) hurt

92. (a) as (b) until(c) from (d) with(e) through

93. (a) decided (b) fell(c) made (d) want(e) left

94. (a) home (b) stuck(c) blended (d) mixed(e) sitting

95. (a) separated (b) leaked(c) died (d) recovered(e) saved

96. (a) so (b) how(c) when (d) tomorrow(e) now

97. (a) called (b) tell(c) replied (d) questioned(e) asked

98. (a) talk (b) push(c) engrave (d) add(e) bury

DIRECTIONS (Qs. 99 to 100): In each of the following sentences,an idiomatic expression or a proverb is highlighted. Select thealternative which best describes its use in the sentence.99. Mohan always keeps himself to himself.

(a) Is too busy (b) Is selfish(c) Is unsociable (d) does not take sides(e) None of these

100. While the ladies continued their small talk in the drawingroom, I felt bored.(a) whispering (b) backbiting(c) gossip (d) light conversation(e) None of these

Page 236: yoursmahboob.wordpress.com SBI · yoursmahboob.wordpress.com iii P 101 Speed Tests for SBI Bank Clerk Exam 101 Speed Tests for SBI Bank Clerk Exam is revised and updated edition on

yoursmahboob.w

ordpress.com

SPEED TEST 99228

Test Code : ............................... Time taken : ......................... Date : ..........................

RESPONSE SHEET

(a) (b) (c) (d) (e)

1.

2.

3.

4.

5.

6.

7.

8.

9.

10.

11.

12.

13.

14.

15.

16.

17.

18.

19.

20.

21.

22.

23.

24.

25.

(a) (b) (c) (d) (e)

51.

52.

53.

54.

55.

56.

57.

58.

59.

60.

61.

62.

63.

64.

65.

66.

67.

68.

69.

70.

71.

72.

73.

74.

75.

(a) (b) (c) (d) (e)

26.

27.

28.

29.

30.

31.

32.

33.

34.

35.

36.

37.

38.

39.

40.

41.

42.

43.

44.

45.

46.

47.

48.

49.

50.

(a) (b) (c) (d) (e)

76.

77.

78.

79.

80.

81.

82.

83.

84.

85.

86.

87.

88.

89.

90.

91.

92.

93.

94.

95.

96.

97.

98.

99.

100.

Page 237: yoursmahboob.wordpress.com SBI · yoursmahboob.wordpress.com iii P 101 Speed Tests for SBI Bank Clerk Exam 101 Speed Tests for SBI Bank Clerk Exam is revised and updated edition on

yoursmahboob.w

ordpress.com

Part-I : General and Financial Awareness1. Currency Swap is an instrument to manage

(a) currency risk(b) interest rate risk(c) currency and interest rate risk(d) cash flows in different currencies(e) All of the above

2. Which of the following voting systems has been adoptedfor the election of Lok Sabha and Legislative assemblyelections in India?(a) Single transferable vote system(b) First-past-the-post system(c) Hare-clark system(d) Single non transferable vote system(e) None of these

3. Money Laundering normally involves(a) placement of funds (b) layering of funds(c) integration of funds (d) All of (a), (b) and (c)(e) None of these

4. Capital Market Regulator is(a) RBI (b) IRDA(c) NSE (d) BSE(e) SEBI

5. Speaker of Lok Sabha is(a) appointed by the President of India on the advice ofthe Cheif Justice of the supreme court(b) appointed by the President of India(c) elected by the members of the Lok Sabha(d) elected by the members of Lok Sabha and Rajya Sabha(e) None of these

6. Which of the following is the Regulator of the credit ratingagencies in India?(a) RBI (b) SBI(c) SIDBI (d) SEBI(e) None of these

7. Lot of Banks in India these days are offering M-BankingFacility to their customers. What is the full form of 'M' in 'M-Banking'(a) Money (b) Marginal(c) Message (d) Mutual Fund(e) Mobile phone

8. Which of the following is mis-matched in relation to leadingbrands of milk and milk products in India?

(a) Amul-Gujarat(b) Parag-Uttar Pradesh(c) Gopal-Rajasthan(d) Verka-Punjab(e) None of these

9. Which of the following terms is used in Banking Field?(a) Interest Rate Swap (b) Input Devices(c) Sedimentary (d) Zero Hour(e) Privilege Motion

10. The term 'Smart Money" refers to __________ .(a) Foreign Currency (b) Internet Banking(c) US Dollars (d) Travelers' cheques(e) Credit Cards

11. Which one of the following is not a 'Money MarketInstrument' ?(a) Treasury Bills (b) Commercial Paper(c) Certificate of Deposit (d) Equity Shares(e) None of these

12. Which one of the following is a retail banking product ?(a) Home Loans (b) Working capital finance(c) Corporate term loans (d) Infrastructure financing(e) Export Credit

13. Which of the following is NOT a function of the ReserveBank of India ?(a) Fiscal Policy Functions(b) Exchange Control Functions(c) Issuance, Exchange and destruction of currency notes(d) Monetary Authority Functions(e) Supervisory and Control Functions

14. With reference to a cheque which of the following is the“drawee bank” ?(a) The bank that collects the cheque(b) The payee's bank(c) The endorsee's bank(d) The endorser's bank(e) The bank upon which the cheque is drawn

15. What is a Debit Card ?(a) It is a card issued by a Rating Agency(b) It is a card which can be used for withdrawing cash or

making payment even in the absence of any balance inthe account

(c) It is a card which can be used for withdrawing cash ormaking payment if there is balance in the account

(d) It is a card which carries prepaid balance(e) It is a card which can be used for making STD calls

Max. Marks : 200 No. of Qs. 190 Time : 2 hr 40 min. Date : ........./......../................

Full Main Test - 4 100100

Page 238: yoursmahboob.wordpress.com SBI · yoursmahboob.wordpress.com iii P 101 Speed Tests for SBI Bank Clerk Exam 101 Speed Tests for SBI Bank Clerk Exam is revised and updated edition on

yoursmahboob.w

ordpress.com

SPEED TEST 100230

16. Bad advances of a Bank are called __________ .(a) Bad debt (b) Book debt(c) Non Performing Asset (d) Out of order accounts(e) Overdrawn accounts

17. The "Sunil Gangopadhyay Memorial Award" is given forexcellence in ___?(a) Telugu (b) Bengali(c) Odisi (d) Rajasthani(e) None of these

18. National Girl Child Day was observed across India by theUnion Government on(a) 24 January (b) 25 January(c) 27 January (d) 21 January(e) None of these

19. Who is the author of the book "My Journey: TransformingDreams into actions"?(a) A P J Abdul Kalam (b) Sachin Tendulkar(c) Pranab Mukherjee (d) Amartya Sen(e) None of these

20. Where is the headquarter of United Nations Children's Fund(UNICEF) located?(a) New York (b) Berlin(c) Italy (d) Washington D.C(e) None of these

21. In which of the following states the Ganga Gram Yojana hasbeen launched?(a) Uttarakhand (b) Bihar(c) West Bengal (d) Uttar Pradesh(e) None of these

22. For what purpose, the Stand Up India Scheme has beenlaunched?(a) To promote entrepreneurship among SC/ST(b) To promote entrepreneurship among Rural Youth(c) To promote entrepreneurship among Woman(d) To promote entrepreneurship among SC/ST and

Women(e) None of these

23. Who of the following is the author of the novel titled - GoSet a Watchman?(a) Cynthia Lord (b) Bill Clegg(c) Marlon James (d) Harper Lee(e) None of these

24. Which of the following companies publishes PurchasingManagers' Index (PMI)?(a) National Stock Exchange of India Limited(b) BSE Institute Limited (c) Markit(d) HSBC Analytics (e) None of these

25. The 'Fountain of Oneness' is located in which of thefollowing cities?(a) Ajmer (b) Delhi(c) Agra (d) Khajuraho(e) None of these

26. Which of the following cities to get India's first river gallery?(a) Bengaluru (b) Mysore(c) Chennai (d) Hyderabad(e) None of these

27. The Indian space agency is soon opening a 100-acre SpacePark in which of the following cities?(a) Bengaluru (b) Nagpur(c) Mumbai (d) New Delhi(e) None of these

28. The World Day of War Orphans is observed on which ofthe following day?(a) 7th January (b) 6th January(c) 8th January (d) 9th January(e) None of these

29. Which of the following companies has won 26th Lal BahadurShastri Hockey tournament?(a) Coal India Limited(b) Oil and Natural Gas Corporation (ONGC)(c) Indian Oil Corporation Limited(d) Steel Authority of India Limited(e) None of these

30. Which of the following cities have made it to a top 30 list ofthe world's most powerful, productive and connected cities,as per the study conducted by international real estateconsultancy JLL?(a) Chennai and Bangalore(b) Kolkata and pune(c) Delhi and Mumbai(d) Delhi and Bangalore(e) None of these

31. Which of the following organisations has launched JeevanLabh Scheme?(a) New India Insurance Ltd(b) National Insurance Company Ltd(c) State Bank of India(d) Life Insurance Corporation of India(e) None of these

32. Who of the following is the author of the book titled 'JinnahOften Came to Our House?(a) Jaswant Singh (b) Kiran Doshi(c) Nisid Hajari (d) Jairam Ramesh(e) None of these

33. On what date, the World Braille Day is observed?(a) January 4 (b) December 30(c) November 9 (d) April 11(e) None of these

34. Which of the following states became the first state in thecountry to sign MoU for UDAY scheme?(a) Rajasthan (b) Gujarat(c) Uttar Pradesh (d) Jharkhand(e) None of these

35. The 14 km long Zojila tunnel will be constructed in which ofthe following states?(a) Sikkim (b) Jammu & Kashmir(c) Uttarakhand (d) Himachal Pradesh(e) None of these

36. Which of the following person appointed as Chairman ofFilm and Television Institute of India (FTII)?(a) Neha Gupta (b) Hrishikesh Kanitkar(c) Gajendra Chauhan (d) Arvind Uppal(e) None of these

Page 239: yoursmahboob.wordpress.com SBI · yoursmahboob.wordpress.com iii P 101 Speed Tests for SBI Bank Clerk Exam 101 Speed Tests for SBI Bank Clerk Exam is revised and updated edition on

yoursmahboob.w

ordpress.com

SPEED TEST 100 23137. Who has been appointed as the new chairman of

Competition Commission of India (CCI)?(a) DK Sikri (b) Amitabh Kant(c) Ashok Chawla (d) Dhanendra Kumar(e) None of these

38. In which of the following cities the world's first slum museumwill be set up?(a) Indore (b) Mumbai(c) Lucknow (d) Ahmedabad(e) None of these

39. Who has been conferred with the 2016 Global and Asia -Pacific Central Bank Governor of the Year?(a) Janet Yellen (b) Mark Carney(c) Raghuram Rajan (d) Haruhiko Kuroda(e) None of these

40. Who has been honoured with the second HarikrishnaDevsare Baalsahitya Award 2015?(a) Gangesh Gunjan (b) Sheela Jhunjhunwala(c) Balkrishna Garg (d) Sherjung Garg(e) None of these

41. In which state, the Union govt has decided to set up NationalOrganic Farming Research Institute (NOFRI) in 2016?(a) Mizoram (b) Meghalaya(c) Assam (d) Sikkim(e) None of these

42. Who was appointed as the CEO of Wipro?(a) Abid Ali Z. Neemuchwala(b) T. Kurien(c) Lalitha Kumaramangalam(d) Ramesh Sippy(e) None of these

43. Who of the following has won the 2015 Costa Novel Prize?(a) Frances Hardinge (b) Kate Atkinson(c) Andrea Wulf (d) Helen Macdonald(e) None of these

44. Vilmos Zsigmond, who passed away recently, belong towhich of the following fields?(a) Politician (b) Scholar(c) Sportsperson (d) Cinematographer(e) None of these

45. National Bank for Agriculture and Rural Development(NABARD) signed an Memorandum of Understanding(MoU) with which of the following for monitoring WatershedProjects?(a) Central Glass and Ceramic Research Institute (CGCRI)(b) National Remote Sensing Centre (NRSC)(c) Central Institute of Technology (CIT)(d) Indian Association for the Cultivation of Science

(IACS)(e) None of these

46. Raghunandan Mandal has passed away recently, he relatedto?(a) Politics (b) Scholar(c) Sportsperson (d) Cinematographer(e) None of these

47. Which of the following companies has acquired Shifu for $8million to enrich customer experience?(a) Paytm (b) Flipkart(c) Amazon (d) Myntra(e) None of these

48. Which of the following financial entity has been penalisedby the Securities and Exchange Board of India (SEBI) forbreaching the stock broker regulations?(a) R. V. Verma (b) Ashwini Kumar(c) Ravindra Kumar (d) Anand Rathi(e) None of these

49. A friendship bus service between India and was flagged offvia Champawat district in Uttarakhand.(a) Nepal (b) Japan(c) Australia (d) Canada(e) None of these

50. Who has been crowned as the FBB 50. Famina Miss Indiaworld 2016?(a) Namrata Sharma(b) Pankuri Gidwani(c) Priyadarshini chatterjee(d) Sushruthi Krishna(e) None of these

Part-II : Quantitative Aptitude

DIRECTIONS (Qs. 51-61): What will come in place of the questionmark (?) in the following questions ?51. 3 × ? + 30 = 0

(a) – 15 (b) 15(c) 10 (d) – 30(e) None of these

52. 40.83 × 1.02 × 1.2 = ?(a) 49.97592 (b) 41.64660(c) 58.7952 (d) 42.479532(e) None of these

53.1 3 1 223 6 1 ?3 7 2 7

(a) 4.4 (b)227

(c)522

(d) 40.5

(e) None of these

54. 1.5625 ?(a) 125 (b) 12.5(c) 1.05 (d) 1.25(e) None of these

55. 3978 + 112 × 2 = ? 2(a) 8400 (b) 8406(c) 8600 (d) 8404(e) None of these

Page 240: yoursmahboob.wordpress.com SBI · yoursmahboob.wordpress.com iii P 101 Speed Tests for SBI Bank Clerk Exam 101 Speed Tests for SBI Bank Clerk Exam is revised and updated edition on

yoursmahboob.w

ordpress.com

SPEED TEST 100232

56.23.7 1.3 ?10 10 10

(a) 6 (b) 7(c) 5 (d) 3(e) None of these

57. 7589 – ? = 3434(a) 721 (b) 4055(c) 3246 (d) 11023(e) None of these

58. 1225 ? 5(a) 3 (b) 2(c) 35 (d) 7(e) None of these

59. 300 + 102 × 2 = ?(a) 450 (b) 800(c) 550 (d) 320(e) None of these

60.5 1.6 2 1.4 ?

1.3(a) 4 (b) 0.4(c) 1.4 (d) 1.2(e) None of these

61.2 1 13 7 5 ?5 5 4

(a)35

10 (b)35

20

(c)75

10 (d)11520

(e) None of these62. If one-third of a number is 3 more than one -fourth of the

number, then the number is :(a) 18 (b) 24(c) 30 (d) 36(e) None of these

63. A boy was asked to write 25 92 but he wrote 2592. Thenumerical difference between the two is:(a) 0 (b) 3(c) 2 (d) 9(e) None of these

64. If the two numbers are respectively 20% and 50% of a thirdnumber, what is the percentage of the first number to thesecond ?(a) 10 (b) 20(c) 30 (d) 40(e) None of these

65. A man gains 10% by selling a certain article for a certainprice. If he sells it at double the price, then the profit madeis:(a) 120% (b) 60%(c) 100% (d) 80%(e) None of these

66. A, B and C enter into a partnership with investments of` 3500, 4500 and 5500, respectively. In the first six months,profit is 405. What is A’s share in the profit ?(a) ` 200 (b) ` 105(c) ` 250 (d) ` 151(e) None of these

67. A tap can fill a cistern in 8 hours and another tap can emptyit in 16 hours. If both the taps are opened simultaneously,the time taken (in hours) to fill the cistern will be :(a) 8 (b) 10(c) 16 (d) 24(e) None of these

68. Pipes A and B can fill a tank in 5 and 6 hours, respectively.Pipe C can empty it in 12 hours. The tank is half full. All thethree pipes are in operation simultaneously. After how muchtime, the tank will be full ?

(a)1793 h (b) 11 h

(c)1182 h (d)

17131 h

(e) None of these69. If the sum of the digits of an even number is divisible by 9,

then that number is always divisible by :(a) 24 (b) 12(c) 18 (d) 27(e) None of these

70. A water tank in the form of a cuboid has its base 20 m long,7 m wide and 10 m deep. Initially, the tank is full but laterwhen water is taken out of it, the level of water in the tankreduces by 2 m. The volume of water left in the tank is :(a) 1120 m3 (b) 400 m3

(c) 280 m3 (d) 140 m3

(e) None of theseDIRECTIONS (Qs. 71-75) : Find the next term in the given seriesin each of the questions below.71. 41, 31, ?, 17, 11, 5

(a) 19 (b) 21(c) 23 (d) 27(e) None of these

72. 8, 15, 28, 53, ?(a) 106 (b) 98(c) 100 (d) 102(e) None of these

73. 24, 49, ?, 94, 15, 31, 59, 58(a) 51 (b) 63(c) 77 (d) 95(e) None of these

74. 5, 10, 13, 26, 29, 58, ?, 122(a) 60 (b) 61(c) 111 (d) 91(e) None of these

75. 2, 3, 10, 15, 26, ?, 55(a) 32 (b) 33(c) 34 (d) 35(e) None of these

Page 241: yoursmahboob.wordpress.com SBI · yoursmahboob.wordpress.com iii P 101 Speed Tests for SBI Bank Clerk Exam 101 Speed Tests for SBI Bank Clerk Exam is revised and updated edition on

yoursmahboob.w

ordpress.com

SPEED TEST 100 233

DIRECTIONS (Qs. 76-80) : What approximate value should comein place of the question mark (?) in the following questions?(You are not expected to calculate the exact value).

76. 3 860000 ?(a) 75 (b) 80(c) 110 (d) 125(e) 95

77.5 1 2

1 5 2 ?8 3 5

(a) 15 (b) 4(c) 19 (d) 9(e) 21

78. 8769 82 4 ?(a) 27 (b) 44(c) 429 (d) 12(e) 512

79. ? % of 45.999 × 16% of 83.006 = 116.073(a) 6 (b) 24(c) 19 (d) 30(e) 11

80. 12.998 × 27.059 × 17.999 = ?(a) 6020 (b) 6320(c) 6800 (d) 6540(e) 6150

81. The area of a circular plot is twice the area of a rectangularplot. If the area of the rectangular plot is 11088 sq. metres.,what is the perimeter of the circular plot?(a) 484 metres (b) 572 metres(c) 528 metres (d) 440 metres(e) None of these

82. The sum of the two digits of a two-digit number and thedifference between the two digits of the two-digit number is8. What is the two digit number?(a) 80 (b) 88(3) 44 (d) Cannot be determined(e) None of these

83. The total number of students studying in a college is 4220.If the number of girls studying in the college is 2420, what isthe respective ratio of the number of boys to the number ofgirls studying in the college?(a) 90 : 131 (b) 90 : 121(c) 121 : 70 (d) 121 : 80(e) None of these

84. In how many different ways can the letters of the word'SCENIC' be arranged?(a) 120 (b) 720(c) 60 (d) 360(e) None of these

85. The cost of 4 bags and 12 purses is 1,520, what is the costof 10 bags and 30 purses?(a) ` 3,600 (b) `3,500(c) `4,000 (d) `3,900(e) None of these

DIRECTION (Qs. 86-90) : Study the following graph carefully toanswer the question given below it.

Production of paper (in lakh tonnes) by 3 differentcompanies A, B & C over the years

2010 2011 2012 2013 2014 2015

A B C

0

10

30

40

50

6070

20

5055

4540

60

5055

5060

45

5560 60

5045

5055

40

86. What is the difference between the production of companyC in 2010 and the production of Company A in 2015?(a) 50,000 tonnes (b) 5,00,00,000 tonnes(c) 50,00,000 tonnes (d) 5,00,000 tonnes(e) None of these

87. What is the percentage increase in production of CompanyAfrom 2011 to 2012?(a) 37.5 (b) 38.25(c) 35 (d) 36(e) None of these

88. For which of the following years the percentage of rise/fallin production from the previous year the maximum forCompany B?(a) 2011 (b) 2012(c) 2013 (d) 2014(e) 2015

89. The total production of Company C in 2012 and 2013 iswhat percentage of the total production of Company A in2010 and 2011?(a) 95 (b) 90(c) 110 (d) 115(e) None of these

90. What is the difference between the average production peryear of the company with highest average production andthat of the company with lowest average production in lakhtonnes?(a) 3.17 (b) 4.33(c) 4.17 (d) 3.33(e) None of these

91. The 63% of a number is 2583. What will be 45% of thisnumber?(a) 1845 (b) 1763(c) 2255 (d) 1927(e) None of these

Page 242: yoursmahboob.wordpress.com SBI · yoursmahboob.wordpress.com iii P 101 Speed Tests for SBI Bank Clerk Exam 101 Speed Tests for SBI Bank Clerk Exam is revised and updated edition on

yoursmahboob.w

ordpress.com

SPEED TEST 100234

92. If the fractions 4 6 2 9 3, , , and5 7 9 11 8

be arranged in descending

order, the fourth fraction will be

(a)9

11 (b)45

(c)38

(d)67

(e) None of these93. A shopkeeper purchased 245 pieces of an article at `30 per

piece. He spent 980 on transport and `1470 on packing thearticles. He sold the articles at the rate of `50 per piece.What is the per cent profit earned?(a) 25 (b) 20(c) 28 (d) 22.5(e) None of these

94. Difference between the compound interest and simpleinterest accrued in two years at 8% per annum is 128.What is the principal amount?(a) `18000 (b) `16000(c) `20000 (d) Cannot be determined(e) None of these

95. The number obtained by interchanging the digits of a twodigit number is less than the original number by 18. If sumof the digits is 6, what was the original two digit number?(a) 51 (b) 24(c) 42 (d) 15(e) None of these

96. Samar spends 52% of his monthly salary on householdexpenditure and 23% on miscellaneous expenditure. If he isleft with `4500, what is his monthly salary?(a) `16000 (b) `17500(c) `17000 (d) `18500(e) None of these

97. Number of girls in a class is 44 which is 55% of the totalnumber of students. How many boys are there in the class?(a) 75 (b) 76(c) 84 (d) 36(e) None of these

98. Average marks obtained in History by student in ‘A’ divisionare 75 and the average marks obtained by 40 students in ‘B’division are 78. What are the overall average marks obtainedin History by the students in both the divisions (roundedoff to two digits after decimal)?(a) 76.81 (b) 77.11(c) 77.41 (d) 76.41(e) None of these

99. The ratio between length and breadth of a rectangular plotis 5 : 3 respectively and its perimeter is 48 m. What will be itsarea in sq m?(a) 120 (b) 116(c) 115 (d) 76.41(e) None of these

100. In a class of 63 children, the children are seated in rows andcolumns such that there are two more children in each

column than the number of children seated in each row.How many children are there in each row?(a) 9 (b) 7(c) 11 (d) 13(e) None of these

Part-III : Reasoning AbilityDIRECTIONS (Qs. 101-103) : Study the following paragraphand answer the questions that follow :

In our society, corruption and abuse of office has beenaggravated by three factors. First, there is a colonial legacy ofunchallenged authority and propensity to exercise powerarbitrarily.

In a society which worships power, it is easy for publicofficials to deviate from ethical conduct. Second, there is enormousasymmetry of power in our society. Nearly 90% of our people arein the unorganized sector. Quite a number of them lead a precariousexistence, depending on subsistence wages with no job security.And nearly 70% of the organized workers with job security andregular monthly wages are employed by the state directly orthrough public sector undertakings. Almost all these employeesare 'educated' in a largely illiterate and semiliterate society andeconomically even the lowliest of public servants are better offthan most people in the country. What is more, their employmentin government comes with all the trappings of power. Suchasymmetry of power reduces societal pressure to conform toethical behaviour and makes it easy to indulge in corruption101. The passage best support the statement that–

(a) Corruption is a consequence of hundred, gears slavaryof the country.

(b) Among the many causes socio-economic disparity isthe main cause of corruption.

(c) Corruption is a status symbol in our society.(d) Mostly it is found that people themselves incourage

the corruption.(e) None of these

102. Which of the following is an inference which can be madefrom the facts stated in the above passage?(a) Corruption is the consequences of can challenged

authority, asymmetry of power and illiteracy etc.(b) People are helpless before the corruption.(c) The government should make a strict law against the

corruption.(d) People should stir a movement against corruption.(e) None of these

103. Which of the following is the most effective statement inthe context of the above passage?(a) Asymmetry of power reduces societal pressure to

conform to ethical behaviour and makes it easy toindulge in corruption.

(b) Society itself nourishes the corruption.(c) Power is worshiped in our society which increases the

corruption.(d) 90% people are in unorganised sector and they always

feel the insecurity of jobs.(e) None of these.

Page 243: yoursmahboob.wordpress.com SBI · yoursmahboob.wordpress.com iii P 101 Speed Tests for SBI Bank Clerk Exam 101 Speed Tests for SBI Bank Clerk Exam is revised and updated edition on

yoursmahboob.w

ordpress.com

SPEED TEST 100 235104. If ‘+’ means ‘divided by’; ‘–’ means ‘added to’; ‘×’ means

‘substracted from’ and ‘÷’ means ‘multiplied by’; then 26 –15 + 5 × 4 ÷ 2 = ?(a) 15 (b) 20(c) 25 (d) 18(e) None of these

105. Four of the following five are alike in a certain way and so forma group. Which is the one that does not belong to that group.(a) RPN (b) WSU(c) HDF (d) LHJ(e) QMO

106. In a row of forty students R is the fifth from right end andthere are ten students between R and D. What is D’s positionfrom the left end of the row ?(a) 26th (b) 23rd(c) 24th (d) 25th(e) Data inadequate

107. In a certain code ORBITAL is written as CSPHMBU. How isCHARGER written in that code ?(a) BIDQSFH (b) BIDSSFH(c) BIDQQDF (d) DIBQSFH(e) None of these

108. Among A, B, C, D and E each having scored different marksin an examination, B scored more than C and E and less thanA and D. C’s marks are not the lowest. Who scored thelowest marks ?(a) O (b) C(c) B (d) Data inadequate(e) None of these

109. How many such pairs of letters are there in the wordOVERWHELM each of which has as many letters betweenthem in the word as in the English alphabet?(a) None (b) One(c) Two (d) Three(e) More than three

110. In a certain code BROWN is written as 531 @% and MEANis written as 26©%. How is ROBE written in that code ?(a) 3@16 (b) 3516(c) 3156 (d) 3©16(e) None of these

DIRECTIONS (Qs. 111-113) : Study the following arrangementcarefully and answer the questions given below :

R 5 # 3 $ M D P 1 4 F © A 6 E W J 2 @ K 8 Q 7% U T I * V 9

111. Which of the following is the seventh to the left of thenineteenth from the left end of the above arrangement ?(a) D (b) U(c) © (d)(e) None of these

112. What should come next in the following series based on theabove arrangement ?53$ P4F 6WJ 8Q ?(a) UI * (b) UT *(c) UIV (d) UTV(e) None of these

113. How many such vowels are there in the above arrangementeach of which is immediately preceded by a symbol andimmediately followed by a number ?(a) None (b) One(c) Two (d) Three(e) More than three

DIRECTIONS (Qs. 114-119) : In the following questions, thesymbols , $, *, @ and © are used with the following meaningsillustrated.

‘P $ Q’ means ‘P is neither equal to nor greater than Q’.‘P © Q’ means ‘P is neither equal to nor smaller than Q’.‘P Q’ means ‘P is neither greater to nor smaller than Q’.‘P @ Q’ means ‘P is not smaller than Q’.‘P * Q’ means ‘P is not greater than Q’.In each question three statements showing relationship have beengiven, which are followed by three conclusions I, II and III.Assuming that the given statements are true, find out whichconclusion(s) is/are definitely true.114. Statements : M @ D, D K, K © R

Conclusions : I. R $ M II. K M III. K $ M

(a) Only I is true (b) Only II is true(c) Only III is true (d) Either II or III and I are true(e) Either II or III is true

115. Statements : F * T, T $ N, N @ RConclusions : I. R $ T

II. N © F III. F $ R

(a) None is true (b) Only I is true(c) Only II is true (d) Only III is true(e) II and III are true

116. Statements : B © N, N @ R, F * RConclusions : I. B © M

II. F * N III. R $ B

(a) I and II are true (b) I and III are true(c) II and III are true (d) All I, II and III are true(e) None of the above

117. Statements : D $ M, M * B, B JConclusions : I. J © D

II. B * D III. J $ M

(a) I and II are true (b) I and III are true(c) II and III are true (d) All I, II and III are true(e) None of the above

118. Statements : W K, K © F, F $ MConclusions : I. M © K

II. W @ F III. F @ W

(a) Only I is true (b) Only II is true(c) Only III is true (d) II and III are true(e) None is true

Page 244: yoursmahboob.wordpress.com SBI · yoursmahboob.wordpress.com iii P 101 Speed Tests for SBI Bank Clerk Exam 101 Speed Tests for SBI Bank Clerk Exam is revised and updated edition on

yoursmahboob.w

ordpress.com

SPEED TEST 100236

119. Statements : F @ T, T K, K * DConclusions : I. D @ F

II. F @ K III. D @ T

(a) II and III are true (b) I and III are true(c) I and II are true (d) All I, II and III are true(e) None of the above

DIRECTIONS (Q. 120-125) : Study the following informationcarefully and answer the questions given below

P, A, D, Q, T, M, R and B are sitting around a circle facing atthe centre. D is third to the left of T who is fifth to the right of P. Ais third to the right of B, who is second to the right of D. Q issecond to the left of M.120. Who is to the immediate right of D ?

(a) M (b) Q(c) B (d) Data inadequate(e) None of these

121. Who is second to the right of M ?(a) B (b) R(c) T (d) Q(e) None of these

122. Who is second to the left of D ?(a) A (b) Q(c) B (d) P(e) Data inadequate

123. Who is third to the right of P ?(a) D (b) M(c) R (d) Data inadequate(e) None of these

124. Which of the following pairs represents the immediateneighbours of A ?(a) PT (b) PB(c) TQ (d) PD(e) None of these

125. In which of the following pairs the first person is sitting tothe immediate right of the second person ?(a) DM (b) BT(c) RA (d) PQ(e) PA

DIRECTIONS (Qs. 126-131) : In each of question below are threestatements followed by three conclusions numbered I, II and III.You have to take the three given statements to be true even if theyseem to be at variance from commonly known facts and thendecide which of the given conclusions logically follows from thethree given statements disregarding commonly known facts. Thendecide which of the answers (a), (b), (c), (d) and (e) is the correctanswer and indicate it on the answer sheet.126. Statements : All cups are benches. Some benches are drums.

All drums are kites.Conclusions : I. Some kites are cups.

II. Some kites are benches. III. Some drums are cups.

(a) None follows (b) Only I follows(c) Only II follows (d) Only III follows(e) II and III follow

127. Statements : Some bikes are cars. Some cars are trains. Sometrains are buses.

Conclusions : I. Some buses are cars. II. Some trains are bikes. III. Some buses are bikes.

(a) None follows (b) Only I follows(c) Only II follows (d) Only III follows(e) I and II follow

128. Statements : All dogs are cats. Some cats are rats. All ratsare mats.

Conclusions : I. Some mats are cats. II. Some mats are dogs. III. Some rats are cats.

(a) Only I follows (b) Only II follows(c) Only III follows (d) I and III follow(e) None of the above

129. Statements : All pens are sticks. All sticks are rings. Allrings are rods.

Conclusions : I. Some rings are pens. II. Some rods are sticks. III. Some rods are pens.

(a) I and II follow (b) I and III follow(c) II and III follow (d) All I, II and III follow(e) None of the above

130. Statements : Some tables are chairs. All chairs are houses.All houses are tents.

Conclusions : I. All houses are chairs. II. Some tents are chairs. III. Some houses are tables.

(a) I and II follow (b) I and III follow(c) II and III follow (d) All I, II and III follow(e) None of the above

131. Statements : Some boxes are walls. No wall is road. All roadsare rivers.

Conclusions : I. Some rivers are walls. II. Some roads are boxes.

III. No wall is river.(a) Only I follows (b) Either I or III follows(c) Only III follows (d) Only II follows(e) II and III follow

132. In a certain code MAIN is written as '9364' and DEAR iswritten as '8532'. How is MEND written in that code?(a) 9548 (b) 9458(c) 9538 (d) 9528(e) None of these

133. In a certain code DREAMING is written as BFSEFMHL.'How is TREATISE written in that code?(a) USFBDRHS (b) BFSUDTHS(c) BFSUSHRD (d) BDQSDRHS(e) None of these

134. The positions of how many digits in the number 5314697will remain unchanged if the digits are rearranged inascending order within the number?(a) None (b) One(c) Two (d) Three(e) More than three

Page 245: yoursmahboob.wordpress.com SBI · yoursmahboob.wordpress.com iii P 101 Speed Tests for SBI Bank Clerk Exam 101 Speed Tests for SBI Bank Clerk Exam is revised and updated edition on

yoursmahboob.w

ordpress.com

SPEED TEST 100 237135. Among A, B, C, D and E each having different amount of

money, C has more money than only E. B has more moneythan D but less than A. Who among them has the highestamount of money?(a) B (b) A(c) D (d) Data inadequate(e) None of these

136. Prakash walked 30 metres towards West, took a left turn andwalked 20metres. He again took a left turn and walked 30metres. He then took a right turn and stopped. Towardswhich direction was he facing when he stopped?(a) South (b) North(c) East (d) Data inadequate(e) None of these

137. How many meaningful English words can be made with theletters RTOU using each letter only once in eachword?(a) None (b) One(c) Two (d) Three(e) More than three

138. If 'P' denotes '–'; 'Q' denotes ' ', 'R' de notes '×' and 'W'denotes '+' then-48 Q 12 R 10 P 8 W 4=?(a) 56 (b) 40(c) 52 (d) 44(e) None of these

139. Which of the following is the middle digit of the secondhighest number among the five three-digit numbers givenbelow ?

512 739 428 843 654(a) 1 (b) 3(c) 2 (d) 4(e) 5

140. In a certain code language 'green grass everywhere' is writtenas 'dik pa sok' and 'cow eats grass' is written as 'nok ta pa'.How is 'cow' written in that code language?(a) nok (b) ta(c) nok or ta (d) Data inadequate(e) None of these

141. A program designed to destroy data on your computer whichcan travel to “infect” other computers is called a(a) disease (b) torpedo(c) hurricane (d) virus(e) None of these

142. What is the most common way to get a virus in yourcomputer’s hard disk ?(a) By installing games from their CDROMS(b) By uploading pictures from mobile phones to the

computer(c) By opening e-mails(d) By sending e-mails(e) None of these

143. What is the main folder on a storage device called ?(a) platform (b) interface(c) root directory (d) device driver(e) None of these

144. A hard copy of a file created on a computer refers to data(a) saved on a floppy disk(b) printed on a printer(c) backed up on a tape drive(d) sent as an e-mail(e) None of these

145. When you quickely press and release the left mouse buttontwice, you are(a) Primary – clicking (b) Pointing(c) Double – clicking (d) Secondary – clicking(e) None of these

146. Underlined text, such as text and folder names is referred toas a(a) icon (b) hyperlink(c) menu (d) source drive(e) None of these

147. To centre a paragraph using shortcut keys, press(a) CTRL + C (b) CTRL + E(c) CTRL + L (d) CTRL + R(e) None of these

148. Word processing, spreadsheet, and photo-editing areexamples of(a) application software(b) system software(c) operating system software(d) platform software(e) None of these

149. Which of the following displays to the right of the spacewhere the text will be inserted when you type?(a) Screen tip (b) Insertion point(c) Rulers (d) Office assistant(e) None of these

150. In a spreadsheet, a cell is defined as the(a) intersection of a table and a tuplet(b) intersection of a file and a database(c) intersection of a row and column(d) intersection of a field and a record(e) None of these

Part-IV : English Language

DIRECTIONS (Q. 151-160) : Read the following passagecarefully and answer the questions given below it. Certain words/phrases have been printed in bold to help you locate them whileanswering some of the questions.

There was a country long time ago where the people wouldchange a king every year. The person who would become theking had to agree to a contract that he would be sent to an islandafter one year of his being a king.

One king had finished his term and it was time for him to goto the island and live there. The people dressed him up in expensiveclothes and put him on an elephant and took him around the citiesto say goodbye to all the people. This was a moment of sadnessfor all kings who ruled for one year. After bidding farewell, thepeople took the king to a remote island in a boat and left him there.On their way back, they discovered a ship that had sunk justrecently. They saw a young man who had survived by holding on

Page 246: yoursmahboob.wordpress.com SBI · yoursmahboob.wordpress.com iii P 101 Speed Tests for SBI Bank Clerk Exam 101 Speed Tests for SBI Bank Clerk Exam is revised and updated edition on

yoursmahboob.w

ordpress.com

SPEED TEST 100238

to a floating piece of wood. As they needed a new king, theypicked up the young man and took him to their country. Theyrequested him to be king for a year. First he refused but later heagreed to be the king. People told him about all the rules andregulations and that how he would be sent to an island after oneyear.

After three days of being a king, he asked the ministers ifthey could show him the island where all the other kings weresent. They agreed and took him to the island. The island wascovered with a thick Jungle and sounds of vicious animals wereheard coming out of it. The king went a little bit further to check.Soon he discovered dead bodies of all the past kings. Heunderstood that as soon as they were left on the island, the wildanimals had come and killed them.

The king went back to the country and collected 100 strongworkers. He took them to the island and instructed them to cleanthe jungle, remove all the deadly animals and cut down all excesstrees. He would visit the island every month to see how the workwas progressing. In the first month, all the animals were removedand many trees were cut down. In the second month, the wholeisland was cleaned out. The king then told the workers to plantgardens in various parts of the island. He also took with himselfuseful animals like chickens, ducks, birds, goats, cows etc. In thethird month, he ordered the workers to build big houses anddocking stations for ships. Over the months, the island turnedinto a beautiful place. The young king would wear simple clothesand spend very little from his earnings as a king. He sent all theearnings to the island for storage. When nine months passed likethis, the king called the ministers and told them: “I know that Ihave to go the island after one year, but I would like to go thereright now.” But the ministers didn’t agree to this and said that hehad to wait for another three months to complete the year.

Three months passed and now it was a full year. The peopledressed up the young king and put him on an elephant to take himaround the country to say goodbye to others. However, this kingwas unusually happy to leave the kingdom. People asked him,“All the other kings would cry at this moment. Why is it that youare laughing?” He replied, “Don’t you know what the wise peoplesay? They say that when you come to this world as a baby, youare crying and everyone else is smiling. Live such a life that whenyou die, you will be smiling and everyone around you will becrying. I have lived that life. While all the other kings were lostinto the luxuries of the kingdom, I always thought about the futureand planned for it. I turned the deadly island into a beautifulabode for me where I can stay peacefully.”151. Why did the people of the kingdom change the king every

year?(a) As their first king had invented this system and had

recorded it in the form of a contract(b) As they believed that the new king would bring better

ideas to the kingdom.(c) As they wanted their king to relax on an island after

one year of hard work(d) Not mentioned in the passage(e) None of the above

152. What would happen to the king once his term of one yearwas over ?(a) He would be paraded in the cities as a farewell and

then be taken to a remote island

(b) He would be gifted with expensive clothes andjewellery

(c) He would be sent on an island which required a lot ofwork to be done

(d) He would be asked to buy an elephant and go to theremote island himself

(e) None of the above153. What did the young man notice on his visit to the remote

island after three days of being king?(a) That the animals on the island were too many to be

killed(b) That the island was very big(c) That the jungle on the island was full of animals that

had killed all the previous kings placed on the island(d) That the island was very beautiful and clean(e) Not mentioned in the passage

154. What happened to the island in the first month of the youngking’s tenure?(A) The wild animals were removed from the island.(B) The whole island was cleaned.(C) Many unnecessary trees were cut down.(a) Only A (b) Only B(c) Only C (d) B and C(e) A and C

155. What could be said about the island after the young kinghad made his servants work on it ?(a) The dangerous island had been turned to a beautiful

place(b) The island still remained the same despite all the young

king’s efforts(c) The island was barely tolerable now(d) The island was safe from all types of attacks(e) None of the above

156. Which of the following describes the young king correctly?(A) He was intelligent(B) He had foresight(C) He was cunning(a) Only A (b) A and B(c) Only C (d) B and C(e) All A, B and C

157. What was the king’s request to the ministers after thecompletion of nine months?(a) That he should not be sent to the island after

completion of one year(b) That he would like to go to the island immediately(c) That he should be paid more in order to improve work

on the island(d) That he should be sent to the island after a year(e) None of the above

158. How did the young king arrange for money on the island?(a) By selling a large amount of his property(b) By spending all the money derived from his income as

a king(c) By borrowing money from the ministers and sending it

to the island(d) By spending very less of his income as a king and

sending it to the island for storage(e) None of the above

Page 247: yoursmahboob.wordpress.com SBI · yoursmahboob.wordpress.com iii P 101 Speed Tests for SBI Bank Clerk Exam 101 Speed Tests for SBI Bank Clerk Exam is revised and updated edition on

yoursmahboob.w

ordpress.com

SPEED TEST 100 239159. Why were the people of the kingdom puzzled when the

young king was taken around the country to say goodbyeto everyone?(a) As they could not believe that one year had elapsed

so soon(b) As they were nor aware that the young king was

actually a wise sage(c) As the young king was happy to go to the island unlike

the previous kings(d) Not mentioned in the passage(e) None of the above

160. What can possibly be the moral of the story?(a) Always put others before yourself(b) Give respect to others(c) Live in the present and forget about the future(d) Do not put things off until tomorrow(e) Always think and plan ahead

DIRECTIONS (Q. 161-163): Choose the word/group of wordswhich is most similar in meaning to the word/group of wordsprinted in bold as used in the passage.

161. CONTRACT(a) work (b) signature(c) deal (d) temporary(e) get

162. BIDDING(a) wishing (b) auctioning(c) wasting (d) playing(e) talking

163. ABODE(a) stop (b) mountain(c) plenty (d) house(e) dwelling

DIRECTIONS (Q. 164-165) : Choose the word / group of wordswhich is most opposite in meaning to the word / group of wordsprinted in bold as used in the passage.164. SURVIVED

(a) scratched (b) died(c) lived (d) fell(e) suffered

165. VICIOUS(a) simple (b) small(c) tough (d) harmless(e) ferocious

DIRECTIONS (Q. 166-170) : Rearrange the following sixsentences (A), (B), (C), (D), (E) and (F) in the proper sequenceto form a meaningful paragraph; then answer the questions givenbelow them.(A) All of a sudden the mother duck saw a fox in the distance,

was frightened and shouted, “Children, hurry to the lake,there’s a fox !”

(B) The mother duck ran, leading the fox away from the lake andas soon as the fox came very close, the mother duck quicklyspread her wings and rose up in the air.

(C) The ducklings hurried towards the lake and the mother duckbegan to walk back and forth dragging one wing on theground.

(D) A mother duck and her little ducklings were on their way tothe lake one day.

(E) The fox stared in disbelief at the mother duck and herducklings as he could not reach the ducklings because theywere in the middle of the lake by now.

(F) When the fox saw her he became happy as he thought thatthe mother duck was hurt and couldn’t fly and that he couldeasily catch and eat her!

166. Which of the following should be the FIRST sentence afterrearrangement?(a) A (b) B(c) C (d) D(e) E

167. Which of the following should be the SECOND sentenceafter rearrangement ?(a) A (b) B(c) C (d) D(e) F

168. Which of the following should be the THIRD sentence afterrearrangement?(a) A (b) B(c) C (d) D(e) E

169. Which of the following should be the FOURTH sentenceafter rearrangement?(a) B (b) C(c) D (d) E(e) F

170. Which of the following should be the LAST (SIXTH)sentence after rearrangement?(a) A (b) B(c) D (d) E(e) F

DIRECTIONS (Q. 171-175) : Which of the phrases (a), (b), (c)and (d) given below each sentence should replace the phraseprinted in bold in the sentence to make it grammatically correct?If the sentence is correct as it is given and ‘No correction isrequired’, mark (e) as the answer.171. As it was a dark and stormy night, Lata was too scared to go

home alone.(a) very scary to (b) much scared to(c) as scared to (d) to scared too(e) No correction required

172. Since it was her engagement party, Riya was dress to kill.(a) dresses to kill (b) dressed to kill(c) dressed to killings (d) dressing to killed(e) No correction required

173. Ramesh worries endlessly about his son’s future as he wasso poor in studies.(a) worry endless (b) worried endless(c) worried endlessly (d) worries endless(e) No correction required

Page 248: yoursmahboob.wordpress.com SBI · yoursmahboob.wordpress.com iii P 101 Speed Tests for SBI Bank Clerk Exam 101 Speed Tests for SBI Bank Clerk Exam is revised and updated edition on

yoursmahboob.w

ordpress.com

SPEED TEST 100240

174. Now that the actual criminal had been caught, Kunal washappy that he was finally let of the hook.(a) off the hook (b) of the hookings(c) off the hooks (d) of the hooks(e) No correction required

175. The little boy appeared all of a sudden out of nowhere andtake everyone by surprise.(a) took everyone as surprised(b) take everyone with surprised(c) took everyone by surprises(d) took everyone by surprise(e) No correction required

DIRECTIONS (Q. 176-185) : In the following passage there areblanks, each of which has been numbered. These numbers areprinted below the passage and against each, five words aresuggested, one of which fits the blank appropriately. Find outthe appropriate word in each case.

A lamb was (176) with a flock of sheep one day. She soonfound some sweet grass at the (177) of the field. Farther andfarther she went, away from the others. She was enjoying herselfso much that she did not (178) a wolf coming nearer to her.However, when it (179) on her, she was quick to start pleading,“Please, please don’t eat me yet. My stomach is full of grass. Ifyou wait a while, I will (180) much better.” The wolf (181) thatwas a good idea, so he sat down and waited. After a while, thelamb said, “If you (182) me to dance, the grass in my stomach willbe digested faster.” Again the wolf agreed. While the lamb wasdancing, she had a new (183). She said, “Please take the bell fromaround my neck. If you ring it as hard as you can, I will be able todance even faster.” The wolf (184) the bell and rang it as hard ashe could. The shepherd heard the bell ringing and quickly senthis dogs to find the missing lamb. The (185) dogs frightened thewolf away and saved the lamb’s life.176. (a) watching (b) laughing

(c) willing (d) tiring(e) grazing

177. (a) height (b) edge(c) midst (d) first(e) base

178 (a) notice (b) trust(c) saw (d) worry(e) maintain

179. (a) lunge (b) visited(c) ate (d) stand(e) pounced

180. (a) walk (b) die(c) taste (d) mix(e) reveal

181. (a) believe (b) thought(c) wished (d) smiled(e) rejoiced

182. (a) make (b) let(c) pay (d) allow(e) request

183. (a) idea (b) luck(c) necklace (d) cry(e) presence

184. (a) opened (b) snatch(c) decorated (d) took(e) ring

185. (a) smiling (b) hurried(c) barking (d) much(e) friendly

DIRECTION (Qs. 186 to 190): In each of the following sentences,an idiomatic expression or a proverb is highlighted. Select thealternative which best describes its use in the sentence.186. He went to his friend's house in the evening as was his

wont.(a) as usual (b) as he wanted(c) as his want was (d) as he wanted that day(e) none of these

187. Why do you wish to tread on the toes?(a) To give offence to them(b) To follow them grudgingly(c) To treat them indifferently(d) To be kicked by them(e) None of these

188. He intends setting up as a lawyer in the adjoining district.(a) To establish himself (b) To migrate(c) To join (d) To settle(e) None of these

189. The autographed bat from the famous cricketer SunilGavaskar is worth a jew's eye.(a) Not a worthy possession(b) unnecessary(c) A costly items(d) A possession of high value(e) None of these

190. The speaker gave a bird's eye view of the political conditionsin the country.(a) a personal view (b) a general view(c) a biased view (d) a detailed presentation(e) None of these

Page 249: yoursmahboob.wordpress.com SBI · yoursmahboob.wordpress.com iii P 101 Speed Tests for SBI Bank Clerk Exam 101 Speed Tests for SBI Bank Clerk Exam is revised and updated edition on

yoursmahboob.w

ordpress.com

Test Code : ............................... Time taken : ......................... Date : ..........................

RESPONSE SHEET

(a) (b) (c) (d) (e)1.2.3.4.5.6.7.8.9.10.11.12.13.14.15.16.17.18.19.20.21.22.23.24.25.26.27.28.29.30.31.32.33.34.35.36.37.38.39.40.41.42.43.44.45.46.47.48.49.50.

(a) (b) (c) (d) (e)51.52.53.54.55.56.57.58.59.60.61.62.63.64.65.66.67.68.69.70.71.72.73.74.75.76.77.78.79.80.81.82.83.84.85.86.87.88.89.90.91.92.93.94.95.96.97.98.99.100.

(a) (b) (c) (d) (e)101.102.103.104.105.106.107.108.109.110.111.112.113.114.115.116.117.118.119.120.121.122.123.124.125.126.127.128.129.130.131.132.133.134.135.136.137.138.139.140.141.142.143.144.145.

(a) (b) (c) (d) (e)146.147.148.149.150.151.152.153.154.155.156.157.158.159.160.161.162.163.164.165.166.167.168.169.170.171.172.173.174.175.176.177.178.179.180.181.182.183.184.185.186.187.188.189.190.

Page 250: yoursmahboob.wordpress.com SBI · yoursmahboob.wordpress.com iii P 101 Speed Tests for SBI Bank Clerk Exam 101 Speed Tests for SBI Bank Clerk Exam is revised and updated edition on

yoursmahboob.w

ordpress.com

SPEED TEST 100242

Page 251: yoursmahboob.wordpress.com SBI · yoursmahboob.wordpress.com iii P 101 Speed Tests for SBI Bank Clerk Exam 101 Speed Tests for SBI Bank Clerk Exam is revised and updated edition on

yoursmahboob.w

ordpress.com

Part-I : General Awareness1. On which one of the following issues can SEBI penalize any

company in India?(A) Violation of Banking Regulation Act.(B) Violation of foreign portfolio investment guidelines.(C) For violation of Negotiable Instrument Act.(a) Only (A) (b) All (A), (B) & (C)(c) Only (A) & (B) (d) Only (B) & (C)(e) Only (B)

2. Expand the term ALM as used in Banking/Finance sector?(a) Asset Liability Mismatch(b) Asset Liability Maturity(c) Asset Liability Management(d) Asset Liability Manpower(e) None of the above

3. What is an ISO series?(a) Documentation of production processes(b) Engineering process flowchart(c) Quality management and quality assurance standards(d) All of the above(e) None of these

4. The Federation of Indian Chambers of Commerce andIndustry (FICCI) was founded in 1927 by(a) Birla and Tata (b) Tata and Thakurdas(c) Thakurdas and Birla (d) Tata and Godrej(e) None of these

5. Every year March 20 is celebrated as what day?(a) World Sparrow Day(b) International Women’s Day(c) World Cuckoo Day(d) International Child Day(e) International Mother’s Day

6. Which of the following is not a Central Government tax?(a) Income tax (b) Customs(c) Land revenue (d) Corporation tax(e) None of these

7. The expansion for the BIS, in the context of the bankingindustry is(a) Bank for International Settlements(b) Bank for Industrial Settlements(c) Bank for Industrial Sectors(d) Bank for International Services(e) None of these

8. Which of the following statement is true?(a) Banks cannot accept demand and time deposite from

public.(b) Banks can accept only demand deposits from public.(c) Banks can accept only time deposits from public.(d) Banks can accept both demand and time deposits from

public.(e) Banks can accept demand and time deposits only from

government.9. Interest payable on saving bank accounts is

(a) not regulated by RBI.(b) regulated by State Governments(c) regulated by Central Government(d) regulated by RBI(e) regulated by Finance minister.

10. Which of the following is the correct statement?(a) State bank of India is the sole authority to issue and

manage currency in India.(b) A nationalized bank is the sole authority to Issue and

manage currency in India(c) A cooperative bank is the sole authority to issue and

manage currency in India.(d) RBI is the sole authority to issue and manage currency

in India.(e) None of these

11. Accounts are allowed to be operated by cheques in respectof(a) Both Savings bank accounts and fixed deposit accounts(b) Savings bank accounts and current accounts(c) Both Savings bank accounts and loan accounts(d) Both Savings bank accounts and cash accounts only(e) Both Current accounts and fixed deposit accounts

12. Which of the following is correct statement?(a) Normally no interest is paid on current deposit

accounts(b) Interest is paid on current accounts at the same rate as

term deposit accounts(c) The rate of interest on current account and savings

account are the same(d) No interest is paid on any deposit by the bank(e) Savings deposits are the same as current deposits.

Max. Marks : 200 No. of Qs. 190 Time : 2 hr. 40 min. Date : ........./......../................

Full Main Test - 5 101101

Page 252: yoursmahboob.wordpress.com SBI · yoursmahboob.wordpress.com iii P 101 Speed Tests for SBI Bank Clerk Exam 101 Speed Tests for SBI Bank Clerk Exam is revised and updated edition on

yoursmahboob.w

ordpress.com

SPEED TEST 101244

13. Which of the following terms is not associated with thegame of cricket?(a) China man (b) Hook(c) Pull (d) Love(e) None of these

14. Larry page and Sergey Brin are well known as(a) Creators of Bluetooth device(b) Founders of Google(c) Stem cell researchers(d) Scientists(e) None of these

15. The book ‘A Bend in the River’ is written by(a) VS Naipaul (b) Chetan Bhagat(c) Vikram Seth (d) Arundhati Roy(e) None of these

16. When a bank returns a cheque unpaid. It is called(a) payment of the cheque(b) drawing of the cheque(c) cancelling of the cheque(d) dishonour of the cheque(e) taking of the cheque.

17. Distribution of Insurance products and Insurance policiesby banks as corporate agents is known as(a) General Insurance (b) Non-life Insurance(c) Bancassurance (d) Insurance banking(e) Deposit Insurance

18. What does the letter 'L' denote in term 'LAF' as referredevery now and then in relation to monetary policy of theRBI?(a) Liquidity (b) Liability(c) Leveraged (d) Longitudinal(e) Linear

19. POSCO is in the process of establishing its plants in India.What does the letter 'P'. denote in the name POSCO?(a) Popular (b) Pallin(c) Pohang (d) Paradeep(e) Petersburg

20. The remuneration payable to the Attorney General of Indiais determined by?(a) President of India (b) Prime Minister(c) Law Minister (d) Parliament(e) None of these

21. Farzad B gas field is located in which of the followingcountry?(a) Iran (b) Oman(c) Saudi Arabia (d) Afghanistan(e) None of these

22. "Mission Bhagiratha" is the prestigious water grid projectin which state of India?(a) Telangana (b) Uttarakhand(c) Bihar (d) Uttar Pradesh(e) None of these

23. The RV Easwar Committee gave recommendations on:(a) Simplification of income tax laws(b) Goods and Services Tax in India(c) Banking reforms(d) Crude oil and gas pricing(e) None of these

24. The RV Easwar Committee gave recommendations onsimplification of income tax laws. . 10. Who among thefollowing is popularly known as "Krishi ka Rishi"?(a) Subhash Palekar(b) Mankombu Sambasivan Swaminathan(c) Subrahmanyan Chandrasekhar(d) Norman Ernest Borlaug(e) None of these

25. "Maa Annapurna Yojna" has been launched in which stateof India?(a) Odisha (b) Tamil Nadu(c) Gujarat (d) Kerela(e) None of these

26. What is the maximum speed of the India's first semi high-speed train "Gatimaan Express"?(a) 160 kmph (b) 170 kmph(c) 180 kmph (d) 190 kmph(e) None of these

27. World's first 'White Tiger Safari' has opened in which stateof India?(a) Rajasthan (b) Madhya Pradesh(c) Gujarat (d) West Bengal(e) None of these

28. Unakoti hill,an ancient Shaivite place of worship, is locatedin which state of India?(a) Uttarakhand (b) Tamil Nadu(c) Odisha (d) Tripura(e) None of these

29. The Central Industrial Security Force ( CISF) Raising Day iscelebrated on which date?(a) February 11 (b) March 10(c) April 26 (d) May 3(e) None of these

30. The world's tallest Jain statue is located in which state ofIndia?(a) Maharashtra (b) Gujarat(c) Karnataka (d) Uttar Pradesh(e) None of these

31. Which is the India's first digital state?(a) Odisha (b) Tamil Nadu(c) Kerala (d) Karnataka(e) None of these

Page 253: yoursmahboob.wordpress.com SBI · yoursmahboob.wordpress.com iii P 101 Speed Tests for SBI Bank Clerk Exam 101 Speed Tests for SBI Bank Clerk Exam is revised and updated edition on

yoursmahboob.w

ordpress.com

SPEED TEST 101 24532. The National Science Day is celebrated on which date in

India?(a) March 1 (b) April 21(c) February 28 (d) June 23(e) None of these

33. The NDDB foundation for Nutrition (NFN) has launchedthe 'Gift Milk' initiative in which state?(a) Telangana (b) Bihar(c) Uttar Pradesh (d) Odisha(e) None of these

34. The world's first fast-acting anti-rabies drug "RMAb" willbe launched in which country?(a) China (b) France(c) India (d) Japan(e) None of these

35. Who has been conferred with the Germany's prestigious'Cross of the Order of Merit'?(a) Arun Joshi(b) Narendra Singh Rathore(c) Goverdhan Mehta(d) Sushmita Sharma(e) None of these

36. Which company has bagged the Global CSR Excellence &Leadership Awards in the category of Best Environment-Friendly Project?(a) GVK Biosciences(b) Dr. Reddy's Laboratories(c) Shantha Biotechnics(d) Bharat Biotech(e) None of these

37. Nargol port is located in which state?(a) Maharashtra (b) Gujarat(c) West Bengal (d) Odisha(e) None of these

38. The Khajuraho Dance Festival is celebrated in which stateof India?(a) Rajasthan (b) Uttar Pradesh(c) Madhya Pradesh (d) Chhattisgarh(e) None of these

39. Which of the following committee is related to rejuvenationof Godavari river?(a) Ashok Kumar committee(b) Nandan Rai committee(c) Rekha Somalia committee(d) Deepak Naik committee(e) None of these

40. India's first ever Gender Park has been inaugurated in whichstate?(a) Odisha (b) Assam(c) Kerala (d) Tripura(e) None of these

41. The World Day of Social Justice is observed on which date?(a) February 20 (b) March 12 (c) April 23 (d) June 14(e) None of these

42. Which state government has launched the Pucca GharYojana for construction workers?(a) West Bengal (b) Gujarat (c) Odisha (d) Madhya Pradesh(e) None of these

43. Which of the following states is also known as 'The land ofdwan lit mountains'?(a) Manipur (b) Arunachal Pradesh (c) Nagaland (d) Mizoram(e) None of these

44. First 'New Silk Road' train from China has completed its tripin which city?(a) Tehran (b) Kabul(c) Islamabad (d) Ankara(e) None of these

45. The Indian Institute of Petroleum and Energy (IIPE) hasbeen established at which of the following city?(a) Visakhapatnam (b) Kochi(c) Kolkata (d) Chennai(e) None of these

46. Who has won the 2016 Formula 1 Gulf Air Bahrain GrandPrix?(a) Kimi Raikkonen (b) Lewis Hamilton(c) Nico Rosberg (d) Daniel Ricciardo(e) None of these

47. In which of the following industry CASA ratio is used?(a) Merchandise Trade (b) Banking & Finance(c) Software Industry (d) Travel & Tourism(e) None of these

48. Which among the following is a opposite activity of hedgingin share / currency /future markets? (a) arbitrage (b) speculation (c) spread (d) short(e) tall

49. Which among the following correctly categorizescollateralized borrowing and lending obligation?(a) A market operation(b) A money market instrument(c) A clearing system(d) A scheme of Reserve Bank of India(e) A policy initiative of Government of India

50. Which of the calling is the oldest joint stock Bank of India(a) Allahbad Bank (b) Bank of Baroda(c) Patiala (d) Bank of India(e) None of these

Page 254: yoursmahboob.wordpress.com SBI · yoursmahboob.wordpress.com iii P 101 Speed Tests for SBI Bank Clerk Exam 101 Speed Tests for SBI Bank Clerk Exam is revised and updated edition on

yoursmahboob.w

ordpress.com

SPEED TEST 101246

Part-II : Quantitative Aptitude

DIRECTIONS (Qs. 51-69): What will come in place of the questionmark (?) in the following questions ?

51.58 of

49 of

35 of 222 = ?

(a) 42 (b) 43(c) 39 (d) 37(e) None of these

52. 56% of 450 + ? = 300(a) 52 (b) 48(c) 42 (d) 56(e) None of these

53. 1.5 3.5 ?27 27 27(a) 5 (b) 7(c) 3 (d) 2(e) None of these

54. 27.06 × 25 – ? = 600(a) 76.3 (b) 76.7(c) 76.5 (d) 76.2(e) None of these

55.7 44 2 ?8 13

(a)1113 (b)

11113

(c)411

13 (d)3118

(e) None of these

56. 4 5 2 ?318 8 8 88

(a) 7 (b) 2(c) 3 (d) 4(e) None of these

57. – (a – b) × ? = b – a(a) – 1 (b) 1(c) – a (d) a(e) None of these

58. (a + b) = ? × (– a – b)(a) 1 (b) – a(c) – 1 (d) –b(e) None of these

59. |? + 14| = 11(a) –3 (b) –25(c) 25 (d) 3(e) Either – 3 or –25

60. 16 + 26 × 2 =?(a) 84 (b) 44(c) 40 (d) 832(e) None of these

61. Which of the following fractions is the least ?

(a)11912 (b)

101

(c)394 (d)

697

(e) None of these62. A number of points are marked on a plane and are connected

pairwise by a line segment. If the total number of linesegments is 10, how many points are marked on the plane ?(a) 4 (b) 10(c) 5 (d) 9(e) None of these

63. A sum of money becomes eight times in 3 years if the rate iscompounded annually. In how much time, the same amountat the same compound interest rate will become sixteentimes?(a) 6 years (b) 4 years(c) 8 years (d) 5 years(e) None of these

64. A machine is sold at a profit of 10%. Had it been sold for` 40 less, there would have been a loss of 10%. What wasthe cost price ?(a) ` 320 (b) ` 200(c) ` 225 (d) ` 250(e) None of these

65. Ram spends ` 3620 for buying pants at the rate of ` 480each and shirts at the rate of ` 130 each. What will be theratio of pants to shirts when maximum number of pants areto be bought ?(a) 7 : 2 (b) 7 : 3(c) 2 : 7 (d) 4 : 5(e) None of these

66. Two trains each of 120 m in length, run in opposite directionswith a velocity of 40 m/s and 20 m/s respectively. How longwill it take for the tail ends of the two trains to meet eachother during the course of their journey ?(a) 20 s (b) 3 s(c) 4 s (d) 5 s(e) None of these

67. Ramesh is twice as good a workman as Sunil and finishes apiece of work in 3 hours less than Sunil. In how many hoursthey together could finish the same piece of work ?

(a)312 (b) 2

(c)321 (d) 8

(e) None of these68. Fifteen years hence, a man will be four times as old as he

was fifteen years ago. His present age is:(a) 25 years (b) 20 years(c) 30 years (d) 45 years(e) None of these

Page 255: yoursmahboob.wordpress.com SBI · yoursmahboob.wordpress.com iii P 101 Speed Tests for SBI Bank Clerk Exam 101 Speed Tests for SBI Bank Clerk Exam is revised and updated edition on

yoursmahboob.w

ordpress.com

SPEED TEST 101 24769. The floor of a rectangular room is 15 m long and 12 m wide.

The room is surrounded by a vrandah of width 2 m on all itssides. The area of the vrandah is :(a) 124 m2 (b) 120 m2

(c) 108 m2 (d) 58 m2

(e) None of theseDIRECTIONS (Qs. 70-74) : Find the next term in the given seriesin each of the questions below.70. 2, 4, ?, 16, 32

(a) 6 (b) 10(c) 8 (d) 12(e) None of these

71. 0, 7, 26, ?, 124, 215(a) 37 (b) 51(c) 63 (d) 88(e) None of these

72. 4, 15, 16, ?, 36, 63, 64(a) 25 (b) 30(c) 32 (d) 35(e) None of these

73. 1, 8, 9, ?, 25, 216, 49(a) 60 (b) 64(c) 70 (d) 75(e) None of these

74. 336, 210, 120, ?, 24, 6, 0(a) 40 (b) 50(c) 60 (d) 70(e) None of these

DIRECTIONS (Qs. 75-82): Find out the approximate value whichis closest to the value that should replace the questions mark (?)in the following questions. (You are not expected to find out theexact value.)

75. 1223.9975 ?(a) 110 (b) 144(c) 34 (d) 12.55(e) 125

76. 503 × 201 = ?(a) 101100 (b) 1000000(c) 110000 (d) 100003(e) 1000103

77. 1205 2.5 = ?(a) 3000 (b) 4800(c) 300 (d) 480(e) 500

78. 22020 0.011 = ?(a) 20020 (b) 2002000(c) 200200 (d) 20002(e) 2000020

79. 20800 ?

(a) 12 (b) 120(c) 140 (d) 102(e) 1020

80.3 1 11 2 7 ?5 7 3

(a) 17 (b) 3(c) 9 (d) 29(e) 25

81. The height of four boys is recorded as 142 cms., 156 cms.,162 cms. and 178 cms. What is the average height of all fourboys?(a) 160 cms. (b) 158.5 cms.(c) 159.5 cms. (d) 162 cms.(e) None of these

82. Pratul's monthly income is onefourth of Manoj's monthlyincome. Manoj's annual income is ` 2.16 lacs. What isPratul's annual income? (In some cases monthly income andin some cases annual income are used.)(a) ` 54.000 (b) ` 5.4 thousand(c) ` 4.500 (d) ` 45.000(e) None of these

DIRECTIONS (83-85) : What will come in place of the questionmark (?) in the following number series?83. 17 98 147 172 181 (?)

(a) 180 (b) 192(c) 184 (d) 182(e) None of these

84. 11 19 31 47 67 (?)(a) 80 (b) 81(c) 86 (d) 96(e) None of these

85. 748 460 316 244 208 (?)(a) 180 (b) 190(c) 172 (d) 182(e) None of these

DIRECTIONS (Q. 86-90): Study the tables carefully to answerthe questions that follow:Number of candidates (in lakhs) appearing in an entranceexamination from six different cities and the ratio of candidatespassing and failing the same

City A B C D E FNumber of

1.25 3.14 1.08 2.27 1.85 2.73Candidates

Ratio of candidates passing and failing within the city.City Passing Failing

A 7 3B 5 3C 4 5D 1 3E 3 2F 7 5

Page 256: yoursmahboob.wordpress.com SBI · yoursmahboob.wordpress.com iii P 101 Speed Tests for SBI Bank Clerk Exam 101 Speed Tests for SBI Bank Clerk Exam is revised and updated edition on

yoursmahboob.w

ordpress.com

SPEED TEST 101248

86. What is the ratio of number of candidates failing the examfrom City D to that of those failing the exam from City A?(a) 289 : 42 (b) 42 : 289(c) 227 : 50 (d) 50 : 227(e) None of these

87. The number of candidates appearing for the exam from CityC is what per cent of the number of candidates appearingfor the exam from City B? (rounded off to the nearest integer)(a) 27 (b) 34(c) 42 (d) 21(e) 38

88. The number of candidates passing in the exam from City F iswhat per cent of the total number of candidates appearingfrom all the cities together ? (rounded off to two digits afterthe decimal).(a) 12.93 (b) 14.46(c) 10.84 (d) 11.37(e) None of these

89. Which city has the highest number of students failing theentrance exam?(a) F (b) C(c) B (d) D(e) None of these

90. What is the number of candidates passing the exam fromCity E?(a) 13,000 (b) 11,10,000(c) 1,13,000 (d) 11,000(e) None of these

91. Sitaram invested an amount of 7450@ 6% per annum rateof simple interest. After how many years will he obtain thetotal amount of 8791?(a) 5 yr (b) 3 yr(c) 2 yr (d) 6 yr(e) None of these

92. The difference between 36% of a number and 12% of thesame number is 82.32. What is the number?(a) 324 (b) 382(c) 343 (d) 336(e) None of these

93. The cost of 16 watches and 21 calculators is 30480. What isthe cost of 32 watches and 42 calculators?(a) `60970 (b) `60480(c) `80960 (d) Cannot be determined(e) None of these

94. If (66)2 added to the square of a number, the answer soobtained is 4840. What is the number?(a) 32 (b) 28(c) 24 (d) 18(e) None of these

95. A sum of money is divided amongst A, B and C in the ratioof 3 : 4 : 5. Another amount is divided amongst P and Q inthe respective ratio of 2 : 1. If Q got 1050 less than B, whatis the amount received by C?(a) `2850 (b) `1000(c) `1840 (d) Cannot be determined(e) None of these

96. If the value of x + y = 18 and xy = 72, what is the valueof (x)2

+ (y)2 ?(a) 324 (b) 54(c) 180 (d) Cannot be determined(e) None of these

97. Rashmi obtained a total of 484 marks out of 750 in anexamination. What is her approximate percentage in theexamination?(a) 61 (b) 56(c) 72 (d) 65(e) 70

98. What approximate amount of compound interest can beobtained on an amount of 5000 at the rate of 3% per annumat the end of 3 yr?(a) `482 (b) `464(c) `450 (e) `425(e) `478

99. Girish attends to 7 customer calls daily. Approximately, howmany calls would he attend to in the span of two months?(a) 8400 (b) 420(c) 217 (d) 7012(e) 336

100. Two number are such that the sum of twice the first numberand thrice the second number is 126 and the sum of thricethe first number and twice the second number is 144. Whatis the smaller number?(a) 18 (b) 24(c) 32 (d) 36(e) None of these

Part-III : Reasoning Ability101. In a certain code language ‘tree is very beautiful’ is written

as ‘ka na da ta’ and ‘this is stong tree’ is written as ‘na pa saka’. How is beautiful written in that code language?(a) da (b) ta(c) sa (d) Data inadequate(e) None of these

102. In a certain code ‘GIVE’ is written as 51@© and ‘FAIL’ iswritten as ‘% 219’. How is LEAF written in that code?(a) 5©2% (b) 9©2%(c) 9@2% (d) 9©1%(e) None of these

103. The positions of the first and the sixth digits in the number5109238674 and interchanged, similarly the positions of thesecond and the seventh digits are interchanged and so on.Which of the following will be the third digit from the rightend after the rearrangement?(a) 9 (b) 0(c) 6 (d) 3(e) None of these

104. Four of the following five are alike in a certain way and soform a group. Which is the one that does not belong to thatgroup?(a) Wheel (b) Tyre(c) Car (d) Door(e) Gear

Page 257: yoursmahboob.wordpress.com SBI · yoursmahboob.wordpress.com iii P 101 Speed Tests for SBI Bank Clerk Exam 101 Speed Tests for SBI Bank Clerk Exam is revised and updated edition on

yoursmahboob.w

ordpress.com

SPEED TEST 101 249105. In a certain code SUBSTANCE is written as RATRUFDOB.

How is TENTHOUSE written in that code?(a) SMDSIFTVP (b) UOFUIDRTN(c) UOFUIFTVP (d) SMDSIDRTN(e) None of these

DIRECTIONS (106-108): Following questions are based on thefive three digit numbers given below:

519 364 287 158 835106. If the positions of the first and the third digits within each

number are interchanged, which of the following will be thethird digit of the second lowest number?(a) 9 (b) 4(c) 7 (d) 8(e) 5

107. If the positions of the first and the third digits within eachnumber are interchanged, which of the following will be themiddle digit of the second highest number?(a) 1 (b) 6(c) 8 (d) 5(e) 3

108. Which of the following is the difference between the seconddigits of the highest and the lowest of these numbers?(a) 3 (b) 1(c) 2 (d) 0(e) None of these

DIRECTIONS (109-113) : Study the following informationcarefully and answer the questions given below:A, M, P, D, Q, R, W and B are sitting around a circle facing at thecentre. D is fourth to the left of A who is third to the right of M. Pis third to the left of Q who is third to the left of M. R is third to theright of W who is second to the right of B.109. Who is second to the left of D?

(a) W (b) B(c) Q (d) Data inadequate(e) None of these

110. Who is third to the left of P?(a) M (b) D(c) R (d) Data inadequate(e) None of these

111. Who is to the immediate right of Q?(a) W (b) D(c) B (d) Data inadequate(e) None of these

112. Which of the following pairs represents the first and secondrespectively to the right of W?(a) DM (b) QB(c) MR (d) Data inadequate(e) None of these

113. In which of the following pairs is the second person sittingto the immediate right of the first person?(a) MD (b) RM(c) AB (d) QB(e) None of these

DIRECTIONS (114-118): In the following questions, the symbols@, S, , © and # are used with the following meaning asillustrated below:‘P Q’ means ‘P is neither greater than nor smaller than Q’.‘P S Q’ means ‘P is neither greater than nor equal to Q’.‘P @ Q’ means ‘P is not smaller than Q’.‘P © Q’ means ‘P is not greater than Q’.‘P # Q’ means ‘P is neither smaller than nor equal to Q.’Now is each of the following questions assuming the givenstatements to be true. Find which of the three conclusions I, IIand III given below them is/are definitely true and give your answeraccordingly.114. Statements: W @ T, T © M, M S D

Conclusions: I. W # DII W @ MIII. D # T

(a) Only I is true(b) Only II is true(c) Only III is true(d) Only II and III are true(e) None of these

115. Statements: F R, R © M, M S DConclusions: I. D # R

II. D # FIII. M @ F

(a) Only I and II are true(b) Only I and III are true(c) Only II and III are true(d) All I, II and III are true(e) None of these

116. Statements: V © M, M B, B S F Conclusions: I. F # M

II. B @ VIII. F # V

(a) Only I and II are true(b) Only II and III are true(c) Only I and III are ture(d) All I, II and III are true(e) None of these

117. Statements: D # N, N @ B, B FConclusions: I. F S D

II. N # FIII N F

(a) Only I is true(b) Only II is true(c) Only III is true(d) Only either II or III is true(e) Only I and either II or III are true

Page 258: yoursmahboob.wordpress.com SBI · yoursmahboob.wordpress.com iii P 101 Speed Tests for SBI Bank Clerk Exam 101 Speed Tests for SBI Bank Clerk Exam is revised and updated edition on

yoursmahboob.w

ordpress.com

SPEED TEST 101250

118. Statements: R S T, T # K, K @ MConclusions: I. R S M

II. T # MIII. R S K

(a) None is true (b) Only I is true(c) Only II is true (d) Only III is true(e) Only II and III are true

DIRECTIONS (Qs. 119-123): Study the following arrangementcarefully and answer the questions given below:W 3 # R @ E J K T 4 B 9 1 D U 8 1 H % A V 5 7 M P 2 Q S 6

119. Which of the following is the sixth to the right of the twentyfirst from the right end of the above arrangement?(a) 8 (b) D(c) P (d) @(e) None of these

120. How many such vowels are there in the above arrangementeach of which is immediately preceded by a symbol andimmediately followed by a consonant?(a) None (b) One(c) Two (d) Three(e) Four

121. How many such symbols are there in the above arrangement,each of which is immediately followed by a number but notimmediately preceded by a number?(a) None (b) One(c) Two (d) Three(e) more than three

122. How many such consonants are there in the abovearrangement, each of which is immediately preceded by anumber and immediately followed by a symbol?(a) None (b) One(c) Two (d) Three(e) More than three

123. Four of the following five are alike in a certain way based ontheir positions in the above arrangement and so form a group.Which is the one that does not belong to that group?(a) 8 1 D (b) 7 5(c) P 2 7 (d) E J R(e) T 4 J

DIRECTIONS (124-128) : In each of the questions below aregiven three statements followed by three conclusions numberedI, II and III. You have to take the given statements to be true evenif they seem to be at variance from commonly known facts. Readall the conclusions and then decide which of the given conclusionslogically follows from the given statements disregardingcommonly known facts.

124. Statements :All keys are locks.No lock is toy.All bags are toys.

Conclusions :I. No bag is key.II. Some bags are keys.III. Some toys are keys.(a) None follows (b) Only I follows(c) Only II follows (d) Only III follows(e) Only I and II follow

125. Statements :Some days are nights.Some nights are months.Some months are years.Conclusions :I. Some years are nights.II. Some months are days.III. No year is night.(a) Only I follows (b) Only II follows(c) Only III follows (d) Only either I or II follows(e) None of these

126. Statements :All cycles are tyres.Some tyres are wheels.All wheels are buses.Conclusions :I. Some buses are tyres.II. Some wheels are tyres.IIi. Some buses are cycles.(a) Only I and II follow (b) Only I and III follow(c) Only II and III follow (d) All I, II and III follow(e) None of these

127. Statements :Some dogs are cats.Some cats are horses.All horses are tigers.Conclusions :I. Some tigers are cats.II. Some horses are dogs.III. Some tigers are dogs.(a) None follows (b) Only I follows(c) Only II follows (d) Only III follows(e) Only II and III follow

128. Statements :All ropes are sticks.Some sticks are hammers.Some hammers are lakes.Conclusions :I. Some lakes are ropes.II. Some hammers are ropes.III. Some lakes are sticks.(a) None follows (b) Only I follows(c) Only II follows (d) Only III follows(e) Only I and III follow

Page 259: yoursmahboob.wordpress.com SBI · yoursmahboob.wordpress.com iii P 101 Speed Tests for SBI Bank Clerk Exam 101 Speed Tests for SBI Bank Clerk Exam is revised and updated edition on

yoursmahboob.w

ordpress.com

SPEED TEST 101 251

DIRECTIONS (129 – 130) : In each of the questions given belowwhich one of the five answer figures on the right should comeafter the problem figures on the left, if the sequence werecontinued?129. Problem Figures

CZ S

C SC =

S =S

=S Z

==Z

CZ

ZC

Answer Figures

SS

S= =

SZ

Z CZ

CZ

S

=

C C

Z= =

C(a) (b) (c) (d) (e)

130. Problem Figures

S=

C=

=

C

=

C

C

=

Answer Figures

=C

==

C

DC

C D=

D= C

(a) (b) (c) (d) (e)

DIRECTIONS (Qs. 131-140) : Study the following paragraphand answer the questions that follow :

Election petitions in India are at present to be filed in theHigh Court. Under the Representation of the People Act, suchpetitions should be disposed of within a period of 6 months. Inactual practice however, such petitions remain pending for yearsand in the meanwhile, even the full term of the House expires thusrendering the election petition infructuous. There have beensuggestions from other high level committees and eminent personsthat a separate judicial set-up may be required. The NationalCommission to Review the Working of the Constitution (NCRWC)recommended that special election benches should be constitutedin the High Courts earmarked exclusively for the disposal ofelection petitions.131. Which of the following conclusion which can be drawn

from the facts stated in the above paragraph?(a) The election commission should itself. make the

disposal of election disputes.(b) The election commission should hand-over the

disputes to an authority.(c) The election petitions remain pending for years while

it should be disposed within six years.(d) To fast disposal of election petitions a separate judicial

set-up should be made or special benches should beconstituted in the High Courts.

(e) None of these

132. Which of the following statement strengthens theconclusion of paragraph?(a) Political parties should patch-up the disputes amongst

them.(b) Election disputes should be disposed the parliament

or concerned house of states.(c) The supreme Court should establish a special judicial

authority for the disposal of election petitions.(d) Existing system is appropriate for the disposal of

election-petitions.(e) None of these

133. Which of the following statement weakens the conclusionof the paragraph?(a) The government should follow the high level

committees and eminent persons.(b) The government should follow the recommendation

of NCRWC.(c) The new system may be unsuccessful so, the

government should continue the existing system.(d) Fast disposal of election petitions is necessary.(e) None of these

134. In a certain code OMNIBUS is written as SUBINMO. Howis TROUBLE written in that code?(a) ELBUORT (b) ELRTBOU(c) EBOURLT (d) ELBTROU(e) None of these

135. How many such pairs of letters arethere in the wordCORPORATE each of which has as many letters betweenthem in the word as in the English alphabet?(a) None (b) One(c) Two (d) Three(e) More than three

136. If it is possible to make only one meaningful word with thefirst, second, fifth, and sixth letters of the word'EDUCATION', which would be the last letter of the word?If more than one such word can be formed, give X as theanswer. Ifno such word can be formed, give K as your answer.(a) T (b) X(c) A (d) E(e) K

137. Rohan walks a distance of 10 km towards North, then turnsto his left and walks 20 km. He again turns left and walks 10km and then, he takes a right turn and walks 5 km. How farhe is from the starting point?(a) 10km (b) 20 km(c) 30 km (d) 25 km(e) 15 km

138. If 'a' means '–', 'B' means '×', 'A' means ' 'and 'F' means '+',then-12 B 20 A 4 Q 10 F 30 = ?(a) 60 (b) 80(c) 70 (d) 90(e) None of these

Page 260: yoursmahboob.wordpress.com SBI · yoursmahboob.wordpress.com iii P 101 Speed Tests for SBI Bank Clerk Exam 101 Speed Tests for SBI Bank Clerk Exam is revised and updated edition on

yoursmahboob.w

ordpress.com

SPEED TEST 101252

139. The positions of the first and eighth digits of the number92753481 are interchanged. Similarly, the positions of secondand the seventh digits are interchanged and so on, which ofthe following will be the fifth digit to the left of the seconddigit from the right end after the rearrangement?(a) 8 (b) 2(c) 4 (d) 1(e) 3

140. What should come next in the following letter series?Z Y X W V U T Z Y X W V U Z YX W V Z Y X W(a) V (b) Y(c) Z (d) U(e) None of these

141. Most of the commonly available personal computers /laptops have a keyboard popularly known as(a) QWERTY (b) QOLTY(c) ALTER (d) UCLIF(e) None of these

142. What are the default file extension for all word documents ?(a) WRD (b) TXT(c) DOC (d) FIL(e) None of these

143. Manipulating data to create information is known as(a) feedback (b) programming(c) processing (d) analysis(e) None of these

144. A _________ is an organised colection of data about asingle entity.(a) file (b) library(c) database (d) dictionary(e) None of these

145. The horizontal and vertical lines on a worksheet are called(a) cells (b) sheets(c) blocking (d) gridlines(e) None of these

146. _________ is when the computer is turned on and theoperating system is loading.(a) Booting (b) Flashing(c) Tracking (d) Taping(e) None of these

147. To print a document, press ________ then press ENTER(a) SHIFT + P (b) CTRL + P(c) ALT + P (d) ESC + P(e) None of these

148. Which of the following are valid Min. & Max. zoom sizes inMS Office(a) 10, 100 (b) 20, 250(c) 10, 500 (d) 10, 1000(e) None of these

149. If you wish to extend the length of the network withouthaving the signal degrade, you would use a

(a) repeater (b) router(c) gateway (d) switch(e) None of these

150. Something which has easily understood instruction is saidto be(a) user friendly (b) information(c) word processing (d) icon(e) None of these

Part-IV : English Language

DIRECTIONS (Q. 151-165): Read the following passage carefullyand answer the questions given below it. Certain words/ phraseshave been printed in bold to help you locate them while answeringsome of the questions.

Long ago there was a poor Brahmin named Krishnan. Hecould not find enough work to do. Sometimes, he and his familyhad to go without food. At last Krishnan decided to leave hisvillage in search of work. Early next morning, he left the house. Hewalked the whole day until he came to a thick jungle. He was tired,thirsty and hungry. While looking around for water to drink, hefound a well. He went to the well and looked in. There he saw ajaguar, a monkey, a snake and a man. They had all fallen into thewell. “O, noble Brahmin”, the jaguar called out to him, “Pleasehelp me out, so that I can go back to my family.”

“But you are a jaguar”, said Krishnan. “How do I know youwill not kill me?” “Don’t be afraid of me, I promise I will not do youany harm”, replied the jaguar. Krishnan reached into the well andpulled out the jaguar. The jaguar thanked him and said, “I’mShersingh. I live in a cave in the mountains. I shall be most delightedif I can repay my debt to you someday.” Krishnan then heard themonkey calling out to him from the well. The Brahmin at oncepulled the monkey out. The monkey thanked the Brahmin. “If youare ever in need of food, just drop in at my place below that bigmountain. Bali is my name.” Now the snake called out to him forhelp. “Help you!” exclaimed Krishnan. “You are a snake. What ifyou bite me?” “I shall never bite you”, said the snake. So Krishnanpulled the snake out of the well. The snake said, “Remember, ifyou are ever in any difficulty, just call out my name-Naagesh, andwherever you are, I shall find you.” The jaguar, the monkey andthe snake took leave of the Brahmin. But before they left, theyspoke to him about the man in the well. “Please do not help him,”said Shersingh. “If you do”, said Naagesh, “you will be in troubleyourself.” As soon as they left, the man in the well began to callout for help. Krishnan felt sorry for the man and pulled him out ofthe well. “Thank you for your kindness”, said the man. “I am SethGhanshyamdas. I am a goldsmith. If you ever need my help, don’thesitate to visit my humble house near the city.” The goldsmiththen left for home.

After some time, the Brahmin continued his journey. But hecould not find any work. He then remembered Shersingh, Bali,Naagesh and Seth Ghanshyamdas. He thought it was time toseek their help. He first went to Bali. The monkey was overjoyedto see him. He gave him a warm welcome and offered him somereally delicious fruits. The Brahmin told him how grateful he was.

Page 261: yoursmahboob.wordpress.com SBI · yoursmahboob.wordpress.com iii P 101 Speed Tests for SBI Bank Clerk Exam 101 Speed Tests for SBI Bank Clerk Exam is revised and updated edition on

yoursmahboob.w

ordpress.com

SPEED TEST 101 253Now Krishnan went to see Shersingh, the jaguar. As soon asShersingh saw Krishnan coming, he ran out to welcome him. Hegave Krishnan a beautiful gold necklace and other preciousjewellery. Krishnan thanked Shersingh for the jewellery anddeparted. His journey had at last brought him luck, he thought. Hewould be able to sell the ornaments for a good price. But who couldhelp him to sell the ornaments? He then remembered SethGhanshyamdas. He went to him. The goldsmith was glad to seeKrishnan. “I have come to ask for your help”, said Krishnan. “Hereare some ornaments. Please give me a good price for them.” SethGhanshyamdas took the jewellery and examined it carefully. “I shallcertainly help you”, he said. “But let me show them to anothergoldsmith. Please wait here, I will be right back.” He then went outwith the ornaments. Seth at once rushed to the Palace of the King.He said, “A man brought these ornaments to me and asked me tosell them. But they are the ornaments I made for the Prince who ismissing.” “Who is this man? Where is he?”, thundered the King.This rogue must have murdered my little Prince and robbed hisjewels!” “He is a Brahmin named Krishnan, your Majesty”, repliedthe goldsmith, and he is there, in my house. The king called for hismost dreaded soldiers. “Arrest the Brahmin who is in the goldsmith’shouse and throw him into the darkest dungeons of the kingdom”,roared the King. The King’s guard stormed into the goldsmith’shouse and seized Krishnan. Krishnan was thrown into a darkdungeon to await his execution. He then remembered the words ofNaagesh, the snake. So he called out to him.

Suddenly, almost like magic, Naagesh slithered his waydown a narrow window into the dingy cell. “O, Lord!” hissedNaagesh, “how did you manage to get yourself arrested?”Krishnan cried and then told the snake what had happened. “Ihave a plan”, hissed Naagesh. “I shall creep into the Queen’sroom and bite her”, said Naagesh. “She will faint. No matter whatthey do, she will remain asleep. The poison will remain in herbody until you place your hand on her forehead”, explainedNaagesh. He then left Krishnan and went to the palace. He creptinto the Queen’s room and bit her. The Queen fainted. The sadnews that the Queen had been bitten by a snake spread all overthe Kingdom. Vaidyas came from far and near, but their medicineshad no effect. No one could revive the Queen. Finally, the Kingdeclared that anyone who could cure the Queen would behandsomely rewarded. Many people went to the palace but all ofthem failed. “I can cure the Queen”, Krishnan told the guards. Atonce they took him to the Queen. Krishnan sat beside the Queenand placed his hand on her forehead. Soon, she opened her eyesand sat up. The King was overjoyed and shed tears of happiness.He embraced Krishnan and thanked him. “Your Majesty”, saidKrishnan. “I was sent to prison for a crime I did not commit.”Krishnan told the King the whole story. The King was fumingwith rage when he heard what the goldsmith had done. He at oncehad the goldsmith arrested. The King then presented Krishnanwith a large house and a thousand pieces of gold. Krishnan sentfor his family and they all lived happily ever after.151. Why did Krishnan decide to leave his village?

(a) As he could not find much work in his own willage andhis family had to starve sometimes because of it.

(b) As his family had requested him to do so.

(c) As his village people had asked him to leave their villageand look for work somewhere else.

(d) As he wanted to search for food in a village differentfrom his own.

(e) None of the above152. Why did the jaguar, the monkey and the snake tell Krishnan

not to save the man in the well?(a) As the man in the well was a goldsmith(b) As the man in the well had cheated the snake, the

monkey and the jaguar(c) As the man in the well was a thief(d) As the snake, the monkey and the jaguar hated the man

as they had known him for a very long time(e) None of the above

153. Why was krishnan afraid to save Naagesh from the well?(a) As Naagesh had threatened him with dire

consequences.(b) As he thought Naagesh would eat him.(c) As he thought Naagesh would bite him once he was

out of the well.(d) As he thought that Naagesh would capture him as soon

as he got out of the well.(e) None of the above.

154. Why did Krishnan go to meet Seth Ghanshyamdas?(a) As he thought that Seth Ghanshyamdas could help

him in selling the ornaments gifted to him by Shersingh.(b) As he knew that Seth Ghanshyamdas had contact with

the King which could prove to be beneficial.(c) As Seth Ghanshyamdas had requested krishnan to sell

ornaments only to him(d) As Krishnan was extremely fond of Seth Ghanshyamdas(e) None of the above.

155. What did Bali do after seeing Krishnan at his house?(1) He gave Krishnan directions to Shersingh’s house.(2) He welcomed Krishnan to his house.(3) He offered tasty fruits to Krishnan.(a) Only 1 (b) Only 2(c) Only 3 (d) Only 2 and 3(e) 1 and 3

156. What plan did Naagesh have to save Krishnan from thedungeon?(a) That he would sneak Krishnan out of the dungeon

without anyone noticing(b) That he would bite the King and make him unconscious(c) That he would bite Krishnan and make everyone believe

that he was dead(d) That he would enter the Queen’s chamber and scare her(e) None of the above

157. What did Seth Ghanshyamdas tell the King about Krishnan?(a) That Krishnan had brought fake ornaments for selling(b) That krishnan was an honest Brahmin who had left his

village(c) That Krishnan had killed the Prince(d) That Krishnan had brought those ornaments for selling

which had been made for the missing Prince(e) None of the above

Page 262: yoursmahboob.wordpress.com SBI · yoursmahboob.wordpress.com iii P 101 Speed Tests for SBI Bank Clerk Exam 101 Speed Tests for SBI Bank Clerk Exam is revised and updated edition on

yoursmahboob.w

ordpress.com

SPEED TEST 101254

158. What did the King do on learning the truth about Krishnanand Seth Ghanshyamdas?(a) He put Krishnan back in the dungeon as he still held

Krishnan responsible for the Prince’s death(b) He called for Krishnan’s wife and family(c) He presented gold to Krishnan and also a house to live in(d) He congratulated the snake on his efforts to save

Krishnan(e) None of the above

159. What did the King do to save the Queen after even theVaidyas failed to revive her?(a) He punished the snake for having harmed the Queen(b) He announced a reward to anyone who could cure the

Queen(c) He immediately called for Krishnan to cure the Queen(d) He asked his guards to immediately look for someone

who could cure the Queen(e) None of the above

160. What can possibly be the moral of the story?(a) Trust oneself before trusting overs(b) A good deed never goes in vain(c) You cannot change people but you can change yourself(d) Try and try until you succed(e) One must be the change one wishes to see in this world

DIRECTIONS (Q. 161-163): Choose the word/group of wordswhich is most similar in meaning to the word/group of wordsprinted in bold as used in the passage.

161. GO(a) report (b) live(c) send (d) leave(e) depart

162. HUMBLE(a) elegant (b) polite(c) modest (d) real(e) vast

163. SEEK(a) hunt for (b) watch for(c) mention (d) ask for(e) force

DIRECTIONS (Q. 164-165): Choose the word/group of wordswhich is most opposite in meaning to the word/group of wordsprinted in bold as used in the passage.164. CONTINUED

(a) remanded (b) presented(c) rested (d) carried on(e) stopped

165. HANDSOMELY(a) Meagrely (b) tidily(c) ugly (d) raggedly(e) plenty

DIRECTIONS (Q. 166-170): Which of the phrases (a), (b), (c)and (d) given below each sentence should replace the phraseprinted in bold in the sentence to make it grammatically correct?If the sentence is correct as it is given and no correction isrequired, mark (e) as the answer.166. A young and successful executive was travelling down a

neighbourhood street, going a bit to fast in his new car.(a) a bit too fastly (b) a bit as fast(c) a bit to fastly (d) a bit too fast(e) No correction required

167. All she could think about was the beautiful dress and howshe could earn enough money to buy it.(a) All she can think (b) All she could thought(c) All she can thought (d) All she can thinking(e) No correction required

168. He told his employer of his plans to leave the business tolead a more leisure life.(a) more leisurely life (b) many leisurely life(c) many leisured life (d) more leisurely live(e) No correction required

169. Padma could convince anyone with her talk as she had thegift of the gabbing.(a) gifting of the gabbing (b) gift of the gab(c) gifting of the gab (d) gift of the gab(e) No correction required

170. For countries undergoing a recession, large cuts in publicspending seem to be the ordering of the day.(a) be the ordering of days(b) being the order of the day(c) be the order of the day(d) being the ordering of days(e) No correction required

DIRECTIONS (Q. 171-175): In each question below, a sentencewith four words printed in bold type is given. These are numberedas (a), (b), (c) and (d). One of these four words printed in bold maybe either wrongly spelt or inappropriate in the context of thesentence. Find out the word which is wrongly spelt orinappropriate, if any. The number of that word is your answer. Ifall the words printed in bold are correctly spelt and alsoappropriate in the context of the sentence, mark (e) ie. ‘Allcorrect’ as your answer.171. The whole (a)/ time she walked with her child in her arms,

the only thing (b)/ that worried (c)/ her was her son’sfeature. (d)/ All correct (e)

172. When the young artist returned (a)/ to his village, his familyheld a festive (b)/ dinner on its lawn to celebrate histriumpant (c)/ homecoming. (d)/ All correct (e)

173. Had she not suppressed (a)/ all the details of her Company’sproject (b)/ her Company would have bagged (c)/ thecontract. (d)/ All correct (e)

Page 263: yoursmahboob.wordpress.com SBI · yoursmahboob.wordpress.com iii P 101 Speed Tests for SBI Bank Clerk Exam 101 Speed Tests for SBI Bank Clerk Exam is revised and updated edition on

yoursmahboob.w

ordpress.com

SPEED TEST 101 255174. She trusted Mira with all her heart (a)/ and thus handled

(b)/ over her life’s (c)/ savings to her instantly. (d)/ Allcorrect (e).

175. It is difficullt (a)/ to see the picture (b)/ when you are inside(c)/ the frame. (d)/ All correct (e)

DIRECTIONS (Q. 176-190): In the following passage there areblanks, each of which has been numbered. these numbers areprinted below the passage and against each, five words aresuggested, one of which fits the blank appropriately. Find out theappropriate word in each case.

One day a father of a very wealthy family (176) his son on atrip to the country with the purpose of (177) his son how the poorpeople live so he could be thankful for his wealth. They spent a(178) of days and nights on the farm of what would be considereda (179) poor family. On their (180) from the trip, the father askedhis son, “How was the trip?" "It was great, Dad.” “Did you seehow poor people can be?”, the father asked. “Oh yeah”, said theson. So what did you (181) from the trip?”, asked the father. Theson answered, “I saw that we have one dog and they had four.We have a pool that (182) in the middle of our garden and theyhave a creek that has no end.” “We have imported lanterns in ourgarden and they have the stars at night.” “Our patio reaches tothe front yard and they have the (183) horizon.” “we have a smallpiece of land to live on and they have fields that go beyound oursight.” “We have (184) who serve us, but they serve others.”“We buy our food, but they grow theirs.” “We have walls aroundour (185) to protect us; they have friends to protect them.”

With this the boy’s father was speechless. Then his sonadded, “Thanks dad for showing me how poor we are”.176. (a) took (b) beat

(c) drag (d) mould(e) showed

177. (a) presenting (b) requesting(c) tell (d) trusting(e) showing

178. (a) two (b) couple(c) much (d) few(e) many

179. (a) major (b) some(c) sorrow (d) very(e) astutely

180. (a) lane (b) journey(c) leave (d) return(e) walking

181. (a) reveal (b) think(c) saw (d) believe(e) learn

182. (a) stands (b) reaches(c) swims (d) leak(e) watery

183. (a) more (b) scene(c) whole (d) last(e) lucky

184. (a) servants (b) mother(c) computers (d) relatives(e) man

185. (a) minds (b) selves(c) property (d) pillars(e) country

DIRECTIONS (Qs. 186-190): In each of the following sentences,an idiomatic expression or a proverb is highlighted. Select thealternative which best describes its use in the sentence.

186. He resigned the post of his own accord.(a) which he liked(b) according to his convenience(c) voluntarily and willingly(d) according to his judgement(e) None of these

187. As a politician he is used to being in the limelight all thetime.(a) giving speeches(b) the object of admiration(c) the centre of attraction(d) an object of public notice(e) None of these

188. I ran out of money on my European tour.(a) exhausted my stock of(b) did not have enough(c) lost(d) carried a lot(e) None of these

189. Madhuri might scream blue murder, but I feel Deepalishould get the promotion since she is better qualified forthe job.(a) someone has been murdered with some blue liquid(b) someone is being murdered and has become blue(c) suffer from persecution complex(d) make a great deal of noise and object vehemently(e) None of these

190. In modern democratic societies lynch law seems to havebecome the spheres of life.(a) law of the mob(b) law of the underworld(c) law of the constitution(d) law of the parliament(e) None of these

Page 264: yoursmahboob.wordpress.com SBI · yoursmahboob.wordpress.com iii P 101 Speed Tests for SBI Bank Clerk Exam 101 Speed Tests for SBI Bank Clerk Exam is revised and updated edition on

yoursmahboob.w

ordpress.com

SPEED TEST 101256

Test Code : ............................... Time taken : ......................... Date : ..........................

RESPONSE SHEET

(a) (b) (c) (d) (e)1.2.3.4.5.6.7.8.9.10.11.12.13.14.15.16.17.18.19.20.21.22.23.24.25.26.27.28.29.30.31.32.33.34.35.36.37.38.39.40.41.42.43.44.45.46.47.48.49.50.

(a) (b) (c) (d) (e)51.52.53.54.55.56.57.58.59.60.61.62.63.64.65.66.67.68.69.70.71.72.73.74.75.76.77.78.79.80.81.82.83.84.85.86.87.88.89.90.91.92.93.94.95.96.97.98.99.100.

(a) (b) (c) (d) (e)101.102.103.104.105.106.107.108.109.110.111.112.113.114.115.116.117.118.119.120.121.122.123.124.125.126.127.128.129.130.131.132.133.134.135.136.137.138.139.140.141.142.143.144.145.146.147.148.149.150.

(a) (b) (c) (d) (e)151.152.153.154.155.156.157.158.159.160.161.162.163.164.165.166.167.168.169.170.171.172.173.174.175.176.177.178.179.180.181.182.183.184.185.186.187.188.189.190.191.192.193.194.195.196.197.198.199.200.

Page 265: yoursmahboob.wordpress.com SBI · yoursmahboob.wordpress.com iii P 101 Speed Tests for SBI Bank Clerk Exam 101 Speed Tests for SBI Bank Clerk Exam is revised and updated edition on

yoursmahboob.w

ordpress.com

Page 266: yoursmahboob.wordpress.com SBI · yoursmahboob.wordpress.com iii P 101 Speed Tests for SBI Bank Clerk Exam 101 Speed Tests for SBI Bank Clerk Exam is revised and updated edition on

yoursmahboob.w

ordpress.com

Page 267: yoursmahboob.wordpress.com SBI · yoursmahboob.wordpress.com iii P 101 Speed Tests for SBI Bank Clerk Exam 101 Speed Tests for SBI Bank Clerk Exam is revised and updated edition on

yoursmahboob.w

ordpress.com

1SOLUTIONS

SPEED TEST 11. (a) A square is a two-dimensional figure consisting of sides

whereas a cube is a three- dimensional figure. Similarly,circle is a two-dimensional figure and a sphere is athree-dimensional figure.

2. (e)

B R I

J S C

+1

G H T

S G F

–1

Similarly,

J O I

J P K

+ 1

N E D

C D M

–1

3. (a) The second number is the product of the digits of thefirst.

4. (d) The first is found in the form of the second.5. (c) In first term, two letters are missing between first two

letters while last two are continous but in second termfirst two letters are continuous and two letters aremissing between last two letters.

6. (a) The third letter of second term is the next letteraccording to alphabet to the third letter of first term.

7. (a) The letters of each group are in reverse order.8. (d) First two letters of the first term are in reverse order in

the second term and so are the next two letters.9. (b) Fifth and third letters of the first term are first and

second letters of the second term and first two lettersof the first term are third and fourth letters of the secondterm.

10. (a) There is a gap of one letter between third and fourth,fourth and first, and first and second letters of eachgroup.

11. (a) The letters in first and second terms are in reverse orderof alphabet.

12. (e) There is a gap of one letter between each correspondingletters of 'QYGO' and 'SAIQ'

13. (d) There is a gap of three letters between eachcorresponding letters of 'YAWC' and 'UESG'.

14. (c) There is a gap of two letters between the twoconsecutive letters of each term.

15. (e) First, Second, and third each term is one more than thesquare of prime number. Hence the fourth term= (19)2 + 1

16. (a) First term = (6)2 + 6.Second term = (7)2 + 7Third term = (10)2 + 10\ Fourth term = (11)2 + 11

17. (d) First term = 32 + 3.Second term = 42 + 4Third term = 52 + 5\ Fourth term = 62 + 6

18. (c) First term = 22 – 1Second term = 32 + 1Third term = 32 – 1\ Fourth term = 42 + 1

19. (c) First term = 32 – 1.Second term = 33 + 1Third term = 42 – 1\ Fourth term = 43 + 1

20. (b) First term = 23.Second term = 32

Third term = 43

\ Fourth term = 52

21. (d) Second term = 4 × First term\ Fourth term = 4 × Third term

22. (a) Second term = (First term)2 – 1\ Fourth term = (Third term)2 – 1

23. (e) Second term = (First term)3

\ Fourth term = (Third term)3.24. (a) Second term = First term + 1/8 First term.

\ Fourth term = Third term + 1/8 Third term.25. (e)26. (c) 1 and 4 are what happens after a disease. 5 is its

symptom but not a definite one. 2 is a probable cause.27. (c) The first is the force fighting on/in the second.28. (e) More of a test of your English.29. (c) Secretly is the opposite of openly, and silently is the

opposite of noisily. Choices a and b are clearly not theopposites of silently. (Choice d) means the same thingas silently.

30. (b) A spring forms or has the shape of a coil, and a ringforms a loop.

SPEED TEST 2

1. (a) 2 3H J G+ -¾¾® ¾¾®1 3P Q N+ -¾¾® ¾¾®1 3D E B+ -¾¾® ¾¾®1 3T U R+ -¾¾® ¾¾®1 3K L I+ -¾¾® ¾¾®

2. (e) Lotus is grown in water (Mud).3. (b) P R O B L E M

2 9 4 8 3 7 5¯ ¯ ¯ ¯ ¯ ¯ ¯

B O R E

8 4 9 7¯ ¯ ¯ ¯ ;

( )

M O E P

5 9 4 7 2¯ ¯ ¯ ¯

L B O R

3 8 4 9¯ ¯ ¯ ¯ ;

O M E P

4 5 7 2¯ ¯ ¯ ¯

E R O L

7 9 4 3¯ ¯ ¯ ¯

4. (e)5. (d) River is a water body.6. (d) Except 255 all other numbers are one more than perfect

square.50 = (7)2 + 1, 65 = (8)2 + 1;170 = (13)2 + 1, 290 = (27)2 + 1But, 255 = (16)2 – 1

Page 268: yoursmahboob.wordpress.com SBI · yoursmahboob.wordpress.com iii P 101 Speed Tests for SBI Bank Clerk Exam 101 Speed Tests for SBI Bank Clerk Exam is revised and updated edition on

yoursmahboob.w

ordpress.com

2 101 SPEED TEST7. (e) The number 49 is a perfect square of a natural

number.8. (e) Except Brinjal, all others grow underground.9. (d) All others are parts of a car.10. (c) Except 529, all others are perfect squares of even

numbers. The number 529 is a perfect square of an oddnumber.196 = 14 × 14; 256 = 16 × 16529 = 23 × 23; 576 = 24 × 24324 = 18 × 18

11. (a) 2 2R P N- -¾¾® ¾¾®

4 2W S U- +¾¾® ¾¾®

4 2H D F- +¾¾® ¾¾®

4 2L H J- +¾¾® ¾¾®

4 2Q M O- +¾¾® ¾¾®12. (b) The number 441 is a multiple of 3

13. (e) 3 3P M; E B;- -¾¾® ¾¾®

3 3T Q; I F;- -¾¾® ¾¾®

3V Y+¾¾®14. (d) 115 = 5 × 23 ;

85 = 5 × 17;95 = 5 × 19;155 = 5 × 31;But, 75 = 5 × 15One factor of 75 is not a Prime Number.

15. (e) Except number 345, all other numbers are product of 23and a Prime Number.115 = 23 × 5;161 = 23 × 7253 = 23 × 11;391 = 23 × 17But 345 = 23 × 15.The number 15 is not a Prime Number.

16. (c) 2 4O M Q- +¾¾® ¾¾®

2 4H F J- +¾¾® ¾¾®

4 2T P R- +¾¾® ¾¾®

2 4T R V- +¾¾® ¾¾®

2 4V T X- +¾¾® ¾¾®

17. (c) In all others, 1st letter – 1 = 2nd letter, and 2nd letter– 2 = 3rd letter.

18. (a) Except Diabetes, all others are infectious diseases.19. (a) Except Mustard, all others are grains. Mustard is an

oilseed.20 (d) All the numbers are multiples of 5. But 25 is a perfect

square.21. (d) All others are synonyms22. (c) All others are synonyms

23. (d) In each pair if first letter is mth from the beginning ofalphabet the second letter is mth from end.

24. (e) In all others,1st letter + 1 = 4th letter.4th letter + 1 = 2nd letterand 2nd letter +2 = 3rd letter

25. (c) In all others4th letter + 1 = 1st letter1st letter + 2 = 2nd letter2nd letter + 1 = 3rd letter.

26. (b) All others are synonyms.27. (d) All others are synonyms.28. (a) All others imply ‘UP’.29. (c) ‘Large’ is an adjective whereas others are noun.30. (d) All others are negative.

SPEED TEST 3

1. (a) 1050 420 168 67.2 26.88 10.752

¸ 2.5 ¸ 2.5 ¸ 2.5 ¸ 2.5¸ 2.5

2. (e)

+6 +18 +36 +60 +90 +126

0 6 24 60 120 210 ?

+12 +18 +24 +30 +36\ ? = 210 + 126 = 336

3. (a) The pattern of the series is :19 – 15 = 4 = 22

83 – 19 = 64 = 43

119 – 83 = 36 = 62

631 – 119 = 512 = 83

\ ? = 631 + 102 = 631 + 100 = 7314. (c) The pattern of the series is :

19 + 1 × 7 = 19 + 7 = 2626 + 2 × 7 = 26 + 14 = 4040 + 4 × 7 = 40 + 28 = 6868 + 8 × 7 = 68 + 56 = 124124 + 16 × 7 = 124 + 112 = 236

5. (d) The pattern of the number series is as given below:11 × 1 – 1 = 1010 × 2 – 2 = 1818 × 3 – 3 = 5151 × 4 – 4 = 200200 × 5 – 5 = 995\ ? = d

6. (b) The pattern of the number series is as given below:14 + 10 = 2424 + 19 (= 10 + 9) = 4343 + 28 (= 19 + 9) = 7171 + 37 (= 28 + 9) = 108108 + 46 (= 37 + 9) = 154

7. (e) The pattern of the number series is as given below:144 + 29 = 173173 – 33 = 140140 + 29 = 169169 – 33 = 136136 + 29 = 165

Page 269: yoursmahboob.wordpress.com SBI · yoursmahboob.wordpress.com iii P 101 Speed Tests for SBI Bank Clerk Exam 101 Speed Tests for SBI Bank Clerk Exam is revised and updated edition on

yoursmahboob.w

ordpress.com

3SOLUTIONS8. (a) The pattern of the number series is as given below:

656 24 328 24 3522

+ = + =

352 24 176 24 2002

+ = + =

200 24 100 24 1242

+ = + =

124 24 62 24 862

+ = + =

86 24 43 24 672

+ = + =

9. (b) The pattern of the number series is as given below:12 × 4 – 30 = 48 – 30 = 1818 × 4 – 36 = 72 – 36 = 3636 × 4 – 42 = 144 – 42 = 102102 × 4 – 48 = 408 – 48 = 360360 × 4 – 54 = 1440 – 54 = 1386

10. (c) 71 78 99 134 183 246

+(7 × 1) +(7 × 3) +(7 × 5) +(7 × 7) +(7 × 9)

11. (b) 342 337.5 328.5 315 297 274.5

–(4.5× 1) –(4.5 × 2) –(4.5 × 3) –(4.5 × 4) –(4.5 × 5)

12. (c) 161 164 179 242 497 1520

+3 +15 +63 +255 +1023

(3 × 4)+3 (15 × 4)+3 (63 × 4)+3 (255 × 4)+313. (b) 3 + 32, 5 + 52, 7 + 72.14. (a) 92 + 10, (19)2 + 20, (29)2 + 3015. (e) 100 × 1.1, 200 × 2.2, 300 × 3.3, 400 × 4.416. (b) + 12 – 02, + 32 – 22, + 52 – 42, + 72 – 62.17. (e) × 3 + 1, × 3 + 3, × 3 + 5, × 3 + 7 ........18. (e) The given series is ¸ 2 – 6

Reqd no. = 50 2 – 6 = 25 – 6 = 1919. (d) (1015 + 1) 2 = 508;

(508 + 2) 2 = 255; (255 + 3) 2 = 129;(129 + 4) 2 = 66.5; (66.5 + 5) 2 = 35.75;(35.75 + 6) 2 = 20.875

20. (a) The series is× 4, 8, × 12, ¸ 16, × 20

21. (d) The series is–1.1, –2.2, –4.4, –8.8, –17.6

22. (e) The series is+112, + 122, + 132, + 142, + 152.

23. (d) The series is abcab, bcabc, cabca.24. (c) First two letters of each term are in reverse order.

Similarly third and fourth letters are also in reverse order.Besides this, second letter of the second term is thenext letter after the first letter of the first term.Second Method -

–1

D C X W, F E V U, H G T S, J I R Q

+1

–1 –1 –1 –1 –1 –1–1

–1... ... ... ...

+1+1

–1

25. (e) The letters are in reverse order while one letter ismissing between two consecutive letters.

26. (b) There is a gap of one letter between two consecutiveletters. Besides, this the letters are capital and lowerrespectively.

27. (b) m m l lm m l lm m l l

28. (e) 1, 12, 123, 1234, 12345, 123456, 123456 7

29. (c) ABCD, ABCDE, ABCDEF, PQRS, PQRST, PQRST U30. (e) N + 3 = Q, Q + 3 = Z, Z + 3 = S

D – 2 = W, W – 2 = F, F – 2 = VP + 3 = B, B + 3 = R, R + 3 = I Hence, ? = SVI

SPEED TEST 4

1. (e) E X C U R S I O N

2. (c) G O L D E N

3. (a) S T R I V E4. (b) Meaningful words are % TAN and ANT

5. (a)C

C

I

I

N

N

T

T

LE

EL

6. (c) ELAN, LEAN and LANE7. (b) Meaningful words are % ARE, ART, ATE8. (a) Given word : A R G U M E N T

Alphabetically the sequence isA E G M N R T U

+1 +1 +1 +1 +1 +1 +1 +1

B F H N O S U V9. (d) New order of letters : GANCARROE10. (c) According to english alphabet, resultant group will be

as follows:

P R I N C E

C E I N P ROnly two letters 'I and N' will remain unchanged.

11. (e) Meaningful words are : ROSE, SORE, EROS and ORES.

12. (d)3 15 14 19 20 1 2 12 5C O N S T A B L E

13. (c) P H Y S I C A L

14. (d) S H I F T E D

15. (b) Meaningful words are : GO and TO16. (a) Meaningful words are : TON, TOE, TEN17. (b) According to order of alphabet

C E F I L M R U

B D E H K L Q T18. (d) Meaningful word are : TIRE, TIER and RITE.

Page 270: yoursmahboob.wordpress.com SBI · yoursmahboob.wordpress.com iii P 101 Speed Tests for SBI Bank Clerk Exam 101 Speed Tests for SBI Bank Clerk Exam is revised and updated edition on

yoursmahboob.w

ordpress.com

4 101 SPEED TEST

19. (a) D I S T A N C E

C J R S B M B FB B C F J M R S

20. (d) FOLK ® FKLO ® EJKN21. (c) MULE = 1 + 3 + 3 + 2 = 922. (e) Meaningful words are : TEARS, STARE, RATES and

ASTER.23. (b) Word : WEBPAGE

Changed word :VFAOBFFSo, F appears thrice.

24. (c) Words : TIPS, SPIT and PITS.25. (b) SKILL, KILLS26. (c) Original word :

HABITUALChanged word :GBAJSVBKSo, fourth from the left is J.

27. (c) Word :W A L K I N GAlphabetical order :A G I K LN WSo, the positions of K and N remain unchanged.

28. (d) Clearly, we have :COMPREHENSION ® (COM) (PREHENS) (ION)

® COMIONSNEHERPThe middle letter is the seventh letter, which is S.

29. (d)

D E P R E S S I O NE D R P S E I S N O1 2 3 4 5 6 7 8 9 10

30. (a) Random, Restaurant, Restrict, Robber, Rocket.

SPEED TEST 5

1. (c) Second and fourth letters of the word PROSE are movedtwo steps backwards in alphabatical order. Similarly,LIGHT can be coded as LGGFT.

2. (d) Z = 52 = 26 × 2ACT = 1 × 2 + 3 × 2 + 20 × 2 = 48 [Alphabetical position

numbers has been doubled]

Þ BAT = 2 × 2 + 1 × 2 + 20 × 2 = 463. (c) Here, 2 ® A, 3 ® L, 5 ® G, 4, 4 ® U and 9 ® T..

Hence, 23549 will be having the code ALGUT.4. (a) RBM STD BRO PUS º the cat is beautiful ....(i)

TNH PUS DIM STD º the dog is brown ....(ii)PUS DIM BRO PUS CUS º the dog has the cat ....(iii)(i) and (ii) Þ STD PUS º is(ii) and (iii) Þ PUS DIM º the dog(i) and (iii) Þ PUS BRO º the cat\ From (iii), CUS º has

5. (b) As, Similarly,

Þ

ïïïïï

þ

ïïïïï

ý

ü

¾¾¬¾¾¬¾¾¬¾¾¬¾¾¬¾¾¬¾¾¬¾¾¬

-

-

-

-

-

-

-

-

CERTGIJLYAVXYALN

2

2

2

2

2

2

2

2

ECTRAYRPGEECTRNLIG

2

2

2

2

2

2

2

2

2

¾®¾¾®¾¾®¾¾®¾¾®¾¾®¾¾®¾¾®¾¾®¾

+

+

+

+

+

+

+

+

+

6. (a) Going through information provided, we get codes forG ® 3, R® 8, E ® 1, C ® 9.Therefore, Greece will be coded as 381191.

7. (b) CAT ® SATC & DEAR ® SEARDClearly, the first letter is transferred to last place and Staken its place. In the same way SINGS would be codedas SINGS.

8. (a) P R O M I S E1 2 3 4 5 6 7Þ MISER will be coded as 45672.

9. (d) In the code, night is called sunshine. As we sleep innight, the correct answer is sunshine.

10. (a) Position of D alphabetically = 4Position of F alphabetically = 6Thus D + F = 10 = position of J in alphabet.

11. (d) ‘ 2 4 7’ ® ‘spread red carpet ’

‘ 2 3 6’ ® ‘dust one carpet ’

‘ 2 4 3 ’ ® ‘ one red carpet ’Hence, ‘2’ ® ‘carpet’; ‘4’ ® ‘red’; ‘7’ ® ‘spread’;‘3’ ® ‘one’; ‘6’ ® ‘dust’.

12. (b) A person sits on a chair. Since ‘chair’ is called ‘cot’, ouranswer is ‘cot’.

13. (a)O V E R V I S T$ # % * # + –´

From above table, SORE is coded as : S O R E

$ * %´

14. (a) As,P U L S E

O T K R D-1 -1 -1 -1 -1

N E W

M D V-1 -1 -1

D R K T O V D M

and

reverse order reverse order

Similarly,

O Q N A D R

P R O B E S-1 -1 -1 -1 -1

R D A N Q O

-1

reverse order

Hence, required code : RDANQO

Page 271: yoursmahboob.wordpress.com SBI · yoursmahboob.wordpress.com iii P 101 Speed Tests for SBI Bank Clerk Exam 101 Speed Tests for SBI Bank Clerk Exam is revised and updated edition on

yoursmahboob.w

ordpress.com

5SOLUTIONS15. (b) As,

R E M I T

£ 3 7and

C O N S U L

= % 8 $ 5b

Similarly,

O C E L O T

% = £ % 7516. (c)

1 2 3 4 5 4 1 3 5 2A M O N G ® N A O G M1 2 3 4 5 4 1 3 5 2S P I N E ® N S I E P

Hence,

1 2 3 4 5 4 1 3 5 2L A M O N ® O L M N A

17. (a) S E A L

$ 7 5 @¯ ¯ ¯ ¯

and D O S E

# 8 $ 7¯ ¯ ¯ ¯

Hence,

S O L D

$ 8 @ #¯ ¯ ¯ ¯

18. (a) Teacher write on blackboard with chalk, here chalk iscalled book, hence here the code of chalk is book.

19. (e) As,

M O

O M

T H

H U

E R

R F

+1 +1

Similarly,

A N

N A

S W

W T

E R

R F

+1 +1

20. (a)

G R O W

= @ % # and

W I T H I N

# ÷ + © ÷ rW I N G

# ÷ =r

21. (a) As,2

2

D FO Q

+

+

¾¾®

¾¾® and 2

2

I KN P

+

+

¾¾®

¾¾®

Similarly,2

2

A CT V

+

+

¾¾®

¾¾®22. (b) W R O M B T ® 7 1 9 4 8 323. (b) As,

1

1

1

1

P QI HN OK J

+

-

+

-

¾¾®

¾¾®

¾¾®

¾¾®

and

1

1

1

1

B CO NL MT S

+

-

+

-

¾¾®

¾¾®

¾¾®

¾¾®

Similarly,

1

1

1

1

M NU TS TT S

+

-

+

-

¾¾®

¾¾®

¾¾®

¾¾®

24. (b) As,

L O C K

M P B J

-1-1+1+1

and

B L O W

C M N V

-1-1+1+1

Similarly,

W I N E

X J M D

-1-1+1+1

25. (b) S O L D I E R J F S C R N K+1+1+1–1–1–1–1

Similarly,

G E N I O U S P V T H F D M+1+1+1–1–1–1–1

26. (e) The colour of blood is red and here red means orange.

Page 272: yoursmahboob.wordpress.com SBI · yoursmahboob.wordpress.com iii P 101 Speed Tests for SBI Bank Clerk Exam 101 Speed Tests for SBI Bank Clerk Exam is revised and updated edition on

yoursmahboob.w

ordpress.com

6 101 SPEED TEST

(27-28) : colours of the sky

Þ ki la fa so

rainbow colours Þ ro ki

sky high rocket Þ la pe jo

the rocket world Þ pe so ne

27. (d) colours sky high Þ ki la jo28. (c) The code of ‘the’ is ‘so’.

29. (e)D R E A M I N G

B F S E F M H L+1 –1

Similarly,T R E A T I S E

B F S U D R H S+1 –1

30. (b) A person sits on a chair. Since ‘chair’ is called ‘cot’, ouranswer is ‘cot’.

SPEED TEST 61. (c) Clearly, Arun is uncle of the lady in the photograph.

Arun — Brother|

Son Wife mother lady|

daugher

+ =

Mother — B|

A C-2. (a) A’s mother = B’s sister

Clearly, A can be a neice of B.3. (b) The only son of Mahesh’s father is Mahesh himself.

Father of Kamla is Mahesh and Mahesh is father ofKamla.

4. (d) Clearly, the grandson of Anil’s mother is son of Aniland wife of Anil’s son is daughter in-law of Anil. Thus,Anil is the father-in-law of the girl.

5. (b) Clearly, from the relationship diagram. Y is the brother-in-law of B.

Married coupleA B

X Y

Brother-in-lawBrother

Brothers6. (c) Woman’s Mother’s husband

Woman’s fatherWoman’s father’s sister ¾¾® Woman’s Aunt.Since, woman’s aunt is man’s aunt\ woman is sister of man.

7. (c) The only son of Vineet’s grandfather is Vineet’sfather. The lady is the daughter of Vineet’s father.Thus, the lady in the park is the sister of Vineet.

8. (a) As X is the son of Y’s father and Y is the sister of X andthus, he has to be the brother of Y.

9. (b) Father

Monika

Mother

Rahul

Only Daughter

10. (a)

Chinky

Daizy Binny Aruna(Binny’s sister

and Chinky’s Mother)(Binny’s sister)

11. (d)Saroj

VaniSister-in-law

Deepakonly brother

Ramesh

Rajesh

father

wife

mother

12. (c)

13. (c)

Husband Sister Man Brother Wife Son

Son

Brother

Mother/Photograph

Mother-in-law

Nephew

For Qs. 14 to 16(Couples)

Roma Mohan

Smita Devika Aman

Romila

(Couples)

(Couples)Madhu Jeevan

Krishna(Couples)

Sunil

Anuj AnkurDaughter

There are in all four married couples.

Page 273: yoursmahboob.wordpress.com SBI · yoursmahboob.wordpress.com iii P 101 Speed Tests for SBI Bank Clerk Exam 101 Speed Tests for SBI Bank Clerk Exam is revised and updated edition on

yoursmahboob.w

ordpress.com

7SOLUTIONS14. (a) Since, Devika is wife of Aman and Krishna is sister

of Aman, therefore, Krishna is the sister-in-law ofDevika.

15. (d) Since, Anuj is son of Krishna and Romila is daughterof Aman and Aman is brother of Krishna, thereforeAnuj and Romila are cousins.

16. (c) Since, Madhu is mother of Krishna who is wife ofSunil. Therefore, Madhu is Sunil’s mother-in-law.

For Qs. 17-20.The information given in the question can besummarised as:

(Mother)D

B

E(Father)

Son/daughter

AF

CWife

Wife

(Son)

Daughter

17. (c) C is the mother to A and F.18. (d) Since we do not know whether A is male or female,

therefore (d).19. (d) We cannot tell from the given facts whether A is a male

or female.20. (b) The two couples in the question are DE (grandmother

and grandfather) and BC (father and mother).21. (e)

R(+)

S

V(–)T(–)Hence T is the daughter of S.

22. (b) V(–)

T(–) R(+) S23. (d)

V(+)

T(+) R(–)

SS is either the nephew or niece of T.

24. (c)

V(+)

T(+) R(–)

S(1)

T(–) V(+)(–) S

R(+)(2)

S is nephew or niece of T. Absurd relationship.

T(–) S(+)

R(+) V(+)(3)

S is husband of T.We need not go further.

25. (a) R(+) SV(–)

T(+)Hence V is the aunt of T.

26. (c) T(–)K(+)

D(+) M(+)B

For Qs. 27 to 30

Adhir Mishra

Urmila Raghu SumitMarried

Roma(eldest)

Mr. & Mrs. Mohan

Roshan Bimla Shilla(youngest)

Shilla

Mr. & Mrs Sharma

Sandeep Shaifali

Leela (daughter)

ShivendarSohan

Sons

27. (b) Sumit’s mother-in-law = Mrs. Mohan.28. (c) As Sohan is son of Sumit and Sumit is son of Mishra.

The surname of Sohan is Mishra.29. (a) Leela is the grand daughter of Mr. Sharma. Hence, the

surname of Leela is Sharma.30. (d) Shivendar is son of Roma. Therefore, he is grandson

of Roma’s father.

SPEED TEST 71. (c) Clearly, I am to the north of my house.

10

5

5

5

Starting point

End Point

(My house)

W

N

E

S

2. (b)100 m

20 10 10 30 50 m My friend

BB'

My self

A A'

70 m

When my friend reaches on the previous track(i.e. on B') again, he had travelled a distance of(30 + 10 + 20 + 10) = 70 m. As I walk with the same speedas that of my friend I have walked 70m, but on thestraight track. Now, he is just [100 – (30 + 20)] = 50mfrom my starting point.Hence, the distance between us = (70 – 50) = 20m

Page 274: yoursmahboob.wordpress.com SBI · yoursmahboob.wordpress.com iii P 101 Speed Tests for SBI Bank Clerk Exam 101 Speed Tests for SBI Bank Clerk Exam is revised and updated edition on

yoursmahboob.w

ordpress.com

8 101 SPEED TEST3. (a) Clearly the direction of the hour’s hand is North-east.

1 2 3 4

5 6 12

H

M

W

N

E

S

4. (b) 33

3

1

1

W

N

E

S

Required distance = 3 + 1 = 4 km5. (a) Here, O is the starting point.

4

3

3 4 A O B

Both A and B are 2 23 4+ = 5 km from the startingpoint.

6. (b) Clearly the school is in north-east

40

5

25

School

Ram’s house W

N

E

S

7. (c) West

River is finally flowing in east direction.For (Qs. 8-9)

30m

60m

Dinesh

Kunal

Prashant

40m25mNitin Atul

8. (d) Left of Kunal is Atul.North East of Atul is Prashant (obvious from the abovediagrams).

9. (c) Total distance travelled = 25 + 40 + 60 + 90 = 215 mts.

10. (d)

AB

C D

NorthWest

South East

90 m

Rahul

Rohan8 km/h

10 km/h

Rohan takes the route BADCB at 8 km/hr 209

= m/sec.

Rahul takes the route CDABC at 10 km/hr 259

= m/sec.

AB = BC = CD = DA = 90 m.First time they meet when Rohan travels 120 m andRahul travels 150 m.For the second time, Rohan is 10 m towards B from C orRahul is 80 m towards C from B. Hence on BC at adistance of 10 m from C.

11. (a)

N

E

S

WReeta Kavita

Kavita’s shadow is right to Reeta i.e., Kavita’s shadowis left to Kavita.\ Kavita is facing North.

12. (a) 45°+180° = 225° clockwise direction270° anticlockwise direction.225 – 270° = – 45°

1st

Movement

2ndMovement

Final Movement

45°

45°

180°

Initial Position

i.e., 45° anticlockwise from initial position.Hence, the required direction is south-west.

Page 275: yoursmahboob.wordpress.com SBI · yoursmahboob.wordpress.com iii P 101 Speed Tests for SBI Bank Clerk Exam 101 Speed Tests for SBI Bank Clerk Exam is revised and updated edition on

yoursmahboob.w

ordpress.com

9SOLUTIONS

13. (d) 50 m20 m

100 m

10m30 mBhavika Sunaina50 m

P

70 m

Q

When Bhavika is at P, Sunaina who is walking at thesame rate will be at Q which is at a distance of 70 m fromSunaina’s point of start.

\ Required distance between them is PQPQ = (70 + 50) – 100 = 20 m

14. (c) A F1 km1 km

5 kmB

E

P (Starting point)

3 km

2 kmC D

W E

N

S

Find the distance PB.By Pythagorus theorem, we have PB2 = PF2 + FB2

PB2 = (4)2 + (3)2 = 25Þ PB = 5 km

15. (c)

12 m

15 m

20 m

15 m

M X

N

Z

Y

XZ = XY + YZ = MN + YZ = 20 m + 12 m = 32 m16. (d) 15 km

12 km

15 km AB

C D N

E

S

W

Obviously, CB = AD = 12 km and B is south of C.

17. (b)

7 km

5 km

A

B C

D

7 km

8 km

E

F

3 km

BF = AB – (AE + EF)= AB – {(AB – CD) + EF}= 8 – {(8 – 5) + 3} = 8 – {3 + 3}= 8 – 6 = 2 km

18. (b) P O

10 k

m

10 k

m

6 km

N

E

S

W

19. (a)10 m

O

15 mN

E

S

W

20. (a) PQ = 10 + 6 = 16 km21. (d)

22. (a)

P

Q

R S5 km

5 km

5 km

5 km

N

E

S

W

Obviously, Q lies North of S.

23. (d) D M

R K

N

E

S

W

24. (b)

20 m

15 m

15 m

P

O

30 m

OP = 30 m + 20 m = 50 m25. (b)

30 m

20 m

30 m

A

BC

D O40 mN

E

S

W

OD = OA + AD = OA + BC= 40 m + 20 m = 60 m

Page 276: yoursmahboob.wordpress.com SBI · yoursmahboob.wordpress.com iii P 101 Speed Tests for SBI Bank Clerk Exam 101 Speed Tests for SBI Bank Clerk Exam is revised and updated edition on

yoursmahboob.w

ordpress.com

10 101 SPEED TEST

26. (d) 21 km

D

B C

5 km

A 13 km8 km

5 km

We have to find out C to D.27. (a)

S

E

N

W 12km

15 km

18 kmO

R

P Q

let O be the starting point and P, Q and R the positionsafter every movement. Hence, Distance from the startingpoint =

Distance of final position R from O = OR = 18 – 12 = 6 km.

28. (b)

W E

NWestSouth

EastS

North

From the figure, it is clear that ‘S’ becomes ‘North-east’in the new figure (dotted line)

29. (e)

5 km

8 km

1 km

2 km

School

5 km

5 km

N

E

S

W

Remaining distance = 8 – (2 + 1) = 5 kmHence, the drivers require to travel 5 km towards northto reach the school again.

30. (c)

25 m

25 m

15 m

South- west direction

SPEED TEST 81. (b) Next train for N. Delhi leaves at 8:30 p.m. Since time

interval between two trains for N. Delhi is 45 minutes. Atrain for New Delhi has left 15 minutes ago.\ Time of information = 8:30 – 45 + 15 = 8 P.M.

2. (b) Time between 1 p.m. on Tuesday to 1 p.m. on Thursday= 48 hrs.The watch gains (1 + 2) = 3 minutes in 48 hrs.Þ it gains 1 min, in 16 hrs.Hence, it will show correct time at 5 a.m. on Wednesday.

3. (b) Anuj reached at = 8 : 15 AMTime when the other man came = 8:15 + 0:30=8:45 AM(who was 40 minutes late)\ scheduled time of meeting = 8:45 – 0: 40 = 8 : 05 AM

4. (d) First clock will gain 11 × 2 minutes in 11 hrs., and secondclock will lose 11 × 1 minutes in 11 hrs.Hence difference will be 33 minutes.

5. (d) His birthday will be in common date of 15 to 18 and 16to 19 i.e., 17th

6. (b) The heights of A, B, C, D and E in ascending orderis E < D < A < B < CClearly, E is the shortest.

7. (a) Position of Kiran from the top = [35 – 7] + 1 = 29 thPosition of Sohan from the top = 9th.Difference of their positions = 29 – 9 = 20\ Mohan’s position from top = 9 + 10 = 19thHence, Kiran’s position from Mohan = 29 – 19 = 10th

For (Qs. 8 to 11)According to the information provided, the order in whichthe boys stand according to their heights is as follows :

D E C A F B8. (c) Clearly, form the above diagram, E is between D and C.9. (b) D is the tallest.10. (d) Counting from the shortest, C is the fourth one in the

line.11. (c) Clearly, from the above diagram, B is the shortest.12. (b) Even if we cannot determine the exact sequence of the

weights of the children, we can conclude on the basisof the information provided that D is the heaviest.

13. (c) The arrangement in the parkingC + S + C + 2S + C + 3S + C + 4S + C + 5S + C +6S + C + 7S + C = 36 vehiclesQ in the second half of the row = 18 vehicles\ C + 7S + C + 6S + 2S = 18 vehiclesHence, no. of scooters= 7 + 6 + 2 = 15

14. (b) After interchanging their positions, position of A fromleft = 11then positions of A form right = 9.\ The total no. of people in the row = (9 + 11) – 1 = 19.

15. (a) Clearly, C comes at the second place – in thedescending order.

1 B

2 C

3 A ¯ Decreasing weights4 E

5 DFor (Qs. 16 & 17) :

The age wise order is as follows :Gita > Kusum > Arti > Archana > Suman

16. (b) 17. (c)

Page 277: yoursmahboob.wordpress.com SBI · yoursmahboob.wordpress.com iii P 101 Speed Tests for SBI Bank Clerk Exam 101 Speed Tests for SBI Bank Clerk Exam is revised and updated edition on

yoursmahboob.w

ordpress.com

11SOLUTIONS18. (b) The order in which the five boys reach the finishing

line is Gaurav, Raj, Mohit, Ashish, Sanchit.Hence Gaurav won the race.

19. (b) From the information given in the question, thenewspaper was read in the following orderB, C, E, A, D.Hence B passed the newspaper to C.

20. (d) There are 18 persons in a row.

7th from left12th from right

(Motilal’s position)

Previous

Present

B

BM

M

15th from left4th from right

21. (d) Positions as given in the question are

7My friend

th 23exactly

in between

rd I

15persons

7th 23rd 39th6persons

\ My position is 6 + 1 + 15 + 1 + 15 + 1 = 39.22. (c) According to Pratap: 20, 21 or 22 ...(i)

According to his sister: not 22 ...(ii)From (i) and (ii), the birthday falls on Apr 20 or 21.

23. (a) R's position = 11th from right\ M's position = (11 + 15 + 1 = ) 27th from right= (40 – 27 + 1 =) 14th from left

24. (a) R = 16th from the right.\ w = (16 – 5 =) 11th from the right.

25. (e) Time at which the train arrived = 7 : 14 PMBut Seema's watch is 6 minutes fast, hence the time inthe Seema's watch = 7 : 20 PM.

26. (c)

R S12th 16th

22ndTotal number of children in the row = 16 + 22 – 1 = 37

27. (d) B > C > D, A, E28. (a) R < M; Q < R, N; N < M

M > N / R > Q29. (e) Total number of boys = 41.30. (d) Geeta > Shilpa

Deepa > GeetaReepa > GayatriFatima is the seniormostBut no other data is there to find who is the juniormost.

31. (e) Correct order can’t be determined.

32. (c)

11th(From front) (From front)

22nd

SunitaNeela

Kamal

3

19th

So there are seven girls between Kamal and Neela.

SPEED TEST 91. (b) The sitting order of the persons are as follows :

B E G F D C A

E

W

SN

Thus, A and B are sitting at the extreme ends.2. (c) Clearly, J is sitting in the middle.3. (d) At one end, we have k and at the other end, we have N.4. (b) Only statement (b) is true.5. (a) The sitting arrangement can be shown as follows:

A

BC

D

E F

Clearly, F is opposite to B.6. (b) Sitting arrangement is as follows :

st nd rd th thth th

G B G B G B G1 1 2 2 3 3 41 2 3 4 75 6The number of boys are less than girls, so we shouldbegin with girl.

(Qs. 7-10) : Seating arrangement is as follows:T

R H

W

YM

D

7. (a) 8. (b) 9. (c) 10. (c)(Qs. 11-15): Seating arrangement is as follows:

KJ P

O

MN

Q

L

11. (b) 12. (c) 13. (e) 14. (a)15. (d)(Qs. 16-20): Seating arrangement is as follows:

JL O

P

M

N

Q

K

16. (e) 17. (c) 18. (d) 19. (a)20. (b)

Page 278: yoursmahboob.wordpress.com SBI · yoursmahboob.wordpress.com iii P 101 Speed Tests for SBI Bank Clerk Exam 101 Speed Tests for SBI Bank Clerk Exam is revised and updated edition on

yoursmahboob.w

ordpress.com

12 101 SPEED TESTFor (Qs. 21 to 25) : Seating arrangement is as follows:

A G

F

D

BC

E

H

21. (a) 22. (b) 23. (e) 24. (c)For (Qs.25 to 30)

VQ

W

P

S

T

R

25. (d) 26. (b) 27. (c) 28. (d)29. (a) 30. (c)

SPEED TEST 10For (Qs. 1 to 5) :

The given information can be summerised in a tablethat follows:

Math ArtsGKStudiesAthleticsStudents

AshaCharuDeepaBeenaEla

× ×××

××××

×

shows good × shows not good

1. (d) From the table above, it is clear that Deepa is goodin Studies, General Knowledge and Arts.

2. (b) Clearly, Beena is good in Studies, General Knowledgeand Mathematics.

3. (a) Obviously, Asha is good in Studies, Mathematics andAthletics.

4. (c) Charu is good in Athletics, General Knowledge andMathematics.

5. (d) From the last row of the above table, it is clear thatEla is good in Studies, General Knowledge and Artsbut not in Athletics.

6. (c) A maximum of five cources can be taken – History,Chemistry, Pschycology, Astronomy andMathematics.

7. (a) There is only one way – History and Psychology.For (Qs. 8 to 12)

A(Farmer)

SonsDaughter

B (Unmarried)(Teacher)

HusbandCD

(Trader)(Doctor)E (Unmarried)(lawyer)

8. (d) Clearly, B and D are the females members in the family.9. (b) From the above table C and D the married couple.10. (c) A, C and E are the male members in the family.11. (d) D, the wife of the trader C is the doctor in the family.12. (c) C is the trader in the family.For (Qs. 13-16)From the given facts, we can summarize,

Friends Game SubjectsA Football Maths, Physics / AccountsB Football Maths, Accounts / PhysicsC Cricket Chemistry, BiologyD Boxing Maths, Accounts.

13. (c) 14. (d) 15. (a) 16. (d)For (Qs. 17-21)

The Organization of the plays will be as follows

5th 6th 7th 8th 9th 10thMon Tue Wed Thu Fri Sat

D B E C F A

17. (c) The organisation would start from play D which is clearfrom above.

18. (b) E is to be organised on 7th.19. (a) The organisation would end with the play A on 10th.20. (a) The play B is organised on 6th which is a Tuesday.21. (d) The correct sequence of the plays is DBECFA.For (Qs. 22-25)

Teacher SubjectsA Hindi, Eng, MathB Hindi, Eng, Geo, History, FrenchC Eng, GeoD Math, HindiE History, French

22. (c) 23. (d) 24. (c) 25. (b)(Qs. 26-30) :

The given information can be summarized as follows :

Flights DayJet Airways Tuesday

British Airways ThursdayDelta Wednesday

Quantas SaturdayEmirates MondayLufthansa SundayAir India Friday

26. (c) 27. (c) 28. (a)29. (e) 30. (d)

Page 279: yoursmahboob.wordpress.com SBI · yoursmahboob.wordpress.com iii P 101 Speed Tests for SBI Bank Clerk Exam 101 Speed Tests for SBI Bank Clerk Exam is revised and updated edition on

yoursmahboob.w

ordpress.com

13SOLUTIONS

SPEED TEST 11

For questions 1 to 5:

Friend Bank OccupationA S ForexB M AgricultureC N EconomistD L TOE R ITF Q ClerkG P Research

1. (b) 2. (c) 3. (a)4. (d) 5. (e)For questions 6 to 10:

Mon PTue RWed WThu VFri SSat QSun T

6. (c) 7. (a) 8. (b)9. (c) 10. (e)For questions 11 to 15:

Friend Shift Day offP II TuesdayQ I MondayR II WednesdayS I SundayT III FridayV III ThursdayW I Saturday

11. (c) 12. (d) 13. (a)14. (d) 15. (c)For questions 16 to 20:

A > D > G ...(ii); C > E > H ...(iii)D > B > F ...(iv); G > C ...(v); F > G ...(vii)Combining these, we getA > D > B > F > G > C > E > H

16. (e)17. (b) A > D > B > F > G. > C , J > E > H18. (a)19. (b) G, C, E and H20. (e)

For questions 21 to 25:Employee Department Sport

A Pers TTB Admin FootballC Admin HockeyD Admin BasketballE Mktg CricketF Pers VolleyballG Mktg LTH Mktg Badminton

21. (c) 22. (b) 23. (e)24. (a) 25. (d)(26-30) : Ascending order of fatness :-

Q < T < V < W < R < S/P < P/SE555F

Ascending order of heightV < W < T < S < R < P < Q

26. (c)27. (e) Even all the conditions are not sufficient to find the

thickest.28. (b)29. (d) P or S can come.30. (c) R is 5th from the left in both arrangements.

SPEED TEST 12

1. (b) Area common to singer and poets.2. (b)

3. (c) Area common to and 4. (b) circle only5. (c) Area not common to rectangle but common to square

and circle.6. (a) Area common to Rectangle, Circle, Square.7. (d) Area common to Rectangle and Circle.8. (a) Only area of square.9. (c) Area common to rectangle, square and circle.10. (d) Sun is star. Moon is a satellite.

Star

SunMoon

11. (a) The required set of students is denoted by regioncommon to any three circle only\ Required number = (13 + 13 + 18 + 18) = 62.

12. (a) The required set of students is denoted by regionslying inside the circles representing History,Mathematics and Science. \ Required number = (9 +14 + 18 + 15 + 16 + 13 + 13 + 20 + 18 + 13 + 16 + 19) = 183.

13. (b) The required set of students is denoted by the regionscommon to the circles representing History andGeography.\ Required number = (20 + 13 + 12 + 18) = 63.

14. (b) Number of students who took History= (16 + 12 + 18 + 20 + 18 + 14 + 13) = 111.

Page 280: yoursmahboob.wordpress.com SBI · yoursmahboob.wordpress.com iii P 101 Speed Tests for SBI Bank Clerk Exam 101 Speed Tests for SBI Bank Clerk Exam is revised and updated edition on

yoursmahboob.w

ordpress.com

14 101 SPEED TESTNumber of students who took Geography= (9 + 16 + 13 + 20 + 13 + 12 + 18) = 101.Number of students who took science= (19 + 15 + 18 + 20 + 18 + 16 + 13) = 119.Number of students who took mathematics= (9 + 14 + 13 + 20 + 13 + 15 + 18) = 102.

15. (c) both students and teachers are parts of college and isdifferents.

16. (b)

Woman

Mother Homosapien

17. (b) Home minister is a minister, minister is part of cabinet.18. (d) All Parrots are birds, but mice is entirely different.19. (a) Some professor may be scientist or researcher.20. (c) Men, Rodents are entirely differents, but both are living

beings.21. (c) Mother and Father are entirely different but both are

parents.22. (d) Nitrogen is air but ice is differents.23. (a) All three items are partly related.24. (d) Tiger is a Carnivore, while elephants is not.25. (b) Herring is type of fish, fish belongs to the class of

animals.26. (c) Nurse and Patient are differents but both are parts of

Hospitals.27. (c) Nose and hand are differents but both are parts of body.28. (b) All diamonds rings are rings, all rings are ornaments.29. (d) Table are furniture but book are differents.30. (c) Chess and table tennis are differents but both are indoor

games.

SPEED TEST 13

1. (c)Radios

Computers Televisions

Phones

Conclusions :I. Not TrueII. True

2. (b) Fingers

Rings

Ears

Neclaces

or

Fingers

Rings Ears

Neclaces

Conclusions :I. TrueII. Not True

3. (d)Cups

Plates

Spoon

Bottles

Cups

PlatesSpoon

Bottles

or

Conclusions :I. Not TrueII. Not TrueBoth conclusions form complementary pair.

4. (a)

Pens

Erasers

Sharpners

Staples

Pens

Erasers

Sharpners

Staples

or

Conclusions :I. Not TrueII. Not True

5. (d) BushesJunglesTrees

Hills

Conclusions :I. TrueII. True

Page 281: yoursmahboob.wordpress.com SBI · yoursmahboob.wordpress.com iii P 101 Speed Tests for SBI Bank Clerk Exam 101 Speed Tests for SBI Bank Clerk Exam is revised and updated edition on

yoursmahboob.w

ordpress.com

15SOLUTIONS

6. (d) WindowsGrills

Glasses or

Windows

Grills

Glasses

Conclusion-I : FalseII: False

7. (d) Painters Artists

Dancers

or Painters Artists

Dancers

Conclusion-I : FalseII: False

8. (b)

BuildingsRoom

Cabins

Conclusion-I : FalseII: True

9. (a)

Necklace

Rings Bracelet

Conclusion-I : TrueII: False

10. (a)

Arms

Hands Muscles

Conclusion-I : TrueII: False

11. (d) According to statement

holidays tripholidaysvacation

or,

holidays trip vacationsIf neither conclusion I and nor II follow.

12. (d) According to statement

birdkites kites aeroplane

or

kites bird Aeroplane

If neither conclusion I and II follows.13. (a) According to statement

plasticsmetals

fibresplastics

or,fibres

plasticsmetal

Only conclusion I follows.14. (a) According to statement

streetsroads

street highway

or, streets

highwayroads

Hence only conclusion I follow.15. (b) According to statement I

plantsanimals

Rocks

plants

or,

Rocksplants

animals

Hence, only conclusion II follows.

16. (d)Holiday Vacation

Trips

or

Holiday

Trips

Vacation

Page 282: yoursmahboob.wordpress.com SBI · yoursmahboob.wordpress.com iii P 101 Speed Tests for SBI Bank Clerk Exam 101 Speed Tests for SBI Bank Clerk Exam is revised and updated edition on

yoursmahboob.w

ordpress.com

16 101 SPEED TESTConclusions :I. Not True II. Not True

17. (d)Kites Birds

Aeroplane

Kites

or

Birds

Aeroplane

Conclusions :I. Not True II. Not True

18. (a) Fibres

Metals

Plastics

Conclusions :I. True II. Not True

19. (b) Rocks

PlantsAnimals

Conclusions :I. Not True II. True

20. (b)

Banks

AcademicsSchool

Institutes

Conclusions :I. Not True II. True

21. (d)Test Exams

QuestionOR

Test Exams

QuestionConclusion I : False Conclusion II : False

22-23.

Force

HeatEnergies

Power

22. (b) Conclusion I : FalseConclusion II : True

23. (a) Conclusion I : TrueConclusion II : False

24-25.

NoteMetals

Coin

PlasticsORMetals

Plastics

Note Coin

ORMetals

Plastics

Note Win

24. (e) Conclusion I : TrueConclusion II: True

25. (d) Conclusion I : FalseConclusion II : False

26. (e)

SymbolsFigures

Graphics

Picture

Page 283: yoursmahboob.wordpress.com SBI · yoursmahboob.wordpress.com iii P 101 Speed Tests for SBI Bank Clerk Exam 101 Speed Tests for SBI Bank Clerk Exam is revised and updated edition on

yoursmahboob.w

ordpress.com

17SOLUTIONSORSymbols

Picture

Graphics

Figures

Conclusion I : True Conclusion II : True27. (d)

Vacancies

Jobs

Occupations

OR

Vacancies

JobsOccupations

Conclusion I : False Conclusion II : FalseFor questions 27 -29 :

According to statements:

Gliders

Parachutes Helicopters

Airplane

or

Gliders

ParachutesHelicopters

Airplane

Parachutes

Gliders Airplane

28. (a) Hence, only conclusion I follows.29. (b) Hence, conclusion II follows.30. (d) According to statements.

Mails

Chats

Updates

or

Mails

Chats Chats

orMails

Hence, conclusion I follows.

SPEED TEST 14

1. (e) Flowers

Bins

Handles

Sticks

orFlowers

Bins

Handles

SticksConclusions I : True II : Not true

III : Not true

2. (b)

towers

Houses

windows

temples

or

towerswindows

temples

Houses

Conclusions I : Not true II : TrueIII : Not true

Page 284: yoursmahboob.wordpress.com SBI · yoursmahboob.wordpress.com iii P 101 Speed Tests for SBI Bank Clerk Exam 101 Speed Tests for SBI Bank Clerk Exam is revised and updated edition on

yoursmahboob.w

ordpress.com

18 101 SPEED TEST3. (c) cots

walls doors chairs

or

wallsdoors

chairs

cots

or

walls doors

chairs

cotsConclusions I : Not true

II : Not trueIII : Not true

As conclusions I and III complement to each other.

4. (d)

trees

gardens stones

fences

Conclusions I : TrueII : TrueIII : True

5. (a)LeavesBox

Books

Jungles

or

BooksLeaves

Jungles

BoxConclusions I : Not true

II : Not trueIII : Not true

6. (d)

ToysDesks

Pens

Roads

or RoadsPens

Toys DesksConclusions I : Not true

II : Not true

7. (c)

Table HutsRing

Bangles

or

Ring

Bangles

Table Huts

or

Table HutsRing

Bangles

Conclusions I : Not trueII : Not true

Since, conclusion I and II form complementary pair.Therefore, either (I) or (II) follows.

8. (e)

Chairs

Rooms

Trees

Poles

Conclusions I : TrueII : True

(Qs. 9-10)

Buildings

Appartment

FlatsHouse

Page 285: yoursmahboob.wordpress.com SBI · yoursmahboob.wordpress.com iii P 101 Speed Tests for SBI Bank Clerk Exam 101 Speed Tests for SBI Bank Clerk Exam is revised and updated edition on

yoursmahboob.w

ordpress.com

19SOLUTIONS

or

Appartment

Flats

BuildingsHouse

9. (b) Conclusions I : Not trueII : True

10. (d) Conclusions I : Not trueII : Not true

(Qs. 11-12)Rivers

Canal

SeasOceans

or

RiversOceans

Seas

Canal11. (d) Conclusions I : Not true

II : Not true12. (e) Conclusions I : True

II : True(Qs. 13-14)

eveningNoon

NightDay

or Noon

NightDay

evening

13. (d) Conclusions I : Not trueII : Not true

14. (a) Conclusions I : TrueII : Not true

(Qs. 15-16)

papers boards

card

or papers

card

boards

15. (c) Conclusions I : Not trueII : Not true

Since conclusions I and II form complementary pair.

16. (d) Conclusions I : Not trueII : Not true

17. (b) CirclesRings

Squares

ellipse

Conclusions I : Not TrueII : True

18. (d)House Appar-

tmentBun-glows

or

House Appar-tment

Bun-glows

Conclusions I : Not TrueII : Not True

19. (a)

Gases LiquidsWater

or

Gases Liquids

Water

Conclusions I : TrueII : Not True

20. (b) HoursSeconds

Day

orHours

Seconds

Day

Conclusions I : Not TrueII : True

(21 -22)

Profess

ors

Teac

hers

Lecturers

or

Page 286: yoursmahboob.wordpress.com SBI · yoursmahboob.wordpress.com iii P 101 Speed Tests for SBI Bank Clerk Exam 101 Speed Tests for SBI Bank Clerk Exam is revised and updated edition on

yoursmahboob.w

ordpress.com

20 101 SPEED TEST

Professors

TeachersLecturers

or

Profess

orsTeachers,Lecturers

21. (a) Conclusions I : True II : Not True22. (b) Conclusions I : Not True II : True(23 - 24) : Possible venn diagram are

Roses Flower Red

or

Roses

Red

Flower

or

Flowers

Red

Red

23. (d)24. (a)25. (a) A possible Venn diagram is

High

Hills

SPEED TEST 15

1. (a) E R W H K A

2 @ 6 # 1

2 @ 6 # 1

©

©

Letter series

Codewithoutcondition

2. (e) M P E K D U

$ 4 2 # 5

4 2 # 5 $

d

d

Letter series

Code

According tocondition (i)

3. (d) T M E I U F

7 $ 2 % 9

9 $ 2 % 9

d

d

Letter series

Code

According tocondition (ii)

4. (a) J T A E R I

3 7 1 2 @ %

% 7 1 2 @ 3

Letter series

Code

According tocondition (i)

5. (c) U K T M I H

# 7 $ % 6

# 7 $

d

« % «

Letter series

Code

According tocondition (iii)

6. (d)2

R

8

N

%

A

9

X

5

H

6

SCondition (iii) is applicable.

7. (a)@

P

6

S

2

R

+

G

7

F

4

TCondition (i) is applicable.

8. (a)+

G

5

H

9

X

6

S

3

D

%

GCondition (ii) is applicable.

9. (a) As, M O D E

# 8 % 6

and D E A F

% 6 7 $Similarly, F O A M

$ 8 7 #10. (e) W E A K

5 % 9 $

W H E N

5 * % 7Therefore,H A N K

* 9 7 $11. (c) 12. (a) 13. (e)14. (b) 15. (d)16. (b) 2 7 0 5 1 4

$ % L T K Qcondition (ii) is applied

Page 287: yoursmahboob.wordpress.com SBI · yoursmahboob.wordpress.com iii P 101 Speed Tests for SBI Bank Clerk Exam 101 Speed Tests for SBI Bank Clerk Exam is revised and updated edition on

yoursmahboob.w

ordpress.com

21SOLUTIONS17. (a) 3 6 4 2 7 9

© H $ Q % ©condition (i) is applied

18. (c) 8 7 5 3 0 6

H % T # L Jcondition (ii) is applied

19. (e) 5 9 2 4 7 6

T @ Q $ % H

20. (d) 4 6 8 9 1 0

$ H J @ K«condition (iii) is applied

21. (a) 2 9 7 6 5 8 1 ® B @ $ © P = B22. (b) 7 2 6 9 5 3 4 ® K B © @P T $23. (c) 8 1 3 5 2 4 6 ® © C T P B K ©24. (e) 4 3 5 2 7 1 8 ® = T P B $ C =25. (a) 4 3 5 9 7 1

? + % « # $

SPEED TEST 161. (e) 3 42 3 #- +¾¾® ¾¾®

3 4O I C- +¾¾® ¾¾®3 4K O 5- +¾¾® ¾¾®

3 4# P I- +¾¾® ¾¾®2 3B $- +¾¾® ¾¾® <

2. (a) 3 3 3 3P # 7 @ D+ + + +¾¾® ¾¾® ¾¾® ¾¾®

3 3 3 3R L I O K+ + + +¾¾® ¾¾® ¾¾® ¾¾®3 3 3 3J 3 2 N C+ + + +¾¾® ¾¾® ¾¾® ¾¾®

3. (c) 11th to the left of 16th from left means 5th from the left.But the sequence has been reversed. Therefore, requiredelement will be 5th from right in the original sequence.5th from right Þ B

4. (c) Consonant Number SymbolSuch combinations are

R3P , N7O , K5D , Q4

5. (e) Number Symbol ConsonantThere is no such combination.

6. (b) Ninth to the right of the 20th from the right means 11thfrom the right, i.e., M.

7. (c) Symbol Consonant Symbol

Such combinations are :

@ F ! : + J C

8. (a) New arrangement

H F 3 U 6 G I T P L 8 9 S 2 7 A M K . . . . . . .

12th from left

9. (d) L +2 S –1 8

A +2 K –1 M

@ +2 I –1 F

6 +1 % +1 G

J +2 D –1 ©

10. (c)

F +5 % +6 L

3 +5 I +6 S

U +5 T +6 ^

+7

+7

+7

7AM

11. (a) D F J T $ # P R Z Q * C M A B @ H K L S + ?only $ # P is the required answer.

12. (e) * Q Z R P # $ T J F D C M A B @ H K L S + ?13. (d) Number of total symbols = 6; Number of total letters

= 16. Since, all the symbols are denoted by 7 and allletters are denoted by 5, sum of the elements of thesequence = 6 × 7 + 16 × 5 = 122

14. (b) When all the symbols are dropped the series becomesas follows:

D F J T P R Z Q C M A B H K L SNow, seventh to the right of twelfth letter from the right= (12 – 7 =) 5th letter from the right, i.e., B.

15. (d) Compare ‘DJ’ and ‘?S’. ‘D’ is the first element from leftend of the series and ‘?’ is the first element from rightend. Similarly, ‘J’ and ‘S’ are third elements from leftand right end respectively. Hence, ‘FT’ is relatedto' + L'.

16. (b) D F J T $ #P R ZQ * C M A B @ H K L S + ?A B C D E F G H I J K L M N O P Q R S T U V

17. (e) Except it there is only one element between first andsecond letter of each group of words when the positionof the letters in the series is taken into consideration.

18. (c) Eighth element from right = (22 + 1 – 8 =)15th elementfrom left.

Hence, the required element which is exactly midwaybetween 5th element from left and 15th element from

left, is15 5

2+æ ö=ç ÷

è ø10th element from left, i.e., Q.

19. (d) Clearly, the given letters, when arranged in the order 5,1, 2, 3, 4 from the word ‘TRACE’.

20. (b) Clearly the given letters, when arranged in the order 4,5, 2, 3, 1, 6 form the word 'STRAVE'.

21. (d) We have to look for Vowel-Number and Number-Vowelsequences.J 1 # P 4 E K 3 A D $ R U M 9 N 51 % T V * H 2 ¸ F 6G 8 Q W4, 3 and 5 are the required numbers.

Page 288: yoursmahboob.wordpress.com SBI · yoursmahboob.wordpress.com iii P 101 Speed Tests for SBI Bank Clerk Exam 101 Speed Tests for SBI Bank Clerk Exam is revised and updated edition on

yoursmahboob.w

ordpress.com

22 101 SPEED TEST

22. (d) $ 9 5 1 % * 2E5555555555F E555555555FD RU M N T V H 7 elements 7 elements

23. (a) After the changing, the series becomes as follows;J 1 # P 4 E K 3 A D $ R U M 9 N W Q 8 G 6 F ¸ 2 H* V T % 15Now, ninth to the right of the eleventh element from theleft® (11 + 9 =) 20th element from the left, i.e., G.

24. (b) We have to look for Symbol Consonant – Consonantsequence and Symbol–Consonant–Symbol sequences.J I # P 4 E K 3 A D $ R U M 9 N 51 % T V H2 ¸ F 6 G 8 Q WOnly T is such a consonant.

25. (e) See the difference between each two successiveelement.(a) A +2 $ – 5 E; A + 2 = $ – 5 = E(b) % + 2V – 5 N; % + 2 = V – 5 = N(c) 2 + 2 F – 5 V; 2 + 2 = F – 5 = V

(d) 4 + 2 K – 51; 4 + 2 = K – 5 = 1(e) 6 + 3 Q – 5 ¸ ; 6 + 2 = 8 – 5 = 2Note that the difference between two successiveelements in 5 is not similar to others.

26. (d) After changing the series becomes as follows :I D 7 1 J P $ 3 E R T 5 £ M 2 N A 4 F H 6 H U 9 # V B @ WNow, twenty-second element from the right end is 3.

27. (c) We have to look forVowel-number-consonant sequence.M £ 5 T R E 3 $ P J 1 7 D I 2 N A 4 F H 6 H U 9 # V B @ WOnly 2 and 4 are such numbers.

28. (e) D 2 JR + 4 P + 4 D + 4 A3 + 4 1 + 4 2 + 4 F£ + 4 E + 4 J + 4 I

29. (d) M £ 5 T R E 3 $ P J 1 7 D I 2 N A 4 F H 6 U 9 # V B @ W30. (b) Fifth element towards right of the seventeenth element

from the right end implies twelfth element from the rightend. Hence, the required element is 4.

SPEED TEST 17

Solution (1 to 5)After analysis of the given input and various steps of rearrangement it is clear that words and numbers are being rearrangedalternately. The words get rearranged in alphabetical order while the numbers get rearranged in descending order.

1. (c) input : won 13 now 25 72 please go 47Step I : go won 13 now 25 72 please 47Step II : go 72 won 13 now 25 please 47Step III : go 72 now won 13 25 please 47Step IV : go 72 now 47 won 13 25 pleaseStep V : go 72 now 47 please won 13 25Step VI : go 72 now 47 please 25 won 13

Step VI is last step.2. (c) Step III : car 81 desk 15 42 39 tall more

Step IV : car 81 desk 42 15 39 tall moreStep V : car 81 desk 42 more 15 39 tallStep VI : car 81 desk 42 more 39 15 tallStep VII : car 81 desk 42 more 39 tall 15

Step VII is last steps.3. (b) Step II : bell 53 town hall near 27 43 12

Step III : bell 53 hall town near 27 43 12Step IV : bell 53 hall 43 town near 27 12Step V : bell 53 hall 43 near town 27 12Step VI : bell 53 hall 43 near 27 town 12

Step VI is last steps.4. (d) Step II : box 93 25 year end 41 32 value

input :Hence step II can not determine input.

5. (a) input : paper dry 37 23 height call 62 51Step I : call paper dry 37 23 height 62 51Step II : call 62 paper dry 37 23 height 51Step III : call 62 dry paper 37 23 height 51Step IV : call 62 dry 51 paper 37 23 heightStep V : call 62 dry 51 height paper 37 23Step VI : call 62 dry 51 height 37 paper 23

Step VI is last step.

Page 289: yoursmahboob.wordpress.com SBI · yoursmahboob.wordpress.com iii P 101 Speed Tests for SBI Bank Clerk Exam 101 Speed Tests for SBI Bank Clerk Exam is revised and updated edition on

yoursmahboob.w

ordpress.com

23SOLUTIONS(Sol. 6-10):

In step I, the word that comes last in the alphabetical ordercomes to the first place, pushing the rest of the line rightward.In step II, the largest number comes at the second place,pushing the line rightward. Thus, words and numbers getarranged alternately till all the words are in reversealphabetically order and numbers in descending order.

6. (d) Input: glass full 15 37 water now 85 67Step I : water glass full 15 37 now 85 67Step II : water 85 glass full 15 37 now 67Step III : water 85 now glass full 15 37 67Step IV : water 85 now 67 glass full 15 37Step V : water 85 now 67 glass 37 full 15Since the arrangement has been done, there will be nostep VI.

7. (d) Step II : ultra 73 12 16 mail sort 39 kiteStep III : ultra 73 sort 12 16 mail 39 kiteStep IV : ultra 73 sort 39 12 16 mail kiteStep V : ultra 73 sort 39 mail 12 16 kiteStep VI : ultra 73 sort 39 mail 16 12 kiteStep VII : ultra 73 sort 39 mail 16 kite 12Hence, Step VI will be the last but one.

8. (d) We can't work backward in an arrangement type.9. (c) Step II : tube 83 49 34 garden flower rat 56

Step III : tube 83 rat 49 34 garden flower 56Step IV : tube 83 rat 56 49 34 garden flowerStep V : tube 83 rat 56 garden 49 34 flowerStep VI : tube 83 rate 56 garden 49 flower 34

10. (a) Input : hunt for 94 37 good 29 48 book.Step I : hunt 94 for 37 good 29 48 bookStep II : hunt 94 good for 37 29 48 bookStep III : hunt 94 good 48 for 37 29 bookStep IV : hunt 94 good 48 for 37 book 29

(Sol. 11-15):Input : for 52 all 96 25 jam road 15 hut 73 bus stop 38 46Step I : all for 52 25 jam road 15 hut 73 bus stop 38 46 96Step II : bus all for 52 25 jam road 15 hut stop 38 46 96 73Step III : for bus all 25 jam road 15 hut stop 38 46 96 73 52Step IV : hut for bus all 25 jam road 15 stop 38 96 73 52 46Step V : jam hut for bus all 25 road 15 stop 96 73 52 46 38Step VI : road jam hut for bus all 15 stop 96 73 52 46 38 25Step VII : stop road jam hut for bus all 96 73 52 46 38 25 15

11. (b) Step IV : hut for bus all 25 jam road 15 stop 38 96 73 5246 Eighth from the right – road.

12. (c) 13. (c)14. (a) Step V : jam hut for bus all 25 road 15 stop 96 73 52 46 38

Sixth from the left is 2515. (d)(Sol. 16-23):

In step I the least number comes to the left most position,pushing the rest of the line rightward . In step II the wordthat comes last in the alphabetical order shifts to secondfrom left, pushing again the rest of the line rightward. Similarly,in step III the second least number shifts to third from left. Instep IV the second from last in the alphabetical order comesto the fourth position. And this goes on alternately till all thenumbers are arranged in ascending order and the words inreverse alphabetical order.Step V : jam hut for bus all 25 road 15 stop 96 73 52 46 38Step VI : road jam hut for bus all 15 stop 96 73 52 46 38 25Step VIII : stop road jam hut for bus all 96 73 52 46 38 25 15

16. (c) Step II : 18 task bear cold dish 81 63 31Step III : 18 task 31 bear cold dish 81 63Step IV : 18 task 31 dish bear cold 81 63Step V : 18 task 31 dish 63 bear cold 81Step VI : 18 task 31 dish 63 cold bear 81Step VII : 18 task 31 dish 63 cold 81 bear

17. (d) Input : 72 59 37 go for picnic 24 journeyStep I : 24 72 59 37 go for picnic journeyStep II : 24 picnic 72 59 37 go for journeyStep III : 24 picnic 37 72 59 go for journeyStep IV : 24 picnic 37 journey 72 59 go forStep V : 24 picnic 37 journey 59 72 go forStep VI : 24 picnic 37 journey 59 go 72 for

18. (a) Input : nice flower 34 12 costly height 41 56Step I : 12 nice flower 34 costly height 41 56Step II : 12 nice 34 flower costly height 41 56Step III : 12 nice 34 height flower costly 41 56

19. (d) Step II : 16 victory 19 36 53 store lake townStep III : 16 victory 19 town 36 53 store lakeStep IV : 16 victory 19 town 36 store 53 lakeSince the line is already arranged, there will be no fifthstep.

20. (d) We can't work out backward.21. (b) Input : milk pot 18 24 over goal 36 53

Step I : 18 milk pot 24 over goal 36 53Step II : 18 pot milk 24 over goal 36 53Step III : 18 pot 24 milk over goal 36 53Step IV : 18 pot 24 over milk goal 36 53Step V : 18 pot 24 over 36 milk goal 53Step VI : 18 pot 24 over 36 milk 53 goalHence step V is the last but one.

22. (a) Step III : 36 win 44 95 86 ultra box queenStep IV : 36 win 44 ultra 95 86 box queenStep V : 36 win 44 ultra 86 95 box queenStep VI : 36 win 44 ultra 86 queen 95 boxHence 6 – 3 = 3 more steps will be required

23. (a) Input : new 22 model 27 pump 38 11 joinStep I : 11 new 22 model 27 pump 38 joinStep II : 11 pump new 22 model 27 38 joinStep III : 11 pump 22 new model 27 38 joinStep IV : 11 pump 22 new 27 model 38 join

SPEED TEST 18

1. (a)– –+ Þ Þ ´

¸ Þ + ´ Þ ¸

10 × 5 3 – 2 + 3 = ?or, ? = 10 ¸ 5 + 3 × 2 – 3or, ? = 2 + 6 – 3 = 5

2. (b) ––

+ Þ ¸ Þ ´¸ Þ + ´ Þ

63 × 24 + 8 4 + 2 – 3 = ?or, ? = 63 – 24 8 + 4 2 × 3or, ? = 63 – 3 + 2 × 3or, ? = 66

3. (b) 6 × 4 – 9 = 154. (c)5. (a) 6 × 5 = 30, 30 × 3 + 1 = 91, 8 × 7 = 56, 56 × 3 + 1 =

169, 10 × 7 = 70, 70 × 3 + 1 = 211Similarly 11 × 10 = 110, 110 × 3 + 1 = 331

Page 290: yoursmahboob.wordpress.com SBI · yoursmahboob.wordpress.com iii P 101 Speed Tests for SBI Bank Clerk Exam 101 Speed Tests for SBI Bank Clerk Exam is revised and updated edition on

yoursmahboob.w

ordpress.com

24 101 SPEED TEST

6. (a) – ,–,

Þ + + Þ ´+ Þ ´ Þ +

Option (a) : 6 20 + 12 + 7 – 1 = 70L.H.S. = 6 – 20 + 12 × 7 ¸ 1 = 6 – 20 + 84

= 90 – 20 = 70 R. H.S.

7. (c) , –, –

+ Þ ´ ´ Þ¸ Þ + Þ ¸

Given expression ® 175 – 25 5 + 20 × 3 + 10After conversion Þ 175 ¸ 25 + 5 × 20 – 3 × 10

= 7 + 100 – 30 = 778. (a) Using the proper signs, we get

Given expression = 14 × 10 + 42 2 – 8 = 140 + 21 – 8 = 153

9. (c) Using proper notations, we have:

(a) Given statement is 3 2 + 4 < 9 ¸ 3 – 2 or 112

< 1 not true

(b) 3 + 2 + 4 < 18 3 – 1 or 9 < 5, which is not true.(c) 3 + 2 – 4 > 8 4 – 2 or 1 > 0, which is true

(d) 3 ¸ 2 – 4 > 9 ¸ 3 – 3 or 52

- > 0, which is not true.

10. (d) Using the proper notation in (d), we get the statementas:8 × 8 + 8 8 – 8 = 64 + 1 – 8 = 57

11. (c) Using the proper notations in (c), we get the statementas:-5 × 2 2 < 10 – 4 + 8 or, 5 × 1 < 18 – 4or 5 < 12 ® which is true.

12. (b) Using the correct symbols, we haveGiven expression = (23 + 45) × 12 = 68 × 12 = 816.

13. (c) Given expression = 32 × 5 – (15 – 3) × 3 = 160 – 12 × 3 = 160 – 36 = 124 = bce

14. (c) Given expression = 105 + 56 – (20 × 7/14) = 105 + 56 – 10 = 151

15. (b) Given expression = 105 15 × 3 = 7 × 3 = 21 = cb16. (a) Using the proper signs in the given expressions, we

get175 ¸ 25 + 5 × 20 – 3 × 10= 7 + 5 × 20 – 3 × 10 = 7 + 100 – 30 = 107 – 30 = 77

17. (c) Using the proper signs, we get:36 – 8 4 + 6 ¸ 2× 3 = 36 – 2 + 3 × 3= 36 – 2 + 9 = 45 – 2 = 43

18. (d) Interchanging (+ and ÷) and (2 and 4), we get :(1) 4 ÷ 2 + 3 = 3 or 5 = 3, which is false(2) 2 ÷ 4 + 6 = 1.5 or 6.5 = 1.5, which is false.

(3) 2 + 4 ÷ 3 = 4 or 103 = 4, which is false.

(4) 4 ÷ 2 + 6 = 8 or 8 = 8, which is true.19. (d) Using the correct symbols, we have:

Given expression = 8 + 36 ÷ 6 – 6 ÷ 2 × 3 = 8 + 6 – 3 × 3 = 5

20. (b) Using the proper notations in (2), we get the statementas 5 × 2 ÷ 2 < 10 – 4 + 2 or 5 < 8 , which is true.

21. (b) Since, 20 × 10 = 200, therefore, – means ×8 + 4 = 12, therefore, ÷ means +.6 – 2 = 4, therefore, × means – .and 12 ÷ 3 = 4, therefore, + means ÷.Now, given expression= 100 × 10 – 1000 + 1000 ÷ 100 – 10= 100 0 – 1000 + 10 – 10 = 0

22. (b) Using the given symbols, we have:Given expression = 8 + 7 × 8 ÷ 40 – 2

= 8 + 7 × 15 – 2

= 375 =

275

.

23. (c) 9 × 8 + 8 ÷ 4 – 9 = 6524. (b) 20 + 12 – 4 ÷ 8 × 6 = 2925. (d) 40 + 12 ÷ 3 × 6 – 60 = 426. (d) Using the correct symbols, we have

Given expression = 24 x 12 + 18 ÷ 9 = 288 + 2 = 290.27. (d) Using the proper notations in (4) we get the statement

as 2 × 5 – 6 + 2 = 6 or 10 – 6 + 2 = 6 or 6 = 6, which is true.28. (c) Using the proper notations in (3), we get the statement

as 5 × 2 ¸ 2 < 10 – 4 + 8 or 5 × 1 < 18 – 4 or 5 < 14,which is true.

29. (a) Using the correct symbols, we haveGiven expression

= ( )36 4 8 44x8 2x16 1

- ¸ -- +

= 32 8 432 32 1

¸ -- +

= 4 40 1

-+

= 0.

30. (b) Using the correct symbols, we haveGiven expression = (3 × 15 + 19) ÷ 8 – 6= 64 ÷ 8 – 6 = 8 – 6 = 2.

SPEED TEST 19

1. (b) No of right angles in one hour = 2\ No of right angles in 24 hours = 24 × 2 = 48

2. (a) Hour hand covers an angle of 360° in 12 hours.\ Time taken to cover an angle of 135°

= 12 135360

´ =4.5 h \ Required time = 3 + 4.5 = 7.5 = 7:303. (d) Angle made by hour hand for 12 hours = 360°

Angle made by hour hand for 1 hour = 36012

°

\ Angle made by hour hand for 6 hours =36012

°× (6) = 180°

4. (b) In a year, number of weeks = 52 extra day = 1From 2002 to 2008, there are 6 years.So number of extra days = 6 (1) = 6While 2004 and 2008 are leap years, having one more extraday apart from the normal extra day.Thus, number of extra days = 6 + 1 + 1 = 8Out of these 8 extra days, 7 days form a week and so 1 dayremains.Hence, March 1, 2002 is 1 day less then March 1, 2008 i.e.,it is Friday.

5. (c) In one hour, hour hand and minute hand are at right angles 2times.Time = 10 p.m – 1 p.m = 9 hr.\ No. of times, when both hands are perpendicular to eachother in 9 hr = 9 × 2 = 18

6. (b) Here H × 30 = 4 × 30 = 1200.(Since initially the hour hand is at 4. \ H = 4).

Page 291: yoursmahboob.wordpress.com SBI · yoursmahboob.wordpress.com iii P 101 Speed Tests for SBI Bank Clerk Exam 101 Speed Tests for SBI Bank Clerk Exam is revised and updated edition on

yoursmahboob.w

ordpress.com

25SOLUTIONSRequired angle A = 900 and since, H × 30 > A° so,there will be two timings.

Required time T = 2

11(H × 30 ± A) minutes past H.

\ One timing = 2

11(4 × 30 + 90) minutes past 4

= 382

11minutes past 4. Or 4 : 38 approx.

7. (a) Since, in one hour, two hands of a clock coincide only once,so, there will be value.

Required time T = 2 (H 30 A )

11´ + ° minutes past H.

Here H = initial position of hour hand = 3(Since 3 o’clock)

A° = required angle = 0° (Since it coincides)

T = 2 (3 30 0)

11´ + minutes past 3

= 41611

minutes past 3.

8. (b) At 5 o’clock, the hands are 25 min. spaces apart.To be at right angles and that too between 5.30 and 6, theminute hand has to gain (25 + 15) = 40 min. spaces55 min. spaces are gained in 60 min.

40 min. spaces are gained in 60 740 min . 43 min .55 11

æ ö´ =ç ÷è ø

7Required time = 43 min. past 511

\

9. (d) At 4 o’clock, the hands of the watch are 20 min. spaces apart.To be in opposite directions, they must be 30 min. spaces apart.\ Minute hand will have to gain 50 min. spaces55 min. spaces are gained in 60 min.

50 min. spaces are gained in 60 650 min . or 54 min .55 11

æ ö´ç ÷è ø

6Required time = 54 min. past 411

\

10. (a) 55 min. spaces are covered in 60 min.

60 min. spaces are covered in 60 60 min.55

æ ö´ç ÷è ø

565 min.11

=

Loss in 64 min. 5 1665 64 min .11 11

æ ö= - =ç ÷è ø

Loss in 24 hrs. 16 1 824 60 min 32 min .11 64 11

æ ö= ´ ´ ´ =ç ÷è ø

11. (c) 100 years contain 5 odd days.\ Last day of 1st century is Friday200 years contain (5 × 2) º 3 odd days.\ Last day of 2nd century is Wednesday.

300 years contain (5 × 3) = 15 º 1 odd day.\ Last day of 3rd century is Monday.400 years contain 0 odd day.\ Last day of 4th century is Sunday.This cycle is repeated.\ Last day of a century cannot be Tuesday or Thursday orSaturday.

12. (a) The century divisible by 400 is a leap year.\ The year 700 is not a leap year.

13. (b) x weeks x days = (7x + x) days = 8x days14. (c) On 31st December, 2005 it was Saturday.

Number of odd days from the year 2006 to the year 2009= (1 + 1 + 2 + 1) = 5 days\ On 31st December 2009, it was Thursday.Thus, on 1st Jan, 2010 it is Friday.

15. (c) The year 2004 is a leap year. It has 2 odd days.\ The day on 8th Feb, 2004 is 2 days before the day on 8thFeb, 2005. Hence, this day is Sunday.

16. (d) Count the number of odd days from the year 2007 onwardsfrom the year 2007 onwards to get the sum equal to 0 odd day.

Year 2007 2008 2009 2010 2011 2012 2013 2014 2015 2016 2017

Odd day 1 2 1 1 1 2 1 1 1 2 1

17. (b) Each day of the week is repeated after 7 daysSo, after 63 days, it will be Monday.\ After 61 days, it will be Saturday.

18. (c) 17th June, 1998 = (1997 years + Period from 1.1.1998 to17.6.1998)Odd days in 1600 years = 0Odd days in 300 years = (5 × 3) º 197 years has 24 leap years + 73 ordinary years.Number of odd days in 97 years = (24 × 2 + 73) = 121 = 2odd days.Jan. Feb. March April May June(31 + 28 + 31 + 30 + 31 + 17) = 168 days= 24 weeks = 0 odd dayTotal number of odd days = (0 + 1 + 2 + 0) = 3Given day is Wednesday.

19. (d) No. of days between 21st July, 1947 and 21 st July, 1999= 52 years + 366 days.= 13 beap years + 39 ordinary years + 366 days= (13 × 2) odd days + 39 odd days + 2 odd days= (26 + 39 + 2) odd days = 67 odd years = 4 odd days.= (7 – 4) = 3 days before the week day on 21st July,1999 = Saturday.

20. (b) Next train for N. Delhi leaves at 8:30 p.m. Since timeinterval between two trains for N. Delhi is 45 minutes.A train for New Delhi has left 15 minutes ago.Time of information = 8:30 – 45 + 15 = 8 P.M.

21. (b) Time between 1 p.m. on Tuesday to 1 p.m. on Thursday= 48 hrs. The watch gains (1 + 2) = 3 minutes in 48 hrs.it gains 1 min, in 16 hrs.Hence, it will show correct time at 5 a.m. on Wednesday.

22. (b) A reverse flow chart will look as follows:Desk officer - FridaySenior clerk - Friday

Same day

Senior clerk’s leave -ThursdayInward clerk - Wednesday

Next day

23. (b) Ashish leaves his house at 6:40 AM.Ashish reaches Kunal’s house at 7:05 AM.They finish Breakfast at 7:05 + 0:15 = 7:20 AM.That’s the time when they leave Kunal’s house for thieroffice.

24. (b) Anuj reached at = 8 : 15 AMTime when the other man came = 8:15 + 0:30=8:45 AM(who was 40 minutes late)\ scheduled time of meeting = 8:45 – 0: 40 = 8 : 05 AM

25. (d) First clock will gain 11 × 2 minutes in 11 hrs., and secondclock will lose 11 × 1 minutes in 11 hrs.Hence difference will be 33 minutes.

26. (d) 1st of month was Tuesday, hence the date on first Saturdaywas 5th.Hence the other Saturdays of the month are 12, 19, 26.Rama met her brother on 26th.

27. (d)12 hrs 150 min2

=

\ Angle covered by hour hand in 1 min = 12°

\ Angle covered by hour hand in 2½ hrs.

= 150 min =150 × 12°

= 75°

Page 292: yoursmahboob.wordpress.com SBI · yoursmahboob.wordpress.com iii P 101 Speed Tests for SBI Bank Clerk Exam 101 Speed Tests for SBI Bank Clerk Exam is revised and updated edition on

yoursmahboob.w

ordpress.com

26 101 SPEED TEST28. (c) Total no. of days between 27.3.1995

and 1.11. 1994 = 27 + 28 + 31 + 31 + 29 = 146Now, 146 is not completely divided by 7. It we have oneday more then we have 147 days which is completely dividedby 7. Thus, the days of the week on 1 Nov, 1994 wasMonday. But the day will be Tuesday ( We have 146 days)

29. (a) 16 - 1- 1997—Thursday.Number of normal year between 1997 and 2000 = 2We know every year has 1odd day.Now, number of leap year = 1Leap year has 2 odd days\ odd days = 2 + 2 = 4 \ 4 Jan, 2000 was Tuesday.

30. (b) Day is on 28th Feb = TuesdaySince, the leap year is excluded\ The day is on 28th March = Tuesday

SPEED TEST 20

1. (c) Age of C = Total age – age of (A + B + D + E)= Total age – 2 × Average ages of (A + B) – 2 × Average agesof (D + E)

2. (c) From I

N

EW

SS-W

Q

P H

P is to the South-West of Q.From II

F Q

PAgain, P is to the South-West of Q.

3. (e) From I : K

(+)N M(–)From II : F (+) Û K (–)

Combining both: F(+) K(–)

N(+) M(–)

Û

Hence K is the mother of N.4. (e) Statement II:

Rani’s age = x yrs.Þ 2x + x = 72 Þ 3x = 72 Þ x = 24 yrs.Using this with Statement I, we get

Age of Nidhi = 3 times younger than Rani’s age =243 = 8 yrs.

\ Both statements I and II are Sufficient.5. (a)6. (e) Since the ages of none of them is given, no conclusion can be

drawn through both the statements.

7. (d) From both the statements:

Left Right

+ 10

20th

B D

D’s position from the left end cannot be determined.8. (e) From both the statements:

N

S

EW

NENW

SESW

K

D

M

Clearly, Town M is towards South-East direction of Town K.9. (d) From both the statements K, M and T are siblings of P.10. (c) From I. M

P TThus, it can be found that M is to the north-west of T. So, Ialone is sufficient to answer the question.From II. M K TThus, it can be deduced that M is to the north-west of T. So,II alone is also sufficient to answer the question. Thus, theanswer can be found by using either of the statements alone.

11. (e) From I. It is clear that K, D and R are siblings while K and Dare females. But no relation can be found between D and M.From II. It can be found that R’s father is certainly the fatherof K and D as well. Since M is married to R’s father, it meansM is the mother of R.Thus, after combining I and II, we can definitely say that Mis the mother of D. So, both statements I and II are requiredto answer.

12. (e) D’s position = 15th from right\ R’s position = (15 – 10 =) 5th from right = (40 – 5 + 1 = ) 36th from left

13. (c)

30 m

20 m

D was facing East when he started his journey, fromstatement I.

30 m

20 mD was facing East when he started his journey, from statementII.

14. (d) From statement I and II.A

B–

C–

D+

Page 293: yoursmahboob.wordpress.com SBI · yoursmahboob.wordpress.com iii P 101 Speed Tests for SBI Bank Clerk Exam 101 Speed Tests for SBI Bank Clerk Exam is revised and updated edition on

yoursmahboob.w

ordpress.com

27SOLUTIONS15. (c) From Statement I

R S10 km

2 km 2 kmT

Q5 kmPNorth

West East

South

Required distance = PT = PQ + QT = (5 + 10) km= 15 km From Statements II

R S13 km

2 km 2 km

TQP

2 km

Require distance = PT = PQ + QT = (2 + 13) km = 15 km16. (e) From the both statements

15thR 21 S

4th

There are 21 students between R and S.17. (d) From the the statements

Suneeta has two children. However, she may have more sons.18. (c) From Statement I

H

YX

From statement IIR

Y

XW

SW

NW

E

NE

SE

N

S

19. (e)20. (a)21. (d) According to statements I

D is the 20th from right end.According to statement (ii)10 girls are in between B and D.Combining statement (i) and (ii).

B D

D B

10 Girls 20thFrom Right

10 Girls

20thFrom Right

or

Both statements are not sufficient to answer thequestion.

22. (e) According to statement ITown K is in north-west with respect to town D.

K

D

N-WN

N-E

EW

W-SS

S-E

According to statement II.Town M is in S-E w.r.t to D

D

M

N-WN

N-E

EW

S-ES

S-E

Combining statement I and II

D

M

K W-NN

N-E

EW

S-WS

S-E

Town M is in S-E w.r.t town, K.Both statements required to answer the question

23. (d) From statement IK and M are sister of T

sister sister

K M

T

From statement II.T’s father is husband of P’s mother.

Page 294: yoursmahboob.wordpress.com SBI · yoursmahboob.wordpress.com iii P 101 Speed Tests for SBI Bank Clerk Exam 101 Speed Tests for SBI Bank Clerk Exam is revised and updated edition on

yoursmahboob.w

ordpress.com

28 101 SPEED TEST

husband

mother father

T PFrom statements - I and II

sister sister

K M

husband

motherfather

T Pbrother or sister

Both statements are not sufficient to answer thequestion.

24. (b) According to Statement IArun did not went London on sunday.

According to statement II:-Arun’s brother went London on Friday

Hence only statement II are sufficient to answer thequestion.

25. (e) From statement I

new 5Þ 3 9good clothesFrom statement II

are costly 6 7Þ 39good clothesCombining statement - I and IInew 5ÞBoth statements are required to answer the question.

SPEED TEST 21

1. (a) H > K ³ R > T > LConclusions:I. H > L : TrueII. K > T : Not True

2. (e) P = N > D ³ G < B = JConclusions:I. G < P : TrueII. G < J : True

3. (d) F £ C ³ V = Z > X = UConclusions:I. V < U : Not TrueII. Z < F : Not True

4. (b) Q £ E = 1 > N ³ R ³ SConclusions :I. E = S : Not TrueII. S £ N : True

5. (d) Accordingly,F @ N Þ F £ NN d R Þ N > RH @ R Þ H £ R

\ F £ N > R ³ HConclusion : I. H d N Þ H > N [not true]

II. F # R Þ F < R [not true]If neither conclusion I not II is true.

6. (b) Accordingly,M # T Þ M < TT @ K Þ T £ KK $ N Þ K ³ N

\ M < T £ K ³ NConclusion : I. M # N Þ M < N [not true]

II. K d M Þ K > M [not true]Only conclusion II is true.

7. (c) Accordingly,T % H Þ T = HH $ W Þ H ³ W

\ T = H ³ WConclusion : I. W # T Þ W < T [true]

II. W % T Þ W = T orIf either conclusion I or II is true. [true]

8. (a) Accordingly,N d K Þ N > KK # D Þ K < DD % M Þ D = M

\ N > K < D = MConclusion : I. M d K Þ M > K [true]

II. D d N Þ D > N [not true]Only conclusion I is true.

9. (e) Accordingly,J $ B Þ J ³ BB % R Þ B = RR d F Þ R > F

\ J ³ B = R > FConclusion : I. F # B Þ F < B [true]

II. R @ J Þ R £ J [true]Both conclusion I and II are true.

10. (e) V # S Þ V £ SS © L Þ S < LL © J Þ L < J hence V £ S < L < JConclusionsI. V © L Þ V < L (True)II. S © J Þ S < J (True)

11. (b) M # R Þ M £ RR © J Þ R < JJ # H Þ J £ H hence M £ R < J £ HConclusionsI. M # H Þ M £ H (False)II. R © H Þ R < H (True)

12. (a) H $ F Þ H ³ FF @ G Þ F = GG « M Þ G > Mhence H ³ F = G > M

Page 295: yoursmahboob.wordpress.com SBI · yoursmahboob.wordpress.com iii P 101 Speed Tests for SBI Bank Clerk Exam 101 Speed Tests for SBI Bank Clerk Exam is revised and updated edition on

yoursmahboob.w

ordpress.com

29SOLUTIONSConclusionsI. H « M Þ H > M (True)II. H « G Þ H > G ( False)

13. (d) R © J Þ R < JJ « T Þ J > TT # L Þ T £ L hence R < J > T £ LConclusionsI. R @ T Þ R = T ( False)II. J @ L Þ J = L (True)

14. (a) W @ T Þ W = TT $ K ÞT ³ KK « F Þ K > F hence W = T ³ K > FConclusionsI. W $ K Þ W ³ K (True)II. W @ K Þ W = K (False)

15. (d) R £ D ...(i); D > W ...(ii); B ³ W ...(iii)None of the inequations can be combined.However, either I (W < R) or III (W ³ R) must be true.

16. (c) H ³ V ...(i); V = M ...(ii); K > M ...(iii)Combining these, we get H ³ V = M < KHence K > V and I follows.Also, M £ H and II follows.But H and K can't be compared. Hence III does not follow.

17. (a) K < T ...(i); T ³ B ...(ii); B £ F ...(iii)Clearly, the inequations can't be combined.

18. (a) Z < F ...(i); R £ F ...(ii); D > R ...(iii)Clearly, the inequations can't be combined.

19. (b) M > R ...(i); R = D ...(ii); D £ N ...(iii)Combining these, we get M > R = D £ NI does not follow as M and N can't be related.N ³ R and II follows.M > D and III follows.

20. (b) 21. (d) 22. (e)

SPEED TEST 221. (a) Only conclusion I seems to be reasonable. Considering

the different nature of IT Companies differentparameters should be employed for rating. It is notnecessary that if separate rating agency is establishedfor IT companies the investors will get protection oftheir investment. Therefore, conclusion II does notfollow.

2. (a) By increasing the manufacturing capacity the Company"Y" would compete reasonably on the cost front. And,as such it can improve the quality of its products. Hence,conclusion I follows. Conclusion II seems to be anassumption.

3. (b) Considering the amount of loss incurred by PublicSector Units it seems to be true that the Governmentdid not take care in the matter of investment in thePublic Sector Units. The use of term "only" in theconclusion I makes it invalid.

4. (b) Clearly, only conclusion II follows. It is not clear howthe population of developing countries will not increasein the future.

5. (c) It is mentioned in the statement that Mr. X has beendeclared successful in the preliminary screening forthe post of Director of KLM Institute. Therefore, eitherhe will be selected or will not be selected as Director ofKLM Institute.

6. (d) II may be an assumption which the professor isassuming before passing his statement but it definitelycannot be a conclusion. Hence II does not follow. Imay or may not be possible. Hence I does not follow.

7. (b) It is clear that either there is no facility for healthinsurance available or it is available for only affluentsections. Hence I cannot be definitely concluded. IIfollows from the given statement, as ‘limited resources’of the person suggests that he will go to a hospitalwhich provides treatment on nominal charges or free.

8. (a) Only conclusion I follows. The statement talks aboutdedicated ordinary doctros but that in no way infersthat extra ordinary specialists are not dedicated to theirprofession. So conclusion II follows. I is true in thecontext of the scenario prevailing in the country.

9. (c) Either I or II can follow. As the government would bereviewing the diesel prices in light of the spurt in theinternational oil prices, the gove can either decide toincrease or keep the price stagnant (increasing subsidy.)

10. (d) The availability of vegetables is not mentioned in thegiven statement. So, I does not follow. Also, II is notdirectly related to the statement and so it also does notfollow. Probably the demand is surpassing the supply.

11. (d) I and II are assumptions and not conclusions.12. (d) The statement does not say why the poor societies

suffer. Hence I does not follow. II also does not followbecause the statement merely states a fact; it does notlook into the merits of the fact.

13. (c) As Praveen has not yet returned, he might have gotkilled or might have survived. Hence (c) is the correctoption.

14. (b) Nothing has been said in the statements which implythat VCRs and being now manufactured indigenously.Therefore, I is invalid. Since import licence on VCR’shas been withdrawn, they can be now freely imported.

15. (d) Nothing of the sort can be concluded as given in twoconclusions on the basis of the statements.

16. (d) Both of the conclusion are invalid.17. (b) Unless absolute figures are given, no conclusin of the

type I can be made. Since average no. of students perteacher (60) in rural areas is higher than the averageno. of students per teacher (50) in urban areas, we canconclude that more students study with the sameteacher in the rural areas as compared to those in theurban areas.

18. (d) This statement does not mention anything abouthealthy people. Neither does it mention about eveningwalks. Hence none of the conclusions follows.

19. (a) Only this can balance the equation.20. (e) All the given choices would lead to an increase in the

number of visits to health facilities.

SPEED TEST 231. (b) Read the last two sentences of the paragraph.2. (b) Read the line-"If the agriculture sector does well and

world trade conditions improve."3. (c) The Indian economy depends on agricultural sector

which depends a lot on monsoon.

Page 296: yoursmahboob.wordpress.com SBI · yoursmahboob.wordpress.com iii P 101 Speed Tests for SBI Bank Clerk Exam 101 Speed Tests for SBI Bank Clerk Exam is revised and updated edition on

yoursmahboob.w

ordpress.com

30 101 SPEED TEST4. (c) Process of poverty measurement needs to take into

account various factors to tackle its dynamic nature.5. (a) It may not be possible to have an accurate poverty

measurement in India.6. (b) Increase in number of persons falling into poverty varies

considerably across the country over a period of time.7. (c) 8. (e) 9. (c) 10. (b)11. (d) 12. (e)13. (c) The argument boils down to the following, including

the unstated assumption provided by (c):Premise: Students get enough reading practice already.Unstated assumption (c): The reading programprovides only reading practice.Conclusion: The reading program is unnecessary.(a) is not a necessary assumption. The argument is notconcerned with whether improved reading skills wouldhelp the students learn history and science. Rather, theargument involves whether the new program wouldhelp improve reading skills.(b) is not a necessary assumption. The argument isthat no additional reading practice is needed, regardlessof which program provides that practice.(d) is not a necessary assumption. The argument doesnot aim to compare the importance of one disciplineover another.

14. (a) (a) is the correct choice as the passage says that“efficiency is present everywhere, this makes it allpervading”. The passage does not suggest thatefficiency does not pay or can be more of a torture.

15. (c) Only this follows by combining the two statements.

SPEED TEST 24

1. (d) 2. (b) 3. (b)4. (c) 3 goes in the opposite direction: it talks about the utility

of fashion whereas the author does not talk of fashionapprovingly.

5. (b) Read the last sentence of the paragraph.6. (d) 7. (a) 8. (a) 9. (a)10. (b) 11. (d) 12. (d) 13. (c)14. (d)15. (d) The use of word ‘hassle-free’ suggests that the

company assumed that people seek convenience andcomfort.

SPEED TEST 251. (d) 1 is correct as it clearly shows that 1 and 5 are the top

and bottom. 2 and 6 are on the sides of 4. So 2 is opposite6. 2 is correct on the same lines. 3 is opposite 4. 3 iscorrect as I & IV clearly tells that 4 is opposite 3. All thethree statements regarding the figure given in thequestion are correct. Hence, the answer is (d).

2. (a) As it is clear from the figure that face 2 is adjacent toface 3.

1

2

54 3

6

3. (a) 1 RG B

W Y 2O 3

G – OR – WB – Y

4. (a) B & K can't opposite to AM & K '' '' to HB & P '' '' to HFrom above statementsH can’t be opposite to B, K, M, PThus H will in opposite of A.

(Qs. 5-8). Since, there are 64 smaller cubes of equal size, therefore,n = no. of divisions on the face of undivided cube = 4

5. (c) no. of cubes with no face coloured = (n – 2)3

= (4 – 2)3 = 86. (d) no. of cubes with one face painted = (n – 2)2 × 6

= (4 – 2)2 × 6 = 247. (a) Number of cubes with two red opposite faces = 0

(none of the cubes can have its opposite facescoloured)

8. (c) Number of cubes with three faces coloured= 4(cubes at top corners) + 4(cubes at bottom corners)= 8

9. (d)10. (d) From figure, 6 is opposite 4

1 is opposite 23 is opposite 5

11. (c) is opposite to =× will opposite to +¸ will opposite to –hence (a) (b) (d) are not identical.

12. (a) When the sheet shown in fig. (X) is folded to from acube, then the face bearing the dot lies opposite to theshaded face, the face bearing a circle (With ‘+’ signinside it) lies opposite to a blank face and the remain-ing two blank faces lie opposite to each other. Clearly,the cubes shown in figures (B) and (D) cannot beformed since they have the shaded face adjacent to theface bearing a dot and the cube shown in fig. (C) can-not be formed since it shown all the three blank faceadjacent to each other. Hence, only the cube shown infig.(A) can be formed.

13. (c) The symbols adjacent to D are d , ×, and .Hence, the remaining symbol (?) will be oppositeto D.

14. (a) The numbers adjacent to 6 are 2, 3, 4 and 5 (from thefirst three figure). Hence, number 1 will be oppositeto 6.

15. (a)

The total no. of all such smaller cubes= 4 × 2 (layers) = 8.

Page 297: yoursmahboob.wordpress.com SBI · yoursmahboob.wordpress.com iii P 101 Speed Tests for SBI Bank Clerk Exam 101 Speed Tests for SBI Bank Clerk Exam is revised and updated edition on

yoursmahboob.w

ordpress.com

31SOLUTIONS16. (b) There are 10 cubes.17. (c) The number on the fall opposite to face having 1 is

6 because 3, 2 and 4 are adjacent faces of 6.18. (a) The number opposite to 4 is 2 because 5, 1, 6 and 3

are adjacent faces of 419. (a) Bottom face of figure I means opposite face of 5. It will

be 3 because 4, 1, 6 and 2 are adjaceent faces of 5.20. (a) Symbols adjacent to are –, ×, ¸ , +. Therefore,

symbol D will be opposite to .

SPEED TEST 261. (d) The shifting of the elements takes place in such a way

that the change is completed in four steps. So thechange from figure five to six will be similar to thechange from figure one to figure two.

2. (a) In each step, two pairs of elements get changed,beginning from the upper left and lower right. Thechange takes place in three successive steps.

3. (c) The main design rotates respectively through 45°clockwise, 180° and 90° anticlockwise after every twofigures. The shaded leaflet rotates through 45°clockwise, 90° clockwise, 135° clockwise, 45°anticlockwise and 90° anticlockwise.

4. (d) Watch the rotation of each element separetely. Thetriangel rotates by 90° ACW and 180°. The circle rotates45°, 135°, 90° CW 45° CW ... ‘C’ rotates 45°, 90°, 135°,180° CW.

5. (b) In each step elements interchange in pairs while oneelement beginning from one end is replaced by a newone. The line of orientation rotates by 45° ACW.

6. (d) In each step the elements of the upper row shift fromleft to right in cyclic order while elements of the lowerrow shift from right to left in cyclic order.

7. (b) In each step, the whole figure rotates by 45° ACW. Themiddle element interchanges with elements on eitherside alternately while the third element is replaced by anew one.

8. (c) In each step the whole figure rotates by 90° ACW whileone of the end elements is replaced alternately on eitherside.

9. (a) In the first step the elements shift from the upper left tolower right ® middle left ® upper right ® lower left ®upper left. In the next step the elements shift one stepCW in cyclic order.

10. (e) In each step the upper element rotates by 90° ACW.The lower element gets inverted and a curve is addedto it on the upper side.

11. (d) In alternate steps the elements shift one-and-a-half sidesCW while one of the elements beginning from the ACWend gets replaced by a new one in each step.

12. (b) In each step the whole figure rotates by 90° CW whileone element is added in each step alternately on CWand ACW end.

13. (b) In each step the whole figure rotates by 90° ACW andan arc is added on the CW side.

14. (b) In each step the triangles rotate by 90° CW. The shadingof the right triangle changes alternately. The shadingsof the middle and left triangles change in each step in aset order.

15. (a) In each step the quadrilateral rotates by 90° ACW whileit shifts half a side CW alternately.

SPEED TEST 271. (a) From problem figure (4) to (5) the lower design is

reversed laterally while the other design moves to theopposite side. Similar changes would occur fromproblem figure (2) to (3).

2. (a) Symbol changes positions as shown in both thediagrams alternately and symbols in place of the sign‘·’ remain unchanged in each of the successive figure.

· ·

·

·

Figure (1) to (2)

·

Figure (2)3. (c) Each symbol of the figure rotates in Anticlockwise and

a new symbol replaces the symbol at the top which isinverted alternatively in each of the successive figures.

4. (e) From problem figure (1) to (2), first and second figureinterchange their position and are reversed at the newpositions and at the same time, third figure remainsunchanged. From problem figure (2) to (3), second andthird figures interchange their positions and arereversed at the new position and the figure (1) remainunchanged. The same problem is repeated alternatively.

5. (a) In each step, both the line segments close to the sidesof the hexagon, move to the adjacent sides in aClockwise. Also the line segments at the corner moveto the adjacent corner Clockwise and their numberincreases by one, in the first, third, fifth steps.

6. (e) From element I to II the design rotates through 180°7. (b) From element I to II the design is mirror image after

being rotated through 90° clockwise.8. (c) From element I to II the design is enclosed by another

design.9. (e) From element I to II the upper design encloses the lower

design.10. (d) From element I to II the design is divided into four

equal parts and the lower left part becomes shaded.11. (c) For the square follow if 1 = 5 then 2 = 6 rule. In alternate

steps the lower-row elements go to the upper row andnew elements appear in the lower row.

12. (b)13. (b) In each step the corner elements rotate 90° CW and

shift one side ACW alternately. The middle figurerotates 180° and 90° CW alternately.

14. (e) In alternate steps the upper left shifts to centre ® lowerright ® upper left. The upper right shifts to right middle® upper-middle ® upper right and lower left ® leftmiddle ® lower middle ® lower left.

15. (e) In each step the side arrow rotates 90° CW and shiftsone side CW. The smaller arrow rotates 45° ACW andthe larger arrow 45° CW alternately.

SPEED TEST 281. (c) Meaningful words : ARE, EAR2. (d) As L 8 and H 7

A & I *T 4 R 3E $ E $

® ®® ®® ®® ®

Page 298: yoursmahboob.wordpress.com SBI · yoursmahboob.wordpress.com iii P 101 Speed Tests for SBI Bank Clerk Exam 101 Speed Tests for SBI Bank Clerk Exam is revised and updated edition on

yoursmahboob.w

ordpress.com

32 101 SPEED TESTSimilarly,

H 7¾¾® A &¾¾®I *¾¾® L 8¾¾®

3. (b) Others relate to ‘parts of tree’.4. (d) All others are parts of a car.5. (d) DG, IG and SN6. (b) 7. (c)8. (c) 9. (b)10. (e) 11. (a)12. (b) 13. (d)14. (c) 15. (d)16. (a) Some hens are fish. (I-Type)

All fish are birds. (A - Type)I + A Þ I-type\ Some hens are birds.This is Conclusion I.

17. (c) Both the Premises are Particular Affirmative. NoConclusion follows from Particular Premises.Conclusions I and II form Complementary Pair. Therefore,either Conclusion I or II follows.

18. (e) All bats are boys. (A-Type)

All boy are gloves. (A-Type)A + A Þ A type Conclusion.\ All bats are gloves.This is Conclusion II.Conclusion I is Converse of this Conclusion.

19. (d)20. (b) Some doctors are nurses. (I-Type)

All nurses are patients. (A-Type)I + A Þ I -type Conclusion.\ Some doctors are patients.Conclusion II is Converse of this Conclusion.

(Qs. 21-25) :The given information can be summarized as follows.

Plays DaysA WednesdayB FridayC TuesdayD SaturdayE MondayF Thursday

21. (c) Play A is staged on Wednesday. So plays F, B and D arestaged after play A is staged.

22. (b) All others are staged one after the other.23. (e) Play E is staged on Monday, the first day.24. (c) 25. (d)26. (a) B ® 9; A ® 2; R ® *; N ® %; I ® #; S ® 427. (b) D ® 2; M ® @; B ® 9; N ® %; I ® #; A ® 6

Condition (ii) is applied.28. (c) I ® $; J ® 8; B ® 9; R ® *; L ® £; G ® #

Condition (ii) is applied.29. (c) 30. (c)31. (a)32. (d) V $ W ÞV < W

W @ T Þ W ³ TT # H Þ T £ H hence V < W ³ T £ HConclusions %I. V © T Þ V > T ( False)II. H % W Þ H = W ( False)

33. (b) H © M Þ H > MM @ E Þ M ³ EE $ C Þ E < C hence] H > M ³ E < CConclusions %I. C @ M Þ C ³ M ( False)II. H © E Þ H > E (True)

34. (e) N @ J Þ N ³ JJ % R Þ J = RR © H Þ R > H hence] N ³ J = R > HConclusions %I. R # N Þ R £ N (True)II. N © H Þ N > H (True)

35. (d) L @ K Þ L ³ KK © A Þ K > AA $ W Þ A < W hence, L ³ K > A < WConclusions % I. W $ L Þ W < L ( False)II. L # W Þ L £ W ( False)

36. (a) 37. (b)38. (c) 39. (d)40. (e)

Page 299: yoursmahboob.wordpress.com SBI · yoursmahboob.wordpress.com iii P 101 Speed Tests for SBI Bank Clerk Exam 101 Speed Tests for SBI Bank Clerk Exam is revised and updated edition on

yoursmahboob.w

ordpress.com

33SOLUTIONS

SPEED TEST 29

1. (b) – 1.5 will lie leftmost on the number line, hence it is thesmallest.

2. (b) According to the question,

the required value = 2 3114 683 4

´ - ´ = 255176 =-

3. (d) Let the number be x.

Now 34x

3x

=- or 312x

= or x = 36

4. (d) Clearly, 1 1 7 77 77

< = <

Þ 71 is the smallest number..

5. (c)81 th part is black. Half of the remaining

i.e. 167

87

21

=´ is yellow..

Therefore, the part left = 7

16 ,

which is equal to 213 = 3.5 cm.

Hence, length of the pencil =

1675.3

= 8 cm

6. (a) 5 22 9 32 81 2592´ = ´ = .\ Difference = 25 × 92 – 2592 = 2592 – 2592 = 0Hence, the numerical difference is 0.

7. (b) Let the number of buffaloes be x and number of ducksbe y. Then number of legs = 4x + 2y and the number ofheads = x + yNow, 4x + 2y = 2(x + y) + 24or 2x = 24 or x = 12

8. (a) The numbers– 264, 396, 792, and 6336 are divisible by132.

9. (c) Let the number be x.

Now, x + 13x = 112 or 14x = 112 or x = 112 814

=

10. (d) We test3600

9= 400, which is not a perfect cube.

360050

= 72, which is not a perfect cube.

3600 12,300

= which is not a perfect cube

Now, 3)2(8450

3600== , which is a perfect cube

Hence, (d) is the correct option.

11. (d) None of options gives us a four digit perfect square.12. (a) Clearly, 1000 + 1> 1000 × 1

Hence, the required positive integer is 1.

13. (a) Number of women = 7253120 =´

Number of men = 48Þ number of married persons = 80and the number of unmarried persons = (120 – 80) = 40If all the men are assumed to be married, then the numberof married women could be 324880 =-Þ maximum number of unmarried women = 403272 =-

14. (a) Number of odd numbered pages = 5452

11089=

+

15. (b) Let ............,13,11,7,5p =

For p = 5, 2(p 1) 24- =

For p = 7, 2(p 1) 48- =

For p = 11, 2(p 1) 120- =

For p= 13, 2(p 1) 168- =.........................................................................................................................................................................................................Clearly, all the above numbers are divisible by 24.

16. (a) 9 stubs are used to make 1 candle

\ 1044 stubs will be used to make 1044

9 or 116 candles

17. (c) Let the whole number be xAccording to question

69x 20x

+ =

Þ x2 + 20x = 69Þ x2 + 20x – 69 =0Þ x2 + 23x – 3x – 69 = 0Þ x ( x + 23) – 3 ( x + 23) = 0 Þ ( x + 23) ( x – 3) = 0\ x = 3 or – 23, Hence, 3 is only whole number.

18. (b) A clock strikes 4 in 9 seconds.

\ clock strikes 1 in 94

seconds.

Þ clock strikes 12 in 912

4´ seconds

i.e., clock strikes 12 in 27 seconds.19. (c) Given, numbers are 50, 35 and 35.

Now, place value of 3 is 30 and 30 in the numbers 35and 35 respectivvely.

\ Sum of the place values = 30 + 30 = 6020. (b) Below given are those numbers between 500 and 600 in

which 9 occurs only once.509, 519, 529, 539, 549, 559, 569, 579, 589, 590, 591, 592,593, 594, 595, 596, 597, 598.These are 18 numbers.

21. (c) From the given alternatives,112 × 114 = 12768\ Larger number = 114

Page 300: yoursmahboob.wordpress.com SBI · yoursmahboob.wordpress.com iii P 101 Speed Tests for SBI Bank Clerk Exam 101 Speed Tests for SBI Bank Clerk Exam is revised and updated edition on

yoursmahboob.w

ordpress.com

34 101 SPEED TEST22. (a) Out of the given alternatives,

137 × 139 = 19043\ Required smaller number = 137

23. (d) Let the numbers be x and (x + 1),\ x(x + 1) = 8556or, x2 + x – 8556 = 0or, x2 + 93x – 92 x – 8556 = 0or, (x2 + 93) (x – 92) = 0\ x = 92

24. (b) 382 = 1444392 = 1521\ Required number = 1521 – 1500 = 21

25. (d) Number of pieces = 455.88.6 = 53

26. (c) Amount received by each student

= 15487

76 = » 204

27. (c) 115

æ ö-ç ÷è ø

of the number = 84

\ number = 84 5

4´æ ö

ç ÷è ø

= 105

28. (d) A + C = 146or A + A + 4 = 146

or A = 146 4

712-

=

\ E = A + 8 = 71 + 8 = 79

29. (d) Lowest number of set A = 280

5– 4 = 52

Lowest number of other set = 52 × 2 – 71 = 33\ Required sum = 33 + 34 + 35 + 36 + 37 = 175

30. (c) Let total number of goats be x.Then, total number of hens = (90 – x)So, x × 4 + (90 – x) × 2 = 248Þ 4x – 2x = 248 – 180

x = 682

= 34

SPEED TEST 30

1. (c) Let Farah’s age at the time of her marriage be x.

Then, 9(x 8) x

7+ = ´

9xx 8

7Þ - =

8 7x 28

Þ = = years

\ Farah’s present age = 28 + 8 = 36 years

\ Daughter’s age 3 years ago = 136 36

´ -

= 3 years

2. (d) Let the number be x.Q x2 – (12)3 = 976\ x2 = 976 + 1728 = 2704\ x = 2704 52=

3. (c) Q 5 chairs + 8 tables = 6574\ 10 chairs + 16 tables = 6574 × 2 = 13148

4. (b) Let the number be x.Q x2 + (56)2 = 4985Þ x2 = 4985 – 3136 = 1849\ x = 1849 = 43

5. (a) Units digit in 1)7( 4 = . Therefore, units digit in 84 )7(

i..e. 327 will be 1. Hence, units digit in

37771)7( 35 =´´´=

Again, units digit in 1)3( 4 =Therefore, units digit in the expansion of

1)3()3( 68174 ==Þ Units digit in the expansion of

73331)3( 71 =´´´=

and units digit in the expanison of 1)11( 35 =Hence, units digit in the expansion of

557135 1137 ´´ = 1173 =´´

6. (b,c) Here, 0 < p < 1, so let 21p =

Clearly, p1p < 1 1 1or 2

2 1/ 2 2æ ö< <ç ÷è øQ

Also, 21 p

p>

21 1

212

>æ öç ÷è ø

Q or 4 > 0.707

7. (a) 9 lemons cost = 1443

948=

´ paise = cost of 4 mangoes

Þ cost of 3 mangoes = 144 3 1084

´ = paise

= cost of 5 applesor cost of 9 oranges = 108 paise

Þ cost of one orange = 108 129

= paise

8. (a) Let x = 1 × 2 × 3 × 4 = 24Therefore, n = 1 + 24 = 25 = oddClearly, n is an odd integer and a perfect square.This is true for all values of x (product of any fourconsecutive integers).

9. (a) By remainder theorem,96 will have the remainder 1 as 9 has the remainder 1.

Also 8

796 + will have the same remainder as

87)1( 6 + which has the remainder equal to 0.

Page 301: yoursmahboob.wordpress.com SBI · yoursmahboob.wordpress.com iii P 101 Speed Tests for SBI Bank Clerk Exam 101 Speed Tests for SBI Bank Clerk Exam is revised and updated edition on

yoursmahboob.w

ordpress.com

35SOLUTIONS

10. (b)84 3 28 287 (7 ) (343)

342 342 342= =

By remainder theorem, 342

)343( 28 will have the same

remainder as 342128

i.e. the remainder is 1.

Alternatively :

342)1342(

342)343( 2828 +

=

= 28 28 27 28

1 27(342) C (342) ... C 342 1342

+ + + +

Clearly, 1 is the remainder.11. (a) Let the number of students in each row is n and the

number of row is r. Then the number of students in theclass will be nr.According to the question,(n + 4) (r – 2) = nr ............(1)and (n – 4) (r + 4) = nr ............(2)on simplifying equations (1) and (2), we get the systemof equations

n – 2r + 4 = 0n – r – 4 = 0

On solving this system, we obtain r = 8; n = 12Hence, nr = 96

12. (d) Let each of them had x bullets after division. Then,total number of bullets they had after using 4 bulletseach (x 4) (x 4) (x 4) 3x 12= - + - + - = -

Now, x12x3 =- or 6x =Original no. of bullets = 6 × 3 = 18

13. (b) I is false as sum of two negative integers is alwaysnegative.II is true, as product of two negative integers is alwayspositive.III is not always true as x –y can be positive or negativeaccording as x > y or x < y.

14. (a) Let the number be x. Then, as per the operationundertook by the student, we have

112612x

=+ Þ x = 660

Hence, the correct answer = 660 12 1226

+ =

15. (d) Let the present age of Harry and George be x and y,respectively.Then,

y31x = ...........(i)

and )5y(41)5x( -=- ...........(ii)

From (i) and (ii), y = 45 yearsHence, the required age = (5 + y) = 50 years

16. (c) Total no. of cats and dogs = 55 .............. (1)Out of these, 1/5 of cats + 1/4 of dogs had been adopted.

It means that number of cats must be divisible by 5 andnumber of dogs must be divisible by 4.This condition follow, if cat, C = 15, dog, D = 40or C = 35, D = 20.In other cases, (1) will not be satisfied.If C = 15, D = 40, then 3 + 10 = 13 pets were adopted.If C = 35, D = 20, then 7 + 5 = 12 pets were adopted.But 13 > 12.

17. (d) 6n2 + 6n = 6(n2 + n) = 6n(n + 1)n(n + 1) is always divisible by 2, as the product of twoconsecutive natural numbers is always divisible by 2.Þ (6n2 + 6n) is divisible by 6 and 12.Hence, option (d) is correct.

18. (d)MW2

= , M – 10 = W + 5

[where M ® men, W ® women]On solving, we get M = 30, W = 15.\ M + W = 45.

19. (b) Let the number of boys = B.3 2B 18 B 244 3

\ = Þ = = of class

\ Strength of the class = 36\ No. of girls = 36 – 24 = 12.

20. (d) Suppose husband’s age be H years.Then wife’s age W = H – 9

Son’s age S = H 9

2-

Daughter’s age D = H3

According to question,H H 973 2

-+ = Þ 2H 42 3H 27+ = -

H 42 27 69Þ = + =\ W = 60.

21. (d) Let the face value of the National Savings Certificatespurchased by Soma in the first year be Rs.x.\ x + (x + 400) + (x + 800) + (x + 1200) + (x + 1600) + (x + 2000) + (x + 2400) + (x + 2800) = 48000Þ 8x + 11200 = 48000Þ 8x = 36800 Þ x = 4600

22. (c) T = P + 4 = 9; N = T – 3 = 6.23. (a) Let the Number be N

N 765x 42\ = +765 is divisible by 17\ if N is divided by 17, remainder will be 42 – 34 = 8.

24. (a) From option (a),210 – 2n = 960 Þ 1024 – 2n = 960 Þ 2n = 64\ n = 6.Similarly we can try for options (b) and (c). Hence, anyother option does not satisfy the given equation.

25. (c) Let the total number of packages be x.

After uploading 2 x5

packages remaining packages are

2 3x – x x

5 5=

Page 302: yoursmahboob.wordpress.com SBI · yoursmahboob.wordpress.com iii P 101 Speed Tests for SBI Bank Clerk Exam 101 Speed Tests for SBI Bank Clerk Exam is revised and updated edition on

yoursmahboob.w

ordpress.com

36 101 SPEED TESTAccording to the question,

When he uploaded another 3 packages then 12

of

original no. of packages remained.3x x

– 35 2

\ = 3 1x

x – 35 2

Þ =

Þ 6x – 5x = 30 Þ x = 30 Hence, 30 packages were in the van before the first delivery.

26. (c) Given : Divisor = 2 × remainder\ Divisor = 2 × 75 = 150

Also, Divisor 2 dividend3

= ´

3dividend 150 2252

Þ = ´ =

27. (a) 24162 = 89x + 43Þ x = (24162 – 43) ÷ 89 = 271

28. (c) By actual division, we find that 999999 is exactly divisibleby 13. The quotient 76923 is the required number.

29. (b) Clearly, unit’s digit in the given product = unit’s digit in7153 × 172.Now, 74 gives unit digit 1.\ 7153 gives unit digit (1 × 7) = 7. Also 172 gives unitdigit 1.Hence, unit’s digit in the product = (7 × 1) = 7.

30. (b) The digit in the unit’s place of 251 is equal to the remainderwhen 251 is divided by 10. 25 = 32 leaves the remainder 2when divided by 10. Then 250 = (25)10 leaves the remainder210 = (25)2 which in turn leaves the remainder 22 = 4. Then251 = 250×2, when divided by 10, leaves the remainder 4×2 = 8.

SPEED TEST 31

1. (b) Q Product of numbers = (LCM × HCF)Þ 480 × second number = 2400 × 16Þ second number = 80

2. (c) Let numbers be x and y.Q Product of two numbers = their (LCM × HCF)Þ xy = 630 × 9Also, x + y = 153 (given)

since x – y = 2(x y) 4xy= + -

Þ 2x y (153) 4(630 9)- = - ´

23409 22680 729 27= - = =3. (a) Product of the numbers

= HCF × LCM = 21 × 4641= 21 × 3 × 7 × 13 × 17= 3 × 7 × 3 × 7 × 13 × 17\ The required numbers can be 3 × 7 × 13 and 3 × 7 × 17 = 273 and 357

4. (b) Required number = HCF of 108 and 144 = 365. (c) Time gap between two consecutive ticks

5857 sec. and

609608 sec.

\ Required time = LCM of 5857

and 609608

LCM of 58 and 609 1218 secHCF of 57 and 608 19

= =

6. (d) To find the capacity we have to take the HCF of 279,341 and 465.

279 = 31 × 9 × 3341 = 31 × 11465 = 31 × 3 × 5

Þ HCF (279, 341, 465) = 31Capacity of the measuring can be = 31 ml.

7. (b) H.C.F of co-prime numbers is 1.So, L.C.M. = 117/1 = 117.

8. (a) Required time = LCM of 200, 300, 360, 450 sec = 1800 sec.

9. (c) The LCM of 18, 22, 30 is 990.So, they will meet each other after 990, ie, 16 min and 30 sec.

10. (d) The required number must be a factor of (11284 – 7655) or3629.Now, 3629 = 19 × 191\ 191 is the required number.

11. (c) The required number = LCM HCFFirst number

´=

2079 27189

´= 297

12. (a) Clearly, the required number must be greater than the LCMof 18, 24, 30 and 42 by 1.Now, 18 = 2 × 32

24 = 23 × 330 = 2 × 3 × 542 = 2 × 3 × 7

\ LCM = 32 ×23 × 5 × 7 = 2520\ the required number = 2520 + 1 = 2521

13. (d) The first number = 2 × 44 = 88

\ The second number = HCF LCM

88´

= 44 264

88´

= 132

14. (b) HCF = 12. Then let the numbrs be 12x and 12y.Now 12x × 12y = 2160 \ xy = 15Possible values of x and y are (1, 15); (3, 5); (5, 3); (15, 1)\ the possible pairs of numbers (12, 180) and (36, 60)

15. (c) Bells will toll together again at a time, which is obtained bytaking L.C.M. of their individual tolling intervals.L.C.M. of 9, 12 and 15 = 180 minThey will toll together again after 180 min, i.e. 3 hours.Time = 8 + 3 = 11 a.m.

16. (b) Since each rod must be cut into parts of equal length and eachpart must be as long as possible, so HCF should be taken.HCF of 78, 104, 117 and 169 = 13.

No. of parts from 78cm. rod = 78 613

=

No. of parts from 104 cm. rod = 10413 = 8

No. of parts from 117 cm. rol = 117 913

=

No. of parts from 169 cm. rod = 169 1313

= .

\ Maximum no. of pieces = 6 + 8 + 9 + 13 = 3617. (c) Let the numbers be x and 4x.

Then, x4x2184 ´=´

or 1764x4 2 =

or 441x2 = or x = 21Þ 4x = 4 × 21 = 84Thus the larger number = 84

18. (b) Let us check each of the options here starting with (a)

(a) 13 11 133 55 6 30

+ = < (b) 51265

38

411

>=+

Page 303: yoursmahboob.wordpress.com SBI · yoursmahboob.wordpress.com iii P 101 Speed Tests for SBI Bank Clerk Exam 101 Speed Tests for SBI Bank Clerk Exam is revised and updated edition on

yoursmahboob.w

ordpress.com

37SOLUTIONS

19. (b)12 10.1008, 0.1

119 10= =

102.0394

= and 101.0697

=

Thus, 101 is the least.

20. (c) Reciprocal of 4

444

23

23

32

=÷øö

çèæ=÷

øö

çèæ =

423

-æ öç ÷è ø

(a), (b), (d) are all equivalent to 4

432

21. (c) 1.65m = 165 cmRequired length = LCM of 25 and 165 = 825 cm = 8.25 m

22. (c) 312.0165

= , 352.0176

= , 388.0187

=

Therefore, 187

176

165

<<

23. (d)48

451161615

1229 +

=+ = 48

161

Therefore, 448

1924831

48161

==+ = a whole number

And 161 17 348 48

- = = whole number

Between 3148

and 1748

; 1748

is the least fraction.

Clearly, the least fraction among the given fractions in

options is 1748

.

24. (b) Correct asecending order is2 9 5 83 13 7 11

< < <

25. (a) Let us take a proper fraction, such as 21 .

Now, the new fraction = 43

2221

=++

Thus, 21

43

>

26. (b) As x is a positive number, we have 1xx

=

1x11

x1x

>+=+

, 11x

x<

+

13x

113x

13x3x2x

<+

-=+

-+=

++

27. (b) LCM of 6, 5, 7, 10 and 12 = 420 seconds420 7 minutes60

= = .

Therefore, in one hour (60 minutes), then will fall

together 8 times 607

æ öç ÷è ø

excluding the one at the start.

28. (b) HCF = 22 × 32

LCM = 24 × 35 × 52 × 72

Ist number = 23 × 34 × 52nd number = 24 × 32 × 52

observing the above situation, we conclude that thethird number must be

2 2 3 2 2 5 22 3 3 7 2 3 7= ´ ´ ´ = ´ ´x29. (d) Product of numbers = HCF × LCM

Þ The other number = 1600480

1604800=

´

30. (c) The traffic lights will again change at three differentroad crossings simultaneously after the LCM of 48, 72and 108i.e., after every (432 sec) 7 minutes and 12 seconds, i.e.the earliest at 8 : 27 : 12 hours.

SPEED TEST 32

1. (a) 1190 ? 30947225

´ = or, 1190 ? 3094

85´

=

or, 3094 85? 221

1190´

= =

2. (a) ( )2? 2 5 5 1 5 1 2 5- = - = + -Q

\ ? = 6

3. (a) ( )? 25

11 3 1836=

´ - ? 5 5 16 33 18 15 3

Þ = = =-

6 23

\= =

4. (c)5. (e) 169 – 25 – 26 + 7 = (?)2 = 125 = ?2 Þ ? = 125 5 5=

6. (e) ( ) ( )2 2316 5 169 ?- + =256 – 125 + 13 = (?)2

144 = (?)2

? = ± 12

7. (b) ( )2? 225 2304 12= + + = 15 + 48 + 144 = 207

8. (b) ? 450 890 685 2025 45= + + = =

9. (c) ( )1 3? 6859 4 19 4 23= + = + =

10. (e)447? 7 73 2621

= ´ + - = 149 73 26 196 14+ - = =

11. (d)60 40 20? 5 5 50

2 2-æ ö= ´ = ´ =ç ÷è ø

12. (d) 2? 4 16= =\ ? = 256

13. (b) ( )2 255? 317 5

= =´

\ ? 3=

14. (d) ( )2 9 16 5? 80 80 20 10036

´ ´= + = + =

? 100 10= =Q

Page 304: yoursmahboob.wordpress.com SBI · yoursmahboob.wordpress.com iii P 101 Speed Tests for SBI Bank Clerk Exam 101 Speed Tests for SBI Bank Clerk Exam is revised and updated edition on

yoursmahboob.w

ordpress.com

38 101 SPEED TEST

15. (a) ( )2? 6 1 2 6= + - = 6 1 2 6 2 6 7+ + - =

16. (c)18412 26 73 ?23

´ + - =

= 96 26 73 ?+ - = = 122 73- = 49 7=

17. (b) 169 – 64 – 676 + 2 = (?)2

= 169 – 64 – 26 + 2 = (?)2 = 171 – 90 = 81\ ? = 9

18. (c) ( )74 676 42 ? 496´ - ´ =

( )74 26 496?

42´ -

= = 1428 ¸ 42 = 34 = (34)2 = 1156

19. (e) 312

20. (e) ?2 = 2 48 8´ = – 21 + 8 + 49 – 14 8 = 14 8 – 21 + 57 – 14 8 = 36 = 62

\ ? = 621. (e) 7365 + 29.16 + ? = 7437.16

? = 473.16 – 7394.16

? = 43 = 1849

22. (b)3756 67

804´æ ö

ç ÷è ø = 250047

23. (b) 17 51 152 289+ + +

= 17 51 152 17+ + + = 17 51 169+ +

= 17 51 13+ + = 17 8+ = 25 = 5

24. (d) 217 64 217 8 225 15+ Þ + = =25. (a)

26. (d)210.25 21025

100 10000+ Þ

145 14510 100

+ Þ 14.5 + 1.45 = 15.95

27. (a) 4 2000 454 1685 4005 425

25-Clearly, the required least number is 25.

28. (b) Let the number be x.Now, according to the question,x2 – (22)3 = 9516or, x2 = 9516 + (22)3 = 9516 + 10648 = 20164Þ x = 20164 142=

29. (a) Let the number be x.According to the question.(4052 – x2) × 15 = 41340

Þ 2 413404052 x 275615

- = = Þ x2 = 4052 – 2756 = 1296

\ x = 1296 = 36

30. (a) Number of rows = 34969

187349691 1

24928 2242569367 2569

´

Þ 34969 = 187Hence, the number of rows = 187

31. (c) 93 × 812 ¸ 273 = (3)?

32 × 3 × 34 × 2 ¸ 33 × 3 = (3)?

36 × 38 ¸ 39 = (3)? or 36 + 8 – 9 = (3)?

35 = (3)? or ? = 5

32. (c) ( ) ( ) ( ) ( )( )3 5 3 ?9 81 27 3´ ¸ =

(3)3 × (9)5 ¸ (3)3 × 2 = (3)? or (3)3 × (3)2 × 5 ¸ (3)6 = (3)?

(3)3 + 10 – 6 = (3)? or (3)7 = (3)?

? = 733. (e) 81.1 × 42.7 × 23.3 = 2?

2? = (23)1.1 × (22)2.7 × 23.3 or 2? = (2)3.3 × (2)5.4 × 23.3

2? = (2)3.3 + 5.4 + 3.3 or 2? = (2)12.0

? = 12

34. (a) ? = (49)3 ¸ (7)2 =49 49 49

7 7´ ´

´ = 2401

35. (a) 643.1 × 84.3 = 8?

Þ (82)3.1 × (8)4.3 = 8? Þ 86.2 × 84.3 = 8?

Þ 86.2 + 4.3 = 8? Þ 810.5 = 8?

? = 10.536. (d) 8? = 87 × 26 ¸ 82.4

8? = 87 × 2

2.48

8 or 8? = 87 + 2 – 2.4

8? = 86.6 or ? = 6.637. (a) ? = (31)31 × (31)– 27

? = (31)31 – 27

? = (31)4

? = (961)2

38. (e)( ){ }( ){ }

22

22

12

12

-

- =

( )( )

4

412

112

-

-=

39. (c)4 4

?3

6 6 66

´= Þ 68– 3 = 6? Þ 6? = 65 Þ ? = 5

40. (c) ( ) ( )6 3? 48 8 64 8= ´ ¸ = 3 6 48 8 8´ ¸ = 3 6 4 58 8+ - =

\ ? = 5

SPEED TEST 33

1. (b)2. (e) ? = 456.675 + 35.7683 × 67.909 – 58.876

= 456.675 + 2428.98 – 58.876= 2885.66 – 66 – 58.876 = 2826.78 » 2830

3. (c)( ) ( ){ }2 252 45 2704 2025?

8 8

+ += = =

47298

= 591.125

4. (b) ? = (12.25)2 – 625 = (12.25)2 – 25= 150.0625 – 25 = 125.0625

5. (e) ? = 572 + 38 × 0.50 – 16= 572 + 19 – 16 = 591 – 16 = 575

6. (a) ? = 1056 7 5 13

3 2 11´ ´ ´´ ´

= 7280

7. (b) ? 9´ = 6318 + 26 = 243

\ ? = 2439 = 27

\ ? = (27)2 = 729

Page 305: yoursmahboob.wordpress.com SBI · yoursmahboob.wordpress.com iii P 101 Speed Tests for SBI Bank Clerk Exam 101 Speed Tests for SBI Bank Clerk Exam is revised and updated edition on

yoursmahboob.w

ordpress.com

39SOLUTIONS

8. (c)78700? 4 120 45 460 5051750

» + ´ = + = 525;

9. (c) ? 78 42 9 129 130+ + =; ;

10. (b)1 2 3 1715 85.75 858 3 5

´ ´ ´ = »

11. (a)561 20 320.5 32035

´ = »

12. (d) 2(15) 730 225 27 6075´ = ´ =

13. (d) Given expression implies 3325 152?25 16

= ´

= 133 × 9.5 = 1263.5

14. (e) 1 2 2? 5 2 35 15 3

= + + + + +

1 2105 15 3

2= + + +

= 3 2 10 1510 10

15 15+ +

+ = +

= 10 + 1 = 1115. (c) ? = – 15 – 27 – 88 – 63 + 255

= –193 + 255 = 6216. (b) Given expression can be written as

2525 0.25 7?5

´ ´= = 883.75

17. (b) 14 57 20 2 3 20 2 1 2 4?19 70 21 1 10 21 1 1 7 7

= ´ ´ = ´ ´ = ´ ´ =

18. (e) 500 32 50 162?100 100

´ ´= + = 160 + 81 = 241

19. (d) 45316 + 52131 – 65229= ? + 15151

Þ 32218 = ? + 15151\ ? = 32218 – 15151 = 17067

20. (c) 184 4 184 4? 8400 23 4 23

100

´ ´= = =

´ ´

21. (a) 4/3 5/ 3? ? 23 128´ = ´Þ ?3 = 25 × 27 = 212

\ ? = (212)1/3 = 24 = 1622. (b) 16% of 450 ?% of 250 = 4.8

Þ16 ?450 250 4.8

100 100´ ¸ ´ =

Þ 72 2.5 ? 4.8¸ ´ =

Þ722.5 ?4.8

´ =

72? 64.8 2.5

\ = =´

23. (e) ? 11 1521- =Þ ? 11 39- =

Þ ? 39 11 50= + =2? (50) 2500\ = =

24. (a) ? × 40 = 8059 – 7263 = 796

\ 796? 19.9

40= =

25. (a)14 ? 4062 5 4062 812.45

´ = ¸ = ´ =

\ 812.4? 203.1

4= =

26. (e) 1? 3.5 (80 2.5) 3.5 802.5

æ ö= ´ ¸ = ´ ´ç ÷è ø

3.5 32 112= ´ =27. (c) 13% of 258 – ? = 10

\ ? = 13% of 258 – 10

= 25813 10 33.54 10 23.54

100´ - = - =

28. (d)4 3 5 4 11 5? 25 4 8 5 4 8

= ´ ¸ = ´ ¸

4 11 8 88 1335 4 5 25 25

= ´ ´ = =

29. (e) ? = 623.15 – 218.82 – 321.43 = 623.15 – 540.25 = 82.930. (b) ? × 50 = 5437 – 3153 + 2284 = 7721 – 3153 = 4568

\ 4568? 91.36

50= =

SPEED TEST 34

1. (a) Assume the third number = xAccording to question2 × 280 + x + 178.5 × 2 = 281 × 5or, 560 + x + 357 = 1405or, x + 917 = 1405or, x = 1405 - 917 = 488

2. (b) Age of the fourth friend = 31 × 4 - 32 × 3= 124 - 96 = 28 years

3. (a) Required average

965 362 189 248 461 825 524 234

8+ + + + + + +

=

= 3808 476

8=

4. (b) 21a + 21b = 1134or, 21 (a + b) = 1134

a + b = 1134 54

21=

\ Required average = 54 27

2 2a b+

= =

5. (e) Let the first number be = 6x\ Second number = 3xand the third number = 2xAccording to the question,6x + 3x + 2x = 154 × 3or, 11x = 154 × 3

Page 306: yoursmahboob.wordpress.com SBI · yoursmahboob.wordpress.com iii P 101 Speed Tests for SBI Bank Clerk Exam 101 Speed Tests for SBI Bank Clerk Exam is revised and updated edition on

yoursmahboob.w

ordpress.com

40 101 SPEED TEST

\ x = 154 3 42

11´

=

\ Required difference = 6x – 2x = 4x = 4 × 42 = 1686. (d)7. (c) Average score

= 16 [221 + 231 + 441 + 359 + 665 + 525]

= 16 [2442] = 407

8. (e) Let A = xAccording to the questionx + x +1 + x + 2 + x + 3 + x +4= 5 × 48Þ 5x + 10 = 240Þ 5x = 230\ x = 46\E = 46 + 4 = 50\ A × E = 46 × 50 = 2300

9. (b) 16a + 16b = 672or, 16 (a + b) = 672

\ a + b = 67216 = 42

Required average = 2

a b+ =

422

= 21

10. (e) Third number = 290– (48.5 × 2) – (53.5 × 2)= 290 – 97 – 107 = 86

11. (b) Total age of the family of five members = 24 × 5 = 120Total age of the family of five members before 8 years= 120 – 5 × 8 = 120 – 40 = 80

So, Required average age = 80 16yr5

=

12. (e) x + x + 2 + x + 4 + x + 6 = 4 × 36Þ 4x + 12 = 144 Þ 4x = 144 – 12

Þ 4x = 132 Þ 132

334

x = =

13. (b) Let the score of Ajay = xRahul = x – 15Manish = x – 25According to question, x = 63 + 30\ x = 93\ Score of Ajay = 93then Rahul = 93 – 15 = 78then Manish = 93 – 25 = 68Total marks of Rahul, Manish and Suresh

= 3 × 63 = 189\ Suresh = 189 – (78 + 68) = 43\ Manish + Suresh = 68 + 43 = 111

14. (a) Suppose marks got in physics, chemistry and mathematicsare P, C and M.P + C + M = C + 120\ P + M = 120

P M 602+

=

15. (c) By Direct Formula :

Average = 2 60 30 2 60 30

40 km/hr60 30 90´ ´ ´ ´

= =+

16. (c) Total of 10 innnings = 21.5 × 10 = 215Suppose he needs a score of x in 11th innings; then average in

11 innings = 215 24

11x+

= or, x = 264 – 215 = 49

17. (a) Let the total journey be x km. Then x3

km at the speed of 25

km/hr and x

km4 at 30 km/hr and the rest distance

53 4 12x xx xæ ö- - =ç ÷è ø at the speed of 50 km/hr..

Total time taken during the journey of x km

= 5

hrs hrs hrs3 25 4 30 12 50

x x x+ +

´ ´ ´ =

18 3hrs hrs

600 100x x

=

\ average speed = 100 133 km/hr

18 3 3600

xx

= =

18. (a) Let the total no. of workers be x.Now, 8000 x = 7 × 12000 + (x – 7) × 6000

Þ 42000 212000

x = =

19. (d) Let the average score of 19 innings be x.

Then, 18x 98x 4

19+

= + Þ x = 22

The average score after 19th innings= x + 4 = 22 + 4 = 26

20. (b) Total weight of 45 students= 45 × 52 = 2340 kgTotal weight of 5 students who leave= 5 × 48 = 240 kgTotal weight of 5 students who join= 5 × 54 = 270 kgTherefore, new total weight of 45 students= 2340 – 240 + 270 = 2370

Þ New average weight kg3252

452370

==

21. (c) Total score of 40 innings = 40 × 50 = 2000Total score of 38 innings = 38 × 48 = 1824Let the highest score be x and the lowest score be y.Sum of the highest and the lowest score= x + y = 2000 – 1824Þ x + y = 176 ...(i)and by question, x – y = 172 ...(ii)Solving (i) and (ii), we get x = 174

22. (c) Let average cost of petrol per litre be Rs x

\ 12000x 4000 4000 40007.5 8 8.5

=+ +

= 12000 3

10 1 101 1 1400075 8 857.5 8 8.5

=æ ö + ++ +ç ÷è ø

32 1 2

15 8 17

=+ +

6120 Rs7.98767

= = per litre

Page 307: yoursmahboob.wordpress.com SBI · yoursmahboob.wordpress.com iii P 101 Speed Tests for SBI Bank Clerk Exam 101 Speed Tests for SBI Bank Clerk Exam is revised and updated edition on

yoursmahboob.w

ordpress.com

41SOLUTIONS23. (d) Let the three numbers are x1, x2, x3 and x3 be the largest

number.Given, average of three numbers = 135

\ x1 + x2 + x3 = 3 × 135 = 405Since, x3 = 180x1 + x2 = 225 .......... (i)Given x1 – x2 = 25 .......... (ii)Solving (i) and (ii), x1 = 125, x2 = 100Hence, the smallest number = 125 – 25 = 100

24. (d) Total age of 10 students =12.5 × 10 = 125 yearsTotal age of 20 students = 13.1 × 20 = 262 years

\ Average age of 30 students 9.1230

262125=

+= years

25. (d) Replacing the two numbers and arranging them inascending order, we get31, 35, 35, 36, 38, 45, 46, 52, 55, 60Since no. of observation is even

\ Median =

N Nth 1 th2 2

2

é ùæ ö+ +ç ÷ê úè øë û observation

= ( )5th 6th obs.

2+

38 45 83 41.52 2+

= = =

26. (b) Total of x and y = 40 × 2 = 80

\ Average of x, y and z = x+y+z3

30390

31080

==+

=

27. (a) Sum of 11 numbers = 11 × 10.9 = 119.9Sum of first 6 numbers = 6 × 10.5 = 63Sum of last 6 numbers = 6 × 11.4 = 68.4The middle number = Sum of the first six + Sum of thethe last six – Sum of all the 11

= 63 + 68.4 – 119.9= 11.5

28. (a) Sum of 10 numbers = 402Corrected sum of 10 numbers = 402 – 13 + 31 – 18 = 402

Hence, correct average 2.4010402

==

29. (d) Required average number of microwave

=1012

10201242+

´+´ =

504 200 70422 22+

= = 32

30. (d) Temperature on the fourth day= 40.2 × 4 + 41.3 × 4 – 40.6 × 7= 160.8 + 165.2 – 284.2 = 41.8°C

SPEED TEST 35

1. (b) 76% of 1285 = 35% of 1256 + ?Þ 976.6 = 439.6 + ? Þ 976.6 – 439.6 = ?\ ? = 537

2. (b) (21.5% of 999)1/3 + (43% of 601)1/2 = ?(1000 × 21.5%)1/3 + (600 × 43%)1/2 = ?(215)1/3 + (258)1/2 = ?(216)1/3 + (256)1/2 = ?(6)3 ×1/3 + (16)2 × 1/2 = ?6 + 16 = ?? = 22

3. (a) 64.5% of 800 + 36.4% of 1500 = (?)2 + 38516 + 546 = (?)2 + 381062 – 38 = (?)2

1024 = (?)2

(32)2 = (?)2

? = 324. (c) 41% of 601 – 250.17 = ? – 77% of 910

» 246 – 250 = ? – 701» 701 + 246 – 250 = ?» 700 = ?

5. (a) 40.005% of 439.998 + ?% of 655.011 = 228.5

» 40 x440 655 229

100 100´ + ´ =

» 176 + 655x 655x229 229 176100 100

= » = -

53 100x 8655´

= »

6. (d) 25% of 84 × 24% of 85 = ?21 × 204 = ?428.4 = ?

7. (b) 20.06%of 599 + 10.01% of 901 = ?20 10600 900 ?

100 100» ´ + ´ =

» 120 + 90 = ?\ ? » 210

8. (a) 14.2% of 5500 + 15.6% of ? = 179515.6781 ? 1795100

+ ´ =

15.6 ? 1014100

´ =

1014 100? 650015.6

´= =

9. (e) 36% of 245 – 40% of 210 = 10 – ?88.2 – 84 = 10 – ?4.2 = 10 – ?? = 10 – 4.2 = 5.8

10. (a)12

of 3842 + 15% of ? = 2449

1921 + 15 2449100

x + =

15 2449 1921100

x´ = -

528 10015

x ´= = 3520

11. (c) 57% of 394 – 2.5% of 996

= 57 2.5394 996

100 100´ - ´

= 225 – 25 = 20012. (a) 40% of 265 + 35% of 180 = 50% of ?

Þ 265 × 0.4 + 180 × 0.35 = ? × 0.5

Þ 106 + 63 = ? × 0.5 Þ 169? 3380.5

= =

13. (b)1 14 3 ?5 3

´ + = 20% of 120 Þ 21 10 ? 120 0.25 3

´ + = ´

Þ 7 × 2 + ? = 24 Þ ? = 1014. (c) 14% of 250 × ? % of 150 = 840

14 150 ?250 840100 100

´´ ´ =

Page 308: yoursmahboob.wordpress.com SBI · yoursmahboob.wordpress.com iii P 101 Speed Tests for SBI Bank Clerk Exam 101 Speed Tests for SBI Bank Clerk Exam is revised and updated edition on

yoursmahboob.w

ordpress.com

42 101 SPEED TEST35 × 1.5 × ? = 840

? = 840

35 15´? = 16

15. (b) ? = 18 27.5609 450

100 100´ + ´

= 109.62 + 123.75 = 233.37 » 233

16. (c)125 853060 ? 408100 100

´ - ´ =

3825 – 408 = ? × 85

1003417 100 ?

85´

=

4020 = ?

17. (d) 500 × 300100 100

x y= ´

Þ 5x = 3y

Þ53xy = .... (i)

Þ200 60

100 100xy ´

Þ xy = 3000

Þ5 30003xx´ =

53xyé ù=ê úë û

Q

Þ 5x2 = 3000 × 3 Þ x2 = 3000 3

Þ 2 1800x =

Þ 1800x = = 2 3 3 10 10´ ´ ´ ´x = 30 2

18. (d)185 35 ?400 240 1648100 100 100

´ + ´ = ´

Þ 740 + 84 = 1648 × ?

100 Þ 824 = 1648 × ?

100

Þ824 100?

1648´

= or ? = 50

19. (b)2 50 25? 6303 100 100

= ´ ´ ´

Þ210 50 25 2?

100 100´ ´ ´

Þ 210?4

= = 52.5

20. (b) Number of transferred employees

= 40% of 1225 = 1225 40 490

100´

=

21. (b) Let the original fraction be xy

.

Then, x 5x 42y 3y 7

+=

+ Þ

6x 184y 7

= Þ x 72 12y 42 7

= =

22. (c) Let the original fraction be = xy

According to the question,350

7100400 9100

x

y

´=

´ Þ

7 7 7 8 88 9 9 7 9

x xy y

= = ´ =Þ

23. (e) Let the maximum aggregate marks = xAccording to the question,40% of x – 4% of x = 261

or x × (40 4)

100-

= 261

\ x = 26136 × 100 = 725

24. (b) Population of the town after 2 years

= 198000 7 51 1

100 100æ öæ ö+ -ç ÷ç ÷è øè ø

= 198000 107 95

100 100´ ´´ = 201267

25. (b) Total expenditure = 44668 + 56732 = 101400Total percentage expenditure= 100 – 22 = 78 %

\ Total amount = ´101400 100

78 = 130000

26. (b) 75 % of 16 = 12\ 12% of monthly salary = 6567

\ 100% = 6567 100

12´

= 54725

27. (d) Total valid votes = 85% of 15200 = 12920\ Number of valid votes to other candidate= 45% of 12920 = 5814

28. (c) If the radius is diminised by r%, then

Area is diminished by 2r2r %

100æ ö

-ç ÷ç ÷è ø

2102 10 19%

100= ´ - =

29. (b) Let B’s salary be 100,then A’s salary = ` 125

% lesser = 125 100 25100 100

125 125-

´ = ´

= %2010051

30. (b) Let the original price of apple be ` x /dozen

New price ` = 45x

/dozen = 54 54 104 125x x

- =

Þ5 1 554

4 6x xæ ö- =ç ÷è ø

Þ 1 5544 6x

æ ö =ç ÷è ø

Þ 4x = 54 6

Þ 4 12.965x

=

SPEED TEST 36

1. (d) Total number of students in the school = 819Number of girls = 364\ Number of boys = 819 - 364 = 455\ Required ratio = 435 : 364 = 5 : 4

2. (d) Share of Urmila in dividend = 2 578346

æ ö´ç ÷è ø

= 19278

Page 309: yoursmahboob.wordpress.com SBI · yoursmahboob.wordpress.com iii P 101 Speed Tests for SBI Bank Clerk Exam 101 Speed Tests for SBI Bank Clerk Exam is revised and updated edition on

yoursmahboob.w

ordpress.com

43SOLUTIONS3. (b) Let the present ages of Richa and Shelly be

5x and 8x years.According to the question,

After 10 years, 5 10 78 10 10

xx

+=

+or, 56x + 70 = 50x + 100 or, 56x – 50x = 100 – 70or, 6x = 30

\ x = 306 = 5

\ Shelly's present age = 8x = 8 × 5 = 40 years4. (d) The sum of money is not known.5. (a) Let the age of woman be 2x years and that of her

daughter be x years.According to the question,

2x + x = 2 × 42 or, 3x = 84 or, x = 843 = 28

\ Daughter's age = 28 years6. (e) Let Arun’s present age be x years.

Then, Deepak’s present age = (x + 14) years

Then, x 7 5

x 14 7 7-

=+ -

7x 5x 35 49Þ - = + 84

x 422

Þ = =

\ Deepak’s present age = 42 + 14 = 56 years7. (d) Data is given in ratio. So age can’t be determined.8. (a) Let number of students in Arts and Commerce were 4x

and 5x respectively. Then,4 8

5 65 11=

+x

x Þ 44x – 40x = 520 Þ x = 520 1304

=

\ Number of students in Arts = 4 × 130 = 5209. (c) Let the original number of boys and girls be 6x and 5x

respectively.

Then, 6 8 115 2 7

xx

+=

-

55 42 56 22x xÞ - = + 78 613

xÞ = =

\ Number of boys = 6 × 6 + 8 = 4410. (a) Present age of Meena

= 8 8 (10 3)

24 10´ ´ -

- = 8 8 7

14´ ´

= 32 years

11. (b) Let the present age of father and son be 17x and 7xrespectively.

Then, 17x – 6 37x – 6 1

=

Þ 21x – 17x = 18 – 6 Þ x = 12 ÷ 4 =3\ Father’s present age = 17 × 3 = 51 years.

12. (c) Rita : Sita : Kavita7 : 15

7 : 1649 : 105 : 240The ratio of money with Rita, Sita and Kavita is49 : 105 : 240We see that 49 º ` 490 \ 240 º ` 2400

13. (a) Iron CopperAlloy I 8 : 6 14 kg.Alloy II 36 : 6 42 kg.

44 : 12 56 kg

14. (b) Ratio of sides = 1 1 1: : 6 : 4 : 32 3 4

=

Largest side = 6104 cm

13æ ö´ç ÷è ø

= 48 cm

15. (d) Let the numbers be x and y.

\ x 4y 7

=

\ 7x = 4y ... (a)

x 30 5y 30 8

+=

+\ 8x – 5y = – 90 ...(b)From eqn (b), 32x – 20y = – 360From eqn (a), 35x = 20y\ 32x – 35x = – 360

\ x = 360 120

3=

y = 210

\ Avg = 330 165

2=

16. (d) Given the ratio = 1 2 3: : 6 :8 : 9.2 3 4

=

\ 1st part = 678223

æ ö´ç ÷è ø

= 204.

17. (b) Let actual distance be x km. Then,3 :1 :: 60 : x4

Þ 3 x 604

= Þ240x 80 km3

= =

18. (d) Let A = 2x, B = 3x and C = 4x. Then,A 2x 2 B 3x 3 C 4x 2, and B 3x 3 C 4x 4 A 2x 1

= = = = = =

A B C 2 3 2: : : : 8 : 9 : 24.B C A 3 4 1

Þ = =

19. (c) Total age of 3 boys = (25 × 3) years = 75 yearsRatio of their ages = 3 : 5 : 7.

Age of the youngest boy = 37515

æ ö´ç ÷è ø

years = 15 years

20. (b) Let income of A = 3x, income of B = ` 2xand expenditure of A = 5y, expenditure of B = ` 3yNow, saving = income – expenditure\ 3x – 5y = 2x – 3y = 200Þ x = 2y and y = 200\ x = 400\ A’s income = 1200

21. (c) Since, A : B = 2 : 3 and B : C = 6 : 5\ A : B : C = 4 : 6 : 5

Then, A’s share 4 750 2004 6 5

= ´ =+ +

`

Page 310: yoursmahboob.wordpress.com SBI · yoursmahboob.wordpress.com iii P 101 Speed Tests for SBI Bank Clerk Exam 101 Speed Tests for SBI Bank Clerk Exam is revised and updated edition on

yoursmahboob.w

ordpress.com

44 101 SPEED TEST22. (c) Let the numbers be 5x and 4x.

Now, difference of numbers = 10i.e. 5x – 4x = 10 Þ x = 10\ Larger number = 10 × 5 = 50

23. (b) Give, Ratio of distribution = 3 : 1 : 5Let the share of P, Q and R be 3x, x and 5x, respectivelySince, Difference between Q’s and R’s share is Rs. 3600.

\ 5x – x = 3600Þ x = 900\ P’s share = 3 × 900 = 2700

and Q’s share = 900Hence, Total of P’s and Q’s share = Rs 3600

24. (b) Mrs. X spends = Rs 535\ Total cost = 43 shirt + 21 ties = 535

By hit and trial, S = 10, T = 5Þ Total cost = 43 × 10 + 21 × 5 = 535Hence, Ratio of shirts to ties = 10 : 5 = 2 : 1

25. (a) Given 52

ba

=

Consider 5

ba7

3ba2

b5a7b3a2

+

+=

++ =

22 3

52

7 55

´ +=

´ +

(By putting value of ab

)

19195

39 395

= =

26. (c) Let the numbers be 6x and 13x.LCM (6x, 13x) = 13 × 6 × x = 312 (given)

Þ 613

312x´

= Þ x = 4

So, the numbers are 24 and 52.\ Sum of the numbers = 24 + 52 = 76

27. (d) We have, 523

b3a2b3a5

=-+

Þ 25a + 15 b = 46a – 69b

Þ 21a = 84 b Þ 14

2184

ba

==

28. (b) Let the no. of one rupee, 50 paise and 25 paise coins be2x, 3x and 4x respectively.

According to question,3x 4x

Rs. 2x Rs. 2162 4

æ ö+ + =ç ÷è ø

8x 6x 4x 2164

+ +Þ =

\ x = 48\ Number of 50 paise coins = 48 × 3 = 144

29. (c) Let the ages of two persons be X and Y respectivelyX 5 X –16 3andY 7 Y –16 5

\ = =

On solving we get X = 40, Y = 56.

30. (b) In the mixture, milk 5 28 207

= ´ = litres, and

Water 2 28 87

= ´ = litres

2 ltrs of water is added, Hence the new ratio ofMilk 20 20 2 :1Water 8 2 10

= = =+

SPEED TEST 37

1. (a) New % of sugar in (3 + 1) litre solution0.04 3 0.03 3%(3 1)

´= = =

+

2. (c) C.P. of mixture 80 15 20 20 1680 20

´ + ´= =

+`

\ S.P. (100 25) 16 20

100+

= ´ =`

3. (b) By the rule of alligation, we haveCost of Alcohol Cost of Kerosene Oil

Rs 3.50 ` 2.50

` 2.75

2.75 – 2.50 = 0.25

cost of mixture

3.50 – 2.75 = 0.75

\ Required ratio0.25 1 i.e. 1: 30.75 3

= =

4. (b) In mixture,Quantity of pure milk 3 0 3 5

Quantity of water 3.6 3 0.6 1-

= = =-

Since in every 5 litres of milk, he adds 1 litre of water.\ In every 25 litres of milk, he adds 5 litres of water.

5. (b) Let C.P. of milk be Re. 1 per litre.Then, S.P. of 1 litre of mixture = Re. 1.Gain = 20%

\ C.P. of 1 litre of mixture = 100 5

1 .120 6

æ ö´ =ç ÷è ø`

By the rule of alligation, we have :

Meanprice

0 Re. 1

Re.

C.P. of 1 litreof water

C.P. of 1 litreof milk

5 11–6 6

= 5 5– 06 6

=

56

\ Ratio of water and milk = 1 5: 1: 5.6 6

=

Page 311: yoursmahboob.wordpress.com SBI · yoursmahboob.wordpress.com iii P 101 Speed Tests for SBI Bank Clerk Exam 101 Speed Tests for SBI Bank Clerk Exam is revised and updated edition on

yoursmahboob.w

ordpress.com

45SOLUTIONS6. (a) Out of 10 litre of solution, there is 1 litres of nitric acid

and 9 litres of water.Let x litres of water be added to the solution so that thediluted solution is of 4% strength.\ 4% of (10 + x) = 1 Þ x = 15.

7. (a) Let C.P. of 1 litre milk be Re. 1.

S.P. of 1 litre of mixture = Re. 1, Gain = 50 %.3

\ C.P. of 1 litre of mixture = 6Re. .7

By the rule of alligation, we have :

Meanprice

0 Re. 1

Re.

C.P. of 1 litreof water

C.P. of 1 litreof milk

67

67

17

\ Ratio of water and milk = 1 6:7 7

= 1 : 6.

8. (b) The first alloy does not have tin. Therefore, quantity of

tin in 2 units of the resulting alloy = 135

Þ Quantity of tin in one unit of the resulting alloy

= 265

2135

=

9. (b) By the rule of alligation,water concentration,

Original solution Water

(mixture)

411

713

14143

1

613

\ water must be added to the mixture in the ratio14 7:143 13 i.e. 2 : 11

Quantity of water to be added 2 55 10 litres

11= ´ =

10. (c) Gold CopperType A 14 4Type B 7 11Type C 21 15\ The ratio of the Gold and Copper in the type Calloy = 7 : 5.

11. (c) C. P. of mixture of 18 kg = 10 × 45 + 8 × 50 = ` 850\ S. P. = C. P. + Profit = 850 + 32 = ` 882\ S. P. = ` 882 for 18 kg

\ S. P. for 1 kg. 882Rs18

= =` 49

12. (d) By the rule of alligation,Alcohol concentration :

Original mixture15%

Pure alcohol100%

25%

75% 10%

\ Alcohol must be added in the ratio of 10 : 75 or 2 : 15\ Quantity of alcohol to be added in 10 litres

2 41015 3

= ´ =

13. (b) C. P. of mixture 100 6048

(100 25)´

= =+

`

Let x kg be mixed. Then,

40 x 55 164816 x

´ + ´=

+

8x 16[55 48] x 14 kgÞ = - Þ =14. (c) The existing solution has 40% sugar. And sugar is to be mixed;

so the other solution has 100% sugar. So, by alligation method:

50% 10%50%

40% 100%

\ The two mixtures should be added in the ratio 5 : 1.

Therefore, required sugar = 300 1

5´ = 60 gm

15. (b)

6 415

19 9Gold Copper

\ Gold : Copper = 6 : 4 = 3 : 216. (c) Apply the alligation on fracfion of milk in each mixture.

0Mixture Water

13

23

13

13

Ratio of mixture to water = 1 : 1Therefore, if there is 60 liture of solution, 60 litres of watershould be added.

17. (e) Initially water (weight) = 45 gm & milk 15 gm. After added15 gmwater the percentage of water

= weight of water

total weight of mixture = 60 10075

´ = 80%

Page 312: yoursmahboob.wordpress.com SBI · yoursmahboob.wordpress.com iii P 101 Speed Tests for SBI Bank Clerk Exam 101 Speed Tests for SBI Bank Clerk Exam is revised and updated edition on

yoursmahboob.w

ordpress.com

46 101 SPEED TEST18. (a) By the rule of alligation:

Cost of 1 kg tea of 1st kind

\ Required ratio = 750 : 250 = 3 : 1.19. (d) Let the C.P. of spirit be Re. 1 per litre.

Spirit in 1 litre mix. of A =57

litre; C.p. of a litre mix. in A = Re.57 .

Spirit in 1 litre mix. of B = 7

13 litre;

C.P. of 1 litre mix. in B = Re.5 .

13

Spirit in 1 litre mix. of 8C=

13litre; Mean price = Re.

8 .13

By the rule of alligation, we have :

57

æ öç ÷è ø

113

æ öç ÷è ø

813

æ öç ÷è ø

713

æ öç ÷è ø

991

Mean price

C.P. of 1 litre mixture in A C.P. of 1 litre mixture in B

\ Required ratio 1 9: 7 : 9.

13 13= =

20. (c) Let the quantity of milk and water be 40 litres and 60litres, respectively.

(Q Ratio of milk to water = 2 : 3)After removing 50% of solutionQuantity of milk = 20 litres andQuantity of water = 30 litresTherefore, the concentration of the solution is reducedfrom 40 to 20 i.e. a reduction of 50%.

21. (b) Quantity of milk = litres274553

Quantity of water = litres184552

Let x litres of water be added to make the ratio 9 : 11.

\ 911

27x18

=+ Þ 18 + x = 33 Þ x =15l

22. (c) Ratio of milk in the containers are,1 3 5 5 3 255 : 4 : 5 : :6 8 12 6 2 12

´ ´ ´ =

and the ratio of water in the containers are,5 5 7 25 5 355 : 4 : 5 : :6 8 12 6 2 12

´ ´ ´ =

Ratio of mixture of milk and water in the containers

= ÷øö

çèæ ´+´+´÷

øö

çèæ ´+´+´ 5

1274

855

65:5

1254

835

61

= 106 : 230 = 53 : 115

23. (c) Let the quantity of pure milk be x litres.If 5 litres of water is added to it, then the cost of (5 + x)litres = Rs. 3xS.P. of ( 5 + x) litres = Rs. (3x + 15)

\ Profit = Rs. 15Given 20% of 3x = 15

Þ 3x 155

= Þ x = 25

\ The amount of pure milk in the mixture is 25 litre24. (c) Let the weight of tea worth Rs 25 per kg = x kg.

According to question

\ 110% of x 25 30 30 30

x 30´ + ´é ù =ê ú+ë û

Þ 110 25x 900 30100 x 30

+é ù =ê ú+ë ûÞ 11 (25x + 900) = 300 (x+30)Þ 275x + 9900 = 300 x + 9000 or 25x = 900\ x = 36 kg.

25. (b) Let, weight of sugar costing Rs 5.75 per kg = x kgx´ 5.75 + 75´ 4.50 = 5.50´ (x + 75)Þ 5.75x + 337.50 = 5.50x + 412.50Þ 0.25x = 75\ x = 300 kg

SPEED TEST 38

1. (b) Let S.P. = ` 100. Then, C.P. = 96; Profit = 4.

\ Profit % 4 25100 % % 4.17%.

96 6æ ö= ´ = =ç ÷è ø

4.2%»

2. (a) Let the cost of production of the table be ` x.Then, 125% of 115% of 110% of x = 1265

125 115 110 x 1265100 100 100

Þ ´ ´ ´ =

253 1265 160x 1265 x160 253

´æ öÞ = Þ = =ç ÷è ø

800

3. (b) Let C. P. = ` x then profit = S.P. – C. P.1 11xx 891 – x 891

10 10Þ ´ = Þ =

891 10x 81011´

Þ = =`

4. (d) C. P. for 50 pencils = 100

\ C. P. for 45 pencils 100 45 9050

= ´ = `

= S.P. of 45 pencils\ No gain , no loss

5. (b) (100 – loss) : S1 : : (100 + gain) : S2\ (100 – 12.5) : 420 : : (100 + 12.5) : S287.5 : 420 : : 112.5 : S2Þ 87.5 × S2 = 420 × 112.5

Þ 2420 1125S 540

875´

= =

Page 313: yoursmahboob.wordpress.com SBI · yoursmahboob.wordpress.com iii P 101 Speed Tests for SBI Bank Clerk Exam 101 Speed Tests for SBI Bank Clerk Exam is revised and updated edition on

yoursmahboob.w

ordpress.com

47SOLUTIONS

6. (b) S.P. for 1 egg = ` 5

10 1Rs2

=

\ C. P. for 1 egg100 1 5

(100 20) 2 12= ´ =

+`

Þ He bought 12 eggs for 5 rupees.

7. (b) C.P. for one coconut 150 3Rs Rs100 2

= =

S.P. for one coconut = ` 2

Profit on one coconut 3 122 2

= - = `

\ Profit on 2000 coconut 1 2000 10002

= ´ = `

8. (a) Let C.P. = Rs 100, then M. P. = ` 150S.P. = 70% of 150 = 105

\ % profit 105 100 100 5%100

-= ´ =

9. (b) C.P. of one litre = 6After adding water to it

One has to pay Rs 7.20 for 23

litre of milk.

So S.P. of 23

litre of milk = Rs 7.20

Þ S.P. of 1 litre of milk 7.20 3Rs 10.80

= =`

S.P. C.P.>Q

Hence gain 10.80 6 4.80

100 1006 6

-= ´ = ´

= 0.80 × 100 = 80%10. (a) He gives 800 grams but charges the price of 1000 grams

(1 kg)Þ on every 800 grams, he gains (1000 – 800) grams i.e.200 grams.

\ His gain % 200 100% 25%800

= ´ =

Short cut : Gain % error

true weight–error=

200

1001000 200

= ´-

= 25%.

11. (d) Here, SP1 = SP2

Þ 140 CP1 = 60CP2 Þ 1

2

CP 6 3CP 14 7

= =

13CP 8000 2400

(3 7)\ = ´ =

+`

and CP2 = 8000 – 2400 = ` 560012. (b) Let the C.P. of the goods be ` 100

Þ Marked price of the goods = ` 120Discount = 10% Þ S.P. is 90% of Rs 120 = ` 108\ Gain% = (108 – 100) = 8%.

13. (d) Let his loss = ` x. Then,C.P. = 5000 + x = 5600 – 2xÞ 3x = 600 Þ x = 200\ C.P. =5000 + 200 = Rs 5200

14. (d) Let C.P. = ` x. Then,

S.P. (100 7) 93x x

100 100-

= ´ =

Also, 93 100x 48 x

100 (100 5)æ ö+ =ç ÷ +è ø

Þ 93x + 4800 = 105xÞ 12x = 4800 Þ x = ` 400

15. (e) CP of DVD player = ` xAccording to the question,

Q x221

100æ ö+ç ÷è ø

= 10980

\ x = 10980 × 5061 = 9000

16. (c) 24 bats + 32 sticks = 5600\ 8(3 bats + 4 sticks) = 5600

Þ 3 bats + 4 sticks = 5600

8 = 700

17. (a) Cost price = +=

1754 14922

1623

18. (c) Required amount = ´ ´

´10500 100 100

120 140 = ` 6250

19. (e) 15 pendants + 24 chains= 3 (5 pendants + 8 chains)= 3 × 145785 = 437355

20. (c) CP = Rs 153, desired gain 20%Þ SP = 153 × 1.2 = Rs 183.60Let the marked price be Rs xThen, 60.18385.0x =´

Þ 216.Rs85.060.183x ==

21. (d) Applying successive discounts of 10%, 12% and 15%on 100, we get 32.6785.088.09.0100 =´´´Þ Single discount = 100 – 67.32=32.68Hence, none of the given options is correct.

22. (b) Let the cost price of the machine be Rs x.

Then, selling price at a profit of 10% = Rs 11x10

And the selling price at a loss of 10% = Rs 9x10

Consequently, we find that11x 9x10 10

æ ö- =ç ÷è ø

805

Þ = Þ =x

x Rs 400

23. (b) Let CP be Rs x and SP be Rs y.Then, profit = Rs (y – x)If SP = 2y, then profit = 3(y – x)Now, )xy(3xy2 -=-Þ y = 2x

Now, profit = 2100 100- -

´ = ´y x x x

x x = 100%

Page 314: yoursmahboob.wordpress.com SBI · yoursmahboob.wordpress.com iii P 101 Speed Tests for SBI Bank Clerk Exam 101 Speed Tests for SBI Bank Clerk Exam is revised and updated edition on

yoursmahboob.w

ordpress.com

48 101 SPEED TEST24. (c) Let CP of 12 pencils = SP of 10 pencils = Re 1

Therefore, CP of 1 pencil = Rs 121 and

SP of 1 pencil = Rs 101

Profit on one pencil = 120

2121

101

=- = Rs 601

% profit = 1/ 60 100 20%1/12

´ =

25. (d) If any two transactions of SP is the same and also gain% and loss % are same then there is always a loss

\ loss % = 2Common gain or loss%

10æ öç ÷è ø

21010

æ ö= ç ÷è ø

= 1%

26. (a) Let the cost price of geyser be Rs x. Thenx × 1.1 × 1.15 × 1.25 = Rs 1265

Þ 800.Rs58125.11265x ==

27. (a) Money paid to buy watch = Rs. 1950.Money paid as interest of 10% on Rs. 1950 = Rs. 195\Total money paid = Rs. 2145Since S.P. = Rs. 2200, therefore the man gained Rs. 55.

28. (c) Let the cost price be Rs 100.\ Marked price is Rs 135.At 10% discount, the customer has to pay= Marked price – discount = 135 – 13.5 = 121.5.

\ % profit = 21.5% = 1

21 %.2

29. (a) S.P. of the 1st chair = Rs. 500Gain = 20%

\ C.P. of the 1st chair = 500 100 500 100100 20 120

´ ´=

+

1250

3=

S.P. of the 2nd chair = Rs. 500Loss = 12%

\ C.P. of the 2nd chair = 500 100 500 100100 12 88

´ ´=

-

6250

11=

Now S.P. of both the chairs = Rs. 1000C.P. of both the chairs

1250 6250 13750 187503 11 33

+= + =

3250033

=

\ Net gain = 1000 – 32500 500

33 33=

Þ Gain % = 500 33 100

32500 33´ =

500 10032500

´

=100 20 1.5%65 13

= = (To one place of decimal)

30. (b) Let x be the selling price of 1 notebook.\ Selling price of 2 note book = 2x = profit

As, we know, profit = selling price – cost price\ Cost price of 12 note book = 12x – 2x = 10 x

\ Profit ( percentage) = 2x 100 20%

10x´ =

SPEED TEST 39

1. (b) Simple Interest = P× R ×T

10031400 8 12 30144

100´ ´

=`

\ Required amount = (31400 + 30144)= 61544

2. (e) Rate = (26350 21250) 100

21250 6- ´

´

= 510000127500 = 4%

3. (c) Let the required time = t yearsSimple interest = (11442 – 9535) = 1907

Simple = P × T × R

100

9535 41907100

t´ ´=

\ t = 1907 100

9535 4´´ = 5 years

4. (b) Let the principal be = 100\ Simple interest

= 100 8 6

100´ ´

= 48

\ Amount (100 + 48) = 148\ When the amount is = 148, the principal = 100\ When amount = 28046, the principal

=10048 × 28046 = 18950

\ Simple interest = (`28046 – 18950) = 9096

5. (c) Simple interest = P×R×T100

= `56500 × 3 × 12 = 20340

100\ Required amount = (56500 + 20340) = 76840

6. (d) Amount invested = 8376 1008 6

´´

= 17450

7. (d) 2000 = P 4 5

100´ ´

\ P = 10000

Now, CI = 2410000 1 1

100

é ùæ öê + - úç ÷è øê úë û

= 10000 × 0.0816 = 816

8. (a) Amount = 11200 + 1120 8.5 3 11200 2856

100´ ´

= + = 14056

9. (b) Ratio of two parts = r2 t2 : r1 t1 = 54 : 50 = 27 : 25

\ Sum lent out at 10% = 2600 27 135052

´ = `

Page 315: yoursmahboob.wordpress.com SBI · yoursmahboob.wordpress.com iii P 101 Speed Tests for SBI Bank Clerk Exam 101 Speed Tests for SBI Bank Clerk Exam is revised and updated edition on

yoursmahboob.w

ordpress.com

49SOLUTIONS10. (d) It dobles in 10 yrs.

Then trebles in 20 yrs.11. (b) Suppose the rate of interest = r% and the sum = ` A

Now, A 4A 600100

r+ ´+ = ;

or, ArA 60025

+ =

or, A 1 60025ré ù+ =ê úë û

...(1)

And, A 6A 650100

r´ ´+ = ;

or, 3A 1 65050

ré ù+ =ê úë û...(2)

Dividing (1) by (2), we have

1+ 600253 650150

r

r=

+; or,

(25 ) 2 1250 3 13

rr

+ ´=

+

or,(25 ) 2 12

50 3 13r

r+ ´

or, (50 + 2r) × 13 = (50 + 3r) × 12or, 650 + 26r = 600 + 36r; or, 10r = 50\ r = 5%

12. (a) Let the sum be ` x.

\ Interest = 8 4 32

100 100x x´ ´

=

32 68100 100

x xx - =

When interest is 68100

x less, the sum is ` x.

\ when interest is 340 less, the sum is 100 340 50068

xx

´ ´ = `

13. (b) S. I. = 81 – 72 = 9

\ 9 100 4T 2 years72 25´ ´

= =´

14. (a) S.I. for 1½ years = ` (1164 – 1008) = 156

S. I. for 2 years = ` 156 2 2 Rs 208

3´ ´æ ö =ç ÷

è ø\ Principal = (1008 – 208) = ` 800Now , P = 800, T= 2, S.I. = 208

\ Rate 100 208 % 13%

800 2´æ ö= =ç ÷´è ø

15. (b) S.I. for 5 years = ` (1020 –720) = 300

SI. for 2 years = 300Rs 2 Rs 120

5´ =

\ Principal = (720 – 120) = ` 600Now, P = 600, T = 2, S.I. = 120

\ 120 100R 10%

600 2´

= =´

16. (b) Let S.I. = x 51.53 10 20

100´ ´

= = 2500.

17. (c) Difference in S.I. 1 2P T (R R )100

´= -

Þ P 4 256

100´ ´

= (Q R1 – R2 = 2)

Þ 56 100P 700

4 2´

= =´

`

18. (a) Q Rate = 5 paise per rupee = 5%200 5 7S.I.

100´ ´

\ = = ` 70

19. (c) Let one gets = ` xthen, second gets = ` (68,000 – x)Given : A1 = A2

x 10 8 (68000 x) 10 6x (68,000 x)100 100

´ ´ - ´ ´+ = - +

x[100 80] (68,000 x)[100 60]Þ + = - +

180x 68,000 – x160

Þ =

34x 68000 16 x Rs32,000Þ = ´ Þ =\ second gets = ` 36,000

20. (b) Interest for one year 3 51212.50 1100 8

= ´ ´ =` `

Thus in 8 years, the interest is 51.21. (c) Shortcut method :

If borrowed amount be ` M and it is to be paid in equalinstalments, then

ra n(n 1)M na100 Y 2

-= + ´

´where Y = no. of instalments per annum a = annual instalmentHere, M = 4200, y = 1, r = 10, n = 5, a = ?

10a 5(5 1)4200 5a100 2

-= + ´

Þ [ ]4200 a 5 1 6a 4200= + Þ =Þ a = ` 700

22. (d) We need to know the S.I., principal and time to find therate. Since the principal is not given, so data isinadequate.

23. (c) Let the principal be P and rate of interest be R%.

\ Required ratio =

P R 66PR 6100 2 : 3.

P R 9 9PR 9100

é ´ ´ ùæ öç ÷ê úè øê ú = = =

´ ´æ öê úç ÷ê úè øë û

24. (a) Let the sum be x. Then,x 6 3 x 9 5 x 13 3 8160

100 100 100´ ´ ´ ´ ´ ´æ ö æ ö æ ö+ + =ç ÷ ç ÷ ç ÷

è ø è ø è øÞ 18 x + 45x + 39x = (8160 × 100) Þ102x = 816000Þx = 8000.

25. (b) Let the sum be 100. Then,

S.I. for first 6 months = 100 10 1

Rs.100 2

´ ´æ ö =ç ÷è ø´` `

Page 316: yoursmahboob.wordpress.com SBI · yoursmahboob.wordpress.com iii P 101 Speed Tests for SBI Bank Clerk Exam 101 Speed Tests for SBI Bank Clerk Exam is revised and updated edition on

yoursmahboob.w

ordpress.com

50 101 SPEED TEST

S.I. for last 6 months = 105 10 1 5.25.

100 2´ ´æ ö =ç ÷è ø´

` `

So, amount at the end of 1 year = ` (100 + 5 + 5.25)= 110.25.\ Effective rate = (110.25 – 100) = 10.25%.

26. (d) Let the rate be R% p.a. Then,5000 R 2 3000 R 4 2200

100 100´ ´ ´ ´æ ö æ ö+ =ç ÷ ç ÷

è ø è ø

2200100R 120R 2200 R 10.220

æ öÞ + = Þ = =ç ÷è ø

27. (b) Let the savings be X and Y and the rates of simpleinterest be 5x and 4x respectively.

Then, 1 1 1 1 X 4X 5x Y 4x or ,2 100 2 100 Y 5

´ ´ ´ = ´ ´ ´ =

i.e., X : Y = 4 : 5.28. (a) Let the principal be P, then amount after 12 years = 2P

Þ P)PP2(SI =-=

Now, 100

P r tI

´ ´= Þ P = 12

100P r´ ´

or 100 25 1r 8 %12 3 3

= = =

29. (b) Let the sum be Rs x

Now, 8 (3 2)56100

x ´ ´ -= Þ x = Rs 700

30. (a) 10,000 6 9SI Rs 45012 100

´ ´= =

´

SPEED TEST 40

1. (a) Required Amount = 15000251

100æ ö+ç ÷è ø

= 16537.50

2. (b) Amount = Principal TimeRate1

100æ ö+ç ÷è ø

281250 1 1100

108 108 100 1001250100 100

é ùæ ö= + -ê úç ÷è øê úë ûæ ö´ - ´

= ´ç ÷´è ø

1250 1664 20810000

´= = `

3. (e) Amount = Principal = TimeRate1

100æ ö+ç ÷è ø

= 5690351

100æ ö+ç ÷è ø

= 5690 × 21 21 2120 20 20

´ ´ = 6586.90

\ Compound interest = (6586.9 – 5690) = 896.9 » 897

4. (e) Rate30240 100 12%84000 3

´= =

´Compound interest

= 840003121

100æ ö+ç ÷è ø

– 84000

= 118013.95 – 84000 = ` 34013.95

5. (a) Simple interest = 4000 5 2

100´ ´

= 400Compound interest

= 254000 1 4000

100æ ö+ -ç ÷è ø

= 4000 105 105 4000

100 100´ ´

= 4410 – 4000 = 410

\ Difference = 410 – 400 = 10

6. (a) Compound interest =2535000 1

100æ ö+ç ÷è ø

– 35000

= 38587.50 – 35000 = 3587.50

7. (d) Principle = 8730 100 48500

6 3´

Compound Interest 2648500 1 1

100

é ùæ ö= ê + - úç ÷è øê úë û

= 48500 × 0.1236 = ` 5994.60

8. (a) 594.5 = 2r5800 1 1

100

é ùæ öê + - úç ÷è øê úë û

= 2594.5 r1 1

5800 100æ ö= + -ç ÷è ø

0.1025 + 1 = 2r1

100æ ö+ç ÷è ø

( )2100 r1.1025

10000+

=

1.1025 × 10000 = (100 + r)2.11025 = (100 + r)2.105 = 100 + rr = 5%

9. (d) Compound interest = 2277400 1 1

200

é ùæ öê + - úç ÷è øê úë û

= 22277400 1

200

é ùæ öê - úç ÷è øê úë û

= 227 227 200 200

7400200 200

´ - ´é ùê ú´ë û

= 51529 40000

740040000

-é ùê úë û

= 11529740040000

´

= 2132.865 = ` 2132.87

10. (c)TrC. I. 1 1

100

é ùæ öê + - úç ÷è øê úë û

4676.25 = 145002r1 1

100

é ùæ öê + - úç ÷è øê úë û

Þ 24676.25 r1 1

14500 100æ ö= + -ç ÷è ø

Þ 24676.25 r1 1

14500 100æ ö+ = +ç ÷è ø

Þ 24676.25 14500 r1

14500 100´ æ ö= +ç ÷

è ø

Þ 19176.25 r1

14500 100= + Þ

r1.3225 1100

= +

Þ 13225 r110000 100

= + Þ 115 r1100 100

= + Þ r 115 1

100 100= -

Þ r 115 100

100 100-

= Þ r 115

100 100= Þ r = 15%

Page 317: yoursmahboob.wordpress.com SBI · yoursmahboob.wordpress.com iii P 101 Speed Tests for SBI Bank Clerk Exam 101 Speed Tests for SBI Bank Clerk Exam is revised and updated edition on

yoursmahboob.w

ordpress.com

51SOLUTIONS11. (c) Compound interest accrued half-yearly.

R = 20% yearly = 10% half-yearlyn = 2 years = 4 half-yearly

nrCI P 1 1100

é ùæ ö= ê + - úç ÷è øê úë û

= 41010000 1 1

100

é ùæ öê + - úç ÷è øê úë û

= 41110000 1

10

é ùæ öê - úç ÷è øê úë û

= 11 11 11 11 10 10 10 1010000

10 10 10 10´ ´ ´ - ´ ´ ´é ù

ê ú´ ´ ´ë û

= 14641 1000

1000010000

-é ùê úë û

= 4641

1000010000

é ùê úë û

= ` 4641

12. (e) Required difference = 2RP

100æ öç ÷è ø

= 214985

100æ öç ÷è ø

= 196985 19.30610000

´ =

13. (d) Difference in amounts = 2977.54 – 2809 = ` 168.54Now, we see that ` 168.54 is the interest on ` 2809 in oneyear (it is either simple or compound interest because bothare the same for a year).

Hence, rate of interest = 168.54 100 6%

2809´

=

Now, for the original sum,

2809 = 261

100x æ ö+ç ÷

è ø or,

253280950

x æ ö= ç ÷è ø

\2809 50 50 2500

53 53x ´ ´

= =´

`

14. (b) 26632.55 6250 1100

tæ ö= +ç ÷è ø

or, 6632.55 51625000 50

tæ ö= ç ÷è ø

or, 663255 51625000 50

tæ ö= ç ÷è ø

or, 132651 51 51125000 50 50

t tæ ö æ ö= =ç ÷ ç ÷è ø è ø

\ t = 3

Hence, the time is 3

2 2t

=

15. (a) Whenever the relationship between CI and SI is asked for 3years of time, we use the formula:

SI = CI

100 1 1100

rrt

é ùæ ö+ -ê úç ÷è øë û

35 3150 CI

100 1 1100

r

´= ´

é ùæ ö+ -ê úç ÷è øë û

9261 8000150 1008000CI =

5 3

-é ù´ ê úë û´

150 100 1261 1261 157.625 3 8000 8

´ ´= = =

´ ´`

16. (b) Let the sum of money be Rs x.

Now, 3

8 1100

rx x æ ö= +ç ÷è ø

or, 3

31 (2)100

ræ ö+ =ç ÷è ø or 1 2

100r

+ =

Again, let the sum becomes 16 times in n years.

Then, 16 1100

nrx x æ ö= +ç ÷è ø

Þ n216 = or n4 22 = or 4n =17. (d) Required difference

= 100

212500050001001215000

2 ´´-

úúû

ù

êêë

é-÷

øö

çèæ +

= 120012528

25285000 -÷

øö

çèæ -´

= 784 6255000 1200 Rs.72625-æ ö - =ç ÷

è ø18. (d) Let the money borrowed be Rs x and the rate of interest

charged = r%Time = 2 years

Now, 4000 = 100

2rx ´´

Þ r x = 200000 ............. (i)

Again, 2rx 1 x 4200

100æ ö+ = +ç ÷è ø

Þ 2 2 4200

10000 100xr xr

+ =

or 20r + 4000 = 4200 [from (i)]or r = 10%

19. (c)n

100r1PA ÷

øö

çèæ += ; A = Amount

P = Principal ; r = rate of interest ; n = time

Required amount 3

1001051000 ÷

øö

çèæ=

= 2021

2021

20211000 ´´´

= Rs 1157.62520. (b) Let the rate of interest be r %.

Therefore, 4

32520 (1 /100)2400

1100

r

r

+=

æ ö+ç ÷è ø

Þ 211

100 20r

+ = or r = 5%

Page 318: yoursmahboob.wordpress.com SBI · yoursmahboob.wordpress.com iii P 101 Speed Tests for SBI Bank Clerk Exam 101 Speed Tests for SBI Bank Clerk Exam is revised and updated edition on

yoursmahboob.w

ordpress.com

52 101 SPEED TEST21. (d) Cash Price (C.P.)= Rs. 39,000

Cash Down Payment (D.P.) = Rs. 17000Balance Due (B.D) = Rs. 22000P = Value of Instalment = Rs. 4800n = No. of Instalments = 5R = Rate of interest

Q nR (n 1)R1 BD 1 nP1200 2400

-æ ö ì ü+ = +í ýç ÷è ø î þ

\ 5R

1 220001200

æ ö+ç ÷è ø = 480052400

R41 ´þýü

îíì +

Þ 5R1 11

1200æ ö+ç ÷è ø

= 4R

1 122400

ì ü+í ýî þ

Þ 55R 24R

11 121200 1200

+ = +

Þ 55R 24R 31R

1 11200 1200 1200

- = Þ =

Þ R = 1200 38.71%

31=

22. (c) Let the amount of each instalment be Rs. x.

Amount of Rs. 100 for 3 years 35100 1

100æ ö= +ç ÷è ø

= 21 21 21

10020 20 20

´ ´ ´ = Rs. 926180

Present value of Rs. 926180

due after 3 years = Rs. 100

Present value of Rs. x due after 3 years

= 100 80

9261´

´ x = Rs. 8000x9261

Amount of Rs. 100 for 2 years = 25

100 1100

æ ö+ç ÷è ø

= 21 21 44110020 20 4

´ ´ =

Present value of Rs. x due after 2 years = Rs. 400 x441

Similarly, present value of Rs. x due after 1 year

= Rs.21

x20

\ 8000 400 20x x x 1261009261 441 21

+ + =

Þ 8000x + 8400x + 8820x = 126100 × 9261Þ 25220x = 126100 × 9261

Þ x = 126100 9261 Rs 46305

25220´

=

23. (b) Given 3r8P P 1

100æ ö= +ç ÷è ø

Where P = Principal amount,r = Compound interest rate

r 100%Þ =

\ let the time in which the principal amount becomes16 times be n

Then 16P n100P 1

100æ ö= +ç ÷è ø

Þ 16 n2 n 4yrs.= Þ =24. (a) Amount = Rs. 176040, Principal = Rs. 16000

Time = 2 yrs, Rate = R

17640 = 160002R1

100æ ö+ç ÷è ø

217640 R116000 100

æ öÞ = +ç ÷è ø

2R1.1025 1

100æ öÞ = +ç ÷è ø

R1 1.05100

Þ + = R 1.05 –1 0.05100

Þ = = Þ R = 5%

25. (a) Let A lent Rs. x and B lent Rs. ySince, A and B together lent out Rs. 81600\ x + y = 81,600Now, given (r) Rate = 4%

\ 4 261 r 1

100 25+ = + =

According to the question, we have3–2x 26 26

y 25 25æ ö= =ç ÷è ø

\ Investment made by B = 25

81600 40,00051

´ =

SPEED TEST 41

1. (b) Relative speed = 42 – 30 = 12 km/hr = 5 1012 m/s18 3

´ =

Time = Total length of both the trains 84 60

10Relative speed3

+=

= 144 3 43.2 seconds

10´

=

2. (a) Speed of the train = 120 24 m/s

5=

\ time taken by the train to pass the platform

= 120 180 12.5 seconds

24+

=

3. (c) Length of the train = Relative speed × time

= 5(40 25) 48

18æ ö- ´ç ÷è ø

= 15 5 48 200 m

18´ ´

=

4. (c) Let the length of the train = x m

Then, speed of the train = 100

15 25x x +

=

or, 25x = 15x + 1500 or, 10x = 1500\ x = 150 m

5. (c)Distance coveredSpeed of bus =

Time taken = 2924

43 = 68 kmph.

6. (a) Distance covered = Speed × Time = 49 × 7 = 343km

Page 319: yoursmahboob.wordpress.com SBI · yoursmahboob.wordpress.com iii P 101 Speed Tests for SBI Bank Clerk Exam 101 Speed Tests for SBI Bank Clerk Exam is revised and updated edition on

yoursmahboob.w

ordpress.com

53SOLUTIONS7. (e) Let the distance between the village and the school be

x km.According to the question,

64 2x x

+ = or,, 2 64

x x+= or,, 3x = 6 × 4

\6 4 8

3x ´

= = km

8. (a) Speed of train (200 400) 1836 5+

= ´ = 60 km/hr..

9. (c) Distance covered in 18 seconds

= 590 18 45018

m´ ´ =

\ length of platform = 450 – 160 = 290 m

10. (b) Stoppage minutes per hour = (64 48) 60

64- ´

= 15 minutes.

11. (c) Speed of bus = 48012 = 40 km/hr

Speed of train = 9405

´ = 72 km/hr

Speed of car = 72

1318

´ = 52 km/hr

Distance covered by car = 52 × 5 = 260 km

12. (a) Length of platform = 5126

18´ × 24 – 300 = 540 meter

\ Speed of man = 540

5 60´ = 1.8 meter/second

13. (b) Distance = 64 × 8 = 512 km

\ Speed = 512

6 = 85 km/hr (approx.)

14. (d) Net distance gained by car over the bus= 40 + 60 = 100m, in 20 sec.

Time =Distance

Relative speed

2

10020536 S

18

Þ =æ ö´ -ç ÷è ø

Þ S2 = 5 m/s = 18 kmph.15. (b) Relative speed of the trains

= (72 – 54) km/h = 18 km/h

= 518

18æ ö´ç ÷è ø

m/sec = 5 m/sec.

Time taken by the trains to cross each other= Time taken to cover (100 + 120) m at 5 m/sec

= 2205

æ öç ÷è ø

sec = 44 sec.

16. (a) Let the distance be x km. Let speed of train be y km/h.Then by question, we have

x x 30y 4 y 60

= -+ ...(i)

andx x 20

y 2 y 60= +

-...(ii)

On solving (i) and (ii), we get x = 3yPut x = 3y in (i) we get

3y 13

y 4 2= -

+ y 20Þ =

Hence, distance = 20 × 3 = 60 km.

17. (b) Rate downstream = 162

æ öç ÷è ø

kmph = 8 kmph;

Rate upstream = 164

æ öç ÷è ø

kmph = 4 kmph.

\ Speed in still water = 12

(8 + 4) = 6 km/h.18. (c) After 5 minutes (before meeting), the top runner covers

2 rounds i.e., 400 m and the last runner covers 1 roundi.e., 200 m.\ Top runner covers 800 m race in 10 minutes.

19. (a)Total distanceAverage speed =

Total time

400 4 9 400 4 9

88 96 89 87 360´ ´ ´ ´

= =+ + +

= 40 metres /minutes

20. (a) Speed = 750 m / sec 5 m / sec150

æ ö =ç ÷è ø

185 km / hr 18 km / hr.5

æ ö= ´ =ç ÷è ø

21. (a) Let each side of the square be x km and let the averagespeed of the plane around the field be y km/h. Then,

x x x x 4x200 400 600 800 y

+ + + =

25x 4x 2400 4y 384.2400 y 25

´æ öÞ = Þ = =ç ÷è ø

\ Average speed = 384 km/h.22. (d) Let after t hours they meet then,

3t + 4t = 17.5 Þ t = 2.5\ Time = 10 am + 2.5 h = 12 : 30pm

23. (a) Let original speed = S km/hHere, distance to be covered is constant

20S 8 (S 5)3

æ ö\ ´ = + ç ÷è ø

20 1008S S3 3

Þ - = 100S 25 km / h

4Þ = =

24. (c) Let the speed of the bus be x km / h.then speed of the car = (x + 25) km / h

\ 500 50010

x x 25= +

+Þ x2 + 25x – 1250 = 0 Þ x = 25Thus speed of the bus = 25 km/hSpeed of the car = 50 km/hAlternative:Difference in speeds 25 km / hr is in only option (c).

Page 320: yoursmahboob.wordpress.com SBI · yoursmahboob.wordpress.com iii P 101 Speed Tests for SBI Bank Clerk Exam 101 Speed Tests for SBI Bank Clerk Exam is revised and updated edition on

yoursmahboob.w

ordpress.com

54 101 SPEED TEST25. (a) Distance to be covered by the thief and by the owner is

same.Let after time 't', owner catches the thief.

140 t 50 t –2

æ ö\ ´ = ç ÷è ø

5 110t 25 t hr 2 hr2 2

Þ = Þ = =

26. (a) Let the duration of the flight be x hours. Then,

600 600 600 1200200 2001x x 2x 1x2

- = Þ - =++

Þ x (2x + 1) = 3 Þ 2x2 + x – 3 = 0 Þ (2x + 3) (x – 1) = 0Þ x = 1 hr. [neglecting the –ve value of x].

27. (d) Required difference = 180 180 15

3 4- = km

28. (c) Let the husband and the wife meet after x minutes. 4500metres are covered by Pradeep in 60 minutes.

In x minutes, he will cover x60

4500 metres.

Similarily,

In x minutes, his wife will cover x60

3750 m.

Now, 726x60

3750x60

4500=+

Þ min28.58250

60726x =´

=

29. (b) Downstream speed = 15 + 5 = 20 km/h.

\ Required distance 2420 8km.60

= ´ =

30. (c) Let the distance travelled during both upward anddownward journey be x km.

Average speed = taken timeTotalcovered distance Total

=

16281628

2

28x

16x

xx

´+

=+

+

2 28 16 20.36 km / h44

´ ´= =

SPEED TEST 42

1. (c) Number of days = 12 812 8

´-

= 24 days

2. (a) In an hour, George and Sonia together can copy1 1 76 8 24

+ = of a 50-page manuscript.

i.e. In an hour they together can copy 748

of the

100-page manuscript.i.e. They together can copy a 100-page manuscript in487

hours, i.e. 667

hours.

3. (a) A’s 1 day’s work = 1

18 and B’s 1 day’s work = 1 .9

\ (A + B)’s 1 day’s work = 1 1 1 .

18 9 6æ ö+ =ç ÷è ø

4. (c) Let C completes the work in x days.

Work done by (A + B) in 1 day = 101

Work done by (B +C) in 1 day = 181

A’s 5 days’ work + B’s 10 days’ work+ C’s 15 days’ work = 1or (A + B)’s 5 days’ work + (B + C)’s 5 days’ work + C’s 10 days’ work = 1

or 5 5 10 110 18 x

+ + =

\ x = 45 days

5. (b) In one min, (A + B) fill the cistern 1 1 1 th10 15 6

= + =

In 3 mins, (A + B) fill the cistern = 3 1 th6 2

=

Remaining part 1 112 2

= - =

Q 1 th

10 part is filled by A in one min.

\ 12

nd part is filled by A in 110 5min2

´ = .

\ Total time = 3 + 5 = 8 min.6. (b) Ratio of times taken by A and B = 100 : 130 = 10 : 13.

Suppose B takes x days to do the work.Then, 10 : 13 : : 23 : x

23 13 299x x .10 10´æ öÞ = Þ =ç ÷

è ø

A’s 1 day’s work = 1 ;23

B’s 1 day's work = 10 .299

(A + B)’s 1 day’s work = 1 10 23 1 .23 299 299 13

æ ö+ = =ç ÷è ø

\ A and B together can complete the job in 13 days.7. (a) Sunil takes 5 days and Pradeep takes 15 days to do the

work.

In a day they would complete 1 15 15

+ i.e., th4

15 work.

The remaining 1115

th work would be completed by

Pradeep in 11 1515

´ i.e. 11 days.

8. (a) Let 1 man’s 1 day’s work = x and1 boy’s 1 day’s work = y.

Then, 6x + 8y = 1

10 and 26x + 48y =

1 .2

Page 321: yoursmahboob.wordpress.com SBI · yoursmahboob.wordpress.com iii P 101 Speed Tests for SBI Bank Clerk Exam 101 Speed Tests for SBI Bank Clerk Exam is revised and updated edition on

yoursmahboob.w

ordpress.com

55SOLUTIONSSolving these two equations, we get :

1 1x and y .100 200

= =

\ (15 men + 20 boys)’s 1 day’s work

= 15 20 1 .

100 200 4æ ö+ =ç ÷è ø

\ 15 men and 20 boys can do the work in 4 days.

9. (a) 1 man’s 1 day’s work = 1 .

108

12 men’s 6 day’s work = 1 26 .9 3

æ ö´ =ç ÷è ø

Remaining work = 2 11 .3 3

æ ö- =ç ÷è ø

18 men’s 1 day’s work = 1 118 .

108 6æ ö´ =ç ÷è ø

16

work is done by them in 1 day..

\ 13

work is done by them in 16 2 days3

´ =

10. (c) A’s one day’s work = 1 th work

16

B’s one day’s work 1 th work

12=

Let the number of days B has worked alone = x days.Then,A’s amount of work + B’s amount of work = 1

1 14 (x 4) 116 12

æ ö æ öÞ + + =ç ÷ ç ÷è ø è ø

1 x 4 31 x 12 44 12 4

+Þ + = Þ = ´ - x 5 daysÞ =

11. (c) (A + B)'s 1 hour's work = 1 1 9 3

12 15 60 20æ ö+ = =ç ÷è ø

(A + C)'s 1 hour's work = 1 1 8 2

12 20 60 15æ ö+ = =ç ÷è ø

Part filled in 2 hrs = 3 2 17

20 15 60æ ö+ =ç ÷è ø

Part filled in 6 hrs = 17 17360 20

æ ö´ =ç ÷è ø

Remaining part = 17 3120 20

æ ö- =ç ÷è ø

Now, it is the turn of A and B and 320 part is filled by

A and B in 1 hour.\ Total time taken to fill the tank = (6 + 1) hrs = 7 hrs.

12. (a) Part filled in 7 min. = 7 × 1 136 45

æ ö+ç ÷è ø

= 720

Remaining part = 7120

æ ö-ç ÷è ø

= 1320

Part filled by (A + B + C) in 1 min.

= 1 1 1

36 45 30æ ö+ -ç ÷è ø

= 160 .

13. (d) (Man + Son)’s one day’s work = 18

Man’s one day’s work = 1

10

Þ Son’s one day’s work = 1 1 18 10 40

- =

\ Son can do it in 40 days.

14. (d) Q A can do 3 of the work in 12 days4

\ A can do 18

of the work in 4 112 days

13 8´ ´ = 2 days

15. (b) (A + B)’s 5 days’ work

= 5 1 1 45 925 20 100 20

æ ö+ = =ç ÷è ø

Remaining work = 2011

2091 =÷

øö

çèæ -

1120

of the work would be finished by B in

.days11

2012011

=

16. (a) 50 men complete 0.4 work in 25 days.Applying the work rule, 122211 wdmwdm ´´=´´we have,

4.025m6.02550 2 ´´=´´

or m2 = men754.025

6.02550=

´´´

Number of additional men required = (75 – 50) = 2517. (c) Let C completes the work in x days.

Work done by (A + B) in 1 day = 101

Work done by (B +C) in 1 day = 181

A’s 5 days’ work + B’s 10 days’ work + C’s 15 days’work = 1or (A + B)’s 5 days’ work + (B + C)’s 5 days’ work

+ C’s 10 days’ work = 1

or 5 5 10 110 18 x

+ + = or x = 45 days

18. (d) In 1 day, work done by 12 men = 1

18

In 6 days, work done by 12 men = 31

186

=

Remaining work = 32

Now, 122211 wdmwdm ´´=´´

Page 322: yoursmahboob.wordpress.com SBI · yoursmahboob.wordpress.com iii P 101 Speed Tests for SBI Bank Clerk Exam 101 Speed Tests for SBI Bank Clerk Exam is revised and updated edition on

yoursmahboob.w

ordpress.com

56 101 SPEED TEST

or 1d16321812 2 ´´=´´

or days916

2184d2 =´´

=

19. (c) 10 men’s 1 day’s work = 1 ;

15

15 women’s 1 day’s work = 1 .

12(10 men + 15 women)’s 1 day’s work

= 1 1 9 3 .

15 12 60 20æ ö+ = =ç ÷è ø

\ 10 men and 15 women will complete the work in20 26 days.3 3

=

20. (a) Work done by A and B in 5 days = 655

151

101

=´÷øö

çèæ +

Work remaining = 61

651 =-

\ C alone can do the work in 6 × 2 = 12 days

Ratio of their share work = 1:2:3122:

155:

105

=

Share of wages = ` 225, 150, 75.

21. (d) 1 man’s 1 day’s work = 1 .

100

(10 men + 15 women)’s 1 day’s work = 1.6

15 women’s 1 day’s work

= 1 10 1 1 1 .6 100 6 10 15

æ ö æ ö- = - =ç ÷ ç ÷è ø è ø

\ 1 woman’s 1 day’s work = 1 .

225\ 1 woman alone can complete the work in 225 days.

22. (a) Let the number of men originally employed be x.9x = 15(x – 6)or x = 15

23. (c) In 8 days, Anil does 1 rd work3

= .

\ in 1 day, he does 1 th work24

= .

\ Rakesh’s one day’s work = 60% of 124

= 1 th work40

.

Remaining work 1 213 3

= - =

(Anil and Rakesh)’s one day’s work

= 1 124 40

+1 th work

15=

Now, 1 th15

work is done by them in one day..

\ 23 rd work is done by them in

215 10days3

´ =

24. (a) Let 1 man’s 1 days’ work= x & 1 boy’s 1 day’s work = y

Then, 2x + 3y = 1

10 and 3x + 2y =

18

Solving, we get : 7

x200

= and 1

y100

=

\ (2 men + 1 boy)’s 1 day’s work

= 7 1 16 2

2 1200 100 200 25

æ ö´ + ´ = =ç ÷è øSo, 2 men and 1 boy together can finish the work in

1122

days.

25. (a) Ratio of time taken by A and B = 160 : 100 = 8 : 5

Suppose, B alone takes x days to do the jobthen, 8 : 5 : : 12 : x8x = 5 × 12

x = 5 12 17

8 2´

= days.

26. (c) Son’s 1 day’s work = 1 13 5

æ ö-ç ÷è ø

= 15 172 2

= days

27. (a) If x complete a work in x days. y will do the same task in3x days.3x – x = 40Þ x = 20y will finish the task in 60 days(x + y)’s 1 days work

= 1 1 120 60 15

+ =

Both of them will complete the work in 15 days.

28. (c) Part filled by (A + B + C) in 1 hour = 1 1 1 15 10 30 3

æ ö+ + =ç ÷è ø

.

\ All the three pipes together will fill the tank in3 hours.

29. (d) Part filled by first tap in one min 1 th12

=

Part filled by second tap in one min 1 th18

=

Now, 1 12 unfilled part = 112 18

é ù+ +ê úë û

13unfilled part = th18

Þ

Q 1 th18

part of tank is filled by second tap in 1min.

\ 13 th18

part of tank is filled by second tap in 1 min.

1318 min18

= ´ = 13 min.

30. (c) Hint : Let the time be t hours after 1 a.m.

\ ( ) ( )t 1 t 2t 1

4 5 2- -

+ - =

Page 323: yoursmahboob.wordpress.com SBI · yoursmahboob.wordpress.com iii P 101 Speed Tests for SBI Bank Clerk Exam 101 Speed Tests for SBI Bank Clerk Exam is revised and updated edition on

yoursmahboob.w

ordpress.com

57SOLUTIONS

SPEED TEST 43

1. (c) Let the age of father and son be 15x years and x yearsrespectively.

Now, according to the question, 15x x 16

2+

=

or, 16 2x 2years

16´

= =

Hence age of the son = 2 years2. (b) Average age = 28.5

\ Total age = 28.5 × 2 = 57

\ Daughter’s age = 5 57

19´ = 15 years

3. (a) Let Sudha’s and Neeta’s present ages be 6x and 7x yearsrespectively.According to the question.

6x 5 57x 5 6

-=

-Þ 36x – 30 = 35x – 25Þ x = 5\ Sudha’s present age = 6 × 5 = 30 years

4. (a) Required average age = 15 36 12 16 years

36 12´ + ´æ ö

ç ÷+è ø

= 540 192 years

48+æ ö

ç ÷è ø

= 732 years48

æ öç ÷è ø

= 15.25 years.

5. (a) Let the ages of Swati and Khyati two years ago be 5x and 7xyears respectively.According to the question,

5x 4 77x 4 9

+=

+Þ 49x + 28 = 45x + 36Þ 4x = 8 Þ x = 2\ Khyati’s present age = 7x + 2 = 7 × 2 + 2 = 16 years

6. (b) Shortcut method :

Son’s age = 5(9 1)(9 4)

-- = 8 yrs

\ Father’s age = 4 × 8 = 32 yrs7. (b) Shortcut method :

Son’s age = 5(7 1) 5(3 1)

7 3- + -

- = 10 yrs

From the first relationship of ages, if F is the age of the fatherthen F + 5 = 3 (10 + 5)\ F = 40 yrs

8. (c) Shortcut method :

Daughter’s age = 10(4 1) 10(2 1)

4 2- + -

- = 20 yrs

9. (e) 10 yrs ago, A was 12

of B’s ago.

AT present, A is 34

of B’s age.

\ B’s age =

110 12 20 yrs1 3

2 4

æ ö-ç ÷è ø=

-

A’s age = 34

of 20 = 15 yrs

10. (b) Let the age of the daugher be x yrs.Then, the age of the mother is (50 – x) yrs.5 yrs ago, 7(x – 5) = 50 – x – 5or, 8x = 50 – 5 + 35 = 80\ x = 10Therefore, daughter’s age = 10 yrsand mother’s age = 40 yrs

11. (c) Let the ratio of proportionality be x, then4x × x = 196 or, 4x2 = 196 or, x = 7Thus, Father’s age = 28 yrs, Son’s age = 7 yrsAfter 5 yrs, Father’s age = 33 yrs.Son’s age = 12 yrs\ Ratio = 33 : 12 = 11 : 4

12. (b) Difference in ratios = 8Then 8 º 24 \ 1 º 3i.e., value of 1 in ratio is equivalent to 3 yrsThus, Rita’s age = 3 × 3 = 9 yrsMother’s age = 11 × 3 = 33 yrs.After 3 years, the ratio = 12 : 36 = 1 : 3

13. (c) Let the present age be x yrs. Then

125% of (x – 10) = x; and 183 %

3 of (x + 10) = x

\ 125% of (x – 10) = 1

83 %3

of (x + 10)

5 5(x 10) (x 10)

4 6= = +

or, 5 5 50 50x x4 6 6 4

- = +

or, 5x 25012 12

= \ x = 50 yrs.

14. (b) Let the father’s present age be x and son’s age be x1 and x2.Now, x = 3(x1 + x2) .....(i)Also, x + 5 = 2(x1 + 5 + x2 + 5)

x + 5 = 2(x1 + x2 + 10) .....(ii)

Putting value of (x1 + x2) = 3x

from (i) in equation (ii)

x + 5 = 2 103xæ ö+ç ÷

è ø = 45

15. (a) Let the present ages of P and Q be 3x and 4x respectively.After 4 years

4x – 3x = 5\ x = 5\ P’s present age = 3x = 3 × 5 = 15 years

16. (c) Let the present ages of Rama and Shyama be 4x and 5x yearsrespectively,

\ 4x 5 55x 5 6

+=

+Þ 25x + 25 = 24x + 30Þ x = 30 – 25 = 5\ Rama’s present age = 4 × 5 = 20 years

17. (d) Let the mother’s age be y years.\ The age of father = (y + 9) years

The age of son = 2y

years

The age of daughter = 72yæ ö-ç ÷è ø

years

Now according to the given condition,

(y + 9) = 3 72yæ ö-ç ÷è ø

Þ y + 9 = 3 42

2y -

Þ 2y + 18 = 3y – 42Þ y = 60 years

18. (d) Suppose age of Ram = Rhis son’s age = Sand his father’s age = F

According to question, RS3

= and 2R F5

= ´

\ 5RF2

= and R S F 46

3+ +

=

R + S + F = 46 × 3

Page 324: yoursmahboob.wordpress.com SBI · yoursmahboob.wordpress.com iii P 101 Speed Tests for SBI Bank Clerk Exam 101 Speed Tests for SBI Bank Clerk Exam is revised and updated edition on

yoursmahboob.w

ordpress.com

58 101 SPEED TESTR 5RR 1383 2

+ + =

R = 3636S 123

= =

5 36F 902

´= =

19. (c) Let the ages of Abhay and his father 10 years ago be x and 5xyears respectively. Then,Abhay’s age after 6 years = (x + 10) + 6 = (x + 16) years.Father’s age after 6 years = (5x + 10) + 6 = (5x + 16) years.

\ (x + 16) = ( )3 5x 167

+ Û 7 (x + 16) = 3 (5x + 16)

Û 7x + 112 = 15x + 48Û 8x = 64 Û x = 8.Hence, Abhay’s father’s present age = (5x + 10) = 50 years.

20. (b) Let their present ages be 4x, 7x and 9x years respectively.Then, (4x – 8) + (7x – 8) + (9x – 8) = 56 Û 20x = 80 Û x = 4.\ Their present ages are 16 yrs, 28 yrs. and 36 yrs. respectively.

21. (d) 16 years ago, let T = x years and G = 8x yearsAfter 8 years from now, T = (x + 16 + 8) years andG = (8x + 16 + 8) years.\ 8x + 24 = 3(x + 24) Û 5x = 48.

8 years ago,

48 8T x 8 548G 8x 8 8 85

++= =

+ ´ +=

88 11424 53

=

22. (d) R – Q = R – T Þ Q = T. Also, R + T = 50Þ R + Q = 50So, (R – Q) cannot be determined.

23. (e) Let the ages of father and son be x and (45 – x) yearsrespectively.Then, (x – 5) (45 – x – 5) = 34Û (x – 5) (40 – x) = 34 Û x2 – 45x + 234 = 0Û (x – 39) (x – 6) = 0 Û x = 39 or x = 6.\ Father’s age = 39 years and son’s age = 6 years

24. (a) Let the ages of children be x, (x + 3), (x + 6)and (x + 12) years.Then, x + (x + 3) + (x + 6) + (x + 9) + (x + 12) = 50Û 5x = 20 Û x = 4.\ Age of the youngest child = x = 4 years.

25. (b) Anup’s age = (5 – 2) years = 3 years. Let Gagan’s age be xyears.

Then, x 6 318-

= Û x – 6 = 54 Û x = 60.

26. (c) Let the school ages of Neelam and Shaan be 5x and 6xrespectively. Then,1 5x 531 96x2

´=

´Û

1 9 5x3

æ ö´ ´ç ÷è ø

= 5 6x2

æ ö´ç ÷è ø Û 15 = 15.

Thus, Shaan’s age cannot be determined.27. (d) Let C’s age be x years. Then, B’s age = 2x years. A’s age = (2x

+ 2) years.\ (2x + 2) = 2x + x = 27 Û 5x = 25 Û x = 5.Hence, B’s age = 2x = 10 years.

28. (d) Let the present ages of the father and son be 2x and x yearsrespectively.Then, (2x – 18) = 3 (x – 18) Û x = 36.\ Required sum = (2x + x) = 3x = 108 years.

29. (d) Let the ages of Preeti and Sonal 1 year ago be 4x andx years respectively.Then, [(4x + 1) + 6] – [(x + 1) + 6] = 9 Û 3x = 9 Û x = 3.\ Required ratio = (4x + 1) : (x + 1) = 13 : 4.

30. (a) Let the present age of the father be ‘x’ and that of the son be

‘y’. Then x 8y 3

=

\ 3x = 8y ...(i)

Further, x 12 2y 12 1

+=

+ \ x + 12 = 2y + 24

\ x – 2y = 12 ...(ii)From eqn (i) and (ii), x = 48, y = 18\ sum = 66 yrs.

SPEED TEST 44

1. (c) Required number of ways= ways of selecting 4 objects out of 6 given objects

= 6C4 256 ´

= = 15

2. (c) Total no. of unrestricted arrangements = (7 – 1) ! = 6 !When two particular person always sit together, the total no.of arrangements = 6! – 2 × 5!Required no. of arrangements = 6! – 2 × 5!= 5! (6 – 2) = 5 × 4 × 3 × 2 × 4 = 480.

3. (c) In MATHEMATICS, the consonants M and T are repeatedtwo times each.Also the vowel A is repeated two times.Since there are four vowels, A, A, E and I; A being repeated,

therefore vowels can be arranged in 42

= 12 ways.

Now remaining 7 consonants, with M, T being repeated, can

be written in 7

2 2´ = 7 × 6 × 5 × 3 × 2 = 1260 ways.

Now four vowels together can take any of the 8 places asshown below:VC VC VC VC VC VC VC V\ Total number of ways in which the letters of the wordMATHEMATICS can be arranged such that vowels alwayscome together = 1260 × 8 × 12 = 120960.

4. (c) There are 8 different letters in the word MATHEMATICS;three letters M, A and T being repeated.The number of ways in which four letters of the wordMATHEMATICS can be arranged = 8

4P = 8 × 7 ×6 × 5 = 1680

5. (c) Number of ways = 6!2!

(Q T letter comes in two time)

= 6 5 4 3 2 1

2 1´ ´ ´ ´ ´

´ = 360

6. (e) CYCLE whereas C comes two times.

So, arrangements are = 5! 5 4 3 22! 2

´ ´ ´= = 60 ways

7. (e) Required no. of ways = 5! 602!

= is

Total no. of letters in the word is 5; T is repeated twice.8. (c) The committee of 4 persons is to be so formed that it has at

least 1 woman.The different ways that we can choose to form such acommittee are:

(i) 1w. 3 m in 4C1 × 6C3 = 6 5 44 803 2 1

´ ´´ =

´ ´

(ii) 2w. 2 m in 4C2 × 6C2 = 4 3 6 5 902 1 2 1

´ ´´ =

´ ´(iii) 3w. 1 m in 4C3 × 6C1 = 4 × 6 = 24(iv) 4w in 4C4 = 1\ Total no. of different ways in which a committee of 4persons can be formed so that it has at least one woman.= 80 + 90 + 24 + 1 = 195

Page 325: yoursmahboob.wordpress.com SBI · yoursmahboob.wordpress.com iii P 101 Speed Tests for SBI Bank Clerk Exam 101 Speed Tests for SBI Bank Clerk Exam is revised and updated edition on

yoursmahboob.w

ordpress.com

59SOLUTIONS9. (d) The committee of 4 persons is to be so formed that it has at

least 2 men. The different ways that we can choose to formsuch a committee are:

(i) 2m. 2w in 6C2 × 4C2 = 6 5 3 3 902 1 2 1

´ ´´ =

´ ´

(ii) 3m. 1w in 6C3 × 4C1 6 5 4 4 803 2 1

´ ´´ =

´ ´

(iii) 4m in 6C4 = 6 5 152 1

´=

´\ Total no. of different ways in which a committee of 4persons can be formed so that it has at least 2 men.= 90 + 18 + 15 = 185

10. (e) One girl can be chosen in 4C1 = 4 waysand 4 boys can be chosen in 6C4 = 15 ways\ Total number of ways = 4 × 15 = 60 ways

11. (a) CORPORATION= 11 letters‘O’ comes thrice, ‘R’ twice.

\ total no. of ways = 11!3!2! = 3326400

12. (b) There are seven letters in the word “COUNTRY” and twovowels O and U. Considering two vowels as one unit, totalnumber of letters will be 5 + 1 = 6. So, number of arrangements= 6!Now, the two vowels can be arranged in 21 ways amongthemselves.\ Total number of ways = 6! × 2! = 1440

13. (c) The word PROBLEM consists of 7 distinct letters.\ Number of arrangements = 7!= 70 × 6 × 5 × 4 × 3 × 2 × 1 = 5040

14. (e) There are 6 letter inthe word ‘ATTEND’ whereas, T comestwo times.

So, required number of ways = 6! 720 3602! 2

= =

15. (a) In word ‘offices’, there are 7 letters and F comes two times.

Required number of ways = 7! 7 6 5 4 3 2!2! 2!

´ ´ ´ ´ ´= = 2520

16. (e) ARMOUR = 6 letter whereas R repeated twice

\ 6! 6 5 4 3 2 1 3602! 2 1

´ ´ ´ ´ ´= =

´17. (c) Total number of ways to stand boys and girls together

= 4! × 3! × 2! = 4 × 3 × 2 × 3 × 2 × 2 = 28818. (e)

O, A, E S F T W RWhen the vowels are always together, then treat allthe vowels as a single letter and then all the letterscan be arranged in 6! ways and also all three vowelscan be arranged in 3! ways. Hence, required no. ofarrangements = 6! × 3! = 4320.

19. (b) Reqd probability = 5

27

2

5 4 107 6 21

CC

´= =

´

20. (b) Treat B and T as a single letter. Then the remainingletters (5 + 1 = 6) can be arranged in 6! ways. Since, O isrepeated twice, we have to divide by 2 and the B and Tletters can be arranged in 2! ways.

Total no. of ways 6! 2! 7202´

= =

21. (e) If the drawn ball is neither red nor green, then it must beblue, which can be picked in 7C1 = 7 ways. One ball canbe picked from the total

(8 + 7 + 6 = 21) in 21C1 = 21 ways.

\ Reqd probability 7 121 3

= =

22. (c) Taking all vowels (IEO) as a single letter (since theycome together) there are six letters

Hence no. of arrangements = 6! 3! 21602!

´ =

[Three vowels can be arranged 3! ways amongthemselves, hence multiplied with 3!.]

23. (b) When 0 is the repeated digit like100, 200, ...., 9 in numberWhen 0 occurs only once like110, 220, ....., 9 in numberWhen 0 does not occur like112, 211, ....., 2 × (8 × 9) = 144 in number.Hence, total = 9 + 9 + 144 = 162.

24. (b) Required number of possible outcomes= Total number of possible outcomes –Number of possible outcomes in which all vowels aretogether= 6 ! – 4 ! × 3 != 576

25. (d) The required number of ways= 8791)17)(19)(110( =-+++ .

26. (c) Three digit number less then 600 will have first element100, and last element 599. First place will not have digitmore than 6, hence, 7 and 9 can not be taken : So, firstdigit can be selected in 4 ways. Second digit can beselected in 6 ways and since repetition of digits areallowed, third digit can also be selected in 6 ways :So, number of ways are 4 × 6 × 6 = 144.

27. (b) Selection of 2 members out of 11 has 11C2 number ofways

11C2 = 5528. (b) From each railway station, there are 19 different tickets

to be issued. There are 20 railway stationSo, total number of tickets = 20 × 19 = 380.

29. (c) To make a 5 digit number, 0 can not come in the bagining.So, it can be filled in 4 ways. Rest of the places can befilled in 4! ways. So total number of digit formed = 4 × 4!= 2 × 24 = 96

30. (b) Total no. of digits = 6To form a odd numbers we have only 3 choice for theunit digits.Now, Extreme left place can be filled in 6 ways themiddle place can be filled in 6 ways.\ Required number of numbers = 6 × 6 × 3 = 108

SPEED TEST 45

1. (a) Here, S = {1, 2, 3, 4, ...., 19, 20}.Let E = event of getting a multiple of 3 = {3, 6, 9, 12, 15, 18}

\ P(E) = n(E) 6 3n(S) 20 10

= = .

2. (c) P (getting a prize) = 10 10 2

(10 25) 35 7= =

+ .

3. (b) Clearly, there are 52 cards, out of which there are 16face cards.

\ P (getting a face card) = 16 452 13

= .

Page 326: yoursmahboob.wordpress.com SBI · yoursmahboob.wordpress.com iii P 101 Speed Tests for SBI Bank Clerk Exam 101 Speed Tests for SBI Bank Clerk Exam is revised and updated edition on

yoursmahboob.w

ordpress.com

60 101 SPEED TEST4. (c) Here, n(S) = 52.

There are 26 red cards (including 2 kings) and there are2 more kings.Let E = event of getting a red card or a king.Then, n(E) = 28.

\ P(E) = n(E) 28 7n(S) 52 13

= = .

5. (b) Total number of balls = (6 + 8) = 14Number of white balls = 8

P (drawing a white ball) = 8 4

14 7= .

6. (d) Total number of balls = (8 + 7 + 6) = 21.Let E = event that the ball drawn is neither red nor green. = event that the ball drawn is red.\ n(E) = 8

\ 8

P(E)21

= .

7. (b) Clearly, n(S) = (6 × 6) = 36.Let E = Event that the sum is a prime number.Then, E = {(1, 1), (1, 2), (1, 4), (1, 6), (2, 1), (2, 3), (2, 5), (3,2), (3, 4), (4, 1), (4, 3), (5, 2), (5, 6), (6, 1), (6, 5)}

n(E) 15\ =

n(E) 15 5P(E)

n(S) 36 12\ = = = .

8. (d) Here S = {HH, HT, TH, TT}.Let E = event of getting at least one head = {HT, TH, HH}

n(E) 3P(E)

n(S) 4\ = = .

9. (b) Here S = {TTT, TTH, THT, HTT, THH, HTH, HHT, HHH}Let E = event of getting at least two heads = {THH, HTH, HHT, HHH}

n(E) 4 1P(E)

n(S) 8 2\ = = = .

10. (b) When a die is thrown, we have S = {1, 2, 3, 4, 5, 6}Let E = event of getting a number greater than 4 = {5, 6}.

n(E) 2 1P(E)

n(S) 6 3\ = = = .

11. (a) We know that in a simultaneous throw of two dice,n(S) = 6 × 6 = 36.Let E = event of getting a total f 7 = {(1, 6), (2, 5), (3, 4), (4, 3), (5, 2), (6, 1)}

n(E) 6 1P(E)

n(S) 36 6\ = = = .

12. (c) In two throws of a die, n(S) = (6 × 6) = 36.Let E = event of getting a sum 9 = {(3, 6), (4, 5), (5, 4), (6, 3)}

n(E) 4 1P(E)

n(S) 36 9\ = = = .

13. (a) In a simultaneous throw of two dice, n(S) = 6 × 6 = 36.Let E = event of getting a doublet = {(1, 2), (2, 2), (3, 3), (4, 4), (5, 5), (6, 6)}

n(E) 6 1P(E)

n(S) 36 6\ = = = .

14. (d) In a simultaneous throw of two dice, we haven(S) = (6 × 6) = 36.Let E = event of getting a total of 10 or 11 = {(4, 6), (5, 5), (6, 4), (5, 6), (6, 5)}

n(E) 5P(E)

n(S) 36\ = = .

15. (a) Total possible outcomes, S ={HHH, HHT, HTH, THT,TTH, THH, TTT, HTT} and desired outcomes E ={HTH,THT}

Þ n(E) = 2 and n(S) = 8

Hence, required probability = n(E)P(E)n(S)

= = 28

= 14

16. (c) 16 tickets are sold and 4 prizes are awarded. A person

buys 4 tickets, then required probability = 4 1

16 4=

17. (c) Total number of letters = 4Total number of vowels = 2 (O and E)

Required Probability = 2 14 2

=

18. (d) Total no. of case = 63 = 216Favourable cases = {(1, 1, 1), (2, 2, 2), (3, 3, 3), (4, 4, 4),(5, 5, 5), (6, 6, 6)}.

Probability = 6 1

216 36=

19. (b) No. of days in leap year = 366No. of complete week = 52 ( )366 7 gives 2 as remainder¸Q

\ No. of days left = 2

Required probability = 27

20. (a) Total case = 6 × 6 = 36Favourable = (1, 6), (2, 5), (3, 4), (4, 3), (5, 2), (6, 1)

Probability = 6 1

36 6=

21. (a) Total number of balls = 5 + 7 + 8 = 20Probability that the first ball drawn is white

51

201

14

CC

= =

If balls are drawn with replacement, all the four eventswill have equal probability.Therefore, required probability

2561

41

41

41

41

=´´´=

22. (b) Total no. of outcomes when two dices are thrown = n(S) = 36 and the possible cases for the event that thesum of numbers on two dice is a prime number, are(1, 1), (1, 2), (1, 4), (1, 6), (2, 1), (2, 3), (2, 5), (3, 2), (3, 4),(4, 1), (4, 3), (5, 2), (5, 6), (6, 1), (6, 5)Number of outcomes favouring the event = n (A) = 15

Required probability ( )( )

1536

n An S

= =125

=

Page 327: yoursmahboob.wordpress.com SBI · yoursmahboob.wordpress.com iii P 101 Speed Tests for SBI Bank Clerk Exam 101 Speed Tests for SBI Bank Clerk Exam is revised and updated edition on

yoursmahboob.w

ordpress.com

61SOLUTIONS

23. (d) The probability of selecting any bag 21

=

Now, probability of getting a white ball from the first

bag 103

53

21

=´=

and probability of getting a white ball from the second

bag 61

62

21

=´=

Required Probability = The probability that a white ballis drawn either from the first or the second bag

157

61

103

=+=

24. (b) Out of 20 consecutive numbers there are 10 even and10 oddWe have to choose 2 numbers out of 20

\ Total outcomes = 202

20 9C 1902´

= =

We can odd sum only when one even and one odd isadded.\ No. of ways of choosing 1 even and 1 oddOut of 10 even and 10 odd = 10C1 × 10 C1 = 10 × 10 = 100

\ Required probability = 100 10190 19

=

25. (b) Here S = {1, 2, 3, 4, 5}Let E be the event of getting a multiple of 3.E = {3, 6}

2 1P(E) .

6 3\ = =

SPEED TEST 46

1. (a) In a circle, circumference = 2pr

Hence, 44 = 2pr 44r2

\ =p

Now, area of circle = p r2 244 44 154 m2 2

= p´ ´ =p p

2. (a) Let the length and breadth of a rectangle are 9 xm and5 xm respectively.In a rectangle, area = length × breadth\ 720 = 9x × 5xor x2 = 16 Þ x = 4Thus, length = 9 × 4 = 36 mand breadth = 5 × 4 = 20 mTherefore, perimeter of rectangle = 2(36 + 20) = 112 m

3. (d) Perimeter of the circle = 2 r 2(18 26)p = +

Þ222 r 887

´ ´ = Þ r = 14

\ Area of the circle

= 2 222r 14 14 616 cm7

p = ´ ´ = .

4. (b) Length of the carpet

= Total costRate / m

æ öç ÷è ø

8100 m 180 m.

45æ ö= =ç ÷è ø

Area of the room = Area of the carpet

= 2 275180 m 135 m .

100æ ö´ =ç ÷è ø

\ Breadth of the room = Area 135 m

Length 18æ ö æ ö=ç ÷ ç ÷

è øè ø= 7.5 m.

5. (a) In a rectangle,2

2(perimeter) (diagonal) 2 area4

= + ´

Þ 2

2(14) 5 2 area4

= + ´

49 = 25 + 2 × area

249 25 24Area 12cm2 2-

\ = = =

6. (a) In an isoscele right angled triangle,Area = 23.3 × perimeter2

= 23.3 × 202 = 9320 m2

7. (b) Required area covered in 5 revolutions

= 5 × 2prh = 5 × 2 × 722

× 0.7 × 2 = 44 m2

8. (c) In a triangle,

Area 1 length of perpendicular × base2

= ´

or 1615 length of perpendicular × 1232

= ´

\ Length of perpendicular 615 2

123´

= = 10 m.

9. (a) 14 m

14 m

C

24 m

40 mA B

D

Area of the shaded portion

( )21441

p´=

= 154 m2

10. (a) Circumference of circular bed = 30 cm

Area of circular bed 2(30)

4p=

Space for each plant = 4 cm2

\ Required number of plants

2(30) 4 17.89 18 (Approx)

4p= ¸ = =

11. (c)

W15

10

Page 328: yoursmahboob.wordpress.com SBI · yoursmahboob.wordpress.com iii P 101 Speed Tests for SBI Bank Clerk Exam 101 Speed Tests for SBI Bank Clerk Exam is revised and updated edition on

yoursmahboob.w

ordpress.com

62 101 SPEED TESTLet the width of the path = W mthen, length of plot with path = (15 + 2W) mand breadth of plot with path = (10 + 2 W) mTherefore, Area of rectangular plot (wihout path)

= 15 × 10 = 150 m2

and Area of rectangular plot (with path)= 150 + 54 = 204 m2

Hence, (15 + 2W) × (10 + 2W) = 204Þ 4W2 + 50 W – 54 = 0Þ 2W2 + 25 W – 27 = 0Þ (W – 2) (W + 27) = 0Thus W = 2 or –27\ with of the path = 2 m

12. (a) If area of a circle decreased by x % then the radius of acircle decreases by

(100 10 100 x)%- - = (100 10 100 36)%- -

(100 10 64)%= -

100 80 20%= - =13. (a) Area of the outer rectangle = 19 × 16 = 304 m2

2m

2m

2m2m

15

12

Area of the inner rectangle = 15 × 12 = 180 m2

Required area = (304 – 180) = 124 m2

14. (a) Radius of a circular grass lawn (without path) = 35 m\ Area = pr2 = p (35)2

Radius of a circular grass lawn ( with path)= 35 + 7 = 42 m

\ Area = pr2 = p(42)2

\ Area of path = p(42)2 – p(35)2

= p(422 – 352) = p( 42 + 35) (42 –35)

= p × 77 × 7 222 77 7 1694 m7

= ´ ´ =

15. (b) Radius of the wheel of bus = 70 cm. Then,circumference of wheel = 2pr = 140 p = 440 cmDistance covered by bus in 1 minute

66 1000 100 cms60

= ´ ´

Distance covered by one revolution of wheel= circumference of wheel= 440 cm

\ Revolutions per minute 6600000 25060 440

= =´

16. (b) Let ABC be the isosceles triangle and AD be the altitude.Let AB = AC = x. Then, BC = (32 – 2x).

A

B CD

xx

Since, in an isosceles triangle, the altitude bisects thebase. So, BD = DC = (16 – x).In DADC, AC2 = AD2 + DC2 Þ x2 = (8)2 + (16 – x)2

Þ 32x = 320 Þ x = 10.\ BC = (32 – 2x) = (32 – 20) cm = 12 cm.

Hence, required area = 1 BC AD2

æ ö´ ´ç ÷è ø

2 21 12 10 cm 60 cm .2

æ ö= ´ ´ =ç ÷è ø

17. (c) Area of field = 576 km2. Then,

each side of field = 576 24 km=Distance covered by the horse

= Perimeter of square field= 24 × 4 = 96 km

\ distance 96Time taken by horse =

speed 12= = 8 h

18. (c) Let the length and breadth of the original rectangularfield be x m and y m respectively.Area of the original field = x × y = 144 m2

\ 144x

y= … (i)

If the length had been 6 m more, then area will be(x + 6) y = 144 + 54

Þ (x + 6) y = 198 … (ii)Putting the value of x from eq (i) in eq (ii), we get

144 6 y 198y

æ ö+ =ç ÷

è ø

Þ 144 6y 198+ =Þ 6y = 54 Þ y = 9 mPutting the value of y in eq (i) we get x = 16 m

19. (b) Perimeter = Distance covered in 8 min.

12000 8 m 1600 m.

60æ ö= ´ =ç ÷è ø

Let length = 3x metres and breadth = 2x metres.Then, 2 (3x + 2x) = 1600 or x = 160.\ Length = 480 m and Breadth = 320 m.\ Area = (480 × 320) m2 = 153600 m2.

20. (c) Length of wire = 222 R 2 56 cm7

æ öp´ = ´ ´ç ÷è ø

= 352 cm.

Side of the square = 352

4 cm = 88 cm.

Area of the square = (88 × 88) cm2 = 7744 cm2.

Page 329: yoursmahboob.wordpress.com SBI · yoursmahboob.wordpress.com iii P 101 Speed Tests for SBI Bank Clerk Exam 101 Speed Tests for SBI Bank Clerk Exam is revised and updated edition on

yoursmahboob.w

ordpress.com

63SOLUTIONS21. (a) Let the length of the room be l m

Then its, breadth = l /2

Therefore, 5000

2 25´ =l

l

or l 2 = 400or l = 20 m

Also, 648002 h 2 h2 240

+ ´ ´ =l

l

Þ 3 l h = 270

or 270 270h 4.5m3 20 60

= = =´

22. (a) In a cube,Area = 6 (side)2

or 150 = 6 (side)2

\ side = 25 5 m=

Length of diagonal = 3 side 5 3 m´ =23. (b) Given, playground is rectangular.

Length = 36 m, Breadth = 21 mNow, perimeter of playground = 2( 21 + 36) = 114Now, poles are fixed along the boundary at a distance3m.

\ Required no. of poles = 114 38

3= .

24. (d) Area of square 12

= ´ (diagonal)2

( )21 128 28 28

2 2= ´ = ´ ´

392= cm2

25. (c) One side of square circumference=4

44 114

= = cm

Circumference of rectangle = 4 × perimeter of square= 4 × 44 = 176 cmwidth of rectangle

circumference of rectangle=2

– length

17651 88 51 37

2= - = - = cm.

\ Required difference = width – side = 37 – 11 = 26 cm.

26. (c) Radius of circle circumference 220× 7(r) = 352 2× 22

= =p

m.

area of circle = 2 222 22r (35) 35 357 7

p = ´ = ´ ´

= 3850 m2 = area of rectangle

\ Length of rectangle area of rectanglewidth

=

3850= = 77m.50

27. (a) 79.2 km/hr 579.2 2218

= ´ = m/s

2 min 40 sec 2 60 40 120 40 160= ´ + = + = sec.Circumference of circular field = speed × time

22 160 3520= ´ = m

Radius of circular field circumference(r)=2p

3520 7 5602 22

´= =

´m

Area of circular field 2 222r (560)7

= p = ´

22 560 5607

= ´ ´

= 985600 m2

28. (d) Area of rectangle = l × b = 240Either length or breadth should be clear, then answer can bedetermined.

29. (c) Area of the circle = 39424 sq cm

pr2 = 39424 Þ 222 r 394247

´ =

Þ 2 39424 7r22

´= Þ r2 = 1792 × 7

Þ r 12544= r = 112 cm 4a = r 4a = 112 a = 28\ Area of the square = a2 = (28)2 = 784 sq cm

30. (b) From the figure, it is required to find the length CD.We have CA = LB = 15m\ LD = BD – LB = 15m

D

BA

C15 m 36 m

36 m

15 mL

\ CD = 2 2 2 2CL DL 36 15 1521 39cm+ = + = =31. (d) Total area = Area of square + 4 (Area of a semi-circle)

= 2 212 4 r

2æ ö+ pç ÷è ø = (4 + 2p) m2 2radius = 1

2é ù=ê úë û

32. (b) In any quadrilateral,Area of the quadrilateral

= 12

× any diagonal × (sum of perpendiculars drawn on

diagonal from two vertices) = 1 21 D (P P )2

´ ´ +

= 1

23 (17 7)2

´ ´ + = 12 × 23 = 276 sq cm

33. (b) Length of the wire = circumference of the circle

= 2p × 42 = 2 22 42

264 cm7

´ ´=

Now, perimeter of the rectangle = 264 cm.Since, perimeter includes double the length and breadth, whilefinding the sides we divide by double the sum of ratio.

Therefore, length = 264

6 72 cm2(6 5)

´ =+

and breadth = 264

5 60 cm2(6 5)

´ =+

Page 330: yoursmahboob.wordpress.com SBI · yoursmahboob.wordpress.com iii P 101 Speed Tests for SBI Bank Clerk Exam 101 Speed Tests for SBI Bank Clerk Exam is revised and updated edition on

yoursmahboob.w

ordpress.com

64 101 SPEED TEST34. (a) Let the final length of the side of the smaller square be a.

Now, a + 10 + 7 = 19a = 19 – 17 = 2

\ Area of the smaller square = (2)2 = 4\ Decrease in the area of the smaller square = 16 – 4

= 12 sq. units.35. (c) Let the breadth be b.

Then, length = b + 13 Perimeter = 50 = 2(l + b)2(b + 13 + b) = 50 b = 6 m l = 6 + 13 = 19 m

\ Area = length × breadth = 19 (6) = 114 m2

SPEED TEST 47

1. (a) Volume of the bucket = volume of the sand emptiedVolume of sand = p (21)2 × 36Let r be the radius of the conical heap.

Then, ( ) 362112r31 22 ´p=´p

or r2 = (21)2 × 9 or r = 21 × 3 = 632. (a) Let the edge of the third cube be x cm.

Then, x3 + 63 + 83 = 123

Þ x3 + 216 + 512 = 1728Þ x3 = 1000 Þ x = 10.Thus the edge of third cube = 10 cm.

3. (b) Area of the inner curved surface of the well dug

= [2p × 3.5 × 22.5] = 222 3.5 22.57

´ ´ ´

= 44 × 0.5 × 22.5 = 495 sq. m.\ Total cost = 495 × 3 = 1485.

4. (d) Let the length of the wire be h cm.and radius of sphere and wire are R and r respectively.Then, volume of sphere = volume of wire (cylinder)

or 3 24 R r h3

=p p or 3 24 R r h3

=

or 3 24 (3) (0.1) h3

=

\ 3

24 (3) 108h 3600cm

0.033 (0.1)´

= = =´

= 36 m

5. (c) In a sphere, volume 34 r3

= p

and surface area 24 r= p

According to question, 3 24 r 4 r 273

¸ =p p

or r = 27 × 3 = 81 cms6. (a) Area of the wet surface = [2(lb + bh + lh) – lb]

= 2(bh + lh) + lb= [2(4 × 1.25 + 6 × 1.25) +6 × 4] m2 = 49 m2.

7. (b) Let l be the length and b be the breadth of coldstorage.L = 2B, H = 3 metresArea of four walls = 2[L × H + B × H] = 108Þ 6BH = 108 Þ B = 6\ L = 12, B = 6, H = 3Volume = 12 × 6 × 3 = 216 m3

8. (c) Let 'A' be the side of bigger cube and 'a' be the side ofsmaller cube

Surface area of bigger cube = 6 A2

or 384 = 6A2

\ A = 8 cm.Surface area of smaller cube = 6 a2

96 = 6a2

\ a = 4 mm = 0.4 cm

So, Number of small cube Volume of bigger cubeVolume of smaller cube

=

3

3(8) 512 8,000

0.064(0.4)= = =

9. (c) Volume of the liquid in the cylindrical vessel= Volume of the conical vessel

= 31 22 12 12 50 cm

3 7æ ö´ ´ ´ ´ç ÷è ø

= 322 4 12 50 cm .

7´ ´ ´æ ö

ç ÷è ø

Let the height of the liquid in the vessel be h.

Then, 22 22 4 12 5010 10 h7 7

´ ´ ´´ ´ ´ =

or h = 4 12 50 24 cm.

10 10´ ´æ ö =ç ÷´è ø

10. (b) Volume of material in the sphere

= { }3 3 3 34 4(4) (2) cm 56 cm .3 3

é ù æ öp´ - = p´ç ÷ê úë û è øLet the height of the cone be h cm.

Then, 1 44 4 h 563 3

æ öp´ ´ ´ = p´ç ÷è ø

4 56h 14 cm.4 4´æ öÞ = =ç ÷´è ø

11. (d) Volume of sphere = 34 9 9 9 cm .

3æ öp´ ´ ´ç ÷è ø

Volume of cone = 31 9 9 9 cm .

3æ öp´ ´ ´ç ÷è ø

Volume of wood wasted

= 34 19 9 9 9 9 9 cm .

3 3é ùæ ö æ öp´ ´ ´ - p´ ´ ´ç ÷ ç ÷ê úè ø è øë û

= (p × 9 × 9 × 9) cm3

\ Required percentage = 9 9 9 100 %4 9 9 9

3

æ öç ÷p´ ´ ´

´ç ÷ç ÷´ p´ ´ ´ç ÷è ø

= 3 100 % 75%.4

æ ö´ =ç ÷è ø

12. (b) Curved surface area of cylinder = 2prh\ Surface area of 50 cylindrical pillars = 50 × 2prhNow, Diameter of each cylindrical pillar = 50 cm

\ Radius = 502

= 25 cm » .25 m

Page 331: yoursmahboob.wordpress.com SBI · yoursmahboob.wordpress.com iii P 101 Speed Tests for SBI Bank Clerk Exam 101 Speed Tests for SBI Bank Clerk Exam is revised and updated edition on

yoursmahboob.w

ordpress.com

65SOLUTIONSAlso, height = 4m\ Surface area = 50 × 2 × 3.14 ×. 25 × 4 = 314 × 1 sq m. = 314 sq. m.Now, labour charges at the rate of 50 paiseper sq. m = 314 ×.5 = 157.0 º 157

13. (c) Let the kerosene level of cylindrical jar be h.

Now, Volume of conical vessel = 21 r h3

p

Since, radius (r) = 2 cm and height(h) = 3cm of conicalvessel.

\ Volume = 1 4 3 43

p ´ ´ = p

Now, Volume of cylinderical jar = pr2h = p (2)2h = 4ph

Now, Volume of conical vessel = Volume of cylindrical Jar

Þ 4 p = 4 ph h = 1cmHence, kerosene level in Jar is 1 cm.

14. (a) Let the rise in water level = x mNow, volume of pool = 40 × 90 × x = 3600 xWhen 150 men take a dip, then displacement ofwater = 8m3

\3600 x 8

150= Þ

900 x 2 x .33m150

= Þ =

Þ x = 33.33 cm15. (d) Let edge of the new cube = x cm.

Volume of the newly formed figure (cube)= sum of volume of smaller cubes.i.e. (x)3 = (3)3 + (4)3 + (5)3 = 27 + 64 + 125 = 216Þ x = 6 cm

16. (b) Volume of the spherical ball = volume of the waterdisplaced.

Þ 343

rp = p (12)2 × 6.75

Þ 3 144 6.75 3 7294

r ´ ´= = Þ r = 9 cm,

17. (d) Let x be the length of a side (edge of the cube)Now, x3 = 12 x Þ x2 = 12Total surface area = 6x2 = 6 × 12 = 72 square units

18. (b) Let an edge of the cube be ‘a’cm.Now, 6a2 = 726 Þ a2 = 121 Þ a = 11mVolume of the cube = (11)3 = 1331 m3.

19. (c) 1 litre = 1 (dm)3

Length of the cistern = 3m = 30 dm (Q 1 m = 10 dm)Breadth of the cistern = 2m = 20 dmHeight of the cistern = 1 m = 10 dm\ Volume of the cistern = 30 × 20 × 10 = 6000 (dm)3

20. (c) Here r = 8 cm, h = 15 cm Þ l 17158 22 =+=Curved surface area of the cone

28 17 136 cmr= p = p ´ ´ = pl

21. (c) Volume of the cone 213

r h= p

31 22 49 51 2618cm3 7

= ´ ´ ´ =

1 cm3 weighs 10 g. Þ 2618 cm3 will weigh

= 2618 10 kg 26.18kg

1000´

=

22. (b) Volume of the steel used in the hemispherical bowl

( )[ ]33 4–5.43

p2

= 125.27722

32

´´= = 56.83 cm3

23. (d) Volume of the parallelopiped = 5 × 4 × 3 = 60 cm3

Volume of the cube = 4 × 4 × 4 = 64 cm3

Volume of the cylinder = p × 3 × 3 × 3 = 27p = 84.8 cm3

Volume of the sphere 3 34 (3) 36 113 cm3

= p = p =

The required decreasing order is D, C, B and A.24. (d) Let R be the radius of bigger sphere. Volume of one

spheres + Volume of other sphere = Volume of biggersphere

Then, ( ) ( ) 33 34 4 4R1 2

3 3 3p + p = p

3 3 34 4(1 2 ) R

3 3Þ p + = p

Þ R3 = 913R 9\ =

25. (b) Given, Diameter of building = 54 cm\ Radius = 27 cm.Also, given area of the base of the wall = 352 cm2

Let Dx be the thickness of the wall\ Area of the Base = 2 p r× D xÞ 352 = 2 × p × 27 ×D x

Þ D x = 352 7 8 7

2 22 27 27´ ´

=´ ´

56 227

= »

Hence, thickness of the wall is 2 cm.

SPEED TEST 48

1. (c) In a right angled D, the length of the median is 21 the

length of the hypotenuse . Hence .cm3AC21BD ==

2. (a) °=-=Ð-=Ð 11070180B180DCADD180ACD Ð-Ð-=Ð\

°=-- 40301101803. (b) The sum of the interior angles of a polygon of n sides

is given by the expression (2n – 4) 2p

Þ ( )2n – 4 16202 180p p

´ = ´

( ) 1620 22n – 4 18180

´= =

or 2n = 22or n = 11

4. (c) Tangent at any point of a circle is ^ to the radiusIn DOPT, OP2=PT2+OT2

(13)2 = (12)2 + OT2 Þ 169 – 144 = OT2

Þ 25 = OT2 Þ 5 = OT

Page 332: yoursmahboob.wordpress.com SBI · yoursmahboob.wordpress.com iii P 101 Speed Tests for SBI Bank Clerk Exam 101 Speed Tests for SBI Bank Clerk Exam is revised and updated edition on

yoursmahboob.w

ordpress.com

66 101 SPEED TEST5. (c) Let the angles of the triangle be 5x, 3x and 2x.

Now, 5x + 3x + 2x = 180°or 10x = 180 or x = 18or Angles are 36, 54 and 90°Given D is right angled.

6. (c) m Ð ABM = 180º –120º = 60º\ D AMB is a 30º – 60º – 90º triangle.

\ AM 3

2 AB =

32

× 8 = 4 3

MB = 12

AB = 12

x 8 = 4

(AC)2 = (AM)2 + (MC)2 = (4 3 )2 + (4 +7)2

= 48 + 121 = 169 ; AC = 169 = 13.7. (d)

BC

A

10 cm

Q

P

R

D ABC and D PQR are similar..

Perimeter of ABCPerimeter of PQR

D=

DABPQ

Þ 3624

=ABPQ

or 36 10 1524

= ´ =AB

8. (a) For the two similar triangles, we have2122

h Area of 1st 9Area of IInd 16h

D= =

D

Þ h1 : h2 = 3 : 49. (c) Circumcentre of a triangle is the point of the intersection

of the perpendicular bisectors of its sides.10. (a) We have, x + y + (y + 20) = 180

or x + 2y = 160 ...(i)and 4x – y = 10 ...(ii)From (i) and (ii), y = 70, x = 20Angles of the triangles are 20°, 70°, 90°. Hence thetriangle is a right angled.

11. (a) Clearly (x + 1) will be the hypotenuse of the righttriangle ]0x[ ³Now, (x + 1)2 = x2 + (x – 1)2

or x2 + 2x + 1 = x2 + x2 – 2x + 1or x2 – 4x = 0 or x (x – 4) = 0 or x = 4The side of a triangle cannot be equal to zero.There fore, x = 4\ Hypotenuse = (4 + 1) = 5

12. (d)2 cm

1 cmO 3 cm

BC A2 2cm

2 23 1 2 2= - =AB cm

\ 4 2=AC cm13. (d) Let the angles be x and 3x

We know, sum of the supplementry angles = 180°\ x + 3x = 180°Þ 4x = 180° Þ x = 45°

14. (d)15. (c) Supplementary angles are pairs of angles whose

measures upto 180 degrees.Hence, let one angle be x. Since they are equal, \ theother angle is also equal to x.So, x + x = 180 Þ 2x = 180\ x = 90

16. (b)

17. (b) Four right angle + 1 rd3

right angle

= 4 × 90 + 13

× 90 = 360 + 30 = 390

18. (b) Two right angles = 2 × 90° = 180°

Half right angles = 1 902

´ ° = 45°

Total two right angles and one half right angle

= 18045

225

°+ °

°

19. (a) Five and two-third of a right angle

= 5 × 90° + 2 903

´ ° = 450° + 60° = 510°

20. (c) Let the angle be q.According to question

( )2 903

q = - q ...(1)

( )1 1804

q = - q ...(2)

From eq. (1)

( )2 903

q = - q

3q = 180 – 2q5q = 180 Þ q = 36º

21. (a) Let the angles are x, 2x, 3x and 4xx + 2x + 3x + 4x = 360º10x = 360ºx = 36º4th angle = 4 × x = 4 × 36 = 144º

22. (b)

3

O

5

MBA

Page 333: yoursmahboob.wordpress.com SBI · yoursmahboob.wordpress.com iii P 101 Speed Tests for SBI Bank Clerk Exam 101 Speed Tests for SBI Bank Clerk Exam is revised and updated edition on

yoursmahboob.w

ordpress.com

67SOLUTIONSIn DOAM52 = 32 + AM2

25 – 9 = AM2

16 = AM2

AM = 4\ Length of chord AB= 2 × AM = 2 × 4 = 8 cm

23. (a) 2x + 3x = 180º5x = 180ºx = 36ºHence, angles are 72º and 108º respectively. As oppositeangle of parallelogram are equal. Therefore, measure ofall angles are 72º, 108º, 72º, 108º.

24. (d) Let the angles be 4x, 3x and 2 x.4x + 3x + 2x = 180° x = 20°

\ angles are 80°, 60° and 40°.25. (c) Second angle of parallelogram

= 180° – 45° = 135°\ Required value

= 135 + 2 × 45= 135 + 90 = 225°

SPEED TEST 49

1. (a)121 117 108 92 67 31

–62–22 –32 –42 –52

2. (b)50 26 14 8 5 3.5

÷2+1÷2+1 ÷2+1 ÷2+1 ÷2+1

3. (c)3 23 43 63 83 103

+20+20 +20 +20 +20

4. (e)748 737 715 682 638 583

–55–11 –22 –33 –44

5. (d)

1 9 25 49 81 121 169

12 32 52 72 92 112 132

6. (b)2 2 2 2 2 2 2 2 2 2

36 20 12 8 6 5¸ + ¸ + ¸ + ¸ + ¸ +

E55555F E55555F E555F E5555F E55555F

7. (c)12 24 48 96 192

2 2 2 2

668 656 632 584 448 296- - - - -

´ ´ ´ ´

E55555F E55555F E55555F E55555F E55555F

E55555F E55555F E55555F E55555F

8. (e)

2 2 2 2 2 2

1 121 441 961 1681 2601

1 11 21 31 41 51­ ­ ­ ­ ­ ­

9. (d)5 4 4 5 5 4 4 5 5 4 4 5

9 49 201 1009 4041 20209 80841´ + ´ + ´ + ´ + ´ + ´ +E55F E555F E555F E555F E55555F E555F

10. (a)2 2 2 2 22 3 4 5 6

31 35 44 60 85 121

+ + + + +E55555F E55555F E5555F E5555F E55555F

11. (d) Pattern of the series would be as follows7 9 12 16 21

+2 +3 +4 +512. (c) Pattern of the series would be as follows

384 192

÷2

96 48 24

÷2 ÷2 ÷213. (e) Pattern of the series would be as follows

5 × 1 + 1 = 66 × 2 + 2 = 1414 × 3 + 3 = 45

\ 45 × 4 + 4 = 18414. (e) Pattern of the series would be as follows

8 9

+(1)2

13 22 38

+(2)2 +(3)2 +(4)2

15. (a) Pattern of the series would be as follows6 11

+5

21 41 81

+10 +20 +4016. (b) The series pattern would be as follows :

+6 +12 +24 +48

7 13 25 49 97

So, the above series is progressing of doubledifferences.

17. (d) The series pattern would be as follows :5 6 10 19 35

(1) (2) (3) (4)2 2 2 2

18. (a) The series pattern would be as follows :8 × 1 + 1 = 99 × 2 + 2 = 2063 × 4 + 4 = 256

19. (e) The series pattern would be as follows :11 13 16 20 25

+2 +3 +4 +520. (c) The series pattern would be as follows :

608 304 152 76 38

÷2 ÷2 ÷2 ÷221. (d) The series is based on the following pattern:

2 × 3 + 5 = 1111 × 4 – 6 = 3838 × 5 + 7 = 197

197 × 6 – 8 = 1174 ; not 11721174 × 7 + 9 = 82278227 × 8 – 10 = 65806Clearly, 1172 is the wrong number and it should bereplaced by 1174.

22. (a) The series is based on the following pattern :

16 + 12 = 17; not 1917 + 22 = 21

Page 334: yoursmahboob.wordpress.com SBI · yoursmahboob.wordpress.com iii P 101 Speed Tests for SBI Bank Clerk Exam 101 Speed Tests for SBI Bank Clerk Exam is revised and updated edition on

yoursmahboob.w

ordpress.com

68 101 SPEED TEST21 + 32 = 3030 + 42 = 4646 + 52 = 7171 + 62 = 107Clearly, 19 should replaced by 17.

23. (d) The series is based on the following pattern :7 + 9 = 169 + 16 = 2516 + 25 = 4125 + 41 = 66; 6841 + 66 = 10766 + 107 = 173Clearly, 68 should be replaced by 66

24. (c) The series is based on the following pattern4 × .5 = 22 × 1.5 = 3 ; not 3.53 × 2.5 = 7.57.5 × 3.5 = 26.2526.25 × 4.5 = 118.125Clearly, 3.5 should be replaced by 3.

25. (b) The series is based on the following pattern:16 × 0.25 = 44 × 0.50 = 22 × 0.75 = 1.51.5 × 1.00 = 1.5 ; not 1.751.5 × 1.25 = 1.875Clearly, 1.75 should be replaced by 1.5.

26. (c) The series is× 1 + 2, × 2 + 3, × 3 + 4, × 4 + 5, × 5 + 6The wrong number is 18.It should be 6 × 2 + 3 = 15

27. (e) The series is × 1.5The wrong number is 366It should be 243 × 1.5 = 364.5

28. (a) The series is× 6 + 42, × 5 + 30, × 4 + 20, × 3 + 12, × 2 + 6,The wrong number is 3674It should be 1220 × 3 + 12 = 3672

29. (b) The series is (2)3, (3)3, (4)3, (5)3, (6)3, (7)3,The wrong number is 218It should be (6)3 = 216

30. (d) The series is + (7)2, + (6)2, + (5)2, + (4)2, + (3)2

The wrong number is 102.It should be 68 + (6)2 = 104

SPEED TEST 50

1. (b) The pattern of the number series is :353 + 1 = 354354 – 3 = 351351 + 5 = 356356 – 7 = 349346 + 9 = 358

2. (c) The pattern of the number series is :1 + 22 = 1 + 4 = 55 + 23 = 5 + 8 = 1313 + 24 = 13 + 16 = 2929 + 25 = 29 + 32 = 6161 + 26 = 61 + 64 = 125

3. (e) The pattern of the number series is :45 + 1 × 12 = 45 + 12 = 5757 + 2 × 12 = 57 + 24 = 8181 + 3 × 12 = 81 + 36 = 117117 + 4 × 12 = 117 + 48 = 165165 + 5 × 12 = 165 + 60 = 225

4. (e) The pattern of the number series is :17 + 13 = 17 + 1 = 1818 + 23 = 18 + 8 = 2626 + 33 = 26 + 27 = 5353 + 43 = 53 + 64 = 117117 + 53 = 117 + 125 = 242

5. (a) The pattern of the number series is :1 1 14 4 2

+ =

1 1 32 4 4

+ =

3 1 14 4

+ =

1 11 14 4

+ =

3 1? 1 24 4

\ = + =

6. (c) The pattern of the number series is :4 + 1 × 15 = 1919 + 2 × 15 = 4949 + 3 × 15 = 9494 + 4 × 15 = 154

154 + 5 × 15 = 2297. (a) The pattern of the number series is :

1 1 12 2

+ =

1 11 12 2

+ =

1 11 22 2

+ =

\ ? = 3 + 1 132 2

=

8. (d) The pattern of the number series is :101 + 2 = 103103 – 4 = 9999 + 6 = 105105 – 8 = 97

97 + 10 = 1079. (b) The pattern of the number series is :

219 – 3 = 216 = 63

344 – 219 = 125 = 53

408 – 344 = 64 = 43

\ ? = 408 + 33

= 408 + 27 435

Page 335: yoursmahboob.wordpress.com SBI · yoursmahboob.wordpress.com iii P 101 Speed Tests for SBI Bank Clerk Exam 101 Speed Tests for SBI Bank Clerk Exam is revised and updated edition on

yoursmahboob.w

ordpress.com

69SOLUTIONS

10. (a) The pattern of the number series is :7 + 3 = 1010 + 12 (= 2 × 3) = 1616 + 12 (= 2 × 6) = 2828 + 24 (= 2 × 12) = 5252 + 48 (= 2 × 24 ) = 100

11. (a) 1 × 1 = 11 × 2 = 22 × 3 = 66 × 4 = 2424 × 5 = 120

12. (b) 7 – 8 = 1 = 13

16 – 8 = 8 = 23

43 – 16 = 27 = 33

? – 43 = 107 = 43

232 – 107 = 125 = 53

13. (b) 4 × 3 + 1 = 134 × 4 + 1 = 17

4 × 5 + 1 = 2114. (c) 982 – 977 = 5

977 – 952 = 25952 – 827 = 125827 – 822 = 5822 – ? = 25? = 822 – 25 = 797

15. (c) 1 × 8 = 88 × 9 = 7272 × 8 = 576576 × 9 = 51845184 × 8 = 41472

16. (e) The series is as follows:64 + 5 = 69;69 + 5 = 74;74 + 5 = 7954 – 5 = 49;49 – 5 = 44

17. (b) The series is as follows:÷ 2 + 8Hence, ? = 1012 ÷ 2 + 8 = 514

18. (c) The series is as follows:× 1, × 3, × 5, × 7, × 9, × 11Hence, ? = 75 × 7 = 525

19. (a) The series is as follows:

× 12 , × 1, ×

112 , × 2, ×

122 , × 3

Hence, ? = 78 × 122 = 195

20. (a) This is a series of prime numbers :21. (d)

3602 1803 604155

36 12154

÷ 1 + 1 ÷ 2 + 2 ÷ 3 + 3 ÷ 4 + 4

3601

÷ 5 + 5 ÷ 6 + 6154 is written in place of 155.

22. (a)45

12 42 196 6066 4251110054

42 is written in place of 45.23. (a)

108 12 20 42 56302

+ 4 + 6 + 8 + 10 + 12 + 148 is written in place of 6.

24. (e)60

16 24 65 945 5197.521032

× 0.5 × 1.5 × 2.5 × 3.5 × 4.5 × 5.565 is written in place of 60.

25. (d)193

13 25 49 194 385977

+ 6 + 12 + 24 + 48 + 96 + 192194 is written in place of 193

26. (a) The series is based on the following pattern :16 + 12 = 17; not 1917 + 22 = 2121 + 32 = 3030 + 42 = 4646 + 52 = 7171 + 62 = 107Clearly, 19 should replaced by 17.

27. (d) The series is based on the following pattern :7 + 9 = 169 + 16 = 2516 + 25 = 4125 + 41 = 66; 6841 + 66 = 10766 + 107 = 173Clearly, 68 should be replaced by 66

28. (e) 32 16 24 65 210 945 5197.5

×0.5 ×1.5 ×2.5 ×3.5 ×4.5 ×5.529. (a) The series is

– 200, –100, –50, –25, – 12.5, –6.25, ......30. (c) The series is

×1.5, × 2, ×1.5, ×2, ×1.5, ×2, .....

SPEED TEST 51

1. (d) Total value of the quantity sold for item D

= 40 150 12.5 90 100

100 100´ ´

´ ´ = 60 × 11.25 × 100 = ` 67500

2. (e) Average price per kg. of items A, B and C

= ( )17.5 10 7.5

3+ +

= 353 = ` 11.667 » 10.50 (approx)

3. (a) Total value of quantity sold for item E = 15 × 25 × 100 = 37500Total value of quantity sold for item F= 10 × 35 × 100 = 35000Required ratio = 37500 : 35000 = 15 : 14

Page 336: yoursmahboob.wordpress.com SBI · yoursmahboob.wordpress.com iii P 101 Speed Tests for SBI Bank Clerk Exam 101 Speed Tests for SBI Bank Clerk Exam is revised and updated edition on

yoursmahboob.w

ordpress.com

70 101 SPEED TEST4. (e) Total value of the quantity sold for item C

= 45 × 100 × 7.50 = 33750Total value of the quantity sold for item E= 22.5 × 100 × 15 = 33750

\ Required percentage = 33750 100 100%33750

´ =

5. (d) Required price

= 120 17.5 12020 100100 100

´´ ´ ´ = 2400 × 21 = ` 50400

6. (d) Total number of students studying in college H= 51.2 + 40 + 36.5 = 127.7 thousandTotal number of students studying in college K.= 30 + 56 + 25 = 111 thousandRequired difference = (127.7 – 111) thousand= 16.7 thousand = 16700

7. (b) Total number of students studying in all the colleges together= [(51.2 + 40 + 36.5) + ( 65 + 50 + 33)+ (44 + 30 + 60) + 30 + 56 + 25)] thousand(127.7 + 148 + 134 + 111) = 520.7 thousand = 520700

8. (c) Number of students from the faculty of science from collegeH and I together = 40 + 50 = 90Number of students from the faculty of science from collegeJ and K together = 30 + 56 = 86

Required ratio = 90 45 45 : 4386 43

= =

9. (a) Number of students from the faculty of science from collegeI = 50Total number of students from college I = 65 + 50 + 33 = 148

Required percentage = 50 100 33.78%

148´ = » 34% (approx.)

10. (e) Average number of students from the faculty of commercefrom all the colleges together.

= ( )36.5 33 60 25

thousand4

+ + +

= 154.4 thousand4

= 38.625 thousand = 38625

11. (a) Required percentage

=

5 142000 17 100 10072000100

æ ö´ - ´ç ÷è ø ´

´=

2 1420007 100 100

140

´ ´´

= 80 100 57%

140´ »

12. (d) Total number of teachers who teach English and History

together = ( )7 27

2000 680100+

´ =

Total number of teachers who teach Mathematics and Biology

together = ( )14 12

2000100

+´ = 520

Required difference = 680 – 520 = 16013. (e) Total number of Biology and History teachers

= 12 100 40 27 100 202000 2000100 100 100 100

+ -æ ö æ ö´ ´ + ´ ´ç ÷ ç ÷è ø è ø

= 12 140 27 80

2000 2000100 100 100 100

æ ö æ ö´ ´ + ´ ´ç ÷ ç ÷è ø è ø

= 336 + 432 = 76814. (b) Required average

=

( )2000 25 27 12100

3

´ + +

= 1280 420

15. (d) Required ratio =

12 200012100 4 :5

15 2000 15100

´

= =´

16. (e) Required difference = 680 – 258 = 422

17. (b) Required percentage = 550 430 100 27%

430-

´ »

18. (b) Required average = 160 708 550 586 501

4+ + +

=

19. (a) Number of flights cancelled by airline R in 2010 due to

technical fault = 880 60 528

100´

=

20. (e) Required percentage = ( )600 546

100365+

´

= 1146 100 314365

´ = (approx.)

SPEED TEST 52

1. (a) 48 % of 525 + ? % of 350 = 399

Þ 48 ?

525 350 399100 100

´ + ´ =

Þ 25200 + ? × 350 = 399 × 100Þ ? × 350 = 39900 – 25200 = 14700

Þ ? = 14700

42350

=

2. (b)3 4 5? of of of 4907 5 8

=

Þ3 4 5? × × ×4907 5 8

=

Þ ? = 35 × 3 = 105

3. (d) 2? 17 335+ =

Þ ? 289 335+ =Þ ? 335 289 46= - =Þ ? = 46 × 46 = 2116

4. (a) ? = 125% of 560 +22% of 450

Þ125 22? 560 450100 100

= ´ + ´

Þ70000 9900?100 100

= +

Þ ? = 700 + 99 = 799

5. (c) 2 228 5 15 6?

7 256 (13)´ - ´

=+ +

Þ140 90

?49 16 169

-=

+ +

Þ50 25?234 117

= =

6. (b) ? = 18.76 + 222.24 + 3242.15Þ ? = 3483.15

Page 337: yoursmahboob.wordpress.com SBI · yoursmahboob.wordpress.com iii P 101 Speed Tests for SBI Bank Clerk Exam 101 Speed Tests for SBI Bank Clerk Exam is revised and updated edition on

yoursmahboob.w

ordpress.com

71SOLUTIONS7. (d) ? = 784 ¸ 16 ¸ 7

Þ784? 716

= ¸

Þ ? = 49 ¸ 7 = 7

8. (c)3 5

? of 455 of 4567 8

= +

Þ3 5

? ×455 ×4567 8

= +

Þ ? = 195 + 285Þ ? = 480

9. (b) ? = 1.05% of 2500 + 2.5% of 440

Þ1.05 2.5

? 2500 440100 100

= ´ + ´

Þ2625 1100

?100 100

= +

Þ3725

? 37.25100

= =

10. (b) ? = 4900 ¸ 28 × 444 ¸ 12Þ ? = 175 × 37Þ ? = 6475

11. (b) QCost price of (12 belts + 30 wallers) = 8940QCost price of 3 × (4 belts + 10 wallets) = 8940

QCost price of 4 belts + 10 wallets = 8940

3 = 2980

12. (c) Q Cost price of an article = 1850For 30% profit, selling price of this article

= 1301850100

´ = 2405

13. (e) Compound Interest after two years

= 210

8500 1 8500100

æ ö+ -ç ÷è ø

= 11 118500 850010 10

´ ´ -

= 10285 – 8500 = 178514. (a) Let length of the train be x m

Speed of the train be 60 km/h = 5 50

6018 3

´ = m/s

Then, 200 27503

x +=

Þ3( 200) 27

50x +

=

Þ 3x + 600 = 1350Þ 3x = 1350 – 600Þ 3x = 750

Þ 750 250

3x = = m

15. (b) Suppose 16 men can complete the same work in x daysThen, Men days

1016

8x

16 : 10 : : 8 : xÞ 16 × x = 10 × 8

Þ 10 85

16x

´= = days

16. (c)

17. (a) Let the original fraction be xy

= .

\ 200 4300 21

xy

´=

´ Þ 4 3 221 2 7

xy

= ´ =

18. (e) The word SIMPLE consists of 6 distinct letters\ Number of arrangements = 6!= 6 × 5 × 4 × 3 × 2 × 1 = 720

19. (d) Let the present age of A = x and B = y yearsAccording to first condition

7 3 4 28 3 21 4 3 77 4

x x y x yy

-= Þ - = - Þ - =

- ........ (i)

According to second condition9 7 8 72 7 639 8

x x yy

+= Þ + = +

+Þ 7y – 8x = 9 ......... (ii)8 6 147 8 9

23years.

x yy x

y

- =- =

=20. (e) x + (x + 1) + (x + 2) = 1383

Þ 3x + 3 = 1383 Þ 3x = 1380

Þ x = 1380460

3=

Largest number = x + 2 = 46221. (b) 8 × 6.5 = 52

52 × 5.5 = 286286 × 4.5 = 1287.

22. (d) 3 × 14 = 4242 × 12 = 504

504 × 10 = 50405040 × 8 = 40320.

23. (c) 403 – 3 = 400400 – 6 = 394394 – 12 = 382382 – 24 = 358358 – 48 = 310

310 – 96 = 214 .

24. (d) 7 8 4 13 –3 22 –14

+5 –9

–3 –7 –11

Page 338: yoursmahboob.wordpress.com SBI · yoursmahboob.wordpress.com iii P 101 Speed Tests for SBI Bank Clerk Exam 101 Speed Tests for SBI Bank Clerk Exam is revised and updated edition on

yoursmahboob.w

ordpress.com

72 101 SPEED TEST25. (a) 250000 ¸ 4 = 62500

62500 ¸ 5 = 1250012500 ¸ 4 = 31253125 ¸ 5 = 625

625 ¸ 4 = 156.25156.25 ¸ 5 = 31.25.

26. (b) Largest fraction = 78

Smallest fraction = 12

Difference = 7 1 7 4 38 2 8 8

-- = = .

27. (e) (?)0.6 × (?)1.4 = 26 × 104or, ?2 = 2704or, ? = ±52.

28. (a) Perimeter of the square = 84 cmPerimeter of the rectangle = 28 cmPerimeter of the rectangle = 2(1 + b)or, 2(8 + b) = 28 cmor, b = 14 – 8 = 6 cm.\ Breadth of the rectangle = 6 cm

Side of the square =844

= 21 cm

Difference = 21 – 6 =15 cm.29. (e) Perimeter of the rectangle = 42 m

2(l + b) = 42 mor, l + 8.5 = 21mor, l = 12.5 m.Area of the rectangle = 12.5 × 8.5 = 106.25 sq.m.\ Area of the circle. = 106.25 sq.m.

30. (d) Let the positive number be x.

Then, 5x 3x

504.6100 100

´ =

5 3x x 504.6

100 100\ ´ ´ ´ =

or, 2 504.6 100 100x

15´ ´

=

\ x = 580.31. (b) Two women alone can complete a piece of work in 16

days.\ Four women can complete the same work in 8 days.Since 12 children can complete the work in

4 8 4 8 88 4 4

´ ´= =

- days.

\ Four children can complete the work in 12 8 24

=

days.32. (d) Let the four odd consecutive numbers be x, x + 2, x + 4

and x + 6.Also, A = x, B = x + 2, C = x + 4 and D = x + 6.\ 4x + 12 = 4 × 40or, 4x = 160 – 12 = 148

or, 148

x 374

= =

\ Numbers are A = 37, B = 39, C = 41, D = 43\ Product of B and D = 39 × 43 = 1677.

33. (a) 2.31 km = 2.31 × 1000 = 2310 mTotal number of days = 3 × 7 = 21

\ Distance covered by Anu each day = 2310

11021

=

m.

34. (b) 43.2 m/hr = 543.2

18´ = 12 m/s

Total distance covered = 12 × 80 = 960 m.Perimeter of the square = 960 m.Side of the square = 240 m.Area = (240)2 = 57600 sqm.

35. (b) Let the number of children be x.Now, according to the question

4800100 (x 4) 4800

xæ ö- + =ç ÷è ø

or, 48

1 (x 4) 48x

æ ö- + =ç ÷è øor, (48 – x)(x + 4) = 48xor, x2 + 4x – 192 = 0or, (x + 16)(x – 12) = 0\ x = 12 sweets

Number of students = 4800

40012

= .

36. (c) Sneha's monthly income = 342000

2850012

=

\ Akruti's monthly income = 28500 116 34800

95´ =

Akruti's annual income = 417600.

37. (b) Time taken by the truck = 256

8 hr.32

=

Distance covered by the car = (256 + 160) = 416 km.Time = 8 hr.

\ Speed of the car = 416 52 km / hr.8

=

38. (a) Required percentage = 663 612 100 5%

1020-

´ = .

39. (c) Sum of the heights of all the girls = 148 × 21 = 3108 cmSum of the heights of the teacher and all the girls = 149× 22 = 3278 cm.Teacher's height = 3278 – 3108 = 170 cm.

40. (a) Radius = 17.5 cm.

Area of the circle = 22

17.5 17.5 962.57

´ ´ = sq cm.

Page 339: yoursmahboob.wordpress.com SBI · yoursmahboob.wordpress.com iii P 101 Speed Tests for SBI Bank Clerk Exam 101 Speed Tests for SBI Bank Clerk Exam is revised and updated edition on

yoursmahboob.w

ordpress.com

73SOLUTIONS

SPEED TEST 53

1. (a)2. (c) They are responsible for national disintegration.3. (d) They are harmful to national integrity.4. (b) India was forged into a nation on account of a common

culture evolved over the centuries.5. (a) The author wants India to remain as an ideal nation

and the passage has certainly a message behind it.6. (e) 7. (b) 8. (a)9. (e) India’s insurgence stood for gaining freedom by

adopting the path of non-violent struggle.10. (c) 11. (d) 12. (e)13. (a) 14. (e)15. (b)16. (a) Option (c) can be rejected as it is out of the context.

Option (d) is also not true as they like to invest abroadthan in India. Option (e) is true in parts. Only optionwhich is in sync with major portion of passage is (a).

17. (a) Other options are not mentioned in the passage.18. (c) Last part of the passage chiefly describes it. Other

options show these sectors in positive light which isagainst passage content.

19. (b) Option (B) is true in its completeness, encompassingthe whole of the passage. But other options are true inparts only. Option (E) is false in parts.

20. (c) Both (1) and (3) are directly mentioned in passage.Statement 2 is just not true.

21. (c) Meaning of Clout is force.22. (e) Marked means distinguished and different.

Imperceptible is that cannot be distinguished orpercepted.

23. (d) Option (D) can be related to the experts as mentionedin the passage.

24. (d) Option (D) has plenty of illustrations, references andstructure to support it in the passage. Other optionslike E and B are true but not the central theme of thepassage. Option (A) and (C) are not true.

25. (d) PROMOTES means to advocate a particular cause.26. (e) A rise from the 4% level to 7% says that there is rise in

education in Egypt.27. (a) All of the options are mentioned in one or other part of

the passage.28. (c) First line of the passage is self explanatory.29. (b) Only option which can be linked with the passage is 1.

Their people will agitate for greater political freedom,culminating in a shift to a more democratic form ofgovernment.

30. (c) Option A is wrong as this is not intended from the useof this phrase. Last part of passage has this phraseand poor-rich divide has been discussed their in votingpattern or why rich people even educated do not vote.Option (D) is out of the context. Option E can also berejected as corruption is not the issue here.

SPEED TEST 54

1. (b) 2. (b) 3. (d) 4. (d)5. (c) 6. (c) 7. (e) 8. (d)

9. (e) 10. (c) 11. (b) 12. (b)13. (a) 14. (d) 15. (d)16. (c) The author wants us to stop debating and implement

policies.17. (c) Stated in the first paragraph.18. (d) Uncertainty about payment is mentioned, hence option

(d).19. (b) Refer to the second paragraph. “State governments

have not implemented agreed plans to ensurerepayment when due...”

20. (a) All the factors are mentioned in the passage.21. (a) Refer to the third paragraph. “The Delhi model has

worked. But it receives no public support.”22. (d) Clearly, populist measures would go against financial

wellbeing.23. (d) It is stated in the passage that the enforcement of the

reforms was inadequate.24. (c) Eminent British economists and political scientists have

strongly attacked the tradition of budget secrecy.25. (e) It leads to the control of public expenditure in order to

set realistic taxation implications.26. (b) He has presented the example of both the open budget

system and the secret budget system, practised byvarious countries and has looked into all their aspects.

27. (d) 28. (e)29. (a) Sir Richard Clarke was the originating genius of nearly

every important development in the British budgetingtechniques during the last two decades.

30. (b)

SPEED TEST 55

1. (b) 2. (e) 3. (c) 4. (c)5. (b) 6. (a) 7. (b) 8. (d)9. (d) 10. (d) 11. (e) 12. (c)13. (c) 14. (a) 15. (b) 16. (c)17. (d) 18. (b) 19. (e) 20. (a)21. (d) 22. (e) 23. (e) 24. (b)25. (a) 26. (b) 27. (d) 28. (b)29. (b) 30. (a)

SPEED TEST 56

1. (b) The word ' vicarious' means 'Endured or done by oneperson substituting for another' which is nearest inmeaning to phrase 'Not experienced personally' whichis option (b), therefore, (b) is the correct answer. Theother 3 words have following meanings: Ambitious -strongly desirous; Nostalgic - homesickness; Vindictive- vengeful.

2. (b) The word 'craven' means 'cowardly, contemptibly timidpusillaninuous which is nearest in meaning to cowardly'which is option (b), therefore, (b) is the correct answers.

3. (d) The word ' Tepid' means moderately warm; Luke warmwhich is nearest in meaning to 'Lukewarm' which is option(d), therefore, (d) is the correct answer. The other 3 wordshave following meanings: Irreversible - uncapable ofbeing changed; Causing fatigue - something that leadsto mental exertion; Fast moving - speedy.

Page 340: yoursmahboob.wordpress.com SBI · yoursmahboob.wordpress.com iii P 101 Speed Tests for SBI Bank Clerk Exam 101 Speed Tests for SBI Bank Clerk Exam is revised and updated edition on

yoursmahboob.w

ordpress.com

74 101 SPEED TEST4. (c) The word 'Tenuous' means thin or slender in form which

is nearest in meaning to 'slender' which is option (c),therefore, (c) is the correct answer. The other three wordshave followings meanings: Contentious - quarrelsome;Dark - little as no light; Malfunctioning - to fail tofunction.

5. (a) The word 'probity' means integrity or uprignt ness,honesty' which is nearest in meaning to integrity whichis option (a), therefore, (a) is the correct answer. Theother 3 words have following meanings: Impudence -insolence; Profane - irreligious; Preface - anintroductory past.

6. (b) The word 'Musty' means absolute, out dated or stalefood' which is nearest in meaning to 'stale' which isoption (b), therefore, (b) is the correct answer. The other3 words have following meanings: Certainty - somethingcertain; Modern - not ancient or remote; Mysterious -implying or suggesting a mystery.

7. (b) The word 'Alleviate' means 'to diminish or lessen ' whichis nearest in meaning to 'lessen' which is option (b),therefore, (b) is the correct answer. The other threewords have following meanings: To release - to freefrom confinement; To deprive - to remove from thepossession; To deceive - delude, to be unfaithful to.

8. (c) Morose means ‘depressed and pessimistic’. Some moresynonyms are : Cantankerous, gloomy, in a bad mood,mournful, moody, splenetic.

9. (c) Protagonist ‘means person who takes the lead or thecentral figure of the narrative’. Some more synonymsare: Central character, Prime mover, Exponent, Hero,Exemplar, Mainstay.

10. (c) Factitous means ‘unnatural, false or artificial’ so in thegiven option artificial would be the right synonym offactitious. Other synonyms : false, artificial, sham.

11. (d) The word ‘hospitable’ means genial, welcoming,congenial, friendly, convivial, but welcoming is the mostsuitable word.

12. (a) The word ‘scarcely’ means hardly, barely, almost not,only just.

13. (c) The word ‘disdain’ means contempt, scorn, disrespect,dislike. From the given options ‘hate’ is the most suitableword.

14. (a) The word ‘absurd’ means stupid not logical andsensible which is nearest in meaning to senseless.

15. (a) The word ‘philanthropy’ means the practice of helpingthe poor and those in need especially by giving money.Hence generosity (willing to give somebody money,gifts etc) is nearest in meaning to it.

16. (a) The word ‘mutual’ is used to describe feelings that twoor more people have for each other equally. Hence,reciprocal is similar word in meaning to it.

17. (d) The word ‘weird’ means very strange or unusual anddifficult to explain which is similar in meaning tounnatural.

18. (d) The word ‘pessimistic’ means bleak, distrustful,hopeless, depressed etc. which is nearest in meaningto ‘not hopeful’.

19. (c) The word ‘analogous’ means like, equivalent, relatedetc. which is nearest in meaning to ‘similar’.

20. (b) Thw word ‘exaggerate’ means amplify, inflate, elaborateetc. which is similar in meaning to ‘overstate’.

21. (d) The word ‘evident’ means obvious, clear, tangible,distinct etc. which is nearest in meaning to ‘quite clear’.

22. (b) The word ‘penalize’ means inflict a penalty on, whichis quite similar in meaning to ‘punish’.

23. (c) The word ‘remedial’ means aimed at solving a problemwhich is similar in meaning to ‘corrective’.

24. (a) The word ‘trivial’ means not worth considering,insignificant etc., which is similar in meaning to‘unimportant’.

25. (a) The word ‘ Incredible’ means beyond belief,unbelievable, unimaginable etc. which is similar inmeaning to ‘hard to believe’.

26. (d) The word ‘frenetic’ means ‘wildly excited or active’,energetic, hectic, fast and furious etc.

27. (c) The word ‘elicit’ means draw out, extract, obtain,provoke etc.

28. (b) The word ‘Lucrative’ means gainful, remunerative, well-paid which is most similar in meaning to ‘profitable’.

29. (c) Vivid means presented in very clear/distinct way. Fromthe options detailed is the other option which canreplace but only if 'clear' as option is not available.Also 'detailed' can be confusing as well. So it is not thebest choice. Ambiguous is opposite for the Vivid.'Categorical' is without any condition.

30. (d) Deplore means to disapprove /to condemn /to regret.Deprive means lack of something. Implored is to begimpatiently. Deny is to refuse. So clear choice is'regretted' as this is direct meaning of the word.

31. (c) Vindictive is revengeful. Other options like violent andcruel can be actions of revenge but not the right answeras synonym. Irritable is not correct clearly.

32. (a) Pragmatic is practical. Pragmatic is replacing 'practical'in English usage for common use.

33. (d) Soporific means sleep inducing. It can not be replacedwith other options. There is not much use of context inthese types of words which have direct meaning withso clear options.

34. (b) Evoke means to 'call for' or 'to rise by a reason'. Fromthe options, (b) is the direct meaning of the word.

35. (a) Abate is to reduce but not to completely remove. SoVanished as answer is rejected. Increased is just theopposite in sense. Stabilized is to remain at same statusor as it is.

36. (c) Insulting behavior is insolent behavior. Violent behaviorcan not be counted as disrespectful it is extremeone.Yes; disrespect can be a reason for violent behavior.

37. (d) Mendacious is intentionally untrue or false. Optionsare not confusing either.

38. (c) Induces means persuades, insist and provoke.39. (d) Authentic means genuine, real, valid and dependable.40. (c) Scarcity means insufficiency, shortage, lack, dearth or

paucity.

Page 341: yoursmahboob.wordpress.com SBI · yoursmahboob.wordpress.com iii P 101 Speed Tests for SBI Bank Clerk Exam 101 Speed Tests for SBI Bank Clerk Exam is revised and updated edition on

yoursmahboob.w

ordpress.com

75SOLUTIONS

SPEED TEST 57

1. (a) 2. (e) 3. (d) 4. (e)5. (c)6. (b) The word ‘obligatory’ means compulsory, mandatory,

required whose opposite is optional or voluntary.7. (d) The word ‘obscure’ means unclear whose opposite is

clear.8. (a) The word ‘repulsive’ means repellent, nasty whose

opposite is attractive.9. (a) The word ‘vital’ means essential, urgent while trivial

means insignificant.10. (d) The word ‘inhibit’ means hinder, hold back whose

opposite word is encourage or promote.11. (c) The word ‘exploit’ means treat unfairly, utilize, misuse

or take advantage of which is opposite in meaning tosupport.

12. (b) The word ‘sharp’ means razor-edged, fierce, shrill etc.which is opposite in meaning to blunt.

13. (d) The word ‘condemn’ means censure, criticize, blameetc. which is opposite in meaning to praise.

14. (d) The word ‘reluctant’ means timid, resistant, opposedetc. which is opposite in meaning to eager.

15. (a) The word ‘scarcity’ means shortage, lack, paucity,meagreness, dearth etc. Its opposite word is ‘plenty’ inthe given options.

16. (a) The word ‘bleak’ means bare, exposed, denuded, dimetc which is quite opposite in meaning to ‘bright’.

17. (d) The word ‘stern’ means serious, austere, unforgiving,harsh etc. Its opposite word will be ‘forgiving’.

18. (a) The word ‘superficial’ means shallow, casual, hasty,trivial, silly, inane etc. which is opposite in meaning to‘profound’.

19. (a) The word ‘elegance’ means grace, stylishness, charm,cleverness which is opposite in meaning to ‘balance’.

20. (d) The word ‘coarse’ means ugly, rough, ill-mannered,rude, crude etc. which is opposite in meaning to ‘soft’from the given options.

21. (c) The word ‘extravagant’ means prodigal, lavish, costlyetc. which is opposite in meaning to the word‘economical’.

22. (b) The word ‘diligent’ means industrious, hard-working,rigorous etc. which is opposite in meaning to the word‘lazy’.

23. (c) The word ‘fictitious’ means fake, spurious etc. Itsopposite word will be genuine or real.

24. (b) The word ‘generous’ means liberal, charitable,benevolent etc. Hence, uncharitable is the oppositeword.

25. (b) Ignored is to overlook /to avoid /not considered/setaside. If something is ignored then it can be removed.So 'remove' gives the same sense as 'ignored'.

26. (c) Lethargic is slow/lazy/inactive/dull. Active is antonym.Aggressive is second best option. Hungry is to make aguessing student go on sense of the sentence anddeviate from the right answer.

27. (d) Hazardous is harmful/perilous/marked by danger/risky;opposite for it is 'safe'. Other options are not confusingso it is easy to answer.

28. (a) Enrich is making richer /more valuable/moremeaningful/more nourished. The opposite of this wordis to make poor - impoverished.

29. (a) Credible is loyal or believable or something in whichfaith can be reposed. Incredible is just opposite of theword. Although from the options Believable is closebut not the best one to choose. If credible is not in theoptions then this is the best among the rest.

30. (a) Inert is without motion. From the options active is clearchoice. As Lazy is similar in meaning.While Resolute(Firm) is irrelevant in context. Strong does not meanactiveness or motion. He has very strong emotions forher.

31. (a) Affluent is extra rich. If richness is a scale then Affluentis on one end and poor is on the other. Ordinary lies inbetween the two. Backwardness is just the result ofpoorness. So 'poor' is the best option.

32. (d) Energetic is full of activity and lethargic meansinactivity/slowness/ laziness/dull. Gloomy is sadness.

33. (b) Here secure means to make safe and precarious is havinga sense of insecurity. So it can be a proper antonym forthe word from the available options.

34. (d) 'Various' word derives from the variety. Variety is rangeof choices and similarity is opposite of it.

35. (b) Recession means depression, slump, downturn,collapse, decline so its antonym can be inflation, boom,rise increase.

36. (d) Denied means deprive, starve, unused, shorn of so itsantonym can be affirmed, avowed, confirmed, declared,stated.

37. (c) Collective means together, mutually, group, joint orcooperative so its antonym can be individual, single oralone.

38. (b) Abducted means kidnap, snatch or seize so its antonymcan be set free, release or freedom.

39. (c) Enhanced means increase, augment, boost or amplifyso its antonym can be decreased, reduce or lessen.

40. (b) Condemn. Adore means esteem, respect and admire soits antonym can be disapprove, criticize, revile, attackor condemn.

SPEED TEST 58

1. (c) shown2. (d) drawn to

Certain Verbs, Nouns, Adjectives, and Participles arealways followed by certain Prepositions.

3. (d) wereThe Past Subjunctive ‘were ‘is used after tile verb wish,to indicate a situation, which is contrary to fact or unreal;as in, I wish I were a millionaire.

4. (c) had seen me5. (c) concur with6. (b) swum

The Past Participle “swum” is to be used and not thepast tense “swam”.The Past Participle represents a completed action or stateof the thing spoken of.

7. (b) The sentence gives a condition, so the principle clausewill use ‘would’ not ‘will’.

Page 342: yoursmahboob.wordpress.com SBI · yoursmahboob.wordpress.com iii P 101 Speed Tests for SBI Bank Clerk Exam 101 Speed Tests for SBI Bank Clerk Exam is revised and updated edition on

yoursmahboob.w

ordpress.com

76 101 SPEED TEST8. (b) Since there is a comparison between two, a comparative

degree verb must be used.9. (a) Gravity is the singular subject which will have the present

indefinite verb i.e., verb + s/es form.10. (b) Since a comparison is being made there must be a subject

in both parts of sentence devided by than. There mustbe ‘that’ in the second part therefore, (d) is incorrectbecause of incorrect usage of article ‘the’.

11. (b) There is no need for ‘who’ or ‘which’ (both of which areincorrect ‘whom’ will be the right pronoun) the clausefollows the subject ‘person’ directly and need not beconnected through a pronoun.

12. (d) It is a passive voice sentence and the phrase before‘that’ should be a complete passive form of verb and nota gerund.

13. (d) ‘of’ is the preposition used with frightened.14. (c) Capitalist society is the singular subject and the statement

made is a dictum, so pesent infinitive tense will be used.15. (a) The use of the word here is as a ‘noun’ not as a verb thus

damage will be the right answer.Tip : A sentence of form this can cause, will always befollowed by a noun.

16. (d) The right expression is ‘conditions necessary for’, sincethe gap is followed by, this. ‘complete’ should be used inthe noun form and not adjective so it will be ‘completionof this’.

17. (b) The right use is as + adjective + as18. (a) There cannot be a subject in the first part of the sentence

as the same subject is given on the second part so (c)and (d) are eliminated. To study is the infinitive verbwhich will not be used here, because it suggests theaction which is the affect and not the cause. e.g. ‘To gainsomething you have to lose something else’.By studying is the right answer because this gives thecause for the verb in the latter part of the sentence.

19. (b) In the given sentence there is no subject or verb, so theadded phrase should be of the form subject + infinitiveverb.

20. (c) Modals such as must or should cannot be used with‘hope’.

21. (a) There must be the preposition ‘to’ to connect the verb‘tend’ with the noun ‘result’, (b) is not correct becausefirst form of verb should be used and not continuoussince its a simple statement made suggesting no actiontaking place at the current moment.

22. (b) The subject of the sentence is candidate, but in the firstpart there is no subject, it should be passive or have anactive subject. (b) is the only option with passive verb.

23. (d) ‘because’ cannot be used as the conjunction in thissentence because for the sentence to be complete itshould be followed by an ‘of’, i.e., because of their, sincethis is not the option ‘of their’ is the most appropriateuse.

24. (c) had better see25. (c) had known26. (c) than one’s speed.

Here ‘than’, used as a preposition, as;I need more than fifty rupees for this magazine.

27. (b) which

28. (b) would not have been misunderstood.29. (d) reformed.30. (c) When he ran across the road

The past tense is needed here as the sentence indicates.

SPEED TEST 59

1. (c) 2. (a) 3. (e) 4. (e) 5. (c)6. (b) 7. (c) 8. (e) 9. (d) 10. (e)11. (c) 12. (b) 13. (b) 14. (d) 15. (b)16. (c) 17. (b) 18. (c) 19. (e) 20. (c)21. (b) 22. (b) 23. (e) 24. (c) 25. (c)26. (c) 27. (d) 28. (e) 29. (c) 30. (e)31. (d) 32. (c) 33. (e) 34. (b) 35. (d)

SPEED TEST 60

1. (d) 2. (c) 3. (d) 4. (a)5. (a) 6. (c) 7. (c) 8. (e)9. (a) 10. (a) 11. (b) 12. (e)13. (d) 14. (d) 15. (e) 16. (e)17. (a) 18. (c) 19. (b) 20. (e)21. (b) Engaged is the word which fits in both the sentences.

In the first sentence it means that couple has made onagreement or a pledge to get married and in statementII it means occupy the attention or efforts of (a personor persons).

22. (b) Option (b) is correct. Application in the first statementmeans a written request for employment. In the IIstatement application means the act of putting to aspecial use or means the act of putting to a special useor purpose.

23. (e) hearingStatement I - an instance or a session in which testimonyand arguments are presented, especially before anofficial, as a judge in a lawsuit.Statement II - the act of perceiving sound.

24. (a) resortStatement I - to have recourse for use help, oraccomplishing something of ten as a final availableoption or resource.Statement II - a place to which people frequently orgenerally go for relaxation or pleasure.

25. (d) Statement I - salaryStatement II - to carry on (a bottle, war, conflict,argument, etc).

SPEED TEST 61

1. (b) up2. (d) up with3. (b) ‘with4. (b) of5. (c) on6. (c) in———with7. (c) on for8. (b) within——to9. (c) for10. (a) from11. (c) for

Page 343: yoursmahboob.wordpress.com SBI · yoursmahboob.wordpress.com iii P 101 Speed Tests for SBI Bank Clerk Exam 101 Speed Tests for SBI Bank Clerk Exam is revised and updated edition on

yoursmahboob.w

ordpress.com

77SOLUTIONS12. (b) in ——— around13. (a) to14. (c) off15. (d) Though —— in16. (d) to17. (c) Although — from18. (d) among19. (b) in20. (d) in21. (b) towards22. (c) to23. (c) concur with24. (d) of25. (b) on26. (c) with27. (d) to28. (c) upon29. (d) of30. (a) at——in——at

In is used with names of countries and large towns, at ismore often used when speaking of small towns andvillages.

31. (b) In the perfect continuous tense only ‘for’ and ‘since’are used. ‘since’ is used to suggest a particular timein past and ‘for’ is used to suggest a time period, ‘along time’ suggest a time period so ‘for’ will be theanswer.

32. (b) ‘With’ is used with overwhelmed33. (b) Preposition ‘about’ is used before the word

‘prohibition’.34. (b) This will be the right preposition.35. (b) The right usage is ‘comes to mind’.

SPEED TEST 62

1. (a)2. (b)3. (a)4. (b)5. (d)6. (d)7. (c)8. (a)9. (e)10. (c)11. (e) Here, too is used as emphatic word. Lata was so scared

that she could not go home alone. Hence, no correctionis required.

12. (b) The structure of sentence is subject + was/were + thirdform of verb + object. Thus, Riya was dressed to kill.

13. (c) The given sentence is the statement of simple pasttense. Hence it should be ‘worried’ instead of worries.

14. (a) The phrase ‘let off’ means to give them only a lightpunishment.

15. (d) It should be ‘took’ instead of ‘take’.16. (d) Replace to by ‘too’ to make a correct phrase ‘a bit too

fast’ which means slightly or to a small extent.17. (e) The word think about refers to consider. Hence no

correction is required.

18. (a) Replace leisure by the adverbial word leisurely19. (b) The phrase ‘gift of the gab’ means ‘to have a talent for

speaking’.20. (c) The phrase ‘the order of the day’ means common,

popular or suitable at a particular time. For example :Pessimism seems to be the order of the day.

21. (d) started carrying out their plan.22. (a) started the discussion.23. (c) A succession of unexpected events.24. (c) be implemented next month25. (b) Idiom put your foot down means : to be very strict in

opposing what somebody wishes to do.26. (d) In most probability27. (a) ignorant about28. (a) in communication with29. (d) discovered by chance30. (b) made a record.

SPEED TEST 63

1. (b) ‘Sent’ is the third form of verb ‘send’ in passive voice.2. (e)3. (a) Replace ‘though’ by ‘As’ to express cause and effect

in the sentence.4. (d) Use ‘and I answered’ properly to express cause and

effect in the sentence.5. (c) Apply the adverb ‘highly’ before the adjective ‘paid’.6. (a) Replace adverb ‘necessarily’ by adjective ‘necessary’.7. (e)8. (c) Use conjunction ‘and/therefore’ in place of ‘yet’.9. (b) ‘Any’ shows uncertainty of selection. So, use ‘one’

instead of ‘any’.10. (d) Use third form of verb ‘signed’ in passive voice.11. (a) Here, Nuclear waste will still remain/be .... should be

used.12. (c) Here, whose attic had been should be used. Whose is

used to say which person or thing you mean.Look at the sentence :It is the house whose door is painted red.

13. (c) Here, subject i.e. A public safety advertising campaignis singular. Hence, hopes to draw attention ... shouldbe used here.

14. (b) Look at the structure of the sentence in Passive Voiceof Past Simple.Subject + was/were + V3 (Past Participle)Hence, awarded to the most .... should be used.

15. (b) Here, a time of reawakening .... should be used.16. (c) Here, and guidance to the mind tortured by doubt

should be used.Look at the sentence:All activities take place under the guidance of anexperienced tutor.

17. (d) Here, Past Simple i.e. the educated class did not supporthim .... should be used as the sentence shows pasttime.

18. (a) Here, Depletion (Noun) of the Ozone layer .... shouldbe used.

Page 344: yoursmahboob.wordpress.com SBI · yoursmahboob.wordpress.com iii P 101 Speed Tests for SBI Bank Clerk Exam 101 Speed Tests for SBI Bank Clerk Exam is revised and updated edition on

yoursmahboob.w

ordpress.com

78 101 SPEED TEST19. (a) Here, Most of the people who should be used. Who is

used to show which person or people you mean.Look at the sentence:The people who called yesterday want to buy thehouse.

20. (b) Here, in reducing human suffering .... should be used.21. (d) Since the sentence begins in past tense. It should end

in past tense also since it the subject is singular theverb will also be singular. Hence “them is missing”should be “them was missing”.

22. (a) The teacher that is the subject is singular so “were”will be replaced with “was”.

23. (b) “Son for help her” should be “son to help her”.24. (b) The verb “ask” will be in the past tense it will become

“asked”.25. (c) “Saving” will be replaced with past tense of the verb

“Save” that is saved because it is preceded by had.26. (a) Change ‘decline of’ to ‘decline in’.27. (b) Delete ‘for’ after order.28. (c) Place on before ‘land’.29. (c) Delete ‘to’ before Chennai.30. (d) Replace ‘for’ by ‘to’.31. (c) Change ‘look for’ by ‘look after’.32. (a) Delete for after await.33. (c) debarred from sending is correct.34. (a) Do not use of with despite. Despite means inspite of.35. (a) Change it to ‘yielded to’36. (b) dying of hunger is correct.37. (c) Change than by ‘to’.38. (d) Change ‘by’ to ‘for’.39. (b) Change ‘to stay in’ by ‘rather than stay in’. She prefers

to write rather than to speak on telephone.40. (c) Change ‘startled by’ to ‘startled at’.41. (b) Change ‘with’ to ‘from’.

SPEED TEST 64

1. (d) Replace ‘their’ by ‘its’. The banker’s association –collective noun – so pronoun ‘its’ singular.

2. (b) Replace ‘are’ by ‘is’. ‘five quintals’ refers a definitequantity (as collective noun) so verb will be singular.

3. (a) Replace ‘have’ by ‘has’. Dickens is the name of a person.4. (d) Insert ‘are’ after ‘hopes’. Hopes is plural, so verb will

be plural.5. (a) Place be after may.6. (d) : Remove' a matter'7. (c) : Remove' for preparing'8. (b) : Either' must' or' 'have to' alone should be used9. (c) : Raplace' were' by 'are' .10. (c) : Replace' indefinite' by , indefinitely' .11. (d) : Replace' days' by daily' .12. (d) : Replace' needs' by' need'13. (d) : Replace' of' by in'14. (c) : Replace' appreciating' by' appreciated15. (a) : Remove' being'16. (b) : Replace 'are' by 'is'17. (d) : Remove 'independent'18. (d) : Replace 'does' by 'did'

19. (d) : Replace 'for' by 'of20. (e) : no error21. (c) : Replace 'enough' by 'any'22. (c) : Remove 'not'23. (a) : Add 'having' after 'after'24. (c) : Replace 'was able to ' by 'could'25. (d) : Replace 'for' by 'to'26. (a) : Replace 'had' by 'would have'27. (c) : Replace 'pumping' by 'pump'28. (a) : Replace 'ential' by 'entails'29. (b) : Replace 'sang' by 'sung'30. (b) : Replace 'at' by 'with'31. (c) : Replace 'would have' by 'had'32. (c) : Replace 'those' by 'which'33. (a) : Replace 'for' by 'two'34. (d) : Add 'they' before 'assume'35. (b) : Add 'which are' before 'available'36. (a) : Replace 'being intelligent' by 'intelligence'37. (a) : The correct form is 'The judges not onlyacquitted38. (e) : No error39. (b) : The correct form is ' different people diferentjobs .'40. (d) : Replace 'promising' by 'promissed'. 366 (c): Replace

'done' by 'made'

SPEED TEST 65

1. (a) 2. (d) 3. (b) 4. (c) 5. (d)6. (c) 7. (d) 8. (a) 9. (d) 10. (b)11. (d) 12. (c) 13. (c) 14. (d) 15. (b)16. (a) 17. (d) 18. (b) 19. (c) 20. (c)21. (e)22. (d) Use ‘divided’ instead of ‘dividend’.23. (a) Use ‘agreed’ properly for ‘consented’.24. (b) Use ‘file’ instead of ‘fill’.25. (c) Apply ‘funds’ for ‘investment’.26. (e)27. (b) Use ‘able’ instead of ‘enable’.28. (d) Use ‘overcome’ instead of ‘resort’.29. (b) Use ‘spread’ instead of ‘spend’.30. (a) Use ‘spent’ properly for ‘initiated’.31. (c) The correct spelling is souvenirs which mean memento,

reminder, memorial etc.32. (a) It should be wasted instead of vested.33. (d) It should be griping which means a sudden strong pain

in stomach.34. (e) No correction is required.35. (b) The correct spelling is charming.36. (d) It should be ‘here was her son’s future’.37. (c) The correct spelling should be ‘triumphant’.38. (a) The word suppressed should be replaced by revealed

or leaked in the sentence.39. (b) It should be ‘handed over’ which means the act of

moving power or responsibility from one person toanother.

40. (a) The correct spelling is difficult.

Page 345: yoursmahboob.wordpress.com SBI · yoursmahboob.wordpress.com iii P 101 Speed Tests for SBI Bank Clerk Exam 101 Speed Tests for SBI Bank Clerk Exam is revised and updated edition on

yoursmahboob.w

ordpress.com

79SOLUTIONS

SPEED TEST 66

1. (b) 2. (d) 3. (d) 4. (b)5. (c) 6. (d) 7. (a) 8. (a)9. (a) 10. (c) 11. (d) 12. (d)13. (c) 14. (d) 15. (c) 16. (d)17. (a) 18. (c) 19. (c) 20. (a)21. (a) 22. (b) 23. (b) 24. (a)25. (d) 26. (b) 27. (d) 28. (c)29. (c) 30. (b)

SPEED TEST 67

Sol: (Qs. 1-5): Clearly C must be followed by D, which must befurther followed by the E as E reitrates the housing shortage andsays that the real deficit will be even higher. D and E provide thestatistical proof of the staggering task mentioned in C. So thisleads us to two options (b) and (d). Among them (b) seems to bemore appropriate as B again emphasises on but is being said is Aand also that B cannot be the concluding statement of theparagraph. Hence, ABCDE gives the correct arrangement.1. (a) 2. (b) 3. (c) 4. (d)5. (e)Sol. For (Qs.6-10) : The arrangement EABDC is correct.The paragraph is clearly taking about Goa state and hence E hasto be opening sentence is the paragraph. This is followed by Awhere the phrase, ‘is an impressive case in point’, which is anexample of what is being said in E. A is followed by E as ‘a similaragitation’ mentioned in B refers to the public activism mentionedin A. B is further followed by D and C.6. (e) 7. (a) 8. (b) 9. (d)10. (c)Sol For. (Q.11-15): The required arrangement is ABDCE.A is the opening sentence as is clear from the given options. A isfollowed by B as ‘the proposal’ mentioned in B is reffering towhatever has been talked in A. B is followed by D as D continuesto talk about the response mentioned in B. Also note that Bmentions a 2 : 1 response against the proposal which is also clearby the 68 : 31 mentioned in D. D is followed by C and C is followedby E. The ‘other immigration organizations’ mentioned in E clearlystates that the previous sentence must have a statement fromsome other organization, which is the immigration lawyers asmentioned in C.11. (a) 12. (b) 13. (d) 14. (c)15. (e) 16. (e) 17. (b) 18. (e)19. (a) 20. (c) 21. (a) 22. (b)23. (c) 24. (d) 25. (d) 26. (d)27. (a) 28. (a) 29. (a) 30. (d)

SPEED TEST 68

1. (b) 2. (b) 3. (a) 4. (b)5. (c) 6. (a) 7. (d) 8. (d)9. (d) 10. (c) 11. (c) 12. (b)13. (b) 14. (a) 15. (b) 16. (a)17. (b) 18. (d) 19. (c) 20. (b)21. (b) 22. (b) 23. (a) 24. (d)25. (b) 26. (b) 27. (a) 28. (d)29. (a) 30. (d) 31. (b)

32. (e) Piece of cake means something easy to do. Therefore,option (e) is the correct choice.

33. (b) Took to one’s heels means to run away. Therefore,option (b) is the correct choice.

34. (e) To pledged means to make a promise. Therefore, option(e) is the correct choice.

35. (d) Crying need means a definite or desparate need forsomeone or something.Therefore, option (d) is the correct choice.

36. (d) Light upon means to arrive at something by chance.Therefore, option (d) is the correct choice.

SPEED TEST 69

1. (d) 2. (c) 3. (a) 4. (a)5. (b) 6. (c) 7. (b) 8. (b)9. (e) 10. (a) 11. (b) 12. (e)13. (b) 14. (a) 15. (c) 16. (b)17. (e) 18. (b) 19. (a) 20. (e)21. (c) 22. (b) 23. (d) 24. (a)25. (e) 26. (a) 27. (d) 28. (b)29. (c) 30. (a) 31. (c) 32. (d)33. (e) 34. (b)

SPEED TEST 70

1. (b) 2. (d) 3. (e) 4. (b)5. (c) 6. (c) 7. (a) 8. (d)9. (c) 10. (b) 11. (d) 12. (a)13. (e) 14. (b) 15. (a) 16. (b)17. (d) 18. (e) 19. (c) 20. (c)21. (d) into 22. (a) around23. (b) translating 24. (c) practice25. (c) chance 26. (d) provided27. (a) other 28. (e) off29. (e) hard 30. (b) equilibrium31. (a) provides 32. (e) form33. (e) symbol 34. (b) made35. (d) emphasis

SPEED TEST 71

1. (b) 2. (a) 3. (c) 4. (b)5. (d) 6. (c) 7. (b) 8. (c)9. (b) 10. (d) 11. (c) 12. (c)13. (a) 14. (e) 15. (b) 16. (e)17. (b) 18. (c) 19. (e) 20. (d)21. (e) 22. (d) 23. (e) 24. (b)25. (b) 26. (d) 27. (d) 28. (a)29. (c) 30. (c) 31. (e) 32. (a)33. (c) 34. (a) 35. (d)36. (b) substitute has for are.37. (b) replace should by will.38. (a) sale should come in place of selling39. (c) earned.40. (d) replace since with from.

Page 346: yoursmahboob.wordpress.com SBI · yoursmahboob.wordpress.com iii P 101 Speed Tests for SBI Bank Clerk Exam 101 Speed Tests for SBI Bank Clerk Exam is revised and updated edition on

yoursmahboob.w

ordpress.com

80 101 SPEED TEST

SPEED TEST 72

1. (a) 2. (c) 3. (a) 4. (b)5. (e) 6. (b) 7. (b) 8. (d)9. (a) 10. (e) 11. (e) 12. (b)13. (a) 14. (b) 15. (a) 16. (e)17. (d) 18. (a) 19. (e) 20. (b)21. (b) 22. (e) 23. (d) 24. (a)25. (d) 26. (b) 27. (a) 28. (c)29. (b) 30. (c)

SPEED TEST 73

1. (e) 2. (a) 3. (d) 4. (a)5. (b) 6. (d) 7. (e) 8. (e)9. (a) 10. (e) 11. (c) 12. (c)13. (b) 14. (a) 15. (b) 16. (d)17. (d) 18. (c) 19. (c) 20. (d)21. (b) 22. (c) 23. (c) 24. (d)25. (b) 26. (e) 27. (e) 28. (c)29. (c) 30. (c)

SPEED TEST 74

1. (e) 2. (a) 3. (d) 4. (b)5. (b) 6. (c) 7. (b) 8. (a)9. (a) 10. (c) 11. (a) 12. (d)13. (d) 14. (a) 15. (b) 16. (d)17. (c) 18. (c) 19. (c) 20. (d)21. (a) 22. (d) 23. (a) 24. (a)25. (a) 26. (c) 27. (b) 28. (a)29. (b) 30. (c)

SPEED TEST 751. (c) 2. (a) 3. (c) 4. (c)5. (a) 6. (b) 7. (c) 8. (b)9. (a) 10. (c) 11. (a) 12. (b)13. (a) 14. (c) 15. (d) 16. (d)17. (e) 18. (a) 19. (b) 20. (a)21. (b) 22. (c) 23. (b) 24. (a)25. (a) 26. (a) 27. (e) 28. (a)29. (a) 30. (c)

SPEED TEST 76

1. (e) 2. (d) 3. (c) 4. (e)5. (d) 6. (e) 7. (d) 8. (c)9. (c) 10. (b) 11. (c) 12. (b)13. (d) 14. (e) 15. (b) 16. (d)17. (b) 18. (a) 19. (d) 20. (a)

21. (a) 22. (a) 23. (a) 24. (b)25. (c) 26. (a) 27. (e) 28. (e)29. (c) 30. (d)

SPEED TEST 77

1. (b) 2. (b) 3. (b) 4. (c)5. (c) 6. (a) 7. (a) 8. (d)9. (a) 10. (d) 11. (b) 12. (d)13. (d) 14. (d) 15. (b) 16. (e)17. (b) 18. (e) 19. (e) 20. (e)21. (b) 22. (b) 23. (e) 24. (c)25. (d) 26. (c) 27. (e) 28. (c)29. (d) 30. (a)

SPEED TEST 78

1. (b) 2. (e) 3. (e) 4. (e)5. (a) 6. (b) 7. (a) 8. (c)9. (a) 10. (d) 11 (a) 12. (c)13. (e) 14. (a) 15. (b) 16. (d)17. (e) 18. (c) 19. (d) 19. (c)21. (d) 22. (d) 23. (e) 24. (e)25. (b) 26. (a) 27. (d) 28. (a)29. (e) 30. (a)

SPEED TEST 79

1. (e) 2. (b) 3. (c) 4. (a)5. (b) 6. (a) 7. (d) 8. (e)9. (b) 10. (c) 11. (c) 12. (c)13. (d) 14. (e) 15. (e) 16. (c)17. (d) 18. (e) 19. (c) 20. (e)21. (c) 22. (d) 23. (d) 24. (e)25. (a) 26. (a) 27. (c) 28. (d)29. (c) 30. (b)

SPEED TEST 80

1. (e) 2. (b) 3. (d) 4. (a)5. (c) 6. (c) 7. (a) 8. (c)9. (c) 10. (a) 11. (d) 12. (b)13. (c) 14. (a) 15. (c) 16. (e)17. (c) 18. (d) 19. (a) 20. (d)21. (e) 22. (c) 23. (a) 24. (a)25. (a) 26. (a) 27. (e) 28. (e)29. (d) 30. (a) 31. (b) 32. (c)33. (e) 34. (e) 35. (c) 36. (e)37. (a) 38. (d) 39. (c) 40. (d)

Page 347: yoursmahboob.wordpress.com SBI · yoursmahboob.wordpress.com iii P 101 Speed Tests for SBI Bank Clerk Exam 101 Speed Tests for SBI Bank Clerk Exam is revised and updated edition on

yoursmahboob.w

ordpress.com

81SOLUTIONS

SPEED TEST 81

1. (b) 2. (c) 3. (a) 4. (b) 5. (b) 6. (c)7. (d) 8. (c) 9. (c) 10. (c) 11. (d) 12. (b)

13. (d) 14. (d) 15. (b) 16. (d) 17. (a) 18. (c)19. (b) 20. (a) 21. (c) 22. (d) 23. (d) 24. (d)25. (b) 26. (c) 27. (b) 28. (a) 29. (d) 30. (b)

SPEED TEST 82

1. (c) The accounting year of RBI starts the month betweenJuly-June.

2. (a) In Article-30, RBI permitted to the co-operative Bankfor special account supervision.

3. (c) Open market operations of RBI refers to trading insecurities.

4. (b) Monetary policy in India is formulated and implementedby RBI.

5. (a) Reserve Bank of India follows minimum reserve systemfor the issuing of currency.

6. (b) RBI controls credit creation by the Commercial Bank inIndia.

7. (b) Note issuing department of RBI should always possessthe minimum gold stock worth 115 crore.

8. (b) RBI and CSO in India is entrusted with the collectionof data of capital formation.

9. (c) The Bank rate is the rate at which RBI gives credit tothe commercial Banks.

10. (b) An increase in CRR by the RBI results in reduction inliquidity in the economy.

11. (c) Commercial Banks provide the largest-credit toagriculture and allied sectors.

12. (a) RBI publishes the financial report on currency andfinance.

13. (a) RBI is the custodian of India’s foreign exchange funds.14. (c) RBI sanctions foreign exchange for the import of goods.15. (d) There are 4 posts of deputy Governor in Reserve Bank

of India.16. (d) RBI regulates the external commercial borrowings.17. (d) All the statements given above are correct except that,

RBI was established in 1949.18. (b) RBI has tweaked upcoming new rules for lending rates.

The central bank said on 29th March 2016 the fixed rateloans of close to three years offered by lenders will belinked to the marginal cost of funding. Loans abovethat tenor should be exempt. Earlier, all fixed rate loanshave been exempted from being set based on marginalcost of funding. The change will apply to new rulesimplemented from April 1, 2016.New rules will forcelenders to adjust lending rates in relation to marketrates removing the sector's discretion in making adecision as to how much to charge for loans.

19. (d) Reserve Bank has provided relief to banks holdingbonds of State Electricity Boards that were debt ridden.Banks have been allowed to keep Ujwal DiscomAssurance Yojana scheme bonds under held tomaturity category. This will lessen pressure on bondyields and debt market. HTM is part of the debt holdingsof the bank not subjected to daily price movementwhich can be held by banks till maturity.

20. (b) As per RBI, growth of card acceptance infrastructure likeautomated teller machines (ATMs) and point-of-sale (PoS)terminals is not on a par with that of card issuance.Debit cards vastly outnumber the volume of credit cardsissued in the country. Further, a high number of debitcards have been issued in recent times under the PrimeMinister's Jan Dhan Yogana, especially to customersin rural areas and smaller towns.Cash continues to be the predominant mode of paymentas it appears to be "costless" in comparison to thevisible costs associated with card/electronic payments.Some of the factors that have inhibited growth in theacceptance infrastructure are lack of adequate and low-cost telecom infrastructure and lack of incentive formerchants for acceptance of cards, among others.

21. (b) Reserve Bank of India said the government will buyback inflation indexed bonds/IIBs maturing in 2023 viareverse auction in February 11th. Not so effective bondshave received a below par response due to lack ofmarketing and associated tax issues. GoI has alsonotified repurchase of 1.44 percent inflation indexedgovernment stock 2023 via a reverse auction for totalamount of INR 6500 crore. Bonds were launched asalternative to gold as an investment in high currentaccount deficit. Repurchase will be prematurely held toredeem government stock through utilization of surpluscash balance and it was an ad-hoc move.

22. (a) RBI has four Deputy Governors. Two are from outside,a commercial banker and an economist and two othersare promoted from within its ranks.- Deputy Governor of RBI can be appointed for a periodof five years or till the age of 62, whichever is earlier.Current other three Deputy Governors of RBI are:R.Khan, S.S. Mundra and R. Gandhi.- The Union Government has reappointed Dr Urjit Patelas Deputy Governor of Reserve Bank of India (RBI).- Dr Urjit Patel headed committee to review the monetarypolicy framework.

23. (b) Urijit Patel, Deputy Governor of RBI in charge of themonetary policy department, has been appointed for aterm of three years for the second time. Patel, 52, willfinish his 3 year term on January 10, and has becomethe longest serving deputy governor in 2015-2016 if heserves the full three year term. Many of the recent deputygovernors served a maximum period of only 5 years. Adeputy governor can serve for the period of three yearsw.e.f taking on the charge on or after January 11,2016or till further orders, whichever is earlier. A PhD fromYale University in Economics and graduate of theUniversity of London and Oxford, he has served as thehead of the U. Patel Committee proposing inflationtargeting as the prime objective of the central bank. Asper the agreement, RBI will target 4 percent inflation atthe close of the financial year 2016-2017 and forsubsequent years, with a band of +/- 2 percent. Otherdeputy governors of the RBI are H. R. Khan, R. Gandhiand S. S. Mundra.

24. (c)25. (c) Since March, 1934 India is a member of the International

Monetary Fund. It has to, therefore, maintain its rate of

Page 348: yoursmahboob.wordpress.com SBI · yoursmahboob.wordpress.com iii P 101 Speed Tests for SBI Bank Clerk Exam 101 Speed Tests for SBI Bank Clerk Exam is revised and updated edition on

yoursmahboob.w

ordpress.com

82 101 SPEED TESTexchange at the level which it has declared to the IMF.The Reserve Bank takes suitable measures to maintainthe value of the rupee at this declared level.

SPEED TEST 83

1. (a) 2. (c) 3. (d) 4. (b)5. (a) 6. (d)7. (c) To provide basic banking services to bankless villages8. (b) 9. (b) 10. (d) 11. (a)12. (d) 13. (a) 14. (a)15. (a) RBI has given nod to Muthoot Finance to set up White

Label ATMs .ATMs set up and run by non-bankingentities are called White Label ATMs (WLAs).

16. (c) It is a special type of credit card which is sponsored byboth the credit card issuing company and theparticipating retail company or vendor. Co-brandedcredit card carries special deals and savings from theparticipating merchants.

17. (a) Credit cards linked to special organizations like sportsclubs, exclusive clubs and charities. Affinity credit cardscan also help raise funds, when a part of income fromevery transaction goes toward the benefit of relevantorganization.

18. (b)19. (a) Visa card In 1977, Visa was adopted internationally and

became the first credit card facility to be recognisedworldwide. It is a conditional authorization given by acompetent authority of a country for a person who isnot a citizen of that country to enter its territory and toremain there for limited duration.

20. (c) A hot card is a lost or stolen card. A hot list is the list ofcaution against the use of a credit card by a defaulterholder.

21. (d)

SPEED TEST 84

1. (e) 2. (a) 3. (b) 4. (e) 5. (a) 6. (b)7. (c) 8. (a) 9. (c) 10. (a) 11. (e) 12. (b)

13. (a) 14. (a) 15. (a) 16. (b) 17. (b) 18. (a)19. (c) 20. (c) 21. (c) 22. (d) 23. (d) 24. (d)25. (c) 26. (b) 27. (b) 28. (d) 29. (c) 30. (d)

SPEED TEST 85

1. (c) Tertiary sector of Indian economy contributes largestto the Gross National Product.

2. (a) Service sector is the main source of National Income inIndia.

3. (d) Toll tax is not a tax levied by the government of India.4. (d) The most appropriate measure of a country’s economic

growth is it’s per capita product.5. (c) Foreign Exchange Management Act (FEMA) was finally

implemented in the year 2002.6. (b) The most common measure of estimating inflation in

India is WPI.7. (d) The national income of India is estimated by CSO.8. (c) Finance Ministry formulates the fiscal policy in India.9. (b) The devaluation of rupee in India took place twice in

the financial year 1991-92.

10. (c) VAT is imposed on all stages between production andfinal sale.

11. (c) Balance of payment is used in terms of Exports andImports.

12. (b) The Indian Economy can be described as a developingeconomy.

13. (b) 14. (d) 15. (d) 16. (d)17. (a) 18. (a)19 (b) A company which has net owned funds of at least

` 300 Crore and has deployed 75% of its total assets inInfrastructure loans is called IFC provided it has creditrating of A or above and has a CRAR of 15%.

20. (a) If the IDF is set up as a trust, it would be a mutual fund,regulated by SEBI.

21. (a) NBFC-MFI is a non-deposit taking NBFC which has atleast 85% of its assets in the form of m microfinance.Such microfinance should be in the form of loan givento those who have annual income of `60,000 in ruralareas and 120,000 in urban areas. Such loans shouldnot exceed `50000 and its tenure should not be lessthan 24 months. Further, the loan has to be given withoutcollateral. Loan repayment is done on weekly,fortnightly or monthly instalments at the choice of theborrower.

22. (d) Factoring refers to the process of managing the salesregister of a client by a financial services company.Basically, there are three parties involved in a factoringtransaction: 1. The buyer of the goods 2. The seller ofthe goods 3. The factor, i.e. The financial institution.

23. (b) All NBFCs are not allowed to take deposits. Only thoseNBFCs which have specific authorization from RBI areallowed to accept/hold public deposits. NBFCs cannottake demand deposits. They can accept only termdeposits with a tenure of minimum 12 months.

24. (b) Liquid assets are those assets that can be exchangedmost readily with minimum number of obstacles andminimum time.

25. (b) Chanakya26. (d) Both (b) and (c)27. (a) Ratio of money held by the public in currency to that of

money held in bank deposits.28. (b) The proportion of total deposits commercial banks keep

as reserves.29. (c) The fraction of the deposits that commercial banks must

keep with RBI30. (b) This includes the currency (notes and coins in

circulation and vault cash of commercial banks) alongwith the deposits held by the Government of India adcommercial banks with RBI.

SPEED TEST 86

1. (c) ‘Eco Mark’ is given to the Indian products forEnvironment Friendly purpose.

2. (b) The Earlier name of WTO was GATT before 1995.3. (b) World Development Report is an annual publication of

IBRD.4. (a) India has the maximum volume of foreign trade with

USA.5. (b) Participatory notes (PNs) are associated with Foreign

Institutional Investors (FII’s).

Page 349: yoursmahboob.wordpress.com SBI · yoursmahboob.wordpress.com iii P 101 Speed Tests for SBI Bank Clerk Exam 101 Speed Tests for SBI Bank Clerk Exam is revised and updated edition on

yoursmahboob.w

ordpress.com

83SOLUTIONS6. (b) The purpose of India Brand Equity Fund to make ‘Made

in India’ a label of quality.7. (a) A trade policy consists of Export-Import policy.8. (b) FERA in India has been replaced by FEMA.9. (b) TRIPS and TRIMS are the terms associated with WTO.10. (c) In the year 2006, SEZ act was passed by the parliament.11. (d) There are total 162 members recently in WTO.12. (d) Capital account is classified into 3 parts in India-

private, banking and official capital.13. (a) The investment in productive assets and participation

in management as stake holders in business enterprisesis foreign direct investment.

14. (b) The portfolio investment by foreign institutionalinvestors is called foreign institutional investment.

15. (c) 16. (c) 17. (b) 18. (b)19. (a) 20. (c) 21. (d) 22. (d)23. (b) 24. (a) 25. (c) 26. (d)27. (c) 28. (c) 29. (c) 30. (b)

SPEED TEST 87

1. (c) Swabhiman scheme is associated with Rural Bankingin India.

2. (c) The unorganised workers social security Act waspassed in 2008.

3. (c) Swadhar scheme launched for the women in difficultcircumstances.

4. (a) The main objective of Pradhan Mantri GramodayaYojana is meeting rural needs like housing, drinking,water, healthcare, etc.

5. (b) Twenty point Economic programme was first launchedin 1975.

6. (a) The disguished unemployment is 9 prominent featuremainly of primary sector.

7. (a) Golden Quadrangle project is associated with Highwaysdevelopment.

8. (d) All the above given statements is the objective ofNational Food security Mission.

9. (c) The period 2012-17 is related to 12th Five year plan inIndia.

10. (b) Liberalization of Economy is not a measure of reducinginequalities.

11. (a) Valmiki Awas Yojana subsumed Integrated Housingand slum development programme.

12. (c) Poverty level in India is established on the basis ofhouse hold consumer expenditure.

13. (b) Nirmal Bharat Abhiyan Yojana is associated withcommunity toilets in slum areas.

14. (c) Crop Insurance scheme in India was started in 1985.15. (d) 'SJSRY' Scheme is not related to the rural development.16. (d)17. (c) ICDS programme focused on the age group of children

upto 6 years.18. (d) The age group of 40-79 years old women are eligible for

India Gandhi Widow Pension Scheme.19. (b) The Project 15B of Indian navy aims to develop stealth

guided missile destroyers. Under the project MazagonDock Limited will construct four stealth guided missiledestroyers. Project 15B destroyers will featureenhanced stealth characteristics as well as

incorporating state of the art weaponry and sensorsincluding the extended range Barak 8 surface-to- airmissiles.

20. (a) Indian railway has launched the E-Samiksha an onlineproject monitoring system with an aim of monitoringimplementation of various ongoing projects includingRail Budget proposals. Apart from budget-relatedprojects, the E-Samiksha can also be used formonitoring the infrastructure target and board meetingfollow-up.

21. (c) The Government of India, through its Ministry ofEnvironment & Forests, is implementing a project titled"Capacity Building for Industrial Pollution Management"under the assistance of World Bank. The projectobjectives are strengthening the environmentalmanagement capacity of central and state levelregulatory authorities with emphasis on rehabilitationof polluted sites and for undertaking area-baseddemonstration projects on remediation of contaminatedsites. The project also aims at developing a "NationalProgram for the Rehabilitation of Polluted Sites" toreduce or eliminate the environmental and health risksassociated with legacy pollution.

22. (a) The union government has set up a panel headed by BN Navalawala to look into various contentious issuesrelating to inter-linking of rivers. The panel will workstowards speedy implementation of the inter-linking ofriver projects. The panel comprising of water expertsand senior officials of different ministries will facilitateinterlinking of intra-state and intra- basin rivers.

23. (a) Dr Nasim Zaidi has been appointed a next Chief ElectionCommissioner of India. He will assume the charge ofthe office from April 19, after the incumbent H S Brahmawill retire on April 18.

24. (d) This scheme was announced to enable minority youthto obtain school leaving certificate and gain betteremployment.

25. (c) Nawaj Shaikh is from National AIDS Research Institute(NARI) and belongs to Pune. The said competition washeld through the MyGov Platform for suggesting theLogo, Slogan and Tagline for the New Education Policyof the Government of India. More than 3000 entrieswere received. It was for the first time ever that thisparticular logo was designed by a common man of thecountry and not by any advertising agency or acorporate house. A cash prize of `10,000 will beawarded to Shaikh.

SPEED TEST 88

1. (a) Ruchir Kamboj, currently the Chief of Protocol, hasbeen appointed as the next Permanent Representativeof India to UNESCO, Paris, with the rank of ambassador.The 1987-batch IFS officer, Kamboj will succeed V SOberoi.

2. (a) Government of India has appointed Justice VangalaEswaraih, former acting Chief Justice of AndhraPradesh as the chairperson of the National Commissionfor Backward Classes. The National Commission ofBackward Classes has been set up under the NationalCommission for Backward Classes Act 1993.

Page 350: yoursmahboob.wordpress.com SBI · yoursmahboob.wordpress.com iii P 101 Speed Tests for SBI Bank Clerk Exam 101 Speed Tests for SBI Bank Clerk Exam is revised and updated edition on

yoursmahboob.w

ordpress.com

84 101 SPEED TEST3. (e)4. (c) Justice Swatanter Kumar is the present Chairperson of

National Green Tribunal of India.5. (b) Prime Minister of India chairs the National Ganga River

Basin Authority.6. (b)7. (b) Shri Ratan Thiyam has been appointed as the

chairperson of the National School of Drama Society.The President of India has appointed Shri Thiyam for aperiod of four years as per the relevant rules andregulations of the Society. Ratan is a playwright andtheatre director.

8. (e) 9. (e) 10. (c) 11. (c)12. (e) 13. (d) 14. (c) 15. (a)16. (c) 17. (b) 18. (b) 19. (a)20. (a) 21. (a) 22. (b) 23. (c)24. (c) 25. (a) 26. (d) 27. (b)28. (c) 29. (b) 30. (c)

SPEED TEST 89

1. (d) Brazil is not a member of ASEAN.2. (b) ‘Sanklap’ project is associated with the eradication of

HIV/AIDS.3. (b) 19th SAARC Summit will held in Islamabad, Pakistan in

2016.4. (d) The 29th summit of ASEAN will be organised in Laos.5. (d) The 2016 BRICS Summit will be held in Panaji, Goa in

India.6. (a) The 2016 annual meeting of WEF organised in

Switzerland.7. (b) There are 21 members in the APEC nations organisation.8. (d) The 17th NAM summit proposes to be held in

Venezuela.9. (c) The G-20 2016 Summit will be hosted in China.10. (a) The 42nd summit of G-7 will be held in Japan.11. (a) Azerbaijan and Fiji became the new members of NAM.12. (b) The 2016 NATO summit organised in the Poland.13. (a) A lady president elected first time in South Korea.14. (a) George Mario Bergoglio elected as a new pope of Roman

Catholic Church.15. (a) The ‘UN Women’ came into existence on 1 July 2010.16. (d) There are 10 members associated with BIMSTEC.17. (a) The Ranking of India in Global Hunger Index list is 63rd

last year.18. (d) There are 8 members associated with MERCOSUR.19. (a) The 9th World Hindi Conference held in South Africa.20. (c) There 8 point mentioned in MDG-2015 of United Nation.21. (a) The headquarter of FAO (Food and Agriculture

organisation) is situated in Italy.22. (b) SAMPRITI-III, 9 a special security forces exercise

organised between India and Bangladesh.23. (a) Shanghai Cooperation Organisation has set to

showcase alignment between China and Russiathrough integration of Beijing marshalled Silk RoadEconomic Belt and Moscow driven Eurasian EconomicUnion. SCO is part of the emerging Eurasia-centredSilk Road geopolitical architecture pillared by Chinaand Russia, along with the Central Asian Republics.

24. (a) Nuclear Security Summit 2016 took place in WashingtonDC on April 1, 2016 and leaders comprised PM NarendraModi, UK PM David Cameron, Canadian PM JustinTrudeau, French President Francois Hollande and otherworld leaders. World leaders have stressed theimportance of Convention on Physical Protection ofNuclear Material and its amendment in 2005 as well asthe International Convention on the suppression ofacts of nuclear terrorism.

25. (a) The 13th edition of the World Spice Congress began inAhmedabad, Gujarat and the theme of the two yearlythree day event is 'Target 2020: Clean, Safe andSustainable Supply Chain'. The World Spice Congresshas been organised jointly by Spices Board, CochinHill Produce Merchant's Association and IndiaPepperand Spice Trade Association. It is given support by AllIndia Spices Importers & Distributors Association andIndian Spices & Foodstuff Exporters' Association.

26. (a) The Union Health and Family Welfare Minister Shri JPNadda chaired the 12th Conference of the CentralCouncil of Health and Family Welfare which discussedthe Draft of the National Health Policy 2016. Some ofthe resolutions adopted towards the close of themeeting were as followed:(i) Health needs of country have changed over time

since the last National Health policy(ii) Draft National Policy formulated by the Ministry

of Health and Family Welfare, GoI has beenformulated keeping this in mind

(iii) Endorsement of the Draft National Health Policyby the Central Council for Health and FamilyWelfare.

27. (d) One day Annual Conference of the State MinoritiesCommission commenced with the focus on inclusivegrowth with the motto "Sabka Saath, Sabka Vikas." Theone day Conference focusing on Development ofMinorities had two Technical sessions besides theinaugural. Valedictory. Mr. Praveen Davar addressedthe Technical I session on "Minority Welfare Schemesof Government of India-An Overview". Technical II session on "Functioning of State MinoritiesCommissions-Problems and Challenges" wasaddressed by Ms. Mabel Rebello. Last year, the themesof the conference were "Capacity building of MinorityEducational Institutions in India" and "Corporate SocialResponsibility and Development of Minorities in India".

28. (a) This is the flagship initiative of the Shipping Ministry.It provides an excellent platform for participants to fundbusiness opportunities. The Maritime India Summit willshowcase investment opportunities in the maritimesector for different activities such as port modernisation,shipbuilding, ship repair and cycling. The aim is toprovide a platform for interacting closely withpioneering international maritime organisations andcreate awareness about trends and potential formaritime sector in India.

29. (d) Mr. Nitin Gadkari, Minister of Shopping, RoadTransport and Highways launched a website entitledwww.maritimeinvest.in to commemorate the MaritimeIndia Summit within the Make in India Summit. User

Page 351: yoursmahboob.wordpress.com SBI · yoursmahboob.wordpress.com iii P 101 Speed Tests for SBI Bank Clerk Exam 101 Speed Tests for SBI Bank Clerk Exam is revised and updated edition on

yoursmahboob.w

ordpress.com

85SOLUTIONSfriendly website will facilitate access to detailedinformation about the summit including registrations.Maritime India Summit 2016 is a global maiden summitbeing carried out by the Shipping Ministry in April. ForMIS 2016, India's partner country is the Republic ofKorea. More than 50 maritime nations have also beeninvited to attend the summit.

30. (a) 5 day long 10th WTO Ministerial Conference wasconcluded on 19th December 2015 in Nairobi, Kenyaaimed at adoption of the Nairobi package for benefittingthe poorest members of the organisation. Theconference was attended by trade ministers of 162member countries of the WTO. India was representedby Minister of State (Independent Charge) forCommerce & Industry Nirmala Sitharaman. This marksthe first time a meeting has been held in Africa by WTOin this capacity.

SPEED TEST 90

1. (d) The Jnanpith award is a literary award which alongwith the Sahitya Akademi Fellowship is one of the twomost prestigious literary honours in the country. Theaward was instituted in 1961. Any Indian citizen whowrites in any of the official languages of India is eli-gible for the honour.

2. (a) Bharat Ratna is India’s highest civilian award. The offi-cial criteria for awarding the Bharat Ratna stipulated itis to be conferred “for the highest degrees of nationalservice which includes artistic, literary, and scientificachievements, as well as “recognition of public serviceof the highest order”. The last recipient of the award isthe cricketer Sachin Tendulkar for the year 2014.

3. (c) The National Film awards, one of the most prominentfilm awards in India, were established in 1954. Everyyear, a national panel appointed by the governmentselects the winning entry, and the award ceremony isheld in New Delhi where the President of India pre-sents the awards.

4. (c) The Vyas Samman is a literary award which was firstawarded in 1991. It is awarded annually by the K.K.Birla Foundation and includes a cash of‘ 250,000 (as of 2005). To be eligible for the award, theliterary work must be in the Hindi language and hasbeen published in the past 10 years.

5. (c) The Saraswati Samman is an annual award for outstand-ing prose or poetry literary works in any Indian lan-guage. It was instituted in 1991 by the K. K. Birla Foun-dation. The award contains ‘ 10 lakh, a citation and aplaque. Candidates are selected from literary workspublished in the previous ten years by a panel thatincludes scholars and former award winners.

6. (a) The Maha Vir Chakra is the second military decoration inIndia and is awarded for acts of conspicuous gallantry inthe presence of the enemy, whether on land, at sea or in theair. The medal may be awarded posthumously.

7. (d) Bihari Puraskar conferred by Rajasthan is a literaryaward instituted by K. K. Birla Foundation. The awardis named after the famous Hindi poet Bihari and isawarded to an outstanding work published in Hindi or

Rajasthani by a Rajasthani writer. It carries a citation, aplaque and prize money of ‘ 1 lakh.

8. (a) Tansen Samman is conferred in the field of music whichcarries a cash prize of ‘ 2 lakh and a citation.

9. (a)10. (b) The Shanti Swarup Bhatnagar award for Science and

Technology (SSB) is an award in India given annuallyby the CSIR. It is named after the founder Director ofthe CSIR and carries an award money of ‘ 5 lakh each.

11. (a) In 2006, Kejriwal was awarded the Ramon MagsaysayAward for Emergent Leadership recognising his in-volvement in a grassroots movement Parivartan usingright-to-information legislation in a campaign againstcorruption. The same year, after resigning from the IRS,he donated his Magsaysay award money as a corpusfund to found the Public Cause Research Foundation,a non-governmental organisation (NGO).

12. (a) Bidhan Chandra Roy Award was instituted in 1976 inmemory of B. C. Roy by Medical Council of India. TheAward is given annually in the categories of States-manship of the Highest Order in India, Medical man-cum-Statesman, Eminent Medical Person, Eminent per-son in Philosophy and Eminent person in Arts. It ispresented by President of India in New Delhi on July 1,the National Doctors’ Day.

13. (a) Pampa Prashasti is the highest literary honour con-ferred by the Karnataka government on a litterateur forhis/her lifetime contribution to Kannada literature. Theaward is named after Adikavi Pampa of 10th Century.The award carries award money of ‘ 3 lakh and a cita-tion.

14. (a) Madhya Pradesh has been awarded 10th NationalAward for Excellence work in Mahatma Gandhi NationalRural Employment Guarantee Act (MGNREGA). Theaward was given to Madhya Pradesh for convergenceof MGNREGA and other schemes for construction ofpermanent assets and for excellent work in providingopportunities for earning permanent income.

15. (b)16. (d) The Nobel prize is a set of an international awards be-

stowed in a number of categories which is given annu-ally to the winners by Swedish and Norwegian Com-mittees in recognition of cultural and/or scientific ad-vances. It was the will of the Swedish inventor AlfredNobel that established the Nobel prizes in 1895 in Swe-den.

17. (c) The Nobel prize in Economics or Economic scienceswas established in 1968 and endowed by Sweden’scentral bank, the Sveriges Riksbank, on the occasionof the bank’s 300th anniversary. While the Nobel Prizein particular was established in 1895.

18. (a) The Academy award is also known as the Oscar awardwhich is presented for various categories in the Filmindustry. It was first given in 1929.

19. (c) The Confucius Peace prize is a prize established in 2010in the People’s Republic of China (PRC). The ConfuciusPeace Prize’s first winner was former Vice President ofthe Republic of China and Kuomintang Chairman LienChan, for his contribution to developing positive tiesbetween Taiwan and mainland China.

Page 352: yoursmahboob.wordpress.com SBI · yoursmahboob.wordpress.com iii P 101 Speed Tests for SBI Bank Clerk Exam 101 Speed Tests for SBI Bank Clerk Exam is revised and updated edition on

yoursmahboob.w

ordpress.com

86 101 SPEED TEST20. (a) The Pulitzer Prize is a U.S. award for achievements in

newspaper and online journalism, literature, and musi-cal composition. It was established in 1917 and admin-istered by Columbia University in New York City byprovisions in the will of American publisher JosephPulitzer.

21. (c) The Nobel awards in literature, medicine, physics, chem-istry, peace, and economics are given in Stockholm,Sweden. The Peace prize is awarded in Oslo, Norway.

22. (a) The British Academy Film awards are presented in anannual award show hosted by the British Academy ofFilm and Television Arts (BAFTA). It is given by UKand is considered to be the counter awards for Oscars.

23. (c) Golden Globe award is given in the field of film andtelevision by Hollywood Foreign Press Association inUnited States of America.

24. (a) The Palme d’Or is the highest prize awarded at theCannes Film Festival and is presented to the director ofthe best feature film of the official competition. It ispresented by Festival International du film de, France.

25. (a) The Kalinga Prize for popularization of Science is aninternational distinction instituted by UNESCO. It wasstarted in 1951 by donation from Mr Bijoyanand Patnaik,founder and president of the Kalinga Foundation Trustin India.

26. (a) International Gandhi Peace prize is given annually byGovernment of India to those individuals and organi-zations which contribute towards changes in the po-litical, social or economic reforms via non-violence. Itwas instituted in 1995.

27. (c) The Royal Institute of British Architects Stirling Prizeis a British prize for excellence in architecture. It isorganised and awarded annually by the Royal Insti-tute of British Architects (RIBA).It is named after thearchitect James Stirling.

28. (a) The World Economic Forum gives Crystal award tothose artists who have improved the state of the worldthrough their art.

29. (d) Ramon Magsaysay award is given annually to thoseAsian people who have contributed extraordinary ser-vice in their respective fields. This award is given byPhilippine in the memoir of Philippine President RamonMagsaysay. He is considered to be one of the greatexamples of integrity, courage, and idealistic democrat.

30. (a)

SPEED TEST 91

1. (c) The book presents the booming maintenance of theworld’s largest democracy and achievements of Indiasince independence.

2. (d) Ambedkar Speaks (Trilogy) is authored by Dr. NarendraJadhav.It is an attempt to develop a comprehensivebibliography on the speeches delivered by Ambedkar.

3. (c) 4. (a) 5. (b)6. (d) A pictorial coffee table book by Alam Srinivas was

launched on 24 June 2013 in Mumbai by former BombayHigh Court Chief Justice C S Dharmadhikari.

7. (b) 8. (a) 9. (b) 10. (a)11. (d) 12. (d) 13. (a)

14. (b) Anand Math is a 1952 Hindi patriotic-historical filmdirected by Hemen Gupta, based on ‘Anandamath’,the famous Bengali novel written by Bankim ChandraChattopadhyay in 1882. The novel and film are set inthe events of the Sannyasi Rebellion, which took placein the late 18th century in eastern India, especiallyBengal.

15. (b) 16. (b) 17. (c) 18. (c)19. (b) 20. (b)21. (a) The Vice President of India Shri M. Hamid Ansari

released a book titled “India and Malaysia: InterwinedStrands” authored by former diplomat Smt Veena Sikri.The book offers a panoramic yet in-depth historicalanalysis of the inter-linkages between India andMalaysia, which are a microcosm of the much largerrelationship between South Asia and South East Asia,as these have evolved more than two millennia.

22. (d) The Global Competitiveness Report is a yearly reportpublished by the World Economic Forum. The reportassesses the ability of countries to provide high levelsof prosperity to their citizens. This in turn depends onhow productively a country uses available resources.Since 2010, Switzerland has led the ranking as the mostcompetitive economy in the world India is at 60thposition.

23. (b) Ashtadhyayi is written by Panini. Panini a sage who isbelieved to have lived around the fifth century B.C.,although other claims trace him to the 4th, 6th and 7thcenturies, is credited with having created theAshtadhyayi, or Ashtak, which is a grammar definingthe structure and syntax of the Sanskrit language.

24. (c) Mitakshara is written by Vigyaneshwar. It was consid-ered one of the main authorities on Hindu Law from thetime the British began administering laws in India.

25. (c) Kautilya’s Arthashastra is an excellent treatise on state-craft, economic policy and military strategy. it is said tohave been written by Kautilya, also known by the nameChanakya or Vishnugupta, the prime minister of India’sfirst great emperor, Chandragupta Maurya.

26. (d) The Post Office (Bengali: Dak Ghar) is a 1912 play byRabindranath Tagore. It concerns Amal, a child confined tohis adopted uncle’s home by an incurable disease.

27. (c)28. (b) English author Rudyard Kipling wrote the Jungle Book

in 1894. It is a collection of stories.29. (a) War and Peace is a novel by the Russian author Leo

Tolstoy, first published in 1869. The work is epic inscale and is regarded as one of the most importantworks of world literature. It is considered as Tolstoy’sfinest literary achievement, along with his other majorprose work, Anna Karenina (1873–1877).

30. (b) The Prince is a 16th-century political treatise by theItalian diplomat and political theorist NiccolòMachiavelli. The descriptions within The Prince havethe general theme of accepting that the aims of princes—such as glory and survival—can justify the use of im-moral means to achieve those ends.

SPEED TEST 92

1. (c) The Olympic Museum is located in Lausanne, Switzer-land. The museum was founded on 23 June 1993, on

Page 353: yoursmahboob.wordpress.com SBI · yoursmahboob.wordpress.com iii P 101 Speed Tests for SBI Bank Clerk Exam 101 Speed Tests for SBI Bank Clerk Exam is revised and updated edition on

yoursmahboob.w

ordpress.com

87SOLUTIONS

the initiative of Juan Antonio Samaranch. The OlympicMuseum was opened again on 21 December 2013 after23 months of renovation.

2. (b) In golf, a caddy or caddie is the person who handles agolf player’s bag and clubs, and gives also some in-sightful advice and moral support to him.

3. (c) The ICC Champions Trophy is a One Day International(ODI) cricket tournament organised by the InternationalCricket Council (ICC). It is second in importance onlyto the Cricket World Cup.

4. (c)5. (a) The four Grand Slam tournaments, also called Majors,

are the most important annual tennis events. The GrandSlam itinerary consists of the Australian Open in midJanuary, the French Open in May/June, Wimbledon inJune/July, and the US Open in August/September.

6. (d) The first Youth Olympic Games (YOG), was held inSingapore from 14 to 26 August 2010. The age limit forthe games is 14-18. The next Summer Youth Olympicswill be held in Nanjing in 2014.

7. (d) Subroto Cup Football Tournament is an inter-schoolfootball tournament in India, named after the Indian AirForce Air Marshal Subroto Mukerjee. Subroto Cup isconducted by the Indian Air Force, with support fromIndia’s Ministry of Youth Affairs & Sports.

8. (a) Wankhede stadium is in Mumbai. It is in this stadiumthat India had won the World cup cricket in 2011 againstSri Lanka.

9. (a) A Gambit is a term associated with the game of Chess.Gambit is a chess opening in which a player sacrificesmaterial, usually a pawn, with the hope of achieving aresulting advantageous position.

10. (a) The term ‘ashes’ is associated with cricket.11. (a) National Sports Day is celebrated on 29th August in the

memory of the great hockey player Major Dhyan Chand.12. (c) A bogey is a score of 1-over par on any individual hole

on a golf course. Golf holes are typically rated as par 3,par 4 or par 5.

13. (c) FINA or Fédération Internationale de Natation or Inter-national Swimming Federation is the International Fed-eration (IF) recognized by the International OlympicCommittee (IOC) for administering international com-petition in Aquatics.

14. (b)15. (a) The Dronacharya award is presented by Indian Gov-

ernment to people showing excellence in sports coach-ing. B.I. Fernandez is the first foreign Coach who wasawarded by Dronacharya Award in 2012.

16. (b) 17. (b)18. (d) The Duleep Trophy is a domestic first-class cricket

competition played in India between teams represent-ing geographical zones of India. The competition isnamed after Kumar Shri Duleepsinhji.

19. (b) The Marquess of Queensberry rules is a code of gen-erally accepted rules in the sport of boxing.

20. (a) 21. (a) 22. (a)23. (a) The Olympic Games were a series of athletic competi-

tions among representatives of city-states and one ofthe Panhellenic Games of Ancient Greece. They wereheld in honor of Zeus, and the Greeks gave them amythological origin. The first Olympics is traditionallydated to 776 BC.

24. (d)

25. (a) Founded in 1961, Netaji Subhas National Institute ofSports commonly known as National Institute of Sports(NIS), is the Academic Wing of the Sports Authority ofIndia (SAI) and Asia’s largest Sports Institute locatedin city of Patiala.

26. (b) Karnam Malleshwari is an Indian weightlifter. She isthe first Indian to win an individual medal in Olympics.

27. (a)28. (b) The first world cup Hockey was played in Barcelona in

1971 whose final winner was Pakistan.29. (b) The average length of the football field is

100 – 110m (110 – 120 yards) with width is in the rangeof 64 to 75 m (70–80 yd).

30. (a) The India national field hockey team had won its firstGold in 1928 at Amsterdam, Nederlands in which Indiadefeated the Nederlands by 3-0. India also won Gold in1932, 1936, 1948, 1952, 1956, 1964, and 1980.

SPEED TEST 93

1. (b) 2. (b)3. (b) With an aim of avoiding a repeat of the 2004 catastrophe,

India is building Tsunami warning device in the SouthChina Sea, which is likely to operate in the next 10months.

4. (a) British drug maker GlaxoSmithKline is seekingregulatory approval for the world’s first malaria vaccineafter trial data showed that it had reduced the numberof cases in African children. The vaccine known asRTS, S was found to have almost halved the number ofmalaria cases in young children in the trial and to havereduced by about 25% the number of malaria cases ininfants. GSK is developing RTS,S with non-profit PathMalaria Vaccine Initiative supported by funding fromthe Bill & Melinda Gates Foundation.

5. (b) Kepler’s laws of planetary motion are three scientificlaws describing the motion of planets around the Sun -The orbit of a planet is an ellipse with the Sun at one ofthe two foci, a line segment joining a planet and the Sunsweeps out equal areas during equal intervals of time andthe square of the orbital period of a planet is proportional tothe cube of the semi-major axis of its orbit.

6. (b)7. (b) A plant that produces both tomatoes and potatoes,

called the TomTato, has been developed for the UKmarket. Ipswich-based horticulture firm Thompson andMorgan said the hybrid plants were not geneticallymodified

8. (b) Australia unveiled its most powerful computer Raijinnamed after the Japanese God of thunder and rain. Raijinis considered the 27th most powerful computer in theworld.

9. (a)10. (b) The Vikram Sarabhai Space Centre (VSSC) is a major

space research centre of the Indian Space ResearchOrganisation (ISRO), which focuses on rocket and spacevehicles for India's satellite programme. It is located inThiruvananthapuram, Kerala.

11. (c) The first fertilizers’ plant was established in Sindri whichis an industrial township within the Dhanbad municipallimits of the Dhanbad District of Jharkhand state.

Page 354: yoursmahboob.wordpress.com SBI · yoursmahboob.wordpress.com iii P 101 Speed Tests for SBI Bank Clerk Exam 101 Speed Tests for SBI Bank Clerk Exam is revised and updated edition on

yoursmahboob.w

ordpress.com

88 101 SPEED TEST12. (a) PARAM is a series of supercomputers designed and

assembled by the Centre for Development of AdvancedComputing (C-DAC) in Pune, India. The latest machinein the series is the PARAM Yuva II.

13. (b) Cinnabar is the common ore of mercury.Generally itoccurs as a vein-filling mineral associated with recentvolcanic activity and alkaline hot springs.

14. (a) 15. (c)16. (a) ETWS installed in Rangachang in Andaman and

Nicobar Islands to predict Tsunami within three minutesof being triggered.

17. (c)18. (c) Hindustan Aeronautics Limited (HAL): Tejas is India’s

indigenously built light combat aircraft. It has comefrom Light Combat Aircraft (LCA) programme.

19. (a) Telefonica, Spanish broadband & telecommunicationsprovider has launched the world's first smartphone withthe Firefox Operating System (OS) to compete withdevices running Google's Android and Apple's IOS.

20. (b) Indian Space Research Organization (ISRO) hassuccessfully launched IRNSS-1A on PSLV C 22 fromSatish Dhawan Space Centre, Sriharikota in AndhraPradesh.

21. (a)22. (a) Global biotechnology company 'Life Technologies' has

launched India's first private DNA forensics laboratoryin Gurgaon which is expected to accelerate samplingprocess thereby reducing the burden on existingforensic laboratories.

23. (c) IBM have made the world's smallest movie "A Boy andHis Atom" using atoms. The movie has been verifiedby the Guinness World Records

24. (b) "Nirbhay", First cruise missile of India has a Long-range (1,000-2,000 km) & strikes targets more than 700km away carrying nuclear warheads.

25. (c) Sqn Ldr Rakesh Sharma was a test pilot in the IAF. In1984 he became the first citizen of India to go into spacewhen he flew aboard the Soviet rocket Soyuz T-11.

SPEED TEST 94

1. (c) 2. (b) 3. (a) 4. (d)5. (c) 6. (b) 7. (d) 8. (a)9. (b) 10. (b) 11. (b) 12. (a)13. (d) 14. (a) 15. (d) 16. (b)17. (c) 18. (d) 19. (b) 20. (a)21. (b) 22. (b) 23. (c)

SPEED TEST 95

1. (c) The Pradhan Mantri Ujjwal Yojana is a scheme to provideLPG to women of all households below poverty line.

2. (b)3. (b) The British mathematician Andrew J.Wiles won the 2016

Abel prize for solving a centuries-old hypothesis-Fermat's Last theorem.

4. (a) The World Happiness Report, 2016 released bysustainable Development Solutions Network, a globalinitiative of the United Nations placed India in 118thrank in terms of Happiness Index.

5. (b) The two day meet of 13th Broadband Commission forSustainable Development was held in Dubai (UAE). Itaims to promote the power of broadband networks &services for the 2030 Agenda for sustainableDevelopment.

6. (c) World Consumer day is celebrated across the world on15th March 2016. The theme for 2016 is- antibiotics offthe menu.

7. (d) President Pranab Mukherjee inaugurated the 5thedition of India Aviation 2016 in Hyderabad which wasjointly organised by the Union Civil Aviation Ministry& the federation of Indian Chambers of Commerce &Industry (FICCI).

8. (c) The fourth Nuclear Security Summit (NSS) is being heldin Washington, D.C. on March 31-April 1, 2016.

9. (d) Divine speaker and spiritualist Chaganti Koteswara Raohas been appointed Adviser to Andhra PradeshGovernment.

10. (b) Hashim Thaci was sworn-in as the President of Kosovoin April 2016.

11. (a) Argentina topped the April 2016 edition of FIFA orCoca-Cola World Rankings of Soccer.

12. (a) Shardul Amarchand Mangaldas advised Future Groupon its acquisition of 100% stake in FabFurnish.com.

13. (a) NASA astronaut Scott Kelly will write a memoir titledEndurance: My Year in Space and Our Journey to Mars,as announced on 6 April 2016. The book will bepublished by Alfred A. Knopf.

14. (b)15. (b) The World Health Day (WHD) 2016 theme is "Beat

Diabetes" to scale up diabetes prevention, strengthencare and enhance surveillance.

16. (b) The Australian grand prix was won by Nico Rosbergovertaking Sebastian Vettel & Lewis Hamilton.

17. (c) Agasthyamala Biosphere Reserve shared by Kerala &Tamil Nadu has been placed in the new 20 BiosphereReserve list released by UNESCO.

18. (a) Union Urban Development, Housing & Urban PovertyAlleviation & Parliamentary Affairs Minister M.Venkaiah Naidu was bestowed with Skoch LifetimeAchievement Award at 43rd SKOCH Summit held inNew Delhi for his contributions to inclusive growth &towards poverty alleviation in India.

19. (b) Union Health Ministry launched Bedaquiline new anti-TB drug for Drug Resistant TB as part of the RevisedNational Tuberculosis Control Program on the eve ofWorld TB Day.

20. (a) Veteran Singer Stevie Wonder was honoured withGlobal Green Hero Award at the 13th Annual GlobalGreen US ceremony.

21. (b) The International Women's Day is celebrated on 8thMarch globally. 2016 Theme is "Planet 50-50 by 2030:Step It up for Gender Equality."

22. (c) Luxembourg 23. (a) Bangladesh24. (a) Justice Permod Kohli was appointed as the new

chairman of the Central Administrative Tribunal (CAT).25. (a) Rajasthan has become the first state to pass land title

bill.26. (b) Italian super-car brand Lamborghini appointed former

Audi India head of field forces, Sharad Agarwal, as itsnew Head of India operations.

Page 355: yoursmahboob.wordpress.com SBI · yoursmahboob.wordpress.com iii P 101 Speed Tests for SBI Bank Clerk Exam 101 Speed Tests for SBI Bank Clerk Exam is revised and updated edition on

yoursmahboob.w

ordpress.com

89SOLUTIONS27. (b) Khadija Ismayilova became the 2016 recipient of the

UNESCO/Guillermo Cano World Press Freedom Prize.28. (a) Historian Srinath Raghavan wrote the book titled "India's

War: The Making of Modern South Asia 1939-1945" whichdetails India's contribution to World War II.

29. (c) Danny Willett has won the 80th edition of US Mastersgolf tournament 2016 at Augusta National in Georgia,United States.

30. (a) UN commemorated the 125th birth anniversary of B. R.Ambedkar on April 13 at the UN headquarters.

SPEED TEST 96

1. (b) 2. (d) 3. (d) 4. (e)5. (e) 6. (b) 7. (e) 8. (e)9. (c) 10. (e) 11. (b) 12. (a)13. (c) 14. (d)15. (a) Narmada is a west flowing river. It flows westward over

a length of 1,312 km before draining through Gulf ofCambay into the Arabian Sea.

16. (d) Floating rate bonds have variable interest rate andprotect investors against a rise in interest rates (whichhave an inverse relationship with bond prices) . Theyalso carry lower yields than fixed notes of the samematurity.

17. (c) As per rules, government has to get all money billsrelated to the union budget passed within 75 days ofthe presentation of the budget.

18. (d) Commodity markets in India are regulated by ForwardMarkets Commission (FMC) headquartered at Mumbai.

19. (d) 20. (a)21. (c) The National Maritime Day of India is celebrated every

year on April 5. On this day, in 1919 navigation historywas created when SS Loyalty, the first ship of theScindia Steam Navigation Company, journeyed to theUnited Kingdom.

22. (b) The Mullaiperiyar Dam is a masonry gravity dam onthePeriyar River in the Indian state of Kerala. It is located881 mabove mean sea level, on the Cardamom Hills ofthe Western Ghats in Thekkady, Idukki District of Kerala.It has been asubject of controversy between TamilNadu and Kerala.

23. (c) The Confederation of Indian Industry (CII) is anassociation of Indian businesses which works to createan environment conducive to the growth of industry inthe country. CII is located at New Delhi.

24. (b) The Maharashtra government has recently tied up withthe Tata Trusts to link all government colleges andmedical colleges with he latter's national cancer grid.With this, Maharashtra has become the India's firststate to join the initiative. The national cancer grid linksall existing and proposed cancer care centres to createa uniform line of treatment for cancer patients acrossIndia.

25. (b) Kachhabali village in Bhim tehsil of Rajsamand districthas become the first liquor-free village in Rajasthan.

26. (a) Kakrapara Atomic Power Station (KAPS) in Gujarat wasrecently making news, when its one unit was shut downafter leakage of heavy water from its coolant system.

27. (b) RM Lodha committee The Lodha committee, which wasprimarily assigned with the task of determining thequantum of punishment for players and others involvedin spot fixing in IPL, also gave recommendation onchange in the structure and ecosystem in the Indiancricket board saying no to politicians' involvement inBCCI.

28. (a) The Shuklaphanta Wildlife Reserve is a protected areain the Terai of the Far-Western Region, Nepal, covering305 km2 (118 sq mi) of open grassland, forests, riverbedsand tropical wetlands at an altitude of 174 to 1,386metres. It was gazetted in 1976 as Royal ShuklaphantaWildlife Reserve.

29. (d) Diarrhoea caused by Rotavirus is one of the leadingcauses of severe diarrhoea and death among childrenless than five years of age in India. Rotavac is the newvacccine launched by Health Ministry to co

30. (c) The International Day of Remembrance of the Victimsof Slavery and the Transatlantic Slave Trade is observedevery year on March 25 to offer the opportunity tohonour and remember those who suffered and died atthe hands of the brutal slavery system. The 2016 themeis "Remember Slavery: Celebrating the Heritage andCulture of the African Diaspora and its Roots".

31. (a) Swayam Shikshan Prayog (SSP) aims to promoteempowerment of women as leaders and entrepreneursthrough self help groups, social enterprises andcommunity led initiatives.

32. (a) Bangladesh Export Processing Zones Authority BEPZAstands for "Bangladesh Export Processing ZonesAuthority", which is a government organization, thatincludes information of investment opportunities andfor investors, proposals, reports, export promotionzones (EPZ) in Bangladesh.

33. (c) Assam rifles is India's oldest paramilitary force, raisedoriginally in 1835 as Cachar Levy. They perform manyroles including the provision of internal security underthe control of the army through the conduct of counterinsurgency and border security operations. AssamRifles is known as "Sentinels of the Northeast". Itsheadquarters are located at Laitkor (Shillong). The mottoof Assam Rifles is "Friends of the Hill People". WhileAssam Rifles functions under the Ministry of Defence,its administrative control is under the Ministry of HomeAffairs.

34. (a) Gurmeet Singh has become the first Indian athlete toclinch a gold medal at the 2016 Asian 20km Race WalkChampionships in Nomi, Japan. It is the first time in 34years that an Indian has won a gold in either an AsianChampionships or Asian Games in this event. Beforehim, Hakam Singh and Chand Ram won a gold each in20km road walk in 1978 and 1982 Asian Gamesrespectively.

35. (b) Bank The Indian Overseas Bank (IOB) has become thefirst bank to commence the sale of Indian Gold Coin(IGC) in the domestic market. The bank has tied up withMetals and Minerals Trading Corporation (MMTC) forthe sale of IGC of 24 carat purity. IGC is the first evernational gold offering by the Centre. The IGC has thenational emblem, the Ashok Chakra, engraved on one

Page 356: yoursmahboob.wordpress.com SBI · yoursmahboob.wordpress.com iii P 101 Speed Tests for SBI Bank Clerk Exam 101 Speed Tests for SBI Bank Clerk Exam is revised and updated edition on

yoursmahboob.w

ordpress.com

90 101 SPEED TESTside and the face of Mahatma Gandhi on the other. Thecoin carries advanced anti-counterfeit features andcomes in tamper-proof packaging. The currentdenominations available are 5, 10 and 20 gm.

36. (a) India has recently signed a loan agreement of $35Million with the World Bank (WB) for "MadhyaPradesh Citizen Access to Responsive ServicesProject". The purpose of the project is to improve accessand quality of public services in Madhya Pradeshthrough implementation of the 2010 Public ServiceDelivery Guarantee Act. The duration of project is 5years.

37. (d) Mossad is the national intelligence agency of Israel. Itis one of the most powerful secret service agencies inthe world. It's headquarters is in Tel Aviv, Israel. Theagency has active agents spread across the world andare involved in intelligence gathering, covert operationsand protecting Jews and Jewish interests.

38. (a) The United Nations Educational, Scientific and CulturalOrganization (UNESCO) Institute of Physics hasrecently released the e-Atlas of Gender Inequality inEducation. The e-Atlas shows gender gaps from primaryto tertiary education and educational pathways of girlsand boys in more than 200 countries and territories.

39. (b) The Songkran festival, also known as the World'sbiggest water festival, is celebrated every year inThailand from 13 to 15 April. Recently, it is in newsbecause Thailand government has decided to imposea curfew during 2016 Songkran festival to showsolidarity with farmers hit by drought.

40. (d) India's first contact-less mobile payment solution "iTap"has been launched by ICICI Bank. Through "iTap" itscustomers will be able to make payments by wavingtheir cell phones near an NFC- enabled merchantterminal. In it, ICICI bank has used the Host CardEmulation (HCE) technology which creates 'virtual'cards for 'physical' credit or debit cards (Visa/MasterCard) of the bank, as selected by the customer.

41. (b) The Uttar Pradesh has launched awareness campaignat school levels to create awareness aboutconservation of birds on March 14, 2016. The awarenesscampaign would also be spread among the students tomake the state clean and green under "CleanUP - GreenUP".

42. (b) The Union Power Ministry has recently named the LEDbased Domestic Efficient Lighting Programme (DELP)as "UJALA". Presently, DELP is running successfullyin over 120 cities across India. UJALA, an acronym forUnnat Jyoti by Affordable LEDs for All, is beingimplemented by Energy Efficiency Services Limited(EESL). The National LED programme was launchedby the Prime Minister on January 2015 with a target ofreplacing 77 crore incandescent lamps with LED bulbs.

43. (d) In 1966, MS Subbulakshmi was invited by Secretary-General of the United Nations, U Thant to give a specialconcert at the United Nations. This was the firstperformance by any Indian classical musician at theUN. .

44. (a) Black Sea and Sea of Azov Crimean Peninsula issurrounded by two seas - Black Sea and Sea of Azov. It

is a land mass towards the north of the Black Sea andhas Ukrainian region of Kherson to the south andRussian region of Kuban to the west. Crimea andadjacent territories were united in the Crimean Khanatebefore becoming part of Russia during 1783. Itseconomy primarily depends on tourism and agriculture,but since Russian annexation of Crimea in 2014, therehas been a considerable drop in the number of peoplevisiting it. With an area of 27,000 square kilometres,Crimea has a population of 2.2 million people. .

45. (a) Infrasound has a frequency lower than 50 hertzInfrasound has a frequency lower than 20 hertz . Thoughit cannot be heard by humans, animals such aselephants, rhinos, alligators and whales produceinfrasound. One interesting feature of these soundwaves is that they can travel longer distances thanhigh frequency waves. Avalanches and earthquakesalso produce infrasound.

46. (c) The ICICI Bank has launched "iWork@home", a first-of-its-kind programme that allows women employeesto work from home for up to a year. The company hasdeployed face recognition technology to facilitatewomen to work from home and provide access to thebank's core banking servers.

47. (c) The Karnataka government has launchedMukhyamantri Santwana Harish Yojana for free medicaltreatment to accident victims. As per the scheme, theaccident victims will be given free medical treatmentfor the first 48 hours at any hospital and financial aidupto Rs. 25,000 will also be provided. The scheme hasbeen named after Harish Nanjappa who had donatedhis eyes moments before his death. .

48. (b) The Jalandhar-based Capital Local Area Bank Ltd(CLABL) will become the India's first small area financebank (SFB) by flagging-off its operations on April 13,2016. The small bank will launch operations under anew name "Capital Small Finance Bank Limited". Apartfrom CLABL, the Reserve Bank of India (RBI) hasgranted 9 entities in-principle licences to open smallfinance banks to expand access to financial services inrural and semi-urban areas. These are Ujjivan FinancialServices Pvt. Ltd, Janalakshmi Financial Services Pvt.Ltd, Au Financiers (India) Ltd, etc.

49. (b) India's first 75-seater solar powered ferry will come upin the favourite tourism spot of Kerala "Allepey (theVenice of the East)".These boats will be commissionedwithin the next three months and will run on the 2.5 km-long Vaikkom-Thavanakkadavu route. The boat is 20 mlong, 7 m wide with a maximum cruising speed of 7.5knots. The boat is built by a Kochi-based NavAlt, incollaboration with a French firm AltEn, which makessolar powered ferries across the world.

50. (a) India's first self cleaning smart toilets have beeninstalled in Chennai, Tamil Nadu. Around 180 self-cleaning public e-Toilets have been installed and arefree for public use. Each toilet automatically cleans itselfbefore entry and after use; and sensors in it enableconsumption of less water and power with each use.

Page 357: yoursmahboob.wordpress.com SBI · yoursmahboob.wordpress.com iii P 101 Speed Tests for SBI Bank Clerk Exam 101 Speed Tests for SBI Bank Clerk Exam is revised and updated edition on

yoursmahboob.w

ordpress.com

91SOLUTIONS

SPEED TEST 97 : PRELIM TEST - 1

Hints & Solutions

1. (d)11 4 5

? 848 21216 5 11

2. (a)150 1.4 480 2.2

?100 100

= 10.50 + 10.56 = 21.06

3. (c)116 3 87 2?

4 3 = 87 – 58 = 29

4. (a)6.96 18.24?1.2 7.6

= 5.8 – 2.4 = 3.45. (b) ? = 32.25 × 2.4 × 1.6 = 123.84

6. (e)250 136 550 ? 670

100 100340 + 5.5 × ? = 6705.5 × ? = 670 – 340 = 330

330? 605.5

7. (d)448? 35 98016

8. (c)14 25 125 225

?120 72 192

= 75 11

164 64

9. (a) ? = 78.45 + 128.85 + 1122.25 = 1392 . 5510. (e) 5598 = ? + 2785

? = 5598 – 2785 = 2813

11. (b)87 37 375

?5 8 8

3219 375

?40 8

3219 1875 134440 40

168 3335 5

12. (d)5616? 3918 8

13. (e)420 288? 13528 32

14. (b) 484 ? 516

? 516 484 32

? 32 32 1024

15. (c)660 45 450 28

?100 100

= 297 + 126 = 42316. (b) The pattern of the number series is:

1 (d) 16 (b) 31 (c) 46 (c) 61 (d) 76 (b) 91 (a)2 (a) 17 (c) 32 (b) 47 (b) 62 (a) 77 (c) 92 (e)3 (c) 18 (d) 33 (e) 48 (e) 63 (c) 78 (a) 93 (b)4 (a) 19 (e) 34 (d) 49 (a) 64 (e) 79 (a) 94 (e)5 (b) 20 (e) 35 (a) 50 (d) 65 (b) 80 (e) 95 (c)6 (e) 21 (a) 36 (c) 51 (a) 66 (c) 81 (a) 96 (b)7 (d) 22 (b) 37 (c) 52 (d) 67 (b) 82 (e) 97 (d)8 (c) 23 (c) 38 (a) 53 (e) 68 (c) 83 (b) 98 (c)9 (a) 24 (c) 39 (d) 54 (c) 69 (a) 84 (c) 99 (d)10 (e) 25 (b) 40 (b) 55 (b) 70 (c) 85 (d) 100 (a)11 (b) 26 (b) 41 (e) 56 (a) 71 (d) 86 (e)12 (d) 27 (b) 42 (d) 57 (d) 72 (d) 87 (a)13 (e) 28 (a) 43 (b) 58 (c) 73 (c) 88 (c)14 (b) 29 (e) 44 (d) 59 (a) 74 (e) 89 (b)15 (c) 30 (d) 45 (e) 60 (b) 75 (b) 90 (d)

Answer Key

Page 358: yoursmahboob.wordpress.com SBI · yoursmahboob.wordpress.com iii P 101 Speed Tests for SBI Bank Clerk Exam 101 Speed Tests for SBI Bank Clerk Exam is revised and updated edition on

yoursmahboob.w

ordpress.com

92 101 SPEED TEST12 + 22 = 1616 + 23 = 2424 + 24 = 4040 + 25 = 72

17. (c) The patern of the number series is :9 + 10 = 1919 + 20 = 3939 + 40 = 7979 + 80 = 159

18. (d) The pattern of the number series is:8 + 32 = 1717 + 52 = 4242 + 72 = 9191 + 92 = 172

19. (e) The pattern of the number series is:7 × 1 + 1 = 88 × 2 + 2 = 1818 × 3 + 3 = 5757 × 4 + 4 = 232

20. (e) The pattern of the number series is:3840 4 = 960960 4 = 240240 4 = 6060 4 = 15

21. (a)75 3

100 7x

y 3 34 7x

y

3 4 47 3 7

xy

22. (b) Speed of the train

= Length of train and platform

Time taken tocrosseach other

275 275m/sec.

33550 18

kmph33 5 = 60 kmph

23. (c)TimeRate

CI P 1 1100

2945000 1 1

100 = 45000 × [(1.09)2 –1]

= 45000 × 0.1881 = 8464.5

24. (c)9 70.82; 0.78

11 9

5 40.83; 0.86 5

11 0.8513

Clearly, 7 4 9 5 119 5 11 6 13

25. (b) Ratio of the profit of Srikant and Vividh= 185000 : 225000 = 37 : 45Sum of the ratios = 37 + 45 = 82Total profit earned

= 82

900045

= 16400

26. (b) Father’s present age = 6x yearsSon's present age = x yearsAfter four years

6 4 44 1

xx 6x + 4 = 4x + 16

2x = 12 12

62

x

Son’s present age = 6 years27. (b) Let the number be x.

× 65 2140

100 5x x 13 2

14020 5

x x

13 8140

20x x

1404x

x = 4 × 140 = 560

560 3030%of 560 168100

28. (a) Let the original number be 10x + y where y > x. 10y + x – 10x – y = 27

9(y – x) = 27 y – x = 3 ....(i)

and x + y = 13 ....(ii)From equations (i) and (ii),y = 8 and x = 5

Original number = 5829. (e) M1D1 = M2D2

22 × 16 = 32 × D2

D2 = 22 16

32= 11 days

30. (d) Let the smallest odd number A be xx + x + 4 = 2 × 59

2x = 118 – 4 = 114114 57

2x

31. (c) 60% of 150 = 90. It means those who obtained either90 or more than 90 marks in the average of fivesubjects will be declared as passed. Therefore,the required number = 31 + 17 = 48

Page 359: yoursmahboob.wordpress.com SBI · yoursmahboob.wordpress.com iii P 101 Speed Tests for SBI Bank Clerk Exam 101 Speed Tests for SBI Bank Clerk Exam is revised and updated edition on

yoursmahboob.w

ordpress.com

93SOLUTIONS32. (b) Reqd no. = 52 + 41 + 13 = 10635. (a) No. of students who obtained more than or equal to

40% marks in Science= 67 + 22 + 14 = 103No. of students who obtained less than 60% marks inHindi = 19 + 59 + 47 = 125

Reqd % 125 103 100 17.60%

12536. (c) Meaningful words : ARE, EAR, ERA

37. (c)

1 1 1 1 1 1 1A D J E C T I V E

B C I F B S J U F

38. (a) na pa ka so birds fly very highri so la pa birds are very beautifulti me ka bo the parrots could flyThus high is coded as na.

39. (d) 1 2 3 4 5 6 7 8 9Difference = 8 – 4 = 4

40. (b) 1 2 3 4 5 6 7 8 9 10 11 12 13C O M P A T I B I L I T Y

Meaningful word L I M B

41. (e) As 2F H 2I G

2N P 2E CSimilarly,

2S U 2L J

2I K 2T R

42. (d) As L 8 and H 7A & I *T 4 R 3E $ E $

Similarly,H 7 A &I * L 8

43. (b) Others relate to ‘parts of tree’.44. (d) Since ‘potato’ is called Banana. Thus, ‘Banana’ grows

underground.45. (e) EI, EG, GI and NL.46. (c) Some suns are planets.

All planets are satellites.(I + A I-type)“Some suns are satellites”.

Conclusions I and II form Complementary Pair.Therefore, either I or II follows.

47. (b) All curtains are rods.

Some rods are sheets.(A + I No Conclusion)

48. (e) Some plugs are bulbs

All bulbs are sockets.(I + A I-type)“Some plugs are sockets”.Conclusion I is Converse of thi3s Conclusion.Conclusion II is Converse of the first Premise.

49. (a) All fishes are birds. (conversion)

All birds are rats.(A + A A-type)“All fishes are rats”.All birds are rats. (conversion)

All rats are cows.(A + A A-type)“All birds are cows”.This is Conclusion I.

50. (d) Some windows are doors.

All doors are roofs.(I + A I-type)“Some windows are roots”.

51. (a) P + S P is daughter of S.S – T S is father of T.Therefore, P is sister of T.

52. (d) P × Q P is wife of Q.Q – T Q is father of T.T is child of P and Q.The sex of T is not known.T is either son or daughter of P.

Page 360: yoursmahboob.wordpress.com SBI · yoursmahboob.wordpress.com iii P 101 Speed Tests for SBI Bank Clerk Exam 101 Speed Tests for SBI Bank Clerk Exam is revised and updated edition on

yoursmahboob.w

ordpress.com

94 101 SPEED TEST53. (e) P × S P is wife of S.

S ÷ T S is son of T.T is either father-in-law or mother-in-law of P.P ÷ S P is son of S.S × T S is daughter of TTherefore, T is father of P.P – S P is father of T.P + T P is daughter of TT ÷ S T is son of S.Therefore, T is father of P.

54. (c) P + Q P is daughter of Q.Q – S Q is father of S.Therefore, P is sister of S.P ÷ Q P is son of Q.Q × S Q is wife of S.Therefore, P is son of S.P ÷ Q P is son of Q.Q + S Q is daughter of S.Therefore, P is grandson of S.

55. (b) P + Q P is daughter of Q.Q × T Q is wife of P.Therefore, T is father of P.

56. (a) B 9; A 2; R *;N %; I #; S 457. (d) D 2; M @; B 9; N %; I #; A 6

Condition (i) is applied.58. (c) I $; J 8; B 9; R *; L £; G #

Condition (ii) is applied.59. (a) B 9; K ©; G $; Q 7; J ©; N %

Condition (iii) is applied.

60. (b) E £; G $; A 2; K 1; R *; L 3Condition (ii) is applied.

For (Qs. 61-65): Given information can be tabulated as follows

Person Sex Company SpecialisationA Male X FinanceB Male Z MarketingC Male Y EngineerD Female X HRE Male Y DoctorF Male Y MarketingG Female Z FinanceH Male Z HR

Thus, ‘G’ is a sister of ‘C’.66. (c) The number 49 is a perfect square of a natural number.

67. (b) 1 3 3 4 5 6 7 8P H Y S 1 C A L

Meaningful word HAIL68. (c) Obviously option (c) may be the conclusion of the

passage. Because the passage also talks about theliteral the actual sense of culture.

69. (a) Option (a) have the same summary hense it strengthensthe conclusion of the passage.

70. (c) Option (c) has the opposite sense of the passage. Thusit weakens the conclusion of the passage.

Page 361: yoursmahboob.wordpress.com SBI · yoursmahboob.wordpress.com iii P 101 Speed Tests for SBI Bank Clerk Exam 101 Speed Tests for SBI Bank Clerk Exam is revised and updated edition on

yoursmahboob.w

ordpress.com

95SOLUTIONS

SPEED TEST 98 : PRELIM TEST - 2

Hints & Solutions1. (d)

35

of 47 of

512

of 1015 = 3 4 5 10151015 1455 7 12 7

2. (c) 2 21.5 0.025 (?) 0.1 (?) 0.1 1.5 0.025

2(?) 0.1 0.0375 ? .0625 0.25

3. (a) 21.5 0.0375 2.25 0.15 0.33755. (e) 125% of 260 + ?% of 700 = 500

?% of 700 = 500 – 125% of 260?% of 700 = 175

175 100? 25700

6. (b) 45% of 750 – 25% of 480

= 45 750 25 480 337.5 120 217.5

100 100

7. (d) 8.5 3.8 (8.5 3.8) 4.775 75 75 758. (c) 39798 + 3798 + 378 = 43974

9. (b)7 3 13 7 111 11 2

= 40 80 3 160 61411 11 2 11 11

10. (e) 10801080 12 10 912 10

11. (d) First start with the option (d).1001× 25 = 250251001× 67 = 67067 etc.

Thus 1001 is the largest number which divides thenumbers of the type 25025, 67067 etc.

12. (b) A three digit number to be exactly divisible by 5 musthave either 0 or 5 at its units place.Such numbers will be 100, 105, 110, ..........., 995.First term = 100, last term = 995Let the required number be n.To find the value of n, we may use the following formulaof arithmetic progression,Tn = a + (n – 1) d.......... (1)Where d = common difference = 5Tn = 995a = 100Hence from (1)995 = 100 + (n – 1) 5 5n = 900n = 180Digits to be used = 0, 1, 2, 3, 4, 5, 6, 7, 8, 9.

13. (a) Let the principal be P, then amount after 12 years = 2P P)PP2(SI

Now, 100

P r tI P = 12100

P r

or 100 25 1r 8 %12 3 3

14. (d) Applying successive discounts of 10%, 12% and 15%on 100, we get 32.6785.088.09.0100

Single discount = 100 – 67.32=32.6815. (a) Let the prices of two houses A and B be Rs 4x and

Rs 5x, respectively for the last year.

1 (d) 16 (b) 31 (b) 46 (b) 61 (c) 76 (b) 91 (d)2 (c) 17 (b) 32 (e) 47 (e) 62 (b) 77 (c) 92 (c)3 (a) 18 (c) 33 (a) 48 (c) 63 (a) 78 (d) 93 (a)4 (a) 19 (b) 34 (c) 49 (d) 64 (d) 79 (a) 94 (a)5 (e) 20 (b) 35 (b) 50 (b) 65 (e) 80 (d) 95 (b)6 (b) 21 (c) 36 (a) 51 (a) 66 (b) 81 (e) 96 (c)7 (d) 22 (a) 37 (c) 52 (e) 67 (c) 82 (a) 97 (b)8 (c) 23 (b) 38 (b) 53 (a) 68 (b) 83 (c) 98 (b)9 (b) 24 (a) 39 (b) 54 (b) 69 (c) 84 (e) 99 (a)

10 (e) 25 (d) 40 (e) 55 (a) 70 (e) 85 (d) 100 (d)11 (d) 26 (d) 41 (d) 56 (d) 71 (b) 86 (b)12 (b) 27 (a) 42 (e) 57 (b) 72 (c) 87 (e)13 (a) 28 (a) 43 (a) 58 (c) 73 (d) 88 (a)14 (d) 29 (e) 44 (c) 59 (a) 74 (e) 89 (d)15 (a) 30 (c) 45 (d) 60 (e) 75 (e) 90 (c)

Answer Key

Page 362: yoursmahboob.wordpress.com SBI · yoursmahboob.wordpress.com iii P 101 Speed Tests for SBI Bank Clerk Exam 101 Speed Tests for SBI Bank Clerk Exam is revised and updated edition on

yoursmahboob.w

ordpress.com

96 101 SPEED TEST

Then, the prices of A this year = `(1.25 × 4x) and that ofB = (5x + 50,000)This year, Ratio of their prices = 9 : 10

109

000,50x5x425.1

450000x45x50 5x = 4,50,000 x = 90,000

Hence, the price of A last year was4x = 3,60,000

16. (b) Average speed = Totaldistance

Total time

=

2020

4060

80 =

80 = 32 km/h2.5

17. (b) Let the usual speed of the aeroplane be x km/h.

Then,)250x(

150021

x1500

Solving, we get x = 750 km/h18. (c) Sum of 40 instalments = S40 = 3600 = 20 (2a + 39d)

or 2a + 39d = 180 ...(i)Sum of 30 instalments = S30 = 2400 = 15 (2a + 29d)or 2a + 29d = 160 ...(ii)From (i) and (ii), we get a = 51 and d = 2The value of first instalment = 51

19. (b) Volume of the earth taken out = 30 × 20 × 12 = 7200 m3

470 m

470 m

30 mTank

30 m

Field

30 m

20 m

Area of the remaining portion (leaving the area of dugout portion)= 470 × 30 + 30 × 10 = 14100 + 300 = 14400 m3

Let h be the height to which the field is raised when theearth dug out is spread.Then, 14400 × h = 7200

h = 0.5 m

20. (b)198

–22

–32

–42

–52

194 185 169 144

21. (c) The first, third, fifth .... and second, fourth .... terms aregroups of consecutive natural numbers.

22. (a) 10609 7938.81

10609 103, by long division method, as below:

10310609100

10

203 6096090

Also, 7938.81 89.1 , by long division method, asbelow:

89.179388.8164

8

169 15381521

1781 17811781

0Hence 103 × 89.1 = 9177.3 9200

23. (b) 18.4% of 656 + 12.7% of 864 = 0.184 × 656 + 0.127 × 864= 120.704 + 109.728 = 230.432 231

24. (a) (98.4)2 + (33.6)2 = 9682.56 + 1128.96 = 10811.52 1081225. (d) Let there be x in place of question mark

So, 8959 8959

26.35 174 5 26.35 20

xx

26. (d) ? = 3739 + 164 × 27 = 3739 + 4428 820027. (a) Required distance covered = 2(6 × 415 + 500) metre

= 2(2490 + 500) metre = 5980 metre = 5.98 km28. (a) Total marks obtained by Amit

= 44 + 55 + 77 + 79 + 76 = 331

Required percentage = 331500 × 100 = 66.2

29. (e) The pattern of the number series is:7 × 2 – 3 = 1111 × 2 – 3 = 1919 × 2 – 3 = 3535 × 2 – 3 = 67

67 × 2 – 3 = 134 – 3 = 131

30. (c) The pattern of the number series is:5 + 12 = 66 + 22 = 1010 + 32 = 1919 + 42 = 35

35 + 52 = 35 + 25 = 60

31. (b) No. of students (of JNU) listening to Radio City

20 65120000 15600100 100

32. (e) The no. of Indraprastha students listening to FMRainbow = 120000 × 13% × 48%The no. of Jamia students listening toFM Gold = 120000 × 18% × 52%

Reqd. percentage

120000 13% 48% 100 66.66%120000 18% 52%

Page 363: yoursmahboob.wordpress.com SBI · yoursmahboob.wordpress.com iii P 101 Speed Tests for SBI Bank Clerk Exam 101 Speed Tests for SBI Bank Clerk Exam is revised and updated edition on

yoursmahboob.w

ordpress.com

97SOLUTIONS

33. (a) Indraprastha University34. (c) The no. of Indraprastha students listening to

Red FM 13 46120000 12 598100 100

The no. of Jamia students listening to Red FM

18 36120000 12 648100 100

Total students = (12 × 598) + (12 × 648)= 12 × 1246 = 14952

35. (b) Radio City36. (a) Since Badminton is not played by using a ball and also

called Volleyball. So, Volleyball is required answer.37. (c) T E C H N O L O G Y

1 2 3 4 5 6 7 8 9 10

Word formed C E N T38. (b) Arranging the number in ascending order, we get

1 2 3 4 5 7 8 94th number (Right) = 53rd number (left) = 3 Difference = 5 – 3 = 2

39. (b) Ja Ki mo pe at a frog's leap

mo la Ki so take a leap ahead

re bo Ja na insects are frog's diet

Thus, at is coded as pe.

40. (e) As W 9E %A 2K $

and S #K $I 7T @

Thus, W 9A 2I 7T @

41. (d) DG, IG and SN

42. (e) 1G F1L K1A B1D C

1I J1O P1L K1U V

1S R

43. (a) G ©Q *

R 1D 7

B 3N ©Condition III is applied.

44. (c) I £P 5E $B 3Q *S £Condition I is applied.

45. (d) R #M @A 9P 5S £I 1Condition II is applied.

46. (b) A 8

R 1

E $

M @

Q *

N 8Condition I is applied.

47. (e) K ©

J 2

B 3

P 5

R 1

D ©Condition III is applied.

48. (c) P × R P is son of R.R – S R is daughter of S.Therefore, P is grandson of S.

49. (d) P + R P is wife of R.R × S R is son of S.Therefore, P is daughter-in-law of S.P R P is father of R.R × S R is son of S.Therefore, S is wife of P.P + R P is wife of R.R S R is father of S.Therefore, S is child of P.

Page 364: yoursmahboob.wordpress.com SBI · yoursmahboob.wordpress.com iii P 101 Speed Tests for SBI Bank Clerk Exam 101 Speed Tests for SBI Bank Clerk Exam is revised and updated edition on

yoursmahboob.w

ordpress.com

98 101 SPEED TESTP R P is father of R.R S R is wife of S.Therefore, S is son-in-law of P.

50. (b) P – Q P is daughter of Q.Q S Q is wife of S.Therefore, S is father of P.

51. (a) P × Q P is son of Q.Q S Q is father of S.Therefore, P is brother of S.

52. (e) P R P is wife of R.R S R is father of S.The sex of S is not clear.S is either son or daughter of P.

53. (a) Lowest number = 189 + 1 = 190Highest number = 972 + 2 = 974

difference = 9 – 4 = 554. (b) Interchanging the digits of given number we get

279, 286, 981, 892 and 157.3rd highest number = 286 or 682

55. (a) Arranging all three digits of given number indescending order, we get

972, 862, 981, 982 and 751.3rd highest number = 972

56. (d) No pen is a mobile. (conversion)

Some mobiles are bottles.E + I O-type of conclusion“Some bottles are not pens”.Some mobiles are bottles. (conversion)

All bottles are papers.I + A I-tpye of conclusion“Some mobiles are papers”.

57. (b) All computers are radios. (conversion)

All radios are televisions.A + A A-type of conclusion“All computers are televisions”.Conclusion II is Converse of this Conclusion.

58. (c) Conclusion I and II from Complementary Pair. Therefore,either Conclusion I or II follows.

59. (a) Some fishes are moons. (conversion)

All moons are birds.I + A I-type of conclusion“Some fishes are birds”.Conclusion I is Converse of this Conclusion.

60. (e) All leaves are roots. (conversion)

All roots are flowers.A + A A-tpye of conclusion“All leaves are flowers”.Conclusion II is Converse of this Conclusion.All stems are roots. (conversion)

All roots are flowers.A + A A-type of conclusion“All stems are flowers”.Conclusion I is Converse of this Conclusion.

For (Qs. 61-67) : The given information can be tabulated asfollows:

Person Sex Relationship Profes sion

L Female W ife of M Architect

Father of P. Businessman

Husband of L.

Son of Q and O.

N Female Daughter-in-law of O and Q.

Housewife

O Female W ife of Q Pilot

P Male Son of M and L. Journalis t

Grandfather of P. Doctor

Husband of Q.

Father of M.

R – – Advocate

Male

Male

M

Q

Two married couples : LM and QO.66. (b) According to question

9 1 4 6 7 5 6 0 8 3

67. (c) ab, abc, abcd, abcde, abcdef, abcd e

68. (b) Clearly option (b) is best support by the passage.69. (c) The passage provides the facts about the preparation

of history. Hence option (c) is the assumption whichcan be implicit in the facts of the passage.

70. (e) Obviously. None of the given option is conclusion

which can be drawn from the facts of the passage.71. (b) Refer to the Ist sentence of para 1 ‘Govind’s father

........................ left large tract of land to Govind’.72. (c) Refer to the 3rd sentence of para 2 “To his

surprise.................into gold”.73. (d) Refer to the last two sentences of para 1 of the passage.74 (e)

Page 365: yoursmahboob.wordpress.com SBI · yoursmahboob.wordpress.com iii P 101 Speed Tests for SBI Bank Clerk Exam 101 Speed Tests for SBI Bank Clerk Exam is revised and updated edition on

yoursmahboob.w

ordpress.com

99SOLUTIONS

75. (e) Refer to the sixth sentence of para 2 of the passage‘You have..................during winter’.

76. (b) Refer to the third last sentence of para 3 of the passage.77. (c) The word spend (verb) means to pay out, to employ

(labour, time etc.) on some objects. For instance,How do you spend your leisure-time?Similarly, the word devote (verb) means to give time andenergy to something. For instance,He is devoted to his studies.Thus, spend and devote are synonymous.

78. (d) The word lie (verb) means to be at rest. For instance,She is lying on the bed.Similarly, the word remain (verb) means continue tostay. For instance,Twenty pens were distributed and the remaining werereturned.Thus, lying and remaining are synonymous.

79. (a) The word dismay (noun) means feeling of fear anddiscouragement. For instance,They were struck with dismay at the news.While, the word joy (noun) means feeling of happiness.Thus, both are antonyms.

80. (d) The word tend (verb) means look after. For instance,The nurse tended the injured persons.While, the word ignore (verb) means take no notice of.

For instance,Raman ignored the advice of his elders.Thus, both are antonyms.

86. (b) ‘Sent’ is the third form of verb ‘send’ in passive voice.88. (a) Replace ‘though’ by ‘As’ to express cause and effect

in the sentence.89. (d) It should be ‘and I answered all of them’ to properly

express cause and effect in the sentence.90. (c) Apply the adverb ‘highly’ before the adjective ‘paid’.

Page 366: yoursmahboob.wordpress.com SBI · yoursmahboob.wordpress.com iii P 101 Speed Tests for SBI Bank Clerk Exam 101 Speed Tests for SBI Bank Clerk Exam is revised and updated edition on

yoursmahboob.w

ordpress.com

100 101 SPEED TEST

SPEED TEST 99 : PRELIM TEST - 3

Hints & Solutions

1. (e) ? 75Squaring on both the sides, we get? = 75 × 75 = 5625

2. (d)21 7 1 ?8 72 171

or ? =21 7 1 38 72 171 19

3. (c)1 2 1? 4 6 52 3 3

3 4 2 9 1(4 6 5) 15 166 6 2

4. (d) ? = 792.02 + 101.32 – 306.76 = 586.585. (a) 300% of 150 = ? % of 600

or , ? of 600 = 45000 or, ? = 756. (d) 34.95 + 240.016 + 23.9800 = 298.9467. (b) 48.95 – 32.006 = ?

or, ? = 48.95 – 32.006 = 16.9448. (a) 3889 + 12.952 – ? = 3854.002

or ? = 3889 + 12.952 – 3854.002 = 47.959. (e) ? + 72.64 = 74.64 or ? = 74.64 – 72.64 = 2.00

10. (d) 6.25 0.0025 = ? or ? 6.25 25000.0025

1 (e) 16 (b) 31 (c) 46 (c) 61 (a) 76 (a) 91 (e)2 (d) 17 (b) 32 (a) 47 (e) 62 (b) 77 (e) 92 (b)3 (c) 18 (d) 33 (e) 48 (a) 63 (b) 78 (b) 93 (a)4 (d) 19 (d) 34 (e) 49 (b) 64 (a) 79 (d) 94 (b)5 (a) 20 (e) 35 (a) 50 (c) 65 (b) 80 (c) 95 (d)6 (d) 21 (c) 36 (c) 51 (d) 66 (d) 81 (c) 96 (e)7 (b) 22 (e) 37 (c) 52 (c) 67 (b) 82 (e) 97 (c)8 (a) 23 (b) 38 (b) 53 (a) 68 (c) 83 (a) 98 (c)9 (e) 24 (d) 39 (b) 54 (d) 69 (c) 84 (c) 99 (c)

10 (d) 25 (a) 40 (e) 55 (b) 70 (d) 85 (e) 100 (d)11 (d) 26 (c) 41 (b) 56 (a) 71 (e) 86 (d)12 (b) 27 (b) 42 (c) 57 (b) 72 (d) 87 (a)13 (b) 28 (b) 43 (b) 58 (c) 73 (c) 88 (b)14 (d) 29 (b) 44 (d) 59 (b) 74 (a) 89 (d)15 (b) 30 (d) 45 (a) 60 (a) 75 (e) 90 (a)

Answer Key

11. (d) Clearly, 1 1 7 77 77

71 is the smallest number..

12. (b) Let the sum be Rs x

Now, 8 (3 2)56100

x x = `700

13. (b) Let the cost price of the machine be `x.

Then, selling price at a profit of 10% = 11x10

And the selling price at a loss of 10% = `9x10

Consequently, we find that

11x 9x10 10

80

805x x 400

14. (d) Let the capacity of the jar be of x bottles.since 6 bottles were taken out from jar and 4 bottles ofoil poured into it 2 bottles were taken out

Page 367: yoursmahboob.wordpress.com SBI · yoursmahboob.wordpress.com iii P 101 Speed Tests for SBI Bank Clerk Exam 101 Speed Tests for SBI Bank Clerk Exam is revised and updated edition on

yoursmahboob.w

ordpress.com

101SOLUTIONS

Therefore, we have

4 325 4

x x

2x43x

54 x = 40

15. (b) Average speed = Totaldistance

Total time

=

2020

4060

80 =

8032 km/h

2.5

16. (b) Let the usual speed of the aeroplane be x km/h.

Then,)250x(

150021

x1500

Solving, we get x = 750 km/h17. (b) Using Venn Diagram

1535-15=20

42-15=27

Failed in only thefirst subject

Failedin boththe subjects

Failed in only thesecond subject

Thus, percentage of students who passed in bothsubjects= 100 – [(35 – 15) + (42 – 15) + 15] = 100 – (35 + 42 – 15)= 100 – (62) = 38%and percentage of students who failed in both subject= 15%Therefore, the percentage of students who passed ineither subject = 100 – (38 + 15) = 100 – 53 = 47%Hence, required no. of students who passed in either

subject but not in both = 2500 × 10047

= 1175

18. (d) Let the length and breadth of the rectangle be x andy cm, respectively.Then, (x – 4) (y + 3) = xy 3x – 4y = 12 ... (i)Also, (x – 4) = (y + 3) [sides of square]

x – y = 7 ... (ii)From (i) and (ii),x = 16 and y = 9Perimeter of the original rectangle = 2(x + y) = 50 cm

19. (d) The series is × 1 + 1 × 7, × 2 + 2 × 6, × 3 + 3 × 5...20. (e) The series is × 1 + 12, × 2 + 22, × 3 + 32.., ...21. (c) The series is × 1 + 2, × 2 + 3, × 3 + 4, ...

22. (e) 3.7 0.9989 ?96 95 95

or ? 3.7 0.998995 95

or ? 2795 95 or , ? = 2.70

23. (b)3.00110004.987

of 1891.992 = ?

or ? = 31005

of 1900 = 100 + 1140 = 1230

24. (d) 12.25 × ? × 21.6 = 3545.64

? = 3545.64

13.4 13264.6

25. (a) ? = 3 4096 = 3 16 16 16 1626. (c) M1D1 = M2D2

6 × 20 = 8 × D2

D2 = 6 20

8 = 15 days

27. (b)363 ?

? 3?2 = 363 × 3 = 121 × 32 = 112 × 32

? = 11 × 3 = 3328. (b) Raju's age at the time of marriage

= 17 + 3 + 2 + 1 + 3 = 26 years29. (b) Required number

= 2040 + 2300 + 2400 + 2200 + 2090 + 2120 = 1315030. (d) Required percentage growth

= 2250 2180 100 3.21%

2180

31. (c) Required %

= 2540 100

2500 2040 2100 2280 2540 2320

= 254000 % 18.43% 18%13780

32. (a) Required ratio = (2250 + 2480) : (2260 + 2440)= 4730 : 4700 = 473 : 470

33. (e) Required average number

= 2500 2250 2450 2150 2020 2300

6

= 13670 2278.33 2278

6

34. (e) 5x + 6x + 7x = 180°18x = 180°

Page 368: yoursmahboob.wordpress.com SBI · yoursmahboob.wordpress.com iii P 101 Speed Tests for SBI Bank Clerk Exam 101 Speed Tests for SBI Bank Clerk Exam is revised and updated edition on

yoursmahboob.w

ordpress.com

102 101 SPEED TEST

x = 180 1018

Sum of the smallest and the largest angles= 12x = 12 × 10 = 120°

35. (a) 232 = 529242 = 576

Required number = 536 – 529 = 7

43. (b) DesksChair

Tables

Mats

or

DesksChair

TablesMats

I. False II. True III. False

44. (d)

Sweets

Fruits

Pencil

Glasses

orSweets

Fruits

PencilGlasses

II. False. From both figures it is clear that either I or IIIfollows.

45. (a) Books

Flowers Chair Hammer

or Books

Flowers

ChairHam

mer

I. False II. False III. False

46. (c)

Roofs

Cameras PhotoStores

orRoofs

CamerasPhoto

Store

I. False II. False III. True

47. (e) Nails Horses

TabletsCrows

I. True II. True III. True63. (b) Since the passage talks about the defense system of

our country and role of the government option (b) maybe a conclusion of the passage.

Page 369: yoursmahboob.wordpress.com SBI · yoursmahboob.wordpress.com iii P 101 Speed Tests for SBI Bank Clerk Exam 101 Speed Tests for SBI Bank Clerk Exam is revised and updated edition on

yoursmahboob.w

ordpress.com

103SOLUTIONS

64. (a) Obviously option (a) strengthens the conclusion ofthe passage.

65. (b) Option (b) has the opposite sense. Thus it weakensthe conclusion of the passage.

66. (d) Except sugar, all others are liquid.67. (b) Meaningful Word DINE68. (c) Except 75, all other numbers are perfect cubes.

343 = 7 × 7 × 764 = 4 × 4 × 427 = 3 × 3 × 3216 = 6 × 6 × 6

69. (c)5 8 6 9 7 29 8 7 6 5 2

70. (d) 9 8 7 6 5 4 3 2 18 7 6 5 4 3 2 1

7 6 5 4 3 2 16 5 4 3 2 1

Page 370: yoursmahboob.wordpress.com SBI · yoursmahboob.wordpress.com iii P 101 Speed Tests for SBI Bank Clerk Exam 101 Speed Tests for SBI Bank Clerk Exam is revised and updated edition on

yoursmahboob.w

ordpress.com

104 101 SPEED TEST

SPEED TEST 100 : FULL MAIN TEST - 4

Hints & Solutions51. (e) 3 × ? + 30 = 0

30? 103

52. (a) ? = 40.83 × 1.02 × 1.2 = 49.97592

53. (e) ? = 1 3 1 223 6 13 7 2 7

10 45 3 22 2.443 7 2 7

54. (d) 1.5625 1.2555. (d) 3978 + 112 × 2 = ? 2

? = (3978 +224) × 2 = 8404

56. (e)23.7 1.3 ?10 10 10

23.7 1.3 ? b c b c10 10 [ a a a ]

2? 510 10

c5 2 b bc 1010 a a 10

57. (b) ? = 7589 – 3534 = 4055

58. (d) ? = 1225 35 75 5

59. (e) ? = 300 + (100 × 2) = 300 + 200 = 500

60. (a)5 1.6 2 1.4 8 2.8 5.2? 4

1.3 1.3 1.3

61. (e)2 1 13 7 55 5 4 = (3 + 7 – 5) +

2 1 15 5 4 =

8 4 5 75 520 20

62. (d) Let the number be x.

Now 34x

3x or 3

12x or x = 36

63. (a) 5 22 9 32 81 2592 .

1 (d) 26 (b) 51 (e) 76 (e) 101 (b) 126 (c) 151 (d) 176 (e)2 (b) 27 (a) 52 (a) 77 (d) 102 (a) 127 (a) 152 (a) 177 (b)3 (d) 28 (b) 53 (e) 78 (a) 103 (a) 128 (d) 153 (c) 178 (a)4 (e) 29 (b) 54 (d) 79 (c) 104 (e) 129 (d) 154 (e) 179 (e)5 (c) 30 (c) 55 (d) 80 (b) 105 (a) 130 (c) 155 (a) 180 (c)6 (d) 31 (d) 56 (e) 81 (c) 106 (d) 131 (c) 156 (e) 181 (b)7 (e) 32 (b) 57 (b) 82 (a) 107 (a) 132 (a) 157 (b) 182 (d)8 (c) 33 (a) 58 (d) 83 (b) 108 (e) 133 (e) 158 (d) 183 (a)9 (a) 34 (d) 59 (e) 84 (d) 109 (e) 134 (c) 159 (c) 184 (d)10 (e) 35 (b) 60 (a) 85 (e) 110 (c) 135 (b) 160 (e) 185 (c)11 (d) 36 (c) 61 (e) 86 (d) 111 (c) 136 (b) 161 (c) 186 (a)12 (b) 37 (a) 62 (d) 87 (a) 112 (a) 137 (c) 162 (a) 187 (a)13 (a) 38 (b) 63 (a) 88 (b) 113 (b) 138 (e) 163 (e) 188 (a)14 (e) 39 (c) 64 (d) 89 (e) 114 (d) 139 (b) 164 (b) 189 (d)15 (c) 40 (c) 65 (a) 90 (c) 115 (c) 140 (c) 165 (d) 190 (b)16 (c) 41 (d) 66 (b) 91 (a) 116 (b) 141 (d) 166 (d)17 (b) 42 (a) 67 (c) 92 (c) 117 (b) 142 (c) 167 (a)18 (a) 43 (b) 68 (d) 93 (a) 118 (e) 143 (c) 168 (c)19 (a) 44 (d) 69 (c) 94 (c) 119 (a) 144 (b) 169 (e)20 (a) 45 (b) 70 (a) 95 (c) 120 (a) 145 (c) 170 (d)21 (d) 46 (a) 71 (c) 96 (e) 121 (c) 146 (b) 171 (e)22 (d) 47 (a) 72 (d) 97 (d) 122 (d) 147 (b) 172 (b)23 (d) 48 (d) 73 (d) 98 (d) 123 (b) 148 (a) 173 (c)24 (c) 49 (a) 74 (b) 99 (e) 124 (e) 149 (e) 174 (a)25 (b) 50 (c) 75 (d) 100 (d) 125 (e) 150 (c) 175 (d)

Answer Key

Page 371: yoursmahboob.wordpress.com SBI · yoursmahboob.wordpress.com iii P 101 Speed Tests for SBI Bank Clerk Exam 101 Speed Tests for SBI Bank Clerk Exam is revised and updated edition on

yoursmahboob.w

ordpress.com

105SOLUTIONSDifference = 25 × 92 – 2592

= 2592 – 2592 = 0Hence, the numerical difference is 0.

64. (d) Let the third number be 100. Then, the first and secondnumbers will be 20 and 50, respectively.

Required % = 20 100 4050

65. (a) Let the cost price of an article be `100then, S.P. = 100 + 10 = `110If S.P. = 2 × 110 = 220

then, profit % = (220 100) 100

100 = 120%

66. (b) Ratio of investment= 3500 : 4500 : 5500 = 35 : 45 : 55 = 7 : 9 : 11Since, Ratio of investment is same as ratio of profit.Ratio of profit = 7 : 9 : 11Now, profit = 405

A’s share = 7 405 Rs 10527

= ` 105

67. (c) Part of the tank filled in one hour = 161

161

81

Hence, the tank will be filled in 16 hours.68. (d) Part of the tank filled by the three pipes working

simultaneously in one hour is = 6017

121

61

51

i.e. it takes 1760 hours to fill up the tank completely..

Now, 21

of the tank is filled with all the pipes open,

simultaneously together in 60 117 2 = hours

17131

69. (c) Any even number is given by 2n for all n Z, where Zis a set of integers. This is divisible by 9 if it form 9 × 2n= 18n, which is divisible by 18.For example, number 36 is even and sum of digits(3 + 6) is 9, which is divisible by 9. Hence, the number36 is divisible by 18.

70. (a) Volume of water left in the tank = × b × h = 20 × 7 × (10 – 2) = 1120 m3

71. (c) This is a series of prime number72. (d) Let x = 8

then 15 = 2x – 1 = y28 = 2y – 2 = z53 = 2z – 3 = mNext term in the pattern should be 2m – 4 = 2 x 53 – 4

= 10273. (d) It is a combination of two series, namely

24, 49, – 94; and 15, 31, 59, 58The two series correspond tox, (2x + 1), (4x – 1), (4x – 1), (4x – 2)Hence the missing term is4 × 24 – 1 = 95

74. (b) Add 3 after doubling the previous number.75. (d) The series exhibits the pattern of n2 + 1, n2 – 1,

alternatively, n taking values 1, 2.............76. (e) Let x be there in the question mark.

So, 33 860000 860000x xTaking log10 on boths the sides 3log10x = 5.9345log x = 1.9782, Taking antilog we get x 95[log x is nearly 2 so, x will be near to but less than 100]

77. (d)5 1 2 13 16 121 5 28 3 5 8 3 5

15 13 40 16 12 24120 =

195 640 288120

= 1123

9.35120

9.

78. (a)8769 87698769 82 44 82 328 = 26.73 27

79. (c) Let x be there in place of question mark so, x% of45.999 × 16% of 83.006 = 116.073.

We take 1646 83 116100 100

x

x × 0.46 × 13.28 = 116or x × 6.11 = 116

x = 18.98 19.80. (b) 12.998 × 27.059 × 17.999

Can be taken as near to 13 × 27 × 18 631881. (c) r2 = 2 × 11088

2227

r = 2 × 11088

r2 = 2 11088 7 705622

r = 7056 = 84 metre

Circumference = 2 r = 2 × 227 × 84 = 528 metre

82. (a) x + y = 8

82 16x y

x

x = 8y = 0Two digit number = 80

83. (b) Required ratio= (4220 – 2420) : 2420= 1800 : 2420 = 90 : 121

84. (d) The word SCENIC consists of 6 letters in which C comestwice.

Number of arrangements = 6!2!

= 6 5 4 3 2 1 3602

Page 372: yoursmahboob.wordpress.com SBI · yoursmahboob.wordpress.com iii P 101 Speed Tests for SBI Bank Clerk Exam 101 Speed Tests for SBI Bank Clerk Exam is revised and updated edition on

yoursmahboob.w

ordpress.com

106 101 SPEED TEST85. (e) 4x + 12y = 1520

Multiplying both sides by 2.5,10x + 30y = 1520 × 2.5= 3800

86. (d) Difference of production of C in 2010 and A in 2015 =5,00,000 tonnes

87. (a) Percentage increase of A from 2011 to 2012

55 40 100 37.5%40

88. (b) Percentage rise/fall in production for B

2011 2012 2013 2014 2015

9% –16.6% 10% – 9% 10%

Here, the maximum difference is from 2011 to 2012, whichis 10. And the second nearest to it is fall or rise of 5. So,undoubtedly the answer is 2012.

89. (e) Percentage production = 120

100 133.3%90

90. (c) Average production of A = 50Average production of B = 54.17Average production of C = 50Difference of production = 54.17 – 50 = 4.17

91. (a) Let the number be x.

63 2583100

x

2583 100 410063

x

45% of this number 454100 1845

10092. (c) Decimal equivalents of the fractions

4 60.8, 0.8575 7

2 90.22, 0.829 11

3 0.3758

Clearly,0.857 > 0.82 > 0.8 > 0.375 > 0.22

6 9 4 3 27 11 5 8 9

93. (a) Purchasing price of articles = (245 × 30) = 7350Total cost = (7350 + 980 + 1470) = 9800

CP per piece 9800 40245

`

SP = `50 per piece

Gain per cent 10 100 2540

94. (c) If difference between the compound interest and simpleinterest on a certain sum of money for 2 yr at r% rate is`x.

Sum 2100x

r

2 2100 25128 1288 2

128 25 25 200002 2

`

95. (c) Let the original number be 10x + y.Number obtained by interchanging the digits= 10y + x

10x + y – 10y – x = 18 9x – 9y = 18 x – y = 2

Again, x + y = 6 x = 4 and y = 2

Original number = 10 × 4 + 2 = 42

96. (e) Let Samar’s monthly salary be x.According to the question,x – (52 + 23)% of x = 4500 x – 75% of x = 4500 25% of x = 4500

4500 100 1800025

x `

97. (d) 55% of total students = 44

Total number of student 44 100 80

55Number of boys = 80 – 44 = 36

98. (d) Required average marks 45 75 40 78

45 40

3375 3120 6495 76.4185 85

99. (e) Perimeter = 2(Length + Breadth)48 = 2(5x + 3x)

48 316

x

Area = (5 × 3) × (3 × 3) = 15 × 9 = 135 m2

100. (b) Let the children in row = xThe children in a column = (x + 2)According to the question,x(x + 2) = 63x2 + 2x – 63 = 0x2 + 9x – 7x – 63 = 0x(x + 9) – 7(x + 9) = 0(x + 9) (x – 7) = 0

Hence, there are 7 children in each row.

Page 373: yoursmahboob.wordpress.com SBI · yoursmahboob.wordpress.com iii P 101 Speed Tests for SBI Bank Clerk Exam 101 Speed Tests for SBI Bank Clerk Exam is revised and updated edition on

yoursmahboob.w

ordpress.com

107SOLUTIONS101. (b) Clearly option (b) is best supported by the passage.102. (a) The given passage talks about the cause of corruption

and option (a) also has the same sense. Hence, it maybe the inference.

103. (a) Obviously, option (a) is the most effective statement inthe context of the given passage.

104. (e) According to the question, after interchanging thesymbols, equations becomes26 + 15 ÷ 5 – 4 × 2 = 26 + 3 – 8 = 29 – 8 = 21

105. (a) R P N W S U H D F L H J Q M O

+ 4 + 2 + 2 + 2 + 2

– 2 + 2 + 2 + 2 + 2

So, RPN is different from other four.

106. (d) D R10 students 5th

40 students

Left Right

So, D’s position from the left end of the row= 40 – (10 + 1 + 5 ) + 1 = 41 – 16 = 25 th

107. (a)

+1

RO B

CS P

+1+1

I

H

–1

+1

T L

M +1+1

R

Similarly,

H

+1

C A

+1+1 –1

E

+1

G R

+1+1

B I D Q S F H108. (e) According to the question, Scored order among the

students = A > D or D > A > B > C > ESo, E student is the lowest marked student.

109. (e) O V E R W H E L M

So, such number of pairs are OR, EH, LM, MR.110. (c) B R O W N

5 3 1 @ %

and M E A N

2 6 © %

R O B E

3 1 5 6111. (c) According to the question, in the above arrangements

seventh to the left of the nineteenth from the left is ©.112. (a) According to the question, in the above arrangements

following series is progressing (+ 2) and ( + l)step in hisgroup and next group comes (+ 3) step,So, next series comes in the group is U I *.

113. (b) According to the question, in the above arrangementsone vowel is immediately preceded by a symbol andimmediately followed by a number, Vowel is © A6

For Q.No.114-119P$Q P<QP©Q P>QP Q P=QP@Q P QP*Q P Q

114. (d) Given statements, M@D M K,D K D = K, K© R K >RConclusions :I. R $ M R < M (True)II. K M K= M (True)III. K $ M K < M (True)Hence, only either II or III and I is true from the givenstatements.

115. (c) Given statements F*T F< T,,T $ N T < N, N@ R N RConclusions :I. R $ T R < T (False)II. N ©F N>F (True)III. F $ R F < R (False)Hence, only conclusion II is true from the givenstatements.

116. (b) Given statements, B © N B> N,N@ R N R, F * R, F RConclusions :I. B © R B > R (True)II. F * N F N (False)III. R $ B R < B (True)Hence, only I and III conclusion are true from the givenstatements.

117. (b) Given statements, D $ M D< M,M * B M < B, B J B =JConclusions :I. J © D J > D (True)II. B @ D B D (False)III. J @ M J M (True)Hence, only I and III conclusion are true from the givenstatements.

118. (e) Given statements, W K W =K,K © F K >F, F $ M F < MConclusions :I. M © K M > K (False)II. W @ F W F (False)III. F @ W F W (False)Hence, any conclusion is not true from the givenstatements.

119. (a) Given statements, F @ T F T,T,T K T =K, K * D K < DConclusions :I. D @ F D F (False)II. F @ K F K (True)III. D @ T D T (True)Hence, conclusion II and III are true from the givenstatements.

Page 374: yoursmahboob.wordpress.com SBI · yoursmahboob.wordpress.com iii P 101 Speed Tests for SBI Bank Clerk Exam 101 Speed Tests for SBI Bank Clerk Exam is revised and updated edition on

yoursmahboob.w

ordpress.com

108 101 SPEED TESTFor Q. 120-125.

Sitting arrangements of 8 persons would be as follows :D

Q

P

A

RT

B

M

120. (a) From the above arrangements, M is to the immediateright of D.

121. (c) From the above arrangements, T is second to the rightof M.

122. (d) From the above arrangements, P is second to the left of D.123. (b) From the above arrangements, M is third to the right of P.124. (e) From the above arrangements, P and R are the immediate

neighbours of A.125. (e) From the above arrangements, in the following pairs, P

is immediate to the right of A.For Q. No. 126-131.126. (c) According to the question, Venn-Diagram is

cups

Benches

Drums

Kites

or cups

Benches

Drums

Kites

Conclusions :I. Some kites are cups. (False)II. Some kites are benches. (True)III. Some drums are·cups. (False)So, only conclusion II follows, given statements.

127. (a) According to the question, Venn·Diagram is

Bikes Cars Trains Buses

or Bikes

Cars

TrainsBuses

or

Bikes

Cars

TrainsBuses

Conclusions :I. Some buses are cars. (False)II. Some trains are bikes. (False)III. Some buses are bikes. (False)So, none conclusion follows given statements.

128. (d) According to the question, Venn-Diagram is

Dogs

Cats

Rats

Mats or Dogs

Cats

Rats

Mats

Conclusions :I. Some mats are cats. (True)II. Some mats are dogs. (False)III. Some rats are cats. (True)So, conclusion I and III follow given statements.

129. (d) According to the question, Venn-Diagram is

Pins

SticksRingsRods

Conclusions :I. Some rings are pens. (True)II. Some roads are sticks. (True)III. Some roads are pens. (True)So, all I, II, III conclusions follow, given statements.

130. (c) According to the question, Venn-Diagram is

Tables Chairs

Hou

ses

Tent

s

Conclusions :I. All houses are chairs. (False)II. Some tents are chairs. (True)III. Some houses are tables. (True)So, conclusion II and III follow given statements.

131. (c) According to the question, Venn-Diagram is

Boxes Walls Roads

Rivers

or

Page 375: yoursmahboob.wordpress.com SBI · yoursmahboob.wordpress.com iii P 101 Speed Tests for SBI Bank Clerk Exam 101 Speed Tests for SBI Bank Clerk Exam is revised and updated edition on

yoursmahboob.w

ordpress.com

109SOLUTIONS

Boxes Walls Roads

Rivers

Conclusions :I. Some rivers are walls. (False)II. Some roads are boxes. (False)III. No wall is river. (True)So, only conclusion III follows given statements.

132. (a) M A I N D E A R

9 3 6 4 8 5 3 2Therefore,M E N D

9 5 4 8133. (e) D R E A M I N G

+1 –1

B F S E F M H LSimilarly,T R E A T I S E

+1 –1

B F S U D R H S

134. (c) 5 3 1 4 6 9 71 3 4 5 6 7 9

135. (b) C > E, A > B > DNow, A > B > D > C > E

136. (b)30 m

20 mW

N

E

S

137. (c) Meaningful wordsTOUR, ROUT

138. (e) 48 Q 12 R 10 P 8 W 4 = ?? = 48 ÷ 12 × 10 – 8 + 4? = 4 × 10 – 8 + 4

? = 40 – 8 + 4 = 36139. (b) Second Highest Number

7 3 9

140. (c) green grass everywhere dik pa sok

cow eats grass nok ta pa

The code for 'cow' is 'nok' or 'ta'151. (d) The reason behind the question is not mentioned in

the passage.152. (a) Refer to the second sentence of the first para of the

passage that the king had to agree to a contract.......being a king.

153. (c) Refer to the sentence that the island was covered.......discovered dead bodies.......past kings of the secondlast sentence of the third para of the passage.

154. (e) Refer to the fourth sentence that in the firstmonth.......trees were cut down.......of the fourth paraof the passage.

155. (a) Refer to the last sentence “I turned the deadlyisland..............a beautiful abode.......peacefully” of thesixth para of the passage.

156. (e) From reading the passage thoroughly we come to theconclusion that the king was intelligent, foresight andcunning as he made deadly island a beautiful place tolive in with all luxuries of the kingdom.

157. (b) Refer to the sentence that ‘I know.......complete theyear’ of the statement made by king in the fourth paraof the passage.

158. (d) Refer to the second last sentence of the fourth parathat he sent all the.......for storage.......of the passage.

159. (c) Refer to the third sentence that however, the king.......leavethe kingdom.......of the last para of the passage.

160. (e) The moral of the story is ‘Always think and planahead’.

161. (c) The word contract means agreement. Compact, bond,deal, bargain etc. are the most similar word in meaning.

162. (a) The word bidding means order, command, wish, desire,request, direction etc. Hence wishing is most similar inmeaning of the given word.

163. (e) The word abode means house, dwelling, residence andhabitation etc.

164. (b) The word survived means remained alive, lived,endured etc. Hence died is the most opposite word inmeaning.

165. (d) The word vicious means brutal, savage, dangerous,cruel etc. Its opposite word will be gentle, kindly,harmless etc.

166-170. Sentences are re-arranged in order as DACFBE. A motherduck is the clue and qualifier of a sentence which isfollowed by A, C, F and finally B and E.

171. (e) Here, too is used as emphatic word. Lata was so scaredthat she could not go home alone. Hence, no correctionis required.

172. (b) The structure of sentence is subject + was/were + thirdform of verb + object. Thus, Riya was dressed to kill.

173. (c) The given sentence is the statement of simple pasttense. Hence it should be ‘worried’ instead of worries.

174. (a) The phrase ‘let off’ means to give them only a lightpunishment.

175. (d) It should be ‘took’ instead of ‘take’.

Page 376: yoursmahboob.wordpress.com SBI · yoursmahboob.wordpress.com iii P 101 Speed Tests for SBI Bank Clerk Exam 101 Speed Tests for SBI Bank Clerk Exam is revised and updated edition on

yoursmahboob.w

ordpress.com

110 101 SPEED TEST

SPEED TEST 101 : MAIN TEST - 5

Hints & Solutions21. (a) Iran Recently, India has offered to invest up to 20 billion

US dollars in Iran's energy industry, subject to provisionof concessional rights.The two nations, also discusseddevelopments regarding Farzad-B gas field. .

22. (a) Telangana Mission Bhagiratha is the water grid projectin Telangana State of India, which aims at reaching outdrinking water supply to even remotest place in theState.

23. (a) simplification of income tax laws24. (a) Subhash Palekar, popularly known as Krishi ka Rishi

(the "sage of agriculture"), is a famous exponent ofnatural farming.

25. (c) The Gujarat Government has launched the state-wide"Maa Annapurna Yojna (Scheme)" at Sanand,Ahmedabad to provide cereals at concessional ratesto poor and middle class families under the NationalFood Security Act (NFSA). This scheme will cover BPL,Antyodaya and APL families.

26. (a) India's fastest train Gatimaan Express "GatimaanExpress" has been flagged off from Hazrat Nizamuddin

Railway station to Agra Cantt. Station. The traincompleted its 200 km journey to Agra within 100minutes. It is capable of running at a maximum speed of160 kmph.

27. (b) The world's first 'White Tiger Safari' has recently openedfor public at Mukundpur in Satna district, MadhyaPradesh. The first of its kind safari has cost Rs. 50 croreand is spread over an area of 25 hectares. It is home tothree white tigers including one male, two females andtwo Royal Bengal tigers.

28. (d) Unakoti hill is an ancient Shaivite place of worship withhuge rock reliefs celebrating Shiva. It is the prime touristspot of Unakoti Tripura District in the KailashaharSubdivision in the North-eastern Indian state of Tripura..

29. (b) The Central Industrial Security Force (CISF) RaisingDay is celebrated every year on March 10. On this day,in 1969, the CISF was set up under an act of theParliament of India with a strength of 2,800. It is directlyunder the Union Ministry of Home Affairs and itsheadquarters are at New Delhi.

1 (d) 26 (a) 51 (d) 76 (a) 101 (d) 126 (a) 151 (a) 176 (a)2 (c) 27 (b) 52 (b) 77 (d) 102 (b) 127 (b) 152 (e) 177 (e)3 (c) 28 (d) 53 (a) 78 (b) 103 (b) 128 (a 153 © 178 (b)4 (c) 29 (b) 54 (c) 79 (a) 104 (c) 129 (e) 154 (a) 179 (d)5 (a) 30 (a) 55 (e) 80 (e) 105 (a) 130 (a) 155 (d) 180 (d)6 (c) 31 (c) 56 (d) 81 (c) 106 (e) 131 (d 156 (e) 181 (e)7 (a) 32 (c) 57 (b) 82 (a) 107 (d) 132 (c) 157 (d) 182 (a)8 (d) 33 (a) 58 (c) 83 (d) 108 (c) 133 (c) 158 (c) 183 (c)9 (d) 34 (c) 59 (e) 84 (e) 109 (c) 134 (a) 159 (b) 184 (a)10 (d) 35 (c) 60 (e) 85 (b) 110 (b) 135 (e) 160 (b) 185 (c)11 (b) 36 (a) 61 (b) 86 (c) 111 (a) 136 (d) 161 (b) 186 (c)12 (a) 37 (b) 62 (c) 87 (b) 112 (a) 137 (d) 162 (c) 187 (c)13 (d) 38 (c) 63 (b) 88 (a) 113 (c) 138 (b) 163 (d) 188 (a)14 (b) 39 (d) 64 (b) 89 (d) 114 (c) 139 (a) 164 (e) 189 (d)15 (a) 40 (c) 65 (a) 90 (e) 115 (d) 140 (c) 165 (a) 190 (a)16 (d) 41 (a) 66 (c) 91 (b) 116 (d) 141 (a) 166 (d)17 (c) 42 (c) 67 (b) 92 (c) 117 (e) 142 (c) 167 (e)18 (e) 43 (b) 68 (a) 93 (a) 118 (c) 143 (d) 168 (a)19 (c) 44 (a) 69 (a) 94 (e) 119 (a) 144 (c) 169 (b)20 (a) 45 (a) 70 (c) 95 (d) 120 (c) 145 (a) 170 (c)21 (a) 46 (c) 71 (c) 96 (c) 121 (b) 146 (a) 171 (d)22 (a) 47 (b) 72 (d) 97 (d) 122 (c) 147 (b) 172 (c)23 (a) 48 (b) 73 (b) 98 (b) 123 (b) 148 (c) 173 (a)24 (a) 49 (b) 74 (c) 99 (b) 124 (b) 149 (d) 174 (b)25 (c) 50 (a) 75 (c) 100 (a) 125 (d) 150 (a) 175 (a)

Answer Key

Page 377: yoursmahboob.wordpress.com SBI · yoursmahboob.wordpress.com iii P 101 Speed Tests for SBI Bank Clerk Exam 101 Speed Tests for SBI Bank Clerk Exam is revised and updated edition on

yoursmahboob.w

ordpress.com

111SOLUTIONS

30. (a) A 108-feet tall idol of Lord Rishabhdeva, the firstTeerthankar of Jains, carved out of a single rock, hasentered the "Guinness World Records" as the world'stallest Jain statue. The impressive statue is located atopMangi Tungi mountain near Teharabad village ofBaglan tehsil in Nashik district of Maharashtra. Beforethis statue, the 57 feet idol of Lord Bahubali inShravanabelagola in Karnataka was considered theworld's tallest Jain statue.

31. (c) The President of India Pranab Mukherjee has declaredKerala as India's first digital state during the launch ofdigital empowerment campaign at Kozhikode, Kerala.The campaign aims to develop better digitalcommunication across the state by 2020. Kerala is thefirst Indian state to successfully link its villages withbroadband connectivity under the National OpticalFibre Network (NOFN) programme.

32. (c) February 28 The National Science Day is celebrated inIndia on February 28 every year to mark the discoveryof the Raman effect by Indian physicist C V Raman.The 2016 theme is "Scientific Issues for Developmentof the Nation" that aims at raising public appreciation.

33. (a) The NDDB foundation for Nutrition (NFN) haslaunched its novel 'Gift Milk' initiative to providenutritional support for school children. NDDB's whollyowned subsidiary, Indian Immunologicals Limited (IIL)partnered with NFN and launched the initiative in theZ.P. School, Laxmapur village, Ranga Reddy District,Telangana. IIL has adopted this school as part of itsCSR activity. .

34. (c) India The World's first fast-acting anti-rabies drug"Rabies Human Monoclonal Antibody (RMAb)" willbe launched in India. The drug has been developed byPune-based Serum Institute of India (SII) and US-basedMassBiologics of the University of MassachusettsMedical School.

35. (c) Prof. Goverdhan Mehta has been conferred with theFederal Republic of Germany's prestigious"Bundesverdienstkreuz" (the Cross of the Order ofMerit). He is a well-known researcher in ChemicalSciences & former Vice-Chancellor of the University ofHyderabad (UoH). He is a Fellow of the Royal Society(FRS), a Foreign Member of the Russian Academy ofSciences and has been conferred with 'Chevalier de laLegion d'Honneur' by the President of France.

36. (a) GVK Biosciences The Hyderabad-based GVKBiosciences has received the Global CSR Excellence &Leadership Awards in the category of BestEnvironment-Friendly Project for its "Go Green, GrowGreen" plantation drive. The global CSR awards areconstituted by the World CSR Congress every year onthe World CSR Day and are presented to companieswho believe in the value of sustainable corporate socialresponsibility. .

37. (b) Gujarat Nargol port is located in Gujarat. Indian Railwaysproposes to undertake implementation of rail connectivityfor the ports of Nargol and Hazira under PPP

38. (c) Madhya Pradesh The 42nd edition of world fameKhajuraho Dance Festival has started at Khajuraho inChhatrapur District, Madhya Pradesh. This culturalfestival highlights the richness of the various Indianclassical dance styles like Kathak, Bharathanatyam,Odissi, Kuchipudi, Manipuri and Kathakali withperformances of some of the best exponents in the field.

39. (d) The Maharashtra State Economic Development Counciland Naik Environmental Research Institute Ltd inNashik have formed Deepak Naik committee to worktowards reviving the river Godavari. Naik is thechairman of Maharashtra state Economic developmentcouncil. He is an expert in river issues.

40. (c) India's first ever Gender Park has been inaugurated byPresident Pranab Mukherjee at Kozhikode, Kerala. Thepark is set up to promote research and other initiativesto enable total gender equality. The Gender Institute atthe park would focus on learning, research and capacitydevelopment, as part of supporting the efforts of theState and Central governments in ensuring an inclusive,discrimination free society.

41. (a) The World Day of Social Justice is observed onFebruary 20 to recognize the need to promote efforts totackle issues such as poverty, exclusion andunemployment. The 2016 theme is "A Just Transition -environmentally sustainable economies and societies".

42. (c) The Odisha Government has launched the Pucca GharYojana for construction workers. The pucca houseswill be built for 50,000 construction workers during thecurrent financial year with a cost of Rs. 500 crore. Thestate government has decided to provide an assistanceof Rs.1 lakh to each registered construction worker ofthe state for construction of his/her house under theYojana. A construction worker registered with theOdisha Buildings and Construction Workers' WelfareBoard for the three years can avail the benefit.

43. (b) Arunachal Pradesh is also known as the "land of thedawn-lit mountains". Arunachal Pradesh was grantedstatehood on 20th February, 1987. . 7. Utkarsha Banglascheme has been launched by the West Bengalgovernment, for what purpose? [A]To train schooldropouts [B]To stop human trafficking [C]To promoteentrepreneurship among women [D]To promote organicfarming Hide Answer To train school dropouts TheWest Bengal government has recently launchedUtkarsh Bangla Scheme, a welfare scheme for school-dropouts. The scheme is aimed at giving vocationaltraining to school dropouts who will be trained free ofcharge in tailoring, driving, repairing television andothers. Each youth will get training ranging from 400 to1200 hours.

Page 378: yoursmahboob.wordpress.com SBI · yoursmahboob.wordpress.com iii P 101 Speed Tests for SBI Bank Clerk Exam 101 Speed Tests for SBI Bank Clerk Exam is revised and updated edition on

yoursmahboob.w

ordpress.com

112 101 SPEED TEST44. (a) The first train connecting China to Iran through the ancient

Silk Road has arrived in the Iranian capital after travelingover 10,000 kilometers. The train had started its journeyfrom China's eastern Zhejiang province and it took 14 daysto reach final destination. It had passed through two CentralAsian countries Kazakhstan and Turkmenistan

45. (a) The Indian Institute of Petroleum and Energy (IIPE)has been established at Visakhapatnam by the Ministryof Petroleum and Natural Gas (MoPNG), Governmentof India. This institute has the mandate to lead thenation forward in education, research and trainingrelated to all areas of Petroleum and Energy.

46. (c) 47. (b) 48. (b) 49. (b) 50. (a)

51. (d)5 4 3? 222 378 9 5

52. (b) Let the number be x56 450 300

100x or x = 300 – 252 = 48

53. (a) 1.5 3.5 ?(27) (21) (27)

? = 5 ( )x y x ya a a54. (c) Let the number be x.

27.06 × 25 – x = 600or, x = 676.5 – 600 = 76.5

55. (e)39 30 45 1? 118 13 4 4

56. (d) 4 5 4 3 5 2 43 21 18 8 8 88 8

? = 4

57. (b) –(a – b) . x = b – aPut x replacing ‘?’ (question mark)or [ ( ) ] [ ]a b x b a or ( )a b x a b

or 1a bxa b

58. (c) ? ( )a b a bor .( )a b x a b[Put x replacing ‘?’ (question mark)]or a + = x. (a + b)or x = – 1

59. (e) |? + 14| = 11 or ? + 14 = 11 or – 11 ? = – 25 or – 3

60. (e) 16 + 26 × 2 = 16 + 52 = 68

61. (b)12 10.1008, 0.1

119 10

102.0394 and 101.0

697

Thus, 101 is the least.

62. (c) Let there be n points marked on the plane.Total number of line segments = 2

n C =10

102

)1n(n or 020nn2

or 0)4n)(5n( or n = 5 [n = – 4 is rejected]

63. (b) Let the sum of money be ` x.

Now, 3

8 1100

rx x

or, 3

31 (2)100

r or 1 2100

r

Again, let the sum becomes 16 times in n years. Then,

16 1100

nrx x

n216 or n4 22 or 4n64. (b) Let the cost price of machine be ` 100

SP of machine at a profit of 10% = ` 110SP of machine at a loss of 10% = ` 90If SP is (110 – 90) = ` 20 less then CP = ` 100Therefore, if SP is ` 40 less, then

CP = 10040

20` 200

65. (a) Let us work with the options.For (a), total cost = 362013024807For (b), total cost = 375013034807For (c), total cost = 187013074802Hence, option (a) is correct.

66. (c) Relative speed of the trains = (40 + 20) = 60 m/sDistance = (120 + 120) = 240 mTime taken by trains to cross each other completely

= 240 4s60

Larger the no. of cogs (tooth of wheel) of wheel,lesser will be that no. of revolution made by it.

67. (b) Let Sunil finishes the job in x hours.

Then, Ramesh will finish the job in 2x hours.

We have, 6x32xx

Therefore, Sunil finishes the job in 6 hours and Rameshin 3 hours.

Work done by both of them in 1 hour = 21

31

61

They together finish the piece of work in 2 hours.68. (a) Let the present age of the man = x years

Now, (x + 15) = 4 (x – 15)or 3x = 75 or x = 25 years

69. (a) Area of the outer rectangle = 19 × 16 = 304 m2

2m

2m

2m2m

15

12

Verandah

Floor of room

Area of the inner rectangle = 15 × 12 = 180 m2

Required area = (304 – 180) = 124 m2

Page 379: yoursmahboob.wordpress.com SBI · yoursmahboob.wordpress.com iii P 101 Speed Tests for SBI Bank Clerk Exam 101 Speed Tests for SBI Bank Clerk Exam is revised and updated edition on

yoursmahboob.w

ordpress.com

113SOLUTIONS

70. (c) The terms exhibit the pattern 21, 22, 23 and so on.71. (c) Try the pattern n3 – 1, n = 1, 2, .......72. (d) Pattern is 22, 42 – 1,42 , 62 – 1, 62 and so on.73. (b) Can you see that the pattern is

12, 23, 32, 43, 52, 63, 72

74. (c) Note that0 = 13 – 1 6 = 23 – 224 = 33 – 3

75. (c) ? 1223.9975 3476. (a) ? = 503 × 201 = 101103 10110077. (d) ? = 1205 2.5 = 482 48078. (b) ? = 22020 0.011 = 2001818 2002000

79. (a) ? 20800 144 12

80. (e)8 15 22

?5 7 3

176

25.147 25

81. (c) Required average height

142 156 162 1784

638 159.5 cm4

82. (a) Manoj's monthly income

= 2.16 10000012

= ` 18000

Pratul's monthly income

= 18000 × 14

= ` 4500

Pratul's annual income= 12 × 4500 = ` 54000

83. (d) The pattern of the number series is:17 + 92 = 17 + 81 = 9898 + 72 = 98 + 49 = 147147 + 52 = 147 + 25 = 172172 + 32 = 172 + 9 = 181

181 + 12 = 181 + 1 = 18284. (e) The pattern of the number series is:

11 + 8 = 1919 + 12 (= 8 + 4) = 3131 + 16 (= 12 + 4) = 4747 + 20 (= 16 + 4) = 67

67 + 24 (= 20 + 4) = 9185. (b) The pattern of the number series is:

748 – 460 = 288460 – 316 = 144316 – 244 = 72

244 – 208 = 36 ? = 208 – 18 = 190

86 (c) Required ratio = 3 32.27 : 1.254 10

= 1.7025 : 0.375 = 227 : 50

87. (b) Required percentage = 1.08 100 343.14

88. (a) Total number of candidates appearing from all the citiestogether= (1.25 + 3.14 + 1.08 + 2.27 + 1.85 + 2.73) lakhs= 12.32 lakhsNumber of candidates passing from City F

= 7 2.73 1.5925

12 lakh

Required percentage = 1.5925 100 12.9312.32

89. (d) Number of failures:

City A 3

10 × 1025 lakhs = 0.375 lakh

City B 38 × 3.14 lakhs = 1.1775 lakh

City C 59 × 1.08 lakhs = 0.6 lakh

City D 34

× 2.27 lakh = 1.7025 lakh

City E 25 × 1.85 lakhs = 0.74 lakh

City F 512

× 2.73 lakh = 0.455 lakh

90. (e) Number of passed students from City E

= 3 1.855 lakhs = 111000.

91. (b) Interest = `8791 – 7450 = 1341

Time 100 1341 100 2235 3 yr

7450 6 745IP R

92. (c) 36% of x – 12% of x = 82.82

or36 12– 82.32

100 100x x

or 9 3– 82.3225 25

x x

or 6 82.3225

x

or 6x = 82.32 × 25

or 82.32 25 343

6x

93. (a) 16 watches + 21 calculators = 3048532 watches + 42 calculators = 2 × 30485 = 60970

94. (e) x2 + (66)2 = 4840or x2 + 4356 = 4840or x2 = 4840 – 4356

Page 380: yoursmahboob.wordpress.com SBI · yoursmahboob.wordpress.com iii P 101 Speed Tests for SBI Bank Clerk Exam 101 Speed Tests for SBI Bank Clerk Exam is revised and updated edition on

yoursmahboob.w

ordpress.com

114 101 SPEED TESTor x2 = 484or x = 22

95. (d) Let the money received by A, B and C be `3x, 4x and5x, respectively and money received by P and Q be `2yand y, respectively.

4x – y = 1050 …(i)Since, we cannot from another equation here. So, we cannotsolve it.

96. (c) x2 + y2 = (x + y)2 – 2xy= (18)2 – 2 × 72= 324 – 144= 180

97. (d)484 100 64.533750

= 65% about

98. (b) CI = p 1 –1100

tr

335000 1 –1100

`

103 103 1035000 –1

100 100 100`

1092727 –10000005000

1000000`

9272750001000000

`

= `5000 × 0.092727= 463.635= `464 (about)

99. (b) In 1 day, Girish attends 7 customer's callIn 60 days, Girish attends 60 × 7 = 420 calls

100. (a) Suppose that numbers are x and yFrom question2x + 3y = 126 …(i)3x + 2y = 144 …(ii)Eq. (i) multiple by 3 and Eq. (ii) multiple by 2 and thensubtract Eq. (ii) from Eq. (i), we get

5y = 90or y = 18Put the value of y in Eq. (i)2x + 3y = 126or 2x + 3 × 18 = 126or 2x + 54 = 126or 2x = 126 – 54 = 72or x = 36

x = 36, y = 18Smaller number = 18

101. (d) treeis very beautiful ka na da ta

this is strong tree na pa sa kaThe code for ‘beautiful’ is either ‘da’ or ‘ta’.

102. (b) G I V E F A I L

5 1 @ % 2 1 9©

Therefore, L E A F

9 © 2 %

103. (b) 3 8 6 7 4 5 1 0 9 2

3rd from right104. (c) All others are parts of a Car.

105. (a) S U B S

–1

R A T R

+1

T

U

A N C E

+1

F D O BSimilarly,

T E N T

–1

S M D S

+1

H

I

O U S E

+1

F T V P106. (e) 519 915; 364 463;

287 782; 158 851;835 538Second lowest number

538 83 5107. (d) Second highest number

581 1 5 8108. (c) Highest number 835

Lowest number 158Required difference 5 – 3 = 2

(109–113) : Sitting arrangement

MD

W

Q

BA

P

R

109. (c) Q is second to the left of D.110. (b) D is third to the left of P.111. (a) W is to the immediate right of W.112. (a) D and M are the first and second respectively to the

right of W.113. (c) B is sitting to the immediate immediate right of A.

(114– 118) : = $ @

#

Page 381: yoursmahboob.wordpress.com SBI · yoursmahboob.wordpress.com iii P 101 Speed Tests for SBI Bank Clerk Exam 101 Speed Tests for SBI Bank Clerk Exam is revised and updated edition on

yoursmahboob.w

ordpress.com

115SOLUTIONS

114. (c) W @ T W TT©M T MM$D M < D Therefore, W T M < D

ConclusionsI. W # D W D : Not TrueII. W @ M W M : Not TrueIII. D # T D T : True

115. (d) F R F RR M R MM$D M DTherefore, F= R M DConclusionsI. D # R D > R : TrueII. D # F D > F : TrueIII. M @ F M F : True

116. (d) V © M V MM B M BB$F B FTherefore, V M B FConclusionsI. F # M F> M : TrueII. B @ V B V : TrueIII. F #V F V : True

117. (e) D # N D > NN@ B N BB F B FTherefore, D> N B = FConclusionsI. F$D F D : TrueII. N # F N > F : Not TrueIII. N F N = F : Not TrueN is either greater than or equal to F. Therefore, either Ior II is true.

118. (c) R $T R TT # K T > KK @ M K MTherefore, R < T > K MConclusionsI. R $M R M : Not TrueII. T # M T > M : TrueIII. R$K R < K : Not True

119. (a) 21st from the right end is B and sixth to the right of Bis 8.Trick : Required answer = 21 – 6 = 15th from the right,i.e., 8.

120. (c) Symbol Vowel ConsonantSuch combinations are:

@ E J ; % A V

121. (b) Number Symbol Number

There is only one such combination: Q $6

122. (c) Number Consonant Symbol

Such combinations are:1H % ; 2 Q$

123. (b) 1 38 1 D1 27 51 3P 2 71 3E J R1 3T 4 J

(124– 130).(i) All petals are trees Universal Affirmative (A-type).(ii) Some days are nights Particular Affirmative (I-type).(iii) No lock is toy Universal Negative (E-type).(iv) Some locks are not toys Particular Negative (O-type)

124. (b) All keys are locks.

No lock is toy.A + E E-type of Conclusion“No key is toy.”All bags are toys.

No toy is lock.A + E E-type of Conclusion“No bag is lock.”All bags are toys.

No toy is key.A + E E-type of Conclusion“No bag is key.This is Conclusion I.

125. (d) All the three Premises are Particular Affirmative (I-type).No Conclusion follows from the two Particular Premises.Conclusions I and III from complementary pair.Therefore, either I or III follows.

126. (a) Some tyres are wheels.

All wheels are buses.I + A I-type of Conclusion“Some tyres are buses.”Conclusion I is Converse of it.Conclusion II is Converse of the second premise.

127. (b) Some cats are horses.

All horses are tigers.

Page 382: yoursmahboob.wordpress.com SBI · yoursmahboob.wordpress.com iii P 101 Speed Tests for SBI Bank Clerk Exam 101 Speed Tests for SBI Bank Clerk Exam is revised and updated edition on

yoursmahboob.w

ordpress.com

116 101 SPEED TESTI + A I-type of Conclusion“Some cats are tigers.”Conclusion I is Converse of it.

128. (a) All ropes are sticks.

Some sticks are hammers.A + I No Conclusion

129. (e) The following changes occur in the subsequent figures.(1) to (2) (1) to (3)

These two steps are continued in the subsequentfigures alternately.

130. (a) The following changes occur in the subsequent figures:(1) to (2) (2) to (3)

These two steps are continued in the subsequentfigures alternately.

131. (d) The given passage talks about the disposal of electionpetitions. It also gives the resolution of the disposal ofthe election petition. While option (d) also have thesame sanse, Hence it may be a conclusion.

132. (c) Obviously, option (c) strengthens the conclusion ofthe passage.

133. (c) The summary of option (c) opposes the conclusion of thepasses. Thus it weakens the conclusion of the passage.

134. (a) The code has been generated by writing the letters inreverse order OMNIBUS SUBINMOTherefore, TROUBLE ELBUORT

135. (e) 3

C

15

O

18

R

16

P

18

R

1

A

20

T

15

O

5

E

136. (d) 1 2 3 4 5 6 7 8 9E D U C A T I 0 N

Meaningful word DATE

137. (d) 20 m

10 m 10 m

5 km

C B

D AE

Required distance = AE = AD + DE = 20 km + 5 km = 25 km138. (b) 12 B 20 A 4 Q 10 F 30 = ?

? = 12 × 20 ÷ 4 – 10 + 30? = 12 × 5 – 10 + 30 ? = 90 – 10 = 80

139. (a) 1 8 4 3 7 25 9

5th of the left of 2140. (c) Z Y X W V U T

Z Y X W V UZ Y X W VZ Y X W

Z Y X151. (a) Refer to the sentence ‘He could not find.........in search

of work’.152. (e) Refer to the sentence “Please do not help him”..........of

the second para of the passage.153. (c) Refer to the sentence “You are...........bite me?”........of

the second para of the passage.154. (a) Refer to the sentence “He would be.......Ghanshyamdas”

.......of the third para of the passage.155. (d) Refer to the sentence, “He gave him........delicious

fruits”........of the third para of the passage.156. (e) Refer to the sentence “I shall creep........bite her........

hand on her forehead” of the fourth para of the passage.157. (d) Refer to the sentence, “A man brought .......prince who

is missing”........of the third para of the passage.158. (c) Refer to the sentence “He at once........pieces of gold”

of the second last sentence of fourth para of thepassage.

159. (b) Refer to the sentence “Finally, the kingdeclared........handsomely rewarded” of the fourth paraof the passage.

160. (b) The moral of the story can possibly be “A good deednever goes in vain”.

161. (b) The word ‘live’ is most similar in meaning to the word‘go’ in the context of the passage.

162. (c) The word ‘modest’ is a synonym of humble, while otherwords are plain, simple, ordinary and unpretentious.

163. (d) The most similar word is ‘ask for’ in the context of thepassage.

164. (e) The word ‘stopped’ is most opposite in meaning likebreak off.

165. (a) The word ‘meagerly’ is the antonym of the wordhandsomely in the context of the passage.

166. (d) Replace to by ‘too’ to make a correct phrase ‘a bit toofast’ which means slightly or to a small extent.

167. (e) The word think about refers to consider. Hence nocorrection is required.

168. (a) Replace leisure by the adverbial word leisurely.169. (b) The phrase ‘gift of the gab’ means ‘to have a talent for

speaking’.170. (c) The phrase ‘the order of the day’ means common,

popular or suitable at a particular time. For example :Pessimism seems to be the order of the day.

171. (d) It should be ‘here was her son’s future’.172. (c) The correct spelling should be ‘triumphant’.173. (a) The word suppressed should be replaced by revealed

or leaked in the sentence.174. (b) It should be ‘handed over’ which means the act of

moving power or responsibility from one person toanother.

175. (a) The correct spelling is difficult.